prep final peds

¡Supera tus tareas y exámenes ahora con Quizwiz!

The nurse is caring for a client diagnosed with chronic renal failure who has been taking Epoetin alfa for 2 months. What should the nurse monitor for pertaining to Epoetin alfa during the client's clinic visit? 1. Hypertension 2. Halitosis 3. Hemoptysis 4. Oliguia 5. Dependent edema

1., 3., & 5. Correct: Epoetin alfa can cause or worsen high blood pressure, induce rapid weight gain, and swelling of feet and hands . Clients may experience coughing up of blood as a result of a rapid increase number of RBCs. 2. Incorrect: Medication has no effect on breath odor. 4. Incorrect: Client in renal failure is likely to be oliguric or anuric. This is not caused by the medication.

The nurse is caring for a client that is receiving blood that was started 2 hours ago. The nurse observes that the client has flushed cheeks. What should the nurse do first? 1. Inform the primary healthcare provider. 2. Stop the blood infusion. 3. Obtain a blood sample from the client. 4. Take vital signs.

2. Correct: Flushing is an adverse reaction to blood transfusion. Stop the infusion immediately, then notify primary healthcare provider. 1. Incorrect: Notify the primary healthcare provider after stopping the blood transfusion. 3. Incorrect: Blood samples will be collected from the client to evaluate the reaction, but stop the transfusion first. 4. Incorrect: Take vital signs after stopping the blood transfusion.

A client reports difficulty sleeping since starting a new job. The nurse's assessment identifies that the client is also working after hours from home. Which teachings are appropriate to promote sleep in this client? 1. Vary bed times to determine time best to promote sleep. 2. Use the bedroom for only sleep. 3. Schedule meal times earlier in the evening. 4. Avoid caffeine in the evening. 5. Use a white noise machine to help lull to sleep.

2., 3., 4. & 5. Correct: The client should associate bed with sleep, not work. Eating late in the evening may interfere with sleep, especially if a heavy meal. Caffeine late in the evening may increase alertness and interfere with sleep. Many people respond positively to white noise. Music, on the other hand, may make it more difficult to sleep. 1. Incorrect: The same time for bed each day will establish a routine and make sleep easier. Varying sleep times will disturb the client's sleep cycle and circadian rhythm. This would not be helpful to facilitate sleep.

What should the nurse include in the plan of care for a child who is receiving chemotherapy for a diagnosis of leukemia? 1. Place the child in a negative pressure isolation room. 2. Administer prophylactic intravenous (IV) antibiotics. 3. Avoid high protein food intake. 4. Teach family and visitors handwashing techniques.

4. Correct: Any client on chemotherapy should have good infection control measures in place such as handwashing by all who they encounter. 1. Incorrect: If the client is immune suppressed, place them in a positive pressure isolation room. A negative pressure room primarily keeps its air inside the room with controlled venting only; whereas a positive pressure room keeps unfiltered air from outside the room out of the room all together. In a hospital, clients with communicable diseases, especially airborne ones, are kept in isolation rooms. In order to ensure the safety of other clients, staff and visitors, it is important that the isolation room contain negative air pressure. This will keep any germs from entering the general airflow and infecting other people. Positive pressure isolation rooms are designed to keep a vulnerable client in isolation safe from contamination from the outside. The air pressure in the room is greater than that outside of it, so it pushes potential infection agents or chemicals away from the client. The most common application is in rooms for client who have compromised immune systems. For these individuals, it is important that no common pathogens, even those that are harmless to healthy people, enter the room. For positive pressure isolation rooms, an anteroom is recommended and incoming air is filtered through both HEPA filters and ultraviolet germicidal irradiation systems, which kill bacteria by exposing them to ultraviolet light. 2. Incorrect: This would be appropriate if there was evidence of a bacterial infection. Just because chemotherapy is being administered does not mean the client has an infection. 3. Incorrect: This client would likely need a high protein diet to meet the nutritional demands of the body during chemotherapy. We need protein for growth, to repair body tissue, and to keep our immune systems healthy. When the body doesn't get enough protein, it might break down muscle for the fuel it needs. This makes it take longer to recover from illness and can lower resistance to infection. People with cancer often need more protein than usual. After surgery, chemotherapy, or radiation therapy, extra protein is usually needed to heal tissues and help fight infection.​

The primary healthcare provider prescribed 0.125 mg of digoxin daily for a client. On hand, the nurse has digoxin 0.25 mg/mL. How many mLs of digoxin should the nurse administer? 1. 5 mL 2. 3 mL 3. 0.5 mL 4. 0.3 mL

0.5

In what position should the nurse place a client post intracranial surgery? 1. Head of bed elevated 30 degrees 2. Supine 3. Dorsal recumbent 4. Recovery position

1. Correct: The goal after intracranial surgery is to keep the intracranial pressure (ICP) from rising while optimizing the cerebral perfusion pressure (CPP). The ideal position for this client is HOB elevated and the head in neutral position. 2. Incorrect: Placing the client in supine position may increase ICP. Supine position is achieved when the client is lying flat. 3. Incorrect: Dorsal recumbent position will increase ICP as this position will increase peripheral return. The client in dorsal recumbent position is lying flat with the knees flexed and separated. 4. Incorrect: The recovery position is side lying position with one knee flexed. This position can also increase ICP.

A child who is 12 hours status post tonsillectomy and adenoidectomy reports feeling nauseated and shows the nurse a moderate amount of red-tinged vomitus in an emesis basin. Which action should a nurse take first? 1. Notify the primary healthcare provider 2. Place ice collar on child's neck 3. Administer an antiemetic as prescribed 4. Apply bilateral pressure to the child's neck

1. Correct: The nurse should notify the primary healthcare provider immediately because the appearance of moderate red-tinged vomitus could indicate hemorrhage in the surgical area. 2. Incorrect: The child should not have a moderate amount of red-tinged vomitus 12 hours post-op. The primary healthcare provider should be notified. Ice collar will not fix the problem. 3. Incorrect: Administering an antiemetic is not an appropriate action because the child's nausea is being caused by blood pooling in the stomach. 4. Incorrect: The application of pressure to the child's neck is contraindicated because this action would not resolve bleeding in the oropharynx and might block the carotid arteries causing harm to the child.

A client presents in the emergency department with acute onset of fever, headache, stiff neck, nausea/vomiting, and mental status changes. What interventions should the nurse initiate? 1. Elevate HOB 30 degrees 2. Pad side rails 3. Provide sponge bath if temperature greater than 101°F (38.3°C) 4. Initiate airborne isolation precautions 5. Darken room

1., 2., 3. & 5. Correct: An acute onset of fever, headache, stiff neck, n/v, and mental status changes are consistent with bacterial meningitis. Elevate the head of the bed to promote comfort and decrease intracranial pressure. The client is at an increased risk for seizures, and the nurse should implement seizure precautions which include padding the side rails. A sponge bath is an independent nursing intervention appropriate for a fever greater than 101°F (38.3°C). Darkening the room is also a comfort measure as this client will have photophobia. 4. Incorrect: Droplet precautions should be initiated for the first 24 hours of antimicrobial therapy. Option 1. Would we want to elevate the HOB? Yes. The client is having mental changes, and is at risk for increased intracranial pressure. Don't lie them supine. That would increase ICP. Option 2, pad the side rails? Is this a necessary intervention? Yes. Could this client have a seizure? Absolutely, so safety is a concern. We want to protect the client from injury. Option 3, Provide a sponge bath for an elevated fever? This would be appropriate - True. A tepid sponge bath will help to lower the fever. Option 4 - Do we need to place this client on airborne isolation? No, This is false. This client needs to be placed on droplet precautions until antimicrobial therapy has been administered for at least 24 hours. Most of the bacteria that cause this form of infection are spread through close personal contact, such as coughing, sneezing, and kissing. Option 5 - darkened room? Do we need to do this? Yes, true. Clients with bacterial meningitis have photophobia (extreme light sensitivity). So darkening the room will help the client's comfort level.

A client who has recurrent episodes of allergic rhinitis asks the nurse what could be done to decrease symptoms. What instruction should the nurse provide to this client? 1. Remove pets from interior of home. 2. Treat a stuffy nose with warm salt water. 3. Remove carpeting. 4. Stay inside when pollen count is at its lowest. 5. Wash bed linens in hot water.

1., 2., 3., & 5. Correct: Controlling pet dander and other pet allergens is one way to reduce allergic rhinitis. Keep pets outside. Cleaning nasal passages with saline solution (warm salt water) will relieve stuffy nose. Control dust and dust mites by dusting regularly, removing carpeting and washing bed linens in hot water. 4. Incorrect: Stay inside with closed doors and windows during high-pollen season.

Which food items, if chosen by a new unlicensed assistive personnel (UAP), would indicate to the nurse that the UAP understands a clear liquid diet? 1. White grape juice 2. Gelatin 3. Vanilla pudding 4. Lemon Popsicle 5. Fat free Broth 6. Tea with honey

1., 2., 4., 5., & 6. Correct: A clear liquid diet is made up of only clear fluids and foods that are clear fluids when they are at room temperature. These choices are considered to be clear liquids. 3. Incorrect. This is considered appropriate for a full liquid diet. A clear liquid diet helps maintain adequate hydration, provides some important electrolytes, such as sodium and potassium, and gives some energy at a time when a full diet isn't possible or recommended. The following foods are allowed in a clear liquid diet: Water (plain, carbonated or flavored) Fruit juices without pulp, such as apple or white grape Fruit-flavored beverages, such as fruit punch or lemonade Carbonated drinks, including dark sodas (cola and root beer) Gelatin Tea or coffee without milk or cream Strained tomato or vegetable juice Sports drinks Clear, fat-free broth (bouillon or consomme) Honey or sugar Hard candy, such as lemon drops or peppermint rounds Ice pops or popsicles without milk, bits of fruit, seeds or nuts Any foods not on the above list should be avoided.

Following report, which newborn infant should the nursery nurse assess first? 1. Positive Babinski reflex noted. 2. Has circumoral cyanosis. 3. Negative Ortolani's sign noted. 4. Has telangiectatic nevi.

2. Correct: Circurmoral cyanosis is bluish discoloration of and around the lips. It is an indicator of cyanotic heart defect. 1. Incorrect: The +Babinski reflex is normal in a newborn up until the child starts to walk. Then it should be negative. 3. Incorrect: A positive (not negative) Ortolani's sign indicates congenital hip dislocation. So this is normal. 4. Incorrect: The common term for this type of nevi is "stork bites", which are a normal newborn finding.

The nurse should teach the client with chronic pancreatitis how to monitor for which problem that can occur as a result of the disease? 1. Hypertension 2. Diabetes 3. Hypothyroidism 4. Graves disease

2. Correct: Insulin is produced in the pancreas. When the client has chronic pancreatitis, the pancreas becomes unable to produce insulin, thus resulting in diabetes. 1. Incorrect: No, not associated with chronic pancreatitis. 3. Incorrect: No, not associated with chronic pancreatitis. 4. Incorrect: No, not associated with chronic pancreatitis.

The nurse is caring for a client on the psychiatric unit. The client is prescribed fluphenazine 10 mg. The drug is available as an elixir: 2.5 mg / 5 mL. How many mL will the nurse give to the client? ______mL. Round answer to the nearest whole number.

2.5 mg : 5 mL :: 10 mg : x mL 2.5 mg/x mL = 50 mg/mL 2.5 mg/x mL = 50 mg/mL x = 20 mL

A preschool child is brought to the emergency department for evaluation following several days of vomiting and diarrhea with low a grade fever. In order to determine if the child is dehydrated, the nurse knows the most reliable indication is what? 1. Dull sunken eyes 2. Blood pressure of 95/50 3. Specific gravity of 1.030 4. Depressed fontanels

3. CORRECT: Physical signs or symptoms can vary from client to client, depending on a variety of factors including age, general health, and history. However, laboratory results provide documented evidence of the body's status. The elevated specific gravity indicates dehydration. 1. INCORRECT: While dull, sunken eyes usually indicate an illness, they are not a reliable specific sign of dehydration. Sunken eyes could be present with malnutrition, weight loss, or other health issues. 2. INCORRECT: Blood pressure varies with age and health status, but is a good indication of the child's status. Normally, preschool children in the 3 to 4 year age group average a systolic of 105 to 110, with a diastolic averaging around 60 to 70. While a blood pressure of 95/50 is low for a preschool child, it is reflective of several days of illness with vomiting. This is not a reliable indication. 4. INCORRECT: Ordinarily fontanels provide a good indication of a child's hydration status. Sunken fontanels indicate dehydration while bulging fontanels mean increased intracranial pressure. However, this child is too old for the nurse to assess fontanels. The posterior fontanel is closed at 2 months of age, and the anterior fontanel can no longer be palpated by 18 months of age. Option 1: No! We tend to associate dull, sunken eyes with an illness and occasionally that may be true. However, in this situation, the nurse is looking for more definite evidence of dehydration. Dull eyes, particularly in the lower conjunctiva membranes, could indicate some dehydration. But the child has been sick several days, and sunken eyes do not provide specific proof. Option 2: Close, but not quite. Did you recognize that this blood pressure is a bit low even for a preschool child? Good catch, but the nurse wants to establish definitive evidence of dehydration. While a slightly lower blood pressure does indicate fluid depletion, there is other data that would provide more substantial proof. Option 3: Excellent! Lab work provides supportive data of what is occurring in the body. This specific gravity is on the high end of normal, indicating concentration and dehydration. This lab work provides the nurse with specific evidence of dehydration. Option 4: Definitely not. Review the specifics about fontanels. Infants are born with two open fontanels, one anterior and one posterior. These areas provide the nurse with a method to determine hydration status. A sunken fontanel does mean dehydration while a bulging fontanel means increased intracranial pressure. But the anterior fontanel closes by 8 weeks of age while the posterior fontanel is usually closed by 1 ½ years of age. This child is too old to still have palpable fontanels.

Following amniotomy, what intervention should the nurse perform? 1. Administer oxygen to client. 2. Have client ambulate to promote labor. 3. Obtain temperature every 4 hours. 4. Monitor fetal heart rate.

4. Correct: The fetal heart rate is assessed for at least 1 minute after amniotomy. The umbilical cord could be displaced in a large fluid gush, resulting in compression and interruption of blood flow through it. 1. Incorrect: Oxygen is not indicated after amniotomy. 2. Incorrect: Ambulation is generally limited following amniotomy to prevent complications such as prolapsed cord if the fetal head is not fully engaged. 3. Incorrect: The client's temperature should be assessed every 2 hours after the membrane's rupture. Elevations above 100.4°F (38°C)should be reported.

The nurse is working with victims of domestic abuse. The nurse should understand which of these factors is a reason why domestic violence or emotional abuse remains extensively undetected? The expenses due to police and court costs are prohibitive Little knowledge is known about batterers and battering relationships There are typically many series of minor, vague complaints Few people who have been battered seek medical care

There are typically many series of minor, vague complaints Signs of domestic violence or emotional abuse may not be clearly manifested and include many series of a minor complaints such as headache, abdominal pain, insomnia, back pain and dizziness. These may be covert indications of violence or abuse that go undetected. These complaints may be vague and reflect ambivalence about the disclosure of any violence or abuse.

An adult client has partial and full thickness burns over the anterior trunk and anterior and posterior aspects of both legs. Utilizing the rule of nines, what percentage of the body surface area is burned? Round your answer to the nearest whole number.

The anterior trunk counts for 18% of the body; entire right leg counts 18%; entire left leg counts 18%. Body surface on this client is 54%.

The nurse has an order to insert an indwelling urinary catheter for a male client. What is the best reason for lubricating the tip of the catheter prior to insertion? Reduce the friction within the urethra Diminish the leakage of urine around the catheter Minimize risk for infection Prevent bladder distention Due to the somewhat long length of the male urethra, lubrication reduces potential discomfort and localized tissue irritation as the catheter is passed.

Reduce the friction within the urethra Due to the somewhat long length of the male urethra, lubrication reduces potential discomfort and localized tissue irritation as the catheter is passed.

The nurse is caring for a 50 year-old client diagnosed with advanced cirrhosis of the liver. Which nursing diagnosis should take priority? Fluid volume excess: ascites Risk for injury related to peripheral neuropathy Altered nutrition: less than body requirements Risk for injury: hemorrhage

Risk for injury: hemorrhage Liver disease interferes with the production of prothrombin and other factors essential for blood clotting. In addition, hemorrhage, especially from esophageal varices, can be life-threatening. This takes priority over the other nursing diagnosis.

Which clients can the nurse assign to the same room? 1. A 48 year old female one day postoperative appendectomy and a 30 year old female with nephrolithiasis 2. A 41 year old male with nausea, vomiting, and diarrhea and a 62 year old male with neutropenia 3. A 41 year old male with Methicillin-resistant Staphylococcus aureus (MRSA) infection and a 42 year old male with Clostridium difficile 4. A 14 year old two days postoperative splenectomy and an 80 year old female with Parkinson's disease 5. A 57 year old female with chronic obstructive pulmonary disease (COPD) and an 68 year old female with asthma

1 & 5. Correct: Both the client with a postoperative appendectomy and the client with nephrolithiasis will need frequent pain assessments. Also neither client has an infection that could be transmitted to the other client.These 2 clients can be assigned to the same room. The clients with asthma and COPD are noninfectious respiratory diseases, so they also can be assigned to the same room. 2. Incorrect: The client with neutropenia has a low number of neutrophils which are a common type of white blood cell important to fighting off infections. The client should be assigned to a single-client room. In addition the other client could be contagious depending on the causative factor of the nausea,vomiting and diarrhea. The client with neutropenia should not be assign with this client since their diagnosis has not been identified. 3. Incorrect: MRSA and C difficile require contact isolation due to different causative organisms. Both of these clients should be assigned to a single-client room. In the healthcare setting it is recommended that clients requiring Contact Precautions should be assigned a single-client room. 4. Incorrect: Think about it an adolescent and an older adult in the same room. The 14 year old client is in the early adolescent stage. Since an admission to the hospital is a stressful situation, the client may exhibit "immature" behaviors and be embarrassed about the healthcare team seeing their bodies. There is a wide gap between a 14 year old and a 80 year old developmental stage. The 80 year old is experiencing developmental changes for older adult client. In addition, this client is exhibiting symptoms of Parkinson's disease. Before we review the options, let's look at the question. The key words in a question should be identified. The key words in this question are clients, assign, and same room. This is a select all question so there will be 2 or more options correct. Also each option stands alone with the question. After reviewing the question, look at each option and identify if it is true or false. Option 1 is true. Ask yourself which groups of clients have something in common. The client post-operative appendectomy and the client with nephrolithiasis will both need to be frequently assessed for pain and interventions aimed at pain management. Also neither client has an infection that could be transmitted to the other client. Option 2 is false. The client with neutropenia has a low number of neutrophils which are a common type of white blood cell important to fighting off infections. The client should assigned to a private room. In addition the other client could be contagious depending on the causative factor of the nausea,vomiting and diarrhea. The client with neutropenia should not be assign with this client since their diagnosis has not been identified.. Option 3 is false. Contact isolation is required for both MRSA and C. difficile but the causative organisms for the diseases are not the same. Option 4 is false. The age difference between teenager and the elderly adult are so vast that the developmental needs of the clients vary too much for them to be placed in a room together. Option 5 is true. Neither client has an infectious disease. The clients with bronchitis and COPD have similar respiratory conditions that are not infectious.

The client has been taking divalproex for the management of bipolar disorder. The nurse should give priority to monitoring which laboratory test? 1. Alanine aminotransferase (ALT) 2. Serum glucose 3. Serum creatinine 4. Serum electrolytes

1. Correct: ALT levels will increase primarily in liver damage/disorders. A side effect of administering divalproex is drug-induced hepatitis. 2. Incorrect: Divalproex is not expected to alter glucose metabolism. 3. Incorrect: Divalproex should not cause a change in renal function. 4. Incorrect: Divalproex should not interfere with electrolytes balance.

Which is the correct method for removing personal protective equipment (PPE)? 1. Contaminated gloves should be removed in the client's room. 2. The glove that is removed first should be placed in the waste basket before the other glove is removed. 3. Remove face shield or goggles first. 4. Shoe covers should be removed last.

1. Correct: Avoid contaminating self, others, or environment when removing equipment. 2. Incorrect: The first glove is held in the still gloved hand and the second glove is slid over the first removed glove. 3. Incorrect: Hand hygiene is performed before removing face shield or goggles. 4. Incorrect: Shoe covers are removed with gloved hands.

A 35 year old client asks a clinic nurse how to find out if the client is overweight or obese. The client weighs 135 pounds and is 5 feet 2 inches tall. What should the nurse educate the client about? 1. Calculating body mass index 2. Measuring abdominal circumference 3. Determining lean body mass 4. Finding the nearest hydrostatic testing location

1. Correct: Calculating body mass index (BMI) would determine if the client is considered overweight or obese. 2. Incorrect: BMI is the most efficient way to determine if a client is overweight or obese. Measuring the abdominal circumference is one assessment for determining if a client is at risk for metabolic syndrome. 3. Incorrect: BMI calculates whether the client is overweight or obese. Once you have the BMI, you can calculate the lean body mass. 4. Incorrect: Although this is the "gold standard" for measuring body fat percentage by weighing the body in water, it is often performed in hospitals and university labs. It is not the most practical means of monitoring weekly progress.

A nurse is caring for a nonambulatory client who must be decontaminated after a chemical exposure event. What nursing action will prevent further chemical exposure? 1. Don appropriate personal protective equipment (PPE). 2. Remove only contaminated clothes. 3. Avoid decontaminating the eyes. 4. Use hot water during decontamination.

1. Correct: PPE should be donned prior to contact with the client to prevent contamination of the healthcare worker. The nurse must protect themselves from exposure of the chemical. 2. Incorrect: All clothes, jewelry, and personal belongings should be removed and placed into appropriate containers. There is no way to be certain which articles of clothing are contaminated and which are not. 3. Incorrect: Decontaminate the eyes with a saline solution via nasal cannula or Morgan lens. Preventing cornea damage is very important. 4. Incorrect: Hot water is unnecessary unless the client is hypothermic during decontamination procedures. Hot water causes vasodilation.

Which assignment would be appropriate for the nurse to delegate to an unlicensed assistive personnel (UAP)? 1. Totaling I & O records on five clients at the end of the shift. 2. Assessing VS on a client who was admitted 30 minutes ago. 3. Administering nasogastric (NG) tube feeding. 4. Changing an abdominal surgical dressing on a client that is 3 days post op.

1. Correct: Totaling I & O is an appropriate task for a UAP to be assigned. This is within the scope of practice for the UAP. 2. Incorrect: New clients should be assessed by an RN; however, it is acceptable for the RN to get assistance with some of the information. The RN must verify all information. The client is a new admit, and is considered unstable; therefore, the RN should get the baseline vitals. 3. Incorrect: Administering a NG tube feeding is not within scope of practice for the UAP. 4. Incorrect: Changing a surgical dressing is not within the scope of practice for the UAP.

To promote rapid diuresis in a client in acute pulmonary edema, which prescription should the nurse administer first? 1. Furosemide 40 mg IVP 2. Dopamine 15 mcg/kg/min 3. Hydrochlorothiazide 25 mg PO 4. Captopril 25 mg PO

1. Correct: Yes, there is a good bit of fluid overload with acute pulmonary edema, so the furosemide should be started first. Furosemine is a loop diuretic that prompts rapid loss of excess fluid. 2. Incorrect: Low doses of dopamine (0.5-5 mcg/kg/min) cause vasodilation and increased diuresis. High doses (5-10 mcg/kg/min) cause vasoconstriction to increase blood pressure and improve cardiac output. The low dose would be better here. 3. Incorrect: A loop diuretic given IVP will work faster than a potassium-sparring diuretic by mouth. 4. Incorrect: Give the furosemide first, then determine if the client needs an ACE inhibitor.

The home care nurse is caring for an elderly client status post total hip replacement and a history of cirrhosis. Which statements by the client's spouse indicates that teaching regarding pain management has been successful? 1. "If the pain increases, I must let the nurse know immediately." 2. "I should have my spouse try the breathing exercises to help control pain." 3. "This narcotic causes very deep sleep, which is what my spouse needs." 4. "If constipation is a problem, increased fluids will help." 5. "My spouse can have one glass of wine to help promote pain relief."

1., 2., & 4. Correct: These are correct responses by the spouse. Increased pain may indicate something else is going on. Breathing exercises would be an excellent non-pharmacological intervention. Increasing fluid is an appropriate intervention for constipation. Narcotics place the client at risk for constipation. 3. Incorrect: The nurse should know that goal of pain relief and administration of narcotics is to use the smallest dose possible to relieve the pain. Narcotics are not used to put clients into deep sleep. Also, clients with liver failure have less tolerance to narcotics because they cannot mobilize drugs like a normal person. Narcotics must be given cautiously to clients with liver disease and the elderly. 5. Incorrect: Alcohol should not be taken when a client has cirrhosis. Alcohol is hepatotoxic to the client with liver disease.

A primary healthcare provider has prescribed chlorpromazine 150 mg by mouth twice a day. The pharmacy sends chlorpromazine oral concentration: 100 mg/mL. How many mL should the nurse administer for each dose? Round answer using one decimal point.

1.5 150 mg : x mL :100 mg : 1 mL 100 x = 150 100 100 divide both sides by 100 X = 1.5

The primary healthcare provider prescribes an intravenous infusion of D5 W at 125 mL per hour. The tubing has a drop factor of 20 gtt/mL. How many drops per minute should the nurse administer? Round answer to the nearest whole number.

125 x 20 60 = 41.666 = 42 Since partial drops cannot be counted, always round to the nearest whole number which, is 42.

The nurse is caring for a client reporting intense headaches with increasing pain for the past month. An MRI is prescribed. In reviewing the client's information, which piece of information is of concern to the nurse? 1. Allergy to iodine 2. Internal cardiac defibrillator 3. Is a diabetic 4. Had a stroke a year ago

2. Correct: If a client with a cardiac pacemaker and internal defibrillator has an MRI, the pacemaker is turned off and the client could die. The MRI uses a magnet. Magnets turn off pacemakers. This needs to be reported to the primary healthcare provider. 1. Incorrect: No, there is no dye involved with MRI. Magnetic resonance imaging (MRI) is a test that uses a magnetic field and pulses of radio wave energy to make pictures of organs and structures inside the body. 3. Incorrect: Diabetics can have an MRI without being made NPO or withholding medications prior to the test. This test should not affect the client's blood glucose in any way. 4. Incorrect: MRI is not contraindicated with stroke. This is not an invasive procedure and does not place the client at increased risk for complications related to having had a stroke.

The nurse is working in the term nursery. Which task should be performed first on a newborn? 1. Prepare the circumcision equipment for a two day old newborn. 2. Assess the five minute APGAR of a newborn. 3. Perform the gestational age assessment on a 30 minute old newborn. 4. Obtain a blood sample for metabolic testing on a 24 hour old newborn.

2. Correct: The APGAR is done to determine whether a newborn needs help breathing or is having heart trouble. It looks at the newborn's breathing effort, HR, muscle tone, reflexes, and skin color and is the most important initial assessment for a newborn. 1. Incorrect: This task is not emergent and can be performed later at an appropriate time. 3. Incorrect: This task is not emergent and can be performed later at anytime during the transition stage of the newborn's nursery care. 4. Incorrect: This task is not emergent and can be performed at anytime between 24 hours and 7 days old. Typically it is done before the newborn is discharged home.

Which clients would be appropriate for the charge nurse to assign to the LPN/VN? 1. Client admitted with exacerbation of asthma. 2. Client needing oral antibiotics for a diagnosis of gastroenteritis. 3. Client 4 hours post lobectomy. 4. Client with terminal cancer refusing pain medication. 5. Client with arthritis who needs scheduled pain medication around the clock. 6. Client who has a chronic graft versus host disease.

2., 5, & 6. Correct. The LPN/VN's scope of practice includes caring for clients with chronic and stable health problems. These clients are stable. 1. Incorrect. This client is unstable and requires ongoing assessment, evaluation and teaching. 3. Incorrect. This client is unstable and requires ongoing assessment, evaluation and teaching. 4. Incorrect. This client is unstable and requires ongoing assessment, evaluation and teaching.

The homecare nurse is visiting a client who recently had a miscarriage at 22 weeks. When is the most appropriate time for the nurse to discuss the topic of another pregnancy? 1. The topic should wait until the nurse builds rapport with the client. 2. Another pregnancy should not be discussed for at least six months. 3. Wait until the client initiates the topic of future pregnancies. 4. Discussion should begin immediately upon the first home visit.

3. CORRECT: A mother who has had a miscarriage will experience all, or some, of the Kübler-Ross's stages of death and dying, and therefore each individual will have a unique, response to the loss of a fetus. The best course of action by the nurse is to utilize therapeutic communication techniques and approach the client with open-ended statements. This allows the client to initiate the topic at whatever point is most appropriate for their own situation. 1.INCORRECT: Building rapport with a new client is an important aspect of establishing therapeutic communication. While each nurse/client relationship is unique, it is expected that rapport will begin to be established during the first visit. However, even after establishing rapport, the nurse must follow the client's lead when discussing the topic of another pregnancy. 2. INCORRECT: When dealing with a client who has suffered loss, there are no hard and fast rules for discussing the topic. Deciding that the topic should not be discussed for a specific length of time, like six months, is inappropriate. Whether the client can safely get pregnant should be discussed with the primary healthcare provider. However, the nurse should take cues from the client about the topic of another pregnancy. 4. INCORRECT: The client has just experienced the loss of a pregnancy and will need the time to come to terms with that situation. Each client grieves in a unique time frame when dealing with such a loss. Unlike teaching that is initiated upon admission, the discussion of another pregnancy is not appropriate at the first visit, unless the client broaches the subject.

What should the nurse teach a client about testicular self examination? 1. This exam should be performed bi-annually. 2. The exam should be performed during a cold shower. 3. Gently roll each testicle with slight pressure between the fingers. 4. The epididymis should feel like a hard, knotty rope.

3. Correct: Examine one testicle at a time. Use both hands to gently roll each testicle, with slight pressure, between the fingers to feel for lumps, swelling, soreness or a harder consistency. 1. Incorrect: All men 15 years and older need to perform this examination monthly. 2. Incorrect: The exam should be performed during or right after a warm shower or bath when the the scrotum is less thick. 4. Incorrect: The epididymis should feel soft, rope like, and slightly tender to pressure. It is located at the top of the back part of each testicle. It is not a lump.

A community health nurse is reconciling medications of a client who was discharged from the hospital with a diagnosis of congestive heart failure, hypertension, and arthritis. After reviewing the client's medications, what action is most important for the nurse to take? *prescribed meds* Furosemide 20 mg tablet by mouth every morning Carvedilol 6.25 mg one tablet by mouth twice daily Potassium Chloride 20 mEq one tablet by mouth every morning *current meds* Saw palmetto one tablet by mouth every morning Adalimumab 40 mg subcutaneously every other week Captopril 25 mg one tablet by mouth every morning . Educate the client on the newly prescribed medications. 2. Inform the client to take the captopril at night. 3. Notify the primary healthcare provider that the client is receiving adalimumab. 4. Tell the client to stop taking saw palmetto.

3. Correct: This is the "most important" action for client safety. Medication reconciliation is "the process of comparing a client's medication prescriptions to all of the medications that the client has been taking. This reconciliation is done to avoid medication errors such as omissions, duplications, dosing errors, or drug interactions. It should be done at every transition of care in which new medications are ordered or existing orders are rewritten. Transitions in care include changes in setting, service, practitioner or level of care. [Adalimumab can cause serious side effects, including heart failure (new or worsening).] 1. Incorrect: Although the nurse will need to teach the client about the new medications, the most important thing for the nurse to do is inform the HCP about the client taking adalimumab. 2. Incorrect: The nurse cannot change the primary healthcare providers RX. Most HF clients go home on an ACE inhibitor, beta blockers, or both (as in this case). Ceptopril is an ACE inhibtor used for the treatment of hypertension and heart failure, and is often prescribed as two -three times daily. 4. Incorrect: Saw palmetto is used as a traditional or folk remedy for urinary symptoms associated with an enlarged prostate gland (also called benign prostatic hyperplasia, or BPH), as well as for chronic pelvic pain, bladder disorders, decreased sex drive, hair loss, hormone imbalances, and prostate cancer. Not saw palmetto contraindicated with prescribed medications.

The nurse is caring for a client on the oncology unit. The client asks, "Why do I need this LifePort to receive my chemotherapy?" What evidence should the nurse consider when answering? 1. IV infusions can be more rapidly administered via an implantable IV port 2. Implantable IV ports are kept sterile and therefore do not become infected 3. Chemotherapeutic agents are more readily absorbed from implantable IV ports 4. Implantable ports are beneficial when long-term and/or multiple IV therapy is indicated.

4. Correct: Clients requiring long-term and/or multiple IV therapy benefit from implantable ports, because they reduces the number of IV sticks, preserve the integrity of peripheral veins, and provide a vessel with adequate blood flow. The part allows chemotherapy agents to be given in a larger vein, decreasing risk of tissue damage that can occur with peripheral administration. 1. Incorrect: Rate of administration is not an indicator for an implantable port, and chemotherapeutic agents are administered at a slower rate than most IV medications. Chemo agents should be given at the prescribed rate. 2. Incorrect: Infection is a concern for any implantable device. Sterile technique is used when accessing port. Inspection of the site is essential, in addition to monitoring vital signs and WBCs. 3. Incorrect: Rate of absorption is not affected by the type of central line or implantable IV port. Implantable ports promote safety and reduce problems during medication administration.

A community health nurse is planning to teach a group of caregivers about early warning signs of Alzheimer's Disease (AD). What signs should the nurse include? 1. Mild disorientation 2. Difficulty with words and numbers 3. Poor personal hygiene 4. Agitation 5. Visual agnosia 6. Dysgraphia

1. & 2. Correct: Early warning signs of Alzheimer's Disease include mild disorientation and difficulty with words and numbers. This client may have difficulty recognizing numbers or doing basic calculations. The person may begin to have trouble with words. 3. Incorrect: Poor personal hygiene occurs as Alzheimer's Disease progresses due to ongoing loss of neurons. 4. Incorrect: Behavioral manifestations occur later in the disease process as a result of changes that take place within the brain. They are not intentional or controllable by the person with this disease. 5. Incorrect: With progression of this disease, additional cognitive impairments are noted, including visual agnosia, which is the inability to recognize objects by sight. 6. Incorrect: Dysgraphia is defined as difficulty communicating via writing and occurs during disease progression.

A client who is 36 weeks gestation has been admitted to the labor and delivery area for evaluation due to worsening signs of pregnancy induced hypertension (PIH). The BP upon arrival is 168/96. While being monitored, she reports a sudden onset of severe abdominal pain. Further nursing assessment reveals vaginal bleeding, abdominal rigidity, and a fetal heart rate of 90/min on the fetal monitor. What nursing actions would be appropriate for this client? 1. Continuously monitor the client's vital signs. 2. Keep the mother informed of the fetus' condition. 3. Careful monitoring of the fetal heart rate electronically. 4. Accurate measurement of I & O. 5. Prepare for emergency vaginal delivery. 6. Monitor for restlessness and decreased level of consciousness (LOC).

1., 2., 3., 4., and 6. Correct: The nurse recognizes that the client is demonstrating signs of placental abruption (abruptio placentae), most likely due to the presence of PIH. Due to the risk of shock, the maternal vital signs are checked immediately and continuously monitored. The mother will be aware of the emergent nature of her situation. She will need to be informed of what is occurring and kept informed of the status of the fetus. Accurate measurement of I&O, in addition to assessing the amount of vaginal blood loss, will be crucial in determining fluid volume status. Restlessness and decreasing level of consciousness would indicate poor cerebral perfusion as a result of decreased vascular volume and decreased cardiac output. Fluid and blood replacement would be indicated. 5. Incorrect: The infant is already demonstrating signs of distress (bradycardia), and the mother is considered unstable. The nurse would need to prepare her for an emergency delivery by cesarean, not a vaginal delivery. Now we have to implement interventions to save mom and baby. Look at the options. Option 1: Continuously monitor client's vital signs. Is this necessary? Yes, True. Mom is at risk for shock, so we need to keep a close eye on her vital signs. Option 2: Keep mom informed of the fetus' condition. True. The mother will be aware of the emergent nature of her situation. She will need to be informed of what is occurring and kept informed of the status of the fetus. Option 3: Careful monitoring of fetal heart rate electronically. True. The fetus is already showing signs of distress so continuous fetal monitoring is essential as mom is prepared for c-section delivery. Option 4: Accurate I&O. True. Accurate measurement of I&O, in addition to assessing the amount of vaginal blood loss, will be crucial in determining fluid volume status. Option 5: Prepare for emergency vaginal delivery. This is false. Emergency deliver is needed, however it needs to be an emergency cesarean delivery. The infant is already demonstrating signs of distress (bradycardia), and the mother is considered unstable. Option 6: Monitor for restlessness and decreasing LOC. What are these a sign of? Shock! So yes, we need to monitor for signs of shock. Restlessness and decreasing level of consciousness would indicate poor cerebral perfusion as a result of decreased vascular volume and decreased cardiac output. Fluid and blood replacement would be indicated.

Which client would be appropriate for the charge nurse to assign to the LPN/VN? 1. The client with a leg cast who needs neurovascular checks. 2. The client diagnosed with arthritis who needs pain medication and heat application. 3. The client reporting abdominal pain and rebound tenderness after a bicycle accident. 4. The client diagnosed with anorexia nervosa who is experiencing muscle weakness and decreased urinary output. 5. The client experiencing nausea and vomiting after receiving chemotherapy.

2. & 5. Correct: These are uncomplicated, stable clients. The LPN/VN can provide pain medication and heat application to the arthritis client and can care for a client experiencing nausea and vomiting after chemotherapy. 1. Incorrect: The LPN/VN can not do a neurovascular assessment independent of the RN. 3. Incorrect: This is a complicated client who has abdominal pain and rebound tenderness. This client is at risk for bleeding, and peritonitis. 4. Incorrect: This client has muscle weakness which is a symptom of hypokalemia. Also a decreased urinary output which could be due to FVD which could lead to shock.

Which instruction should the nurse include when developing a discharge teaching plan for a client diagnosed with a deep vein thrombosis (DVT) who has been prescribed rivaroxaban? 1. Importance of coagulation monitoring. 2. Avoid foods high in Vitamin K. 3. Have protamine sulfate readily available. 4. Methods to reduce risk of bleeding.

4. Correct: This client is at risk for bleeding. Educate on ways to reduce the risk of bleeding, such as using a soft toothbrush, floss with waxed rather than unwaxed floss, shave with an electric razor, take care when using scissors or knives, avoid potentially harmful activities, do not take aspirin or other NSAIDS. 1. Incorrect: No routine coagulation monitoring is required with this medication. 2. Incorrect: There are no dietary restrictions with this medication. Rivaroxaban doesn't interact with vitamin K like warfarin does. 3. Incorrect: There is no specific antidote for this medication. Protamine sulfate is a medication that is used to reverse the effects of heparin.

What is the priority nursing intervention when caring for a client with an eating disorder? 1. Encourage the client to cook for others 2. Weigh the client daily and keep a journal 3. Restrict access to mirrors 4. Monitor food intake and behavior for one hour after meals

4. Correct: Yes, this is the primary problem and the most life-threatening. Provide a pleasant, calm atmosphere at mealtime and one hour after meals. Meal times become episodes of high anxiety and monitoring for one hour after prevents the client from vomiting up food. 1. Incorrect: No, remember the focus is on control and attention to food. These clients need to eat. They may enjoy preparing food for others but that does not mean they will partake. 2. Incorrect: No, we don't let them know their weight. If they gain one ounce, these clients will try anything to lose it. The nurse wants to know if the client is gaining weight. Knowledge of the weight increasing induces feelings of becoming out of control. 3. Incorrect: They still need to brush their hair and put on make-up. We focus on these clients' eating to keep them healthy and alive. The client should have limited time with mirrors but it is appropriate for them to have access when getting ready.

As part of the screening process to identify if a client is obese, the nurse calculates the client's body mass index (BMI). Weight - 180 pounds Height - 5' 5" Calculate the BMI to the whole number.

Calculate BMI by dividing weight in pounds by height in inches squared and multiplying by a conversion factor of 703. 5'5" = 65" [180 pounds ÷ (65)2 ] x 703 = [180 pounds ÷ 4225] x 703 = 0.04260355 x 703 = 29.95 or 30

A client asks the nurse about including her 2 year-old and 12 year-old sons in the care of their newborn sister. Which response is an appropriate initial statement by the nurse? "Focus on your sons' needs during the first days at home."! "Suggest that your partner spend more time with the boys." "Tell each child what he can do to help with the baby." "Ask the children what they would like to do for the newborn."

"Focus on your sons' needs during the first days at home."! In an expanded family, it is important for parents to reassure older children that they are loved and as important as the newborn.

The nurse is caring for a client in the emergency department following an argument with the spouse. The client describes a verbal argument that began to get physical with shoving of the client. There is a history of domestic violence. Which phase of the cycle of violence is the client describing? 1. Honeymoon phase 2. Tension-building phase 3. Acute battering phase 4. Remorse phase

2. Correct: In the tension-building phase, minor physical or emotional abuse may occur as well as verbal arguments. The victim feels growing tension and tries to control the situation. 1. Incorrect: The honeymoon phase is characterized by remorse with promises never to hurt the victim again. The abuser is sorry and apologetic. 3. Incorrect: The acute battering phase includes the release of tension through extreme physical violence. This is also called the explosion phase. 4. Incorrect: There is no remorse phase, but remorse is expressed during the honeymoon phase. There are 3 phases: tension building, acute battering (explosion) and honeymoon phase.

A client is seen in the clinic expressing feelings of hopelessness and despair after losing his wife two months ago. He tells the nurse, "I think I am ready to go meet her. Please don't tell anyone." How should the nurse respond? 1. "I can see that you miss your wife very much." 2. "Tell me about your wife." 3. "I will keep your secret if you promise me you won't do anything until we talk again." 4. "I can't keep a secret like that. Are you planning to harm yourself?"

4. Correct: This elderly client is contemplating suicide. Elderly mean are at a high risk for succeeding at suicide because they tend to use lethal methods. The nurse has a responsibility to get the client help. 1. Incorrect: This ignores the problem. The issue is that the client is attempting suicide. 2. Incorrect: This question does not address the issues of suicide. Talking about his wife may make the client more depressed. 3. Incorrect: It is not appropriate to keep this information secret. The client may commit suicide in the meantime.

The RN, who is functioning as the charge nurse, needs to determine shift assignments. How will the charge nurse determine which client assignments are appropriate for the licensed practical nurse (LPN)? Ask the LPN about prior experience caring for clients with similar diagnoses Determine how many nursing assistants are available to help the LPN with client care Refer to the list of technical tasks LPNs are trained to perform Review the procedure manual with the LPN prior to making an assignment

Ask the LPN about prior experience caring for clients with similar diagnoses The definition of assignment is the routine care, activities and procedures that are within the authorized scope of practice of the RN or LPN/LVN. The RN must determine the needs of the clients and make assignments not only based on scope of practice, but also education, demonstrated competency and skill level. Regardless if the LPN received education and training to perform specific skills, the RN needs to determine the LPN's experience with caring for clients with similar diagnoses. While the RN is responsible for ensuring an assignment given to a delegatee is carried out completely and correctly, the LPN must be able to perform the skills or tasks independently.

The homecare nurse is providing family teaching on safety issues for a client diagnosed with Parkinson's disease. What adaptations should the nurse instruct the family to initiate? You answered this question 1. Install grab bars on tub walls. 2. Place nightlights in hallways. 3. Add bran and fiber to daily diet. 4. Remove scatter rugs or loose cords. 5. Keep bedroom dark, cool and quiet. 6. Put tennis balls on wheels of walker.

1., 2., 3., 4., & 5. Correct: Parkinson's disease causes deterioration of the basal ganglia, ultimately impacting motor control and function. As muscles become stiff and rigid, mobility slows, resulting in poor coordination and loss of balance. Safety is a chief concern in all ADLs, requiring modifications in activity, nutrition, and the client's environment. Because Parkinson's disease affects mobility, modification such as grab bars and night lights are essential. Clients develop constipation because of decreased peristalsis, so adding bran and fiber can address impending bowel issues. Scatter, or throw, rugs along with loose extension cords on the floor create a fall risk because the client is unable to pick up feet during ambulation. The shuffling gait that develops increases the risk for falls. These clients also have problems with insomnia along with poor REM sleep, leading to daytime drowsiness. Making the bedroom conducive to sleep may help alleviate symptoms for a period of time. A dark, cool room with no distractions is the most appropriate sleep environment. 6. Incorrect: The proper method of utilizing a walker is to step into the walker, pause and then move it forward before stepping again. Even though clients with Parkinson's disease have a shuffling gait and stooped posture, sliding a walker with tennis balls on the wheels presents a serious safety issue. The client would not have the ability to control the speed or hold the walker steady while stepping into it. Option 1: Of course! Prevention of falls is of primary importance since Parkinson's clients quickly develop unsteady gait and bradykinesia. Grab bars and nonslip rugs in bathing areas are important actions, particularly when clients try to remain independent in selfcare activities as long as possible. Option 2: Another good choice. Although the client's vision is not affected, the ability to ambulate steadily does deteriorate. Gait becomes slow, shuffling and uneven overtime. Adequate lighting at night, particularly enroute to the bathroom area, is another necessary safety modification. Option 3: Absolutely. As muscles in the body stiffen and become rigid, peristalsis also declines, resulting in constipation issues. It is very important to start adding bran and fiber to the diet daily to try to preserve bowel function as much as possible. Option 4: This one is correct also, and makes sense. Loss of motor control along with muscles stiffness contributes to balance problems. The worst thing to do is allow obstacles to exist in the walking area. Scatter rugs, also called throw rugs, are not attached and therefore present a fall risk. Extension cords on the floor also are dangerous because this client develops a shuffling gait. That inability to pick up feet during ambulation creates a fall hazard. Option 5: Definitely. Sleep disturbances like insomnia and poor REM sleep contribute to daytime drowsiness or even depression. Because these problems are part of the disease process, it is important to try to create a sleep environment that is conducive to rest. A dark, quiet atmosphere that is slightly cool with no distraction, like television, is most appropriate for Parkinson's clients. Option 6. Oops! Not this one, even though you may have seen this done before! Placing tennis balls onto the wheels of a walker was originally intended to make it easier for the client to propel the walker on all types of flooring, including carpet and tile. Even though clients with Parkinson's disease have difficulty with movements, altering the wheels with tennis balls would allow the walker to slide too easily, presenting a serious fall risk.

When teaching a client about lactose intolerance, what should the nurse include? 1. Common symptoms of lactose intolerance include abdominal bloating, diarrhea, and gas. 2. Symptoms of lactose intolerance generally occur three hours after consuming foods high in lactose. 3. Calcium rich foods should be consumed. 4. The client can drink lactose-free milk. 5. Vitamin D foods should be increased in the diet.

1., 3., 4., & 5. Correct: These statements are correct. Symptoms include abdominal bloating, pain, diarrhea, and gas. Because milk and milk products cause symptoms, the client may not get enough calcium and vitamin D. Supplementing with calcium or foods high in calcium and vitamin D is important to maintain these levels. The client may have lactose-free milk. 2. Incorrect: Symptoms occur 30 minutes to 2 hours after drinking milk or milk products.

What turning method should the nurse use to turn a client who has a spinal injury? 1. Lateral transfer 2. Slide sheet procedure 3. Logrolling 4. Mechanical lift transfer

3. Correct: Logrolling is used for the client who has a spinal injury. This technique keeps the client's body in straight alignment at all times. 1. Incorrect: Lateral transfer uses a spinal board to move the client from one bed to another. 2. Incorrect: Slide sheets enable clients to be slid up a surface or over to their side, that is, up the bed or rolled over in the bed. The difference is that all of the body may not be kept in perfect alignment as with logrolling. 4. Incorrect: A mechanical lift is used to move client from a bed to chair or chair to bed.

Four clients are admitted to the medical-surgical unit. The nurse is aware that what client will need standard precautions only? 1. The client with chicken pox. 2. The client with rubeola. 3. The client with impetigo. 4. The client with pancreatitis.

4. CORRECT: Standard precautions are observed with all clients admitted to the hospital, without the need for additional safeguards. The client with pancreatitis is not contagious and does not present any unique concerns other than the need for gloves and hand-washing. 1. INCORRECT: Chicken pox, also known as varicella zoster, requires airborne precautions. The virus can be spread through contact with the droplets, either touching or inhaling the droplet, while providing care for this client. 2. INCORRECT: Measles, also called rubeola, is spread through droplet contact with the contaminated individual, including inhaling the droplets. Airborne precautions are necessary when caring for a client diagnosed with rubeola. 3. INCORRECT: Impetigo is a severe skin infection characterized by itchy, red, fluid-filled blisters caused by either staphylococcus or streptococcus bacteria. This skin infection is highly contagious, and requires contact precautions to protect staff and visitors.

A client with Bell's palsy is having difficulty eating. Which action by the nurse will be most helpful? 1. Teach the client to perform active facial exercises several times a day. 2. Provide a liquid diet high in protein and calories that will be easily swallowed. 3. Provide oral hygiene after eating. 4. Teach the client to chew food on the unaffected side of the mouth.

4. Correct: Maintenance of good nutrition is most important. Teaching the client to chew on the unaffected side will help the client avoid food trapping. This will decrease the risk of aspiration which prioritizes higher than the other options. 1. Incorrect: Performances of facial exercises is important in recovery from Bell's palsy and will help over a long period of time. This intervention is not the highest priority. 2. Incorrect: Liquids are too difficult for the client to manage, as lip closure and chewing are impaired. A purely liquid diet increases the risk for aspiration. 3. Incorrect: Providing oral hygiene is important to prevent dental caries; however, this is not more important than preventing aspiration.

An unresponsive client with a respiratory rate of 14/min arrives at the emergency department after attempting suicide in a running car with the garage door closed. What action should the nurse perform FIRST? 1. Connect to an O2 saturation monitor. 2. Hyperventilate with an ambu bag. 3. Notify the xray department for a ventilation/perfusion scan of the lungs. 4. Administer 100% O2 per nonrebreather mask.

4. Correct: Do you see the clues? Suicide attempt, running car. Are you thinking carbon monoxide poisoning? Yes! Good for you! How do you treat carbon monoxide poisoning? It is treated with 100% oxygen. 1. Incorrect: Treatment is not monitoring. Do something. Yes, we want to monitor the client's O2 sat, but remember to start the oxygen first. Additionally, what will the O2 sat reading look like? It will look good because it is just telling us that something has attached to the hemoglobin. But what is attached here? Carbon monoxide. We will need ABGs 2. Incorrect: Hyperventilation is used when there is acidosis. Is a respiratory rate of 14/min normal? Yes, normal is 12-20 breaths per minute. 3. Incorrect: This procedure may be appropriate for emboli in the lungs and pneumonia but not for carbon monoxide poisoning.

Which task would be appropriate for the RN to delegate to the LPN/VN when caring for a client with COPD? 1. Auscultate breath sounds every 2 hours. 2. Assist client to sit on the side of the bed. 3. Ask the client about pertinent medical history. 4. Observe client's ability to purse-lip breathe.

4. Correct. LPN/VN's can observe clients to gather data regarding client performance of interventions that have already been taught. 1. Incorrect. It is the RNs task to assess, evaluate and teach. This task should not be delegated to the LPN/VN. 2. Incorrect. This intervention can be done by the UAP. The LPN/VN is not needed to reposition this client. 3. Incorrect. This is an RN task and should not be delegated to the LPN/VN.

The lactation consultant is preparing to make rounds on the breastfeeding clients on the Labor, Delivery, Recovery, Postpartum (LDRP). Which client should the consultant see first? 1. The mother who is nursing her newborn every 2-3 hours for 15-20 minutes at a time. 2. The mother who stated that her newborn sucks in short bursts and has audible swallowing. 3. The mother who reported blisters on her nipples and pain whenever the newborn latches on. 4. The mother who stated that her baby was so good that she has to wake him for each feeding.

4. Correct: A baby who is so sleepy that he doesn't wake on his own for feeding is at high risk for dehydration and malnourishment. This newborn needs further evaluation and close monitoring to prevent serious complications. 1. Incorrect: This is a normal finding for a breastfeeding mother and is not the priority concern for the consultant. 2. Incorrect: This is a normal finding for a breastfeeding mother and is not the priority concern for the consultant. 3. Incorrect: Blisters and pain are concerns that need to be assessed, but the sleepy baby situation has first priority. This would be the next client for the consultant to see, but not the first.

A community health nurse is assessing a migrant farmer who raises chickens. The nurse notes the client has developed a cough, fever, dyspnea, and hemoptysis. What infection should the nurse suspect? 1. Lyme disease 2. Toxoplasmosis 3. Tuberculosis 4. Histoplasmosis

4. Correct: Histoplasmosis is a fungal infection transmitted through ingestion of soil contaminated by bird manure. 1. Incorrect: The classic symptom of Lyme disease is usually an expanding target-shaped or "bull's-eye" rash which starts at the site of the tick bite. Fever, headache, muscle aches, and joint pain may also occur. 2. Incorrect: Toxoplasmosis occurs from contact with cat feces. Symptoms may be influenza-like: swollen lymph nodes, headaches, fever, and fatigue, or muscle aches and pains. 3. Incorrect: TB is often suspected; however, the primary difference is exposure to bird feces.

Which assignment would be most appropriate for the charge nurse to make for a nurse from the adult medical unit reassigned to the general pediatric unit? 1. 6 year old who has cystic fibrosis 2. 5 month old diagnosed with Respiratory Syncytial Virus (RSV) 3. 12 year old with Hirschsprung's Disease 4. 18 year old with suspected new onset of ankylosing spondylitis 5. 15 year old with Crohn's disease 6. 10 year old with an exacerbation of asthma

4., 5., & 6. Correct: The reassigned nurse should be assigned clients that have diagnoses similar to adults that medical surgical nurses are used to caring for. This would promote a safe environment for the clients at the reassigned nurse's skill level. These clients all have adult diseases that this nurse would be aware of. Ankylosing spondylitis is a type of arthritis that involves long term inflammation of the spine and joints, causing pain and decreased flexibility. Crohn's disease is a chronic disease that results in inflammation and irritation in the GI tract, most commonly the small intestine and ileum. It has periods of exacerbation and remissions. The client has chronic diarrhea, abdominal pain, weight loss and nutritional deficiencies. Asthma is a chronic disease characterized by inflammation in the airways, resulting in hyperresponsiveness, mucosal edema, and increased mucus production. The client has a tight, irritative cough, wheezing with chest tightness, and dyspnea. A medical surgical nurse should possess the knowledge and skills needed to care for these clients. 1. Incorrect: Cystic Fibrosis is basically a childhood disease, even though some clients do live into adulthood now. The pathophysiology and associated care may not be familiar to the reassigned nurse. Cystic fibrosis is an exocrine gland dysfunction and causes multisystem involvement. The client has increased viscosity of mucous, elevated sweat electrolytes, and autonomic nerve dysfunction. The pancreas and bronchioles are common areas obstructed by the mucous. 2. Incorrect: A 6 month old with RSV needs different care than an adult. This child would require respiratory isolation. The airways of infants are much smaller and therefore, more readily occluded by mucous. This infant would need the assessment and treatment skills from the pediatric nurse. The medical surgical nurse would not need to be assigned to care for this infant. 3. Incorrect: Hirschsprung disease is specific to children and is also known as aganglionic megacolon. It is present at birth and is characterized by an inability to pass stools. The affected part of the colon that is missing the ganglion cells does not relax to allow stool to pass. Surgery to remove the aganglionic part of the intestines is often done within the first few days or months of life, soon after the disease is diagnosed.

Which action by an unlicensed nursing assistant would require the nurse to intervene? 1. Collecting I & O totals for unit clients at the end of shift. 2. Elevating the head of the bed 30°- 40° for the client post thoracotomy 3. Ambulating a client who is 2 days post vaginal hysterectomy 4. Turning off continuous tube feeding to reposition a client, then turning the feeding back on

4. Correct: The unlicensed nursing assistant should not turn tube feedings off or on. The nurse should do this when repositioning is needed. Prior to turning feeding back on, tube placement needs to be verified. 1. Incorrect: Obtaining the urinary output of a client at the end of the shift is appropriate for the nursing assistant and should be documented and reported to the RN. 2. Incorrect: This is appropriate because this position will improve gas exchange and breathing for a client after thoracic surgery. 3. Incorrect: The hysterectomy client needs to be ambulated to avoid post op complications. This is an appropriate and safe action for the unlicensed nursing assistant to do.

The nurse is preparing to administer nadolol to a hospitalized client. Which client data would indicate to the nurse that the medication should be held and the primary healthcare provider notified? 1. Blood pressure 102/68 2. Glucose 118 mg/dL 3. UOP 440 mL over previous 8 hour shift. 4. Heart rate 56/min

4. Correct: This is a beta blocker. If a client's heart rate is less than 60 beats per minute, notify the primary healthcare provider and ask if the client should receive this medication. You can identify that nadolol is a beta blocker because it ends in "lol". 1. Incorrect: Beta blocker are prescribed to lower BP. When the baseline BP is not known, worry about a BP of 90/60 or below. If the client's BP drops below 90/60, this beta blocker should be held and the primary healthcare provider notified. 2. Incorrect: This is a normal glucose level. If the client is a diabetic, beta blockers can mask the signs of hypoglycemia. 3. Incorrect: Urinary output is adequate. Beta blockers do not alter renal function.

The nurse has been talking with a depressed client at an outpatient clinic. When asked how the client feels to live alone, the client simply stares straight ahead. How should the nurse respond? 1. Ask, "Why won't you answer me?" 2. Leave the client alone for awhile. 3. Tell a joke to lighten the mood. 4. Use therapeutic silence.

4. Correct: Use of silence allows the client time to think over what he or she wants to say and gives the client a chance to collect thoughts. 1. Incorrect: This is not therapeutic and appears aggressive and confrontational. 2. Incorrect: This is not therapeutic. Depressed clients may need extra time to formulate their thoughts. 3. Incorrect: This is not therapeutic and demonstrates disregard for the client's feelings.

A nurse is caring for a pediatric client who has been diagnosed with hypothyroidism. What is essential for the nurse to teach the parents of this child? 1. Administer the liquid medication with soy milk. 2. Notify primary healthcare provider of slow heart rate. 3. Monitor glucose before meals and at bedtime. 4. Wait 4 hours after giving medication before giving iron supplements.

4. Correct: Wait for 4 hours before giving child iron supplements, antacids that contain calcium or aluminum hydroxide, or calcium supplements as it interferes with medication. 1. Incorrect: Give the medication with a liquid, except soy milk, which interferes with the ability to absorb the thyroid hormone. 2. Incorrect: Bradycardia is seen with hypothyroidism. When taking thyroid medication, we want to watch for signs of hyperthyroidism such as tachycardia, rapid weight loss, sweating, restlessness. 3. Incorrect: Hypothyroidism does not affect glucose.

A nurse is providing care to a post-operative parathyroidectomy client. Which occurrence takes highest priority? 1. Psychoses 2. Renal calculi 3. Positive Trousseau's sign 4. Laryngospasm

4. Correct: Yes, airway is most important here. But don't pick it just because it sounds scary all by itself. Think about the why. When the parathyroids are removed, calcium is affected because these glands help control calcium levels in the blood. 1. Incorrect: This is disturbing, and important, but AIRWAY is priority. 2. Incorrect: Renal calculi can cause problems and lead to pain and possibly renal failure but are not as important as airway obstruction. 3. Incorrect: A positive Trousseau's sign is seen with hypocalcemia but is not the highest priority. Airway is the most important in this question.

The nurse leader is planning to change the method of client documentation on the unit. Some employees accept the change without difficulty; however, some of the employees are resistant to change and try to sabotage the plans for change. Which action should the nurse leader take to reduce resistance to change on the unit? 1. Allow staff on the unit a voice in the plan for change. 2. Discourage discussion between supporters and resisters. 3. Set an implementation date and begin the new method. 4. Announce that the plan for change is set by administration.

1. Correct: Allowing everyone an opportunity to speak may reveal the reasons behind the resistance. If everyone has a voice, each person is more likely to buy into the new method. 2. Incorrect: Supporters and resisters should communicate. Perhaps the supporters can persuade the resisters. Encouraging discussion keeps communication lines open and is more likely to decrease resistance. 3. Incorrect: Setting a date for implementation should come after discussion and training on the new process. A target date must be set; however, the groundwork for change must occur first. 4. Incorrect: Staff is more likely to accept change that affects them if they have a voice. Administration can take staff suggestions and possibly make a better plan.

Which referral would the nurse anticipate that the primary healthcare provider would make for a client who has difficulty eating using regular utensils? 1. Occupational therapist 2. Physical therapist 3. Rehabilitation nurse 4. Registered Dietitian

1. Correct: An occupational therapist helps physically disabled clients adapt to physical limitations and is most qualified to help clients improve their ability to perform activities of daily living. OT's help clients learn to approach tasks differently, use assistive devices or equipment, make adaptations to the home or work environments and find ways to assist the client in meeting personal goals. 2. Incorrect: The physical therapist is trained to deal with problems that limit their abilities to move, perform daily functions, or remain active and independent. However, physical therapists do not assist with special adaptations needed to perform activities of daily living such as eating. 3. Incorrect: A rehabilitation nurse can help a client eat, but isn't trained in modifying utensils. The rehabilitation nurse assists clients as they adapt to altered lifestyles and assists clients to attain and maintain the highest level of functioning. Some of the aspects included in the role of the rehab nurse includes encouraging self care, preventing complications and further disability, setting goals for independent functioning, and assisting clients to access additional care needed. The rehabilitation nurse would work collaboratively with the occupational therapist (OT). The OT is the one who will best meet the needs of this client who is experiencing difficulty eating with regular utensils. 4. Incorrect: A registered dietitian manages and plans for the nutritional needs of clients but isn't trained in modifying or fitting utensils with assistive devices. This would be the role of the OT.

A client who only speaks Spanish is admitted to the surgical unit. What is the best method for the nurse to inform the client about a pre-surgical procedure? 1. Use an audiotape made in Spanish to inform the client of the pre-surgical procedure. 2. Draw pictures of what to the client can expect prior to surgery. 3. Facial expressions and gestures can be used to let the client know what to expect. 4. Enlist the help of a Spanish speaking family friend to tell the client what to expect prior to surgery.

1. Correct: Audiotapes made in the language of high volume clients who speak a language other than English is helpful to inform clients about admission procedures, room and unit orientation, and pre-surgical procedures. 2. Incorrect: This is not the best option. Some pre-surgical procedures may be difficult to draw or difficult to understand. 3. Incorrect: This is called "Getting by" and may have to be used when the nurse cannot speak the client's language, without interpreters, audiotapes, or written materials available to inform the client in their language. This is not the best option. 4. Incorrect: Disadvantages of using ad hoc interpreters include compromising the client's right to privacy and relying on someone without training as an interpreter. They may leave out important words, add words, or substitute terms that make communication inaccurate.

The nurse is reviewing medications for a client who is being treated for major depression. The client is prescribed a selective serotonin reuptake inhibitor (SSRI). Which over the counter medication/supplement taken by the client should be reported to the primary healthcare provider immediately? You answered this question Correctly 1. Daily intake of St. John's Wort. 2. Daily intake of a multi-vitamin. 3. Occasional use of ibuprofen. 4. Twice daily intake of an antacid.

1. Correct: St. John's Wort is an herbal supplement often used in the treatment of mild depression. It should not be taken in combination with a selective serotonin reuptake inhibitor due to the risk of serotonin syndrome, which can be fatal. 2. Incorrect: A multi-vitamin taken with an SSRI poses no risk. 3. Incorrect: This medication taken with the SSRI would not warrant immediate reporting to the primary healthcare provider. 4. Incorrect: Antacids would not require immediate reporting.

When providing instructions, the nurse asks the client to repeat the techniques for crutch walking. The nurse is aware that further teaching is needed when the client makes which statement? 1. "The elbows should be flexed at 10 degrees." 2. "I should not lean on the crutches with my armpit." 3. "When going upstairs, my non-surgical leg goes up first." 4. "Both crutches are held in one hand when sitting down".

1. Correct: The nurse is looking for an incorrect statement from the client. This statement indicates the client will need further instruction prior to discharge. When using crutches, the client's elbows should be flexed at 30 degrees. 2. Incorrect: This is a correct statement by the client. The weight of the body is placed on the hands and handgrips rather than being supported by the armpits, which could cause axillary nerve damage. This is a correct statement by the client; however, the question asks for an incorrect statement by the client. 3. Incorrect: The client is aware that the non-surgical "good" leg should be placed on the steps first when going upstairs, while the surgical "bad" leg is placed on the stairs first when coming down steps. This is a correct statement, indicating that the client did understand teaching; however, this question is looking for an indication that the client needs further instructions. 4. Incorrect: When sitting down in a chair, the client would indeed place both crutches in one hand while safely reaching for the chair with the free hand. This is a correct statement and does not indicate the need for further teaching.

The nurse is reviewing the medication prescriptions with a client for which English is a second language (ESL). Which nursing intervention most likely will prevent a medication error with this client? 1. Use the teach-back method so that client is repeating the instructions back to the nurse. 2. Give printed information to the client. 3. Ask the client if they have questions before the client leaves the healthcare setting. 4. Refer medication questions to the pharmacist.

1. Correct: The teach-back method of asking the client to repeat the teaching instructions to the nurse will most likely reveal any misunderstanding. This allows the nurse to reinforce any areas where clarification is needed. 2. Incorrect: Printed information may or may not be helpful, depending on the client's level of understanding. 3. Incorrect: The client may not know which questions to ask regarding the medication, particularly if there is a language barrier. 4. Incorrect: The client may not ask another person for help. There has been no relationship established with the pharmacist since the nurse has been providing the teaching. The nurse should not put this responsibility on someone else in the interdisciplinary team.

A farm worker comes into the clinic reporting headache, dizziness, and muscle twitching after working in the fields. What condition does the nurse suspect? 1. Pesticide exposure 2. Heat stroke 3. Anthrax poisoning 4. Gastroenteritis

1. Correct: These are symptoms of pesticide exposure when combined with the details given of coming from the fields. Death can result from severe acute pesticide poisoning. 2. Incorrect: The data provided does not lead the nurse to suspect heat stroke. The stem does not tell the temperature the farmer is working in. Heat stroke signs and symptoms include increased sweating, tachypnea and temperature greater than 105.8°F (41.0°C). 3. Incorrect: The data provided does not lead the nurse to suspect anthrax poisoning. The worker has been outside in a field. This is not a risk factor for anthrax exposure. Inhalation anthrax develops when you breathe in anthrax spores. It's the most deadly way to contract the disease, and even with treatment it is often fatal. Initial signs and symptoms of inhalation anthrax include: Flu-like symptoms, such as sore throat, mild fever, fatigue and muscle aches, which may last a few hours or days. Mild chest discomfort, Shortness of breath, Nausea, Coughing up blood, Painful swallowing 4. Incorrect: The data provided does not lead the nurse to suspect gastroenteritis. These signs and symptoms do not go with gastroenteritis. Gastroenteritis signs and symptoms include diarrhea, nausea, vomiting, fever and abdominal cramping.

The school nurse has educated a group of teens concerned about acquiring the Ebola virus. Which statement by the students would indicate to the nurse that further teaching is necessary? 1. "I can get a vaccine to prevent getting the Ebola virus." 2. "Ebola is not spread through casual contact, so my risk of getting the virus is low." 3. "The Ebola virus is passed from person to person through blood and body fluid." 4. "Ebola viruses are mainly found in primates in Africa."

1. Correct: This is an incorrect statement. At present, there is no vaccine to prevent Ebola. 2. Incorrect: This is a correct statement about the Ebola virus. Ebola is not spread through casual contact. 3. Incorrect: This is a correct statement about the Ebola virus. Ebola virus is passed from person to person through blood and body fluid. 4. Incorrect: This is a correct statement about the Ebola virus. Ebola viruses are mainly found in primates in Africa.

A client receiving torsemide 20 mg every day reports an onset of cramping in the lower extremities. Based on this report, what current lab finding would the nurse expect? 1. Potassium level of 3.1 mEq/L (3.1 mmol/L) 2. Calcium level of 11 mg/dL (2.75 mmol/L) 3. Sodium level of 140 mEq/L (140 mmol/L) 4. pH level of 7.40

1. Correct: Torsemide is a loop diuretic, which causes the excretion of K+. Hypokalemia can result from use of this diuretic. Normal range for potassium is 3.5 to 5.0 mEq/L (3.5 to 5.0 mmol/L). Therefore the level of 3.1 mEq/L (3.1 mmoL/L) is hypokalemia, and a common sign and symptom includes muscle cramps. 2. Incorrect: Normal calcium levels in the serum are 9.0-10.5 mg/dL (2.25-2.62 mmol/L). The level of 11 mg/dL (2.75 mmol/L) is hypercalcemia. Calcium acts like a sedative, so you would expect the client's muscle tone to be weak and flaccid rather than experiencing muscle cramping. 3. Incorrect: The normal sodium level is 135 to 145 mEq/L (135 to 145 mmol/L). Therefore, a level of 140 mEq/L (140 mmol/L) is WNL and would not be a factor in the client's report of muscle cramping. 4. Incorrect: The pH level of 7.40 is also WNL and is not a lab finding that would be consistent with muscle cramping.

Which signs and symptoms would indicate to the nurse that the client is having an anaphylactic response after receiving penicillin? 1. Reports a scratchy throat 2. Faint expiratory wheeze on auscultation. 3. Client statement, "I feel like something is wrong." 4. Bounding radial pulse rate of 100/min 5. BP 100/70

1., 2. & 3. Correct: Swelling of face, mouth, throat, and a scratchy throat are indicative of an inflammatory response that could obstruct the airway. Wheezes and stridor are indicators of breathing difficulties seen with anaphylactic reaction. A sense that something bad is happening should serve as a warning that something bad is really going on. Suspect anaphylactic response. 4. Incorrect: The pulse rate would be increased, but the client would have a thready, weak pulse, not bounding. The pulse may also be irregular. 5. Incorrect: This blood pressure is not below 90 systolic which could indicate shock. Although on the low side, simply getting this BP reading does not tell you if perfusion is adequate. Once blood pressure decreases, other symptoms may appear such as dizziness, blurred vision and loss of bladder/bowel control.

A client who has been trying to lose weight reports to the nurse that it is just easier to stop by the fast food restaurant on the way home from work than to go home and prepare a meal. Which interventions could help the client stay on track? 1. Suggest that the client eat yogurt and a piece of fruit upon returning home. 2. Suggest that the client order low fat options at the restaurants. 3. Encourage the client to pack a healthy snack to eat on the way home from work. 4. Inform the client that fast food restaurants do not have healthy food options. 5. Suggest that the client alter her route home from work in the evenings to avoid the fast food restaurants.

1., 2. & 3. Correct: The client is describing lack of convenience, a barrier to making better choices. The client can consume yogurt and fruit on the way home and still be making a good choice for dinner. Accessibility of healthier items will help the client stay on track. Availability of healthy foods will help the client stay on her food plan. 4. Incorrect: There are healthier choices currently at most fast food restaurants. Clients should be encouraged to choose from those. 5. Incorrect: While this may help the client stay on track, it may make a healthy choice inaccessible. The client is more likely to make healthy choices when they are accessible, available, and affordable. Options 2 and 4 are opposites, so one must be wrong. Option 4 is false; more restaurants are serving low fat, low calorie meal choices. Option 2 is true. Identify client-centered options. The focus of the nurse should be the client. Items that test your ability to be client-centered tend to explore client feelings, identify client preferences, empower the client, provide the client choices, or in some way put emphasis on the client. The client is the priority. Option 5 is false. It does not take into account the client's desired route home or that preparing a meal is inconvenient. Options 1 and 3 are true. These options are easy, convenient and healthy alternatives to fast food choices.

The nurse is teaching the family of a homebound client about ways to increase the client's safety while bathing independently. Which strategies should the nurse include? 1. Install grab bars in the tub or shower. 2. Install hand bars on sides of tub. 3. Use tub/shower seat for bathing. 4. Provide a long handled bath scrubbie for bathing. 5. Schedule bathing routines three times per week

1., 2., 3. & 4. Correct: Grab bars will assist the client in getting into or out of the tub or shower, thus reducing the chance for falls. Hand bars are very helpful as one enters or exits the tub. The increased stability offered by these devices reduces risk of falls. Using a shower seat will allow the client to remain independent in terms of entering or exiting the tub or shower. The use of handled scrubbies or sponges allows the client to reach lower extremities or back with greater ease. 5. Incorrect: The bathing routine may need to be more often than three times per week depending on the client. The bathing schedule does not relate to a client's independence.

What assessment data would a nurse expect to find in a client diagnosed with acute inflammatory bowel disease? 1. Bloody stools that contain mucus 2. Pallor 3. Anorectal excoriation 4. Urine output below 30 mL/hr 5. Increased serum prealbumin

1., 2., 3., & 4. Correct What do we know about acute inflammatory bowel disease? Inflammatory bowel disease (IBD) involves chronic inflammation of all or part of the digestive tract. IBD primarily includes ulcerative colitis and Crohn's disease. Both usually involve severe diarrhea and bloody stools, abdominal pain with cramping, fatigue, decreased appetite and weight loss. IBD can be debilitating and sometimes leads to life-threatening complications. Now, look at the options. Option 1 is true. We just said blood stools. And yes they contain mucus because of the damage occurring to the intestinal lining. Option 2. True. These clients tend to be deficient in iron causing iron deficient anemia. Pallor is a sign of anemia. Option 3. If you had a lot of diarrhea, would your rectal area be excoriated? Yes. Option 4 Low urine output? True. Dehydration, right? right. Option 5. If the client is not eating, the prealbumin would be decreasing not increasing, so this option is false.

What discharge education should a nurse provide to a client post hip replacement with a metal joint? 1. Weight bearing limits. 2. Use of a high seated chair. 3. Sexual intercourse in dependent position for up to six months. 4. Avoid taking showers. 5. Use of long handled tongs to assist with dressing.

1., 2., 3., & 5. Correct: Weight bearing limits on the involved extremity varies according to the healthcare providers preference but are commonly prescribed. The client needs to avoid flexion. This includes sitting in low chairs and getting into a bath tub; elevated toilet seats and raised seats are necessary. Sexual intercourse should be carried out with the client in a dependent position (flat on the back) for 3-6 months to avoid excessive adduction and flexion of the new hip. To avoid flexion when dressing, adaptive devices and utensils may be used to help with bathing, dressing and personal hygiene. 4. Incorrect: Showers are preferable as getting into a tub would cause flexion of the new hip. This could cause the hip to dislocate.

A client is being evaluated for possible Rheumatoid Arthritis (RA). Which lab data and assessment findings by the nurse would be indicative of RA? 1. Joint pain, swelling, and warmth. 2. Decreased movement in joints. 3. Presence of Rheumatoid factor on lab analysis. 4. Presence of Dupuytren's contractures. 5. Elevated erythrocyte sedimentation rate (ESR). 6. Presence of Cyclic Citrullinated Peptide Antibody.

1., 2., 3., 5., & 6. Correct: Classic features of RA include joint pain, swelling, and tenderness worsened by movement and stress placed on joint. Morning stiffness that often lasts for one hour or more and limited movement in joints are common manifestations as well. The Rheumatoid Factor is present in 80% of adults who have rheumatoid arthritis. The ESR blood test is elevated with RA and is used to determine if an abnormal level of inflammation exists in the body. The cyclic citrullinated peptide antibody, if present, helps to confirm the diagnosis of RA and may indicate the risk of having severe symptoms. Levels that are at a moderate to high level may indicate that the client is at increased risk for damage to the joints. 4. Incorrect: Dupuytren's contractures are a type of hand deformity where a layer of tissue under the skin in the palms of the hands is affected. Hard knots form in the palm areas and eventually create a thick cord that can pull one or more of the fingers into a bent position. However, this is not associated with RA. So signs and symptoms of RA include redness, swelling, tenderness, and pain in the joints. Morning stiffness can occur or 30 minutes or more. Along with pain, many people experience fatigue, loss of appetite and a low-grade fever. RA can also affect the eyes, mouth, skin, lungs, blood and blood vessels. Look at the options. Option 1: Joint pain, swelling, and warmth. True. These are classic features of RA. Option 2: Decrease movement in joints. True. Pain, swelling, and tenderness are worsened by movement and stressed placed on the joint. Remember, morning stiffness and limited movement is common. Option 3: Presence of Rheumatoid factor on lab analysis. True. Rheumatoid factor is an antibody found in about 80 percent of people with RA during the course of their disease. Option 4: Presence of Dupuytren's contractures. False. Dupuytren's contractures are a type of hand deformity where a layer of tissue under the skin in the palms of the hands is affected. Hard knots form in the palm areas and eventually create a thick cord that can pull one or more of the fingers into a bent position. However, this is not associated with RA. Option 5: Elevated ESR. True. The ESR blood test is elevated with RA and is used to determine if an abnormal level of inflammation exists in the body. Option 6: Presence of Cyclic Citrullinated Peptide Antibody. True. The cyclic citrullinated peptide antibody, if present, helps to confirm the diagnosis of RA and may indicate the risk of having severe symptoms. Levels that are at a moderate to high level may indicate that the client is at increased risk for damage to the joints.

The nurse has been teaching the client about warfarin for prevention of pulmonary emboli. Which comments by the client indicate understanding of the medication? 1. "I must get my blood levels checked regularly." 2. "I shouldn't change my diet to include a lot of foods containing vitamin K without supervision." 3. "I should eat lots of foods containing vitamin K." 4. "I should report this medication to any primary healthcare provider that I see." 5. "I should not change the dosage without talking with my primary healthcare provider."

1., 2., 4. & 5. Correct: The client should comply with regular follow up visits for checks of INR level. INR is the international normalization ratio and is used for clients taking anticoagulants (blood thinning medications). The client should eat a normal healthy diet, but should not increase foods containing high amounts of vitamin K. The client should report using warfarin to any primary healthcare provider, as treatment may be changed due to this medication. The client should not manipulate the dosage unless instructed by the primary healthcare provider. An identification card or bracelet may also be recommended in case of emergencies. Clients should inform dentists and other healthcare providers especially before a medical procedure. The anticoagulant effect must be closely monitored. 3. Incorrect: Vitamin K reverses the anticoagulant effects of warfarin, so instruct the client to avoid foods high in vitamin K (examples are green leafy vegetables, brussels sprouts, prunes, cucumbers and cabbage). Do you know what warfarin is? Well you know from the stem that it is prescribed for the prevention of pulmonary emboli. How? Warfarin is an anticoagulant that reduces the formation of blood clots by inhibiting vitamin K dependent coagulation factors. So know you have an idea of what options could be correct. Let's look at them. Option 1. What do you think? True. Too much warfarin can lead to bleeding/hemorrhage. The client needs regular follow up visits to check the INR level. INR is the international normalization ratio and is used for clients taking anticoagulants. Option 2. This is true. What would happen if the client ate too much vitamin K? Vitamin K is the antidote for warfarin. So the client's INR level would not be therapeutic. Can you say pulmonary emboli?! So the client should eat a normal healthy diet, but should not increase foods containing high amounts of vitamin K. Option 3. Well if option 2 is true, can option 3 be true? No! Watch out for opposites. They cannot both be correct. Vitamin K reverses the anticoagulant effects of warfarin, so instruct the client to avoid foods high in vitamin K (examples are green leafy vegetables, brussels sprouts, prunes, cucumbers and cabbage). Option 4. This is a safety issue isn't it? Yes. This is correct. In fact, a list of all medications should be provided to any healthcare providers caring for a client. What about option 5? This is True. The client should not manipulate the dosage unless instructed by the primary healthcare provider. An identification card or bracelet may also be recommended in case of emergencies.

Which food items, if chosen by a client diagnosed with diverticulosis, would indicate to the nurse that the client understands the prescribed diet? 1. Avocados 2. Acorn squash 3. Applesauce 4. Lima beans 5. Raspberries 6. Cottage cheese

1., 2., 4., & 5. Correct: High fiber foods include raw fruits, legumes, vegetables, whole breads, and cereals. Avocados have 10.5 grams of fiber per cup. Acorn squash has 9 grams of fiber per cup. Lima beans 13.2 grams of fiber per cup. Raspberries have 8 grams of fiber per cup. 3. Incorrect: Raw fruits have more fiber than cooked or processed fruits. A raw apple would provide more fiber than applesauce. 6. Incorrect: Milk and foods made from milk: such as yogurt, pudding, ice cream, cheeses, cottage cheese and sour cream are low fiber. The average adult only eats 15 grams of fiber per day. Women need 25 grams of fiber per day, and men need 38 grams per day, according to the Institute of Medicine. Total dietary fiber intake should come from food. A high fiber diet Is prescribed in the prevention or treatment of a number of gastrointestinal, cardiovascular, and metabolic diseases. Diverticulosis is one disease where a high fiber diet may help to relieve symptoms. But do not get confused. Diverticulosis is high fiber. Diverticulitis (active inflammation) needs low fiber! Increased fiber should come from a variety of sources including fruits, nuts, legumes, vegetables, whole breads, and cereals. Top sources of fiber are: beans (all kinds), peas, chickpeas, black-eyed peas, artichokes, whole wheat flour, barley, bulgur, bran, raspberries, blackberries, and prunes. Good sources of fiber include: lettuce, dark leafy greens, broccoli, okra, cauliflower, sweet potatoes, carrots, pumpkin, potatoes with the skin, corn, snap beans, asparagus, cabbage, whole wheat pasta, oats, popcorn, nuts, raisins, pears, strawberries, oranges, bananas, blueberries, mangoes, and apples. Avoiding refined grains -- such as white flour, white bread, white pasta, and white rice -- and replacing them with whole grains is a great way to boost the amount of fiber.

Which signs/symptoms would lead a nurse to suspect Fifth disease in a child brought into a pediatric clinic? 1. Erythema on the cheeks. 2. Joint pain. 3. Temperature 102°F (38.88°C). 4. Swollen knees. 5. Pruritic rash on soles of feet.

1., 2., 4., & 5. Correct: These are common signs/symptoms of Fifth disease. 3. Incorrect: Low grade fever is seen with this disease. What is Fifth's Disease? It is a mild rash illness caused by parvovirus B19. It is more common in children than adults. A person usually gets sick with fifth disease within 4 to 14 days after getting infected with parvovirus B19. After several days, the client may get a red rash on the face called "slapped cheek" rash. This rash is the most recognized feature of fifth disease. Some people may get a second rash a few days later on their chest, back, buttocks, or arms and legs. The rash may be itchy, especially on the soles of the feet. The client may also have painful or swollen joints. So let's look at options to see which are true. Option 1 is true. Remember that "slapped cheek" rash. Option 2 is true. The client can develop joint pain. Option 3 is false. The client may have a low grade fever, not a high fever such as 102ºF or 39ºC Option 4 is true. Swollen knees. That's swollen joints. Option 5 is true. Itching on the soles of the feet.

When caring for a client on bedrest, which interventions should the nurse implement to decrease the risk of deep vein thrombosis? 1. Apply compression hose. 2. Place pillow under knees while supine. 3. Assist client to perform active foot and leg exercises. 4. Place client on intermittent pneumatic compression device. 5. Assess extremities for negative Homan's sign.

1., 3., & 4. Correct. The client will need compression or compression hose and/or intermittent pneumatic compression device. The client should perform leg and foot exercises to decrease stagnation of blood. Compression hose, foot and leg exercises as well as pneumatic compression devices increase venous return and prevents stasis of blood. Other interventions to decrease deep vein thrombosis (DVT) include early ambulation, passive and active range of motion, isometric exercises and anticoagulant drugs such as heparin. 2. Incorrect: Do not compromise blood flow by placing pillows under the knees, crossing legs, or sitting for long periods of time. Pillows under the knees help with pressure on the lower back. However, if pillows are left under the knees for an extended time, venous return could be compromised. A pillow under the knees is not a recommended intervention for DVT prevention. 5. Incorrect: Do not assess Homan's sign, as it may dislodge a clot. Homan's sign is not a preventative intervention. Assessing a Homan's sign is considered to be controversial, and this test may contribute to the release or dislodgement of a clot.

What should the nurse check when assessing a client's balance? 1. Walking on tiptoes 2. Babinski reflex 3. Romberg test 4. Muscle strength of legs 5. Dorsalis pedis pulses

1., 3., & 4. Correct: Asking the client to walk on the tips of the toes assesses foot strength and balance. Muscle strength is needed to maintain balance and a Romberg's test asks the client to stand erect with arms at their side and feet together. The nurse notes any sway or unsteadiness. Then the client does the same thing with their eyes closed for 20 seconds again noting imbalance and sway. A positive Romberg is seen with swaying and moving feet apart to prevent a fall. It indicates a problem with balance. 2. Incorrect: Babinski sign is an important neurologic examination based upon what the big toe does when the sole of the foot is stimulated. If the big toe goes up, that may mean trouble with the central nervous system. This is not part of assessment for balance 5. Incorrect: Assessing the dorsalis pedis pulse is done as part of a circulatory check not while assessing balance.

When performing an admission assessment, what should the nurse recognize as signs/symptoms of hyperthyroidism? 1. Nervousness 2. Weight gain 3. Exophthalmos 4. Loss of appetite 5. Constipation 6. Hot and sweating

1., 3., & 6. Correct: With hyperthyroidism, the client has too much energy. They report being nervous and feeling hot. Exophthalmos is an irreversible eye condition where the eyes bulge. This condition is associated with hyperthyroidism that has not been treated early enough to prevent this from occurring. Due to the hypermetabolic state, the client will often report feeling hot and will be sweating. 2. Incorrect: The client with hyperthyroidism has an increased appetite but experiences weight loss due to the excessive energy consumption. 4. Incorrect: Loss of appetite is seen in the client with hypothyroidism. The client with hyperthyroidism has an increased appetite but experiences weight loss due to the excessive energy consumption. 5. Incorrect: Constipation is a sign of hypothyroidism due to slowed GI motility. In hyperthyroidism, the nurse would expect increased GI motility.

Which task would be appropriate for the nurse to delegate to an unlicensed assistive personnel (UAP)? 1. Check client for signs of skin breakdown. 2. Check client's vital signs after ambulating. 3. Administer 8 ounces of polyethylene glycol electrolyte solution every 10 minutes. 4. Obtain a stool specimen. 5. Determine what activities the client can do independently.

2. & 4. Correct. These tasks are within the scope of practice for the UAP. 1. Incorrect. The UAP cannot assess the client for signs of skin breakdown. 3. Incorrect. The UAP cannot administer medication. 5. Incorrect. The UAP cannot assess which activities that the client can perform.

Question 22: A client has been admitted with a stroke on the right side of the brain. What clinical manifestations does the nurse expect to find when assessing this client? 1. Right sided hemiplegia 2. Impaired judgment 3. Depression 4. Impaired language comprehension 5. Impulsiveness 6. Impaired speech

2. & 5. Correct. The client with right sided brain damage will have left sided hemiplegia and will exhibit impulsive behavior and impaired judgment. 1. Incorrect. This is seen with left-brain damage. 3. Incorrect. This is seen with left-brain damage. 4. Incorrect. This is seen with left-brain damage. 6. Incorrect. This is seen with left-brain damage.

A home health nurse is visiting an adolescent with a myelomeningocele. The nurse realizes more instruction is needed when the client makes what statement? 1. "I might need to get glasses." 2. "I catheterize myself twice a day." 3. "I drink bottled water all day long." 4. "I do upper arm exercises every day."

2. CORRECT: The nurse is looking for a statement that indicates the need to do further instruction for this client. Self-catheterization should be completed every four to six hours. Allowing urine to remain in the bladder longer than six hours greatly increases the risk of infection. This comment indicates that the adolescent needs more instruction on the importance of timing catheterization. 1. INCORRECT: The potential need for glasses is not an issue that the nurse is in a position to evaluate. There are additional health issues the client may encounter, such as the possible need for glasses. But the nurse is not qualified to determine visual needs. 3. INCORRECT: This is a factual statement and requires no further instruction. It is very important for clients with spina bifida to stay well hydrated throughout the day, no matter what type of water is used. 4. INCORRECT: Those with spina bifida often need to use either a wheelchair or bilateral crutches over the course of their life. Maintaining upper body strength is crucial for mobility and decreasing the possibility of complications. This is an accurate statement that will not need addressed.

A community health nurse prepares a presentation about decreasing the risk of the spread of influenza in the community. Which information should the nurse include in the presentation? 1. The flu is spread via the influenza vaccine. 2. Use a shirtsleeve when coughing or sneezing. 3. Tissues are the most effective means to decrease the spread of the influenza. 4. Antibiotics are effective in treating influenza.

2. Correct: A shirtsleeve should be used as a barrier when coughing or sneezing. This prevents germs being spread via the hands. 1. Incorrect: The flu vaccine contains a dead virus that is not capable of causing the flu. Clients may experience flu-like symptoms from the flu vaccine, but they won't contract the virus. 3. Incorrect: Tissues are effective in decreasing the spread of the flu if disposed of in the trash after use. Hand washing is also very important in decreasing the spread of germs. 4. Incorrect: Antibiotics are not effective in treating the flu. The flu is treated with antipyretics, fluids, and rest. Antibiotics are used for infections, not viruses.

What would be most important for the nurse to teach parents in order to promote sleep and rest in the preschool child? 1. Allow the child to choose own bedtime based on degree of fatigue. 2. Develop a consistent routine before going to bed. 3. Assess how much sleep the child requires. 4. Set a consistent wake-up schedule.

2. Correct: A consistent routine helps to prepare the child for sleep. Reading or telling stories before bedtime may help the child to relax and fall asleep more easily. Routines are very important for this age group. Doing specific things before bedtime can signal to the child that it is time to get ready for bed and to go to sleep. 1. Incorrect: Although important, this is not the priority. Establishing a routine is most important. A cool environment will promote rest. A child's sleep cycle is sensitive to light and temperature. Melatonin levels help to regulate the drop in internal temperature needed to sleep. 3. Incorrect: Assessing the amount of sleep needed can help with promoting sleep and rest but routine is priority in the preschool age group. 4. Incorrect: Setting a wake-up time prevents a child from over sleeping on weekends and holidays. Those extra hours can disturb the sleep cycle. For a preschooler routine is the priority answer to promote sleep and rest at night.

The nurse is talking with the spouse of an alcoholic client. Which statement by the client's spouse is evidence of codependent behavior? "I frequently tell my spuse that drinking alcohol is ruining our relationship." 2. "I go and pick my spouse up from the bar when not home by midnight." 3. "I do not go out drinking with my spouse, and will not drink at home either." 4. "I have told my spouse that I am willing to attend a counseling session when my spouse wants to stop drinking."

2. Correct: The spouse is attempting to please the alcoholic client. Codependent people are people pleasers, and they make excuses for others. The spouse is enabling the client to continue to drink. The spouse may feel keeping the client from driving while intoxicated will keep people safe. 1. Incorrect: This is a response by a person who is not codependent. This person is not afraid to show feelings and does not deny that there is a problem. 3. Incorrect: By not drinking with the client, the spouse shows that this behavior is not condoned. 4. Incorrect: Again, the spouse does not deny a problem and wants to help the client quit rather than making excuses.

The home care nurse is admitting a new client with a diagnosis of COPD, atrial fibrillation and gout. After reviewing the client's medication list, the nurse would arrange for periodic monitoring of blood drug levels for which of the following medications? (Select all that apply.) Beclomethasone inhaled (Qvar) Digoxin (Lanoxin) Theophylline (Elixophyllin, Theo-24, Uniphyl) Allopurinol (Aloprim, Zyloprim) Glipizide (Glucotrol)

Digoxin (Lanoxin) Theophylline (Elixophyllin, Theo-24, Uniphyl) It is necessary to monitor blood levels for the client taking theophylline and digoxin to prevent the client from developing toxicity.

A RN is observing an unlicensed nursing personnel (UAP) feed a client who is on aspiration precautions. Which action by the UAP would require the nurse to intervene? 1. Elevating the head of the bed to a 90 degree angle 2. Instructing the client to lean the head back slightly when swallowing. 3. Adding a thickening agent to liquids. 4. Feeding the client small amounts of food per bite.

2. Correct: This is an incorrect action, and needs intervention by the nurse. The chin should be flexed to prevent the risk of aspiration. A chin down or chin tuck maneuver is widely used in dysphagia treatment to prevent aspiration. 1. Incorrect: This is a correct action. The head of the bed should be elevated which aids in esophageal peristalsis and swallowing is aided by gravity. 3. Incorrect: This is a correct action. Thickened liquids are easier to swallow without aspirating. Drinking liquids thickened will help to prevent choking and stops fluid from entering the lungs. 4. Incorrect: This is a correct action by the UAP, so the nurse does not need to intervene. Smaller amounts of food can be chewed more thoroughly and swallowed with less risk for aspiration.

What teaching points should the nurse include when educating a client how to prevent a venous stasis ulcer? 1. Elevate legs above heart for 5 minutes, twice a day. 2. Perform leg exercises regularly. 3. Wear graduated compression stockings. 4. Treat itching with prescribed topical corticosteroids. 5. Minimize stationary standing.

2., 3., 4., & 5. Correct: Regular leg exercises improve calf muscle function. Wearing graduated compression stockings will help prevent dilation of lower extremity veins, pain, and a heavy sensation in the legs that typically worsens as the day progresses. Itching can cause the client to scratch, which leads to skin breakdown. Topical corticosteroids can decrease itching and should be used as prescribed. Minimize stationary standing as much as possible to decrease pooling of blood in the lower extremities. 1. Incorrect: Elevate legs above heart for 30 minutes three times a day will minimize edema and reduce intraabdominal pressure. Venous stasis is a problem with the flow of blood from the veins of the legs back to the heart. It's also called chronic venous insufficiency. Veins have valves that keep the blood moving in one direction-toward the heart. In venous stasis, the valves in the veins of the leg don't work right. So fluid pools in the legs. This can lead to problems that include varicose veins. Venous stasis is sometimes caused by deep vein thrombosis and high blood pressure inside leg veins. Risk factors include: Elderly, female, overweight, sedentary lifestyle, smoking, family history. Symptoms affect the legs and may include: Swelling, often in the ankles; Varicose veins; itching; Cramping; Ulcers; Aching or a feeling of heaviness; Changes in skin color. Prevention includes wearing compression stockings to reduce swelling and to relieve pain. They also can help venous skin ulcers heal. Other points the nurse should include in teaching: Get more exercise, especially walking. It can increase blood flow; Avoid standing or sitting for a long time, which can make the fluid pool in the legs; Keep legs raised above the heart when lying down. This reduces swelling. Option 1 is false. When lying down elevate the legs above the heart to decrease swelling. Elevate legs above heart for 30 minutes three times a day will minimize edema and reduce intraabdominal pressure. Option 2 is true. Exercising increase blood flow. Option 3 is true. This prevents swelling and relieves pain. Option 4 is true. The primary healthcare provider may prescribe topical steroid creams and ointments to help with itching. Teach the client to avoid using lanolin, calamine and other lotions that dry the skin, topical antibiotic ointments such a neomycin, benzocaine and other numbing medications Option 5 is true. Standing or sitting for a long time, can make the fluid pool in the legs

The nurse is assisting a client with right-sided weakness to transfer from the hospital bed to a wheelchair. The client has an IV attached to an IV pole on the right side of the bed. How should the nurse complete this transfer? 1. Place the wheelchair on the left side of the bed. 2. Place the wheelchair on the right side of the bed. 3. Face the wheelchair toward the foot of the bed. 4. Face the wheelchair toward the head of the bed. 5. Have client grab the wheelchair with the right arm. 6. Have client grab the wheelchair with the left arm.

2., 4., & 6. Correct: The wheelchair should be placed on the right side of the bed where the equipment is located. It needs to face the head of the bed so the client can reach the chair with the strong left arm to help with the transfer. The client should grab the wheelchair arm with the strong left arm. 1. Incorrect: Since the IV and IV pole are on the right side of the bed, the wheelchair should be placed on the right side rather than the left side of the bed. There would not be enough slack in the IV tubing to get out on the left side. 3. Incorrect: If the wheelchair faces the foot of the bed, then the client would not be able to reach with the wheelchair arm with the strong left arm. The client needs to be able to use the left arm for stability. 5. Incorrect: The client should grab the wheelchair arm with the strong left arm. The right side is weak and grabbing with this side puts the client at an increased risk for falls and injury.

The charge nurse on the pediatric unit has several tasks that need completed. What tasks can be assigned to the unlicensed assistive personnel (UAP)? (Select All That Apply) 1. Obtain a urine sample from an infant. 2. Empty a nasogastric (NG) canister for client with ileus. 3. Feed a child with bilateral burns of hands. 4. Change an ostomy appliance on child with stoma. 5. Ambulate an adolescent two days post appendectomy.

3 and 5. CORRECT: A UAP can perform any activities of daily living (ADL), including transfers in or out of bed and ambulation. Feeding clients is considered an ADL which can be performed by a UAP, so feeding a child whose hands are bandaged is an appropriate task. Also, ambulating the adolescent is definitely within the scope of duties for the UAP. 1. INCORRECT: Obtaining a urine sample from an infant is too complex for a UAP. The two methods used for collecting this urine sample is either straight catheterization of the infant or use a "wee bag". Neither of these methods can be performed by a UAP. 2. INCORRECT: Emptying containers can be within the realm of duties for a UAP. However, that does not include a NG canister. A nurse must assess the color, consistency, and amount of drainage in the canister in addition to location and position of the NG tube. This particular action should be completed by a nurse. 4. INCORRECT: Changing an ostomy appliance is a complex task. A nurse needs to assess the skin for evidence of skin breakdown or excoriation that needs treated before another flange is applied. The nurse must also assess the condition of the stoma. This is not a task appropriate for a UAP, although emptying the ostomy bag would be appropriate.

The school nurse is screening the children for scoliosis. At what time of development should the nurse expect to see early findings of scoliosis? During the years when children begin to run and jump During a preadolescent growth spurt! In early infancy before 8 months of age When a child begins to play competitive sports

During a preadolescent growth spurt! Idiopathic scoliosis is seldom apparent before 10 years of age and is most noticeable at the beginning of the preadolescent growth spurt. It is more common in females than in males.

The nurse is reviewing morning laboratory results for multiple clients. Which client laboratory results should the nurse immediately report to the Healthcare provider? 1. Client with chronic obstructive pulmonary disease (COPD) and a PCO2 of 50 mm Hg. 2. Diabetic client with fasting blood sugar of 145 mg/dL (8.0 mmol/L). 3. Cardiac client on furosemide with potassium of 3.1mEq/L (3.1 mmol/L). 4. Client with sepsis and total white blood cell count of 16,000 mm3. 5. Client following a thyroidectomy with calcium level of 8.0 mg/dL (2 mmol/L).

3. & 5. Correct: Although all the laboratory results are outside of standard accepted levels, two particular clients are the most concerning. The cardiac client's potassium level of 3.1 is extremely concerning, since normal potassium levels should be between 3.5-5.0 mEq/L. Hypokalemia can cause muscle weakness and heart arrhythmias, such as PVC's. Secondly, after the client's thyroidectomy, their calcium level is 8.0 mg/dl (normal 9.0-10.5 mg/dl), indicating possible removal of parathyroid glands. Because hypocalcemia places the client at risk for seizures or laryngospasms as well as arrhythmias, the primary healthcare provider needs to be notified immediately so that corrective therapy can be initiated. 1. Incorrect: While this client's PCO2 of 50 is elevated (normal is 35-45 mm Hg), this is neither unexpected or unusual for an individual with COPD. This client will frequently experience elevated levels of PCO2; therefore, the nurse should just continue monitoring for any changes in respiratory status. 2. Incorrect: This diabetic client has a fasting blood sugar of 145, which is elevated above normal levels of 70-110. However, it is not uncommon for diabetics to occasionally have elevated glucose levels, even early in the morning. The nurse can address this issue by referring to the sliding scale for insulin administration. This does not need to be reported immediately to the primary healthcare provider. 4. Incorrect: It is expected that clients diagnosed with sepsis will have extremely elevated white blood cell counts. Despite the fact that this lab result is outside of normal values (4,500 - 10,000 mm3), this level is not concerning enough to immediately contact the primary healthcare provider.

The nurse in the pediatric intensive care unit (PICU) is caring for a preschool child three days after open heart surgery. What assessment finding should the nurse report immediately to the primary healthcare provider? 1. Increased episodes of fussy crying. 2. A hacking, non-productive cough. 3. Oral temperature of 100.9 F (38.3 C). 4. Chest tube draining 30 mL per shift.

3. CORRECT: An oral temperature of 100.9 F (38.3 C) is considered too elevated for 3 days post-op. An oral body temperature greater than 100.5 F (38.1 C) indicates the potential for infection. Although no other vital signs are given in the scenario, a temperature this elevated would need to be reported by the nurse immediately to the primary healthcare provider. 1. INCORRECT: Increasing episodes of crying could indicate many things in a preschool child, including pain, fear, loneliness or even elevated body temperature. While this change in the client's status will need to be investigated further, the nurse would not need to report this behavior at this time. 2. INCORRECT: A hacking, non-productive cough, even several days after open heart surgery, could be attributed to the effects of intubation, anesthesia, or even certain cardiac medications. Clients are always encouraged to cough and deep breathe in order to prevent pulmonary complications. If the cough becomes productive or breathing becomes labored, the nurse would need to report this to the primary healthcare provider. This is not an urgent concern for the nurse. 4. INCORRECT: Chest tube drainage is common following open-heart surgery, even three days later. It is impossible to evaluate whether 30 mL in one shift is a change since there are no parameters to compare the previous shifts output. The nurse would not need to report this drainage at this time.

The nurse is reviewing client assignments at the beginning of the shift. Which task could be safely assigned to an unlicensed assistive person (UAP)? Stay with a client during the self-administration of insulin Clean and apply a dressing to a small pressure ulcer on the leg Empty a client's colostomy bag Monitor a client's response to passive range of motion exercises

Empty a client's colostomy bag If the UAP has demonstrated competency in the task, s/he may empty a client's colostomy bag. This is an uncomplicated, routine task with an expected outcome. The other tasks involve one or more parts of the nursing process and cannot be assigned to an UAP.

The nurse is caring for a client with acute renal failure. The morning assessment findings indicate the client has become confused and irritable. Which finding is most likely responsible for the change in behavior? 1. Hyperkalemia 2. Hypernatremia 3. Elevated blood urea nitrogen (BUN) 4. Limited fluid intake

3. Correct: A client with acute renal failure will have an increased (BUN). Significant elevation in BUN may result in nausea, vomiting, lethargy, fatigue, impaired thought processes, and headache. 1. Incorrect: Hyperkalemia can result from acute renal failure. Symptoms of hyperkalemia do not include confusion and irritability. Hyperkalemia may cause muscle weakness, muscle twitching, and flaccid paralysis. 2. Incorrect: Clients with renal failure retain fluid and are at risk for dilutional hyponatremia. Increased or decreased sodium levels can cause confusion, but this client is not at risk for hypernatremia. 4. Incorrect: Clients in acute renal failure should have limited fluid intake. This will not lead to confusion.

A client is admitted with new onset hyperthyroidism. Which medication is of concern to the nurse while reviewing the client's routine medications? 1. Ranitidine 2. Furosemide 3. Amiodarone 4. Propranolol

3. Correct: Amiodarone, a class III anti-arrhythmic drug, has multiple effects on myocardial depolarization and repolarization that make it an extremely effective antiarrhythmic drug. However, amiodarone is associated with a number of side effects, including thyroid dysfunction (both hypo- and hyperthyroidism), which is due to amiodarone's high iodine content and its direct toxic effect on the thyroid. 1. Incorrect: Ranitidine has not been found to contribute to the development of hyperthyroidism or hypothyroidism. 2. Incorrect: Furosemide has not been found to affect the thyroid. 4. Incorrect: Beta blockers are given to hyperthyroid clients to decrease myocardial contractility BP, and HR. It also decreases anxiety. This will help the hyperthyroid client.

The school nurse has been observing a 13 year-old student during the past few months as the student has steadily lost weight. Which assessment finding would be the best indication of the beginning of an eating disorder? 1. Clothing size has decreased by 2 sizes. 2. Student eats most meals with peers. 3. Client reports a fear of gaining weight. 4. Diet consists mostly of fruit or raw vegetables.

3. Correct: An adolescent reporting a fear of gaining weight may indicate the beginning of an eating disorder. This is the best indicator of an eating disorder. 1. Incorrect: A decrease in clothing size does not indicate a problem. It may be an indicator of an eating disorder but in itself does not mean there is an eating disorder. 2. Incorrect: A client with an eating disorder may eat alone, or not at all. Eating with peers shows the feeling of acceptance which is not usually present with an eating disorder. 4. Incorrect: Eating snacks of fruit and vegetables is a healthy behavior. This alone does not contribute to an eating disorder. Also, it says the diet is "mostly" fruit and vegetables.

A client at 34 weeks gestation with pregnancy induced hypertension (PIH) reports "heartburn." Which action by the nurse has priority? 1. Administer an antacid per standing orders. 2. Check client's blood pressure. 3. Call the primary healthcare provider immediately. 4. Assure client this is a normal discomfort of pregnancy.

3. Correct: Epigastric discomfort is commonly described as "heartburn" by pregnant clients, but epigastric discomfort is a symptom of impending rupture of the liver capsule and seizures associated with worsening PIH and eclampsia. As a new nurse we need to assume the worst. Call the primary healthcare provider. 1. Incorrect: Not a concern as much as impending seizure symptoms. Administering an antacid will not fix the problem if PIH is worsing. This is delaying care. 2. Incorrect: Not a concern as much as impending seizure symptoms. Checking the client's blood pressure is not the priority in this situation. It will not fix the problem. 4. Incorrect: Not in this situation. Heartburn is a normal discomfort or right upper quadrant pain in a client with PIH may indicate impending rupture of the liver capsule which is a life threatening complication.

The nurse is evaluating the outcomes of nursing interventions for the client on the long-term care unit. The nurse has determined that the goal was partially met. What should the first nursing action be at this point to maintain quality of care? 1. Identify a new goal for the client since this one has not been achieved. 2. Consider new nursing interventions for achievement of the goal if the condition still warrants it. 3. Determine that the nursing interventions were performed as planned. 4. Allow more time for achievement of the goal.

3. Correct: First, the nurse will want to determine that the interventions were performed. If they were not carried out, the goal could not be achieved. In addition, the nurse should determine if the nursing interventions were carried out appropriately and completely. Evaluation of the effectiveness of the nursing interventions would follow. 1. Incorrect: New goals may need to be identified; however, in this case it is not yet known if the interventions were carried out appropriately. Until it is determined that the current nursing interventions were implemented and performed appropriately, there is no way to accurately explore if new goals are needed. The original goals may be the most appropriate for the client. 2. Incorrect: New interventions may be appropriate; however, there is another option that is better. The original nursing interventions should have been identified based on the client's needs. Until the nurse determines if these were carried out appropriately, it would be premature to establish new nursing interventions. 4. Incorrect: Additional time for goal attainment may be appropriate; however, other actions should be performed first. Before extending time for achieving the goal, the nurse should determine if the nursing interventions have been carried out appropriately. If these have been performed, extending the time for goal attainment may delay making changes that are needed.

The emergency department nurse is assuming care of a client with full thickness burns to both legs. Which primary healthcare provider prescription should be implemented first? 1. Administer IV morphine 2. Insert oropharyngeal airway 3. Start two large bore IVs 4. Apply silver sulfadiazine to burn area

3. Correct: Full thickness burns of both legs would result in a severe fluid volume deficit. A priority treatment for burns include fluid replacement; therefore, insertion of 2 large bore IVs is a priority. 1. Incorrect: Pain is important but not priority over fluid volume status. Remember, pain never killed anybody. 2. Incorrect: This client does not have airway involvement. These burns are on the legs; there is no indication in the stem that the airway is involved. 4. Incorrect: Application of silver sulfadiazine does not take priority over fluid replacement.

A middle-aged client has a strong positive family history of type 2 diabetes mellitus. What should the nurse teach the client regarding the best method to prevent or delay the development of this disease? 1. Test serum glucose values monthly. 2. Avoid starches and sugars in the diet. 3. Obtain a normal body weight and exercise regularly. 4. Maintain a normal serum lipid panel.

3. Correct: Genetics and body weight are the most important factors in the development of type 2 diabetes mellitus. The client cannot alter his genetics. Therefore, a normal body weight is imperative. Regular exercise reduces insulin resistance and permits increased glucose uptake by cells. This serves to lower insulin levels and reduce hepatic production of glucose. 1. Incorrect: Monthly glucose monitoring is not sufficient. It will tell you when the client becomes a diabetic but will not prevent it from happening. 2. Incorrect: Starch and sugar intake should be decreased, not avoided. 4. Maintaining a normal serum lipid panel may not be achievable in some clients, but it is always the goal. Medication may be needed.

What is the first intervention the emergency department (ED) nurse should implement when caring for a lethargic toddler with a diagnosis of near-drowning? 1. Torso warming 2. Start intravenous infusion 3. Administer oxygen 4. Prepare for nasogastric intubation

3. Correct: Hypoxia is the primary problem because it results in brain cell damage 1. Incorrect: While warming protocols will likely be needed, hypoxia is the first priority 2. Incorrect: Fluid resuscitation will most likely be needed, hypoxia is the first priority 4. Incorrect: Nasogastric intubation may be needed but hypoxia is the first priority 1. Look at each option as True or False. 2. Option 1 is false. Because of submersion, the patient is usually hypothermic. Prescribed rewarming procedures (e.g., extracorporeal warming, warmed peritoneal dialysis, inhalation of warm aerosolized oxygen, torso warming). The client has blunted sensorium but is not unconscious so delivery of supplemental oxygen is appropriate. 3. Option 2 is false. Fluid resuscitation will most likely be needed to help correct acidosis but this intervention is secondary to oxygen administration. 4. Option 3 is true. Near-drowning victims typically suffer hypoxia and mixed acidosis. The priority is to restore oxygenation and prevent further hypoxia. 5. Option 4 is false. Near-drowning victims are at heightened risk for aspiration. Nasogastric intubation is used to decompress the stomach and to prevent the client from aspirating gastric content.

A client has been admitted to the emergency department after repeated food binging and purging by vomiting and laxative abuse. The client reports leg pains and weakness. ECG reveals a depressed ST segment and flattened T wave. Based on this data, what does the nurse anticipate that this client will need to receive first? 1. Oral fluids 2. Kayexalate enemas 3. Intravenous potassium (KCL) 4. An antidiarrheal medication

3. Correct: Look at the clues in the stem: vomiting, laxative abuse, symptoms of hypokalemia including weakness, muscle cramps, and arrhythmias. Due to repeated laxative abuse and vomiting, the client has lost potassium. Normal potassium is 3.5-5.0 mEq/L. IV potassium is required for a severely low potassium. 1. Incorrect: Oral fluids are needed, but with symptoms this severe, IV resuscitation is needed with potassium. The client is exhibiting symptoms of severe hypokalemia. The potassium is prescribed to correct this imbalance. 2. Incorrect: Kayexalate is given for high potassium. This client's potassium is low. The therapeutic effect of kayexalate is to reduce the serum potassium level. 4. Incorrect: We are worried about low potassium here. This won't solve the problem. An antidiarrheal medication may be prescribed, but the client is exhibiting symptoms of hypokalemia. The client should be administered the IV potassium first to correct the low potassium level.

A client is returned to the surgical unit following gastric/esophageal repair of a hiatal hernia, with an IV, NG tube to suction, and an abdominal incision. To prevent disruption of the esophageal suture line, what is most important for the nurse to do? 1. Assess the wounds for drainage. 2. Give ice chips sparingly. 3. Maintain the patency of the NG tube. 4. Monitor for the return of peristalsis.

3. Correct: Maintain the patency of the NG tube. On ANY post-op client, the nurse is responsible for preventing disruption of the suture line. (Disrupture of any suture line, since disruption could be life-threatening.) The nurse is responsible for keeping the NGT patent to prevent accumulations of gastric secretions and blood in the stomach. Accumulation of fluid in the stomach can cause pressure on the suture line and places the client at risk for disruption of the suture line and hemorrhage. The nurse knows NEVER to allow pressure or stretching on suture lines. 1. Incorrect: Assessing the wound for drainage is important, but when there is something more life-threatening, that is the priority answer. Disrupting the sutures is more life-threatening. 2. Incorrect: This person is ABSOLUTELY NPO. Giving ice chips is contraindicated as it could disrupt the suture lines. 4. Incorrect: It is important to monitor for return of peristalsis, but this is not life-threatening.

After the unexpected death of a Jewish teenager, the coroner tells the family that an autopsy has been requested. The teen's mother starts crying hysterically and refuses to allow the autopsy. After calming the mother, what should the nurse do next? 1. Explain that the coroner does not need the family's permission to perform the autopsy. 2. Ask the primary healthcare provider for a sedative for the mother. 3. Notify the coroner that the family is Jewish. 4. Call the rabbi of the family's synagogue to discuss the nature of the autopsy.

3. Correct: Mutilation of the body is forbidden. Autopsy is allowed only when mandated by civil authorities, such as when murder is suspected. If an autopsy is performed, all body parts must be returned for burial. 1. Incorrect: Permission is not needed when foul play is suspected. The keyword is "unexpected". The law can require an autopsy be performed when death is the result of foul play, homicide, suicide or accidental causes such as motor vehicle crashes, falls, the ingestion of drugs or deaths within 24 hours of hospital admission. 2. Incorrect: The nurse has calmed the mother. The sedative is not needed and does not solve this problem. Remember to stay away from medications as long as possible. 4. Incorrect: A rabbi is usually requested at the time of death, but this will not solve the autopsy problem. The rabbi may pray in a minyan, a group of 10 adults over the age of 13.

A client arrives at the emergency department (ED) in obvious emotional distress, reporting perioral numbness and tingling of the fingers and toes. The nurse notes a respiratory rate is 56/min. What should be the initial intervention performed by the nurse? 1. Send the client for a CT of the head. 2. Place on 100% O2 per non-rebreathing face mask. 3. Have the client breathe into a paper bag. 4. Administer diazepam 2 mg IV push.

3. Correct: Recognize the respiratory rate is too fast. This client is hyperventilating and blowing off too much CO2 which has resulted in symptoms of respiratory alkalosis, perioral numbness, and tingling of the fingers and toes. The nurse should try to help calm the client and encourage the client to slow the rate of breathing. This will help hold onto CO2. By breathing into a paper bag, the client will re-breathe CO2 therefore increasing the CO2 level. 1. Incorrect: The client is not demonstrating signs of a stroke. A CT is not warranted based on the information provided. 2. Incorrect: Administration of O2 is not warranted at this time. The client is blowing off too much CO2 and needs to re-breathe CO2 using a paper bag. Increasing O2 will not fix the problem of emotional distress. 4. Incorrect: Diazepam has sedative effects. Although hysterical clients may have to be sedated to decrease the respiratory rate, the less invasive means of using the paper bag should be attempted first.

The nurse is caring for a client who is taking an antipsychotic medication for the treatment of schizophrenia. The nurse is told in report that the client has akathisia, as a side effect of their antipsychotic medication. What symptom should the nurse expect this client to have? 1. Upward gaze of the eyes. 2. Involuntary movement of the tongue. 3. Reports of restlessness. 4. Lack of movement or slowed movement.

3. Correct: Reports of restlessness, inability to sit still, and nervous energy indicate akathisia. These symptoms respond poorly to treatment. If possible, the dose of the medication may be reduced. 1. Incorrect: Upward gaze of the eyes indicates dystonia, a possible adverse reaction to the antipsychotic medications. 2. Incorrect: Tardive dyskinesia has the symptoms of involuntary movement of the tongue, chewing movements of the mouth, and lip smacking. These symptoms may be irreversible. 4. Incorrect: Slowed movement refers to the side effect of bradykinesia. Lack of movement is referred to as akinesia.

The nurse is caring for a client in an outpatient clinic. The client's spouse died 8 months ago. Which statement by the client suggests that the client is achieving resolution of grief? 1. "I am starting a new life, so I have removed all of the pictures from the wall that remind me of my spouse." 2. "I'm so lonely and I'm not sure life is worth living now." 3. "Although it hasn't been easy, I accept the loss of my soul mate." 4. "If only we had spent more time together before the illness got so severe."

3. Correct: This client has begun to achieve resolution of grief by walking through the tasks of mourning: to accept the reality of the loss, to experience the pain of grief, to adjust to an environment in which the deceased is missing, and to withdraw emotional energy and perhaps invest in another relationship. 1. Incorrect: This client is still in the grieving process. Behavioral manifestations of grief include crying, withdrawal, avoiding reminders of the deceased, seeking or carrying reminders of the deceased, over activity, and a variety of changes in relationships with other people. 2. Incorrect: This client is still in the grieving process. The manifestations of grief can vary widely. This client has not accepted the reality of the loss, invested in relationships with other nor allowed themselves to go through the process of grief. 4. Incorrect: This client is still in the grieving process.

A nurse is caring for a client injured in a motor vehicle accident while driving intoxicated. After hearing that someone was critically injured because of the accident, the client mumbles, "But I only had just a few drinks". What is the most therapeutic statement the nurse could make to the client? 1. "If you only had a few drinks, how did you wreck?" 2. "What do you mean by 'just a few drinks'?" 3. "Tell me what you remember about the accident." 4. "You were driving when the accident happened."

3. Correct: While providing care to this client, it is important for the nurse to remain professional and non-judgmental. Because no life-threatening injuries are indicated, the most therapeutic approach would be to allow the client to verbalize feelings at this time. Additionally, having the client recall any specifics about the incident may provide the nurse with additional data for a neuro assessment. 1. Incorrect: Though it may be challenging to remain non-judgmental, this response demands an explanation from the client and can seem threatening. The client may have no memory of the accident; furthermore, although the client was intoxicated, there may be unknown circumstances that contributed to this accident. 2. Incorrect: This response might be helpful in situations where the nurse needs to determine the amount of alcohol a client ingests on a daily basis. However, in this circumstance, the amount of alcohol is not the issue for the nurse. The legal authorities may pursue this line of questioning. 4. Incorrect: This is a closed-ended statement that does not provide the client an opportunity to verbalize feelings. The nurse is making a statement that may, or may not, be factual. This would not be therapeutic to the client.

What should the nurse emphasize when teaching clients how to decrease the risk of chronic obstructive pulmonary disease? 1. Avoid exposure to individuals with respiratory infections. 2. Increase intake of Vitamin C. 3. Eliminate exposure to second hand smoke. 4. Avoid prolonged exposure to occupational dusts and chemicals. 5. Get a yearly influenza and pneumococcal vaccination

3., & 4. Correct: The most important environmental risk factor for COPD is cigarette smoking. Second hand smoking also contributes to COPD. Risk factors for COPD include prolonged and intense exposure to occupational dust and chemicals as well as indoor and outdoor air pollution. 1. Incorrect: Exposure to individuals with respiratory infections does not increase risk of chronic obstructive pulmonary disease. Respiratory infections may cause an acute exacerbation in a client with existing COPD. 2. Incorrect: Increasing intake of vitamin C does not decrease risk of obstructive pulmonary disease. 5. Incorrect: Clients should get the influenza vaccine annually in autumn. The pneumococcal vaccine should be administered every 5 years, rather than yearly.

What information should the nurse include in teaching an oncology client the purpose of taking epoetin? 1. Emergency treatment of anemia. 2. Improves quality of life. 3. Used for the prevention of pure red cell aplasia (PRCA). 4. Decreases the need for transfusion.

4. Correct: Epoetin is prescribed to treat a lower than normal number of red blood cells (anemia) caused by chronic kidney disease in clients on dialysis, in HIV clients receiving zidovudine and in cancer clients receiving chemotherapy that develop anemia. Epoetin stimulates the bone marrow to produce more RBCs. 1. Incorrect: Epoetin does not work raoidly enough to be used for the emergency treatment of anemia (RBC transfusion). 2. Incorrect: Epoetin has not been proven to improve quality of life, fatigue, or sense of well-being in clients with cancer. 3. Incorrect: Pure red cell aplasia (PRCA) is a type of anemia that starts after treatment with epoetin or other erythropoetin medications.

The hospice nurse has been assigned a new client who is being cared for at home by family members. Based upon the client's physical assessment, the nurse is aware that the client's death is imminent. What is the nurse's most important role in the care of the family at this time? 1. Providing care for the client, allowing the family to rest. 2. Providing education regarding the symptoms the client will likely experience. 3. Allowing the family to express their feelings and actively listening. 4. Communicating the client's impending death to the family while they are together.

4. Correct: Communicating news of the client's impending death to the family while they are together. The nurse's most important role in the care of the family is compassionate communication. The family needs to be informed about the situation so that they are prepared for the client's death and can provide support to one another. 1. Incorrect: Providing respite time when death is imminent is not a priority. Family should be allowed to spend time with the client. They will, more than likely, want to be with the client in the last hours. 2. Incorrect: When death is imminent, education of what to expect is appropriate, but does not take priority over compassionate communication. Compassionate communication is most important at this time. 3. Incorrect: Silence and listening sends a message of acceptance and comfort. Although important, allowing for expression of feelings is not more important than preparing the client for the imminent death.

What is the priority nursing action for a pregnant client in labor who is having an epidural catheter inserted for pain management? 1. Perform a thorough skin prep of the insertion site. 2. Obtain the client's consent for the procedure. 3. Assure the client that residual effects of the procedure won't be felt. 4. Monitor maternal blood pressure.

4. Correct: Epidural anesthesia may result in distal vasodilation and a precipitous drop in maternal blood pressure, which will adversely affect placental blood flow. Evidence-based practice guidelines from the Association of Women's Health, Obstetric and Neonatal Nurses (AWHONN) suggests assessing the maternal blood pressure and fetal heart rate every 5 minutes during the first 15 minutes after initiation of epidural medication. 1. Incorrect: Preparing the insertion site is the responsibility of the primary healthcare provider. 2. Incorrect: Obtaining consent is the responsibility of the primary healthcare provider. This is not the priority nursing action. 3. Incorrect: Residual effects of epidural anesthesia include infection and headache. So this is an incorrect statement.

A client taking phenelzine is admitted to the hospital. Which healthcare provider prescription should the nurse question? 1. Take blood pressure lying, sitting, and standing once per shift. 2. Order a complete blood count and liver profile studies. 3. Eliminate foods containing tyramine from diet. 4. Discontinue phenelzine. Begin fluoxetine 20 mg by mouth at bedtime.

4. Correct: Phenelzine is a non-selective monamine oxidase inhibitor (MAOI). Fluoxetine is a selective serotonin reuptake inhibitor (SSRI). Both of these medications are antidepressants, but are in different drug classifications. They should not be taken in combination due to the risk of serotonin syndrome. Additionally 2 weeks should be allowed for phenelzine to be cleared from the body before starting a different classification of antidepressant. There should be at least two weeks between giving phenelzine and fluoxetine. 1., 2., 3. Incorrect: These are correct prescriptions/interventions for this client. Clients taking antidepressants can have a sudden drop in blood pressure upon rising. Instruct them to rise slowly. The liver can be affected by these drugs so routine liver screening is acceptable. Foods containing tyramine can lead to hypertensive crisis when ingested while taking a monoamine oxidase inhibitor (MAOI).

A client with a head injury manifests symptoms of increasing intracranial pressure. The primary healthcare provider prescribes mannitol IV. How would the nurse plan to evaluate the effectiveness of this medication? 1. Monitor urine output hourly 2. Take vital signs every 15 minutes 3. Measure head circumference every 8 hours 4. Assess the level of consciousness (LOC) every hour

4. Correct: The stem of the question states the client manifests symptoms of increased ICP. Assessing the LOC is the only answer that assesses for increased ICP. Even if you do not know how mannitol works, the only answer that assesses the client for increased ICP is to assess the LOC. 1. Incorrect: Mannitol causes an osmotic diuresing effect. Urinary output is expected to increase, but this does not assess changes in ICP. Assessing LOC is the only answer that assesses for changes in ICP. 2. Incorrect: Taking frequent vital signs is an answer that sends the message to the NCLEX people that you don't know what to do, so you'll get a set of vital signs. 3. Incorrect: Measuring head circumference is useful if your client is an infant, but frequently assessing the LOC is a more sensitive indicator.

Which statement by a client would indicate to the nurse that education about gastroesophageal reflux disease (GERD) has been successful? 1. It would be better for me to eat 3 small meals a day. 2. I need to avoid eating foods high in purine. 3. When going to sleep, I should lie on my side. 4. My last daily meal should not be within 2 hours of bedtime.

4. Correct: To avoid reflux the client should not eat within 2 hours of bedtime. Late night meals may increase discomfort and should be avoided. 1. Incorrect: The client should eat at least 6 smaller meals per day to help decrease reflux. Small, frequent meals help prevent over distention of the stomach. 2. Incorrect: The client with GERD should avoid high fat foods and increase high protein foods in an effort to lose weight. Eliminate foods high in purine with disorders such as gout. 3. Incorrect: The client should sleep with the HOB elevated on six inch blocks and several pillows under the upper body. Gravity fosters esophageal emptying.

What foods should the nurse inform the client to avoid for three days prior to a guaiac test? 1. Chicken 2. Carrots 3. Apple 4. Raw broccoli 5. Steak 6. Turnip greens

4., 5., & 6. Correct: Foods that affect this test include raw broccoli, red meats such as steak, turnip greens, cantaloupe, radish, and horseradish. All of these could cause a false positive reading for the guaiac test. 1. Incorrect: Red meats such as steak should be avoided, but chicken is okay. 2. Incorrect: Carrots are not prohibited and will not affect the results of the test. 3. Incorrect: The client can eat apples with no effect on the test results.

The nurse is caring for a Puerto Rican client. The client has several injuries from a car accident and is experiencing pain. Which behavior is likely to be noted? 1. Loud crying with pain. 2. Enduring the pain in order to bring honor. 3. Quiet and stoic responses to pain. 4. Refusing pain medication because it is God's will.

1. Correct: Puerto Rican clients tend to cope with pain by loud and outspoken reports of pain. This is consistent with Puerto Rican culture and their response to pain. 2. Incorrect: Quietly enduring pain is consistent with the Japanese culture. This is consistent with the Asian culture and brings honor. 3. Incorrect: Stoic responses are consistent with Asian culture. The client is likely to be quiet about the pain thinking that complaints of pain will bring dishonor to the family. 4. Incorrect: Filipino clients tend to view pain as God's will. They may refuse medication to relieve the pain.

Which action by a nurse would indicate that this nurse is following standard precautions? 1. Clean gloves while performing a heel stick on an infant. 2. Sterile gloves to empty a indwelling urinary catheter bag. 3. Shoe covers when entering the room of a client with influenza. 4. Clean gloves while inserting a urinary catheter.

1. Correct: Standard precautions when drawing blood is to wear gloves so blood will not get on the nurse's hands. Clean gloves are appropriate. 2. Incorrect: Clean gloves for the nurse's protection are needed. Sterile gloves are not needed as part of standard precautions. 3. Incorrect: Shoe covers are not a standard precaution and not needed when entering the room of a client with influenza. 4. Incorrect: Sterile gloves are needed to insert a urinary catheter. Standard precautions are meant to reduce the risk of transmission of bloodborne and other pathogens from both recognized and unrecognized sources. They are the basic level of infection control precautions which are to be used, as a minimum, in the care of all clients. ASSESS THE RISK of exposure to body substances or contaminated surfaces BEFORE any health-care activity. Make this a routine! Select PPE based on the assessment of risk: clean non-sterile gloves, clean, non-sterile fluid-resistant gown, mask and eye protection or a face shield.

The nurse is working with a group of elderly clients to promote better nutrition. Prior to developing the health promotion plan, the nurse assesses individual members of the group. Which assessment findings are expected as the nurse works with this group? 1. Some clients may have dental issues, making chewing difficult. 2. There may be a decreased appetite in clients. 3. Caloric and nutritional needs may vary somewhat depending on activity levels. 4. Access to fresh foods is adequate. 5. The desire and interest in cooking is increased.

1., 2. & 3. Correct: Many elderly people have dental issues that affect chewing and intake of nutritionally dense foods. Appetite may decrease due to changes in taste, medications, depression or isolation. Many elderly people are active; therefore, it is important to assess each one individually in regard to activity levels. 4. Incorrect: Many elderly clients may not have access to fresh foods due to infrequent grocery shopping, limited budgets, and a desire to not waste good food. 5. Incorrect: Many elderly do not have a desire to cook for one or two. Pain and physical impairment may also decrease desire or interest in cooking.

Which interventions should be included in the nutritional teaching plan to accomplish the goal of a diet lower in fat? 1. Use 2% milk instead of whole milk. 2. Eat air-popped popcorn instead of potato chips. 3. Eat more red meat instead of fish. 4. Incorporate plant sources of protein. 5. Use olive oil instead of vegetable oil when frying.

1., 2. & 4. Correct: Two percent milk can reduce the amount of fat consumed daily, not only in milk that the client drinks, but also in foods that contain milk as an ingredient. Air-popped corn contains no fat unless butter is added after popping. The client still is able to have a crunchy snack without the fat. Plant proteins such as kidney, black, or lima beans are good sources of protein without the fat from a meat source. 3. Incorrect: Red meats are high in fat. Chicken, fish, and seafood are better meat choices. 5. Incorrect: Olive oil is low in saturated fat but still a source of fat. While olive oil may be a healthier choice, all fats have essentially the same number of calories per serving. The goal is to reduce the amount of fat in the diet.

A client is seen in an outpatient clinic for anxiety after losing the family home in a hurricane. What nursing interventions would be appropriate for the nurse to make? 1. Teach the client how to use progressive muscle relaxation. 2. Assist the client in correcting any distortion being experienced. 3. Suggest that the client might recover faster by moving away from the coastal area. 4. Refer the client to the family primary healthcare provider for a complete physical examination. 5. Allow the client time to talk about the loss.

1., 2. & 5. Correct: The correct answers are appropriate interactions for this client and will help the client with anxiety reduction. Allowing the client time to talk shows them that someone cares. Muscle relaxation helps relax the client. Helping the client see the situation accurately helps decrease a distorted view of the experience. When a person is feeling anxious or stressed, these strategies can help him or her cope: Practice yoga, listen to music, meditate, get a massage, or learn relaxation techniques. Stepping back from the problem helps clear your head. Eat well-balanced meals. Do not skip any meals. Do keep healthful, energy-boosting snacks on hand. Limit alcohol and caffeine, which can aggravate anxiety and trigger panic attacks. Get enough sleep. When stressed, your body needs additional sleep and rest. Exercise daily to help you feel good and maintain your health. Check out the fitness tips below. Take deep breaths. Inhale and exhale slowly. Talk to someone. Tell friends and family you're feeling overwhelmed, and let them know how they can help you. Talk to a physician or therapist for professional help. 3. Incorrect: This does not address the anxiety related to this crisis. Additionally, the nurse is trying to solve the client's problem through relocation, which may not be the client's desire. 4. Incorrect: This refers the client to someone else and does not address the problem. This may make the client feel as though their anxiety is not important. Option 1 is true. Progressive muscle relaxation is a form of relaxation. Option 2 is true. Clarify distortions by getting persons to look at the situation realistically, focus on what can be changed versus what cannot. Option 3 is false. This may not be a viable solution and the client may not want to leave home. Do not place your judgment or suggestions on the client. The goal is to empower the person by allowing them to make informed choices. Option 4 is false. The problem at this point is not physical. It is psychological based on crisis. Option 5 is true. Encourage the expression of feelings.

An occupational health nurse works in a factory where loud equipment is used in production of the factory's product. What should the nurse emphasize to factory management persons to reduce the risk of hearing impairment? 1. Supply workers with earplugs when exposed to noise. 2. Replace high noise machinery with low noise machinery. 3. Limit amount of time a person spends at a noise source. 4. Operate noisy machines during shifts when fewer people are exposed. 5. Supply personal noise monitoring to identify employees at risk from hazardous level of noise. 6. Have all employees make an appointment for a hearing test.

1., 2., 3., 4., & 5. Correct: All of these are primary prevention methods of controlling hearing loss in employees exposed to hazardous noise levels. Earplugs, or earmuffs, are considered an acceptable but less desirable option to control exposures to noise and are generally used during the time necessary to implement engineering and administrative controls to protect worker's hearing. Engineering controls include choosing low noise tools and machinery, maintaining and lubricating machinery and equipment, placing a barrier between the noise source and employed, and enclosing or isolating the noise source. Administrative controls are changes in the workplace that reduce or eliminate the worker's exposure to noise. This includes operating noisy machines during shifts when fewer people are exposed, limiting the amount of time a person spends at a noise source, and using monitoring equipment to monitor hazardous noise level. 6. Incorrect: Some employees may need a hearing test but this will not reduce the risk of hearing impairment. This is secondary prevention, which focuses on screening and early diagnosis of disease.

A nurse suspects that a client admitted to the emergency department is in diabetic ketoacidosis. What data would lead the nurse to this conclusion? 1. Dry mucous membranes 2. Fruity-smelling breath 3. Biot's respirations 4. Glycosuria 5. Client report of abdominal pain

1., 2., 4., & 5. Correct: The client with diabetic ketoacidosis will have signs of dehydration due to polyuria and includes dry mucous membranes. Fruity breath odor is from the acetone that occurs with breakdown of fats and formation of ketones, which are acids.. With DKA, the client would be spilling glucose into the urine. Vomiting and abdominal pain are frequently the presenting symptoms of DKA. 3. Incorrect: The client will have Kussmaul respirations. Biot's respiration is a respiratory pattern characterized by periods of rapid respirations, then apnea periods. These are not the type of respirations that occur with diabetic ketoacidosis (metabolic acidosis).

A client diagnosed with cancer has been losing weight. What should the nurse teach the client regarding methods for improving nutritional needs to maintain weight? 1. Add butter to foods. 2. Cup of cubed beef broth. 3. Add powdered creamer to milkshake. 4. Use biscuits to make sandwiches. 5. Fish sauted in olive oil. 6. Put honey on top of hot cereal.

1., 3., 4., & 6. Correct: Butter added to foods adds calories. This client needs more calories and more protein. Spread peanut butter or other nut butters, which contain protein and healthy fats, on toast, bread, apple or banana slices, crackers or celery. Use croissants or biscuits to make sandwiches which provides more calories. Add powered creamer or dry milk powder to hot cocoa, milkshakes, hot cereal, gravy, sauces, meatloaf, cream soups, or puddings to add more calories. Top hot cereal with brown sugar, honey, dried fruit, cream or nut butter. 2. Incorrect: One cube of beef broth is 11 calories. Supplementing the diet with beef broth would not add significant calories. 5. Incorrect: Although cooked in olive oil, fish is low in calories.

A home care nurse is making an initial visit to an elderly client recently discharged following hip surgery. When evaluating the home environment, what environmental hazard is most concerning to the nurse? 1. Lamp plugged into extension cord. 2. Throw rugs on kitchen tile floor. 3. Gas fireplace in the living room. 4. Non-working wall socket in hall.

2. CORRECT: It is quite common to find throw rugs, or "scatter rugs" in homes to protect carpets and absorb moisture or dirt. However, throw rugs are a common hazard, posing the potential for tripping or catching on wheels. In this situation, a tile floor is generally smooth, making it even more likely to slip on the rugs. 1. INCORRECT: An extension cord by itself is not a problem. If the cord was placed under carpeting, it could be a fire hazard. Also, if the cord were too short, and hung suspended in air, the client could trip. However, just using an extension cord does not present a problem. 3. INCORRECT: Many homes use a gas fireplace which is actually safer than wood burning. The presence of a gas fireplace does not create an immediate danger. 4. INCORRECT: A non-working wall socket does not present any danger to the client. If the socket were sparking, there would be cause for concern. But there are many reasons for a socket not to work, and without any information, the nurse cannot consider this a danger.

A client is admitted to a chemical dependency unit for addiction treatment. Which of the client's belongings should the nurse remove from the client's room? 1. Shampoo and conditioner 2. Mouthwash and hand sanitizer 3. Toothpaste and dental floss 4. Lotion and foot powder

2. Correct: Mouthwash and hand sanitizers have alcohol in them, which the client may drink. 1. Incorrect: No alcohol content in these items, so these would be safe and contribute to the well-being of the client while in treatment. 3. Incorrect: No alcohol content in these items, so these would be safe and contribute to the well-being of the client while in treatment. 4. Incorrect: No alcohol content in these items. These would be safe and provide comfort and maintain skin integrity for the client.

A nurse is to administer a time release capsule to a client who has difficulty swallowing. Which intervention would be the best course of action for the nurse to take? 1. Open the capsule and sprinkle it on applesauce. 2. Melt the capsule in juice or water. 3. Call the primary healthcare provider to change the order. 4. Break the capsule in half using a pill splitter.

3. Correct: If the client has difficulty swallowing a capsule or tablet, ask the primary healthcare provider to substitute a liquid medication if possible. 1. Incorrect: Sprinkling the medication over applesauce or pudding may be the only option the nurse has if there is no other form, but since this medication is time-released, the best answer and priority would be to get a liquid form, if available, for the drug. 2. Incorrect: Never melt a time release capsule or tablet as this would release the medication all at once. 4. Incorrect: Breaking or splitting would also release the medication in boluses and could cause harm to the client.

While completing the admission history on an elderly client diagnosed with advancing Alzheimer's disease, the client's spouse begins to sob and states, "After all these years, we won't be together anymore." What would be the best response by the nurse? 1. "You can come to visit anytime you want to." 2. "Would you like to see the room and facilities?" 3. "Let's find a quiet place to sit and talk awhile." 4. "You did the best you could in this situation."

3. Correct. The nurse recognizes that the client's spouse is emotionally distraught at this moment, and is most in need of the nurse's focus at this time. Major life events have affected this family unit, including the client's terminal diagnosis and separation to a new living environment. This spouse is understandably overwhelmed by the changes occurring and, while the nurse will need to complete the admission paperwork, family needs must be met. Focusing on the spouse's emotional needs and allowing time to verbalize feelings could positively affect the client's adaptation to the situation. 1. Incorrect. Although this statement may be factual, it is a closed-ended statement, which belittles the spouse's expression of distress by presuming that unlimited visitation will rectify the situation. Though the spouse is verbalizing sadness because of physical separation, the grief may be a reflection of deeper concerns, considering the client's diagnosis. 2. Incorrect. This response focuses on the facility surroundings, rather than the spouse's distress and expression of sorrow. Changing the topic both ignores and belittles the client's grief. The nurse must address the needs of family as well as those of the client. 4. Incorrect. Though this response may seem encouraging, a closed-ended statement does not allow the openly distressed spouse an opportunity to verbalize further feelings. Family dynamics can significantly impact the client's well-being and potential to adapt to new surroundings.

When preparing an intramuscular injection for a neonate, which needle should a nurse select? 1. 18 G, 7/8 inch 2. 21 G, 1 inch 3. 25 G, 5/8 inch 4. 25 G, 1.5 inch

3. Correct: The most appropriate needle to select for use in administering IM injection to a neonate would be a 25 gauge, 5/8 inches long. Intramuscular injections are given in the vastus lateralis muscle of the thigh. 1. Incorrect: This needle is too large a diameter for a newborn infant. An 18 gauge needle is appropriate for the intravenous (IV) medication or blood administration in adults. 2. Incorrect: This needle would be too large for a newborn infant. A 21 gauge needle is typically used to draw up medication from vials or ampules (filtered needle required). 4. Incorrect: This needle would be far too long for a newborn infant and also for most children. A 1.5 inch needle is often needed to administer intramuscular injections to obese adults.

The nurse is caring for a client immediately following a bilateral salpingo-oophorectomy. Which position would be best for this client? 1. Fowler's 2. Modified Sims 3. Side-lying 4. Supine

3. Correct: We want to position for comfort with the knees flexed and on the side for airway. 1. Incorrect: Avoided to prevent pooling and edema in pelvis 2. Incorrect: Partial lying on stomach is going to be painful 4. Incorrect: Stretching out straight puts pressure on the abdomen and should be avoided

A nurse is evaluating an unlicensed assistive personnel (UAP) for proper body mechanics while lifting a heavy object off of the floor. What action by the UAP would indicate a need for further instruction by the nurse? 1. Testing the weight to determine if additional assistance is needed. 2. Keeping the feet shoulder width apart. 3. Bending from the waist to pick up the object. 4. Holding the object close to the body upon rising.

3. Correct: You should not bend at the waist. This will injure your back. Lower your knees, and stay close to the object to use thigh muscles. 1. Incorrect: This is correct. If it is too heavy do not attempt to lift alone. 2. Incorrect: This is correct and will help maintain balance as you lower yourself to the floor. 4. Incorrect: This is correct. Holding close to the center of gravity will help prevent injury to your back and arms.

The client needs assistance to apply anti-embolism stockings each day in the long-term care facility. Today, as the nurse enters the room to apply the stockings, she finds that the client has been walking about the unit for 30 minutes. What should the nurse do first to lessen the risk of swelling of the lower extremities? 1. Ask the client to lie down and place the stockings on the legs. 2. Ask the client to sit on the bedside and place the stockings on the legs. 3. Tell the client that the nurse will return later to assist with the application. 4. Elevate the extremities in bed for 30 minutes before application.

4. Correct: The client should have extremities elevated to encourage venous return and reduce the risk of swelling before the stockings are applied. 1. Incorrect: To place the stockings on immediately will cause further venous stasis and swelling. 2. Incorrect: The extremities should be elevated for a period of time before application. 3. Incorrect: This instruction alone does not give the client adequate information about the need to keep the lower extremities elevated before applying the stockings.

Which manifestations, if noted in a pregnant client, would the nurse need to report to the primary healthcare provider? 1. Calf muscle irritability 2. Facial edema 3. Pressure on the bladder 4. Blurry vision 5. Hemoglobin of 11 mg/dL 6. Epigastric pain

1., 2., 4., & 6. Correct: These are danger signs/symptoms of pregnancy and need further investigation by the primary HCP. These signs could indicate preeclampsia, fluid and electrolyte disturbances, and other high risk complications during pregnancy. 3. Incorrect: As the baby gets bigger, it pushes on the bladder, causing pressure, so this is an expected symptom in pregnancy. 5. Incorrect: This is normal for the pregnant client and within the normal range for the female client.

A charge nurse is teaching a new nurse on the labor and delivery floor the proper positioning of a client following an epidural. The charge nurse knows the teaching was successful when the new nurse places the client in which position? 1. Lithotomy 2. Left-lateral 3. Semi-Fowler's 4. Right-lateral

2. CORRECT: The left-lateral position is most appropriate following epidural anesthesia. In this position, the placenta is well perfused and the client is less likely to experience side effects from anesthesia such as hypotension. 1. INCORRECT: The lithotomy position is supine with legs separated, knees flexed and elevated with feet supported in stirrups. Such a position is appropriate for gynecologic exams, but would place too much pressure on the vena cava at this time. 3. INCORRECT: In this position, the client is supine with the head of the bed elevated between 30 and 90 degrees. This is a good position for those with breathing difficulties; however, following an epidural, elevating the head may drop the blood pressure while leaving the client supine and putting pressure on the vena cava. 4. INCORRECT: The right-lateral position is on the right side, with left leg flexed toward the head, and is useful to avoid hypotension. But this is not the best position following an epidural for improving uteroplacental perfusion.

A client is admitted with irritable bowel syndrome (IBS) and shingles. The nurse is discussing the client assignments with the charge nurse. Which staff member should not be assigned to this client? 1. The nurse with a history of roseola. 2. The unlincesed assitive personnel (UAP) with no history of roseola. 3. The UAP with a history of chicken pox. 4. The LPN/VN with no history of chicken pox.

4. Correct: A nurse who has not had chicken pox could contract it and should not be assigned a client with shingles. Those who have not developed antibodies to the varicella zoster virus are susceptible to chicken pox. Chicken pox and shingles are both from the varicella virus. 1. Incorrect: Roseola is a rose colored rash and would not have any effect on the assignment. It is a generally mild infection that usually affects children by age 2, and rarely adults. It is caused by 2 strains of herpes virus, rather than the varicella virus. 2. Incorrect: Roseola is a rose colored rash and would not have any effect on the assignment. It is a generally mild infection that usually affects children by age 2, and occasionally adults. It is caused by 2 strains of herpes virus, rather than the varicella virus. There is no relationship between roseola and shingles. 3. Incorrect: Shingles is caused by a reactivation of the varicella-zoster virus (which causes chicken pox). Those who have not developed antibodies to the varcella-zoster virus are susceptible to chicken pox. Therefore, the UAP who had chicken pox could be assigned this client.

A nurse has arrived late to work twice in the last week. What should be the nurse manager's first action? 1. Confront the nurse with the consequences of tardiness. 2. Ask the nurse to consent to a drug screening test. 3. Document the tardiness in the nurse's record. 4. Ask the nurse the reason for being tardy.

4. Correct: The first action should be discussing the tardiness with the nurse. There may be a situation that is impacting the nurse's ability to be on time. This will allow the nurse to explain the tardiness. The nurse manager may have to consider alternate scheduling for the nurse. 1. Incorrect: The first action should not be confrontational. The nurse manager needs to find out the reason for tardiness in a non-confrontational manner. This will demonstrate that the nurse manager is showing concern for the nurse. 2. Incorrect: The nurse is not exhibiting any impairment behaviors. Although drug use may be characterized by behaviors such as tardiness to work, this should not automatically be assumed. The nurse should have the opportunity to first provide an explanation for the tardiness. 3. Incorrect: Documentation should be done after the meeting with the nurse. The documentation would include the nurse's explanation for the tardiness.

Which assessment finding by the nurse is most indicative of fluid volume overload? 1. Client has pitting edema in lower extremities. 2. Client's blood pressure is 120/80. 3. Client's CVP measurement is 6 mmHg. 4. Weight gain of 1.5 pounds (0.68 kg) in one day.

1. Correct. A client in fluid volume overload may experience pitting edema in lower extremities, a bounding pulse, increased blood pressure, and shortness of breath. 2. Incorrect. This blood pressure reading is considered normal and is not a characteristic of fluid volume overload. 3. Incorrect. This CVP is within the normal range therefore not indicative of a fluid volume excess. In a fluid volume excess, the CVP would be elevated. 4. Incorrect. A weight gain in excess of 2 pounds (0.9 kg) is of concern for fluid volume excess. Any weight gain overnight is reason for concern; however, the stem asked which finding was most indicative.

Which male client condition in the after-hours clinic should the nurse assess first? 1. Scrotal pain and edema. 2. Erection lasting for 2 hours. 3. Inability to void with a history of benign prostatic hyperplasia (BPH). 4. Purulent drainage from the penis.

1. Correct: This client is likely to have testicular torsion, which requires immediate intervention. Infarction of the testes can occur if not treated promptly. 2. Incorrect: This is not the most life threatening problem. Priapism, a persistent, often painful erection that lasts for more than 4 hours should be treated. 3. Incorrect: With BPH the prostate gland increases in size, leading to disruption of the outflow of urine. This can cause inability to void and needs to be assessed but is not the first priority. 4. Incorrect: This client does not have the most serious condition and would not take priority.

The nurse, caring for a client who has chronic renal failure, suspects that the client is experiencing anxiety. Which statements by the client would validate the nurse's suspicion? 1. "I do not think I can continue working." 2. "My husband has taken over the house cleaning and cooking." 3. "I fear I am dying." 4. "I have an "uneasy" feeling most of the time." 5. "Most of the time I feel very 'down and blue'."

1., 2., 3. & 4. Correct: The inability to maintain employment is of concern to most clients who have been used to working. With a chronic illness, the client is unlikely to be able to return to work. Anxiety related to role strain is common. The client may not be able to perform the duties that she once did, thus causing others to have to assume their roles. Death is a possible outcome if transplant does not occur. Fear may be a later diagnosis as the client's condition deteriorates. Clients with anxiety often report feeling uneasy or on edge. 5. Incorrect: These comments are more indicative of a depressed mood than anxiety. Depression may also occur in the client who has chronic renal failure.

After determining that a client diagnosed with a stroke has adequate swallowing ability, the nurse develops interventions to safely provide oral feedings to the client. What interventions should the nurse include in this plan of care? 1. Provide mouth care prior to feeding. 2. Flex head forward for eating. 3. Have dietary puree foods. 4. Use crushed ice as a stimulant for swallowing. 5. Offer thickened liquids to drink. 6. Position client in semi fowler's position after feeding.

1., 2., 4., & 5. Correct: These interventions will stimulate sensory awareness, salivation, swallowing, and decrease the risk of aspiration. 3. Incorrect: Pureed foods are not usually the best choice because they are often bland and too smooth making it difficult to swallow. 6. Incorrect: The client should remain in a high Fowler's position, preferably in a chair with the head flexed forward, for feeding and for 30 minutes afterward. After careful assessment of swallowing, chewing, gag reflex, and pocketing, oral feedings can be initiated. The nurse wants to implement ways that will stimulate sensory awareness and salivation and can facilitate swallowing, while reducing the risk of aspiration. Mouth care before feeding helps stimulate awareness and salivation. Saliva helps you chew, taste, and swallow. So option 1 is true. Option 2 is true as well. Flexing the head forward bends the cervical spine at its highest point C1 through C2. Head flexion moves the chin against the neck. This type of chin-tuck is used to reduce residual particles of food pocketing in the epiglottic valleculae. The epiglottic valleculae are two depressions situated between the base of the tongue and the epiglottis, one on each side of the median glosso-epiglottic ligament. Their normal function is to collect saliva prior to a swallow, and, for people without swallowing difficulties, they do not collect food particles during a bolus being swallowed. Option 3 is false. Pureed foods are not usually the best choice because they are often bland and too smooth. Foods should be easy to swallow and provide enough texture, temperature (warm or cold), and flavor to stimulate swallowing. Option 4 is true. Crushed ice can act as a stimulant. Have the client swallow and then swallow again. Option 5 is true. Thin liquids are often difficult to swallow and many promote coughing. Thin liquids can be thickened with commercially available thickening agents. Avoid milk products because they tend to increase the viscosity of mucous and increase salivation. Option 6 is false. The client should remain in a high Fowler's position, preferably a chair with the head flexed forward, for the feeding and for 30 minutes afterward. This will help to prevent aspiration.

The nurse is caring for a client in the emergency department who presents with hematemesis. What information is most important for the nurse to obtain during the initial assessment? 1. Vital signs 2. History of prior bleeding episodes 3. Medications the client is taking 4. Urinary output 5. Level of consciousness

1., 4., & 5. Correct: A set of vital signs and assessment for hypovolemic shock take priority for this client. S/S of shock include thready, rapid pulse, decreased LOC, shortness of breath, cold and clammy skin, and decreased urinary output. 2. Incorrect: History of prior bleeding episodes is important but does not address the immediate problem. 3. Incorrect: Medication history is important, but the nurse must first determine whether or not the client is in shock. What is hematemesis? It is the vomiting of blood. So what are your worried about? Hemorrhage/Shock! So what assessments will help you determine if the client is in or going into shock? Option 1? True. BP will drop, HR will increase. Option 2. False. Will this assess the current problem? No. Option 3. False. Will knowing medications fix the problem? No. Option 4? True. Are the kidneys being perfused? If not, the UOP will be low. Option 5? True. Level of consciousness will decrease as perfusion to the brain decreases.

A nurse is caring for a multipara client in active labor who received morphine 4 mg IVP for pain. Thirty minutes later, the client had a precipitous delivery. What should the nurse prepare to administer to the newborn? 1. Oxygen 2. Naloxone 3. Glucose 4. Vitamin K

2. Correct: The primary side effect of opioids is respiratory depression, which is more likely to affect the newborn. Naloxone reverses opioid-induced respiratory depression. This newborn will need naloxone to reverse the effects of the narcotic that was given to mom 30 minutes earlier. 1. Incorrect: Prepare to administer the naloxone to reverse the respiratory depression first. Then if oxygen is needed, provide it. 3. Incorrect: Glucose is not indicated in this situation. 4. Incorrect: Vitamin K is given to the newborn after delivery to prevent vitamin K dependent bleeding but it is not the priority here.

A client admitted to the hospital following a fall has a history of Alzheimer's disease with apraxia. The nurse knows the client will need priority assistance with what activity? 1. Ambulating to the bathroom. 2. Understanding instructions. 3. Using utensils at mealtime. 4. Identifying objects in room.

3. Correct: Apraxia is a motor disorder of voluntary movements in which the individual can no longer execute purposeful activity, even though there is adequate mobility, strength, and coordination. This loss of ability to carry out previously learned movements could occur secondary to brain injury or a disease process such as Alzheimer's disease. The client has the ability to pick up utensils but is unable to use them correctly, which may affect the client's nutritional status. 1. Incorrect: Apraxia does not affect the ability to ambulate to the bathroom, although the client may not be able to follow cleanliness procedures once in the bathroom. However, there is another activity is of more concern. 2. Incorrect: The ability to understand is not affected by apraxia, which is a disorder in which the client loses the ability to perform purposeful movement. The client is still able to comprehend instructions at this point. There is another situation in which the client will need assistance. 4. Incorrect: The client is still able to identify objects in the environment; however, the diagnosis of apraxia indicates the client cannot use previously known objects correctly. Because of this situation, there is another area in which assisting the client is of more importance.

Question: An elderly client receives instructions regarding the use of warfarin sodium. Which statement indicates to the nurse that the client understands the possible food interactions which may occur with this medication? 1. "I'm going to miss having my evening glass of wine now." 2. "I told my daughter to buy spinach for me. I'll have to eat more servings now." 3. "I will have to watch my intake of salads, something that I really love." 4. "I am going to begin eating more fish and pork and leave beef alone now."

3. Correct: Clients taking warfarin sodium must watch their intake of vitamin K, which is present in leafy green vegetables and tomatoes. 1. Incorrect: Wine does not affect the use of warfarin sodium. 2. Incorrect: These clients need to monitor their intake of spinach, which is a source of vitamin K. 4. Incorrect: These clients need to monitor their intake of fish, which is a source of vitamin K.

A client's membranes spontaneously rupture at 10 cm dilation and +2 station. The nurse notes that the fluid is colored green. What client preparation is the priority nursing action? 1. Emergency cesarean delivery 2. Immediate high forceps delivery 3. Equipment for immediate suctioning of the newborn 4. Administration of IV oxytocin

3. Correct: Green stained fluid indicates fetal passage of meconium. The fetus must be suctioned by the healthcare provider when the head is still on the perineum and before the baby takes its first breath. This will remove any particulate matter from the meconium that may cause aspiration. 1. Incorrect: Delivery will probably occur soon and vaginal delivery is preferable to cesarean. This is an unrealistic and inappropriate action for this client. 2. Incorrect: High forceps are never indicated and would not provide safe delivery for the baby. The concern is the meconium stained fluid and potential aspiration for the baby. 4. Incorrect: The meconium passage is an indicator of fetal stress, and increased uterine contractions may stress the fetus further. This would not be safe for the baby or the mother at this stage of labor.

The nurse enters a client's room and finds the client masturbating. Which action by the nurse would be most appropriate for the nurse to take? 1. Ask the client to stop 2. Remain in the room until client has finished. 3. Document the activity in the client's chart. 4. Quietly leave the room

4. Correct: Leaving the client's room, allows the client to have privacy. The client has the right to express self sexually in private. 1. Incorrect: The client has a right to express sexuality through masturbation, which is a normal way of finding sexual release. 2. Incorrect: Ignoring the behavior and continuing presence in the room will embarrass the client. 3. Incorrect: The nurse can chart the client's sexual activity in the chart. However, when the nurse enters the client's room and finds the client masturbating, the nurse first needs to leave the client's room quietly. "Most" is used in this stem. What would be least embarrassing for the client, while allowing the client to express sexuality? Option 4, leaving the room quietly.

The nurse is caring for a client in the 8th week of pregnancy. The client is spotting, has a rigid abdomen and is on bedrest. What is the most important assessment at this time? 1. Protein in the urine 2. Fetal heart tones 3. Cervical dilation 4. Hemoglobin and hematocrit levels

4. Correct: The client may be bleeding, and that is an emergency! Common causes of hemorrhage during the first half of pregnancy include abortion and ectopic pregnancy. Ectopic pregnancy is a significant cause of maternal death from hemorrhage and the classic signs of ectopic pregnancy include positive pregnancy test, abdominal pain and vaginal "spotting". Remember that in the ruptured ectopic pregnancy, bleeding may be concealed and severe pain could be the only symptom. 1. Incorrect: Protein in the urine indicates preeclampsia, which is a condition in which hypertension develops during the last half of pregnancy. 2. Incorrect: We can't hear them yet because the client is just 8 weeks pregnant. It may be possible to detect heart beat with a Doppler transducer at 10 weeks, but this client is only in the eighth week of pregnancy. 3. Incorrect: A vaginal exam may stimulate heavier bleeding and will not provide information about concealed bleeding. A transvaginal ultrasound will be performed to determine whether a fetus is present and if so, whether it is alive.

When assessing for the development of an infection following the application of a plaster cast to the leg, the nurse should teach the client to observe for the presence of which sign of infection? 1. Hot spots 2. Cold toes 3. Warm toes 4. Paresthesia

1. Correct: Hot spots is the best answer. Redness and increased warmth are indicators of localized infection. If the cast covers the extremity, redness cannot be visualized, but the client can feel more warmth (a "hot spot") in an area becoming infected. 2. Incorrect: "Cold toes" is a neurovascular check, not an indication of infection. 3. Incorrect: "Warm toes" is a neurovascular check, not an indication of infection. 4. Incorrect: Paresthesia is a neurovascular check, not an indication of infection.

A 6 year old admitted from the emergency department (ED) with a fractured tibia is scheduled for surgery in the morning. All of the private rooms are full so the child must be admitted to a semi-private room. What room assignment is appropriate for the nurse to make for this client? 1. Rooming with an 8 year old in sickle cell crisis. 2. Rooming with a 2 year old admitted with bacteremia. 3. Rooming with a 3 year old with pneumonia. 4. Rooming with a 4 year old with gastroenteritis.

1. Correct: Sickle cell disease and a child in a sickle cell crisis is not considered contagious. This is the only option that does not have an infectious process, so this would be the best room assignment for the child with the fracture. In addition, the children are close in age with the same development tasks, so activities for the children may be similar. 2. Incorrect: Bacteremia is an infectious process in which there is viable bacteria in the blood stream. The source of the infection is not noted. The child with a fracture who will be having surgery should not be placed in a room with a child who has a known infection. 3. Incorrect: The child with pneumonia has an infectious process that may be viral or bacterial. The child with the fracture should not be assigned to this room due to the risk of air-borne exposure to the infectious agent. 4. Incorrect: Gastroenteritis is a diarrheal illness with inflammation in the stomach and small intestine. This is contagious, so if all possible, this child should be kept in a private room, so other children would be less likely to contract the gastroenteritis. It may be viral, bacterial, or parasitic in origin. The child with the fracture should not be assigned to the room with the child with gastroenteritis.

The adult child of a client diagnosed with bipolar disorder asks the nurse if they will one day be diagnosed with the same disorder. What is the nurse's best response? 1. "There is a familial tendency for developing this disorder; however, it doesn't mean you will definitely develop this disorder." 2. "You should not worry about developing this disorder. You are young and healthy." 3. "If you were going to develop this disorder, you would have it by now." 4. "You have not been exposed to anything that would contribute to the development of this disorder, so you will not develop this disorder."

1. Correct: Studies to determine if an illness is familial compare the percentages of family members with the illness to those in the general public or within a control group. Bipolar disorder is an example of psychiatric illness's with familial tendencies. Other psychiatric illness include schizophrenia, major depression, anorexia nervosa, panic disorder, somatization disorder, antisocial personality disorder, and alcoholism. 2. Incorrect: This adult child has a predisposing risk of developing this disorder. Do not give the adult child false assurance. Awareness of the family tendency will promote early detection and treatment. 3. Incorrect: There is no particular time frame for developing this disorder. You are brushing off the adult child here and not providing accurate information. 4. Incorrect: Exposure to outside elements is not indicative of development of this disorder. Do not dismiss the adult childs concern ever but surely not with inaccurate information.

A client with an automated internal cardiac defibrillator (AICD) was successfully defibrillated. The telemetry technician shouts out that the client was in ventricular fibrillation (VF). What should the nurse do first? 1. Go to the client to assess for signs and symptoms of decreased cardiac output. 2. Call the primary healthcare provider to report that the client had an episode of VF so medication adjustments can be made. 3. Notify the "on call" person in the cath lab to re-charge the ICD in the event that the client has a recurrence. 4. Document the incident on the code report form and follow up regularly.

1. Correct: The client comes first. Check to see how they are doing by completing a head to toe cardiac output assessment. make sure to include LOC, vital signs, skin and urinary output assessment. 2. Incorrect: Do not call before you assess the client who may be unconscious if the arrhythmia has decreased their cardiac output. 3. Incorrect: This is not needed because there is a battery that keeps it charged, so that they don't have to re-charge after each shock. 4. Incorrect: Documentation is not appropriate until the client has been assessed first.

While reviewing the prescriptions written by a primary healthcare provider, the nurse notes that ibuprofen 30 mg by mouth every 6 hours is prescribed for a child weighing 6 kg. The drug information book states that the appropriate dosage range is 20-30 mg/kg/24 hours. What action should the nurse take? 1. Administer the ibuprofen at 30 mg by mouth every 6 hours. 2. Contact the nursing supervisor regarding the prescription. 3. Ask the pharmacist to calculate the appropriate dose. 4. Notify the primary healthcare provider.

1. Correct: The maximum dose in 24 hours would be 30 x 6 = 180 mg. 30 mg every 6 hours is a safe dose. 2. Incorrect: The nursing supervisor does not need to be notified. This is a safe dose. 3. Incorrect: The nurse can calculate the appropriate dose based on the information provided. The primary healthcare provider does not need to be notified. The prescription is within the safe range. 4. Incorrect: The primary healthcare provider does not need to be notified since the prescription is written within the safe range.

Question 32: What information should be included in the health promotion plan for parents regarding the promotion of adequate bowel elimination in their toddler? 1. Include adequate fiber in the diet through whole grains and fruits. 2. Increase intake of water daily. 3. Provide toileting opportunities that are free from distractions. 4. Encourage the toddler to go to the bathroom at least three times daily. 5. Take away attention from the toddler unable to potty.

1., 2. & 3. Correct: Fiber is important for achieving adequate bowel elimination. Fruits and whole grains may help. Water intake is important, coupled with adequate fiber. Distractions at toileting times may result in poor elimination results. 4. Incorrect: The toddler should be taken to the bathroom after meals and at bedtime to encourage adequate elimination. Routine is very important. Peristalsis increases after meals. 5. Incorrect. Embarrassment or punitive measures will not yield positive results. Rather, the toddler should be praised for using the potty. Options 1 and 2 are true. Increasing fiber and fluid in the diet will promote adequate bowel elimination by increasing peristalsis. Option 3 is true. Toddlers who are distracted may not wait for complete bowel elimination before getting up from the toilet. Option 4 is false. The best time to have a bowel movement is after a meal when peristalsis increases and prior to bedtime. Option 5 is false. A client's emotional status must be maintained. You do not want to punish the child if unable to have a bowel movement.

The nurse is planning to teach a group of senior citizens about modifiable risk factors for developing a stroke. Which factors should the nurse include? 1. Diabetes mellitus 2. Hypertension 3. Hispanic ethnicity 4. Atrial fibrillation 5. Sleep apnea 6. Smoking

1., 2., 4., 5., & 6. Correct: These are all modifiable risk factors that can be managed through lifestyle changes or medical treatment. 3. Incorrect: Hispanics, African Americans, Native Americans, and Asian Americans have a higher incidence of strokes than whites. You cannot change your race or ethnicity so this is a non-modifiable risk factor for stroke. Modifiable risk factors are those that can potentially be altered through lifestyle changes and medical treatment, thus reducing the risk of stroke. Diabetes and hypertension damages vessels by causing plague buildup which can lead to decreased perfusion to the brain. Atrial fibrillation can cause a clot to be thrown from the heart and travel to the brain. Sleep apnea causes hypoxia that can affect brain perfusion. Smoking causes vasoconstriction which can decrease oxygenation to the brain. All of these problems however, can be altered through lifestyle changes or through medical treatment.

The nurse is providing preprocedural education to the client preparing for a barium enema. What statement made by the client indicates a need for further education? "I will need to drink plenty of fluids and eat foods high in fiber after the procedure." "I will use the prescribed laxative before the procedure." "I will not eat or drink anything after midnight before the procedure." "A barium enema is used to examine the upper and lower GI tracts."

"A barium enema is used to examine the upper and lower GI tracts." A barium enema involves filling the large intestine (lower GI tract) with diluted barium liquid while x-ray images are taken. After the procedure, a small amount of barium will be immediately expelled and the remainder will be excreted in the stool. Because barium liquid may cause constipation, clients should eat foods high in fiber and drink plenty of fluids to help expel the barium from the body.

A client diagnosed with angina has been instructed about the use of sublingual nitroglycerin. Which statement made by the client is incorrect and indicates a need for further teaching? "I'll call the health care provider if pain continues after three tablets five minutes apart." "I will rest briefly right after taking one tablet." "I understand that the medication should be kept in the dark bottle." "I can swallow two or three tablets at once if I have severe pain."

"I can swallow two or three tablets at once if I have severe pain." Clients must understand that just one sublingual tablet should be taken at a time and placed under the tongue. After rest and a five-minute interval, a second and then eventually a third tablet may be necessary.

The nurse is caring for a client who is experiencing frightening hallucinations that are markedly increased at night. The client's partner asks to stay a few hours beyond the visiting time, in the client's private room. What would be the best response by the nurse? "Yes, staying with the client and orienting the client to the surroundings may decrease any anxiety." "No, your presence may cause the client to become more anxious." "No, it would be best if you brought the client some reading material that the client could read at night." "Yes, would you like to spend the night when the client's behavior indicates that the client is or will be frightened?"

"Yes, staying with the client and orienting the client to the surroundings may decrease any anxiety." Encouragement of a family member or a close friend to stay with the client in a quiet surrounding cannot only help increase orientation, but can also minimize confusion and anxiety. The visitor could also report to the nurse any unusual findings of the client. This would be the most supportive approach for this client.

Prior to signing a consent form for surgery, the client states, "I am not sure that I understand the possible risks for this surgery and what the alternative treatments are." What should the nurse do first? 1. Clarify any questions that the client may have and then share the client's concern with the primary healthcare provider. 2. Reinforce that it is not unusual for clients to have questions about surgery. 3. Inform the primary healthcare provider that the client has concerns about the surgery. 4. Use open ended questions to explore client's concerns.

*3 Correct: The nurse should call the primary healthcare provider. Further discussion with the client is warranted from the primary healthcare provider that has scheduled and most likely will be performing the surgery. This also provides the client the opportunity to ask questions appropriately.* 1. Incorrect: The client has the right to make informed decisions. The client should not sign until all questions are answered by the primary healthcare provider. 2. Incorrect: Recognizes client concerns, but does not take care of problem. The nurse has a responsibility to be an advocate for the client and practice within the law. 4. Incorrect: The informed consent comes from discussion between the primary healthcare provider and the client. The nurse can do this, but it doesn't fix the problem.

The nurse checks the results of a urinalysis performed on a client with dehydration. Which results should the nurse expect to find? 1. Increased white blood cells 2. Presence of protein 3. Presence of ketones 4. Increased specific gravity

*4. Correct: Specific gravity is an indicator of hydration status and urine osmolality. In a dehydrated client, specific gravity is increased, indicating highly concentrated urine.* 1. Incorrect: White blood cells should not be found in the urine unless an infection is present. Dehydration does not cause white blood cells in the urine. 2. Incorrect: Protein should not be found. Presence of protein indicates renal disease. In order to have proteinuria there must be damage to the glomeruli 3. Incorrect: Ketones should not be present. They are found in clients with poorly controlled diabetes or hyperglycemia, because ketones are a by-product of fat breakdown. Fats are broken down and used for energy when glucose cannot be transported into the cells because of lack of insulin.

How should the nurse respond to a pregnant client who asks, "How will I know when it is time to go to the hospital?" 1. "Go to the hospital immediately if your membranes rupture." 2. "You should leave for the hospital as soon as you lose your mucus plug." 3. "Go to the hospital when you have a burst of energy followed by a backache." 4. "You need to go to the hospital when contractions are 2 minutes apart."

. Correct: Yes! This is the appropriate teaching. A gush or trickle of fluid from the vagina should be evaluated regardless of whether contractions are occurring. Infection and compression of the umbilical cord are possible complications. 2. Incorrect: No. The mucus plug is lost prior to the beginning of active labor, so too early to go to the hospital. Some women lose their mucus plug weeks before labor begins, others lose it right as labor starts. 3. Incorrect: Nesting? That's too early and not specific enough. This is not labor. 4. Incorrect: The client should go when contractions are 5 minutes apart, for 1 hour if it is her first pregnancy. Labor may be faster for the woman who has given birth before than for the nullipara. Multiparas are instructed to go to the hospital when contractions are regular, 10 minutes apart, for 1 hour.

The community health nurse is presenting information about birth control measures to a group of young females. The nurse explains that an intrauterine device (IUD) is most appropriate for what individuals? (Select all That Apply) 1. A mother of a toddler who wants another child in three years. 2. The client with a recent exacerbation of sickle cell anemia. 3. A client with stage II breast cancer who has finished chemotherapy. 4. An adolescent who has recently become sexually active. 5. The client with a double mastectomy seven years ago.

1 and 5. CORRECT: An IUD is a surgically placed method of birth control in which a small, t-shaped piece of plastic, or even copper, is inserted into the uterus to decrease the chance of pregnancy. The client must be very healthy, emotionally amenable to a foreign body to prevent pregnancy, and aware that an IUD is not 100% fail-proof. The mother of a toddler who would like to have another child in a couple years is an excellent candidate for the use of an IUD. Also, a client who had a double mastectomy over seven years ago is a good candidate, since treatment that long ago means the client would no longer be receiving any type of immunosuppressant therapy. 2. INCORRECT: A client who has had a recent exacerbation of sickle cell anemia is at high risk for several complications, including infection and clots. This is a foreign body in an already compromised client, leading to many potential complications. 3. INCORRECT: The client being actively treated for cancer is also immunosuppressed and would not be a good choice for an IUD. The risk of infection is much too high. 4. INCORRECT: An adolescent who has recently become sexually active presents a challenge. Remember that an adolescent does not have regular menstrual cycles yet, and can experience intermittent bleeding. Many primary healthcare providers argue that the use of an IUD may be safer since the client would not have to remember a pill, a ring, or a patch. But an even greater concern is the fact that an IUD is NOT 100% effective, still presenting the risk of an unwanted pregnancy. Also, an IUD does not protect against sexually transmitted disease (STDs), which is often a concern in those who have become sexually active.

A school nurse is planning a lesson on inhalant abuse for a high school health class. Which information does the nurse need to include? 1. Substances used for inhaling include lighter fluid, spray paint, and airplane glue. 2. Inhalants are absorbed through the lungs and cause central nervous system depression rapidly. 3. Although inhaling can make a person very ill, death is highly unlikely. 4. Inhaling substances can cause abdominal pain, lethargy, and renal failure. 5. Inhalants cause the heart to beat slowly

1, 2, & 4 Correct: Products such as glues, nail polish remover, lighter fluid, spray paints, airplane glue, deodorant and hair sprays, whipped cream canisters, and cleaning fluids are widely available. Many young people inhale the vapors from these sources in search of quick intoxication without being aware that using inhalants, even once, can have serious health consequences. Inhaled chemicals are absorbed rapidly into the bloodstream through the lungs and are quickly distributed to the brain and other organs. Within seconds of inhalation, the user experiences intoxication along with other effects similar to those produced by alcohol. Alcohol-like effects may include slurred speech; the inability to coordinate movements; euphoria; and dizziness. Inhalants also are highly toxic to other organs. It causes abdominal pain, and vomiting. Chronic exposure can produce significant damage to the heart, lungs, liver, and kidneyIn addition, users may experience lightheadedness, hallucinations, and delusions. This information needs to be included in a teaching plan on inhalant abuse. All of these statements are correct and need to be included in a teaching plan on inhalant abuse. 3. Incorrect: With inhalant use, death can occur from respiratory depression or cardiac arrest. 5. Incorrect: Inhalants force the heart to beat rapidly and erratically, leading to cardiac arrest. Inhalants are volatile substances that produce chemical vapors that can be inhaled to induce a psychoactive, or mind-altering, effect. Common household products contain volatile solvents or aerosols. So what options did you chose? Option 1: This statement is correct. Products such as glues, nail polish remover, lighter fluid, spray paints, airplane glue, deodorant and hair sprays, whipped cream canisters, and cleaning fluids are widely available. Many young people inhale the vapors from these sources in search of quick intoxication without being aware that using inhalants, even once, can have serious health consequences. What do you think about Option 2? True. Inhaled chemicals are absorbed rapidly into the bloodstream through the lungs and are quickly distributed to the brain and other organs. Within seconds of inhalation, the user experiences intoxication along with other effects similar to those produced by alcohol. Alcohol-like effects may include slurred speech; the inability to coordinate movements; euphoria; and dizziness. In addition, users may experience lightheadedness, hallucinations, and delusions. Look at option 3. True or false? False. Death is HIGHLY likely!! The highly concentrated chemicals in solvents or aerosol sprays can induce irregular and rapid heart rhythms and lead to fatal heart failure within minutes of a session of prolonged sniffing. This syndrome, known as "sudden sniffing death," can result from a single session of inhalant use by an otherwise healthy young person. Option 4? True. Inhalants also are highly toxic to other organs. It causes abdominal pain, and vomiting. Chronic exposure can produce significant damage to the heart, lungs, liver, and kidneys. Option 5? This one is false. Inhalants force the heart to beat rapidly and erratically, leading to cardiac arrest.

An alert elderly client has been admitted to the hospital and placed on bedrest following a fall at home. During evening medication rounds, the nurse notes the client has become disoriented to time and place. The nurse is aware a new onset of confusion could be the result of what factors? 1. Admission to the hospital. 2. Amount of physical pain. 3. Current bed confinement. 4. Advanced age. 5. Response to analgesic.

1, 2, 3, & 5. Correct: The nurse is aware that multiple factors can contribute to acute confusion in clients. The sudden relocation to a new environment, along with pain from injury, could definitely contribute to an acute onset of confusion. The client's ordered bedrest and response to new pain medications are additional factors that could produce an acute change in mental status. 4. Incorrect: Age alone is not a factor for confusion. New onset of confusion may be successfully resolved once any contributing factors are addressed. Well, look at the many clues in this stem! The client is alert on admission and then that evening becomes disoriented. This is important information to answer this question. Option 1: Yes. Change of environment especially in the elderly can lead to confusion. The stem does not tell you the client's injuries from the fall but specifically says the client was elderly and alert on admission. Option 2: Good choice. Pain, especially when uncontrolled can cause disorientation and confusion. The client may be so focused on the pain that they become confused and disoriented. Option 3: Great choice! The client is now looking at the same four walls all day long. Bed rest will have effects on the brain causing confusion. Option 4: Oh no! Simply being elderly does not lead to disorientation and confusion. Do not be misled by the client being elderly. Remember in the stem, the clue is this client is an alert, elderly person. Option 5: Yes. Medications have many side effects. Analgesic pain medications can alter mental status leading to disorientation and confusion.

What clients could safely be delegated to the LPN/VN? 1. A client two days post appendectomy needing to ambulate. 2. A client with bronchitis receiving nebulizer treatments. 3. A newly diagnosed diabetic client awaiting discharge home. 4. A client newly admitted with exacerbation of myasthenia gravis. 5. A client admitted yesterday for observation following a fall. 6. A client with a nasogastric tube (NG) hooked to low suction.

1, 2, 5 & 6. Correct: Thes clients are appropriate and stable enough for the LPN/VN's scope of practice. While an LPN/VN cannot be assigned a fresh post-op, the first client had an appendectomy two days ago. The LPN/VN could even delegate ambulating this client to unlicensed assistive personnel (UAP). A client with bronchitis will need a respiratory assessment by the RN at some point, but the LPN/VN is definitely qualified to administer aerosol treatments. The third client was admitted for observation following a fall a day ago, indicating no injuries serious enough for a full admission. PNs can insert and monitor NG tubes. 3. Incorrect: This client is a newly diagnosed diabetic who will require extensive teaching about selfcare at home. Additionally, discharging a client always involves teaching, which cannot be initiated by an LPN/VN. This option does not indicate that any teaching had been presented, so the client is not an appropriate assignment for the LPN/VN. 4. Incorrect: Myasthenia Gravis is a progressive weakening of the neuromuscular system placing the greatest risk on the respiratory system. Although this client is on a medical-surgical floor, there is a need for close monitoring and frequent assessment of the respiratory system, requiring an RN.

A cardiac step down unit has requested float staff because of multiple impending admissions. The supervisor can only send one LPN/VN to the floor. Which clients would be appropriate assignments for the LPN/VN? 1. A client with COPD complaining of shortness of breath on exertion. 2. A post-cardiac catherization needing assistance with bedpan. 3. A client receiving heparin injections for deep vein thrombosis. 4. A client with atrial fibrillation currently on a diltiazem drip. 5. A client receiving a blood transfusion that requires monitoring. 6. A client post pacemaker insertion, awaiting discharge instructions.

1, 3 & 5. Correct: The LPN is being floated to a specialty floor and appropriate assignments would include clients who are stable. Client #1 has COPD, and, although complaining of shortness of breath, that is not unusual for clients with this diagnosis. Client #3 is receiving heparin sub-q for deep-vein thrombosis, and sub-q injections are within the LPN's scope of practice. Client #5 -It is considered within the scope of practice for an LPN/VN to monitor a transfusion of a blood product. 2. Incorrect: This client is post cardiac catherization and remains on bedrest; therefore, the affected leg must be kept straight to prevent femoral hemorrhaging. Because positioning on a bedpan requires rolling of the client, an RN should be assigned to assess the insertion site and monitor for the presence of bleeding. 4. Incorrect: Atrial fibrillation places the client at risk for blood clots. Diltiazem is a calcium channel blocker that has been ordered as a titrated drip to slow heart rate and restore a regular rhythm. Assessing this client and titrating the diltiazem requires the skills of an RN. 6. Incorrect: Discharging a client includes teaching and a review of medications to be taken at home. These areas require the expertise of an RN and would not be appropriate for an LPN/VN. Option 1. Remember the LPN can care for stable clients who are complicated and have a long term illness. This client has SOB on exertion which is a typical finding in the COPD client. Option 3. The LPN can give heparin injections to a client with DVTs. Option 5. Can the LPN monitor a client receiving blood? Yes. The RN starts the blood but the LPN can monitor the blood once started. So why can't the LPN care for clients in options 2, 4, and 6? These are all cardiac clients who are complicated and unstable or need teaching,

An oncology client with a Hickman catheter is being discharged to receive chemotherapy via cassette pump at home. The nurse is aware that discharge instruction should include what information? (Select All That Apply). 1. Always use two pairs of gloves when preparing chemo medications. 2. Discarded chemo cassettes and tubings can be placed in regular trash. 3. Used needles or syringes must be placed into plastic chemo receptacle. 4. Linens soiled with chemo drugs can be washed with regular laundry. 5. Waste is placed into chemo bags and picked up by medical supplier. 6. Regular home cleaning products are appropriate for spilled chemo.

1, 3 and 5. CORRECT: Administering chemotherapy medications at home would require the same diligence and precautions that are used in the hospital setting. In order to prevent contamination, the individual preparing the chemo should wear two pair of gloves and should not prepare the drugs in an area where food is prepared. Used needles or syringes must be discarded in a hard, yellow plastic receptacle marked "chemo". Any "soft" waste products, such as dressings or towels used to clean up spills, must be double-bagged and then placed into the designated "chemo bag". These wastes are then picked up by the medical supplier for disposal. 2. INCORRECT: No equipment used to prepare or administer chemotherapy medications can ever be placed in regular trash. Specially designated chemo disposal receptacles must be used for all types of chemo waste. 4. INCORRECT: Several days after receiving chemotherapy, the human body eliminates unused or excess product through body waste such as stool, urine, or even emesis. Linens that become soiled with such waste products must be washed separately from normal linens for the first washing. A second washing is necessary, although the linens may be thrown in with other clothing for the second washing. 6. INCORRECT: It is never appropriate to use regular home cleaning products when cleaning up spilled chemo drugs. The medical supplier who delivers the equipment will also deliver the specific "chemo spill kit" needed for the medication in use. In the home setting, it is advised to clean up the area of the spill at least three times.

A pregnant client who had been on a magnesium drip for severe pregnancy induced hypertension (PIH) has had an emergency cesarean section at 35 weeks. The nursery nurse should anticipate what findings in the newborn related to that magnesium therapy? (Select All That Apply) 1. Hypotension 2. Hypoglycemia 3. Hyperreflexia 4. Flaccid muscle tone 5. Respiratory depression

1, 4 and 5. CORRECT: When magnesium sulfate is administered to the mother for preeclampsia, the intent is to prevent seizures and decrease blood pressure by suppressing the central nervous system, thus preventing premature labor. The dose of this drug and the length of time administered will determine what side effects might be seen in the newborn, since magnesium crosses the placental barrier. The nurse will most likely note hypotension and some degree of respiratory depression in the infant. Additionally, the infant may have flaccid or weak muscles along with poor, or even absent, reflexes. Treatment of the newborn will be based on the degree of depression. 2. INCORRECT: The use of magnesium sulfate in the mother prior to delivery does not affect the blood glucose level of the fetus/newborn. Magnesium sulfate affects the central nervous system, not the pancreas, so blood sugar should be within normal limits. 3. INCORRECT: Magnesium is a central nervous system depressant that crosses the placental barrier. Side effects to the newborn would be similar to those noted in the mother, including depressed, or absent reflexes. The nurse would not find hyperreflexia.

A client with Hepatitis C has returned from surgery with a total laryngectomy. The nurse knows that what personal protective equipment is necessary when providing trach care? (Select All That Apply). 1. Face mask 2. Shoe covers 3. N-95 mask 4. Goggles 5. Gloves 6. Gown

1, 4, 5 and 6. CORRECT: The client has had a total laryngectomy which will initially produce large amounts of thick, bloody mucus. Hepatitis C is transmitted through blood and body fluids. During trach care, the nurse needs to be protected by specific personal protective equipment (PPE's). For this procedure, the nurse should utilize gown, gloves, goggles and face mask. 2. INCORRECT: Tracheostomy care is completed in close proximity to the client. Splattering of blood and body fluids on the floor is unlikely, so shoe covers are unnecessary. 3. INCORRECT: The N-95 face mask is a specially fitted mask used by nurses when providing care for clients with active tuberculosis. It is not necessary for a client with Hepatitis C.

Which observation of denture care by the unlicensed assistive personnel (UAP) would require the nurse to intervene? 1. Soaking the dentures in hot water 2. Donning gloves and using a gauze pad to grasp and remove dentures 3. Moistening the dentures prior to inserting them 4. Wrapping the dentures in tissue while the client sleeps 5. Placing a washcloth in the bathroom sink prior to cleaning.

1. & 4. Correct: Hot water may damage dentures so intervention is needed. Dentures should be stored in a denture cup. 2. Incorrect: Gloves should be worn to remove dentures and a gauze used to grasp the dentures. 3. Incorrect: Moistening the dentures will ease insertion. 5. Incorrect: The wash cloth is placed in the sink to prevent the dentures from breaking if they are dropped. Hot water can make denture material sticky. Cool water should be used. Standard precautions are to don gloves anytime coming into contact with body fluids. The gauze helps to grip the dentures. Moistening dry dentures help with insertion. Wrapping dentures in tissue may cause them to be accidently thrown away. A towel needs to be placed in the sink prior cleaning to prevent damage to the dentures if they are dropped.

A primary healthcare provider prescribes contact precautions for a newly admitted client. What equipment does the nurse need to place outside of the client's room for use when entering the room? 1. Gown 2. Gloves 3. Goggles 4. Surgical mask 5. N95 respirator

1. & 2. Correct: Healthcare personnel caring for clients on Contact Precautions wear a gown and gloves for all interactions that may involve contact with the client or potentially contaminated areas in the client's environment. 3. Incorrect: Goggles are not required with contact precautions.It is used when splashing is anticipated. 4. Incorrect: A surgical mask is not required with contact precautions. It would be used for droplet precautions. 5. Incorrect: A N95 respirator is not required with contact precautions. It is used for airborne precautions. Option 1 is true. Contact precautions require the nurse to don a gown when entering the room of a client. Remove the gown when leaving the room, dispose of it in a waste container, and perform hand hygiene. This makes option 1 correct. Option 2 is true. Contact precautions require the nurse to don gloves when entering the room of a client since the environment surrounding the client may be contaminated as well as the client. Option 3 is false.This is false. Goggles are not required for contact precautions. Option 4 is false. This is also false. A surgical mask is not required for contact precautions. Option 5 is false. Special air handling and ventilation are not required to prevent contact transmission because infected secretions are not suspended in the air.

The home health nurse is caring for a client who is identified as high risk for falls. What evaluation would indicate a therapeutic response to home fall prevention education? 1. Installs a grab bar in the tub. 2. Turns night lights on at bedtime. 3. Only uses assistive devices when leaving home. 4. Goes barefoot while in the home. 5. Uses throw rugs in walking areas to prevent slipping.

1. & 2. Correct: Placing a grab bar in a slippery tub can assist the client in getting into and out of the tub. Turning on night lights at night ensures that the client can navigate safely, thus reducing the risk of falls. 3. Incorrect: If the adult has an assistive device, it should be used inside and outside the home. The client should be encouraged to use assistive devices, such as canes or walkers, at all times. 4. Incorrect: The client should always wear properly fitting shoes that have nonskid protection. The client increases their risk for injury when properly fitting shoes are not worn. 5. Incorrect: Throw rugs actually may increase the risk of tripping.

Which nursing statements about a client reflect correct documentation in the hospital medical record? 1. 20% of breakfast consumed. 2. 4 inch by 2 inch wound noted on right arm. 3. Enema administered. 4. Appears upset at spouse. 5. Lying in bed.

1. & 2. Correct: The nurse should record findings or observations precisely and accurately. Percent of breakfast eaten is accurate documentation. An arm wound should include its exact size and location. 3. Incorrect: Documentation of enema administered should also include type of solution, amount and results. 4. Incorrect: Documenting observed behaviors or conversations is appropriate; however, drawing conclusions about feelings is not. A better notation would be to describe facial expression and any emotions exhibited,( i.e. crying, laughing, etc.). 5. Incorrect: This documentation does not give body position and does not provide pertinent information about the position of bed and side rails or light placement.

The nurse plans to teach a client how to manage the use of a behind the ear hearing aid. What teaching strategies should the nurse include? 1. Hairspray should not be used while wearing the hearing aid. 2. A whistling sound when the hearing aid is inserted indicates proper placement. 3. Submerse hearing aid in cool water daily to clean. 4. Illustrate where damage commonly occurs on a hearing aid. 5. Batteries last 6 months with daily wearing of 10-12 hours.

1. & 4. Correct: The residual from the hair spray causes the hearing aid to become oily and greasy. The client should routinely inspect the hearing aid for damage, especially where damage is more likely: ear mold, earphone, dials, cord, and connection plugs. 2. Incorrect: A whistling sound indicates incorrect ear mold insertion, improper fit of aid, and buildup of earwax or fluid. 3. Incorrect: Do not submerse hearing aid in water, as it will damage the device. 5. Incorrect: Batteries last 1 week with daily wearing of 10-12 hours.

The nurse is caring for a client in end-stage renal disease with an arteriovenous dialysis access. What assessment by the nurse indicates patency of the access? You answered this question Incorrectly 1. Presence of bruit 2. Warm and dry skin 3. Presence of a thrill 4. Positive Trousseau's 5. Capillary refill within 3 seconds

1. & 3. Correct: A functioning arteriovenous access device is critical for the client receiving hemodialysis. Assessment of the device to ensure patency includes auscultation for the presence of bruit and palpation for the presence of a thrill. Failure of the permanent dialysis access accounts for most hospital admissions of clients receiving ongoing hemodialysis. 2. Incorrect: Assessment of warm and dry skin does not ensure patency of the arteriovenous access device. Warm and dry skin indicates adequate circulation. 4. Incorrect: The presence of a positive Trousseau's sign would indicate hypomagnesemia or hypocalcemia. 5. Incorrect: Capillary refill within 3 seconds is an expected finding in any client with adequate circulation. The question is saying, "Hey new nurse, do you know how to assess the patency of an AV dialysis access?" Well do you? Remember the phrase: "Feel a thrill, hear a bruit?" A thrill is a vibratory sensation felt on the skin overlying an area of turbulence. A bruit is heard through a stethoscope, placed gently on the AV Fistula. You should hear a constant swishing sound. No sense of pulsating should be noticed. What are the correct options? Options 1 and 3: Presence of a bruit and thrill. Look at option 2. Does warm dry skin indicate that the shunt is functional? NO. Option 4? No. What is a positive Trousseau's an indication of? Low Calcium, right? Yes Option 5? Does a capillary refill within 3 seconds indicate that the shunt is patent? No. It indicates adequate circulation to the fingers/toes.

The Emergency Department triage nurse encounters a client who says that he has received exposure to a liquid hazardous chemical at work. He reports that he is only 1 of about 20 people. What should the nurse do? 1. Call the supervisor and inform of the possibility of contamination in the surrounding space. 2. Obtain vital signs immediately. 3. Call personnel trained in containment and decontamination immediately. 4. Direct the individual to a bed space immediately. 5. Instruct the client to remove clothing and put on disposable hospital gown.

1. & 3. Correct: The nurse should report this to the supervisor who can determine the next action to take regarding isolation, decontamination, and use of the current space. Those who are trained in hazardous exposures should be informed immediately so that appropriate action is taken. These actions are priority for minimizing the exposure of clients and staff to the hazardous chemical. 2. Incorrect: The nurse should avoid contact with the client until personnel trained for handling hazardous exposures are present. 4. Incorrect: Containment is necessary to prevent further contamination of the space and individuals in the area. Directing the client to a bed space would not be containment. 5. Incorrect. The client may need to be directed to a decontamination area to prevent further contamination of the area, so removing clothing before going to this area would put others at risk for exposure to the hazardous chemical. Remember client safety is always a priority. This question is talking about the strategic plan for exposures to a liquid hazardous chemical. You want to focus on the contamination of the client and others. So let's look at the options. Option 1 is true. After the client states that a exposure to a liquid hazardous chemical has occurred, the nurse should report this to the supervisor who will initiate the strategic plan for a exposures to a liquid hazardous chemical. The other 19 persons who were exposed will need to be treated according to the strategic plan. Option 2 is false. Safety issues are a priority for the client, but also for the nurse and the healthcare team. The nurse should avoid contact with the client until personnel trained for handling hazardous exposures are present. The client should be monitored within designated distance. Option 3 is true. Those who are trained in hazardous exposures should be informed immediately so that appropriate action is taken. These actions are priority for minimizing the exposure of clients and staff to the hazardous chemical. Option 4 is false. Containment is necessary to prevent further contamination of the space and individuals in the area. Directing the client to a bed space would not be containment. The strategic plan will describe what action to implement for the client prior to their decontamination process. Option 5 is false. Safety issues are a priority for the client and the healthcare team. The client may need to be directed to a decontamination area to prevent further contamination of the area, so removing clothing before going to this area would put others at risk for exposure to the hazardous chemical..

The pediatric nurse is planning an educational seminar for new parents. The seminar will focus on tips for administering medication to children. Which points should the nurse include? 1. Demonstrate proper measuring techniques for liquid medications. 2. Put crushed medications into the child's favorite food. 3. Place liquid medication in an 8-ounce bottle of formula. 4. Call medication "candy" to encourage children to take the medicine. 5. Do not place medications in a container other than the original container.

1. & 5. Correct: Demonstration with return demonstration by the parent is an appropriate teaching strategy. Give clear examples and demonstrations and speak in layman's terms. Never put medications in dishes, cups, bottles, or other household containers that children or other family members may be unaware of. 2. Incorrect: Do not place crushed drugs into the child's favorite food or snack. The medication can change the taste of the food, and the child may refuse, therefore missing part or all of the dose. Additionally, the effectiveness of some medications may be harmed by the crushing of the drugs. 3. Incorrect: Do not place liquid drugs in a large bottle of formula. Unless the child drinks the entire amount, he or she will not receive the correct dose. 4. Incorrect: Don't refer to drugs as candy. Children may try to take more candy leading to overdose.

The charge nurse of a large medical-surgical unit is admitting several clients requiring specific infection control precautions. The nurse is aware that droplet precautions are necessary for which client diagnosis? 1. Mumps 2. Methicillin resistant Staphylococcus aureus (MRSA) 3. Shingles (Herpes Zoster) 4. Human immunodeficiency virus (HIV) 5. Pertussis

1. & 5. Correct: Droplet precautions are utilized whenever a client has specific microorganisms that are spread by coughing, sneezing or talking. Individuals within three feet of the client can be contaminated by breathing in those respiratory droplets. Mumps require approximately 9 days of isolation with droplet precautions after the swelling becomes visible. Clients with pertussis also require droplet precautions. Pertussis is a very contagious disease only found in humans. It is spread from person to person. People with pertussis usually spread the disease to another person by coughing or sneezing or when spending a lot of time near one another where you share breathing space. 2. Incorrect: MRSA is a bacterial infection that can be spread by either direct contact with the client, or indirect contact with environmental surfaces that have been contaminated by the client. This client will require contact precautions. Depending on the type of care being provided to this client, staff may need gloves, gown, mask or eye protection. 3. Incorrect: Shingles (Herpes Zoster) is transmitted through air and direct contact with respiratory secretions and the lesions. Therefore, airborne and contact precautions should be initiated. This client should be placed in a private room with negative pressure to prevent contaminating others. Staff entering this room must wear a special filter mask (N95), gown and gloves. Additionally, pregnant staff or family should not enter the room at all during this time. 4. Incorrect: HIV is a viral disease transmitted through the blood or body fluids of contaminated individuals, blood transfusions, or needles from IV drug usage. Standard precautions are adequate for most HIV clients unless the client has an open wound.

While completing the nutritional history of a client admitted with pernicious anemia, the nurse determines that the client follows a strict vegan diet. What education should the nurse provide to the client? 1. Vitamin B12, a nutrient needed to prevent pernicious anemia, is found in some foods like meat, fish, eggs, and milk. 2. In order to increase intake of vitamin B12, your diet must contain beef or chicken liver at least once per week. 3. In addition to eating plants, you should eat dairy products and eggs in order to prevent pernicious anemia. 4. Vegetables high in protein include cabbage, carrots and squash. 5. Pernicious anemia occurs when the body produces red blood cells that are larger than normal and result in a lower than normal red blood cell count.

1. & 5. Correct: Pernicious anemia is a type of vitamin B12 anemia. The body needs vitamin B12 to make red blood cells. You get this vitamin from eating foods such as meat, poultry, shellfish, eggs, and dairy products. 2. Incorrect: The client does not have to eat meat or dairy products in order to obtain vitamin B-12. Supplements can be taken and the client can eat vegetables that are considered to be high in protein. 3. Incorrect: A strict vegan will not eat dairy products or eggs. 4. Incorrect: For a vegetable to qualify as a low-protein source, it must contain 4g or less of protein. Green vegetables, such as lettuce, cabbage, bell pepper and asparagus provide only 1 to 2g of protein per serving. Orange vegetables, including carrots, sweet potatoes and squash also contain only 1 to 2 g.

A client diagnosed Alzheimer's disease has been prescribed memantine. What should the nurse teach the caregiver about this medication? 1. When beginning this medication provide ambulatory assistance. 2. This medication is prescribed to help improve mild dementia. 3. This medication must be taken without food. 4. If a dose is missed, double the next dose. 5. If the client cannot swallow the capsule you sprinkle on applesauce.

1. & 5. Correct: This medication can cause dizziness, so safety precautions should be taught to the caregiver. Extended release caps should not be crushed, chewed, or divided. If the client cannot swallow it whole, it can be opened and sprinkled on a small amount of applesauce. 2. Incorrect: Memantine is used for moderate to severe dementia associated with Alzheimer's disease. 3. Incorrect: Memantine can be taken with or without food. 4. Incorrect: If the client misses a single dose of memantine, that client should not double up on the next dose. The next dose should be taken as scheduled. Memantine is used to treat moderate to severe dementia associated with Alzheimer's disease. Common side effects of this medication are dizziness or drowsiness. So a primary goal is to prevent injury by initiating safety precautions. These effects may be worse if the client drinks alcohol. The client should not drive or perform other possibly unsafe tasks until they know how they will react to it. The client should NOT take more than the recommended dose. If the client misses a single dose of memantine, that client should not double up on the next dose. The next dose should be taken as scheduled. Memantine can be taken with or without food. If stomach upset occurs, take with food to reduce stomach irritation.

Which assessments will provide the nurse with the most information regarding a client's neurologic function? 1. Level of consciousness 2. Doll's eyes reflex 3. Babinski reflex 4. Reaction to painful stimuli 5. Verbal ability

1. & 5. Correct: Yes, the most important and subtle changes are related to the client's level of consciousness, verbal ability, orientation, and ability to move to command. 2. Incorrect: No, only helps with the determination of brain death. 3. Incorrect: Identifies diseases of the brain and spinal cord. 4. Incorrect: This should be last resort.

A client has been given information about several complementary therapies for the treatment of anxiety disorder. Which therapy selected by the client would require the nurse to check for allergies? 1. Aromatherapy 2. Biofeedback 3. Guided Imagery 4. Acupuncture

1. CORRECT. Aromatherapy is the use of essential oils from plants and herbs in the form of baths, inhalation, or compresses applied directly to the skin to promote relaxation, decrease depression and enhance sleep. Because these oils come in contact with the client's skin, or by inhalation, it would be important to verify any allergies the client may have prior to initiating therapy. 2. INCORRECT. Biofeedback is progressive muscle relaxation with the use of electrodes placed on the client's skin. This therapy has been used to treat medical issues such as migraines or chronic pain. Allergies would not be a concern with this complementary therapy. 3. INCORRECT. Guided Imagery, also called "visualization", uses words or sounds to direct the client on an imaginary journey within the mind. This technique for dealing with anxiety would not present any concerns about allergies. 4. INCORRECT. Acupuncture is a complementary therapy that entails stimulating certain areas of the body by penetrating the skin with a variety of tiny needles in order to treat a variety of physical and emotional disorders.The nurse would not be concerned about allergies with this therapeutic treatment.

The nurse is instructing the mother of a pre-school child newly diagnosed with cystic fibrosis (CF). What statement by the mother indicates to the nurse that further teaching will be necessary? 1. "I will prepare a low fat, low protein diet for meals." 2. "Pancreatic enzymes are given with meals and snacks." 3. "I should encourage extra fluids during the day." 4. "I do not need to limit salt when preparing foods."

1. CORRECT: Children diagnosed with cystic fibrosis have difficulty getting enough nutrients because of an inherited defect that impairs the body's ability to absorb fats or protein. This child needs high calorie meals and snacks, ideally with extra calcium and salt. In addition, the child will take supplemental fat-soluble vitamins like A, D, E and K. This statement indicates the need for further teaching. 2. INCORRECT: Cystic fibrosis is a defect in mucus producing cells and digestive enzymes. Therefore, supplemental pancreatic enzymes must be administered with all meals and snacks. This is an accurate statement by the mother and no need for more instruction. 3. INCORRECT: Children with cystic fibrosis lose large amounts of salt in body sweat, and therefore are usually dehydrated. Large amounts of fluid should be provided during the day. Additionally, extra fluids help to thin the thick, sticky mucus that clogs the lungs and digestive tract. This is an accurate statement so no further teaching is needed. 4. INCORRECT: Cystic fibrosis symptoms can vary, even in the same client. But one consistent sign is the loss of copious amounts of salt in sweat. Parents often notice this on the skin when they kiss the child. Because of the hyponatremia, extra salt in meals and snacks is important. This is a correct statement, so the nurse will not need to reinstruct the mother.

A client at 31 weeks gestation is being seen by the primary healthcare provider for reports of generalized illness. When assessing the client, the nurse would immediately report what symptom to the primary health care provider? You answered this question Incorrectly 1. Right upper quadrant pain 2. Nausea with vomiting 3. Severe headache 4. Blurred vision

1. CORRECT: The symptoms being reported by the client indicates hemolysis of blood cells, elevated liver enzymes and low platelet count (HELLP) Syndrome, a life-threatening liver disorder related to preeclampsia. Occurring generally in the third trimester, or even right after birth, the exact cause is unknown. The only treatment is to deliver the fetus. 2. INCORRECT: Nausea and vomiting are generally complaints reported early in the first trimester. The return of these symptoms could indicate a general illness but would need to be investigated. However, nausea and vomiting is not the greatest concern with this client. 3. INCORRECT: A severe headache would lead the nurse to investigate blood pressure, as preeclampsia could cause elevated blood pressure. However, this is not the most serious reported symptom by the client. 4. INCORRECT: Blurred vision is definitely alarming and should be reported to the primary healthcare provider by the nurse. Combined with a severe headache, the nurse will need to report these findings. But there is another finding that is more urgent to report.

A nurse is observing two unlicensed assistive personnel (UAP) changing sheets for an immobile, obese client. What unacceptable action by the UAPs would require the nurse to intervene? 1. Stands straight with feet together. 2. Asks client to lift head off the bed. 3. Pulls draw sheet with both hands. 4. Faces slightly towards head of bed.

1. CORRECT: When moving a client, the most important safety action for the staff doing the lifting is to spread their feet apart to shoulder width, with knees slightly bent, to prevent back injury. The feet should never be placed together. The most stable part of the body is at the hips, and moving feet apart stabilizes the lifter. The nurse would intervene in this scenario before the UAPs is injured. 2. INCORRECT: The UAPs are aware when sliding a client up in bed, if the client does not lift their head, the sudden movement could hyperextend the client's neck, causing severe trauma. The client must lift head off bed just before the staff moves the draw sheet to prevent neck injury. This is a correct action. 3. INCORRECT: When moving an obese client, there should be at least two staff members on each side of the bed, grasping the draw sheet with both hands. With a firm grasp on the draw sheet, the staff then slides the client upward in the bed. The UAPs completed this action correctly. 4. INCORRECT: Before moving the client upward, all staff should turn slightly toward the head of the bed, feet planted shoulder width apart and firmly grasp the draw sheet with both hands. This position is correct for both client and staff safety.

Which statement made by a client post-thyroidectomy would require further investigation by the nurse? 1. "I have a tingling feeling of my fingers." 2. "It hurts when I move my head." 3. "I feel pressure in my arm when you take my blood pressure." 4. "My legs are weak."

1. Correct. After this procedure the nurse should worry about the possibility of some of the parathyroids being accidentally removed with resulting hypoparathyroidism. Hypoparathyroidism results in hypocalcemia. Signs and symptoms include tingling, burning, or numbness of lips, fingers, and toes. The muscles may become tight and rigid, and seizures can result. 2. Incorrect. Pain is expected here. The incision is at the base of the neck, so movement of the head would increase the pain. 3. Incorrect. The sensation of pressure in the arm is considered normal when the BP is being measured. You worry if you see carpal spasm (+ Trousseau's) which is indicative of neuromuscular excitability caused by hypocalcemia secondary to the inadvertent removal of some of the parathyroids. 4. Incorrect. Weak/flaccid extremities would be seen with hyperparathyroidism. In this case, we are concerned that the parathyroids may have been removed, resulting in hypoparathyroidism. The weakness in the legs is apparently from a different cause. However, the signs of possible hypoparathyroidism would be the priority to investigate.

A client was prescribed a monoamine oxidase inhibitor (MAOI) for the treatment of depression. Which comment by the client indicates adequate understanding of the tyramine restrictions that apply? 1. I cannot eat avocados or smoked ham. 2. I can eat sausage for breakfast, but not bacon. 3. At least I can still have my beer. 4. I can have blue cheese on my salad but not ranch dressing.

1. Correct. Clients taking MAOIs cannot consume foods containing large amounts of tyramine. MAOIs block monoamine oxidase which breakdown tyramine. Having a MAOI prescribed and eating a diet high in tyramine can cause a severe increase in blood pressure. Smoked ham and avocados are high in tyramine. 2. Incorrect. Clients taking these medications cannot eat the following foods: sausage, salami, liver, or bologna which have high levels of tyramine. 3. Incorrect. Clients taking these medications cannot consume beer, sherry, chianti wines, or ales due to their high tyramine levels. 4. Incorrect. Consuming blue cheese on a salad may result in a hypertensive crisis due to the presence of tyramine.

The nurse is preparing to transfer a client from the delivery room to the postpartum unit. Which statement by the client would cause the nurse to re-assess the client prior to transfer? 1. "I just felt something gushing." 2. "I feel like I am still having contractions." 3. "When I stand up I feel dizzy for several moments" 4. "My nipples hurt since I breastfed my baby."

1. Correct. This could indicate postpartum hemorrhage (PPH) and requires immediate assessment by the nurse. PPH can be caused by the following: placenta previa, cervical lacerations, vaginal tear or a ruptured or inverted uterus. 2. Incorrect. This is normal postpartum contractions of the uterus to help dispel clots and to return the uterus to normal size. The contractins may occur for several days after delivery. 3. Incorrect. Due to the fluid loss during the delivery, the client may be experiencing orthostatic hypotension. Teach safety measures. This statement does not require the client to stay in the delivery room. . 4. Incorrect. The breast may become tender after breast feeding the infant. This is not the priority and can be dealt with on the postpartum unit.

A client returns to the nursing unit post-thoracotomy with two chest tubes in place connected to a drainage device. The client's spouse asks the nurse about the reason for having two chest tubes. The nurse's response is based on the knowledge that the upper chest tube is placed to do what? 1. Remove air from the pleural space 2. Create access for irrigating the chest cavity 3. Evacuate secretions from the bronchioles and alveoli 4. Drain blood and fluid from the pleural space

1. Correct: A chest tube placed in the upper chest is to remove air from the pleural space. Remember air rises and fluid settles down low. 2. Incorrect: Chest tubes are placed in the pleural space to get rid of air, blood, fluid, or exudate so that the lung can re-expand. The purpose is not to create an access for irrigating the chest cavity. 3. Incorrect: The chest tube is inserted into the pleural space because the lung has collapsed due to air, blood, fluid, or exudate. The chest tube does not go into the lung so secretions can not be removed from the bronchioles and alveoli by way of the chest tube. 4. Incorrect: You have to know the purpose of the upper chest tube. Fluid drains down, so the lower one is for fluid.

A 40 year old client reports a diminished ability to visually focus on close objects and has also noticed a need for a well lit environment to enhance vision. To what would the nurse attribute these changes? 1. Normal changes associated with aging. 2. A cataract is forming. 3. Symptoms of a brain tumor. 4. Precipitated by diabetic retinopathy.

1. Correct: Aging results in stiffening of the lens, thus lessening the ability to focus. The retina is less sensitive to light, making accurate vision in low-light situations more difficult. Pupillary response diminishes, affecting the ability to adjust to changing light levels. 2. Incorrect: Cataracts present with blurred vision and a glare from lights. 3. Incorrect: Brain tumors increase intracranial pressure, resulting in blurring of vision. 4. Incorrect: Diabetic retinopathy is caused by changes in retinal blood vessels and results in blurred vision and outright impairment in some fields.

A client has been admitted voluntarily to the psychiatric unit. During the admitting interview, the client confides to the nurse that they have a lethal plan for committing suicide. At the end of the interview the client asks the nurse, "How long will I have to stay here?" What should the nurse say to this client? You answered this question 1. "Let's discuss this after the health team has assessed you." 2. "Since you signed papers to be admitted, you cannot leave until the primary healthcare provider discharges you." 3. "A lawyer will have to make that decision." 4. "You can leave when you are no longer suicidal."

1. Correct: A client may sign out of the hospital at any time, unless following a mental status examination the healthcare professional determines that the client may be harmful to self or others and recommends that the admission status be changed from voluntary to involuntary. 2. Incorrect: A client may sign out of the hospital at any time, unless following a mental status examination the healthcare professional determines that the client may be harmful to self or others and recommends that the admission status be changed from voluntary to involuntary. 3. Incorrect: Lawyers do not make that decision. This client was voluntarily admitted, not involuntary. Involuntary admission can be from three different commitment procedures: judicial, administrative and agency determination. Involuntary admission can be further categorized as emergency, observational/temporary or indeterminate/extended. 4. Incorrect: This is not the best response, since the client has told you of a plan. They might decide to tell you they have no plan when in fact they do.

Which assigned postpartum client should the nurse identify as being at highest risk for hemorrhage? 1. C-section delivery 2. Vaginal delivery of twins 3. Vaginal delivery of premature baby 4. Precipitous delivery of gravida 5

1. Correct: A client with a surgical wound is at risk for hemorrhage and is at greater risk than birth from a vaginal delivery. The surgical opening of the abdomen and the uterus makes this the highest risk. 2. Incorrect: If the placenta is removed and the fundus massaged properly, risk of hemorrhage decreases. The risk of hemorrhage goes up with multiple births, such as twins, as compared with a single birth, but it is still not as high a risk as a c-section. 3. Incorrect: Premature does not place the client at higher risk of bleeding. The premature newborn is generally smaller with less risk of damage to the uterus and perineum of mom. 4. Incorrect: A precipitous delivery could make you think tear, but the client is Gravida 5. Tearing is less likely after having 5 children. This question is looking for the test taker to determine which client is at highest risk for hemorrhage? Which option could be eliminated immediately? The gravida 5 precipitous delivery. Since this is the fifth delivery, there is little chance of tearing. Because the nurse will be assessing each client, the test taker must determine which client is a priority. Although hemorrhage is likely in all of these clients, which once is most likely to develop this complication? Surgical clients are at a high risk for hemorrhage. So, the c-section client is at greatest risk. There is a surgical incision to the abdomen and the uterus with a c-section.

The nurse is caring for a client who has a history of sleep apnea. The client is scheduled for a colon resection the following morning and asks if the sleep apnea machine should be brought to the hospital. What is the nurse's best response? 1. Yes, bring the sleep apnea machine. 2. No, do not bring the sleep apnea machine. 3. It is your choice. 4. Call your primary healthcare provider.

1. Correct: A client with sleep apnea is at risk for cardiac and respiratory complications postop due to decreasing oxygenation. So yes, the client needs to use the CPAP machine. Remember this client will also be receiving narcotics for pain and have a decreased activity level as well. All of these things can decrease oxygenation. 2. Incorrect: The client will need to have the machine after surgery. 3. Incorrect: Best response is for nurse to recommend that the client bring machine. 4. Incorrect: The nurse can answer this question. Sleep apnea is a concern for clients who are about to have surgery. These clients are at an increased risk for developing respiratory and cardiovascular complications in the postoperative period. Complications can include irregular heart rhythms, oxygen deficiency, high blood pressure, diabetes, stroke, heart attack, and even death. Positive airway pressure (PAP) is considered the gold standard treatment for sleep apnea. With continuous positive airway pressure (CPAP), the client will wear a mask over the nose and/or mouth while sleeping. An air blower will force air through the airway, which will prevent it from closing during sleep. The nurse should closely monitor the client's vital signs both during and after surgery.

The nurse walks into a client's room and finds the client exposed while the unlicensed assistive personnel (UAP) is giving the bath. After covering the client with a sheet, what should the nurse do first? 1. Tell the UAP to keep the client covered at all times. 2. Talk with the UAP about providing appropriate care for all clients. 3. Provide teaching to the UAP about privacy for clients. 4. Use the call light to ask for additional assistance in the room.

1. Correct: A comment should be made about keeping the client covered. This instruction is the first action after covering the client. 2. Incorrect: The nurse should talk with the UAP but the discussion should focus specifically about providing privacy for clients. 3. Incorrect: The nurse may want to provide teaching, but this is not first action. Teaching would require allowing enough time to give instructions and then arranging time for return demonstration. 4. Incorrect: The UAP should be allowed to finish the bath. Additional assistance is not needed.

The nurse is caring for a client taking a selective serotonin reuptake inhibitor (SSRI). The client tells the nurse "I am sweating more than ever!" What is the nurses best response? 1. This is a common side effect of antidepressant medications. Perhaps a different antidepressant would cause less side effects. 2. Excessive sweating can have many causes. 3. I think that you should report this side effect to your primary healthcare provider. 4. This symptom should go away within a few days.

1. Correct: A common side effect of SSRIs is increased sweating. This option also gives the client an explanation. 2. Incorrect: This response shows a lack of understanding of the side effects of anti depressant medications. 3. Incorrect: This option does not acknowledge the client's problem and possible causes. 4. Incorrect: Increased sweating may continue throughout treatment with an antipsychotic medication.

The out patient surgical unit has admitted multiple clients currently awaiting early morning procedures. What client should the nurse assess first? 1. The client awaiting repair of hiatal hernia reporting chest pain. 2. The client with a torn right rotator cuff reporting shoulder pain. 3. The client with an inguinal hernia repair reporting skin irritation. 4. The client awaiting a hemorrhoidectomy reporting rectal bleeding.

1. Correct: A hiatal hernia occurs when a portion of the stomach pushes up through the esophageal ring (hiatus) of the diaphragm. Surgical intervention is generally a last resort and only when there is evidence of serious complications. Although chest pain could be the result of reflux within the esophagus, it could also indicate a strangulated hiatal hernia. The nurse needs to assess this client immediately. 2. Incorrect: A torn rotator cuff is generally only repaired when other treatment options have been ineffective, such as rest, ice, NSAIDs and even steroid injections. This client has been ordered a surgical repair, indicating other therapies have failed. Shoulder pain on the affected side is to be expected and not an urgent need. 3. Incorrect: An inguinal hernia is the protrusion of intestine through abdominal muscles, creating a painful bulge which worsens with lifting, bending, or straining. Skin irritation usually results from wearing a supportive garment known as a truss. The purpose of this belted device is to apply pressure and provide support to the area of the hernia until surgical repair. Skin irritation is not the nurse's priority. 4. Incorrect: Large or engorged rectal hemorrhoids may require surgical repair because of excessive bleeding, pain, or prolapse. This type of bleeding is not unexpected nor does it present any major concerns about shock. This client would not need to be seen first.

A client consumes a lacto-ovo vegetarian diet at home. During hospitalization, the primary healthcare provider prescribes an increased calorie diet. Which foods are appropriate for the nurse to serve as between meal snacks to boost caloric intake? 1. Cheese sandwich and milk 2. Boiled eggs but no dairy products 3. Fish sticks and cocktail sausages 4. Fresh vegetables but no milk or eggs

1. Correct: A lacto-ovo vegetarian diet is a vegetarian diet that does not include meat, but does contain eggs and dairy. The client can eat milk and dairy products along with grain products on this diet. 2. Incorrect: Dairy products and eggs are allowed on this diet. Milk, cheese and yogurt can be consumed on a lacto-ovo vegetarian diet. 3. Incorrect: The client does not consume meats. Meats should not be provided as a snack. 4. Incorrect: The client can consume milk and eggs as well as fresh fruits and vegetables. Milk and eggs can be consumed on a lacto-ovo vegetarian diet.

Which room assignment would be most therapeutic for the nurse to make for a client with bipolar disorder in manic phase who is hyperactive and has difficulty sleeping? 1. A private bedroom. 2. A semi private room with a roommate who has a similar problem. 3. Either a private or a semi private room. 4. Direct admission to the seclusion room until his activity level becomes more subdued.

1. Correct: A private room will help to decrease stimulation. The client with bipolar disorder needs a calm environment especially when in the manic phase. Avoid excessive stimulation. 2. Incorrect: Don't put two manics together. This room assignment will not help to decrease stimulation which is what the manic client needs. 3. Incorrect: They need a private room. The client with psychosis maybe suspicious and have delusion or hallucinations. 4. Incorrect: There's no need for this right now. The client is hyperactive and has difficulty sleeping. A seclusion room is needed for severe agitation and acute aggression.

What term should the nurse use to document that a woman is pregnant for the first time? 1. Primigravida 2. Multigravida 3. Primipara 4. Multipara

1. Correct: A woman pregnant for the first time. The prefix "primi" means first. "Gravida" refers to a woman who is or has been pregnant, regardless of the duration or outcome of the pregnancy. 2. Incorrect: A woman who has had 2 or more pregnancies. The term "multi" means more than one. "Gravida" refers to a woman who is or has been pregnant. 3. Incorrect: A woman who has completed 1 pregnancy with a fetus or fetuses who have reached 20 weeks of gestation. The term "primi" means first. "Para" refers to the number of pregnancies that have progressed to 20 or more weeks at delivery. 4. Incorrect: A woman who has completed 2 or more pregnancies to 20 or more weeks of gestation. The term "multi" means more than one. "Para" refers to the number of pregnancies that have progressed to 20 or more weeks at delivery whether the fetus was born alive or still born: refers to the number of pregnancies, not the number of fetuses.

The parents of a 4 year old child have recently had a new baby and the parents report that the 4 year old had been dry all night for 8 months and is now wetting the bed again. What should the nurse assess first? 1. Urinalysis 2. Normal urination habits. 3. Adjustment to the new baby. 4. Fluid intake after 6 pm.

1. Correct: Always assess the physiologic problem first to rule out a urinary tract infection (UTI). Once a physiologic cause is removed as the cause other assessment should be performed. If a UTI is present, treatment should start immediately. 2. Incorrect: Assessing the normal urination habits is not first. Assessing the urinalysis is priority. 3. Incorrect: Regression is the likely cause but the physiologic problems should be assessed first. 4. Incorrect: The child's fluid intake may be too high after 6 pm, but ruling out a urinary tract infection is the first assessment and requires immediate treatment if there is an infection.

A night nurse is receiving report from the day nurse when the day nurse states, "I have an appointment and I need to leave. Can you get the rest of the client's information from the medical records?" What client right may be compromised by the day nurse's request? 1. Reasonable continuity of care 2. Confidentiality 3. Considerate and respectful care 4. Participation in decision making

1. Correct: An incomplete or uninformative client report from one healthcare provider to the next may compromise the client's right to reasonable continuity of care. The properly done hand-off report shares essential information and helps to provide client safety and continuity of care. The oncoming nurse needs an opportunity to clarify information from the outgoing nurse. 2. Incorrect: The client's right to confidentiality is not violated because both nurses have been assigned to care for the client. Information pertaining to client care is passed between healthcare providers during care reports to safeguard continuity of care. 3. Incorrect: The right to considerate and respectful care is not addressed in this scenario. 4. Incorrect: The right to make decisions about the plan of care is not addressed in this scenario.

The nurse is assessing the injection site of a healthy client who received a Mantoux skin test 48 hours ago. Which finding at the injection site indicates a need for further evaluation? 1. 16 mm induration 2. 4 mm erythrokeratodemia 3. 0.1 mL bluish colored hard wheal 4. Multiple fluid-filled vesicles

1. Correct: An induration of 15 mm or greater is usually considered positive in people who have normal or mildly impaired immunity. A client with a positive reaction of 15mm or greater will need further evaluation by a primary healthcare provider. 2. Incorrect: This is a small, red, hard area that is smaller than 10 mm. Therefore the size is not considered significant. Induration is roughness, not hardness. The induration is what nurses assess to determine significance. 3. Incorrect: When administering a Mantoux skin test, 0.1 mL of solution is injected under the top layer of the skin to produce a wheal. The presence of the 0.1 mL wheal is not expected at this time. 4. Incorrect: This is the significant reaction that one would find with a multiple puncture tine, which is sometimes used with mass screening for TB. This is not expected with a Mantoux skin test.

The nurse is assessing the injection site of a healthy client who received a Mantoux skin test 48 hours ago. Which finding at the injection site indicates a need for further evaluation? 1. 15 mm induration 2. 4 mm erythrokeratodemia 3. 0.1 mL bluish colored hard wheal 4. 0 mm induration

1. Correct: An induration of 15 mm or greater is usually considered significant in people who have normal or mildly impaired immunity. 2. Incorrect: This is a small, red, hard area that is smaller than 10 mm. Therefore the size is not considered significant. Induration is roughness, not hardness. The induration is what nurses assess to determine significance. 3. Incorrect: When administering a Mantoux skin test, 0.1 mL of solution is injected under the top layer of the skin to produce a wheal. The presence of the 0.1 mL wheal is not expected at this time. 4. Incorrect: This is normal finding in someone who has not been exposed to TB.

A client is diagnosed with a duodenal ulcer due to Helicobacter pylori (H Pylori). In addition to antibiotic therapy, the nurse anticipates that the client will also receive what class of pharmacologic agent? 1. Proton pump inhibitor 2. Mitotic inhibitor 3. Serotonin antagonist 4. Acetylsalicyclic acid

1. Correct: Antisecretory agents like proton pump inhibitors are indicated for the treatment of peptic ulcer disease. Antisecretory agents decrease the secretion of gastric acids. Protein pump inhibitors, a combination of antibiotics and bismuth salts are most commonly used for treatment of H Pylori. 2. Incorrect: Mitotic inhibitors are chemotherapeutic agents that are indicated for the treatment of malignancies and cancerous cells. They are most often used in combination chemotherapy regimens to enhance the overall cytotoxic effect. 3. Incorrect: Serotonin antagonists are antiemetic agents that are indicated for the treatment of nausea and vomiting. Serotonin antagonists block the serotonin receptor sites located throughout the body responsible for the mediation of nausea and vomiting. 4. Incorrect: Acetylsalicylic acid is a non narcotic analgesic that inhibits the cox-2 protective mechanisms to the gastric mucosa. This could make the ulcer worse. Clients are advised to avoid the use of NSAIDs and acetylsalicylic acid due to increased bleeding potential.

The nurse is caring for a client with questionable loss of consciousness in the emergency department following a motor vehicle crash. Which action should the nurse take first? 1. Assess airway patency, breathing, and circulation. 2. Assess level of consciousness and movement. 3. Cover wounds with a sterile dressing. 4. Maintain cervical spine immobilization.

1. Correct: Any time a nurse is faced with emergency management, the primary survey should be followed. The beginning order of the primary survey is airway, breathing, circulation. 2. Incorrect: After completion of airway, breathing and circulation assessment, the nurse should assess for neurologic disability. This would include assessment of the client's level of consciousness (LOC). 3. Incorrect: Measurement of a full set of vital signs would occur after assessment of airway, breathing and circulation. 4. Incorrect: The secondary survey includes stabilization of the neck and assessment for signs of neck injury.

A 9 month old with asthma symptomology has montelukast sodium oral granules prescribed. What is the most appropriate way for the nurse to instruct the parent on how to administer the medication? 1. Mix the granules with a spoonful of baby food such as applesauce. 2. Pour the granules directly on the back of the infant's tongue. 3. Dissolve the granules in an 8 ounce (240 mL)bottle of juice. 4. Administer the medication in the morning mixed in a bowl of rice cereal.

1. Correct: Applesauce is an appropriate baby food for a 9 month old infant. The medication is being mixed with a very small amount of baby food to facilitate all of the medication being consumed. 2. Incorrect: Although the medication can be administered directly into the mouth, a 9 month old is not likely to tolerate medication granules being placed in the back of the mouth and would likely spit the medication out or gag when the medication is placed in the back of the mouth, 3. Incorrect: The medication is being placed in too much juice. The infant might not drink this amount and would not receive all of the medication ordered. 4. Incorrect: If the child does not eat the entire amount of the cereal, the child would not receive the prescribed dose of the medication.

A five year old is in kindergarten and goes to the nurse's office where she reports a "stomachache". While there, the nurse observes that the child has a large bruise on her upper arm and bruises on both ears. What should the nurse do first? 1. Ask the student about the bruises on the arms and ears. 2. Do nothing as bruises are common in 5 year old children. 3. Report the injuries immediately to the parents. 4. Discuss the findings with the child's teacher.

1. Correct: Assessment and gathering information should be the first response. The child may have experienced a severe accident that does not indicate abuse. The nurse needs further information before assuming abuse in the family. 2. Incorrect: Bruises on the upper arms and ears are not typical for a child of this age. 3. Incorrect: The parents may have inflicted the injuries on the child which will be important to assess later, but is not the first action for the nurse. 4. Incorrect: The nurse should discuss the observations with the teacher to determine if other indicators of abuse are present. The first step is to communicate with the child and further assess the situation.

A client on the in-patient psychiatric unit was found to have lacerations on the wrist when the nurse made rounds. Which change in routine on the unit is most likely to prevent such an event from occurring in the future? 1. During the end-of-shift report, assign specific staff to check on each client. 2. Place newly admitted clients close to the nursing station. 3. Monitor level of suicide precaution needed on each client daily. 4. Ask clients to check on each other throughout the shift.

1. Correct: Assigning specific staff to perform client checks during the shift will assure that the clients, that the staff are concerned about their welfare. In addition, it assures that someone is specifically monitoring the client each shift, therefore, promoting the clients right to a safe environment. Client safety is a priority in Maslow's Hierarchy of Needs. The nurses will play a key role in reducing these self-harming behaviors through recognition of the problem, being alert to risk factors when assessing the client, and ultimately guiding the client into more acceptable outlets for stress, anxiety, anger, low-self esteem, or other related causes. 2. Incorrect: This routine may or may not prevent an injury. The clients may learn the "routine" of the nurses and will perform the self-harming behaviors when the nurses are not likely to be making individualized checks on them. 3. Incorrect: Each client should be monitored daily at irregular intervals. Self-harming behaviors typically increase the risk of suicide in the client. The nurse should determine the level of imminent risk of suicide in the client. This should be routinely performed with client checks, not simply assessed on a daily basis. 4. Incorrect: It is not the clients' responsibility to check on each other. All clients have the right to a safe environment, and it is the responsibility of the nurses and healthcare team to provide this safety.

A nurse observes a psychiatric client sitting alone. The client is talking, but occasionally stops and leans to the side as if listening to someone. The client then laughs. What is this client most likely experiencing? 1. Auditory hallucinations 2. Delusions 3. Catatonic excitement 4. Anergia

1. Correct: Auditory hallucinations are false sensory perceptions of sound not associated with real external stimuli. When the client begins to respond to a stimuli that is not visible to the nurse, this is a hallucination. 2. Incorrect: Delusions are false personal beliefs that are inconsistent with the person's intelligence or cultural background. These beliefs are not consistent with reality and guide the client's behavior. Typically the client thinks they are all powerful, or have unrealistic fears. 3. Incorrect: Catatonic excitement is manifested by a state of extreme psychomotor agitation. This client is not showing any symptoms of agitation. 4. Incorrect: Anergia is a deficiency of energy to carry out activities of daily living. There are no indications in the question that this client cannot carry out activities of daily living.

The nurse is having an education class for pregnant women. A question is raised about exercise. What is the nurse's best response? 1. Discuss with healthcare provider your current exercise regimen and history. 2. You can continue any exercise that you have been doing before pregnancy. 3. If you haven't already started an exercise program, you should wait until after delivery. 4. Exercise is required during pregnancy for a minimum of 15 minutes each day.

1. Correct: Best advice for pregnant women. The healthcare provider can individualize according to the physical condition of the woman and the stage of pregnancy. 2. Incorrect: As pregnancy progresses, the exercise program may need modification because the change in the woman's center of gravity makes her more prone to falls. Therefore, an activity that is safe in the first trimester may not be safe in the third trimester. Those women who have been exercising strenuously before pregnancy should consult the healthcare provider but may be able to continue much of their usual routine. Recreational sports generally can be continued if no risk of falling or abdominal trauma exists. 3. Incorrect: Exercise during pregnancy is generally beneficial and can strengthen muscles, reduce backache, reduce stress and provide a feeling of well-being. The amount and type of exercise recommended depend on the physical condition of the woman and the stage of pregnancy. 4. Incorrect: Women who have no medical or obstetric complications should exercise in moderation each day for 30 minutes or more during pregnancy.

The nurse assessing clients in a pediatric clinic would refer which child for further assessment? 1. A 20 month old who only says "no." 2. A 1 year old who says three words 3. A 9 month old who says "dada" and "mama" 4. A 4 month old who laughs out loud

1. Correct: By 18 months of age, a child should be able to speak 10 or more words. 2. Incorrect: By 1 year of age, a child should be able to say "mama," "dada," and an additional 3 to 5 words. 3. Incorrect: By 9 months of age, a child should be able to say "mama" and "dada." 4. Incorrect: By 4 months of age, a child should be able to laugh out loud. Option 1 is the true answer because the child needs further assessment. By 18 months of age, a child should be using 5 to 20 words. Option 2 is false because the child is meeting developmental expectations. The average 12 month old uses 2 to 3 words. Option 3 is true in that the child is meeting developmental expectations. Around 9 to 12 months, a child begins saying "mama" and "dada" as well as imitates other speech sounds. Option 4 is true in that the child is meeting developmental expectations. At 4 to 7 months old, squealing and yelling begin. At 4 to 5 months old a child begins to make simple vowel sounds, laughs aloud, and vocalizes in response to voices.

Which comment made by a new nurse regarding calcium gluconate 1000 mg (10 mL) IV indicates to the charge nurse that further education is needed? 1. "Infusion rate should be 5 mL/minute." 2. "Calcium gluconate will counteract the effects of the client's hyperkalemia." 3. "I will monitor for hypophosphatemia after administering this medication." 4. "This medication is given to reverse the effects of hypermagnesemia."

1. Correct: Calcium gluconate is administered IVP very slowly. Rapid injection may cause vasodilation, decreased blood pressure, bradycardia, cardiac arrhythmias and even cardiac arrest. The max rate is 1.5- 2 mL/min. Administration at a faster rate would indicate further education is needed. 2. Incorrect: This is a correct statement by the new nurse, indicating that the nurse understands the use of this medication. It counteracts the effects of hyperkalemia on cardiac excitability. 3. Incorrect: This is a correct statement. Calcium and phosphorus have an inverse relationship to each other. As calcium goes up, phosphorus goes down. Hypophosaphetemia may occur after administration. 4. Incorrect: This is a correct statement. Calcium gluconate is used to treat calcium deficiencies as well as magnesium sulfate overdose.

A client is admitted to the critical care unit after suffering from a massive cerebral vascular accident. The client's vital signs include BP 160/110, HR42, Cheyne-Stokes respirations. Based on this assessment the nurse anticipates the client to be in which acid/base imbalance? 1. Respiratory acidosis 2. Respiratory alkalosis 3. Metabolic acidosis 4. Metabolic alkalosis

1. Correct: Causes of respiratory acidosis include any causes of decreased respiratory drive, such as drugs (narcotics) or central nervous system disorders. With a massive cerebral vascular accident (CVA or stroke), the respiratory center in the brain is impaired and affects oxygenation. Cheyne-Stokes respirations are characterized by progressively deeper and sometimes faster respirations followed by periods of apnea. This leads to acidosis and often times respiratory arrest. 2. Incorrect: Respiratory alkalosis includes hyperventilation and tachypnea which does not describe the characteristics of Cheyne-Stokes respirations. 3. Incorrect: Metabolic acidosis includes tachypnea with deep respirations called Kussmaul's respirations caused by disorders like DKA. 4. Incorrect: The most common cause of metabolic alkalosis is vomiting, and this is clearly a respiratory problem, not metabolic.

The nurse is planning care for four clients with different medical issues. With which diagnosis would a client benefit most from an integrative medicine healthcare strategy? 1. Chronic fatigue syndrome who has had no relief of fatigue. 2. Diabetes whose blood sugars are out of control and refuses to take the prescribed oral and injection medications. 3. Cholecystitis who wants surgery to treat the symptoms definitively. 4. Productive cough with green sputum, fever of 104.2 degrees Fahrenheit (40.1 degrees C), and chest pain.

1. Correct: Chronic fatigue syndrome is a chronic health problem that is difficult to treat using only traditional medicine and responds well to the use of an integrative medicine healthcare strategy by using a combination of traditional and holistic therapies. Integrative medicine is an approach to care that puts the patient at the center and addresses the full range of physical, emotional, mental, social, spiritual and environmental influences that affect a person's health. 2. Incorrect: Clients with acute illness symptoms are more appropriately treated with traditional medicine strategies. 3. Incorrect: Clients with acute illness symptoms are more appropriately treated with traditional medicine strategies. 4. Incorrect: Clients with acute illness symptoms are more appropriately treated with traditional medicine strategies.

A client admitted to the inpatient mental health unit asks if mail can be received from family. Which statement by the nurse indicates adequate understanding of client rights? 1. Clients can receive and send mail, but staff must check for hazards. 2. Clients are not allowed to receive mail while hospitalized. 3. Receiving mail from family is not encouraged. 4. Clients are allowed to send or receive mail after the first 72 hours after admission.

1. Correct: Clients are allowed to send and receive mail. Mail must be checked for hazards to protect the client and the safety of others on the unit. 2. Incorrect: This statement indicates lack of understanding of client's rights. This includes the right to send and receive "sealed, unopened, uncensored mail." If the client is present, staff may open and check mail for contraband, but may not read it. The superintendent, director, or designee of the superintendent or director must document with specific facts the reason for opening the mail. 3. Incorrect: This statement indicates lack of understanding of client's rights on the mental health unit. 4. Incorrect: The client has a right to send and receive mail throughout their hospital stay.

Which discharge referral would be a priority for the nurse to make in order to promote continuity of care for a client following a colectomy and colostomy formation due to colon cancer? 1. Home health 2. Meals on Wheels 3. Hospice care 4. Registered dietitian

1. Correct: Clients often go home quickly and do not completely understand discharge instructions. The first priority would be for colostomy care, which can be provided by home health. 2. Incorrect: Meals on Wheels will be important later during rehabilitation but is not the priority. 3. Incorrect: Hospice care is premature. The question does not reveal if surgery was successful or not to remove the colon cancer. 4. Incorrect: A dietary consult may be necessary later, but is not the priority at present.

The nurse is assigned a group of clients on the inpatient psychiatric unit. Which client presents the greatest risk for violence toward others? You answered this question Correctly 1. 24 year old man with paranoid delusions 2. 62 year old woman with bipolar disorder 3. 72 year old man with major depression 4. 28 year old woman with borderline personality disorder

1. Correct: Clients with paranoid delusions believe that others may harm them. Because they cannot determine what is accurate, they may react in a violent manner. The clients age falls within the range for males who are most likely to present a risk of violence toward others. 2. Incorrect: The client with bipolar disorder may exhibit mood changes from elated to hopeless episodes. This client may be irritable; however, it is not likely that she will present a great risk for violence. Her age does not fall within the range for women that are most likely to present a threat of violence. 3. Incorrect: This client is more likely to hurt themselves than others. The client may exhibit sadness, anxiety or exhaustion. 4. Incorrect: The client with borderline disorder may exhibit impulsive and dangerous behavior. This client is more likely to hurt herself, perhaps through self-mutilation.

The nurse instructs a client about deep breathing and coughing exercises that will be performed postoperatively. Which statement by the client indicates that teaching has been effective? 1. "Coughing and deep breathing should be performed hourly to prevent pneumonia." 2. "Coughing and deep breathing are needed to prevent blood clots." 3. "Coughing and deep breathing will aide with healing by increasing available oxygen." 4. "Coughing and deep breathing will help resolve any blood clots that have formed. "

1. Correct: Coughing and deep breathing exercises are done to expand the lungs and prevent pneumonia and atalectasis. After surgery, due to the pain, clients are prone to shallowly breath which can lead to atelectasis and thick secretions and increased risk of pneumonia. 2. Incorrect: Coughing and deep breathing exercises are done to expand the lungs and prevent pneumonia and atalectasis after surgery. Coughing and deep breathing will not prevent blood clots. 3. Incorrect: Coughing and deep breathing will increase available oxygen. The main reason client's should cough and deep breath however, is for lung expansion and pneumonia prevention. 4. Incorrect: Coughing and deep breathing exercises are done to expand the lungs and prevent pneumonia and atalectasis after surgery. Coughing and deep breathing will not resolve blood clots.

What is the priority nursing action for a pregnant client who has dilated to 6 centimeters while receiving an epidural? 1. Continuous monitoring of maternal blood pressure. 2. Frequent auscultation of the fetal heart rate. 3. Administer an IV fluid bolus of at least 500 mL. 4. Frequent monitoring of the maternal temperature.

1. Correct: Decreased blood pressure is dangerous to both the laboring mother and fetus because of the decrease in cardiac output and placental perfusion. The most common negative side effect of epidural anesthesia is a precipitous drop in blood pressure. 2. Incorrect: The fetal heart rate should be continuously monitored, but with an epidural, the first priority is maternal circulation. 3. Incorrect: Even though an IV fluid bolus may prevent hypotension, it should be administered before the epidural placement. 4. Incorrect: We are not worried about infection at this time.

The nurse, performing an initial physical assessment on a client determines that the client has difficulty hearing questions. The nurse also notices an empty eyeglass case. Based on this information, which action should first be taken by the nurse? 1. Determine which ear the client hears best from or if there is a hearing deficit is bilateral. Then ask about the empty eyeglass case. 2. Ask client about use of any assistive devices and document the client's response. 3. Look through client's belongings to determine if there is a pair of glasses and a hearing aid. 4. Notify the primary healthcare provider of client's difficulty hearing and the empty eyeglass case.

1. Correct: Determine out of which ear the client hears best or if there is a hearing deficit is bilateral. Ask if the client uses glasses and how often the glasses are used at home. Assessment is a first action many times. 2. Incorrect: This answer is very similar to option 1. Documenting the client's response is not going to intervene. 3. Incorrect: Inappropriate to look through client's belonging without asking. This is an invasion of privacy unless the client gives permission to do so. This should not be the first action either. Assessment should be first. 4. Incorrect: This is not first action to be taken by the nurse. Notifying the physician should not be the first action. The nurse should take steps to help the client directly.

The school nurse suspects that a 5 year old has been physically abused. What would be the best way for the nurse to establish trust with this child? You answered this question Correctly 1. Using play therapy. 2. Asking the mother to come to the school. 3. Hugging the child. 4. Conducting an in-depth interview with the child.

1. Correct: Establishing a trusting relationship with an abused child is extremely difficult. He or she may not even want to be touched. Play activities can provide a nonthreatening environment that may enhance the child's attempt to discuss painful issues. 2. Incorrect: The child is less likely to talk about an event in front of the parent or possible abuser. 3. Incorrect: The child may not even want to be touched, so trying to hug the child would not help to establish trust and potentially could scare the child away. 4. Incorrect: This is not developmentally appropriate for the child. This may be done with the parent but may cause defensiveness.

Which statement by the nurse would be the correct response to a client who is postmenopausal with a uterus when the client asks about temporary hormonal therapy for hot flashes? 1. "Hormonal therapy with a combination of low doses of estrogen and progestin may be prescribed." 2. "Unopposed estrogen hormonal therapy would be most appropriate." 3. "Hormonal therapy is an outdated treatment and can no longer be prescribed so you should try an alternative such as ginseng." 4. "Hormonal therapy is not an option for women with a uterus so you may need to consider a hysterectomy."

1. Correct: Estrogen and progestin are prescribed for women who have not had a hysterectomy.​ 2. Incorrect: Only women who no longer have a uterus can take estrogen without progestin.​ 3. Incorrect: Few data exists about the safety and effectiveness of alternative treatments. 4. Incorrect: Combination therapy for women with a uterus; and estrogen alone for women without a uterus.​ 1. Look at each option as True or False. 2. Option 1 is true. Progestin prevents proliferation of the uterine lining and hyperplasia as well as decreases the risk of endometrial cancers. 3. Option 2 is false. Only women with a uterus are prescribed estrogen without progestin (unopposed estrogen) because there is no longer a risk of estrogen-induced hyperplasia of the uterine lining. 4. Option 3 is false. Few data exists about the safety and effectiveness of alternative treatments. 5. Option 4 is false. Even with the associated risks of HT, some women elect to use HT in low doses as prescribed by their health care provider for short periods of time.

After reviewing the client assignments, the LPN/VN tells the RN the assignment is very unfair and requests that some of the clients be redistributed to the other staff. What should the RN do first? 1. Ask the LPN/VN how the client assignment should be adjusted. 2. Assign one of the LPN/VN's clients to another nurse. 3. Encourage the LPN/VN to use teamwork skills in caring for the clients. 4. Develop a strategic plan to assist with client assignments.

1. Correct: Explore her concerns; this is most therapeutic and helpful response. Finding out what are LPN/VN's concerns first will help the RN address the LPN/VN's request and build trust in the healthcare team relationship. 2. Incorrect: This statement does not help the RN understand the LPN/VN's concern about the assignment, an negates the confidence in the LPN/VN's abilities and skills. 3. Incorrect: This answer does not acknowledge the LPN/VN's concern. 4. Incorrect: This action will not help address the LPN/VN's immediate concern with the assignment and makes resolution of the issue much more complicated than it should be.

Which victim would the nurse decontaminate first in a biological terrorist event? 1. Client who was exposed but is exhibiting no symptoms 2. Client who has an open leg fracture and head injury 3. Client who is not breathing and has no palpable pulse 4. Client with minor cuts and abrasions

1. Correct: Exposed victims with no symptoms are first priority 2. Incorrect: Victims needing maximum medical care are third priority 3. Incorrect: Deceased victims are the last priority 4. Incorrect: Those with minor injuries are second priority 1. Look at each option as true or false. 2. Option 1 is true. The goal is to decontaminate victims who have been exposed, yet are salvageable. 3. Option 2 is false. Clients are ranked for decontamination by who is the most salvageable. Therefore those with injuries needing maximum care are decontaminated after those who have no symptoms and those with minor injuries. 4. Options 3 is false. Clients who are dead are unsalvageable and have the lowest priority for decontamination. 5. Option 4 is false. Clients are ranked for decontamination by who is the most salvageable. Therefore those with minor injuries are decontaminated after those who have no symptoms.

A primipara at 36 weeks gestation is seen in the OB/GYN clinic. Which sign/symptom should the nurse immediately report to the primary healthcare provider? 1. Puffy hands and face 2. Reports indigestion 3. Pedal edema 4. Trace proteinurea

1. Correct: Facial and upper extremity edema can be a sign of pre-eclampsia, which can endanger both the mother and fetus. Preeclampsia is a pregnancy complication characterized by high blood pressure and signs of damage to another organ system, often the kidneys. Preeclampsia usually begins after 20 weeks of pregnancy in a woman whose blood pressure had been normal. Even a slight rise in blood pressure may be a sign of preeclampsia. Left untreated, preeclampsia can lead to serious, even fatal, complications. Signs and symptoms of preeclampsia include hypertension and may include: Proteinuria; Severe headaches; Changes in vision; Upper abdominal pain; Nausea or vomiting; Decreased urine output; Thrombocytopenia; Impaired liver function; Shortness of breath; Sudden weight gain, and edema, particularly in face and hands. 2. Incorrect: Indigestion should be assessed for severity, but it is a common symptom in 3rd trimester of pregnancy. 3. Incorrect: Pedal edema should be assessed but is common in 3rd trimester of pregnancy. 4. Incorrect: Trace proteinurea is a benign sign in 3rd trimester of pregnancy.

What is the first nursing action that should be taken in caring for a client with symptoms of tuberculosis? 1. Identify the client's symptoms promptly. 2. Instruct the client to cover the mouth and nose with tissues when sneezing. 3. Isolate the client in a negative pressure room. 4. Place a surgical mask on the client.

1. Correct: First, identify the client's symptoms. 2. Incorrect: Not before proper identification of client's symptoms. 3. Incorrect: Not before proper identification of client's symptoms. 4. Incorrect: Not before proper identification of client's symptoms.

Which statement by the client with children ages 5 months to 8 years old requires follow up by the clinic nurse? 1. "I give all my children a spoonful of honey at night when they have a cough." 2. "I serve my 8 year old a glass of orange juice with breakfast before school." 3. "I have children use Lavender scented soap to wash their hands before they eat. 4. "I play music for my children when it is time for them to take a nap."

1. Correct: Giving honey to children younger than 1 year should be avoided because it is a reservoir of Clostridium botulinum spores. Clostridium botulinum produces toxins which may cause botulism. 2. Incorrect: Vitamin C stimulates the immune system and helps to reduce the risks of cardio vascular disease, cancer and eye disease. 3. Incorrect: Lavender is an aromatherapy used for relaxation and has been also been found to be useful in treating wounds and burns, and for skin care. 4. Incorrect: Music therapy has been shown to help achieve relaxation. Music should not be limited to adults; it can be used for infants and children. Identify specific determiners in options. These convey a thought or concept that has no exceptions. Words such as just, always, never, all, every, none, and only are absolute and place limits on the statement that generally is considered correct. Statements including these words generally make the statement false as the statement is general and broad and does not allow for exceptions.

Which information should the nurse plan to teach family members of a client diagnosed with hepatitis B? 1. Do not share personal items with the client, such as razors or toothbrushes. 2. Wash dishes separately from the rest of the family's. 3. Wear a surgical mask when in close proximity to the client. 4. Use a separate bathroom from the client.

1. Correct: Hepatitis B is a bloodborne pathogen that can spread via sharing personal items, such as razors or toothbrushes where infected blood can get into a person's cut, mucous membranes, etc. 2. Incorrect: Unlike some forms of hepatitis, Hepatitis B is not spread through sharing eating utensils, contaminated food or water. Hepatitis B is spread by infected blood or body fluids. 3. Incorrect: Hepatitis B is not airborne, therefore, there is no need to wear a mask. 4. Incorrect: Hepatitis B is not spread by sharing a bathroom. It is blood borne, not spread by the fecal route.

Which comment by the client indicates understanding of possible complications of long term hypertension? 1. "I would like to have my serum creatinine checked at this visit." 2. "My blurred vision is part of getting older." 3. "I have leg pain caused by excessive exercise." 4. "Adding salt to my food is permissible."

1. Correct: Hypertension is one of the leading causes of end stage renal disease. The client understands that renal function is reflected by serum creatinine levels. This request demonstrates understanding of the disease and possible complications. 2. Incorrect: The appearance of the retina provides important information about the severity and duration of hypertension. Manifestations of severe retinal damage include blurred vision, retinal hemorrhage, and loss of vision. 3. Incorrect: Intermittent claudication is a complication of peripheral vascular disease (PVD). Hypertension speeds up the process of PVD. 4. Incorrect: Lifestyle modifications include dietary sodium reduction, weight reduction, Dietary Approaches to Stop Hypertension (DASH) eating plan, moderation of alcohol consumption, regular physical activity, avoidance of tobacco use, and management of psychosocial risk factors.

The nurse is assessing a client admitted with acute gastritis. Which client information is most significant? 1. Takes ibuprofen for arthritis pain. 2. Had an upper respiratory infection two weeks ago. 3. Has a stressful job. 4. Enjoys spicy food.

1. Correct: Ibuprofen is a non-steroidal anti-inflammatory drug (NSAID). NSAIDs are highly associated with GI irritation. 2. Incorrect: Upper respiratory infections have nothing to do with gastritis. 3. Incorrect: Research does not support an association between gastritis and stressful jobs. 4. Incorrect: Spicy foods may not be tolerated by clients with gastritis, but spicy foods have not been linked to causing gastritis. This question is using the word "most". The test taker would need basic knowledge about gastritis. Option 1 states the client takes ibuprofen for arthritis pain. What do you know about ibuprofen? It is a non-steroidal anti-inflammatory drug (NSAID). NSAIDs contribute to stomach lining irritation. Thus, option 1 is most significant. NSAIDs can cause damage to the gastroduodenal mucosa via several mechanisms, including the topical irritant effect of these drugs on the epithelium, impairment of the barrier properties of the mucosa, suppression of gastric prostaglandin synthesis, reduction of gastric mucosal blood flow and interference with the repair of superficial injury. Option 2- upper respiratory infection does not contribute to acute gastritis. Option 3, a stressful job. Research does not support a connection between a stressful job and gastritis. Option 4, spicy foods, has not been linked to causing gastritis.

Which finding indicates to the nurse that a client is at risk for skin breakdown? 1. Weakness requiring assistance to move in bed. 2. Daily intake of at least 85 percent of food offered. 3. Occasional forgetfulness. 4. Continent of bowel and bladder.

1. Correct: Immobility or weakness puts a client at risk for skin breakdown, particularly if combined with other indicators such as inadequate nutrition, confusion, incontinence, or limited sensory perception. 2. Incorrect: This level of intake is considered adequate, as the client seems to be consuming protein, dairy, fruits, and/or vegetables. 3. Incorrect: Occasional confusion should not put the client at risk for skin breakdown. If the client was confused and did not keep skin dry, move about, or have adequate nutrition, the client would be at greater risk. 4. Incorrect: Moisture presence and feces on the skin from incontinence place the client at risk for skin breakdown. If the client is continent of urine and feces, there is no increased risk for skin breakdown.

The nurse is monitoring a client in diabetic ketoacidosis (DKA). Which arterial blood gas value would be expected? 1. pH 7.32 2. PaCO2 47 3. HCO3 25 4. PaO2 78

1. Correct: In DKA, the client is acidotic. Normal pH is 7.35-7.45. A pH of 7.32 indicates acidosis and will be expected for a client in DKA. 2. Incorrect: Normal PaCO2 is 35-45. Remember CO2 is considered an acid. The client in DKA will have an increased respiratory rate, so the PaCO2 will either be normal or low. This value of 47 is high and not an expected finding. 3. Incorrect: Normal HCO3 is 22-26. For a client in DKA, the expected HCO3 would be less than 22. HCO3 is a base. In acidosis, the expected finding is low HCO3​. 4. Incorrect: Normal PaO2 is 80-100. An expected finding in DKA will be normal or increased PaO2, not decreased.

Which client would be most appropriate for the emergency department charge nurse to obtain a social service consult? 1. Six year old who ingested diluted bleach. 2. Ten year old who suffered burns in a house fire. 3. Twelve year old who fractured his arm in a fight at school. 4. A 16 month old without any oral intake for the last 12 hours.

1. Correct: In most areas, laws mandate certain situations/circumstances involving children be reported to social services/child protection. Among these things are: ingestion of toxic substances, fractures, suspected neglect or abuse, burns. For older children and adults, the healthcare provider uses their judgment as to whether the situation indicates neglect or abuse by the parents or caregivers. 2. Incorrect: The child in a burned house would be reported only if the story were inconsistent as to how the house caught on fire, or if foul play is suspected. 3. Incorrect: A child fighting at school is inappropriate, but this doesn't mean there is family abuse/neglect at home. 4. Incorrect: A 16 month old who is sick may not take liquids, but the fact that the mother brought the child in means she is attentive and concerned. The nurse would determine why the 16 month old is not drinking liquids then rehydrate the child to prevent dehydration.

What room assignment by the charge nurse is most appropriate for a client who is being admitted with poor appetite, malaise, and temperature of 101.5ºF (38.6ºC)? 1. Private room. 2. Room with a client who has biliary colic. 3. Room with a client who is 3 days post operative hip replacement. 4. Room with a client who is in skeletal traction due to broken femur.

1. Correct: In this particular situation, a private room is best due to the elevated temperature. This could mean the client has an infection and is contagious. All of the often clients do not need to be exposed to this client with fever of unknown cause. 2. Incorrect: Does not need to be exposed to infection. Biliary colic is pain due to a gallstone blocking the bile duct. The client may need surgery and definitely should not be exposed to infection. 3. Incorrect: Post op client already at risk for infection. This is not the most appropriate client to room with the new admit. 4. Incorrect: Does not need to be exposed to infection. The client is already at risk for infection due to the skeletal traction. Complications of skeletal traction include risk for bone infection due to a screw being placed in a bone.

A client newly diagnosed with insulin dependent diabetes mellitus is started on insulin aspart protamine suspension/insulin aspart solution mixture. The nurse would teach the client that the insulin should start to lower the blood sugar within how many minutes? 1. 15 2. 30 3. 45 4. 90

1. Correct: Insulin aspart mixture is a rapid-acting insulin and starts to work within 15 minutes after given subcutaneously. 2. Incorrect: Regular insulin has an onset of 30 minutes to 1 hour. Aspart is a rapid-acting insulin, and begins to work within 15 mnutes. 3. Incorrect: Long acting insulin has an onset of 45-48 minutes. An example of long acting insulin would be lantus. 4. Incorrect: Intermediate acting insulin such as NPH insulin has an onset of 90 minutes.

A client with sleep apnea has been ordered a Continuous Positive Airway Pressure (CPAP) machine. Which action could the RN delegate to an unlicensed assistive personnel (UAP)? 1. Reminding the client to apply the CPAP at bedtime 2. Obtaining oxygen saturation levels every three hours 3. Teaching the client how to turn on the CPAP machine 4. Assessing for fatigue or depression caused by poor sleep

1. Correct: It is appropriate delegation for a UAP to remind the client to do a previously taught intervention. The UAP cannot perform actual teaching because this is outside the scope of practice, but reminding the client about what was taught may help with compliance. 2. Incorrect: This is an assessment function and may be outside the UAP's scope of practice in some states. Since oxygen saturation requires every three hour monitoring, it is best not to assign this to the UAP. The nurse should be the one to check the oxygen saturation levels every three hours because additional assessment of the client status may be warranted. 3. Incorrect: Initial teaching about the CPAP machine is the responsibility of the RN. The LPN can reinforce this teaching, but teaching is outside the UAP's scope of practice. 4. Incorrect: Assessment is outside the UAP's scope of practice. Independent assessment requires additional education and skills and should be carried out by the RN.

A teenage client asks the nurse, "Do you think I should tell my parents about my sexuality?" What is the nurse's best response? 1. "What do you think you should do?" 2. "Absolutely, I think you should tell your parents." 3. "Don't you think your parents have the right to know about your sexuality?" 4. "I do not think now is the right time to tell your parents. Wait until you are 21."

1. Correct: It is better to say "What do you think you should do?" This helps the client reflect on options and does not have the nurse tell the client what to do. It is much more therapeutic to help the client make the decision for themselves, instead of the nurse. This prevents any biases from impacting the outcome. 2. Incorrect: All of these responses give advice to the client. Telling the client what to do or how to behave which implies that the nurse knows what is best and that the client is not capable of making any decisions. 3. Incorrect: All of these responses give advice to the client. Telling the client what to do or how to behave which implies that the nurse knows what is best and that the client is not capable of making any decisions. 4. Incorrect: All of these responses give advice to the client. Telling the client what to do or how to behave which implies that the nurse knows what is best and that the client is not capable of making any decisions.

The nurse is caring for a client that is undergoing an induction for fetal demise at 34 weeks. Immediately after delivery the mother asks to see the infant. What is the nurse's best response? 1. Bring the swaddled baby to the mother. 2. Explain that the cause of death must be determined before she can see the baby. 3. Ask her if she is sure she wants to see the baby. 4. Tell her it would be better to wait until she is in her room before she sees the baby.

1. Correct: Let the grieving mother see the infant to continue the grieving process. The mother has the right to make her own decision. 2. Incorrect: This is an untrue statement. In some cases, the cause may never be found. 3. Incorrect: This is non-therapeutic and implies that the nurse disagrees with the mother's decision to see the infant. 4. Incorrect: This is non-therapeutic and delays the mother's request. This response may also cause additional fear and anxiety.

In which client should the nurse question the prescribed medication levofloxacin? 1. History of myasthenia gravis. 2. Has a prescription for verapamil. 3. Thrombocytopenic 4. Admitted with renal arterial stenosis.

1. Correct: Levofloxacin is contraindicated in clients with a history of myasthenia gravis because it may cause the condition to become worse. Myasthenia gravis results in a breakdown in the communication between muscles and nerves and is characterized by muscle weakness. The most commonly affected muscles are those of the eye, face, throat, neck and limbs. 2. Incorrect: Levofloxacin and verapamil are not known to be incompatible. Levofloxacin is a quinolone antibiotic and there are no contraindications for use with verapamil, a calcium channel blocker. 3. Incorrect: You would worry about thrombocytopenia in clients taking anticoagulants. Thrombocytopenia is a decrease in platelets in the blood. 4. Incorrect: ACE inhibitors are contraindicated with renal arterial stenosis. There are no contraindications of using levofloxacin with renal arteral stenosis.

A female client with a history of frequent exacerbations of asthma asks the nurse to explain to her why she is at greater risk for fractures than other women her age. What is the nurse's best response? 1. "The steroids you are taking decrease calcium in the bone by sending it to the blood." 2. "Taking steroids causes bone calcium to increase, thus causing osteoporosis." 3. "Clients who have asthma are not able to exercise enough to prevent fractures from occurring." 4. "Asthma should not put you at increased risk for fractures but you are at risk for decreased blood glucose levels."

1. Correct: Long term use of steroids decreases serum calcium, so the body takes calcium from the bone and puts it in the blood in order to bring the serum calcium back to a normal level. Every time a steroid is given, calcium is removed from the bone, thus leading to a greater risk for osteoporosis and fractures. 2. Incorrect: Osteoporosis is a decrease in bone calcium not an increase. 3. Incorrect: There are many types of exercise that asthma clients may participate in, including walking at short intervals. 4. Incorrect: Drug therapy for asthma (not asthma itself) may put a client at risk for osteoporosis, but not hypoglycemia.

A client is admitted to the hospital with acute exacerbation of COPD following an upper respiratory infection. His daughter found him at home, confused and in respiratory distress, a day after he developed a cold. He was placed on 4 L/min of oxygen via nasal cannula, but oxygen saturation remains at 89%. Based on this assessment, the nurse suspects that the client has developed which acid base imbalance? 1. Respiratory acidosis 2. Respiratory alkalosis 3. Metabolic acidosis 4. Metabolic alkalosis

1. Correct: Look at all the hints in this stem: COPD, upper respiratory infection, respiratory distress, confused, oxygen saturation of 89%. This client is having lung problems. So you should be able to identify the acid base imbalance as a respiratory problem, right? Yes. Why is it acidosis? Poor gas exchange! Respiratory failure, COPD, and muscular weakness can lead to respiratory acidosis. So you would expect the pH to be < 7.35, and the pCO2 to be > 45. The HCO3 would be normal. 2. Incorrect: Not alkalosis. You would expect respiratory alkalosis with a client who is hyperventilating, such as the hysterical client. The client in this question would be hypoventilating and having poor gas exchange. 3. Incorrect: Not a metabolic related acid/base imbalance. Metabolic problems do not start with a respiratory problem. Metabolic acidosis is seen with diabetic ketoacidosis or starvation. 4. Incorrect: This is not a metabolic problem but a respiratory problem. Metabolic alkalosis may be seen with prolonged vomiting and hypokalemia.

A client with an acute exacerbation of systemic lupus erythematosus (SLE) is hospitalized with incapacitating fatigue and fever. A urinalysis reveals proteinuria and hematuria. The primary healthcare provider prescribes corticosteroids. During the acute phase of the client's illness, what is most important for the nurse to do? 1. Monitor intake and output and daily weight. 2. Allow for frequent, uninterrupted rest periods. 3. Institute seizure precautions. 4. Protect client from injury that may cause bleeding.

1. Correct: Look at the clues in the stem. Proteinuria and hematuria. When you see proteinuria what do you need to worry about? The kidneys! Protein is a great big molecule. The only way for protein to be seen in the urine is if there are holes in the glomerulus. So the kidneys are being damaged. Thus, the nurse knows that the biggest problem to "worry" about here is renal failure. The best methods for monitoring fluid status and renal status for a client are to monitor I and O and daily weights. (Also, remember that one weight doesn't mean anything. The hematuria indicates that there has already been glomerular damage.) 2. Incorrect: Systemic lupus erythematosus (SLE) is an autoimmune disease. In this disease, the body's immune system mistakenly attacks healthy tissue. It can affect the skin, joints, kidneys, brain, and other organs. Fatigue is a major symptom so allowing for frequent, uninterrupted rest periods is important for this client but monitoring for renal failure is more acute. 3. Incorrect: Seizures are a potential problem with SLE, but the ACTUAL problem depicted in the stem of the question, renal failure, takes priority. Look for the option that relates to the renal system. 4. Incorrect: Hemolytic problems can occur with SLE, but this is not the ACTUAL problem depicted in the stem of the question. The stem is indicating a renal problem, so look for a renal answer.

A nurse is planning to conduct parenting classes for first time parents in an attempt to decrease child abuse in the community. What type of prevention is the nurse utilizing? 1. Primary prevention 2. Secondary prevention 3. Tertiary prevention 4. Case management

1. Correct: Primary prevention is aimed at reducing the incidence of mental disorders within the population. Primary prevention targets individuals and the environment. Emphasis: assisting individuals to increase their ability to cope effectively with stress and targeting and diminishing harmful forces (stressors) within the environment. Teaching parenting skills and child development to prospective new parents is primary prevention. 2. Incorrect: Secondary prevention services are aimed at reducing the prevalence of psychiatric illness by shortening the course or duration of the illness. 3. Incorrect: Tertiary prevention services are aimed at reducing the residual effects of severe or chronic mental illness. 4. Incorrect: Case management occurs at the secondary level of prevention and is a way to organize client care so that specific outcomes are achieved within an allotted time frame.

A client with type II diabetes reports normal blood glucose levels at bedtime and high blood glucose levels in the morning for the past week. What instruction would the nurse give the client? 1. Monitor blood sugar around 2am. 2. Decrease bedtime snacking. 3. Decrease intermediate acting insulin. 4. Increase intermediate acting insulin.

1. Correct: Morning hyperglycemia may be the result of dawn's phenomenon or the Somogyi effect. The client must take their blood sugar between two and three o'clock in the morning for several days to determine the cause of morning hyperglycemia. If the client has decreased blood sugar between two and three o'clock in the morning, suspect Somogyi effect. 2. Incorrect: This is an intervention; assessment should come first. The nurse must determine the cause of morning hyperglycemia in order to treat the condition appropriately. 3. Incorrect: This is an intervention; assessment should come first. The nurse must determine the cause of hyperglycemia in order to treat the condition appropriately. An appropriate intervention for a client with Somogyi effect would be to decrease the evening dose of intermediate acting insulin, however, the nurse must first determine that the client is in fact experiencing the Somogyi effect. 4. Incorrect: This is an intervention; assessment should come first. Increasing the intermediate acting insulin would not be appropriate action for a client experiencing Somogyi effect.

A 15 year old is being admitted with pelvic inflammatory disease. Which client could the charge nurse assign the new admit to room with? 1. 18 year old who sustained a compound fracture. 2. 15 year old diagnosed with anorexia nervosa. 3. 13 year old admitted with pneumonia. 4. 14 year old who is taking steroids for chronic asthma.

1. Correct: The best choice would be the client with a fracture who is also an adolescent. Neither of these clients require visitor limiting or potential to transmit infections. 2. Incorrect: Usually adolescents with anorexia nervosa losing weight are put on a behavior modification program, and visitors are limited. Therefore, it would probably be best if this client did not have a roommate. 3. Incorrect: Pneumonia could be contagious and should not have a roommate. This client could be on contact or droplet precautions. 4. Incorrect: Long term steroid therapy could make this client immunosuppressed. Visitors should be limited and a private room would be recommended.

Which action by a nurse requires intervention by the charge nurse? 1. The two-handed method is used to recap a needle. 2. A needleless system is used to give medication through an intravenous (IV). 3. A blunt cannula is used to withdraw medication from a vial. 4. An engineered sharp injury protective device is used to recap a used needle.

1. Correct: Needles should be recapped using a one hand scoop method to prevent accidental sticks. Two-handed method increases the risk that the nurse's non-dominant hand will be punctured with the needle. Think about it. You do not want the hand holding the cap to get close to the needle. What if you miss the needle and stick your hand. The best solution is to not recap at all. Place the needle in the sharps container at once. But if the sharps container is not close by then the one hand scoop method is appropriate. You are not exposing one hand to the needle. 2. Incorrect: This is a correct method to use. To prevent injury during injection administration or body fluid retrieval, use a one-handed scoop method, needleless system, blunt cannula for medication withdrawal from a vial, or an engineered sharp injury protective device whenever possible. 3. Incorrect: This is a correct method to use. To prevent injury during injection administration or body fluid retrieval, use a one-handed scoop method, needleless system, blunt cannula for medication withdrawal from a vial, or an engineered sharp injury protective device whenever possible. 4. Incorrect: This is a correct method to use. To prevent injury during injection administration or body fluid retrieval, use a one-handed scoop method, needleless system, blunt cannula for medication withdrawal from a vial, or an engineered sharp injury protective device whenever possible.

After completing several rounds of chemotherapy, a client's laboratory results indicate severe neutropenia. Following admission assessment, what is the nurse's priority action for this client? 1. Notify dietary no fresh, unpeeled fruits or vegetables. 2. Avoid all venipunctures or IM injections. 3. Have client wear mask when leaving room. 4. Instruct client to use a soft toothbrush.

1. Correct: Neutropenia is an abnormally low white blood cell count caused, in this case, by the recent chemotherapy. The greatest concern is the client's inability to fight off infection. Fresh fruits and vegetables have a high bacterial count and present an increased risk for infection. Asking dietary to remove fresh fruits and vegetables from meal trays is an important priority action by the nurse. 2. Incorrect: Avoiding venipunctures of any type, including IM injections, is an important precaution for neutropenia, in which infection is the main concern. However, the word "ALL" makes this statement to definite. The client may need an IV. Remember, nothing is that definite in the world. 3. Incorrect: This immunocompromised client is at risk for infection, as indicated by a low neutrophil count. While airborne bacteria may be a concern at some point, there is another action by the nurse which takes priority. 4. Incorrect: A soft toothbrush is used as part of the precautions for clients at risk for bleeding, which would not apply to this client. The nurse here is concerned about infection control secondary to a low neutrophil count. Option 1: Excellent choice! You realized that fresh fruits and vegetables have a high bacterial count, no matter how thoroughly they are cleaned. Instructing dietary not to place any fresh fruits on the tray is also much safer than removing the fruit once it has touched the tray surface. It is better to serve the client canned fruits or cooked vegetables. Option 2: Not a necessary action. Neutropenia indicates the client is at increased risk of infection, not bleeding. Avoiding venipunctures or intramuscular injections is a precaution taken with clients at increased risk of hemorrhaging. Option 3: Did you pause on this one? You may have remembered that neutropenia means increased risk for infection, and that includes airborne bacteria. Immunocompromised clients rarely leave the assigned room; however, if it is necessary to go to another department for tests or treatments, having the client wear a mask might be considered. But it is not the priority action for the nurse following admission. Option 4: What does neutropenia mean for this client? It indicates the nurse must be concerned about increased risk of infection and needs to take an appropriate action to reduce that risk. Although it is true that brushing with stiff bristles may irritate gums, having the client use a soft toothbrush is a precaution used for those at risk for bleeding.

Which ethical principle is involved when a nurse reports a medication error to the primary healthcare provider? 1. Nonmaleficence 2. Beneficence 3. Justice 4. Fidelity

1. Correct: Nonmaleficence is best illustrated with the nurse's action, as the goal is to do no harm to the client. With timely reporting of an error, further complications may be prevented. 2. Incorrect: Beneficence refers to doing good. This may include compassion and kindness. 3. Incorrect: Justice refers to equitable distribution of resources. Triage in the ED is one action that illustrates justice. 4. Incorrect: Fidelity refers to truth-telling. If the client were to ask if a medication error was made, the nurse would answer yes to the question as a way of demonstrating fidelity.

The nurse in the emergency department is caring for a client admitted in diabetic ketoacidosis (DKA). Which central venous pressure (CVP) reading would the nurse anticipate? 1. 1 mm of Hg 2. 3 mm of Hg 3. 6 mm of Hg 4. 12 mm of Hg

1. Correct: Normal CVP is 2-6 mmHg. This is a CVP reading that would indicate fluid volume deficit. A client in DKA will have polyuria. A Hurst strategy is "with polyuria, think shock first". Less volume, less pressure! 2. Incorrect: This is a normal CVP reading. Normal CVP is 2 to 6 mm of Hg. 3. Incorrect: This is a normal CVP reading. 4. Incorrect: This CVP reading indicates fluid volume overload. The client in DKA will not be experiencing fluid volume excess.

A client has arrived in the emergency department with partial thickness burns to 52 percent of the body. Which central venous pressure (CVP) reading would the nurse anticipate? 1. 1 mm of Hg 2. 2 mm of Hg 3. 6 mm of Hg 4. 10 mm of Hg

1. Correct: Normal CVP is 2-6mmHg. This CVP reading indicates fluid volume deficit. A client with 52 percent of the body burned with partial thickness burns would lose fluid from the vascular space out into the tissues resulting in fluid volume deficit. 2. Incorrect: This is a normal CVP reading. Normal CVP is 2 to 6 mm of Hg. 3. Incorrect: This is a normal CVP reading. Normal CVP is 2 to 6 mm of Hg. 4. Incorrect: An increased CVP reading indicates fluid volume excess. There is no indication in the stem that the client is experiencing a fluid volume excess.

The nurse is providing prenatal education for a couple expecting a first child. The expectant mother asks about fetal movements. What is the best explanation by the nurse? 1. "You should feel activity between weeks 16 to 20." 2. "The fetus is too small to feel any movements." 3. "Maybe around the end of the 1st trimester." 4. "It is different for each individual woman."

1. Correct: Quickening is the term used to refer to fetal movement when first sensed by the expectant mother. It is challenging for first time mothers to differentiate between actual fetal movement and other sensations, but usually between weeks 16 and 20, actual kicks or changes in fetal body position are noted by the mother. 2. Incorrect: As a first time mother, the client is requesting information about fetal movement. This response by the nurse does not provide the healthcare facts needed by the mother. While this is statement is accurate, the nurse has not addressed the client's question. 3. Incorrect: Not only is this response vague, it is also incorrect. The end of the first trimester is too soon, even though there is in fact movement, because the fetus is too small. 4. Incorrect: Though this is an accurate statement, the nurse has not specifically addressed the client's question. Expectant parents have many questions, and the mother in particular is anxious about changes or sensations within her body. The nurse needs to provide specific and detailed information when teaching clients.

While making rounds, the nurse discovers a small fire in a client's room. What should the nurse do first? . Remove the client from the room immediately. 2. Leave the client's room to obtain a fire extinguisher. 3. Instruct the unlicensed assistive personnel (UAP) to pull the fire alarm. 4. Evacuate all clients from the unit.

1. Correct: Rescue/Remove the client; first step in Rescue, Alarm, Contain, Extinguish (RACE). 2. Incorrect: Never leave the client in an unsafe environment. Remove the client from the area. 3. Incorrect: Not first action in RACE. Get the client out of the area first. The UAP may need to help you with this. Don't send the UAP away. 4. Incorrect: Not first action in RACE.

A client has recently been diagnosed with systemic scleroderma. Which of the following client complaints would be of most concern to the homecare nurse? 1. "I feel like food gets stuck in my throat when I eat." 2. "I have a hard time brushing my teeth properly." 3. "My fingers burn when I go outside in the winter." 4. "I get short of breath whenever I exercise."

1. Correct: Scleroderma is an autoimmune disorder characterized by the excess production of collagen and hardening of tissues. In systemic scleroderma, body organs lose the ability to function as the disease progresses. When parts of the digestive system build up collagen, clients experience frequent acid reflex, constipation, and difficulty swallowing. The nurse would be most concerned about aspiration during or after meals. 2. Incorrect: Because facial skin tightens, clients have difficulty opening the mouth completely. It becomes challenging to properly brush teeth or perform personal mouth care, increasing the likelihood of tooth decay. However, this would not be the biggest concern to the nurse at this time. 3. Incorrect: It is common for clients with one autoimmune disorder to develop other disorders. These symptoms indicate Raynaud's phenomenon, which is often reported by scleroderma clients. Advance of the disease triggers skin on the hands to become tight, stiff, and slightly shiny. The client begins to experience severe pain when fingers are exposed to the cold. Fingertips start out white in color, progressively turning red until re-warmed. Impaired circulation and pain are certainly an area of concern but not the most immediate worry to the nurse at this time. 4. Incorrect: Clients with scleroderma develop scarring ("fibrosis") of lung tissue, decreasing respiratory capabilities and eventually leading to pulmonary hypertension. This client reports shortness of breath just during exercise, indicating that simple daily activities are still achievable. It is obvious the disease has not yet progressed enough to impact ADL's, and therefore this is not the most concerning complaint at this time.

A client experiencing a manic episode tells the night nurse, "If you do not go to bed with me, I am going to have you fired." What is the nurse's best response? 1. "That is inappropriate behavior. You will have to go to your room if you say that again." 2. "You've got to be kidding! You can't get me fired for not sleeping with you." 3. "I don't want to hear that again! Don't ever say that again." 4. "I can see that you need attention, but this is not the way to get it."

1. Correct: Set limits on manipulative behaviors. Explain what is expected and what the consequences are if limits are violated. The nurse needs to set limits on and control dangerous behavior. 2. Incorrect: Do not argue with the client. The behavior of a manic client is often aimed at decreasing the effectiveness of staff control. 3. Incorrect: This is confrontational and does not set appropriate boundaries or consequences. The manic client can elicit numerous intense emotions even in the nurse caring for them. 4. Incorrect: Remember to set limits without demeaning the client, and do not encourage this behavior. Don't acknowledge that the client is seeking attention.

The son of an elderly diabetic client reports that his mother is frequently having low blood sugar. What should the nurse teach this family member about symptoms of hypoglycemia in the elderly? 1. Elders may not be aware that blood sugar is dropping due to decreased release of epinephrine in response to the lowered blood sugar. 2. Suggest that the client and family check with primary healthcare provider to ensure that the medication prescribed has low incidence of hypoglycemic episodes. 3. Symptoms of hypoglycemia may be averted if the client maintains routines and regular meal schedules. 4. Stress the importance of proper foot care and regular eye exams. 5. Check blood glucose levels if client becomes unsteady, has difficulty concentrating, or is tremulous.

1., 2., 3. & 5. Correct: Older clients are at risk for hypoglycemia unawareness. Blood sugar levels should be checked frequently. Some oral medications are more likely to cause hypoglycemia episodes. If the client has frequent episodes, perhaps a medication change is warranted. The elderly must maintain regular meal schedules and adequate food intake. This may present challenges for the elder who lives alone. If an elder develops unsteady gait, loss of concentration, and/or lightheadedness, the blood glucose levels should be checked. These symptoms are typical in a hypoglycemic episode. 4. Incorrect: Proper foot care and regular eye exams should be done to avoid complications caused by hyperglycemia, not hypoglycemia. All this question is saying is: Hey new nurse do you know about hypoglycemia in the elderly? So focus on answering that question as we look at the options provided. Option 1: Elders may not be aware that blood sugar is dropping due to decreased release of epinephrine in response to the lowered blood sugar. Is this a true or false statement. It is true. Aging modifies the cognitive, symptomatic, and counter-regulatory hormonal responses to hypoglycemia. Although hypoglycemia in the elderly is the most common complication of tight glycemic control, multiple co-morbidities like renal impairment, chronic heart disease, malnutrition and polypharmacy may increase risk of this complication. Option 2: Suggest that the client and family check with primary healthcare provider to ensure that the medication prescribed has low incidence of hypoglycemic episodes. ​What do you think? Did you say true? Good. Some oral medications are more likely to cause hypoglycemia episodes. If the client has frequent episodes, perhaps a medication change is warranted. Option 3: Symptoms of hypoglycemia may be averted if the client maintains routines and regular meal schedules. ​This one is true. The elderly must maintain regular meal schedules and adequate food intake. This may present challenges for the elder who lives alone. Remember, routine is very important in the diabetic client to maintain a stable blood glucose level. Option 4: Stress the importance of proper foot care and regular eye exams.​ This option may have challenged you because we do want clients with diabetes to properly care for their feet and eys, however, does this statement address the issue of hypoglycemia presented in the question? No, so this is false. Option 5: Check blood glucose levels if client becomes unsteady, has difficulty concentrating, or is tremulous. True. If an elder develops unsteady gait, loss of concentration, and/or lightheadedness, the blood glucose levels should be checked. These symptoms are typical in a hypoglycemic episode.

On morning rounds, the nurse finds a somnolent client with a blood glucose of 89 mg/dL(4.9 mmol/L). A sulfonylurea and a proton pump inhibitor are scheduled to be administered. What is the nurse's best action? 1. Give the proton pump inhibitor and hold the sulfonylurea until the client eats. 2. Hold the medications and notify the primary healthcare provider. 3. Arouse the client and give some orange juice with sugar packets added. 4. Give the medications as prescribed and re-check the blood sugar in one hour.

1. Correct: Sulfonylureas are a class of oral hypoglycemics and should be held until after a meal in a client with a blood glucose of 89mg/dl. 2. Incorrect: It is not necessary to call the primary healthcare provider; you are just waiting until the client eats. Also, the proton pump inhibitor does not affect blood glucose levels and should be administered. 3. Incorrect: A blood glucose of 89mg/dl is not hypoglcemia, do not treat unless the blood glucose drops to or below the 70-80 range. 4. Incorrect: If you administer the sulfonylurea, you are going to cause the client to secrete insulin from their pancreas, causing the blood sugar to drop and cause hypoglycemia.

A client who is 20 weeks pregnant and diagnosed with pelvic inflammatory disease is given a prescription for metronidazole. What should the nurse inform the client to avoid in order to prevent an interaction with metronidazole? 1. Furosemide 2. Alcohol 3. Doxycycline 4. St. John's Wort

2. Correct: Metronidazole is an antibiotic used for the treatment of vaginal infections. Metronidazole and alcohol can interact with each other, causing severe nausea and vomiting as well as cramping and flushed appearance. 1. Incorrect: Furosemide is a diuretic and does not interact with metronidazole. 3. Incorrect: Doxycycline is a tetracycline antibiotic and does not interact with metroindazole. 4. Incorrect: St. John's wort is an herbal supplement and does not interact with metronidazole.

What is the priority nursing action for a client that was admitted with tingling of the toes and feet after having the flu for several days when the client begins to have numbness in the legs and hips? 1. Notify the primary healthcare provider 2. Monitor for paresthesia in the fingers and hands 3. Insert an indwelling urinary catheter 4. Assist the client with performing passive range of motion

1. Correct: Symptoms are classic for Guillain-Barre. The possibility of rapid progression and respiratory failure make this a medical emergency. The nurse's priority action is to notify the healthcare provider. 2. Incorrect: The nurse should continue to monitor for paresthesia in the upper body and arms. The first priority in this situation is to notify the primary healthcare provider of the potential life threatening situations. 3. Incorrect: Urinary retention is a possible complication with Guillain-Barre, and the client may require an indwelling urinary catheter, but the immediate priority is to notify the primary healthcare provider. 4. Incorrect: Passive range of motion is performed to prevent complications of immobility, but this is not the priority at this time. The client is presently able to move their extremities. Passive range of motion is not the priority at this time. Option 1 is true. A client that has numbness and tingling in the lower extremities that advances upwards, especially after having a viral infection, has clinical manifestations characteristic of Guillian-Barre Syndrome. The primary healthcare provider must be immediately notified of the change because the disease is progressively paralytic and must be treated before paralysis of the respirator muscles occurs.

What should the nurse teach the mother about appropriate sleep in teenagers? 1. Teens need about 8 to 10 hours of sleep each night. 2. Biological sleep patterns shift toward earlier wakening. 3. Typically do not require as much sleep as adults. 4. Teenagers do not exhibit the normal signs of sleep deprivation.

1. Correct: Teens need approximately 8 to 10 hours of sleep per night. 2. Incorrect: Teens tend to go to bed later and wake up later. A natural shift in a teens circadian rhythm is called "sleep phase delay". The need for sleep is delayed about two hours. They naturally get sleepy later in the evening. 3. Incorrect: Teens need adequate sleep. The adolescent period is a time of biological, psychological, and social change. Sleep is necessary to support this important stage of growth and development. The sleep deprivation can affect brain and bodily development. 4. Incorrect: When teens are deprived of sleep, they become irritable, fall asleep in class, or experience anxiety. Teens may display even more signs of sleep deprivation than adults. Depression, issues with learning and behavior, substance use or abuse and obesity can be long term effects on a teenager's health.

The nurse assesses bruises on a child's face, the hands, and the feet. When questioned, the parents state their child is so clumsy. What action by the nurse demonstrates client advocacy? 1. The nurse reports the incident to the Child Protective Services. 2. The nurse notifies the parent's clergy. 3. The nurse reports the assessment to the primary healthcare provider. 4. The nurse speaks to the parents privately about any concerns.

1. Correct: The action is appropriate. The nurse is serving as an advocate for the child who cannot advocate for self. Early identification of possible child treatment by the nurse is crucial. A pattern or combination of indicators should arouse suspicion and cells for further investigation. Incompatibility between the reported cause the injuries noted is the most important criterion to base decision to report suspected abuse. All states as well as the provinces in North America have laws that require child maltreatment or suspected child abuse to be reported. Therefore, the nurse has an obligation to report any suspicions of child maltreatment. An advocate pleads the cause of another person. In this case, the nurse pleads the cause of the child to help protect the child's human rights. 2. Incorrect: The nurse should not contact the parent's clergy without their permission. This would be a violation of HIPAA. 3. Incorrect: The nurse is acting in the caregiver role. Although client finding will be documented and reported to the primary healthcare provider, client advocacy would involve proper reporting to the appropriate child protection agency and authorities. 4. Incorrect: If abuse is occurring, the parents will usually deny it. If a child is old enough to talk, the history of the injuries may be reported if the child id separated from the parents for this discussion.

A client in a psychiatric unit tells the nurse, "I wanted to take the car to work, but the train station took all the tracks. Driving is the ticket when you want to go to the movies. No one needs money in heaven. We have money in our foods." How should the nurse document this conversation? 1. Associative looseness 2. Circumstantiality 3. Echopraxia 4. Anhedonia

1. Correct: Thinking is characterized by speech in which ideas shift from one unrelated subject to another in an unrelated manner. The person is not aware that the topics are unconnected. Speech may be incoherent at times. 2. Incorrect: With circumstantiality, the person is delayed in reaching the point of a communication because of unnecessary and tedious details. The point or goal is usually met, but only with numerous interruptions by the interviewer to keep the person on track. The person gets caught up in countless details and explanations. 3. Incorrect: The client who exhibits echopraxia may imitate or mimic the movements made by others. 4. Incorrect: Anhedonia is the inability to experience pleasure in acts that are normally pleasurable.

The nurse is caring for a postoperative client. The client asks the nurse the purpose of anti-embolic stockings. What is the nurse's best response? 1. Promotes the return of venous blood to the heart and assists in preventing blood clots. 2. Stabilizes any clots to prevent embolization. 3. To increase the blood pressure in the venous system in the legs to promote perfusion. 4. Promotes lymphatic drainage to prevent swelling and arterial congestion.

1. Correct: The anti-embolic stockings promote return of venous blood to the heart and assist in preventing the stasis of blood that can lead to blood clots. 2. Incorrect: The purpose of the anti-embolism stockings is to promote venous return and prevent blood stasis which can result in blood clot formation. Anti-embolitic stockings will not stabilize existing blood clots. 3. Incorrect: Anti-embolism stockings are used to increase venous return. They are not used to increase blood pressure or perfusion to the legs. 4. Incorrect: Compression garments, not anti-embolitic stockings, are used by persons with lymphedema to reduce edema by promoting the flow of lymph fluid out of the affected limb. Anti-embolitic stockings are to help with venous return and preventing stasis of blood and blood clots.

The home health nurse is concerned about the safety of the client who lives alone in a poorly maintained home. The nurse convenes the interdisciplinary team to discuss the situation. Which action should occur first? 1. Share the assessment findings with the interdisciplinary team. 2. Suggest that the social worker visit the client in the home. 3. Ask the primary healthcare provider about possible nursing home placement. 4. Suggest a "meals on wheels" solution to nutrition.

1. Correct: The assessment findings from the home health nurse will allow each person of the team to offer input based on their particular expertise. After assessment findings have been discussed, the problem solving approach can begin. The interdisciplinary team works together and shares their expertise, knowledge and skills to improve client care. 2. Incorrect: Suggesting a social worker visit may be appropriate; however, this situation would best be served by a discussion with the entire team first. 3. Incorrect: Nursing home placement may be appropriate; however, this is not the first step in collaboration with the team. The team will discuss the home health nurse's concerns and problem solve to provide solutions. 4. Incorrect: Nutrition is a pertinent issue that may need to be addressed; however, the entire team's input is needed at this point. Also the nurse's concern in the safety of the client in a poorly maintained home.

The nurse is caring for a burn client in the emergent phase. The client becomes extremely restless while on a ventilator. What is the priority nursing assessment? 1. Patency of endotracheal tube. 2. Adventitious breath sounds. 3. Fluid in the ventilator tubing. 4. Ventilator settings.

1. Correct: With restlessness, think hypoxia so the nurse should start assessment with airway first. Check for patency of the ET tube. If this is patent, then the other options would be next. 2. Incorrect: This is the next best answer, but hypoxia and airway comes first. 3. Incorrect: This is the third step. Rule out the other two before checking tubing for kinks or obstructions. 4. Incorrect: Start with the client first. Then move toward the ventilator. Always assess the client first.

The charge nurse walks into the client's room as the staff nurse is preparing the client for discharge. The charge nurse overhears the staff nurse giving the client her phone number. The staff nurse says, "Call me when you get home, and maybe we can get together sometime." What should the charge nurse do first? 1. Interrupt the staff nurse and complete the discharge. 2. Tell the staff nurse in the client's presence that the action is inappropriate. 3. Make no comment, and let the staff nurse continue to talk with the client. 4. Stay with the client until ready to leave the unit.

1. Correct: The charge nurse should make sure that professional boundaries are maintained; therefore, the charge nurse should interrupt the process and continue with the discharge procedure. Then the nurse should be counseled immediately so that further inappropriate behavior does not occur. 2. Incorrect: The nurse should be counseled; however, counseling does not need to be done in front of the client. The better option is to counsel the staff nurse in private. 3. Incorrect: The charge nurse must make sure that professional boundaries are maintained. To make no comment indicates acceptance of the behavior. 4. Incorrect: No, the charge nurse interrupts the staff nurse and completes the discharge then counsels the staff nurse on professional boundaries.

A client is admitted to the surgical unit with cholelithiasis and a history of psychosis and a known allergy to phenothiazines. Which prescription should the nurse discuss with the primary healthcare provider? *Exhibit (prescriptions & allergies)* *prescriptions* Clear liquid diet Gallbladder ultrasound today IV of LR with KCL 20 mEq at 125 ml/hr Thioridazine 50 mg PO TID ​Ciprofloxicin 200 mg IVPB every 12 hours Haloperidol 5 mg by mouth twice daily Ondansetron 4 mg IM as needed for nausea or vomiting *allergies* Phenothiazines Penicillin 1. Thioridazine 50 mg PO tid 2. Ciprofloxicin 200 mg IVPB every 12 hours 3. Haloperidol 5 mg PO bid 4. Ondansetron 4 mg IM prn nausea or vomiting

1. Correct: The client is allergic to phenothiazines. Thioridazine is a phenothiazine and should not be given to this client. 2. Incorrect: Ciprofloxicin is an antibiotic but is not a penicillin drug; therefore, it can be administered to this client. 3. Incorrect: Haloperidol is an antipsychotic medication. The classification is butyrophenone, not a phenothiazine. 4. Incorrect: Ondansetron is an antiemetic and is an appropriate drug for this client.

A client with a history of peptic ulcer disease arrives at the emergency department reporting weakness, and vomiting "a lot of dark coffee-looking stomach contents." The client's skin is cool and moist to the touch. BP 90/50, HR 110, RR 26, T 98, O2 sat 88%. Which primary healthcare provider prescription should the nurse perform first? 1. Initiate oxygen at 2 liters/nasal cannula. 2. Start an IV of NS at 150 ml/hr. 3. Insert nasogastric (NG) tube to low suction. 4. Attach client to the electrocardiography (ECG) monitor.

1. Correct: The client is showing signs of shock and needs all of the above interventions. However, go back to the ABC's. Oxygen needs to be initiated first because the O2 sat and the increased respiratory rate. 2. Incorrect: Fluids are needed to increase blood pressure and tissue perfusion. If O2 sats were above 90 then this would be the first priority. 3. Incorrect: The "coffee looking" contents indicate GI bleeding. The NG tube will empty the stomach and monitor the bleeding but is not the top priority to prevent harm to the client. 4. Incorrect: The client has an increased heart rate and if the oxygen and circulation are not improved, problems could occur. Attaching the client to an ECG monitor will allow you to monitor thew heart for arrthymias or impending damage due to decrease oxygen. Necessary but not the first priority.

Which client should the nurse see first? 1. 53 year old client with chest pain scheduled for a stress test today 2. 62 year old client with mild shortness of breath and chronic obstructive pulmonary disease 3. 66 year old client with angina scheduled for a cardiac catheterization this AM 4. 78 year old client who had a left hemispheric stroke 4 days ago

1. Correct: The client may be experiencing a myocardial infarction and requires further assessment. Therefore, this client would not be a priority over a client who may be experiencing a MI. 2. Incorrect: Dyspnea is one of the three (chronic cough, sputum production, and dyspnea) primary symptoms characteristic of chronic obstructive pulmonary disease. 3. Incorrect: The client is scheduled for the procedure needed for further assessment of angina. This client would be considered more stable than the client who may be having a MI. 4. Incorrect: After a stroke has occurred, medical management is aimed at preventing a second stroke from occurring and rehabilitation. This client may have significant sequelae related to the stroke, but would not be considered acute nor a priority over the client possibly having a MI. It is important for the nurse to see all of the clients. Assessment of the client with chest pain scheduled for a stress test today is the priority. The nurse must assess the client to determine if the pain may be caused by a myocardial infarction (MI). A MI would be the most life threatening problem of the listed clients.

Which client should the nurse see first after receiving report on assigned clients? 1. Having dyspnea after surgery. 2. Needing an IV started for the administration of blood. 3. Crying with pain after back surgery. 4. Vomiting dark brown, granular material.

1. Correct: The client may be having a pulmonary embolism after surgery. This client with oxygenation needs takes priority over the other three clients. 2. Incorrect: Needing an IV started for blood administration does not take priority over oxygenation. If blood is needed, tissue perfusion could be altered, so this would need to be addressed in a timely manner after airway issues and other potentially deteriorating situations have been addressed. 3. Incorrect: Pain is expected after back surgery and is not a priority over oxygenation. When possible, the pain should be assessed and medications administered. Remember, pain never killed anyone. 4. Incorrect: This client with dark brown emesis may have an upper GI bleed that has slowed or stopped. This is the second client to see but is not a priority over oxygenation. This could potentially return to active GI bleeding and the client's condition could deteriorate rapidly, so the client would need to be seen following the client with dypnea.

A 3 day post-operative client with a left knee replacement is reporting chills and nausea. Temperature: 100.8ºF/38.2ºC, pulse: 94, respiration: 28 and blood pressure is 146/90. What is the nurse's best action? 1. Call the surgeon immediately. 2. Administer extra strength acetaminophen per prescription. 3. Assess the surgical site. 4. Offer extra blankets and increase fluids.

1. Correct: The client's symptoms are indicative of infection, and the primary healthcare provider needs to be notified and may want diagnostic tests performed. The other actions are appropriate to treat the symptoms and provide comfort, but they are not the best action to fix the problem. 2. Incorrect: While this may be appropriate, it may also delay treatment of the problem, which is infection. Remember, you can only pick one answer to fix the problem and this action will only treat the symptoms. 3. Incorrect: The primary healthcare provider may want the site assessed, but this also delays treatment. Since you can only pick one option, this is not the best. 4. Incorrect: Comfort measures are always appropriate, but this is not the best action available.

A nurse with less than one year of experience reports to an experienced nurse, "The charge nurses are always checking up on me and evaluating my client care. I feel as if the charge nurses do not trust me to give good care to my clients." Which response by the experienced nurse demonstrates an understanding of appropriate staff supervision? 1. The charge nurses are accountable for supervising client care and client safety after delegating the client care assignments. 2. The charge nurses do that to everyone. It can be annoying sometimes, wwhen they ask about your client care. 3. Why don't you speak to the charge nurses about your perception of not being trusted to care for your clients? This is probably not their intention. 4. You are a new nurse, and the charge nurses know that you do not have the experience and knowledge base yet to handle some of your assignments.

1. Correct: The experienced nurse demonstrates an understanding of appropriate staff supervision by answering that the charge nurses are accountable for supervising client care and safety after they have made client care assignments, and by clarifying that the charge nurses are probably attempting to be supportive of the new graduate nurse. 2. Incorrect: This answer does not address the nurse's question correctly. This answer is an example of nontherapeutic communication. The nurse is giving the opinion that the charge nurse's supervision technique is annoying. 3. Incorrect: The nurse is making a statement about the charge nurse's intentions, but does not know what the nurse's intentions are. The role of the charge nurse should be addressed. 4. Incorrect: This is a negative statement about the nurse's job performance. A more positive approach is to explain the charge nurse's role.

A newly appointed nurse manager on the unit has a stable staff who have worked together for 5 or more years. The unlicensed assistive personnel (UAPs) are accustomed to informally arranging their lunch time; however, the nurse manager has implemented a plan to assign breaks and lunch. The UAPs are angry and refuse to change to the new system. What should be the nurse manager's first action in this situation? 1. Plan a unit staff meeting to discuss the problem and receive input for resolution. 2. Inform the staff that the plan will be implemented and those not following the plan will be disciplined. 3. Ask the charge nurse to address the problem daily as it occurs. 4. Plan a meeting with all

1. Correct: The key word in the stem is first. So yes, get everyone together and discuss the problem and find areas of compromise where possible. 2. Incorrect: Too authoritative. This is good staff that has worked together on the unit for a long time. We want them to be happy and get the work done. Again, the key word in the stem is first. 3. Incorrect: No, this is a manager's issue resulting from a new system. This may need to be done but is not the first action. 4. Incorrect: Explaining the rationale to one group does not promote teamwork. It is better to plan a unit staff meeting and not a meeting for only the UAPs.

What is the best method for the nurse to verify correct nasogastric (NG) tube placement after insertion? 1. X-ray of the upper GI 2. Gastric aspiration and pH testing 3. Auscultation of air instilled into the stomach 4. Visualization of the tube markings

1. Correct: The gold standard for nasogastric feeding tube placement is radiographic confirmation with X-ray. This is the most reliable method! 2. Incorrect: Both respiratory and gastrointestinal aspirates may be similar in color and may be misinterpreted. 3. Incorrect: This method cannot differentiate tube placement in the stomach or lung. The practitioner may still hear a rush of air. 4. Incorrect: Visualization of tube markings does not provide a reliable verification that the tube is in the stomach. This has never been a reliable way of verifying placement.

Which action by a nurse would require the charge nurse to intervene? 1. Walking in the hallway outside the operating room without a hair covering. 2. Putting on a surgical mask, gown and cap shoe cover before entering the operating room (OR). 3. Wearing a surgical mask into the holding area. 4. Wearing scrubs from home into the nursing station.

1. Correct: The hallway outside the OR is restricted to personnel with surgical attire and coverings. This area requires boot covers and hair covering. 2. Incorrect: Putting on a surgical mask, gown, cap and shoe covers are all required prior to entering the OR. You are walking into a sterile area that requires these coverings. 3. Incorrect: Surgical mask may be worn in the holding area, but is not required. This area is a clean area, but not sterile. 4. Incorrect: Wearing scrubs into a nursing station is appropriate. This area is not considered part of the surgical suite.

Which client is at the greatest risk for developing pancreatic cancer? 1. 70 year old obese client who smokes one pack of cigarettes a day 2. 64 year old client who had gallbladder surgery less than 5 years ago 3. 58 year old client with Chron's Disease 4. 52 year old client whose mother died from pancreatic cancer

1. Correct: The incidence of pancreatic cancer increases with age. Cigarette smoking, exposure to industrial chemicals or toxins in the environment, and a diet high in fat, meat, or both are associated risk factors. 2. Incorrect: Diabetes and pancreatitis are associated with pancreatic cancer. 3. Incorrect: Diabetes and pancreatitis are associated with pancreatic cancer. 4. Incorrect: The inherited risk is small. 1. Look at each option as True or False. 2. Option 1 is true. The incidence increases with age. Almost all patients are older than 45. About two-thirds are at least 65 years old. The average age at the time of diagnosis is 71. Cigarette smoking is an associated risk factor and the risk of developing pancreatic is twice as high in smokers. 3. Option 2 is false. Gallbladder disease or gallbladder surgery is not associated with pancreatic cancer. 4. Option 3 is false. Chron's Disease is not associated with pancreatic cancer. 5. Option 4 is false. Although family history is a risk factor, most people who get pancreatic cancer do not have a family history of it.

An experienced RN and LPN are working with a new nurse who has just recently passed NCLEX®. The team is assigned to care for 12 clients on the medical-surgical unit. Which factor is most important for consideration when delegating? 1. Lack of experience of the new nurse. 2. The preferences of the LPN who has experience. 3. RN's desire to avoid confrontation. 4. Assignment of equal number of clients to the RN, LPN and new nurse.

1. Correct: The lack of experience of one of the team members (the new RN) must be considered when delegating for client safety. The new nurse may not have the knowledge, assessment skills, and experience needed to care for clients who are unstable or have complex health issues. 2. Incorrect: Preferences by nurses should not guide delegation decisions. This takes the focus off what is best for the clients and places the focus on the nurse. 3. Incorrect: The possibility for conflict when delegation decisions are made should not influence these decisions which are made in the best interest of the client. 4. Incorrect: Although it seems like the "fair" thing to do by each nurse caring for the same number of clients, the delegation decisions should be based on the experience of the new RN. Client safety could be compromised by assigning the new nurse to clients who are unstable or have complex health issues. Delegation to the LPN must include consideration of the LPN's scope of practice.

Following a thyroidectomy, a client reports shortness of breath and neck pressure. Which nursing action is the best response? 1. Remove the dressing and elevate the head of bed. 2. Call a code, open the trach set, and position the client supine. 3. Obtain vital signs. 4. Immediately go to the nurse's station and call the primary healthcare provider.

1. Correct: The nurse should identify that the client is in respiratory distress. So get the dressing off the neck, elevate the HOB and see if they can breathe any better. Stay with the client. 2. Incorrect: Calling a code and opening a trach set is premature. What is likely the problem? Swelling around the airway. Do something that will decrease swelling. Placing the client flat will make the swelling and breathing worse. 3. Incorrect: Don't just look and check. The nurse must do something. This is delaying treatment. Checking the vital signs will not correct the problem. 4. Incorrect: Never leave an unstable client. If the client is having trouble breathing, then that client is unstable. The nurse can call the primary healthcare provider from the room.

A client has been admitted to the orthopedic floor following application of a long leg cast for a fractured femur. What nursing action takes priority? 1. Perform neurovascular checks of the extremities. 2. Cover the edge of the cast near the groin area. 3. Instruct client not to insert anything into cast. 4. Use palms of hands to lift and position the cast.

1. Correct: The most vital aspect of care for clients with a fracture and/or cast is frequent neurovascular checks. Circulation can quickly become compromised secondary to edema from the injury or application of the cast, leading to permanent nerve and tissue damage. Neurovascular checks are performed every two hours for the first 24 hours, or more often per hospital protocols, and both extremities must be compared when looking for problems. 2. Incorrect: While this is a vital action by the nurse, it is not the initial priority. Because this client has a long leg cast for a fractured femur, there is the potential for urine to contaminate the cast close to the groin. That would impair the integrity of the cast, or potentially cause an infection. The nurse definitely needs to cover the upper edges of the cast near the groin with water proof material, but there is another action to complete first. 3. Incorrect: Clients must always be instructed on self care or equipment function as part of the recovery process. Proper cast care following discharge is essential and, in particular, the importance of not placing anything down inside the cast. Clients tend to complain of itching skin beneath a cast and may put baby powder, corn starch or other objects inside the cast to scratch. All these can cause serious complications, and the nurse must provide specific teaching to prevent such problems. However, those instructions are not the most immediate priority for the nurse at this time. 4. Incorrect: Casting material can take up to 24 hours to dry hard enough to protect the client's injury. In the meantime, careful handling of the cast when positioning the client is crucial. The nurse is aware that the cast must be lifted using only the palms of the hands to prevent indentations which could injure the client's skin beneath the casting material. These instructions must also be relayed to any personnel providing care to the client; however, this is not the first priority.

The nurse is supervising the care of a client on bedrest with a skull fracture from head trauma. Which action, when performed by an unlicensed assistive personnel (UAP), should the nurse interrupt? 1. Assisting with turn, cough, and deep breathing (TCDB) 2. Elevating the head of the bed to 30 degrees. 3. Measuring urinary output every hour. 4. Turning off room lights.

1. Correct: The nurse should interrupt the UAP assisting with TCDB because this may increase intracranial pressure (ICP). TCDB increases intrathoracic pressure which then increases ICP. 2. Incorrect: Maintain client with head trauma in the head up position. This position promotes drainage from the head and decreases vascular congestion. 3. Incorrect: This is an acceptable action and one the UAP can do. 4. Incorrect: You want to decrease stimulation and turning off room lights will provide restful environment in an effort to decrease ICP.

A client who is suicidal confides to the night nurse, "I will try again when I get out of this place." What is the nurse's best response? 1. "What do you plan to do?" 2. "You will try what again?" 3. "Why would you want to do that? You have everything to live for." 4. "Are you trying to get back at your family for sending you here?"

1. Correct: The nurse must assess the seriousness of the client's intent. Does the client have a plan and the means? How lethal are the means? Direct questions are appropriate when suicide is a possibility. 2. Incorrect: Inappropriate. This is an indirect question. The nurse needs to get to the point when suicide is a possibility. This question does not address the plan. 3. Incorrect: Inappropriate. This is also an indirect question. Get to the point. The nurse is making a judgment about the client's life. The nurse does not know the specifics of the client's life. 4. Incorrect: Inappropriate. Although this is a direct question, it does not explore the ability of the client to successfully accomplish the task. Get to the point. This type of question could cause the patient to become defensive.

A client diagnosed with pancreatic cancer is being discharged home to live with an adult child. What action should the nurse take to promote continuity of care? 1. Identify community services available for the client and family. 2. Refer client for hospice care. 3. Advise family that client would benefit more from nursing home placement. 4. Make arrangements for around the clock home health aides.

1. Correct: The nurse promotes continuity of care at discharge by providing a smooth transition from one level of care to another. The nurse should include in the discharge plan appropriate community support services available to the client and family so that they can obtain support as needed. 2. Incorrect: This may be premature at this point. Hospice referral is provided when any person with a life threatening illness, which measures life in months rather than years, qualifies for hospice care. 3. Incorrect: It is not appropriate for the nurse to impose personal opinions about what is best for the client. 4. Incorrect: This may be premature at this point. Further assessment is needed and can be provided as the cancer progresses.

Which nurse would be the most appropriate for the charge nurse to assign to a 5 year old admitted in sickle cell crisis? 1. The nurse who is taking care of a 4 year old who had a routine appendectomy, a 3 year old who had bowel surgery, and a 10 year old with developmental delays. 2. The nurse who is taking care of a 6 month old with Respiratory Syncytial Virus (RSV), a 3 year old with exacerbation of asthma, and a 6 year old with a urinary tract infection for 2 weeks. 3. The nurse taking care of a 9 year old newly diagnosed with diabetes, a 6 year old with end stage renal disease, and a 2 year old with contact dermatitis. 4. The nurse taking care of a 8 year old with skeletal traction, a 5 year old with cerebral palsy, and a 12 year old with cystic fibrosis.

1. Correct: The nurse taking care of the appendectomy, bowel surgery, and developmentally delayed child has the set of clients that is less busy and has fewer client care needs. Routine appendectomy and bowel surgery will need observation and assessment but should be stable. The child with developmental delays will need assistance but no life threatening concerns with any of these clients. 2. Incorrect: This set of clients are not appropriate primarily, because of the RSV client. The client with sickle cell already has an oxygen problem and does not need RSV too. RSV is very contagious. 3. Incorrect: This set of clients are very labor intensive. The newly diagnosed diabetic requires constant assessment and interventions to prevent complications. The 6 year old with end stage renal disease also will require a great deal of nursing assessment. 4. Incorrect: Assignment requires much care for clients. This set of clients are inappropriate because of the labor intensive needs. Skeletal traction will require pin care, skin care and prevention of immobility. The cerebral palsy client will require assistance with hygiene and self care and the cystic fibrosis client requires respiratory and GI care including assessment f

Which nurse would be the most appropriate for the charge nurse to assign to a 5 year old admitted in sickle cell crisis? 1. The nurse who is taking care of a 4 year old who had a routine appendectomy, a 3 year old who had bowel surgery, and a 10 year old with developmental delays. 2. The nurse who is taking care of a 6 month old with Respiratory Syncytial Virus (RSV), a 3 year old with exacerbation of asthma, and a 6 year old with a urinary tract infection for 2 weeks. 3. The nurse taking care of a 9 year old newly diagnosed with diabetes, a 6 year old with end stage renal disease, and a 2 year old with contact dermatitis. 4. The nurse taking care of a 8 year old with skeletal traction, a 5 year old with cerebral palsy, and a 12 year old with cystic fibrosis.

1. Correct: The nurse taking care of the appendectomy, bowel surgery, and developmentally delayed child has the set of clients that is less busy and has fewer client care needs. Routine appendectomy and bowel surgery will need observation and assessment but should be stable. The child with developmental delays will need assistance but no life threatening concerns with any of these clients. 2. Incorrect: This set of clients are not appropriate primarily, because of the RSV client. The client with sickle cell already has an oxygen problem and does not need RSV too. RSV is very contagious. 3. Incorrect: This set of clients are very labor intensive. The newly diagnosed diabetic requires constant assessment and interventions to prevent complications. The 6 year old with end stage renal disease also will require a great deal of nursing assessment. 4. Incorrect: Assignment requires much care for clients. This set of clients are inappropriate because of the labor intensive needs. Skeletal traction will require pin care, skin care and prevention of immobility. The cerebral palsy client will require assistance with hygiene and self care and the cystic fibrosis client requires respiratory and GI care including assessment fro complications.

Which task would be appropriate for the nurse to assign to an LPN/VN? 1. Changing a colostomy bag. 2. Hanging a new bag of total parenteral nutrition (TPN). 3. Teaching insulin self administration to a diabetic client. 4. Administering IV pain medication to a two day post op client.

1. Correct: The only procedure listed that is within the LPN/VN's practice range is changing the colostomy bag. This is a task that can be delegated to the LPN/VN. 2. Incorrect: Hanging a new bag of TPN is parenteral therapy requiring a central line. This is outside the scope of practice for the LPN/VN. Therefore, the RN must perform this task and cannot delegate this to the LPN/VN. 3. Incorrect: Teaching is outside the scope of practice for the LPN/VN. Teaching can be reinforced by the LPNVN, but they cannot perform the initial teaching. Teaching insulin self administration cannot be delegated to the LPN. 4. Incorrect: The administration of parenteral pain medications is not in the scope of practice for the LPN/VN. This should not be delegated to the LPN/VN.

The nurse is caring for a client in the outpatient mental health clinic. The client recounts several incidences of spousal abuse. The client says to the nurse, "I know that he loves me. Sometimes I can be quite irritating." Which response is most appropriate by the nurse? 1. "You are not responsible for the abuse." 2. "Sometimes we can irritate our spouses." 3. "The worst is over now." 4. "You should think about leaving him."

1. Correct: The perpetrator is responsible for his/her own actions, but the abused partner may take responsibility or make excuses for them. This mindset needs to be clarified and corrected to prevent further abuse and keep the client safe. 2. Incorrect: Behavior of the perpetrator is not the responsibility of the victim. This statement reinforces the client's belief that they are at fault for the abuse. 3. Incorrect: The severity of the abuse is usually increases over time. This is giving false reassurance to the client. 4. Incorrect: The nurse is offering advice with the "should" statement. A decision to leave must be made by the victim, and the victim should understand that, at the point of leaving, violence may become fatal.

A client comes to an obstetric clinic for a routine prenatal checkup at 32 weeks gestation. The nurse palpates the client's abdomen to determine fetal position so that fetal heart sounds can be assessed. It is determined that the fetal position is left occipital anterior (LOA). Where should the nurse place the Doppler to hear fetal heart sounds? 1. Below the umbilicus, on the mother's left side. 2. Below the umbilicus, on the mother's right side. 3. Above the umbilicus, on the mother's right side. 4. Above the umbilicus, on the mother's left side.

1. Correct: The point of maximal intensity of the fetus is on the mom's abdomen where the fetal heart tones (FHT) is the loudest, usually over the fetal back. Divide the mom's pelvis into 4 quadrants (right and left anterior and right and left posterior). The occiput of the head is the most common presenting part and is abbreviated O. The occiput and back are pressing against left side of mom's abdomen; FHT would be heard below umbilicus on left side. 2. Incorrect: Fetal heart sounds (FHS) would be found below the umbilicus, but not on the mother's right side if the fetal position is LOA. 3. Incorrect: FHS heard above the umbilicus indicate a breech presentation. If the fetal position is determined to be LOA the FHS would be on the mother's left side. 4. Incorrect: FHS heard above the umbilicus indicate a breech presentation, rather than LOA. The FHS would be heard on the mother's left side, but below the umbilicus.

A client receiving 50 mL/hr of enteral feedings has a gastric residual volume of 200 mL and is reporting nausea. What is the appropriate nursing intervention? 1. Stop the feeding and assess gastric residual volume in 1 hour. 2. Reduce the infusion rate to 25 mL/ hour and reevaluate residual volume in 4 hours. 3. Change the feeding schedule from continuous to intermittent delivery. 4. Discard the 200 mL and continue the feedings at the same rate.

1. Correct: The safest response is to STOP the feedings and re-assess in 1 hour. Nausea may be a sign of intolerance. Continuing the feeding may also result in vomiting with possible aspiration. 2. Incorrect: Reducing the rate requires a primary healthcare provider's prescription and does not fix the problem. In this answer, the NCLEX people are giving you a scope of practice question. If you select this answer, you are telling the people who write the test that you are going to write prescriptions for your clients. 3. Incorrect: Changing the feeding schedule requires a primary healthcare provider's prescription and does not fix the problem. Again, with this answer, the NCLEX people are giving you a scope of practice answer. If you choose this answer, you are telling the people at NCLEX that you are going to write prescriptions for your clients. 4. Incorrect: Do not discard residual volumes. Discarding residual volumes can disrupt a client's fluid and electrolyte balance. Standard practice is to give it back. Discarding the residual requires a prescription. Continuing at the same rate is not safe when you have high residuals. The feedings should be stopped.

The nurse is developing a teaching plan for a female client who is taking one of the thiazolidinediones for the treatment of type 2 diabetes. What instruction should be included in the teaching plan? 1. Make sure that you use effective contraception while taking this drug. 2. The drug may lead to weight loss. 3. Therapeutic effect is reached within one to two weeks. 4. Therapeutic effect is reached within one month.

1. Correct: Thiazolidinediones may reduce the plasma concentration of the contraceptives. Additionally, post-menopausal women may resume ovulation. 2. Incorrect: Thiazolidinediones may lead to weight gain and exacerbate congestive heart failure. 3. Incorrect: With thiazolidinediones therapy, therapeutic effect may not be reached until 8 to 12 weeks of treatment. 4. Incorrect: With thiazolidinediones therapy, therapeutic effect may not be reached until 2 to 3 months of treatment.

The nurse is caring for a client taking digoxin. Which electrolyte imbalance should be of most concern? 1. Hypokalemia 2. Hyponatremia 3. Hypomagnesemia 4. Hypocalcemia

1. Correct: The serum potassium level is monitored because the effect of digoxin is enhanced in the presence of hypokalemia and digoxin toxicity could occur. 2. Incorrect: Hyponatremia, hypomagnesemia, and hypocalcemia do not interfere with digoxin. Any electrolyte imbalance can predispose the client to digoxin toxicity, but hypokalemia is the imbalance that can potentiate digoxin toxicity the most. 3. Incorrect:Hyponatremia, hypomagnesemia, and hypocalcemia do not interfere with digoxin. Any electrolyte imbalance can predispose the client to digoxin toxicity, but hypokalemia is the imbalance that can potentiate digoxin toxicity the most. 4. Incorrect: Hyponatremia, hypomagnesemia, and hypocalcemia do not interfere with digoxin. Any electrolyte imbalance can predispose the client to digoxin toxicity, but hypokalemia is the imbalance that can potentiate digoxin toxicity the most.

The client has been prepared for surgery. As the nurse is discussing the post-op expectations, the client says to the nurse, "I am not sure what other options are available to me." What should the nurse do? 1. Request the surgeon visit the client again before surgery. 2. Check client records to see if the client signed the consent form. 3. Explain that the surgery is scheduled for 30 minutes from now. 4. Tell the client that the surgeon explained those options yesterday.

1. Correct: The surgeon is responsible for informing the client about the surgical procedure, the options available,and the benefits and risks of each treatment modality. So, if the client has concerns the surgeon should be told and requested to see the client again prior to surgery. Surgery should be delayed until the client is sure of decision. 2. Incorrect: The consent form signature is important; however, the client has the right to have questions answered and to change his mind. 3. Incorrect: The client should not be encouraged to have the surgery if he still has questions about other options. The consent must be informed, so the client must have all questions answered. The surgery can be delayed until the client's concerns are addressed. 4. Incorrect: The surgeon may have explained the options, however; it is obvious that the client did not understand the options. The client's concerns must be addressed prior to surgery.

The nurse is discharging the client after removing sutures from an abdominal wound. Which instructions should the nurse give the client at the time of discharge to reduce the risk of complications? 1. inspect the wound daily for any changes 2. Resume normal activities when you go home. 3. Keep the incision covered at all times. 4. Follow up with primary healthcare provider when scheduled.

1. Correct: The wound should be inspected daily for any signs of infection once the client goes home. Healing has only just begun by discharge. Signs of wound infection include: Increased pain, swelling, redness, or warmth around the affected area; Red streaks extending from the affected area; Drainage of pus from the area; Fever. 2. Incorrect: The client may be restricted in some activities, such as lifting, that would place undue strain on the suture line. 3. Incorrect: It is likely that the incision can be uncovered, but the primary healthcare provider prescription would apply here. Look for words like "all" which generally make the option wrong. Things are not that definite. 4. Incorrect: This is true; however, the signs and symptoms of infection should be given to the client. If signs/symptoms develop, the primary healthcare provider should be notified prior to the next appointment.

A client presents to the after-hours clinic with reports of pain that occurs with walking but generally subsides with rest. The nurse's assessment reveals coolness and decreased pulses in lower extremities bilaterally. What condition would the nurse recognize these symptoms being most indicative of? 1. Chronic Arterial Insufficiency 2. Chronic Venous Insufficiency 3. Chronic Unstable Angina 4. Chronic Coronary Artery Disease

1. Correct: These symptoms are indicative of arterial insufficiency as there is pain with walking that is relieved by rest. This pain is known as intermittent claudication. In addition, the pulses are decreased or may be absent with arterial insufficiency and the extremities are cool to touch. Other s/s include: paleness of extremity when elevated or possible redness when lowered, loss of hair on affected extremity, and thick nails. 2. Incorrect: Venous insufficiency is not characterized by pain with walking. Pulses are generally normal and color is generally normal with the exception of the brown pigmentation that may be noted (especially around the ankles). 3. Incorrect: The description in the stem is evident of peripheral arterial insufficiency and is not descriptive of decreased coronary artery perfusion. No reports of chest pain were noted. 4. Incorrect: The description is evident of peripheral arterial insufficiency and is not descriptive of decreased coronary artery perfusion. The symptoms listed in the stem are indicative of a peripheral artery problem.

Two days after being prescribed enoxaparin the nurse notes hematemesis. Lab work has been obtained. Based on this data what action is most important for the nurse to take? *Exhibit (labs)* Guaiac stool: + for occult blood Hemaglobin: 10.0 g/dL Hematocrit: 40% RBCs: 4.5 Platelets: 90,000 1. Administer protamine sulfate. 2. Administer the next dose of enoxaparin. 3. Obtain vital signs. 4. Insert a nasogastric tube.

1. Correct: This client has a low hgb, hct, and platelet count and is actively bleeding. Protamine sulfate is the antidote for enoxaparin. 2. Incorrect: Administering another dose of enoxaparin would make the problem worse. The client is actively bleeding and has a low platelet count. 3. Incorrect: The client is actively bleeding. Obtaining vital signs is delaying treatment. The client needs protamine sulfate. 4. Incorrect: The client needs protamine sulfate to correct the problem. Enoxaparin is a low molecular weight heparin (LMWH). It works by blocking the formation of blood clots. Enoxaparin Sodium Injection is indicated for the prophylaxis of deep vein thrombosis (DVT), which may lead to pulmonary embolism (PE) and to treat Acute Deep Vein Thrombosis. Periodic complete blood counts, including platelet count, and stool occult blood tests are recommended during the course of treatment.

A client admitted with somnolence has a history of chronic bronchitis and heart failure. Vital signs on admit are T 101.8ºF (38.8ºC), HR 106, R 26/shallow, BP 90/58. ABGs are pH 7.2, PCO2 75, HCO3 26. The nurse determines that this client has which acid/base imbalance? 1. Respiratory acidosis 2. Respiratory alkalosis 3. Metabolic acidosis 4. Metabolic alkalosis

1. Correct: This client has a respiratory problem. Respiratory failure, COPD, and muscular weakness can lead to respiratory acidosis. Signs & symptoms: hypoventilation, sensorium changes, somnolence, semicomatose to comatose state. pH < 7.35, pCO2 > 45, HCO3 normal. 2. Incorrect: This is not alkalosis since the pH is 7.2 showing acidosis. 3. Incorrect: Not a metabolic related acid/base imbalance, because the HCO​2 is 26 and within the normal range. 4. Incorrect: Not a metabolic related acid/base imbalance, because the HCO​2​ is 26 and within the normal range.

Twelve hours post coronary artery bypass surgery (CABG), the nurse notes the client's level of consciousness has decreased from alert to somnolent. BP 88/50, HR 130 and thready, resp 32, urinary output (UOP) has dropped from 100 mL one hour earlier to 20 mL this hour. What would be the nurse's first action? 1. Administer 100% oxygen per mask. 2. Lower the head of the bed. 3. Give furosemide STAT. 4. Re-check the BP in the other arm.

1. Correct: This client has developed signs of cardiogenic shock, one of the complications post CABG. Cardiac output is decreased, so the client needs more oxygen for the circulating blood volume. 2. Incorrect: Lowering the HOB will not help in cardiogenic shock but will actually make it harder for the heart to pump. 3. Incorrect: Poor kidney perfusion is the reason for the decreased UOP. The kidneys are trying to conserve what little volume the body has to maintain vital organ perfusion as long as possible. 4. Incorrect: Rechecking the BP will not help the problem. With the other symptoms, this BP is most likely accurate. This would only delay treatment and would not fix the problem.

The nurse is preparing to make initial shift rounds. Which primipara client should the nurse see first? 1. 39 weeks with a board like abdomen and scant dark red bleeding. 2. 38 weeks gestation with blood streaked vaginal discharge 3. 40 weeks gestation reporting urinary frequency 4. 36 weeks gestation with pitting pedal edema

1. Correct: This client has symptoms of a placental abruption (abruptio placentae). There is an extremely high risk for fetal loss and maternal disseminated intravascular coaculation (DIC) which is a potentially life threatening clotting disorder in which blood clots form throughout the body's small blood vessels. 2. Incorrect: This describes loss of the mucous plug, which is a normal occurrence at term. 3. Incorrect: Urinary frequency without dysuria at term indicates descent of the fetus. This is a normal occurrence at 40 weeks gestation. 4. Incorrect: Edema confined to the feet and ankles is a normal discomfort of pregnancy at term.

All of the beds in a 10 bed Labor, Delivery, Recovery, Postpartum Unit (LDRP) are full when one of the nurses assigned that day calls in sick. A nurse from the Med surg unit is transferred to the LDRP unit. Which client should the charge nurse assign to this nurse? 1. Client at 32 weeks gestation on oral terbutaline with 4 contractions/hour. 2. One hour postpartum client with a continuous trickle of vaginal bleeding. 3. 2 hours postpartum client reporting intense perineal pain. 4. Client at 36 weeks gestation with a blood pressure of 148/92.

1. Correct: This client is at lowest risk for complications. She is having infrequent contractions and is not at high risk for preterm delivery. She is also receiving an oral tocolytic, terbutaline. Tocolytic agents are used to inhibit uterine contractions and suppress preterm labor. The medical surgical nurse should be able to safely provide care for this client. 2. Incorrect: Continuous vaginal bleeding (even a trickle) with a firm fundus indicates excessive bleeding and is suggestive of lacerations of the vagina, cervix or perineum. This client needs the assessment skills and nursing care of a trained LDRP nurse. The medical surgical nurse may not have the specialized assessment skills and knowledge needed to appropriately care for this client. 3. Incorrect: Intense perineal pain is a symptom of a genital tract hematoma. A client can lose 500 mL of blood into the perineal tissues in a very short period of time. Immediate intervention is also needed for this client. Small hematomas may be managed with ice packs and close observation. However, clients with enlarging hematomas may have to return to surgery for incision and removal of the clot. This client would need a trained LDRP nurse for close monitoring and care. This would not be a client to assign to the medical surgical nurse. 4. Incorrect: This client's blood pressure indicates the presence of a complication known as pregnancy induced hypertension (PIH). This condition is characterized by high blood pressure during pregnancy which can lead to a serious condition known as preeclampsia. Protein will be found in the urine. If not treated, serious complications for the mother and fetus can develop. This client would need the specialized care of the LDRP nurse.

A client delivered a term infant four hours ago. The infant was stillborn. Which room would be most appropriate for the nurse to assign to this client? 1. A private room on the gynocological unit. 2. A private room on the postpartum unit. 3. Discharge her home as soon as her condition is stable. 4. Room her with another client with a pregnancy loss.

1. Correct: This client needs a private room so she can feel free to grieve and have family members stay with her for support. She should be transferred to a gynocological unit so the sights and sounds of the maternity unit do not contribute to her pain. 2. Incorrect: Difficult for mother with stillborn to be on postpartum unit with mothers and their babies. The mother should not be surrounded by these reminders. 3. Incorrect: She does not need to be rushed out of the hospital. She needs to have time with her stillborn and also still needs to be assessed for postpartum complications. Remember that she is going through all of these postpartum stages of normal delivery and requires observations. 4. Incorrect: I know we say like illnesses go together but not here. This client needs privacy and time with her family.

Which postpartum client should the nurse assign to a private room? You answered this question Incorrectly 1. Has antibodies for Hepatitis C. 2. Is rubella non-immune. 3. Is rubella immune. 4. Has lupus antibodies.

1. Correct: This client should be in a private room for her protection and the protection of other postpartum women. The presence of antibodies for Hepatitis C indicates HCV infection and possibly impaired immune function due to liver damage. In addition, Hepatitis C is transmitted by contact with body fluids and it is likely that lochia will be found on toilet surfaces. It is also common for postpartum women to have some kind of wound (perineal laceration or episiotomy) and they will be at increased risk of HCV contaminated lochia coming into contact with their wound. 2. Incorrect: Rubella non-immunity carries risks only to an unborn fetus. If a non-immune pregnant woman contracts rubella during the first trimester, it can result in serious complications in the fetus. Being rubella non-immune is not of concern when making room assignments for postpartum clients. Being non-immune to rubella does not make this client a risk to other postpartum clients. 3. Incorrect: Rubella-immune woman has no risks. Being rubella immune indicates that the client has developed antibodies in response to a previous rubella infection or immunization. Rubella immunity is desired for pregnant women or those planning to become pregnant. This will help to prevent a rubella infection during pregnancy which could cause birth defects. Being rubella immune would not prevent this client from being able to be placed in the room with other postpartal clients. 4. Incorrect: The woman with lupus antibodies is not at increased risk for infection to herself or to others. The pregnant woman with lupus antibodies requires more vigilant monitoring of the fetus because they may increase the risks of neonatal lupus syndrome, certain heart defects, and miscarriage or pre-term birth. However, these antibodies in the postpartal client does not pose any risks to other postpartal clients, so this would not be a factor when making room assignments.

A client is admitted to the pediatric unit with a diagnosis to rule out tuberculosis (TB). What room assignment should the charge nurse make? 1. Private room. 2. Private room and place on protective isolation. 3. Room with a client diagnosed with a respiratory infection. 4. Room with a client who is 24 hours post appendectomy.

1. Correct: This client should be in a private room to prevent the spread by airborne contamination. In addition, standard precautions should be implemented. Remember, you are trying to protect staff and others without the disease from contracting TB. 2. Incorrect: Airborne isolation is needed to protect staff and others. There are no indications for protective isolation. The term 'protective isolation' describes a range of practices used to protect highly susceptible hospital clients from infection. 3. Incorrect: A respiratory infection client needs a private room. Also, it is best for the client with suspected TB to be in a private room. 4. Incorrect: 24 hours postoperative client does not need exposure to infection. Cross contamination is always a concern with a surgical client.

The women's health charge nurse is making assignments for the next shift. The unit is short one staff member and will receive a nurse from the medical surgical unit. Which group of clients should she assign to the medical surgical nurse? 1. Total abdominal hysterectomy, bladder suspension with A&P repair, client with breast reduction. 2. C-section planning discharge, post-partal infection, mastectomy. 3. Vaginal delivery of fetal demise, C-section with pneumonia, 32 week gestation with lymphoma. 4. 28 week gestation of bed rest, post-partal with HELLP syndrome, breast reconstruction.

1. Correct: This group of clients is primarily med surgical. 2. Incorrect: This group of clients needs specific teaching. 3. Incorrect: This group of clients needs specialized care. 4. Incorrect: No, the monitoring is too specific for the med-surg nurse.

The nurse is caring for a client who is preparing to undergo a total hysterectomy for stage 4 cervical cancer. The client is crying and states, "I want to have more children, and I am unsure if I should have the procedure." What is the nurse's best action? 1. Allow the client to discuss her fears, and encourage her to talk more with her primary healthcare provider. 2. Discuss the fun things that she will be able to do after her surgery, and encourage her to make a list of positive things. 3. Explain to the client that her ovaries can be frozen for egg harvesting at a later time, and she can find a surrogate. 4. Advise the client to put off having the surgery until she is certain, and notify the surgeon of the decision.

1. Correct: This is likely anticipatory grieving and fear. Let the client talk and encourage her to talk again to the primary healthcare provider. The client needs reassurance that she is making the right decision. 2. Incorrect: This is not the client's fear and not helpful in this situation. The client may have a hard time thinking about fun and positive things while she is upset and crying. 3. Incorrect:This is not an appropriate answer, and we don't freeze ovaries. The nurse should not give false hope or information. 4. Incorrect: The cancer is already in stage 4. Postponing surgery is dangerous. It is not the nurse's place to advise the client to put off surgery. Postponing surgery is something that needs to be decided between the surgeon and client.

A Hispanic client is considering treatment options for cancer. The client says that she needs to discuss the options with her sons before she makes her final decision. What should the nurse say to the client? 1. You are wanting your sons to assist you in deciding about treatment options. 2. It is really your decision about which option you choose. 3. I will be happy to discuss this issue with you. 4. This shows that you are proud of your sons.

1. Correct: This is paraphrasing the client's statement and is a therapeutic response. Within this culture the family plays a very important role when making decisions about healthcare. 2. Incorrect: Although clients have the right to make autonomous decisions, it is important to remember cultural variations regarding the decision making process. 3. Incorrect: The nurse can discuss the issue; however, the males in the family have much influence on decisions. 4. Incorrect: This is giving an opinion on the relationship of the mother and sons. While this may be true, it does not focus on the cultural aspect of the question and is not the best response.

A nurse is caring for a client that is undergoing outpatient psychiatric treatment for somatization disorder. Which statement by the client indicates that teaching has been successful? 1. "I will keep a diary of times of stress and the appearance of physical symptoms." 2. "I will simply ignore any physical symptoms I get from now on." 3. "The best way for me to stop having physical symptoms is to avoid all the stress in my life." 4. "I will take a sedative when I start having physical symptoms."

1. Correct: This will help the client see the relationship between stress and physical symptoms, which is the first step in recognizing that the symptoms are related to stress. Somatization is the expression of psychological stress through physical symptoms. 2. Incorrect: The possibility of organic pathology must always be considered. Failure to do so could jeopardize client safety. The client should not ignore symptoms. The word "any" is too much like the word "all". 3. Incorrect: Stress cannot be totally avoided for the rest of one's life. The client needs to find more effective ways to cope with stress, such as relaxation exercises, physical activities, and/or assertiveness training. The word "all" is too limiting. 4. Incorrect: This will not solve the problem. The client needs to find more effective ways to cope with stress, such as relaxation exercises, physical activities, and/or assertiveness training. Remember, to stay away from medications on the NCLEX. Drugs should not be the first choice.

When administering an intravenous push (IVP) medication through a continuous intravenous infusion, which intervention is most important for the nurse to take? 1. Assess for drug and solution compatibility. 2. Clamp the tubing of the large volume infusion above the injection port. 3. Stop the large volume infusion and flush the tubing . 4. Use the port nearest the client to administer the IVP medication.

1. Correct: This would have the most life threatening affect on a client if it is not done and an incompatibility exists. Checking for incompatibility between the large volume solution and the medication is a safety issue. 2. Incorrect: This is an action that can be taken when administering an IVP medication; however, clamping the tubing does not have to be done. If the tubing is not clamped when administering the IVP medication, the medication would first go up the tubing toward the large volume container, then go toward the client when the pressure from the push is stopped. 3. Incorrect: This needs to be done if the large volume infusion solution is incompatible with the IVP medication. The action would not have to be implemented when administering all IVP medications. If incompatible, then it should be flushed. 4. Incorrect: This is recommended when administering IVP medication, but would not cause the greatest life-threatening consequences. Using the port closest to the client minimizes the distance the medication must travel, so that the medication gets to the client's circulation faster.

What is the most important action for the nurse to take in order to decrease an adverse drug reaction/interaction in an elderly client who takes multiple medications? 1. Implementing a thorough client assessment. 2. Instructing the client about adverse drugs reactions. 3. Explaining to the client that hospital admissions of older adults are often due to a drug reaction. 4. Teaching the client that adverse reactions are directly proportional to the number of medications taken.

1. Correct: To prevent complications of medication administration, such as adverse drug reactions and interactions, careful planning is priority. A thorough assessment of the client is vital when planning care. 2. Incorrect: Instructing the client about adverse drug reactions is a true statement that supports client education, but not more important than thorough client assessment. 3. Incorrect: Explaining the prevalence of drug reactions in the elderly is a true statement that supports client education, but not more important than thorough client assessment. 4. Incorrect: Teaching the client that risk increases with the number of medications taken is a true statement that supports client education, but not more important than thorough client assessment.

Which food selections would need to be removed from the tray by the nurse for a client recovering from thyroidectomy? 1. Roasted almonds 2. Mashed vegetables 3. Scrambled eggs 4. Ice cream

1. Correct: Too hard and crunchy. Need soft diet because esophagus is right behind the thyroid and trachea. This would be difficult to swallow after surgery due to pain. 2. Incorrect: Mashed vegetables will be soft and easy to swallow. 3. Incorrect: This would be good for the client. The food is soft and easy to swallow. 4. Incorrect: Ice cream with neck surgery. Cold and soft. Good food choices include juice, mashed vegetables and potatoes, ice cream, pudding, milk shakes, eggs, broth, pasta, beans, hummus, guacamole, tender meats, fish, and gelatin. The key is to provide soft, easy to swallow foods.

A nurse is working in a walk-in clinic where a mother brings in her 6 year old child stating, "My child is just not right." The nurse notes an unusual odor to the child's breath, new onset of bed-wetting, and lethargy. What prescription by the primary healthcare provider should be performed first? 1. Blood glucose 2. Urinalysis for white blood cells (WBC) 3. Oxygen saturation 4. Toxicology screen

1. Correct: Type I diabetes usually has a sudden onset and many times diabetic ketoacidosis (DKA) is the first encounter. The symptoms in the stem: unusual odor to the breath, bed wetting, and lethargy are symptoms of DKA. The blood glucose is one of the most important tests for the diagnosis of DKA. 2. Incorrect: A urinalysis to assess WBC will not support the diagnosis of DKA. 3. Incorrect: In this case, oxygen saturation is not the priority. This child is not in respiratory distress. 4. Incorrect: A toxicology screen will not support a diagnosis of DKA; however, if the blood glucose was not elevated, it could provide further assessment data.

For a client with a major burn, which evaluation criterion identified by the nurse best indicates that fluid resuscitation has been effective during the first 24 hours of care? 1. Urine output of 860 mL / 24 hours. 2. Increase in weight from preburn weight. 3. Heart rate of 122 beats per minute 4. Central venous pressure of 18 mm

1. Correct: Urine output is the best indicator of adequate fluid replacement during the first 24 hours. 2. Incorrect: The weight is not a good indicator now because of the large volume of fluids being infused. These extra fluids would increase the weight. Edema is a problem because of third spacing. 3. Incorrect: The heart rate should come down with adequate fluid replacement. 4. Incorrect: The CVP reading is too high. This indicates that too many fluids have been given.

The nurse is working on the inpatient mental health unit and determines that one of the clients has suicidal thoughts. The nurse initiates suicide precautions. Which rationale best validates the action? 1. The client has the right to a safe care environment. 2. The nurse may be sued for malpractice if injury occurs. 3. All clients on mental health units are placed on suicide precautions. 4. Clients are most likely to act on suicidal thoughts when energy is low.

1. Correct: Verbalizing suicidal thoughts is a risk factor for client suicide. Safety must be maintained while the client is in this vulnerable state. The nurse identifies client at risk of suicide and intervenes to prevent harm for those identified as being at risk. 2. Incorrect: Client safety is the primary issue here. 3. Incorrect: This is not a true statement. All clients have the right to a safe environment; however, not all clients on the mental health unit are placed on suicide precautions. Only clients identified at risk for suicide are placed on suicide precautions. 4. Incorrect: This is an untrue statement. Clients are likely to act on suicidal thoughts as energy levels improve. The issue here is client safety, and the client's right to a safe environment.

The nurse routinely screens injury victims for the possibility of intimate partner violence (IPV). Which statement correctly supports the nurse's action? 1. Victims of abuse are likely to report injuries and causes that do not fit the normal profile. 2. IPV is not routinely seen in the upper socioeconomic level. 3. All women should be screened, but men are not routinely screened. 4. Only victims who enter the emergency department alone should be screened for IPV.

1. Correct: Victims of abuse most often report causes of injuries that don't fit with the type of injury observed. For example, a victim may report that a bruised eye came from "running into" a door. The victim may feel the abuse is a personal incident or is afraid of the abuser. 2. Incorrect: Though many IPVs are from low income families IPV occurs across all socioeconomic levels and cultures. All suspected IPV cases should be assessed and reported regardless of their socioeconomic levels and cultures. 3. Incorrect: Men are also victims of IPV, though not as frequently as women. All potential victims should be assessed and reported if needed. 4. Incorrect: Many times the perpetrator will come to the emergency department (ED) with the victim. The victim may be afraid to give an accurate report of the accident with the perpetrator in the ED exam room. If so, more discreet screening is necessary.

Parents of school-aged children are working toward a goal of healthy family TV viewing. Which parental statement indicates adequate understanding of appropriate use of TV in the family? 1. I don't allow my kids to watch violent TV shows. 2. They usually watch the kid shows on the kids' networks. 3. I don't usually worry about the time watching TV on weekends. 4. They can choose one TV show per day without my input.

1. Correct: Violent TV shows are not recommended for school-aged children. They may be disturbing and may desensitize them to violence. 2. Incorrect: Even shows on kids' networks may demonstrate values that are not congruent with the healthy family. Input from the parents is needed here as well. 3. Incorrect: TV time should be limited each day to allow time for physical activity, social interaction, and development of hobbies and other skills. 4. Incorrect: The child needs the input of the parents. Parents and children may have an agreed upon list of shows that the child may watch.

Which intervention would the nurse recommend to a client with rheumatoid arthritis to best help relieve joint stiffness? 1. Take a warm shower prior to performing activities of daily living. 2. Take an aspirin after activity to help decrease inflammation. 3. Lose 10 pounds of weight. 4. Apply cold compresses to joints for 30-45 minutes.

1. Correct: Warm water may provide muscle relaxation, increase blood flow, and reduce stiffness. 2. Incorrect: A mild analgesic may be taken before activity or exercise to decrease pain and inflammation. 3. Incorrect: Weight reduction may be recommended to relieve stress on joints but does not address joint stiffness. 4. Incorrect: Apply cold compresses for 15-20 minutes at a time. Longer than 20 minutes may cause tissue damage. 1. Look at each option as True or False. 2. Option 1 is true. Warm baths and/or showers can help relieve joint stiffness and allow the client to more comfortably perform ADLs. 3. Option 2 is false. Aspirin or other anti-inflammatory drugs or analgesics should be taken before activity to help decrease inflammation and reduce joint pain. 4. Option 3 is false. Weight loss may decrease stress on joints but pain and stiffness will still be a problem. 5. Option 4 is false. Cold compresses may be effective for reducing joint pain; heat is used for decreasing pain and stiffness. When a client uses a cold compress, the time should be limited to 10 to 15 minutes to decrease risk of tissue damage.

The nurse is caring for a burn victim with a skin graft to the hand. The area is pale and mottled but has good capillary refill. What is the nurse's best action at this time? 1. Warm the room. 2. Submerge the hand in warm water. 3. Order a K pad and apply to hand. 4. Have the client exercise the fingers to increase blood flow.

1. Correct: When caring for clients with skin grafts, we want good circulation, so warm that room up. 2. Incorrect: This will not improve circulation and can lead to infection. 3. Incorrect: This will not improve circulation. Someone who has a skin graft doesn't have good sensation so there is risk of another burn to the graft with this. 4. Incorrect: Working those stiff, cold fingers will further imbalance the oxygen supply. This will not help, particularly if the environment remains cool.

A nurse on the unit has had a disagreement with the family of a client regarding the client's dressing change. What is the best action by the nurse manager? 1. Meet with the family member and the RN to discuss the disagreement. 2. Assure the family member that the nurse followed the hospital procedure. 3. Discuss the dressing change procedure with the RN and compare to a current textbook. 4. Report the argument to the hospital administrator.

1. Correct: When conflict occurs, meet with both parties together to discuss the problem. Each party can hear what the other is saying and the nurse manager is not caught in the middle. They will be able to come up with solutions together or the manager can mediate. 2. Incorrect: It is ok to clarify that the nurse followed hospital procedure. However, the nurse is sing the nontherapeutic communication technique of blocking. The family member may still believe that there is another procedure that could have been initiated. 3. Incorrect: You may want to do this as well, but it will not address the conflict. The conflict is that the family member disagrees with the nurse's procedure for dressing change. 4. Incorrect: The nurse manager must try to resolve the conflict between the family member and the nurse first. If the conflict cannot be resolved the nurse manager would notify the person that is next in the chain of command.

The client with mania has repeatedly interrupted group session with the counselor. The client explains that they already know this information about family roles and paces around the room. What should the nurse do at this time? 1. Ask the client to take a walk with you and make another pot of coffee. 2. Ask the client to reflect on their behavior to determine if it is appropriate. 3. Ask the group to tell the client how they feel when they are interrupted. 4. Tell the client to perform jumping jacks and count out loud.

1. Correct: Yes, get them away from the group and do something purposeful. Purposeful activities help the client use energy and focus on something. Distractibility is the nurse's most effective tool. 2. Incorrect: That is embarrassing and humiliating to the client. Singling out the client during group activity, does not fix the problem. This may lead to arguing and escalate the client's mania. 3. Incorrect: Sometimes this will be helpful during times of therapy, but the client is manic at this time. They may not believe them. Also, the client may be aggressive toward other group members. 4. Incorrect: This is getting them active, but the group will be interrupted by this behavior. Do not let the client continue with this attention seeking behavior. Remove the client from the group activity. The purpose of the group is to work toward a common goal. The client performing jumping jacks is not working toward a common goal.

The nurse is providing care to a client who has a history of violent episodes against his wife. The client has made a specific threat that he plans to kill his wife when he gets out of the hospital. What should the nurse do first? 1. Discuss the threat with the treatment team immediately. 2. Call the wife immediately to report her husband's intention. 3. Do nothing because the client may change his thinking. 4. Tell the client that he shouldn't make threats like that.

1. Correct: Yes, immediately discuss the treat with the treatment team. The duty to warn is an obligation of healthcare providers. The threat should be discussed with the treatment team, and agency policy for notification of the threatened party should be followed. 2. Incorrect: The team should discuss this first; the next action may be to call the wife. At this point, the client is in the hospital and the wife is not in immediate danger. 3. Incorrect: A threat cannot be ignored. The threatened party has the right to know. The treat team will decide and take the appropriate action. 4. Incorrect: The client has reported valuable information and follow up is required.

The schizophrenic client tells the nurse, "I am Jesus, and I am here to save the world!" The client is reading from the Bible and warning others of hell and damnation. The other clients on the unit are upset and several are beginning to cry. What nursing intervention is most appropriate? 1. Set verbal limits and have the client return to assigned room. 2. Explain to the client that not all people are Christians. 3. Remove the Bible from the client and explain that the client is not Jesus. 4. Ask the client to share with the group how the client is Jesus.

1. Correct: Yes, the nurse must set limits. This is disrupting others and so the client needs to be redirected to their room for a cool down and then another activity shortly thereafter. This client is experiencing delusions of grandeur, which are not reality based, and require intervention that does not reinforce the behavior. 2. Incorrect: No, this will only reinforce the clients thought process of religion. 3. Incorrect: No, don't argue with the client. This is not therapeutic and does nothing to help resolve the disruption to the other clients. 4. Incorrect: This is ridiculing the client and also inflaming the situation. This is not desirable.

The nurse is planning care for a preschool child who is being treated in the hospital for respiratory syncytial virus (RSV). What should the nurse recognize as the child's likely view of this illness in order to properly plan care? 1. Punishment 2. Disturbance to body image 3. Rejection from parents 4. Change in routine with friends

1. Correct: Yes, the preschool child views illness as punishment. The preschooler may believe that the illness occurred because of some personal deed or thought or perhaps just because the child touched something or someone. 2. Incorrect: To the adolescent, appearance is crucial. Illness or injury that changes an adolescent's self-perception can have a major impact. 3. Incorrect: The adolescent fears rejection and criticism from parents. 4. Incorrect: Friends are important to the school aged child and fear that friends will forget them while they are ill. They fear a change in routine.

Which client could the telemetry charge nurse safely transfer in order to admit a new client? 1. Twenty-four hour post operative carotid endarterectomy. 2. Unstable angina with onset of atrial fibrillation. 3. Status post coronary artery bypass grafting (CABG) with atrial flutter. 4. Myocardial infarction with a history of heart failure.

1. Correct: Yes, this client is the least critical. The carotid endarterectomy is to open or clean the carotid artery and hopefully prevent a stroke. Since no evidence of complications or being unstable is presented, this client should be able to be cared for on a general nursing unit. 2. Incorrect: Needs a telemetry bed with new onset of atrial fibrillation. The description provided also tells you that the client is experiencing unstable angina. Unstable angina can occur at rest and may develop suddenly, progressively worsening in a short time period. Unstable angina should be treated as an emergency because the client is at increased risk for a myocardial infarction (MI). 3. Incorrect: The client who had a CABG and is experiencing an should be considered unstable and requires further cardiac monitoring. Arrhythmias are common complications after CABG and are major causes of morbidity and longer hospital stays. The client should be carefully assessed for how well the rhythm is being tolerated. Ventricular response to the rapid rate from the atria may be slower and the cardiac output can be reduced. 4. Incorrect: Myocardial infarction and history of heart failure needs telemetry. This client is considered unstable. The client needs to be monitored for arrhythmias, signs of decreased cardiac output and for any signs of recurring infarction. 1. Client safety is always a priority. 2. Never delay treatment or choose an option that ignores client symptoms. Option 1 was the only option that did not have symptoms or active process that would require continued monitoring. Selections of options 2, 3 or 4 would be ignoring client symptoms.

The nurse is caring for a client who is scheduled to receive furosemide 40 mg IVP twice daily, as well as 20 meq (20 mmol/l) of potassium chloride twice daily. The client's lab work reveals that the potassium level is 2.4 mEq/L (2.4 mmol/L) this morning. How should the nurse proceed? 1. Notify the primary healthcare provider of the potassium level immediately. 2. Administer the medications as scheduled and notify the primary healthcare provider on rounds. 3. Give the potassium, but hold the furosemide until primary healthcare provider rounds. 4. Assess the client for muscle cramps.

1. Correct: Yes, this is a very low level. Normal values are 3.5-5.0 mEq/L (3.5-5.0 mmol/L). This client will need more potassium and less furosemide (a potassium wasting diuretic). 2. Incorrect: No, potassium is dangerously low. Giving the furosemide will drop the potassium level further since it is potassium wasting. Do not wait for the primary healthcare providers to make rounds as they often do not make predictable rounds. 3. Incorrect: This is delaying care and confuses the issue of how much potassium needs to be administered now. 4. Incorrect: Delays care. What if there are no symptoms? Will you wait for symptoms to treat? Hypokalemia is generally defined as a serum potassium level of less than 3.5 mEq/L (3.5 mmol/L). Severe hypokalemia is a level of less than 2.5 mEq/L. Hypokalemia is a potentially life-threatening imbalance. Signs and symptoms may include: Weakness and fatigue (most common); Muscle cramps and pain (severe cases); Worsening diabetes control or polyuria; Palpitations; Psychological symptoms (eg, psychosis, delirium, hallucinations, depression). Severe hypokalemia may manifest as bradycardia with cardiovascular collapse. Cardiac arrhythmias and acute respiratory failure from muscle paralysis are life-threatening complications that require immediate diagnosis.

What is indicated when caring for a client admitted with meningitis? 1. The client should be placed in a negative pressure room and health care providers should wear a N95 protective mask when in contact with the client. 2. The client's room door may remain open and health care providers should wear a facemask within 3 to 6 feet of the client. 3. The client should be placed in a private room and no face mask is needed. 4. The only precaution needed is hand hygiene.

1. Incorrect: Meningococcal infections are not spread by small airborne organisms. Meningococcal infection is spread by large particle droplets. 2. Correct: Meningococcal infections are transmitted by large particle droplets. Meningitis can be spread by respiratory and throat secretions or lengthy contact. 3. Incorrect: Meningococcal infections are not spread by contact with a person's skin or contaminated items. Meningococcal infection is spread by large particle droplets. 4. Incorrect: Meningococcal is not spread by contact with a person's skin or contaminated items, but by large particle droplets. Hand hygiene alone is not sufficient to prevent the spread of meningococcal infections. Option 1 is false. Airborne precautions are required for clients with TB, varicella, or other airborne pathogens and these clients should be in negative pressure rooms and N95 masks should be worn by health care providers. Option 2 is true. Droplet precautions are used for organisms such as influenza or meningococcal infections that are transmitted by close contact with respiratory or pharyngeal secretions. Health care providers should wear a face mask when 3 to 6 feet from the client; however, because the risk of transmission is limited to close contact, the door may remain open. Option 3 is false. Contact precautions are used for organisms spread by skin-to-skin contact such as C. difficile. Masks are not needed and doors do not need to be closed. Option 4 is false. Use of standard precautions are used to care for all clients but more is needed.

A nurse notes late decelerations in the fetus of a client who is receiving oxytocin via IV infusion. What nursing interventions should the nurse perform? 1. Administer naloxone. 2. Place client in side-lying position. 3. Stop oxytocin. 4. Increase the rate of IV fluids 5. Notify primary health care provider. 6. Administer oxygen at 8 L/min per face mask.

1. Incorrect: Naloxone is not indicated here. Naloxone reverses the effects of morphine. There is nothing in the stem indicating that the client received a narcotic. 2. Correct: The side-lying position will relieve pressure from the aorta thus getting more oxygen to the fetus. 3. Correct: Stop the oxytocin infusion. During uterine contraction, blood flow through the uterus slows reducing fetal oxygenation. These intense contractions may be the cause of the late decelerations. 4. Correct: Increasing the IV fluid expands the client's blood volume and improves placental perfusion. 5. Correct: The primary healthcare provider should be notified as continued late decelerations may mean the fetus needs to be delivered immediately via C-section. 6. Correct: Administering oxygen to increase the client's blood oxygen saturation will make more oxygen available to the fetus.

What interventions would be appropriate for the nurse to make for a child who is in Bryant's traction? 1. Perform neurovascular checks every 2 hours. 2. Maintain hip flexion at 90 degrees with buttocks raised 1 inch (2.54 cm) off the bed. 3. Reposition child infrequently so that traction is maintained. 4. Place child prone for one hour daily to prevent contractures. 5. Remove adhesive traction straps daily to prevent skin breakdown. 6. Use wrist restraints to keep child from turning over.

1.& 2. Correct: Both legs are extended in a vertical position in order to maintain hip flexion at 90 degrees. This helps to keep the femur in the hip socket. Because the legs are extended upward the circulation and nerves can be affected. The feet should be assessed for color, pulses, warmth, and sensation every 2-4 hours. 3. Incorrect: The child should be repositioned slightly every 1-2 hours to avoid skin breakdown. 4. Incorrect: The child cannot be placed prone while in Bryant's traction. 5. Incorrect: Traction should not be relieved, which is what would happen if straps are removed. 6. Incorrect: A jacket restraint is used to keep the child from turning over in the bed.

A pediatric nurse is providing anticipatory guidance to a group of parents who have children nearing the age of 1 year old. What milestones should the nurse teach the parents to expect to see in their 1 year old child? 1. Gets to a standing position without help. 2. Puts out arm or leg to help with dressing. 3. Able to say several single words. 4. Pulls toys while walking. 5. Builds a tower of 4 blocks.

1., & 2. Correct: A 1 year old should be able to get to a standing position without help. May stand alone. Can assist in getting dressed by putting out arm or leg. 3. Incorrect: Children at 18 months are able to say several single words. 4. Incorrect: Children at 18 months are able to pull toys while walking. 5. Incorrect: Children at 2 years of age can build a tower of 4 or more blocks. Look at options 1 and 4. They are opposite. Option 1 says they can stand up without help. Option 4 says they can walk and pull a toy. A one year old cannot walk and pull a toy yet. Remember Erickson's stages of psychosocial development and growth and development. This question wants to know what a one year old should be able to do. Option 1 is true. A one year old should be able to get to a standing position without assistance. Option 2 is true. A one year old should be able to put out arms and legs when getting dressed. Option 3 is false. A one year old is not at a stage to put several words together to make sentences. Option 4 is false. A one year old cannot walk and pull a toy yet. Option 5 is false. A one year old can build a tower of two blocks.

A nurse is monitoring a newly hired unlicensed assistive personnel (UAP) perform a bed bath on a client needing total care. Which action by the UAP would require further teaching? 1. Lowers side rails on both sides of bed. 2. Washes eyes with mild soap and water from the inner to outer canthus. 3. Makes certain bath water temperature is between 110-115°F (43-46°C). 4. Uses long, firm strokes to wash from wrist to shoulder of each arm. 5. Performs a back massage after completing the bath.

1., & 2. Correct: The nurse needs to intervene in these situations. Both side rails should not be lowered because the client could fall out of the bed. The UAP should lower the side rail closest to themselves and keep the opposite rail up. Wash eyes with water only since soap is very irritating to the eyes. 3. Incorrect: This would be a correct action by the UAP. The nurse does not need to intervene. Temperatures less than 110°F (43°C) can chill the client, and a temperature greater than 115°F (46°C) may be too hot and burn the client. 4. Incorrect: This is a correct action and does not require intervention by the nurse. Firm strokes from distal to proximal areas promote circulation by increasing venous blood return. 5. Incorrect: A back massage is appropriate after a bath and does not require nursing intervention. A back massage is a way of providing relaxation for the client.

Which signs/symptoms should the nurse assess for the presence of in a client diagnosed with valvular heart disease? 1. Orthopnea. 2. Paroxysmal nocturnal dyspnea. 3. Petechiae on the trunk. 4. Increasing CVP with decreasing BP. 5. Pericardial friction rub. 6. Widening pulse pressure.

1., & 2. Correct: These are signs seen with valvular heart disease. Orthopnea is a condition where the client must sit or stand to breathe comfortably. Paroxysmal nocturnal dyspnea occurs when the client is reclining. It is sudden respiratory distress. 3. Incorrect: This is a sign of endocarditis. 4. Incorrect: This is the hallmark sign for cardiac tamponade. 5. Incorrect: This is a sign of pericarditis. 6. Incorrect: This is a sign of increased intracranial pressure. Now in this question, we have not been told which valve or valves are damaged, so we must think in general about all the valves. So let's look at our options. Option 1, orthopnea. Orthopnea is the sensation of breathlessness in the recumbent position, relieved by sitting or standing. So if my valves are messed up, this can put a strain on the heart. There may be an increase in pressure behind the affected valve. This back pressure can cause blood and fluid to build up in the lungs or lower part of the body (depending on which valve is affected). If it backs up to the lungs we see respiratory problems like orthopnea. So True Option 2, paroxysmal nocturnal dyspnea. This refers to attacks of severe shortness of breath and coughing that generally occur at night. Fluid is backing up into the lungs again. True Option 3, petechiae on the trunk. Petechiae are pinpoint, round spots that appear on the skin as a result of bleeding. There are many causes of petechiae, but valvular heart disease is not one of them. Causes include prolonged straining, certain medical conditions such as endocarditis, specific types of injuries, medications such as anticoagulants, or crushing injuries and sunburn. False Option 4, increasing CVP with decreasing BP. What hallmark sign does this indicate? Not valvular heart disease. Cardiac tamponade. False Option 5, pericardial friction rub. This is the rubbing together of inflamed membranes of the pericardium that produces a sound heard with a stethoscope. It occurs in pericarditis or after a myocardial infarction. So this is false. Option 6, widening pulse pressure. Well we know this is bad and indicates something, but do you remember what? Not valvular heart disease. It is indicative of increasing intracranial pressure.

Which signs/symptoms should the nurse monitor for in a client admitted with a diagnosis of pheochromocytoma? You answered this question Incorrectly 1. Headache 2. Hypotension 3. Hyperglycemia 4. Bradycardia 5. Polycythemia 6. Leukopenia

1., & 3. Correct. This disease is characterized by hypertension, hypermetabolism, hyperglycemia, and headache due to increased release of epinephrine and norepinephrine. 2. Incorrect. Hypertension, rather than hypotension, would be seen in this client. 4. Incorrect. The heart rate will increase rather than decrease. 5. Incorrect. Polycythemia is elevated red blood cell count, which is not seen with this disease. 6. Incorrect. Leukopenia is a low white blood cell count, which is not seen with this disease. Pheochromocytoma is a rare, catecholamine-secreting tumor that may precipitate life-threatening hypertension. The tumor is malignant in 10% of cases but may be cured completely by surgical removal. Signs and symptoms of pheochromocytomas often include: Signs and symptoms include: High blood pressure, hyperglycemia, rapid or forceful heartbeat, profound sweating, severe headache, tremors, paleness in the face and shortness of breath. Less common signs or symptoms may include anxiety or sense of doom, abdominal pain, constipation, weight loss. These signs and symptoms often occur in brief spells of 15 to 20 minutes. Spells can happen several times a day or less often.

What should the nurse include in the teaching plan for a client receiving external beam radiation? 1. Small marks will be placed on the skin to mark the treatment area. 2. Lotion may be used around the treatment area to decrease dryness. 3. The radiation therapist can see, hear, and talk with you at all times during treatment. 4. Stay away from babies for 24 hours. 5. You will have to hold your breath during radiation treatment.

1., & 3. Correct: Small ink marks or small tattoos will be placed on the skin to mark the treatment area. Do not remove the marks. The radiation therapist can see, hear, and talk to the client at all times during treatment. Relieve anxiety by letting client know he/she is not alone. 2. Incorrect: The client should avoid the use of potential irritants (perfume, powders or cosmetics) on the skin in the treatment field. 4. Incorrect: The client receiving external beam radiation is not radioactive and will not radiate others. The client can safely be around other people, babies, and children. Clients who are undergoing brachytherapy or receiving radio pharmaceuticals require that you be aware that the client is emitting radioactivity. 5. Incorrect: The client will need to stay very still so radiation goes to the exact same place each time but can breathe as always. The client does not have to hold breath.

A client was admitted 48 hours ago in septic shock. Treatment included oxygen at 40% per ventimask, IV therapy of Lactated Ringer's (LR) at 150 mL/hr, vancomycin 1 gm IV every 8 hours, and methylprednisolone 40 mg IVP twice a day. Which clinical data indicates that treatment has been successful? 1. Blood pressure 96/68; HR- 98; RR- 20 2. WBC 12,000 cells/mm3 (12 x 109/L) 3. CVP- 6 mmHg 4. pH- 7.30; pCO2- 44; pO2 -92; HCO3- 20 5. Urinary output 0.3 mL/kg/hr

1., & 3. Correct: The systolic BP should be greater than 90. Normal CVP is 2-6 mmHg. 2. Incorrect: Incorrect: WBC is elevated. 4. Incorrect: The client is still in metabolic acidosis, so no improvement. 5. Incorrect: Urinary output should be at least 0.5 ml/kg/hr. Option 1: True. The BP is above 90 systolic which means that the client is perfusing vital organs. The HR and RR are normal. Option 2: False. The WBCs are still elevated, indicating that the client is still sick. Normal range is 4,500 - 11,000 cells/mm3 ​Option 3: True. Normal CVP is 2-6 mmHg. This client is well hydrated at this point. Option 4: False. This client is in metabolic acidosis, so is still sick. Metabolic acidosis occurs with severe sepsis. Look at the pH of 7.3 (normal 7.35-7.45). This pH is low or acidosis. What other chemical says acidosis? The HCO3 of 20 (normal 22-26) is acid. Note that the CO2 is normal at 44 (normal 35-45). Option 5: Urinary output should be a minimum of 0.5/kg/hr in order to be adequate.

The nurse receives new primary healthcare provider prescriptions on a client diagnosed with Addison's disease. What prescription should the nurse question? 1. Weigh QD 2. IV of Normal Saline at 125 mL/hr 3. MRI of pituitary gland 4. Fludrocortisone acetate 0.1 mg by mouth T.I.W. 5. Dehydroepiandrosterone (DHEA) 5 mg by mouth every other day

1., & 4. Correct: QD is listed on the Joint Commission on Accreditation of Healthcare Organizations (JCAHO) official "do not use" list of abbreviations. This should be prescribed as "daily" instead of "QD". The abbreviation T.I.W. stands for three times a week; however, it is an unapproved abbreviation. Use "three times a week". 2. Incorrect: This is a correct action, for a client with Addison's disease, and it is written properly. 3. Incorrect: The primary healthcare provider may suggest a MRI scan of the pituitary gland if testing indicates the client might have secondary adrenal insufficiency. This is an approved abbreviation. 5. Incorrect: This is written correctly and may be given to women to treat androgen deficiency in cases such as this client with Addison's disease. At first glance, it looks like you are being asked to identify which prescriptions would not be acceptable for a client with Addison's disease, doesn't it? But look at the options. Actually all of these prescriptions are acceptable, but look at the way the prescriptions are written. Some of the options are using unapproved abbreviations. Now as we look at the option remember what is being asked: "What prescription should the nurse question?". So we are looking for options that are wrong! Option 1. True. This prescription is written incorrectly. "QD" is listed on the Joint Commission on Accreditation of Healthcare Organizations (JCAHO) official "do not use" list of abbreviations. This should be prescribed as "daily" instead of "QD". Option 2. IV of Normal Saline at 125 mL/hr. This is a correct action for a client with Addison's disease, and it is written properly. We are looking for the options that are incorrect, so don't pick this one. 3. Incorrect: MRI of pituitary gland. Nothing wrong here! The primary healthcare provider may suggest a MRI scan of the pituitary gland if testing indicates the client might have secondary adrenal insufficiency. This is also an approved abbreviation. Option 4. True. This prescription is written incorrectly. The abbreviation T.I.W. stands for three times a week, however, it is an unapproved abbreviation. Use "three times a week". Option 5. DHEA 5 mg by mouth every other day. Nothing wrong with this prescription or how it is written. DHEA may be given to women to treat androgen deficiency in cases such as this client with Addison's disease.

The charge nurse is observing a new nurse administer cortisporin otic to the left ear of a 2 year old child. What action by the new nurse would indicate that the charge nurse needs to intervene? 1. Position the client prone, with affected ear up. 2. Pull pinna down and back. 3. Administers medication at room temperature. 4. Allow child to sit up once medication is instilled. 5. Educate parents that the medication may burn when instilled.

1., & 4. Correct: The charge nurse needs to intervene if the new nurse does not position the client supine, with affected ear up. The child may lie in a parents lap to decrease anxiety and increase cooperation. The child should remain supine for 5 minutes after medication is instilled to assure medication remains in ear canal. Remaining supine for several minutes permits to fluid to be absorbed and not drain back out of the ear canal. 2. Incorrect: The charge nurse does not need to intervene if the new nurse pulls the pinna down and back. For children 3 years of age and younger, pull pinna down and back. For adults, pull pinna up and back for medication administration or otoscopic exam. 3. Incorrect: The charge nurse does not need to intervene if the new nurse administers the medication at room temperature. It should not be placed in the ear cold as it can cause nausea/vomiting and dizziness. 5. Incorrect: The charge nurse does not need to intervene if the new nurse teaches the parents that generally mild adverse reactions include ear irritation, local stinging or burning, and/or dizziness.

A client with cancer refuses treatment and asks about options for hospice home care. The client's daughter asks the case manager to talk the client into agreeing to cancer treatment. The nurse explains to the daughter that this violates which client right? 1. To self-determination 2. To decline participation in research studies and experimental treatments 3. To expect reasonable continuity of care 4. To make decisions about the plan of care 5. To advocacy

1., & 4. Correct: Under the Patient Self-Determination Act (PSDA), healthcare institutions provide clients with a summary of their rights when making health care decisions as well as the facility's policies regarding recognition of advanced directives. The client is advised of the right to consent to or refuse treatment. Client rights refer to such matters as access to care, dignity, confidentiality, and consent to treatment. The competent adult client has the right to participate in the plan of care, to refuse a proposed treatment, and to accept alternative care and treatment. Documentation should be made that the client fully understands the risks and benefits of their decision. 2. Incorrect: The right to decline participation in research or experimental studies is incorrect because no research or experimental treatment is proposed to the client. 3. Incorrect: The right to expect reasonable continuity of care appears to be a possible correct answer, but is incorrect because the client has not been transferred to hospice home care. Attempting to convince the client to agree to cancer treatment would not be pertinent to continuity of care in this situation. 5. Incorrect: The right to advocacy relates to the right to have another person present during interviews or examinations. This right would not be violated by the nurse if attempts were made to convince the client to have cancer treatment.

A client diagnosed with heart failure has been prescribed a 2 gm sodium diet. Which food choices selected by the client would indicate to the nurse that the client understands this diet? 1. Pork loin 2. Frozen cheese ravioli dinner 3. Instant vanilla pudding 4. Thin crust pepperoni and ham pizza 5. Fresh salad with fresh citrus juice dressing 6. Bottled tomato juice

1., & 5. Correct: A 3 ounce serving of pork loin contains approximately 54 mg of sodium. Slices of lemon, lime, or even oranges squeezed over a salad is low sodium (0-85 mg). 2. Incorrect: Canned entrees, and frozen dinners are high in sodium (Up to 1000 mg). 3. Incorrect: Instant puddings and cakes are high in sodium (1400 mg). 4. Incorrect: Pizza is high in sodium, particularly with meats such as pepperoni and ham (690 mg). 6. Incorrect: Bottled or canned tomato juice (980 mg), vegetable juice, mineral water, and softened water is high in sodium. Select coffee, tea, fruit juices, soft drinks, and low sodium tomato and vegetable juices. The average American adult's daily intake of sodium ranges from 7-15 grams per day. Clients diagnosed with heart failure are generally prescribed a 2 gm sodium diet. This is a drastic difference and education is imperative for the client to achieve this goal. All foods over 400 mg/serving of sodium should be avoided. Processed meats, cheese, bread, cereals, canned soups, and canned vegetables must be limited. Teach the client how to read labels to look for sodium content. The client should also be aware of the high sodium content in most restaurant foods. Fresh fruits, vegetables, lean meats, seafood are much better choices.

The community health nurse is planning to teach nutritional education to a group of adults attending a health fair. What tips about health eating should the nurse include? 1. Pay attention to fullness cues during meals. 2. Make one fourth of the plate fruits and vegetables. 3. Drink sweet tea rather than soft drinks with meals. 4. Eat foods low in dietary fiber. 5. Consume less than 30% of calories from saturated fatty acids. 6. Use a smaller plate for meals.

1., & 6. Correct: Pay attention to hunger and fullness cues before, during, and after meals. Use them to recognize when to eat and when you have had enough. Portion out foods before eating. A smaller plate will make the amount of food look larger. 2. Incorrect: Make half the plate fruits and vegetables. 3. Incorrect: Cut calories by drinking water or unsweetened beverages rather than drinks with sugar, such as soft drinks and sweet tea. 4. Incorrect: Diets should be high in fiber coming from fruits, vegetables, and whole grains. 5. Incorrect: Individuals should consume less than 10% of calories from saturated fatty acids (approximately 20 grams of saturated fat per day in a 2000 calorie diet).

The homecare nurse is instructing the family of a client recently diagnosed with Parkinson's disease about potential neurologic changes. During the discussion, what signs should the nurse include? 1. Unsteady gait 2. Muscle rigidity 3. Hyperactive reflexes 4. Bradykinesia (slowed movements) 5. Expressive aphasia

1., 2 & 4. Correct: Parkinson's disease is a debilitating, progressive neurological disorder of unknown cause. The most classic symptoms include unsteady gait secondary to increasing muscle rigidity and bradykinesia, plus difficulty with purposeful movement. These symptoms worsen over time and are often accompanied by tremors in the extremities at rest. 3. Incorrect: Reflexes in clients with Parkinson's disease become progressively slowed, not hyperactive. Because this disorder affects the midbrain, and ultimately the connection of the basal ganglia, deep tendon reflexes decrease over the course of the disease. Hyperactive reflexes are associated with other neurologic disorders such as multiple sclerosis. 5. Incorrect: Expressive aphasia is associated with brain trauma or cerebral vascular accident (CVA) and prevents the client from verbalizing appropriate or desired terminology. In Parkinson's disease, the client's speech volume becomes too low and very monotone. Also, because of facial muscle rigidity, there is great difficulty articulating words enough to be clearly understood.

A nurse is completing a nursing history on a client admitted with peripheral vascular disease (PVD). Which data from the nursing history should the nurse identify as contributing to this diagnosis? 1. Family history of hyperlipidemia 2. Postmenopausal 3. BMI of 24 4. Swims three times a week 5. Leg pain when walking

1., 2 & 5. Correct: A family history of hyperlipidemia, hypertension, or PVD increases the risk of a client developing PVD as well. Men over age 50 and postmenopausal women are at increased risk. A decline in the natural hormone estrogen may be a factor in heart disease increase among post-menopausal women. Estrogen is believed to have a positive effect on the inner layer of artery wall, helping to keep blood vessels flexible. Developing PVD risk also increases if the client has hyperlipidemia, cerebrovascular disease, heart disease, diabetes, hypertension, and/or renal failure. Leg pain with activity such as walking is a sign of PVD increases risk. 3. Incorrect: Overweight clients are at increased risk for PVD. A BMI of 24 means the client is of normal weight for height. Body mass index (BMI) is a measure of body fat based on height and weight that applies to adult men and women. Underweight = <18.5; Normal weight = 18.5-24.9; Overweight = 25-29.9; Obesity = BMI of 30 or greater. 4. Incorrect: Swimming three times per week is good exercise for the client. Sedentary life style increases the risk for development of PVD.

The nurse is teaching a community education course regarding complementary and/or alternative therapies. Which therapies would the nurse include in the course as complementary and/or alternative therapies? 1. Acupuncture 2. Yoga 3. Tai chi 4. Reiki 5. Zumba

1., 2, 3, & 4. Correct: All are considered complementary and/or alternative therapies. Acupuncture involves stimulating specific points on the body. This is most often done by inserting thin needles through the skin, to cause a change in the physical functions of the body. Research has shown that acupuncture reduces nausea and vomiting after surgery and chemotherapy. It can also relieve pain. The practice of yoga makes the body strong and flexible, and improves the functioning of the respiratory, circulatory, digestive, and hormonal systems. Yoga brings about emotional stability and clarity of mind. Tai chi is an ancient Chinese discipline involving a continuous series of controlled usually slow movements designed to improve physical and mental well-being. Reiki is a healing technique based on the principle that the therapist can channel energy into the client by means of touch, to activate the natural healing processes of the body and restore physical and emotional well-being. 5. Incorrect: Zumba is a type of dance exercise and is not considered a form of alternative therapy.

A nurse is planning to educate diabetic clients on how to decrease their risk for developing renal failure. What educational points should the nurse include? 1. Avoid daily use of non-steroidal antiinflammatory medications. 2. Aggressive blood pressure management is necessary. 3. Aim to keep Glycosylated Hemoglobin (HgbA1c) less than 7%. 4. Have estimated glomerular filtration rate measured every five years. 5. Increase protein intake to 30% of total calories eaten per day.

1., 2. & 3. Correct: NSAIDs can damage the kidneys with chronic use. Risk factors for diabetic related renal complications include hypertension and hyperglycemia; therefore, management of blood pressure and blood glucose is necessary. The ADA treatment goal for HgbA1c is < 7%. 4. Incorrect: The estimated glomerular filtration rate (eGFR) should be assessed at least yearly if not more frequently. 5. Incorrect: A diabetic client's diet should consist of no more than 15-20% caloric intake of protein because protein makes the kidneys work harder. First of all, we know that diabetic are at risk for developing renal failure, right? Right. High glucose levels and fluctuations will damage the glomerulus. So we know that we want the diabetic to keep their glucose under control. With that in mind which option deals directly with glucose? Option 3. Keep HgbA1c under 7% should be the goal. Now let's back up and look at option 1. Should the diabetic client take NSAIDS on a daily basis? No. NSAIDS alone can cause renal damage. Add diabetes and routinely taking NSAIDS will increase the risk of renal problems. Option 2, Monitor BP. True. We know that hypertension damages the kidneys. Again a diabetic who is hypertension increases the risk for kidney damage. Option 4, Test glomerular filtration rate every 5 years? This looks at renal function. Do you think every 5 years is adequate? No, it should be done at least yearly. Option 5, Increase protein intake to 30% of total calories eaten each day. A diabetic client's diet should consist of no more than 15-20% caloric intake of protein because protein makes the kidneys work harder.

The nurse manager is planning a leadership development workshop for new charge nurses. Which components of the communication cycle should the manager include as necessary for effective verbal communication? 1. There is a sender for every message. 2. A clear message is formulated. 3. There is a receiver for every message. 4. The sender and receiver share the same life experiences. 5. There can be incongruence between the verbal and nonverbal message.

1., 2. & 3. Correct: The communication cycle includes the sender, a clear and concise message, the receiver, plus verbal or nonverbal feedback to acknowledge understanding of the message. The sender is the person who delivers the message, and the receiver is the person who receives the message. 4. Incorrect: The sender and receiver may not share the same life experiences; however, therapeutic communication can still be achieved. The more the sender and receiver have in common and the closer the relationship, the more likely they will accurately perceive one another's meaning and respond accordingly. However, this is not required for effective verbal communication. 5. Incorrect: There should be congruence between verbal and nonverbal communication. Incongruency can lead to misunderstanding and miscommunication.

A pediatric nurse plans care for a child diagnosed with acute post-streptococcal glomerulonephritis. Which assessment findings should the nurse anticipate? 1. Edema 2. Proteinuria 3. Hematuria 4. Anasarca 5. Decreased urine specific gravity

1., 2. & 3. Correct: The nurse should anticipate that the child may be edematous and exhibit signs of proteinuria and hematuria. Glomerulonephritis results in a decrease in kidney filtration of the blood, which leads to fluid retention (edema), leakage of protein from the blood into the urine (proteinuria), and leakage of a large number of red blood cells into the urine. 4. Incorrect: Anasarca is total body edema. It is much greater than edema of the extremities. The entire body is edematous.This would be seen with nephrotic syndrome. 5. Incorrect: Urine specific gravity (density as compared to water) increases because of the inability of the kidney to filter out particulates.

A 37 week pregnant woman presents to triage with reports of a headache and begins to have a seizure. What actions should the nurse take? 1. Place the client's head in the nurse's lap. 2. Administer oxygen. 3. Monitor tonic-clonic activity. 4. Place an oral airway into the client's mouth. 5. Administer diazepam.

1., 2. & 3. Correct: This client in triage experiencing a seizure should be gently lowered to the floor, with her head protected. Oxygen is needed to ensure supply of oxygen to mom and fetus. Seizure activity should be monitored for tonic and clonic phases of seizure, timing, and body part affected. 4. Incorrect: Never place an object in a client's mouth who is experiencing a seizure. 5. Incorrect: Magnesium sulfate is administered to control BP and decrease seizures. Magnesium sulfate leads to fewer maternal deaths and fewer future seizures when given for eclamptic seizures. Diazepam is contraindicated for use in pregnancy. What actions do you take when a client has a seizure? The key is safety and protecting the clients from injury. In this case you have two clients, don't you? Yes. Mom and baby. So let's look at the options. Option 1: Place client's head in the nurse's lap. Sound like an odd thing to do but will this protect the client's head from injury? Yes it will. So this is true. This client in triage experiencing a seizure should be gently lowered to the floor, with her head protected. Option 2: Administer oxygen. During the seizure, is the fetus going to get enough oxygen? No. So administering oxygen will help mom and baby get enough oxygen. Option 3: Monitor the seizure activity. This is true. You want to be able to tell the primary healthcare provider about the seizure. You don't just want to say, "Oh, it was bad!" Seizure activity should be monitored for tonic and clonic phases of seizure, timing, and body part affected. Option 4: Place an oral airway into the client's mouth. FALSE. Never place an object in a client's mouth who is experiencing a seizure. Option 5: Administer diazepam. The clues in the stem tell you that the client is pregnant and is experiencing a headache prior to having a seizure. What do you think has caused this seizure? Eclampsia. What is the treatment? Magnesium sulfate is administered to control BP and decrease seizures. Magnesium sulfate leads to fewer maternal deaths and fewer future seizures when given for eclamptic seizures. Diazepam is contraindicated for use in pregnancy. The cure for eclampsia however, is delivery.

The client reports having trouble sleeping at night. "My mind is constantly working, and I can't fall asleep until 2:00 or 3:00 a.m."Which behaviors found in the assessment are likely to contribute to sleep difficulty? 1. Performs office work before going to bed. 2. Watches night-time drama shows on TV. 3. Drinks caffeine after dinner each evening. 4. Reads for pleasure before going to bed. 5. Exercises 45 minutes at 5 pm each evening.

1., 2. & 3. Correct: Working on job-related tasks before bedtime may increase anxiety and contribute to difficulty sleeping. Suspenseful night-time drama TV shows may be too stimulating prior to going to bed. Caffeine following dinner may interfere with sleep. 4. Incorrect: Quiet reading is likely to ease the transition from wakefulness to sleep and may be an important intervention to promote sleep. 5. Incorrect: Exercising early in the evening may be an effective intervention. If exercise is performed prior to going to bed, it may interfere with falling asleep.

The nurse is planning an educational seminar on ophthalmic health. Which risk factors for cataract formation should be included in the discussion? 1. Diabetes mellitus. 2. Cigarette smoking. 3. Family history of glaucoma. 4. Long-term use of corticosteroids. 5. Thin cornea.

1., 2. & 4. Correct: All these factors put a client at greater risk for development of cataracts. 3. Incorrect: A family history of glaucoma places a client at risk for the development of glaucoma, not cataracts. 5. Incorrect: Thin cornea is a risk factor for glaucoma, not cataracts. Option 1. True. People with diabetes are at increased risk for cataracts. Damage to the lens of the eye results from persistent high blood glucose levels. Option 2. True. People who smoke are more likely to develop cataracts. Smoking may damage the lens of the eye by leading to the formation of chemicals called free radicals. High levels of free radicals can damage cells, including those in the lens of the eye. Option 3. ​False. A family history of glaucoma does not increase a person's risk for cataract. However, surgery to treat glaucoma may raise the risk of cataracts.​ Option 4. True. Long-term use of high doses of steroid medicines for conditions such as asthma or emphysema increases a person's risk for cataracts. Option 5. False. Thin cornea is a risk factor for glaucoma, not cataracts.

The nurse has been teaching the parents of a child taking methylphenidate for the treatment of attention deficit hyperactivity disorder (ADHD). Which comments by the parents indicate adequate understanding of the important considerations for methylphenidate? 1. "I know that I need to monitor weight." 2. "I am supposed to call if my child has decreased attentiveness." 3. "This medication may cause increased drowsiness." 4. "I know that I need to monitor my childs height." 5. "If my child can't sleep, the dosage may need to be increased."

1., 2. & 4. Correct: Continued use of the medication may cause delays in growth and loss of appetite. Lack of appetite may cause weight loss. This drug may affect child's growth rate. The child's attentiveness should increase with this medication and if there is no improvement in attentiveness with this medication then notify the primary healthcare provider. 3. Incorrect: The medication is more likely to cause insomnia especially if administered late in the day. If this medication can cause insomnia. 5. Incorrect: If the client cannot sleep, it is likely that the afternoon dose will be decreased or omitted.

A home health nurse is interpreting Mantoux skin test results of clients who received the test 48 hours ago. Which clients have a positive tuberculin skin test reaction? 1. HIV+ client with an induration of 6 millimeters. 2. Client who immigrated from Haiti 6 months ago who has an induration of 10 millimeters. 3. Client working at a nursing home with an induration of 8 millimeters. 4. 3 year old client with an induration of 12 millimeters. 5. Healthy client with no known TB exposure who has an induration of 5 millimeters.

1., 2. & 4. Correct: HIV infected clients are considered to have a (+) TB skin test with an induration of 5 millimeters or more. An induration of 10 millimeters or more is considered positive in recent immigrants (less than five years) from high-prevalence countries such as Haiti, and in children less than 4 years of age. 3. Incorrect: An induration of 10 millimeters or more is considered positive for residents and employees of high-risk congregate settings. 5. Incorrect: An induration of 15 millimeters or more is considered positive in any person with no known risk factors for TB.

What physical changes should the nurse discuss with a client who is entering menopause? 1. Loss of bone density 2. Loss of muscle mass 3. Improved skin elasticity 4. A reduction in waist size 5. Increased fat tissue

1., 2. & 5. Correct: Changes associated with menopause, with its dramatic decline in estrogen, include loss of muscle mass, increased fat tissue leading to thicker waist, dryness of the skin and vagina, hot flashes, sleep abnormalities, and mood changes. 3. Incorrect: A decrease in turgor and elasticity may occur as we grow older. Skin becomes dry and thin and collagen levels decrease after menopause. 4. Incorrect: There is increased abdominal fat. The waist size swells relative to hips after menopause.

A 3-year-old child refuses to take a prescribed medication. Which statements by the mother, regarding the child's refusal, indicate to the nurse that parental education is needed? 1. "My child is trying to make me angry". 2. "I feel like such a bad mother when my child acts this way". 3. "I promise my child a reward for taking medicine". 4. "I am unfazed by my child's actions". 5. "My child doesn't have to take medicine if he doesn't want to".

1., 2. & 5. Correct: If the mother feels the child is trying to "make her angry" she may respond with inappropriate discipline. The nurse can help the mother understand that developing independence is one of the developmental tasks of a child this age, and that the movement toward independence reflects good, not bad, parenting. The child must take the prescribed medicine in order to get well. 3. Incorrect: Rewarding the client for taking medication is not contraindicated. 4. Incorrect: Being "unfazed" by the child's actions does not reflect bad parenting. Option 1 is true. The nurse needs to educate the parent. Option 2 is true. The nurse needs to educate the parent regarding a 3 year old's typical behavior. Option 3 is false, as further teaching would not be necessary if the parent made this statement. Option 4 is false, as further teaching would not be necessary if the parent made this statement. Option 5 is true. The child needs the medication or it would not have been prescribed.

A client in labor is placed on an external fetal monitor. Which interventions should the nurse perform if a late fetal heart rate deceleration occurs? 1. Turn the client to the left side. 2. Administer oxygen. 3. Start an intravenous line. 4. Prep the mother for cesarian section. 5. Notify the primary healthcare provider.

1., 2. & 5. Correct: Late fetal heart rate decelerations are associated with fetal hypoxia and acidosis. Positioning the mother on her left side prevents compression of the vena cava. Oxygen administration increases maternal, then fetal blood level, thus treating current and preventing further development of hypoxia and acidosis. Failure to recognize fetal monitoring strip abnormalities and failure to report abnormalities to the primary healthcare provider are deviations from the standard of care. 3. Incorrect: Intravenous line would already be in place. This will not help the fetus 4. Incorrect: Prepping the client for caesarian section is premature.

The nurse is developing a teaching plan covering emergency responses to smallpox. This presentation will be used with newly hired hospital employees. What information is essential for the presentation? 1. People may be exposed to smallpox but not get the disease. 2. People may contract the disease by handling contaminated clothing or bedding. 3. Smallpox is fatal is about 50% of cases. 4. Smallpox victims are contagious for two weeks. 5. Smallpox victims are isolated from others.

1., 2. & 5. Correct: Not everyone who is exposed will develop the disease. Handling contaminated bedding or clothing is one way to contract the illness. If a person comes in contact with the wound discharge, infection may occur. Persons must be isolated for approximately 18 days after an exposure, and persons with smallpox should be isolated until the last scab falls off. 3. Incorrect: Smallpox is fatal in 10 - 30% of victims. 4. Incorrect: Smallpox is considered contagious until the last scab falls off. There is no specific time frame to determine this and is different for each individual.

Which assessment findings would the nurse expect in a client diagnosed with Paget's disease? 1. Severe back pain 2. Walking with a limp 3. Upper extremity grip weakness 4. A shuffled gait 5. Difficulty hearing

1., 2. & 5. Correct: Paget's disease is a chronic skeletal bone disorder in which there is excessive bone resorption followed by the marrow being replaced by fibrous connective tissue. The new bone is larger, disorganized, and weak. These clients have severe pain, may walk with a limp, and may have a bowed leg look. Don't forget hearing is controlled by a bone so they may have hearing deficits. 3. Incorrect: Muscle weakness is not a symptom. 4. Incorrect: This is with Parkinson's disease related to brain and nerve signals.

In caring for a client exposed to radiation, the nurse knows that the type of damage due to radiation exposure depends on which factors? 1. Dose rate. 2. Organs exposed. 3. Type of tumor being treated. 4. Presence of metastatic disease. 5. Type of radiation.

1., 2. & 5. Correct: The extent of damage due to radiation exposure depends on the quantity of radiation delivered to the body, the dose rate, the organs exposed, the type of radiation, the duration of exposure, and the energy transfer from the radioactive wave or particle to the exposed tissue. 3. Incorrect: The type of tumor being treated is important to know, but this will not affect the type of damage the client receives from the radiation. 4. Incorrect: The client may be receiving radiation therapy for palliative treatment. Damage to the client due to the radiation exposure will not increase or decrease due to the metastatic disease. External radiation (or external beam radiation) uses a machine that directs high-energy rays from outside the body into the tumor.The extent of damage due to radiation exposure depends on the quantity of radiation delivered to the body, the dose rate, the organs exposed, the type of radiation, the duration of exposure, and the energy transfer from the radioactive wave or particle to the exposed tissue. Option 1. True. The higher the dose, the greater the risk of damage to exposed tissue. Option 2. True. It makes sense that organs exposed to the radiation will be damaged more than organs that have not been exposed. Right? Right. Option 3. False. The type of tumor does not determine how radiation affects tissue and organs. Rather it is the type and dose of the radiation, and what area is being radiated. Option 4. False. Same here as for option 3. It is the type and dose of the radiation, and what area is being radiated. Option 5. True. External radiation will cause different damage than internal radiation. So type of radiation is true.

Which interventions decrease risk of infection or damage to delicate tissue when the nurse is changing a wound dressing? 1. Warm cleansing solutions to body temperature. 2. Clean the wound when there is drainage present. 3. Use cotton balls to clean the suture site. 4. Use sterile gauze squares to dry the wound 5. Use sterile forceps when cleaning the wound.

1., 2. & 5. Correct: Using cleansing solutions at body temperature enhances the healing process by not lowering the temperature of the wound and enhancing circulation to the wound bed. Drainage should be removed so that it does not become infected because drainage and exudate can create an environment where bacteria can thrive. Sterile forceps should be used so that contaminated hands/gloves do not increase the risk of infection at the wound site. 3. Incorrect: Cotton balls may leave small cotton filaments behind that may serve as a site for infection. 4. Incorrect: Moisture is important for the healing process, so drying the wound could delay the healing process and cause undue harm to the client. Cells that are kept moist and hydrated promote epidermal growth. This will promote the healing of the cell base of the wound. Before we review the options, let's look at the question. The key words in a question should be identified. The key words in this question are decrease infection,damage, tissue, and wound dressing. This is a select all question so there will be 2 or more options correct. Also each option stands alone with the question. After reviewing the question, look at each option and identify if it is true or false. Remember client safety is always a priority. This question is talking about the correct procedure for changing a wound dressing. Remember your wound care principles. So let's look at the options. Option 1 is true. The nurse should use cleansing solutions at body temperature to increase the healing process by enhancing circulation to the wound bed. The vessels at the wound site would be constricted if the temperature of the cleansing solution was lower than the body temperature. Option 2 is true. Drainage should be removed so that it does not become infected because exudate and loose tissue create a good medium for for bacteria to thrive. A wound which is free exudate and damaged tissue heals faster. Options 3 is false. When cleaning with cotton balls, there maybe cotton threads that break loose.The cotton threads will decrease the healing process. Option 4 is false. Moisture is important for the healing process, so drying the wound could delay the healing process. Cells that continue to stay dehydrated will die. Option 5 is true. Any bacteria that enters the wound site has a higher risk of creating an infection. Utilizing sterile forceps when cleaning the wound will decrease the incidence of accidental entrance of bacteria into the wound.

A client's central venous pressure (CVP) reading has changed significantly from the last hourly reading. Which data would the nurse assess that reflect changes in the CVP reading? 1. Heart sounds 2. Skin turgor 3. Temperature 4. Nail bed color 5. EKG rhythm 6. Urinary output

1., 2. & 6. Correct: The CVP reading reflects the client's fluid volume status. If the CVP is elevated, indicating FVE, then the nurse is likely to hear S3 sounds when auscultating the heart sounds. The client's skin turgor and urine output would reflect the client's fluid volume status. 3. Incorrect: The CVP reading reflects the client's fluid volume status. The client's temperature would not reflect the client's fluid volume status. 4. Incorrect: The CVP reading reflects the client's fluid volume status. The nail bed color would not reflect the client's fluid volume status. 5. Incorrect: The CVP reading reflects the client's fluid volume status. The EKG rhythm would not reflect the client's fluid volume status. Option 1, heart sounds? Do heart sounds change with CVP changes? Yes, so this is true. The CVP reading reflects the client's fluid volume status. If the CVP is elevated, indicating FVE, then the nurse is likely to hear S3 sounds when auscultating the heart sounds.​ Option 2, Skin turgor. Would skin turgor be affected by CVP changes reflecting FVD? Yes, so True. Option 3, Temperature? What do you think? No, false. temperature changes do not occur with fluid volume changes. What do we assess temperature for? Infection. Option 4, nail bed color? False. The nail bed color is assessed for cardiac output and heart function. Option 5, EKG rhythm? Does this reflect fluid volume status? No, false. It looks at the function of the heart. Option 6, urinary output? Is output affected by fluid volume? Yes it is. If you are in FVE, urine output goes up. If you are in FVD, then urine output goes down.

The nurse has determined that a bedridden client diagnosed with a stroke is at risk for venous thromboembolism (VTE). What interventions should the nurse initiate? 1. Measure the calf and thigh daily. 2. Apply sequential compression device to legs. 3. Position paralyzed leg with each distal joint higher than the proximal joint. 4. Place a trochanter roll at the hip. 5. Perform passive range of motion exercises once daily. 6. Monitor for pain by assessing Homan's sign.

1., 2., & 3. Correct: Assessment for VTEs is accomplished by measuring the calf and thigh daily, observing swelling, noting unusual warmth of the leg, and asking the client about pain in the calf. Prevention of VTEs include the use of sequential compression devices for bedridden clients. This device helps promote venous return. Positioning the paralyzed leg with each distal joint higher than the proximal joint will prevent dependent edema. 4. Incorrect: A trochanter roll does not prevent VTEs. They are used for the prevention of external hip rotation. 5. Incorrect: Passive range of motion exercises should be done several times a day to promote venous return and muscle tone. Once a day is not adequate. 6. Incorrect: Routinely checking the Homan's sign can actually cause a clot to dislodge. The nurse should not perform this procedure.

What should the nurse include about transmission of the chickenpox virus while teaching a group of parents about the importance of vaccination? 1. Direct contact 2. Indirect contact 3. Airborne 4. Droplet 5. Common vehicle

1., 2., & 3. Correct: Chickenpox is transmitted from person to person by directly touching the blisters, saliva or mucus of an infected person (Direct contact). Chickenpox can be spread indirectly by touching contaminated items freshly soiled, such as clothing, from an infected person (Indirect contact). The virus can also be transmitted through the air by coughing and sneezing (Airborne). Airbone transmission of infectious agents occurs either by: Airborne droplet nuclei (small particles of 5 mm or smaller in size); Dust particles containing infectious agents. Microoganisms carried in this manner remain suspended in the air for long periods of time and can be dispersed widely by air currents. Because of this, there is risk that all the air in a room may be contaminated. Some examples of microorganisms that are transmitted by the airborne route are: M. tuberculosis, rubeola, varicella, and hantaviruses. 4. Incorrect: Transmission occurs when droplets containing microorganisms generated during coughing, sneezing and talking are propelled through the air. However, these infected droplets may linger on surfaces for long periods of time, so these surfaces (within the range of the coughing/sneezing person) will need additional cleaning. 5. Incorrect: Applies to microorganisms that are transmitted by contaminated items such as food, water, medications, medical devices, and equipment. Option 1: Direct contact. This is true. Chickenpox is transmitted from person to person by directly touching the blisters, saliva or mucus of an infected person (Direct contact). Option 2: Indirect contact. This is true. Chickenpox can be spread indirectly by touching contaminated items freshly soiled, such as clothing, from an infected person (Indirect contact). Option 3: Airborne. This is also true. You see, some organisms can be transmitted in more than one way, such as with chickenpox. This virus can be transmitted through the air by coughing and sneezing (Airborne). Airbone transmission of infectious agents occurs either by: Airborne droplet nuclei (small particles of 5 mm or smaller in size); Dust particles containing infectious agents. Microoganisms carried in this manner remain suspended in the air for long periods of time and can be dispersed widely by air currents. Because of this, there is risk that all the air in a room may be contaminated. Some examples of microorganisms that are transmitted by the airborne route are: M. tuberculosis, rubeola, varicella, and hantaviruses. Option 4: Droplet. This is false. Droplet transmission occurs when bacteria or viruses travel on relatively large respiratory droplets that people sneeze, cough, drip, or exhale. They travel only short distances before settling, usually less than 3 feet. Option 5: Common vehicle. This is false. This type of transmission applies to microorganisms that are transmitted by contaminated items such as food, water, medications, medical devices, and equipment.

What potential contributing factors for stress urinary incontinence should a nurse assess for in an elderly female client? 1. Lack of estrogen 2. Rising abdominal pressure 3. Multiparous vaginal births 4. Spinal cord injury 5. Dementia

1., 2., & 3. Correct: During pregnancy and childbirth, the sphincter and pelvic muscles stretch out and are weakened. Increased age, decreased estrogen, and a history of multiple vaginal births/pregnancies are contributing factors for stress incontinence. 4. Incorrect: Spinal cord injury results in urge incontinence because of damage to the nerves of the bladder. Urge incontinence means there is a sudden, involuntary contraction of the muscle wall. 5. Incorrect: With functional incontinence the person knows there is a need to urinate but cannot make it to the restroom. The dementia client cannot make the conscious decision or carry out the task of ambulating to the restroom.

A nurse plans to educate a group of new parents about how to prevent burn injuries in children. What should the nurse include? 1. Eliminate use of placemats. 2. Establish "no" zones for space heaters. 3. Cover unused electrical outlets. 4. Warm baby bottle in microwave for 30 seconds. 5. Set the hot water heater thermostat to 140°F (60°C).

1., 2., & 3. Correct: Placemats and tablecloths can be pulled down by children. If something hot is sitting on it, the child can be scalded. The parents should be taught to block access to stove, fireplace, space heaters, and water heaters. They need to be inaccessible to small children. Covering unused electrical outlets will prevent a child from sticking things, such as a fork, in it which could result in an electrical burn. 4. Incorrect: The parents should not use microwave at all for warming the bottle. Food and liquids can heat unevenly and burn the child. 5. Incorrect: Hot water heater thermostats should be set to below 120°F (48.9°C). Bath water should be around 100°F (38°C) to prevent burn injuries with children. The water should be tested before allowing the child to step into the bath also.

A nurse from the maternity unit is pulled to the medical-surgical unit for the first four hours of the shift. Which clients would be appropriate for the charge nurse to assign to the nurse from the maternity unit? 1. Client with rheumatic fever 2. Client scheduled for an appendectomy 3. Client one day post cardiac catheterization 4. Client diagnosed with Methicillin-Resistant Staphylococcus Aureus 5. Client newly admitted with Guillian-Barre Syndrome

1., 2., & 3. Correct: Rheumatic fever is an inflammatory disease that can develop later as a complication of untreated or inadequately treated Group A beta hemolytic strep infections such as strep throat and scarlet fever. It is not contagious at this point, so the maternity nurse could be assigned to this client. The maternity nurse should have the knowledge and skill needed to provide preop care to a client scheduled for a routine appendectomy.This nurse routinely cares for preop surgical clients on the maternity unit. Since the client who had a cardiac catheterization is one day post procedure, the client is stable and could be appropriately assigned to the maternity nurse if needed. 4. Incorrect: This client is contagious and should not be delegated to the maternity nurse. The nurse will be going back to the maternity unit after four hours and will be a potential agent for spreading the infection. Client safety could be compromised. 5. Incorrect: Although the client with Guillian-Barre Syndrome is not contagious, this is a newly admitted client with a complex, possibly life threatening condition. The maternity nurse may not have the knowledge and assessment skills needed to care for this client. This would not be an appropriate assignment for this nurse. In this question a maternity nurse is pulled to a medical surgical unit for four hours and will then return to the maternity unit. This nurse specializes in maternity nursing so consider her a brand new nurse on the med surg unit. Do not assign this nurse unstable clients. Also, keep in mind that the nurse will be going back to care for new moms and babies. Think safety and infection control! Let's look at the options. Option 1 is a client with rheumatic fever. Fever scared you didn't it? But is this disease contagious? No. Rheumatic fever is an inflammatory disease that develops as a complication of untreated or inadequately treated Group A beta hemolytic strep infections such as strep throat and scarlet fever. It is not contagious at this point, so the maternity nurse could be assigned to this client. Option 2 is a client scheduled to have an appendectomy. The maternity nurse should have the knowledge and skill needed to provide preop care to a client scheduled for a routine appendectomy.This nurse routinely cares for preop surgical clients on the maternity unit. Option 3, a client one day post cardiac cath? Since the client who had a cardiac catheterization is one day post procedure, the client is stable and could be appropriately assigned to the maternity nurse if needed. Look at option 4, the client diagnosed with MRSA? Are you going to assign the maternity nurse to this client for four hours then send her back to moms and newborns? I hope not. This client is contagious and should not be delegated to the maternity nurse. The nurse will be going back to the maternity unit after four hours and will be a potential agent for spreading the infection. Client safety could be compromised. What about option 5, the new admit client diagnosed with Guillian-Barre Syndrome. Is this client contagious? No, but the client is a new admit with a complex, possibly life threatening condition. The maternity nurse may not have the knowledge and assessment skills needed to care for this client. This would not be an appropriate assignment for this nurse.

A palliative care client is suffering from persistent diarrhea. What foods should the nurse suggest? 1. Applesauce 2. Rice 3. Bananas 4. Tea 5. Yogurt

1., 2., & 3. Correct: The BRAT diet is recommended for clients with persistent diarrhea. This diet consists of bananas, rice, applesauce, and toast. Rice and potatoes help to reduce diarrhea. Bananas will help replace potassium. Once the diarrhea subsides, the client can add easily digestible foods like eggs. 4. Incorrect: Avoid coffee and tea because caffeine containing beverages may have a laxative effect. Caffeine is a stimulant and will increase the peristalsis even more. 5. Incorrect: Dairy products may make the diarrhea worse. Avoid these until the diarrhea subsides.

The palliative care nurse is instructing the family of a client who is experiencing nausea and vomiting on methods of controlling these symptoms. What methods should the nurse include? 1. Offer electrolyte replacement drinks or broths. 2. Avoid cooking close to the client 3. Provide light, bland food. 4. Drink liquids less often 5. Chew 5-30 paw paw seeds

1., 2., & 3. Correct: These are all methods that can help control n/v symptoms. Sports drinks and broths can help with hydration. Juices and soft drinks should be avoided. Smells from foods cooking can lead to nausea and vomiting. Bland foods in small portions may be tolerated vs. fried or heavy foods. 4. Incorrect: The client should drink small amounts of liquid more often. If tolerated, fluids will help prevent dehydration. Avoid milk products and sugary drinks as they will increase nausea and loss of fluids. 5. Incorrect: Paw paw seed is an herb that can be used for constipation. The question is not related to relieving constipation. It is related to nausea and vomiting prevention/control.

The nurse is working with a LPN/VN and an unlicensed assistive personnel (UAP). Which client would be appropriate for the nurse to assign to the LPN/VN? 1. In Bucks traction requiring frequent pain medication 2. 24 hours post appendectomy 3. Diagnosed with cholelithiasis and scheduled for surgery in the AM 4. Admitted 6 hours ago in adrenal insufficiency 5. In diabetic ketoacidosis receiving IV insulin

1., 2., & 3. Correct: These are stable clients that can be assigned to the LPN. The LPN can provide medications for pain management. Since the postop client is not requiring frequent assessments and is considered stable at this point, the RN can assign the LPN to care for this client. The client having surgery in the AM is stable and will require predictable preop care the evening prior to surgery, so the LPN can care for this client as well. 4. Incorrect: This client has adrenal insufficiency. It occurs when at least 90 percent of the adrenal cortex has been destroyed. As a result, often both glucocorticoid (cortisol) and mineralocorticoid (aldosterone) hormones are lacking. This puts the client at risk for fluid volume deficit and shock. This would require higher level knowledge and skills of the RN and should not be delegated to the LPN. 5. Incorrect: The client in diabetic ketoacidosis is not considered a stable client. The administration of IV insulin is outside the scope of practice for the LPN. Caring for this client would require higher level assessment skills, knowledge, and nursing care that is within the RNs scope of practice. This client should not be assigned to the LPN.

Which signs/symptoms should the nurse assess for in the client admitted with a diagnosis of myasthenia gravis? 1. Difficulty holding head erect 2. Limited facial expressions 3. Ptosis 4. Hemiparesis 5. Writhing, twisting movements of the body 6. Pill rolling

1., 2., & 3. Correct: These are three of the primary symptoms of myasthenia gravis. The muscles of the head and neck are weak and have difficulty holding the head up. Facial paralysis occurs and drooping of the eyelids develops as the client's muscles get tired. 4. Incorrect: Hemiparesis is a type of physical condition with weakness on one side of the body. 5. Incorrect: Writhing, twisting movements of the face, limbs, and body is known as chorea and is seen in Huntington's disease. 6. Incorrect: The hand tremor described as "pill rolling" is a sign of Parkinson's disease. The thumb and forefinger appear to move in a rotary fashion as if rolling a pill, coin, or other small object.

A hiker that was lost in the mountains for 3 days experienced exposure to below freezing temperatures. Upon arrival to the emergency department (ED), the nursing assessment reveals hard, mottled, bluish-white toes bilaterally, and the client reports being unable to feel the toes. Which actions should the nurse initially take? 1. Remove any wet or constricting clothing. 2. Initiate a controlled and rapid rewarming process with warm water. 3. Wrap each toe individually with sterile gauze. 4. Encourage the client to walk. 5. Apply a heating pad to the feet. 6. Massage the frozen digits.

1., 2., & 3. Correct: Wet clothing is removed to eliminate continued exposure to the cold and allow the warming process to begin. Swelling is common so anything, such as clothing or jewelry that could cause constriction to blood flow should be removed. A controlled and rapid re-warming process is accomplished using a continuous flow of warm water until flushing is noted in the affected areas. Antiseptics or antibiotics are often used, and each digit is wrapped individually with sterile gauze (not constricting) to minimize the risk of infection and assist in the warming process. The core should be re-warmed first to prevent "afterdrop" which is a further drop in core temperature caused by cold peripheral blood returning to the central circulation. 4. Incorrect: Movement of frostbitten areas can cause ice crystals to form in the tissue and cause further damage. In addition, lack of sensation places the client at risk for falls or other injury. 5. Incorrect: External heat such as heating pads, fireplaces, etc. should not be used because burns are more likely to result due to the presence of decreased sensation in the affected areas. 6. Incorrect: Initial rubbing or massage of the frostbitten digits is an absolute contraindication as it can cause further tissue damage. Gentle handling is required to prevent stimulation of the cold myocardium.

What signs and symptoms of ovarian cancer should a nurse include when educating women? 1. Urinary frequency. 2. Menorrhagia with breast tenderness. 3. Watery vaginal discharge. 4. Increasing abdominal girth. 5. Fullness after a heavy meal.

1., 2., & 4. Correct: Signs and symptoms of ovarian cancer include irregular menses, increasing premenstrual tension, menorrhagia with breast tenderness, early menopause, abdominal discomfort, dyspepsia, pelvic pressure and urinary frequency. Flatulence, fullness after a light meal, and increasing abdominal girth are significant symptoms. 3. Incorrect: Watery, vaginal discharge is a sign of advanced cervical cancer. 5. Incorrect: A sense of fullness occurs after ingesting a light meal.

Which tasks can the RN safely delegate to an LPN/LVN when caring for a client scheduled for an adrenalectomy? 1. Check fingerstick glucose level. 2. Administer regular insulin SQ based on sliding scale prescription. 3. Assess client's cardiac rhythm. 4. Reinforce teaching regarding postoperative care. 5. Review client's pre-surgical laboratory values.

1., 2., & 4. Correct: The LPN/LVN can perform these tasks and can reinforce teaching. 3. Incorrect: The RN must assess, evaluate, and teach. The LPN/LVN can collect data to assist the RN, but the RN must validate that the data is correct. 5. Incorrect: The RN must assess, evaluate, and teach. The LPN/LVN can collect data to assist the RN, but the RN must validate that the data is correct.

What should the nurse include in a discharge plan for a client diagnosed with lymphoma who will be receiving outpatient treatment? 1. Avoid uncooked meats, seafood or eggs and unwashed fruits and vegetables. 2. Take bleeding precautions. 3. Do not take influenza or pneumonia vaccine during treatment. 4. Avoid individuals with infections. 5. Emphasize importance of frequent oral hygiene with an alcohol based mouthwash.

1., 2., & 4. Correct: The client with lymphoma is susceptible to infection and should eat foods low in bacteria. The client should avoid uncooked meats, seafood or eggs and unwashed fruits and vegetables as the bacteria count will be higher than desired. Instruct client and family about bleeding precautions and management of active bleeding due to thrombocytopenia. They should be advised to avoid activities that place them at risk for injury or bleeding (including excessive straining). This client is at risk for infection due to low white count, so the client should avoid individuals who are ill. 3. Incorrect: Encourage clients to maintain current immunizations for influenza and pneumonia. They are more susceptible to infection. Cancer and cancer treatment can weaken the immune system, which puts them at higher risk of serious problems if they get the flu or pneumonia. Only live vaccines (MMR, varicella, oral polio) are contraindicated in clients receiving chemotherapy. 5. Incorrect: This client is at risk for bleeding and infection due to low platelet and white cell counts. The client needs frequent oral care with a soft toothbrush and alcohol free mouthwash. Alcohol-based mouthwashes can dry out the gum and increase bleeding.

What actions should the nurse take when administering fentanyl? 1. Remove old fentanyl patch prior to applying new patch. 2. Cleanse area of old fentanyl patch. 3. Shave hair where fentanyl patch will be applied. 4. Place fentanyl patch over dry skin. 5. Apply adhesive dressing over the fentanyl patch. 6. Dispose of fentanyl patch in trash.

1., 2., & 4. Correct: These are correct actions. Apply patch to dry, hairless area of subcutaneous tissue, preferably the chest, abdomen, or upper back. The old patch should be removed prior to applying a new patch so that too much medication is not given. This is also why the old site should be cleaned. The patch should be placed on dry skin. Do not place over emaciated skin, irritated or broken skin, or edematous skin. 3. Incorrect: Do not shave area where patch will be applied and do not apply over dense hair areas. If there is hair on the skin, clip the hair as close to the skin as possible, but do not shave. 5. Incorrect: Do not apply adhesive dressing over patch. It can interfere with absorption. If the patch comes loose, you may tape the edges and remove and apply a new patch. 6. Incorrect: Dispose of fentanyl patch in sharps container. Fentanyl patches that have been worn 3 days still contain enough medication to cause serious harm to adults and children.

What should the nurse include in the teaching plan for a client who has iron deficiency anemia? 1. Consume iron rich foods such as dried lentils, peas, and beans. 2. Notify primary healthcare provider of glossitis, anorexia, and paresthesia. 3. Iron is needed for white blood cell development. 4. Educate about ferrous sulfate supplement. 5. After drinking liquid iron, follow immediately by water.

1., 2., & 4. Correct: These are examples of iron rich foods. Foods high in iron will help with correcting iron deficiency anemia. Glossitis, anorexia, and paresthesias can result from iron deficiency anemia. Foods high in vitamin C such as citrus fruits, dark green leafy vegetables and strawberries help with absorption. 3. Incorrect: Iron is needed for red blood cell development and oxygen transport to the cells. Iron is not needed for white blood cell development. White blood cell development. White blood cells are produced in the bone marrow. 5. Incorrect: Clients should dilute liquid iron with water or juice, drink with a straw, and rinse mouth after swallowing. Iron will stain the teeth.

A 19 year old client preparing to enter college asks the clinic nurse about immunizations. What immunizations should the nurse suggest the client discuss with the primary health care provider? 1. Meningococcal conjugate vaccine 2. Tdap vaccine 3. HPV vaccine 4. Seasonal flu vaccine 5. Hepatitis B 6. Polio

1., 2., 3., 4, & 5. Correct: These vaccine are specifically recommended for young adults ages 19-24. Meningococcal conjugate vaccine is recommended as it protects against bacterial meningitis. It is required for students living in a dorm. Tdap vaccine protects against tetanus, diphtheria, and pertussis. HPV vaccine protects against the human papillomavirus, which causes most cases of cervical and anal cancers, as well as genital warts. Seasonal flu vaccine is recommended. Hepatitis B is a blood-born infection, which can also be transmitted through sexual activity. 6. Incorrect: The inactivated polio (IPV) vaccine is a 4-dose series given during early childhood. IPV is not routinely recommended for children aged 18 years or older.

A mother brings her 6 week old infant to the ED and reports that the baby isn't gaining weight, and has not wet a diaper in 12 hours. The baby vomits after every feeding. Which nursing interventions would help this infant? 1. Upright position with feedings and at night 2. Small frequent feedings that are thickened 3. Supine position for sleeping 4. Administration of H2 blockers 5. Give Pedialyte only until vomiting stops

1., 2., & 4. Correct: These interventions will promote stomach emptying and prevent gastric reflux. 3. Incorrect: First, did you recognize that this infant has Gastric Reflux? And the ideal position for sleeping with Gastric reflux is HOB elevated 30 degrees and the prone position. Y'all elevated and prone will help to improve stomach emptying, not supine. 5. Incorrect: We definitely don't want to give this baby thin liquids, they need thickened feedings. This is really important to decrease chance of aspiration.

A charge nurse is observing a new nurse for proper use of standard precautions for infection control. Which actions indicate that standard precautions are being followed? 1. Wearing clean gloves to convert an IV to a saline loc 2. Donning sterile gloves for a cesarean dressing change 3. Wearing a N95 respirator while caring for a child who has respiratory syncytial virus (RSV) 4. Putting on a gown to take care of a client who has toxoplasmosis 5. Performing hand hygiene after removing gloves

1., 2., & 5. Correct: Clean gloves are needed when there is a chance of coming into contact with blood or body fluids, which is likely to happen when converting an IV to a saline loc. Dressing changes in the hospital are a sterile procedure and require the use of sterile gloves. Hand hygiene should be performed before and after contact with a client immediately after touching blood, body fluid, non-intact skin, mucous membranes, or contaminated items. It should also be performed after removing gloves, before eating, and after using the restroom. 3. Incorrect:The N95 respirator is used with airborne precautions and RSV requires droplet precautions. 4. Incorrect: Toxoplasmosis is transmitted through the feces of infected cats or through ingestion of raw or rare meats. A gown is not required in the care of this client because transmission occurs through ingestion of the parasites.

A nurse is planning to conduct primary prevention classes in a local community. Which initiatives should the nurse include? 1. Parenting classes for first time parents 2. Healthy diet classes for school-age children 3. Breast self-examination classes 4. Cardiac rehabilitation classes 5. Community exercise classes to promote weight loss

1., 2., & 5. Correct: Primary prevention is aimed at reducing the incidence of mental or physical disorders within the population. Primary prevention targets individuals and the environment. Emphasis is on assisting individuals to increase their ability to cope effectively with stress and targeting and diminishing harmful forces (stressors) within the environment. Teaching parenting skills and child development to prospective new parents, teaching healthy eating to school-age children, and promoting weight loss through exercise classes are all examples of primary prevention. 3. Incorrect: This is an example of secondary prevention (initiatives that work to identify and detect disease in its early stages). 4. Incorrect: This is an example of tertiary prevention (designed to restore self-sufficiency and to limit complications and disabilities associated with a disease state, such as after an MI). Before we review the options, let's look at the question. The key words in a question should be identified. The key words in this question are primary prevention classes. This is a select all question so there will be 2 or more options correct. Also each option stands alone with the question. After reviewing the question, look at each option and identify if it is true or false. This question is focusing on primary prevention class. So let's look at the options. Option 1 is true. Primary prevention is aimed at reducing the incidence of mental or physical disorders within the population. Primary prevention targets individuals and the environment. Teaching parenting classes to first time parents is an example of a primary prevention class. Option 2 is true. Teaching children to chose healthy food choices is an example of a primary prevention class. A primary prevention class strives to prevent the children from being obese or developing a disease from unhealthy food choices. Option 3 is true. This is an example of secondary prevention which identifies and detects disease in its early stages.. The client is performing a self breast examination to identify the presence of a breast mass. Option 4 is false.Cardiac rehabilitation classes are organized to assist client's manage their current chronic illness or injury. This class is a tertiary prevention class. Option 5 is true. Encouraging weight loss through exercise classes are all examples of primary prevention because the goal is to prevent weight gain.

The nurse is observing a new nurse inserting a nasogastric (NG) tube. Which action by the student nurse needs to be corrected by the nurse? 1. Measures from the tip of the nose to the xiphoid process of the client. 2. Lubricates the NG tube with petroleum gel. 3. Aspirates the NG tube to test gastric contents with a pH stip. 4. Marks the tubing at measurement mark with tape and secures to nose. 5. Places tube end into a glass of water to assess for bubbling.

1., 2., & 5. Correct: These actions by the new nurse are not done properly. The measurement for tube placement should be nose to ear and then xiphoid process. Lubricate the tube with a water solution, not a petroleum gel. Never place the tube in water because if the tube is in the trachea, the client can aspirate the water into the lungs. 3. Incorrect: This is the proper technique for checking placement of the NG tube. The pH should be less than 5 if in the stomach. 4. Incorrect: Yes, the tubing should be marked with a piece of tape and secured to the nose with tape or a commercial device if available.

A community health nurse, participating in a health fair, is educating a community group about risk factors for developing varicose veins. What risk factors should the nurse include? 1. Sitting for prolonged periods 2. Obesity 3. Female 4. Leg exercises 5. Wearing high-heeled shoes

1., 2., 3, & 5. Correct: These are all risk factors for developing varicose veins: Sitting or standing for prolonged periods of time, obesity, female gender, wearing high-heeled shoes. 4. Incorrect: Exercise is good for preventing varicose veins. Get moving. Walking is a great way to encourage blood circulation to the legs. Varicose veins are swollen, twisted veins that you can see just under the skin. They usually occur in the legs, but also can form in other parts of the body. For example, hemorrhoids are a type of varicose vein. Veins have one-way valves that help keep blood flowing toward the heart. If the valves are weak or damaged, blood can back up and pool in the veins. This causes the veins to swell, which can lead to varicose veins. The risk of developing varicose veins increase with age, being female, obesity, lack of exercise or a family history. They can also be more common in pregnancy. Calf muscles contract less in women wearing high-heeled shoes than those wearing flat bottom shoes. Less blood is pushed out of the leg when the calf muscles do not contract. This causes the venous blood pressure to increase and can stress the valves in the veins. Like crossing the legs, wearing heels can contribute to the overall venous reflux disease problem. Exercising, losing weight, elevating the legs when resting, and not crossing them when sitting can help keep varicose veins from getting worse. Wearing loose clothing and avoiding long periods of standing can also help.

A client who has diabetes calls the nurse hot-line reporting shakiness, nervousness, and palpitations. Which questions would yield information that would help the nurse decide that this is a hypoglycemic episode? 1. What have you eaten today and at what times? 2. Are you using insulin as a treatment of diabetes, and if so, what kind? 3. Do you feel hungry? 4. Do you have access to a glucose monitor to check your current glucose level? 5. Does your skin feel hot and dry?

1., 2., 3. & 4. Correct. This question will give the nurse information about how much time has elapsed since the last meal and will indicate the amount of protein and carbohydrates consumed at the last meal. Even a minor delay in meal times may result in hypoglycemia. Insulin type will give the nurse information about duration of action and peak time. Hunger is a symptom of hypoglycemia.If the client has a glucose monitor, an accurate reading would give the nurse valuable information about how much food the client should consume now. 5. Incorrect. Hot and dry skin is not an indicator of hypoglycemia and would not help the nurse determine if the client is experiencing a hypoglycemic episode. Cool, clammy skin is a symptom of hypoglycemia. This question wants the nurse to verify that the client on the phone is having a hypoglycemic episode. So what questions could the nurse ask to verify this diagnosis? Option 1. Is it important to know if the client has eaten and what time? Yes. If the client has not eaten, would that lead to hypoglycemia? Yes. So this is true. Option 2. True. We know that insulin does what to glucose? Decreases it. Option 3. True. Hunger is a symptom of hypoglycemia. Option 4. Is it important to know if the client can check to see what their glucose level is? Yes, true. Option 5. Does the skin get hot and dry with hypoglycemia? No. It gets cool and clammy doesn't it? Yes, so this statement is false.

A nurse is developing a proposal to implement a pet therapy program at a nursing home. What information should the nurse include in the proposal to support this program? 1. Evidence has shown that animals can directly influence a person's mental and physical well-being. 2. Bringing a pet into a nursing home for the elderly has been shown to enhance social interaction. 3. Petting an animal can be helpful in lowering a client's blood pressure. 4. Some researchers believe that animals actually may retard the aging process among those who live alone. 5. Nursing home clients are more submissive after petting an animal.

1., 2., 3. & 4. Correct: All of these statements are correct in reference to pet therapy programs. Petting a dog or cat has been shown to lower blood pressure. Studies indicate a 7 mm Hg drop in systolic and an 8 mm Hg decrease in diastolic BP when volunteers talked to or would pet their dogs as opposed to reading aloud or resting quietly. 5. The client's are not more submissive or passive after participating in pet therapy. Evidence does show increased mental and physical well being with pet therapy. Pet therapy involves any interaction that clients have with animals to make them feel better. Generally, studies show that pet therapy leads to positive outcomes. You must know about pet therapy to answer this question. Then remember with SATA questions to read each option as a true or false statement. Option 1: Yes. This is a general statement about pet therapy and it is true. Research shows that there is a positive relationship between pet therapy and improved mental and physical well being. Option 2: Good choice. Bringing a pet into the nursing home will increase the elderly client's social interaction. Social interaction involves reciprocal stimulation or response among people. Option 3: True statement! Studies show there is a positive connection between lowering of BP and pet therapy. Option 4: Yes. Research shows that there is a positive correlation between pet therapy and slowing of the age process. Option 5: No! There is nothing in research to support pet therapy and the elderly client becoming more submissive. False statement.

A nurse is developing a proposal to implement a pet therapy program at a nursing home. What information should the nurse include in the proposal to support this program? 1. Evidence has shown that animals can directly influence a person's mental and physical well-being. 2. Bringing a pet into a nursing home for the elderly has been shown to enhance social interaction. 3. Petting an animal can be helpful in lowering a client's blood pressure. 4. Some researchers believe that animals actually may retard the aging process among those who live alone. 5. Nursing home clients are more submissive after petting an animal.

1., 2., 3. & 4. Correct: All of these statements are correct in reference to pet therapy programs. Petting a dog or cat has been shown to lower blood pressure. Studies indicate a 7 mm Hg drop in systolic and an 8 mm Hg decrease in diastolic BP when volunteers talked to or would pet their dogs as opposed to reading aloud or resting quietly. 5. The client's are not more submissive or passive after participating in pet therapy. Evidence does show increased mental and physical well being with pet therapy. Pet therapy involves any interaction that clients have with animals to make them feel better. Generally, studies show that pet therapy leads to positive outcomes. You must know about pet therapy to answer this question. Then remember with SATA questions to read each option as a true or false statement. Option 1: Yes. This is a general statement about pet therapy and it is true. Research shows that there is a positive relationship between pet therapy and improved mental and physical well being. Option 2: Good choice. Bringing a pet into the nursing home will increase the elderly client's social interaction. Social interaction involves reciprocal stimulation or response among people. Option 3: True statement! Studies show there is a positive connection between lowering of BP and pet therapy. Option 4: Yes. Research shows that there is a positive correlation between pet therapy and slowing of the age process. Option 5: No! There is nothing in research to support pet therapy and the elderly client becoming more submissive. False statement.

The nurse is caring for a client with cirrhosis of the liver and suspects that the client may be developing hepatic encephalopathy. Which assessments by the nurse suggest that the client is developing this complication? 1. Asterixis 2. Lethargy 3. Amnesia 4. Behavioral changes 5. Kussmaul respirations

1., 2., 3. & 4. Correct: Hepatic encephalopathy results in changes in neurologic and mental responsiveness due to the accumulation of ammonia. All of the correct options are either mental or neurologic changes. 5. Incorrect: Kussmaul respirations are not a characteristic of hepatic encephalopathy. They are seen in diabetic ketoacidosis (DKA). Hepatic encephalopathy results in changes in neurologic and mental responsiveness due to the accumulation of ammonia. So what options will result in neuro and mental changes due to ammonia? In order to answer this, you also have to know what these terms mean, don't you. So let's look at each option. Option 1: Asterixis is a hand-flapping tremor, often accompanying metabolic disorders. The tremor is usually induced by extending the arm and dorsiflexing the wrist. Asterixis is seen frequently in hepatic encephalopathy. So this option is true. Option 2: lethargy. If you are full of ammonia or toxins, what would happen to your LOC? Decrease right? Right. So true. Option 3: Amnesia, partial or total memory loss. Again this is seen because of the buildup of toxins in the body. Option 4: Behavioral changes. The brain does not like it when it is full of toxins. Option 5: False. Kussmaul respirations are not a characteristic of hepatic encephalopathy. They are seen in diabetic ketoacidosis (DKA).

The homecare nurse is instructing a client with chronic obstructive pulmonary disease (COPD) about the importance of a nutritious diet to avoid weight loss. The nurse knows that teaching has been effective when the client selects which foods for a breakfast menu? 1. Scrambled eggs 2. Cheese omelet 3. Sliced banana 4. Orange juice 5. Whole milk 6. Dry toast

1., 2., 3. & 4. Correct: Maintaining weight and nutrition is vital to the health of clients with (COPD). Extreme fatigue along with excessive mucus production decreases the client's ability to eat complete, well-balanced meals, leading to weight loss or malnourishment. Therefore, the nurse would instruct the client to eat small, frequent meals high in protein and fiber. Good sources of protein include eggs, cheese, fish and poultry, beans and even nuts. Fresh fruit such as bananas along with non-carbonated beverages such as orange juice are excellent breakfast food choices. 5. Incorrect: Although milk and dairy products like yogurt could be considered part of a healthy breakfast, it is recommended that COPD clients use 1% or 2 % milk products to avoid increasing mucus production. This client should select the orange juice from the choices provided. 6. Incorrect: Dry toast provides little nutrient value, and may actually increase coughing because of its brittle nature. Coughing quickly leads to exhaustion rather than eating. This client would benefit more from a more palatable choice such as muffin or French toast.

The nurse is providing foot care to the client who has diabetes. The nurse teaches the client proper care of the feet. What should the nurse include in the teaching? 1. Inspect the feet daily for abrasions or pressure areas. 2. Check water temperature with the hands before getting into tub. 3. Do not use heating pads on the feet or lower legs. 4. Thoroughly dry the feet, especially between the toes. 5. Cut toenails rather than file them. 6. Cut nails in a rounded fashion.

1., 2., 3. & 4. Correct: The feet should be inspected daily. Small tears or abrasions can occur without the client's awareness due to decreased sensation in the feet. The client may be burned by getting into water that is too hot due to decreased sensation in the feet. There is less chance of decreased sensation in the hands. Heating pads may burn the client's feet. It is better to apply blankets for warmth. Drying the feet and between the toes will prevent skin breakdown. 5. Incorrect: Filing is safer, as it is not likely to result in cutting or irritating the skin around the nail. A cut on the lower extremity can result in an infection. Clients should not cut their nails. Filing is safer. 6. Incorrect: The nails should be filed straight across. Filing into a round shape may result in an ingrown toenail, which may lead to infection. Skin breaks on the lower extremity can lead to infection.

The home health nurse is assessing the home environment for threats to the safety of the toddler who lives in the home. Which observations should be included in this assessment? 1. Do stairs have guard gates? 2. Are safety covers on electrical outlet plugs? 3. Is the swimming pool inaccessible to the toddler? 4. Are cleaning supplies located out of the toddler's reach? 5. Are stairs brightly lit?

1., 2., 3. & 4. Correct: Toddlers may fall if left unsupervised around stairs. Make sure that gates are in place and that they are used. Toddlers are at risk for exploring the outlets by putting metal objects into the outlets or putting their fingers in them. They should be covered unless in use. Toddlers can drown in small amounts of water and they may try to explore swimming pools if they are accessible. Pools should have fences or locking stairs and the child should never be left unsupervised around the pool. Toddlers are curious and may get into cabinets containing harmful substances. 5. Incorrect: This assessment would be important for the visually impaired or elderly, but not specifically for toddlers. The guard gates should be in place so that the toddler does not have access to the stairs. Remember client safety is always a priority. This question is asking about toddler safety in the home. There are all kinds of safety hazards in and around the home for the toddler So let's look at the options.. Option 1 This is true. There should be hardware-mounted safety gates at the top and bottom of every stairway. Otherwise the toddler might fall and sustain a bad injury. Option 2 is also true. All unused outlets should be covered with safety plugs so the toddler cannot stick anything into the outlet. Doing so could electrocute the child. Option 3 is true. There should be climb-proof fencing at least 5 feet (1.5 meters) high on all sides of the pool? Does the fence have a self-closing gate with a childproof lock? It should. Option 4 is true. Low cabinets within easy reach of the toddler, such as under the sink should be free of cleaning supplies, bug sprays, dishwasher detergent, and dishwashing liquids. These supplies should be out of the reach of children. Option 5 is false. The toddler should not have access to the stairs at all. A well lit stairway is good for people who are older and have difficulty seeing in dim light, however.

What signs and symptoms will the nurse look for when caring for an infant with severe dehydration? 1. Dark, yellow urine 2. Lethargic 3. Bulging fontanels 4. Tachypnea 5. Decreased urine output

1., 2., 4., & 5. Correct: These would be signs and symptoms of dehydration in an infant. Amber or dark urine is an indication of dehydration. Urine should be a clear, pale yellow. Fussiness and irritability are seen in infants when they do not feel well. As dehydration worsens, lethargy and unresponsiveness can develop. Tachypnea or rapid respiration along with tachycardia and low blood pressure are present with severe dehydration. With severe dehydration, there will be decreased urine ouput. The body is trying to conserve volume. 3. Incorrect: The fontanels will be sunken rather than bulging. Bulging fontanels indicate brain swelling or fluid build up in the brain. Sunken fontanels are related to dehydration.

A home health nurse is planning home safety education for a client and spouse. Which actions should be included to promote fire safety in the home setting? You answered this question Correctly 1. A fire extinguisher should be kept on each level of the home. 2. Keep matches and lighters away from children by storing them in a locked cabinet. 3. Install carbon monoxide smoke alarms, and test them monthly. 4. You may leave Christmas lights lit all night as long as the tree is artificial. 5. Have a planned route of exit and a place where all family members will meet.

1., 2., 3. & 5. Correct: A fire extinguisher should be placed on each level of the home, near an exit, but out of reach of children. Keeping matches and lighters away from children by storing them in a locked cabinet can prevent fire-related deaths. Carbon monoxide smoke alarms will alarm for smoke as well as carbon monoxide, which is an odorless gas than can kill quickly. Alarms should be tested every month and repaired or replaced immediately if malfunction occurs. A plan facilitates exit from the building, and a place to meet helps identify that all family is out of the building. 4. Incorrect: Lit Christmas lights should be turned off when no one is home and when people go to bed for the night. It does not matter whether the tree is real or artificial.

The nurse is educating a group of sexually active teenagers about Chlamydia. What should the nurse teach these clients to prevent them from acquiring or transmitting this disease ? 1. Use a latex condom when having sex to protect against Chlamydia. 2. Seek the advice of a primary healthcare provider if there is vaginal discharge or burning on urination. 3. Suggest that the teens be screened for Chlamydia. 4. Reassure the teens that if they have no symptoms, they have no disease. 5. Take prescribed medication if diagnosed with Chlamydia, and repeat screening in three months.

1., 2., 3. & 5. Correct: Consistent use of latex condoms protects against STIs. Although chlamydia may have no symptoms, burning and discharge should be reported for further evaluation. It is recommended that all sexually active young women less than 25 years of age be screened for chlamydia on an annual basis. Medication should be taken as prescribed, and rescreening should occur in 3 months to make sure that there is no more disease present. 4. Incorrect: Chlamydia does not always produce visible symptoms, and, if left untreated, can lead to pelvic inflammatory disease (PID). False security may lead to unsafe sex practices. Women with symptoms may notice an abnormal vaginal discharge; a burning sensation when urinating. Symptoms in men can include a discharge from their penis; a burning sensation when urinating; pain and swelling in one or both testicles. So let's look at each option as a true/false statement. Option 1 is true. Using latex condoms correctly every time one has sex will help to prevent acquiring and transmitting chlamydia. Option 2 is true. Vaginal discharge and burning on urination can be a symptom of chlamydia. Option 3 is true. If a person younger than 25 years old and is sexually, or an older person with risk factors such as new or multiple sex partners, or a sex partner who has a sexually transmitted infection, then testing for chlamydia should be done every year. Gay, bisexual, and men who have sex with men; as well as pregnant women should also be tested. Option 4 is false. Although a person may not have symptoms, that person could still have chlamydia. Option 5 is true Repeat infection with Chlamydia is common. The client should be tested again about 3 months after treatment.

The nurse has been trained to work in a decontamination station for hazardous exposure victims. What should the nurse tell the victim about the process? 1. First you will remove clothing and dispose of it in hazardous material containment area. 2. You will be placed in a warm shower for decontamination. 3. You will spend a minute or so using soap over the entire body before rinsing. 4. You will spend approximately 15 minutes in the shower. 5. You will apply soap from head to toe and then rinse for a few minutes.

1., 2., 3. & 5. Correct: If the victim can remove his/her own clothing, then instructions should be given to do so and dispose of in hazardous material container. The person will wash for several minutes, beginning with a minute or so of full body rinsing with water to remove any visible contaminants, followed by soap and finally the rinse. The length of time for washing and rinsing will vary with institution and known contaminants. Using soap with good surfactant qualities is important. Generally, the victim is instructed to rinse with tepid water, apply soap from head to toe, and then rinse again with copious amounts of water. 4. Incorrect: Most procedures require about 5 to 6 minutes for the decontamination process. Times may vary depending on policy, contaminants, and the level of ability of the victim.

Which symptoms should the nurse anticipate when caring for a client with acute cholecystitis? 1. Chills 2. Fever 3. Nausea and vomiting 4. Increased appetite 5. Rigidity of upper right abdomen

1., 2., 3. & 5. Correct: Many clients with acute cholecystitis present with acute onset of right upper quadrant pain associated with nausea and vomiting. Epigastric pain may also be present as well as fever, chills, and anorexia. A physical examination often reveals rigidity of the upper right abdomen that may radiate to midsternal area or right shoulder. Rebound and guarding are present in some cases. 4. Incorrect: The client with cholecystitis will have nausea and vomiting which usually results in a decreased appetite. In order to get this question correct, you need to know what acute cholecystitis is. The gallbladder is an organ that sits below the liver. It stores bile, which your body uses to digest fats in the small intestine. Acute cholecystitis occurs when bile becomes trapped in the gallbladder. This often happens because a gallstone blocks the cystic duct, the tube through which bile travels into and out of the gallbladder. When a stone blocks this duct, bile builds up, causing irritation and pressure in the gallbladder. This can lead to swelling and infection. So let's look at our options. Option 1: True. Any disease process ending in "itis" may present with fever and chills. So knowing that will make Option 2...True. Option 3: N/V? True. Many clients with acute cholecystitis present with acute onset of right upper quadrant pain associated with nausea and vomiting. Option 4: False. If you have nausea and vomiting do you want to eat? No. You would not want to eat, so anorexia is seen. Option 5: True. A physical examination often reveals rigidity of the upper right abdomen that may radiate to midsternal area or right shoulder. Rebound and guarding are present in some cases.

A school nurse is planning a session on the effects of cannabis use for a high school health class. Which information does the nurse need to include? 1. Cannabis ingestion can cause tachycardia. 2. Inhaled cannabis produces a greater amount of tar than tobacco. 3. Cannabis smoke contains more carcinogens than tobacco smoke. 4. Cannabis ingestion reduces the risk for heart disease 5. Orthostatic hypotension can be caused by cannabis ingestion.

1., 2., 3. & 5. Correct: Tetrahydrocannabinol (THC) is the chemical compound in cannabis. THC enters the blood stream quickly and is transported to the brain and other organs. Within minutes, the heart rate may increase by 20-50 bpm and last for up to 3 hours. Cannabis ingestion may cause tachycardia and orthostatic hypotension. Cannabis smoke contains more carcinogens and tar than tobacco. Lowering of blood pressure during use is common and can lead to orthostatic hypotension. 4. Incorrect: Research has indicated that the ingestion of cannabis increases the risk for heart disease.

The nurse is working on a health promotion plan for a young family whose child has severe allergies and asthma symptoms. Which interventions would be important to include in the health promotion plan? 1. Wash stuffed animals/toys frequently in hot water. 2. Make sure that bathrooms and high humidity areas are properly vented. 3. Limit carpet in the bedrooms. 4. Use humidifiers regularly. 5. Vacuum floors and upholstered furniture regularly.

1., 2., 3. & 5. Correct: The frequent washing in hot water removes dust mites. Adequate venting lessens the likelihood of fungal/mold spores. Carpet harbors dust and other allergens. The floors and upholstered furniture may harbor dust, pollen from clothing, and other irritants. 4. Incorrect: Humid air may contribute to mold or fungal spores in the house. Less humidity is appropriate. Option 4 is different or unique as compared to the other options, especially from option 2. Both discuss humidity. Option 4 provides more humidity, while option 2 provides less. Since this is a select all that apply question, serious consideration to whether Option 2 or 4 is true or false should be made. Humid air can contribute to the growth of mold, which often triggers asthma symptoms, so statement 4 is false. The other options are true. The child's room and belongings must be cleaned often. Carpet harbors a lot of allergens and should be limited.

Question 10: Which independent nursing actions should the nurse initiate for a client admitted with heart failure? 1. Monitor for distended neck veins 2. Measure abdominal girth 3. Evaluate urine output from diuretic therapy 4. Educate client regarding signs and symptoms of heart failure 5. Administer medications as prescribed

1., 2., 3., & 4. Correct: An independent nursing intervention is one that an RN can prescribe, perform, or delegate based on their skills/knowledge. A collaborative intervention is one that is carried out in collaboration with other health team members (physical therapist, healthcare provider). Dependent nursing intervention is one prescribed by a healthcare provider but carried out by the nurse. These actions do not require an order by a healthcare provider nor collaboration with another. They are independent nursing functions. 5. Incorrect: Administering prescribed medications is a dependent nursing intervention and cannot be initiated without an order being in place. This is the only option that is dependent on the primary healthcare provider's actions first before the nurse can initiate it.

A client has been admitted to the telemetry unit with a diagnosis of a cerebral vascular accident. What should the nurse assess to determine the client's risk for aspiration? 1. Ability to swallow 2. Gag reflex 3. Level of consciousness 4. Cough reflex 5. Ability to follow commands

1., 2., 3., & 4. Correct: Assessing the ability of a client to swallow is something the nurse can and should do. A small amount of water should be given to the client as the nurse observes for coughing or gurgling. If the nurse suspects a client is having difficulty safely swallowing, further assessment by a speech and language therapist is recommended. To test for a gag reflex use a tongue depressor. Ask the client to open the mouth and look at their throat with a penlight. If the uvula and pharynx rise as the client says "aaahh" then the gag reflex is intact. If it does not rise, touch the back of the throat at the soft palate and watch for the rise in the pharynx in a gag response, If intact,the client should not be at risk for aspiration with eating. A client with a decrease level of consciousness is always at risk for dysphagia and aspiration. A cough reflex is assessed by administering a small sip of water and observing for a cough. if the client coughs, feeding should be withheld until further testing can be performed. 5. Incorrect: Assessing ability to follow commands does not identify a problem with swallowing. It does not provide a great deal of information about cognitive function. The other tests provide more information specific to aspiration.

What should be included in the discharge teaching plan for a client who has lymphedema post right mastectomy? 1. Use a thimble when sewing. 2. Wear a heavy duty oven mitt for removing hot objects from the oven. 3. Long sleeves should be worn to prevent insect bites. 4. Shave underarms with an electric razor. 5. Avoid wearing jewelry.

1., 2., 3., & 4. Correct: Because lymphedema is a lifelong threat, teach the client hand and arm precautions to minimize the risk of injury, infection, and impaired circulation. All of these options are correct options to minimize these risks. Even a minor injury such as a pin prick or sunburn can cause painful swelling after lymph node removal. 5. Incorrect: The client may wear jewelry that does not inhibit lymph drainage. They should avoid jewelry that constricts the affected arm.

What actions should a nurse take to provide continuity of care when discharging a client diagnosed with hemiparesis to a long-term care facility for rehabilitation? 1. Document current functional status assessment 2. Notify the primary healthcare provider of transfer completion. 3. Transfer essential medical record to the receiving facility. 4. Phone report to the receiving nurse. 5. Send a day's worth of medications with the client to the receiving facility.

1., 2., 3., & 4. Correct: Documentation of the client's baseline functional status is important for the receiving facility to work with in further goal setting. It is the primary healthcare provider's role to receive acceptance for transferring a client to another facility. A copy of select parts of the medical recording, according to facility policy, is another form of communication that will support continuity of care. It is the nurse's responsibility to communicate the client's condition and care plan to the receiving facility nurse in order to support continuity of care. 5. Incorrect: Medications are not transferred with the client to a new facility. A list of current medications is sent to the facility.

A nurse is planning an educational session on safety for parents of young children. What safety points should the nurse include? 1. Teach children the basics of swimming. 2. Plan an escape route in the event of fire. 3. Make sure that sand surrounds the playground equipment. 4. Gates should be placed at the top and bottom of stairs when toddlers are present. 5. Vitamins should be referred to as candy so that children will take them. 6. A child at age 7 may sit in the front seat of a car.

1., 2., 3., & 4. Correct: Everyone should know the basics of swimming (floating, moving through the water) and cardiopulmonary resuscitation (CPR). Create and practice a family fire escape plan and involve kids in the planning. Make sure everyone knows at least two ways out of every room and identify a central meeting place outside. Falls on playgrounds are common and can cause serious injury. Wood chips or sand, not dirt or grass should be under playground equipment. Having a gate at the top and bottom of stairs can prevent falls. 5. Incorrect: Do not tell children that medication, even vitamins, is candy. Children may take medicine because they think it really is candy when they see medication not intended for them. 6. Incorrect: Children should not sit in the front seat of a car until 8-12 years of age or they are 4ft. 9in. (145 cm) in height. Remember client safety is always a priority. This question is asking about safety points for young children. So let's look at the options. Option 1 is true. Everyone should know the basics of swimming (floating, moving through the water) and cardiopulmonary resuscitation (CPR). Option 2 is true. Create and practice a family fire escape plan, and involve kids in the planning. Make sure everyone knows at least two ways out of every room and identify a central meeting place outside. Option 3 is true. Falls on playgrounds are common and can cause serious injury. Wood chips or sand, not dirt or grass should be under playground equipment. Option 4 is true. Having a gate at the top and bottom of stairs can prevent falls. The approved safety gates should be attached to the wall.The manufacturer's instructions should be reviewed to make your the parent's have the right gate for their gate. Not all gates are also safe for use at the top of stairs. Option 5 is false. Do not tell children that medication even vitamins candy. Children may take medicine because they think it really is candy when they see medication not intended for them. Option 6 is false. Children should not sit in the front seat of a car until 8-12 years of age or they are 4ft. 9in. (145 cm) in height. ​

What signs and symptoms would a nurse assess for in a client who is receiving hospice care and is close to death? 1. Cool extremities 2. Mottling 3. Cheyne-Stokes respirations 4. Loss of appetite 5. Increased blood pressure

1., 2., 3., & 4. Correct: In the hours before death, blood will be shunted to the vital organs and not the periphery. This will make the extremities cool to the touch and mottled in appearance. Both cool extremities and mottling are due to reduced blood flow. Cheyne-Stokes respirations is a respiratory pattern that consists of loud deep inhalations followed by a pause of apnea. Loss of appetite will occur as energy needs decline. The use of moistened clothes around the mouth and lip balm may help with keeping lips moist and comfortable. 6. Incorrect: Blood pressure will not increase as death nears. The pumping action of the heart declines when death is occurring which leads to a decrease in cardiac output and blood pressure

A nurse working in a clinic is planning to assess a client for any sensory deficits. What assessments should the nurse include? 1. Ask the client about any recent changes in vision. 2. Observe the client's conversation with others. 3. Assess two-point discrimination. 4. Perform the Rinne test. 5. Test near vision with the Snellen chart.

1., 2., 3., & 4. Correct: Physical assessment determines whether the senses are impaired. During the physical examination, the nurse assesses vision and hearing and the olfactory, gustatory, tactile, and kinesthetic senses. The exam should reveal the client's specific visual and hearing abilities, perception of heat, cold, light touch, pain in the limbs, and awareness of the position of body parts. Start with a history and ask about recent changes in vision. Observing client conversation with others can indicate hearing or communication problems. Two-point discrimination will assess tactile sense. The Rinne test assesses hearing. 5. Incorrect. Near vision is assessed by using the Rosenbaum eye chart. It consists of paragraphs of text or characters in different sizes on a 3.5 x 6.5 inch card. Be sure the client has a literacy appropriate for the text used.

A home care nurse is visiting a client who delivered her first baby one week ago. What behavior by the client would indicate to the nurse that maternal-infant bonding is occurring? 1. Holds baby face to face 2. Talks about the baby's features 3. Touches baby frequently 4. Talks to baby 5. Allows baby to cry vigorously for 15 minutes

1., 2., 3., & 4. Correct: Positive behaviors that would indicate that maternal-infant bonding is occurring include making eye contact, assuming en face position when holding the infant, pointing out common features, smiling and gazing at the infant, touching infant, progressing from fingertips to holding, speaking soft, high-pitched tones and speaking positively about the infant. 5. Incorrect: Crying vigorously for 15 minutes is an indication that the baby has a need that the mom is ignoring. This is not a common behavior promoting maternal-infant bonding. Options 1, 2, 3, and 4 are all true. A positive maternal-infant bond has formed when baby and mom become intimately involved with each other through behaviors and stimuli that are complementary and provoke further interactions. These options illustrate such behaviors. Option 5 is false. This is not a behavior that is seen when there is a positive maternal-infant bond.

What test should the nurse use to test a client's gross hearing acuity? 1. Weber's 2. Rinne 3. Audiometry 4. Whisper 5. Monofiliment testing

1., 2., 3., & 4. Correct: The Weber test uses a tuning fork to assess bone conduction by examining the lateralization of sounds. The Rinne test compares air to bone conduction. Audiometric testing determines the degree and type of hearing loss. The audiometer produces pure tones at varying intensities to which the client can respond. The ticking of a watch has a higher pitch than the normal voice. Have client occlude one ear. Out of the client's sight, place a ticking watch 1 inch (2-3 cm) from the unoccluded ear. Ask what the client can hear. Repeat with the other ear. With the whisper test, the examiner stands 12-24 inches (30-61 cm) to the side of the client and, after exhaling, speaks using a low whisper. The client is asked to repeat numbers or words or answer questions. Each ear is tested. 5. Incorrect: Monofiliment testing identifies sensory neuropathy, particularly of the feet.

The charge nurse tells a nurse that multiple sick calls from the upcoming shift has occurred. The charge nurse asks the nurse who works in a state where mandatory overtime is legal to work an additional 8 hours of mandatory overtime. The nurse has just completed a 12 hour shift. What options would be appropriate for the nurse to take? 1. Assess personal level of fatigue prior to making a decision regarding accepting or refusing assignment. 2. Suggest splitting the shift with another nurse. 3. Accept assignment, documenting personal concerns regarding work conditions. 4. Refuse the overtime assignment, being prepared for disciplinary action. 5. Simply accept the assignment since overtime is mandatory.

1., 2., 3., & 4. Correct: The nurse's level of fatigue must be considered especially under conditions of mandatory overtime. Splitting the overtime shift is an acceptable option that the nurse could suggest in order to solve the staffing problem and decrease the amount of time the nurse will be working. The nurse can accept the assignment, documenting your personal concerns regarding working conditions in which management decides the legitimacy of employee's personal concerns. This documentation should go to your manager. Refuse the assignment, being prepared for disciplinary action. If your reasons for refusal were client safety, nurse safety, or an imperative personal commitment, document this carefully including the process you used to inform the facility (nurse manager) of your concerns. Keep a personal copy of this documentation, provide a copy to the immediate supervisor, and send a copy to the Local Unit Officer. 5. Incorrect. The nurse can also accept the assignment; however, that nurse should document professional concern for client safety and the process you used to inform the facility (manager) of your concerns. Remember if you work overtime, fatigue is not a viable reason for a error and will not stand up in court Option 1. Assess person level of fatigue. This is true. Remember assessment is the first part of the nursing process. Shouldn't the nurse identify whether personal fatigue will be an issue in caring for clients? Yes. Option 2. Suggest spitting a shift with another nurse. Could this be a reasonable option to help staff the unit appropriately? Yes. This will decrease the amount of overtime required by the nurse. Option 3. Accept the assignment and document concerns. The nurse could decide to do this and would be have the right to make this decision. The nurse manager should receive the documentation regarding the nurse's concern about working the required overtime. Option 4. Refuse the assignment. The nurse has the right to do this as well, but may have to accept any disciplinary action. Again documentation is critical. Why was the assignment refused? Option 5. Simply accept the assignment. This is not as complete an answer as option 3. Documentation is still required. Pick the more global answer (option 3) and eliminate this option as false.

What strategies for smoking prevention could the school nurse recommend to the community task force? 1. Have a "Pledge Campaign" asking students not to use tobacco. 2. Include effects of smoking in health classes. 3. Enlist help from celebrities who are against smoking. 4. Conduct a "Don't Smoke" poster contest aimed at seventh graders. 5. Start a smoking cessation class for students who currently smoke.

1., 2., 3., & 4. Correct: These are all activities that the nurse could recommend. All are primary prevention strategies that may educate and influence students to abstain from smoking. 5. Incorrect: This is a worthy activity; however, it is for students who are already smoking and wish to stop. This is not primary but tertiary prevention.

What signs of cannula displacement should the nurse monitor for at an arterial line insertion site? 1. Swelling 2. Fluid leakage 3. Blanching 4. Poor arterial waveform 5. Pyrexia 6. Purulent drainage

1., 2., 3., & 4. Correct: These are signs of cannula displacement. Observe for signs of cannula displacement into the tissues which will be swelling, bleeding, lack of a normal arterial waveform, fluid leakage, blanching, and pain or discomfort. 5. Incorrect. This is a sign of infection rather than cannula displacement. Signs of infection include pain, redness, purulent drainage, and fever. 6. Incorrect. This is a sign of infection rather than cannula displacement.Signs of infection include pain, redness, purulent drainage, and fever. In this question we are looking for evidence that an arterial line catheter has migrated out of the artery. Well, some signs are the same as you would see with a plain ole IV. So let's look at our options. Option 1, Swelling. If fluid is being delivered into the interstitial space around the artery, would you see swelling? Yes. This is true. Just like an IV catheter into a vein. Option 2. Fluid leakage. If the cannula is not in the artery, would fluid begin to leak around the catheter and be seen at the catheter insertion site? Yes. This is true. Same as with a plain ole IV. Option 3. Blanching. Would you see this with a plain IV into a vein? No, But what about one going into an arterial line that might be displaced and partially occluding the artery? Yes, This is true. Remember, blanching of the skin typically indicates a temporary obstruction of blood flow. If you press gently on an area of your skin, it likely turns lighter before resuming its natural color. Option 4. Poor arterial waveform. When you have an arterial line, we can measure the client's blood pressure and we see an arterial waveform. A flattened or poor waveform indicates a possible cannula displacement. Option 5. Pyrexia - fever. Would the client have an infection with a cannula displacement? No, This is false. Option 6. Purulent drainage? False. This is a sign of infection as well.

The client diagnosed with active tuberculosis has been prescribed isoniazid 300 mg by mouth every day. What should the nurse teach this client? 1. "Notify your healthcare provider if your urine turns dark." 2. "Your healthcare provider has prescribed B6 along with the isoniazid to prevent neuritis." 3. "You should avoid eating aged cheeses and smoked fish." 4. "Eat foods such as tuna twice a week." 5. "Rise slowly from lying to sitting, or sitting to standing."

1., 2., 3., & 5 Correct: Signs of hepatotoxicity from this medication include dark urine, jaundice, and clay-colored stool. Isoniazide- induced pyridoxine (Vitamin B6) depletion causes neurotoxic effects. Vitamin B6 supplementation of 10-50 mg usually accompanies isoniazid use. Aged cheeses and smoked fish are high in tyramine which may cause palpitations, flushing, and blood pressure elevation while taking isoniazid. Avoid these foods during treatment. Isoniazid should be taken on an empty stomach, one hour before or two hours after food. Some clients experience orthostatic hypotension while taking isoniazid, so caution against rapid positional changes. 4. Incorrect: Histamine containing foods such as tuna and yeast extracts may cause exaggerated drug response (H/A, hypotension, palpitations sweating, itching, flushing, diarrhea).

What potential contributing factors for transient urinary incontinence should a nurse assess for in an elderly female client? 1. Fecal impaction 2. Use of a diuretic 3. Diabetic 4. Urinary retention 5. Vaginitis

1., 2., 3., & 5. Correct: Fecal impaction can compress the urethra resulting in urinary incontinence. Use of diuretics can make it difficult to get to the toilet in time to void, thus causing urinary incontinence. Diabetics have polyuria, which can contribute to urinary incontinence. Vaginitis can contribute to urinary incontinence. 4. Incorrect: Urinary retention is an established or chronic form of urinary incontinence.

A client is admitted to the Labor & Delivery Unit with severe preeclampsia. Which nursing intervention does the nurse include in the plan of care for this client? 1. Monitor for headache. 2. Place client in left recumbent position. 3. Insert indwelling urinary catheter. 4. Administer propranolol for BP > 100 diastolic. 5. Initiate external fetal heart monitoring.

1., 2., 3., & 5. Correct: Headache is a sign of increasing BP and increasing ICP. The left recumbent position moves the fetus off the mom's aorta and will help decrease the BP. This client needs to have UOP closely monitored because of the fluid volume excess (FVE), so an indwelling urinary catheter is needed. The fetus needs to be monitored for complications, and the fetal heart rate (FHR) should be 120-160/minute so close monitoring is required. 4. Incorrect: Propranolol, a beta blocker, is not given during pregnancy as it decreases HR and the amount of blood pumped by the heart. This can cause fetal bradycardia, decreased cardiac output, and potential for fetal demise.

Which assessment findings would the nurse expect to see in a client diagnosed with idiopathic thrombocytopenic purpura (ITP)? 1. Ecchymosis 2. Bleeding gums 3. Palpable spleen 4. Pain 5. Petechiae

1., 2., 3., & 5. Correct: The word thrombocytopenia means low platelets. Any client with low platelets is at risk for bleeding, which is indicated by ecchymosis (bruising), bleeding gums, and petechiae (red to purple dots on the skin, 1-3 mm in size). Spleen and liver are often slightly palpable. 4. Incorrect: Pain is not associated with ITP unless there are other associated problems. However, the stem of the question gave no indication that other problems exist.

A nurse is teaching a client the advantages of having a PICC line inserted rather than a peripheral IV. What information should the nurse include? 1. TPN may be infused using a PICC line. 2. Use of a PICC can allow for early client discharge. 3. PICC lines do not have to be replaced as often as a peripheral IV line. 4. PICC lines have the same risk of infection as a peripheral IV line. 5. PICC lines do not need to be flushed as frequently. 6. PICC placement decreases the need for skin puncture when blood sampling is needed.

1., 2., 3., & 6. Correct: Peripheral IV lines must be changed every 72-96 hours. PICC lines may remain in place for extended periods of time. A PICC can be cared for at home by home care nurses, family members, or in outpatient clinics. TPN cannot be administered via a peripheral line since it is hypertonic. PICC lines offer a lower chance for infection than a peripheral line. As long as the PICC is functioning and there is no evidence of infection, the PICC line can remain in place until it is no longer needed. 4. Incorrect: PICC lines are long lasting, so the risk of infection from changing sites is eliminated. Additionally, sterile technique is used for insertion, with sterile dressing changes. Precautions should still be taken to prevent complications. 5. Incorrect: Both peripheral and central lines need to be flushed to maintain patency.

The nurse providing palliative care to a client would include which outcomes in the teaching plan? 1. Maintaining the client's quality of life 2. Minimizing family caregiver stress 3. Managing the client's pain 4. Managing the client's and family's emotional needs 5. Attending to the client's spiritual needs 6. Ensuring the client understands that disease focused treatments will cease

1., 2., 3., 4. & 5. Correct: Palliative care includes supporting the client's and family's quality of life. Palliative care includes managing pain and symptoms. Palliative care includes managing the client's and family's emotional needs and attending to their spiritual needs. 6. Incorrect: Palliative care includes treatment of discomfort, symptoms, and stress of serious illness. Disease focused treatments will not cease. First, you must understand palliative care. Palliative care may be used in any serious illness to provide treatment of discomfort, s/s, and stress. Palliative care is broader than hospice care and aims to support best possible quality of life regardless of stage of disease. Look at each option and see which ones are true and thus would be appropriate for the teaching plan. Option 1: Yes. We want the client's quality of life to be maintained. This includes all aspects of the client's life. Much support is needed during a serious illness. Option 2: Good choice. The family caregiver is under much stress caring for a loved one with a serious illness. Option 3: A must. Appropriate control of pain is so important and affects all areas of the client's life. The pain should be controlled as much as possible for the client to maintain a quality of life. Option 4: Yes. Emotional needs will vary with the type and severity of the illness. A client requiring palliative care has an illness that will require changes in their life style possibly including work, marriage, children, and other responsibilities. The emotional support needed can be great. Option 5: Yes. Due to the serious nature of illnesses requiring palliative care the nurse should include spiritual care. Everyone has spiritual needs and they affect our health. Option 6: This is a false statement. Palliative care can include disease focused treatments. The nurse may at appropriate times help the client and family weigh the benefits of continued diagnostics and disease focused treatments.

Which signs/symptoms does the nurse expect to note when caring for a client with a suspected cystitis? 1. Incontinence 2. Urgency 3. Frequency 4. Hematuria 5. Nocturia 6. Flank pain

1., 2., 3., 4. & 5. Correct: Signs and symptoms of cystitis include burning on urination, nocturia, incontinence, suprapubic or pelvic pain, hematuria, and back pain. 6. Incorrect: Flank pain is seen when the urinary tract infection progresses to the kidneys. Cystitis is a fairly common lower urinary tract infection, it refers to an inflammation of the bladder, specifically, the wall of the bladder. All this question is asking is "Hey new nurse, do you know the signs/symptoms of cystitis? Well do you? Remember to look at each option individually and decide if it is a sign or symptom of cystitis. Option 1. Incontinence - loss of bladder control? True or False? True. Option 2. Urgency - A strong desire to urinate, accompanied by a fear of leakage.? True Option 3 Frequency - the need to urinate many times during the day, at night (nocturia), or both but in normal or less-than-normal volumes. ? True Option 4 Hematuria - the presence of blood in urine.? True Option 5. Nocturia - Frequency at night? True Option 6. Flank pain? False. Flank pain occurs with a kidney infection, pyelonephritis.

A nurse is planning a health fair in a Hispanic community composed of primarily young adults. What would be essential for the nurse to provide to this community at the health fair? 1. Blood pressure screening 2. Glucose monitoring 3. Influenza vaccination 4. BMI calculation 5. Test urine for protein. 6. Pneumococcal vaccination

1., 2., 3., 4., & 5. Correct: In order to answer this question correctly, the test taker needs to understand what is considered young adulthood (ages 18 to 40 yrs). Middle adulthood is from ages 40 to 65 yrs and late adulthood is greater than 65+ yrs. You also need to know what Hispanics are at risk for. Hispanics have a higher incidence of death from heart disease and stroke. Blood pressure monitoring is essential to detect and control hypertension. Diabetes is prevalent in the Hispanic community. Early diagnosis is critical to manage and control for the risk of complications. Flu vaccination is recommended for all ethnic groups. Obesity is very high among Hispanic Americans at >70%. Chronic renal failure is a high risk for Hispanic Americans particularly since diabetes is prevalent. Early testing for protein in urine is recommended. 6. Incorrect: Pneumococcal vaccination is recommended for older adults (greater than age 65).

Which vaccines would a nurse participating at a health fair encourage a 65 year-old adult to receive? 1. Influenza 2. Herpes Zoster 3. Diphtheria 4. Pertussis 5. Pneumococcal vaccine 6. Measles, mumps, and rubella (MMR)

1., 2., 3., 4., & 5. Correct: Influenza is often quite serious for people 65 and older due to weaker immune defenses. CDC recommends a single dose of herpes zoster vaccine for people 60 years of age or older to prevent shingles.Tetanus, diptheria and pertussis (Tdap) vaccine is given to older adults to protect against whooping cough (pertussis), tetanus and diptheria. Adults should get one dose of the tetanus and diptheria (Td) vaccine every 10 years. For adults who did not get Tdap as a preteen or teen, they should get one dose of Tdap in place of a Td dose to boost protection against whooping cough. However, adults who need protection against whooping cough can get Tdap at anytime, regardless of when they last got Td. Pneumococcal vaccines protect against infections in the lungs and blood stream and are recommended for all adults over 65 years old and for adults younger than 65 who have certain chronic health conditions. 6. Incorrect. A booster for measles, mumps, and rubella is not indicated for this age group.

A case manager is evaluating a client diagnosed with hemiplegia due to a cerebral vascular accident for assistive devices that will be needed upon discharge. Which resources should the case manager include for this client? 1. Plate guards 2. Transfer belt 3. Raised toilet seat 4. Long handled shoe horn 5. Wide grip utensils 6. Large button closures on clothes

1., 2., 3., 4., & 5. Correct: The goal is to promote self-care by the client as much as possible. The case manager should evaluate the need for assistive devices to help with eating, bathing, dressing, and ambulating. The plate guard will prevent food from being pushed off of the plate. The transfer belt will provide safety for the client and family member who is assisting the client to get up into a chair or back in bed. A raised toilet seat makes it easier for the client to sit on the toilet without falling. The long handled shoe horn allows the client to put on shoes without assistance. Wide grip utensils accommodate a weak grip. 6. Incorrect: It is hard for someone with hemiplegia to use buttons. Velcro fasteners are best. An Assistive Device is any device that helps someone do something that they might not otherwise be able to do well or at all. Generally the term is used for devices that help people overcome a handicap such as a mobility, vision, mental, dexterity or hearing loss. The case manager in this question is identifying resources that will help a client with hemiplegia. Do you remember the definition of hemiplegia? It is paralysis of one side of the body. Well if the client cannot move one side of the body, what kind of assistive devices would be helpful for the client in achieving activities of daily living? Option 1. Plate guards. Yes. A plate guard will prevent food from being pushed off of the plate. The plate has sides to it. Option 2. A transfer belt. The client cannot move one side of the body. So a transfer belt is a safety feature for the client and the care provider who is assisting the client in moving. Option 3. A raised toilet seat. Will this help the client? Yes. A raised toilet seat makes it easier for the client to sit on the toilet without falling. Remember safety! Option 4. Long handled shoe horn. Will this help with activities of daily living? Yes, it will help the client put on shoes. Remember the goal is to promote self-care by the client as much as possible. Option 5. Wide grip utensils. Wide grip utensils accommodate a weak grip. After a stroke, clients may not have fine motor dexterity. It will be easier for the client to hold a spoon with a big handle then to hold one with a small handle. Option 6. Large button closures on clothes. Well I just told you that they have a weak grip and have difficulty with fine motor dexterity. So trying to button up clothes even with large buttons may be difficult. Velcro fasteners would make dressing easier for this client. They put big in this option to make you think that the larger button would be easier to close, but don't be mislead.

A child is brought into the emergency department (ED) after accidently ingesting 3 grams of acetylsalicylic acid. Initial assessment reveals lethargy, excessive sweating, hyperventilation, and hyperthermia. What interventions should the nurse initiate? 1. Provide tepid water sponge bath. 2. Start an IV for fluid resuscitation. 3. Insert a nasogastric tube. 4. Pad side rails. 5. Obtain blood gases. 6. Administer ipecac syrup orally.

1., 2., 3., 4., & 5. Correct: This client has hyperthermia. Methods to decrease temperature include external cool down, such as with a tepid water sponge bath. Dehydration occurs early in aspirin poisoning due to vomiting and hyperventilation. IV fluid is needed to offset the dehydration. Gastric lavage and activated charcoal are needed to deactivate the aspirin. The child is at risk for seizures so pad the side rails for safety. Care is based on blood gas results. Metabolic acidosis is the imbalance of the most concern. 6. Incorrect: Although ipecac syrup was used commonly in the past to make a client vomit, it is rarely recommended today. It would not be suggested in aspirin poisoning due to the chance that the client might develop altered mental status or convulsions.

A nurse manager has several issues regarding staff maintaining proper infection control while caring for clients. What actions should the manager take regarding this issue? 1. Place colorful posters regarding infection control in conspicuous places on unit. 2. Monitor staff providing client care for the use of appropriate infection control. 3. Give staff a written test on proper infection control. 4. Have all staff read agency policy and procedures regarding infection control. 5. Dock pay of staff who do not maintain proper infection control. 6. Provide mandatory in-service sessions on infection control for every shift.

1., 2., 3., 4., & 6. Correct. Each of these actions can be taken by the nurse manager. The staff needs further education, reminders, and follow-up observation. Posters are great reminders of concepts. All nurses need to supervise those under their direction. Testing can be done as pretest or post test along with in service education. Staff development or in service sessions are required by Joint Commission on Accreditation of Healthcare Organizations (JCHO) on infection control. 5. Incorrect. This is not the best solution, because most people want to do what is right. Education should be tried first, then documentation of the infractions. You must support, supervise and educate!

A nurse is planning to provide information to a group of adults considering smoking cessation. What information should the nurse include? 1. Nicotine is the drug in tobacco products that produces dependence. 2. Withdrawal symptoms may include irritability, difficulty concentrating, and increased appetite. 3. Stopping smoking reduces the risk of coronary heart disease. 4. All smokers need to have a prescription for bupropion SR in order to quit. 5. Refer to smoking quit-lines that offer free support, advice, and counseling from experienced coaches.

1., 2., 3., 5. Correct: These are correct statements. Nicotine is the drug in tobacco products that produces dependence. Other withdrawal symptoms include anxiety and cravings for a cigarette. There are many health benefits to smoking cessation including reducing the risk of coronary heart disease, stroke, peripheral vascular disease, COPD and reduced risk for infertility in women. Clients should be referred to educational programs and support groups. 4. Incorrect: The majority of cigarette smokers quit without using this prescription; however, treatments can help the smoker quit, so they should discuss possible medications with their primary healthcare provider. Other medications such as the nicotine patch or varenicline (chantix) may also be used to assist with smoking cessation.

Which signs/symptoms does the nurse expect to see in a client diagnosed with Bell's Palsy? 1. Drooping of one side of the face. 2. Inability to wrinkle forehead. 3. Excessive tearing. 4. Decreased sensitivity to sound. 5. Inability to taste. 6. Numbness of affected side of face.

1., 2., 3., 5., & 6. Correct. Symptoms of Bell's Palsy include sudden weakness or paralysis on one side of the face that causes it to droop (main symptom), drooling, eye problems (such as excessive tearing or a dry eye), loss of ability to taste, pain in or behind ear, numbness in the affected side of face, increased sensitivity to sound. 4. Incorrect. There would be increased sensitivity to sound with Bell's Palsy. Bell's palsy is a paralysis or weakness of the muscles on one side of your face. Damage to the facial nerve that controls muscles on one side of the face causes that side of to droop. The nerve damage may also affect the client's sense of taste and the amount of tears and saliva produced. This condition comes on suddenly, often overnight, and usually gets better on its own within a few weeks. Looking at the options. Which ones are associated with this diagnosis? Did you pick options 1, 2, 3, 5, and 6? Good for you. Why is option 4 wrong? The client will actually have increased sensitivity to sound. Hyperacusis also sometimes appears after 7th nerve (Bells palsy) injuries, which paralyze one of the two small ear muscles that protect the ear from loud noise.

The nurse suspects a client admitted with myasthenia gravis is going into a cholinergic crisis. Which signs and symptoms would validate the nurse's suspicions? 1. Abdominal cramping 2. Lethargy 3. Salivation 4. Hypertension 5. Lacrimation 6. Miosis

1., 2., 3., 5., & 6. Correct: The signs of cholinergic crisis include Diarrhea and abdominal cramping, Urination increased, Miosis (pinpoint pupils), Bradycardia, Emesis (nausea and vomiting), Lacrimation, Lethargy, Salivation. Remember this: DUMBELLS as a mnemonic to help you recall these signs and symptoms. 4. Incorrect: Hypertension is not a sign of cholinergic crisis. Muscles get weaker so BP would go down. Cholinergic crisis is an episode of excessive stimulation to one of the body's neuromuscular junction points. Such an event results from a buildup of acetylcholine stemming from acetylcholinesterase inactivity or insufficiency. A common cause of cholinergic crisis episodes is the unintended overdose of treatment drugs in myasthenia gravis clients. Remember DUMBELLS - Diarrhea and abdominal cramping, Urination increased, Miosis (pinpoint pupils), Bradycardia, Emesis (nausea and vomiting), Lacrimation, Lethargy, Salivation to help you remember these signs and symptoms.

A client who has been taking phenytoin for several years arrives to the clinic for follow-up care. During the nurse's history and physical of the client, which findings indicate a possible side effect to the phenytoin? 1. Skin rash 2. Reports fatigue 3. Dyspnea on exertion 4. Pale conjunctiva 5. Heart rate 60/min

1., 2., 3.,& 4. Correct: An adverse effect of phenytoin is aplastic anemia. Phenytoin is an anticonvulsant. Aplastic anemia is a blood disorder where not enough new blood cells are produced in the bone marrow. The blood cells include red blood cells, white blood cells and platelets. The most common symptom of decreased RBC's is fatigue and dyspnea upon exertion because RBC's are responsible for oxygen transport throughout the body. A common sign/symptom of aplastic anemia is also skin rashes. Collectively, these are signs/symptoms of aplastic anemia caused by this medication. 5. Incorrect: This is a normal heart rate, and there is no concern for vital signs within normal limits.

A nurse works in the operating room (OR) as a circulator. Which actions should the nurse perform to help prevent surgical-site infections? 1. Keep the OR doors closed during a surgical case. 2. Minimize traffic in the OR. 3. Ensure the room has negative air flow. 4. Monitor the sterile field at all times. 5. Immediately discard any object that becomes contaminated.

1., 2., 4. & 5. Correct: Before we review the options, let's look at the question. The key words in a question should be identified. The key words in this question are operating (OR), circulator, and prevent surgical-site infections. This is a select all question so there will be 2 or more options correct. Also each option stands alone with the question. After reading each answer, you need to ask yourself is this answer true or false. Remember client safety is always a priority. This question is identifying interventions to prevent surgical-site infections. Let's look at each option. Option 1 is a true statement. Keeping the door closed will allow the air exchange system to work properly within the OR, so that organisms are vented out of the room. Option 2 is true as well. The more people entering the OR, the greater the chance for contamination. Option 3 is false. Positive air flow is needed in the OR. The purpose of positive pressure is to ensure that airborne pathogens do not contaminate the client or supplies in that room. Positive pressure could be used in an operating room to protect the client and sterile medical and surgical supplies. Positively pressured rooms are typically considered the cleanest rooms in the hospital. Option 4 is true. The sterile field should be observed continuously to maintain that contamination has not occurred. Option 5 is also true. Items that become contaminated should be discarded. Option 5 is also true. Items that become contaminated should be discarded.

The nurse is caring for a client who has hypercholesterolemia. When evaluating the effects of atorvastatin, the nurse should monitor the results of which laboratory tests? 1. AST 2. Alkaline phophatase 3. Complete blood count 4. Serum cholesterol levels 5. Serum triglyceride levels

1., 2., 4. & 5. Correct: AST is a liver function test. Liver function tests including AST should be monitored before, at 12 weeks after initiation of therapy or after dose elevation, and then every 6 months. If AST levels increase to 3 times normal, atorvastatin should be reduced or discontinued. Atorvastatin may increase alkaline phosphatase and bilirubin levels. Atorvastatin is a lipid-lowering agent/HMG-CoA reductase inhibitor. The expected outcome of treatment with atorvastatin is lower serum cholesterol and triglycerides. 3. Incorrect: The CBC results would not be used to evaluate the effectiveness of treatment with atorvastatin. The CBC is used to evaluate your overall health and can be used to measure components and features of your blood such as RBC'c, WBC's, Hgb, Hct and platelets. What is atorvastatin? If you don't know, you should have been clued in by the word, hypercholesterolemia. High Cholesterol, right? Yes, so this is a statin drug to lower serum cholesterol and triglycerides. Now you should be able to pick out at least two correct answers. Did you say options 4 and 5 are correct? Yes. If Atorvastatin is a lipid-lowering agent and the expected outcome of treatment with atorvastatin is to lower serum cholesterol and triglycerides, then we need to monitor those levels don't we? Yes we do. Let's look at the rest of the options now. Option 1. True. Cholesterol and triglycerides are produced in the liver, so this drug can affect the liver's function. AST is a liver function test. Liver function tests including AST should be monitored before, at 12 weeks after initiation of therapy or after dose elevation, and then every 6 months. If AST levels increase to 3 times normal, atorvastatin should be reduced or discontinued. Option 2. This is true. Atorvastatin may increase alkaline phosphatase and bilirubin levels. What about option 3? No, this is false. The CBC results would not be used to evaluate the effectiveness of treatment with atorvastatin. The CBC is used to evaluate the client's overall health and can be used to measure components and features of blood such as RBC'c, WBC's, Hgb, Hct and platelets.

A client is admitted to the hospital due to alcohol toxicity. Which interventions should the nurse initiate? 1. Pad side rails 2. Attach client to pulse oximeter 3. Monitor closely for hyperthermia 4. Place in recovery position 5. Monitor fluids and electrolytes

1., 2., 4. & 5. Correct: Alcohol toxicity can lead to behavior changes and alcohol-induced central nervous system depression which can lead to respiratory and circulatory failure. The client can also experience unconsciousness, or coma leading to possible death. The client can also experience hypokalemia, hypomagnesemia and hypoglycemia. Client is at risk for seizures so pad the side rails. Client is at risk for hypoventilation and may stop breathing. Pulse oximeter will measure oxygen levels. The recovery position decreases the risk for aspiration. Alcohol has a diuretic effect, so I&O should be monitored. 3. Incorrect: Due to the fluid shift, this client is at a higher risk for hypothermia. The client with alcohol toxicity is not at risk for hyperthermia. Alcohol acts like a sedative, right? Yes. It depresses the central nervous system. What else does alcohol do to a person? Alcohol is hypertonic, so it makes the client void a lot. So they can go into a fluid volume deficit and lose potassium and magnesium through the kidneys. Option 1: Pad the side rails? True or False? This is true. Could this client have seizures? Yes. Seizures occur as a result of low blood sugar levels. So pad the side rails. Option 2 Attach client to pulse oximeter? True. Alcohol acts like what? A sedative, so decreased respiratory rate or apnea can occur. Option 3. Watch for hyperthermia? This is false. The client is at risk for hypothermia, not hyperthermia. Alcohol is a vasodilator, meaning that it causes your blood vessels to dilate, particularly the capillaries under the surface of your skin. Thus, the volume of blood brought to the skin's surface increases, making you feel warm. However, this overrides one of your body's defenses against cold temperatures, constricting your blood vessels, thereby minimizing blood flow to your skin in order to keep your core body temperature up. Option 4, place in recovery position? True. This client's level of consciousness is decreased because the client is sedated. So the client is at risk for aspiration. Put the client in the recovery position, side lying. Option 5, Monitor fluids and electrolytes. True. What are the two electrolytes we are worried about? Potassium and magnesium.

A nurse is educating a group of community citizens about risk factors for developing peripheral neuropathy. Which risk factors should the nurse include? 1. Uncontrolled diabetes 2. Alcohol abuse 3. Vitamin A deficiency 4. Rheumatoid arthritis 5. Varicella-zoster virus

1., 2., 4. & 5. Correct: All are risk factors for peripheral neuropathy. 3. Incorrect: Vitamin A deficiency can result from inadequate intake, fat malabsorption, or liver disorders. Deficiency impairs immunity and hematopoiesis and causes rashes and typical ocular effects (e.g., xerophthalmia, night blindness). Vitamin B deficiency can cause peripheral neuropathy.

A client diagnosed with rheumatoid arthritis has been prescribed dexamethasone orally as part of initial treatment therapy. What side effects should the nurse teach the client are expected? 1. Fatigue 2. Insomnia 3. Hypoglycemia 4. Truncal obesity 5. Increased appetite 6. Low blood pressure

1., 2., 4. & 5. Correct: Dexamethasone (Decadron) is a corticosteroid used short term to treat severe inflammation occurring in rheumatoid arthritis (RA). Expected side effects are associated with the body's response to excessive steroids in the system. Even short term use of corticosteroids will produce fatigue, secondary to insomnia, truncal obesity accompanied by thin extremities, and an increased appetite resulting in weight gain. Despite the short and intermittent use of corticosteroids for this auto-immune disease, some side effects remain permanently. 3. Incorrect: Excessive steroids in the body cause blood glucose levels to increase, resulting in hyperglycemia. Clients taking corticosteroids will need regular finger stick glucose monitoring while taking these medications for rheumatoid arthritis. 6. Incorrect: The body's response to increased corticosteroids in the system is an elevated blood pressure, often accompanied by headaches or nausea. Clients taking steroids will need to have blood pressure checked frequently.

A nurse notes that a client with end-stage chronic renal failure has dry, itchy skin, white crystals on the skin, and uremic halitosis. Which nursing interventions would be appropriate for this client? 1. Encourage use of cotton gloves during sleep 2. Apply emollients to the skin 3. Increase protein rich foods in the diet. 4. Cut fingernails short 5. Provide mouth care prior to meals

1., 2., 4. & 5. Correct: The build up of uremic frost associated with end stage renal disease causes pruritus. Gloves reduce the risk of dermal injury. Emollients and lotion will aid dry, itchy skin. Apply after bathing. Cutting nails short will decrease risk of skin breakdown when scratching. Uremic halitosis occurs from a build-up of urea in the body. It produces a metallic taste in the mouth. Mouth care prior to meals will help in eliminating this taste. 3. Incorrect: A client in end stage renal disease needs to decrease the amount of protein in the diet. Dietary restrictions include protein, sodium, potassium, and phosphate.

The nurse has been teaching the client about warfarin for prevention of pulmonary emboli. Which comments by the client indicate understanding of the medication? 1. "I must get my blood levels checked regularly." 2. "I shouldn't change my diet to include a lot of foods containing vitamin K without supervision." 3. "I should eat lots of foods containing vitamin K." 4. "I should report this medication to any primary healthcare provider that I see." 5. "I should not change the dosage without talking with my primary healthcare provider."

1., 2., 4. & 5. Correct: The client should comply with regular follow up visits for checks of INR level. INR is the international normalization ratio and is used for clients taking anticoagulants (blood thinning medications). The client should eat a normal healthy diet, but should not increase foods containing high amounts of vitamin K. The client should report using warfarin to any primary healthcare provider, as treatment may be changed due to this medication. The client should not manipulate the dosage unless instructed by the primary healthcare provider. An identification card or bracelet may also be recommended in case of emergencies. Clients should inform dentists and other healthcare providers especially before a medical procedure. The anticoagulant effect must be closely monitored. 3. Incorrect: Vitamin K reverses the anticoagulant effects of warfarin, so instruct the client to avoid foods high in vitamin K (examples are green leafy vegetables, brussels sprouts, prunes, cucumbers and cabbage). Do you know what warfarin is? Well you know from the stem that it is prescribed for the prevention of pulmonary emboli. How? Warfarin is an anticoagulant that reduces the formation of blood clots by inhibiting vitamin K dependent coagulation factors. So know you have an idea of what options could be correct. Let's look at them. Option 1. What do you think? True. Too much warfarin can lead to bleeding/hemorrhage. The client needs regular follow up visits to check the INR level. INR is the international normalization ratio and is used for clients taking anticoagulants. Option 2. This is true. What would happen if the client ate too much vitamin K? Vitamin K is the antidote for warfarin. So the client's INR level would not be therapeutic. Can you say pulmonary emboli?! So the client should eat a normal healthy diet, but should not increase foods containing high amounts of vitamin K. Option 3. Well if option 2 is true, can option 3 be true? No! Watch out for opposites. They cannot both be correct. Vitamin K reverses the anticoagulant effects of warfarin, so instruct the client to avoid foods high in vitamin K (examples are green leafy vegetables, brussels sprouts, prunes, cucumbers and cabbage). Option 4. This is a safety issue isn't it? Yes. This is correct. In fact, a list of all medications should be provided to any healthcare providers caring for a client. What about option 5? This is True. The client should not manipulate the dosage unless instructed by the primary healthcare provider. An identification card or bracelet may also be recommended in case of emergencies.

A client reports crushing chest pain 3 hours prior to arrival in the emergency department. Initial assessment by the nurse reveals a BP of 90/50, a weak, thready pulse at 108/min, cool, clammy skin, and confusion. Which interventions should the nurse perform? 1. Initiate cardiac monitoring. 2. Monitor intake and output hourly. 3. Position client in recumbant position. 4. Limit physical activity. 5. Administer dopamine at 5 micrograms/kg/min.

1., 2., 4. & 5. Correct: This client exhibits signs of cardiogenic shock, a complication of myocardial infarction. Hypotension accompanied by clinical signs of increased peripheral resistance (weak, thready pulse and cool, clammy skin) and inadequate organ perfusion (altered mental status and decreased urinary output) are found in this client. Initiate cardiac monitoring, watching for dysrhythmias, monitor I&O hourly to make sure kidneys are perfused. Limit activity to decrease oxygen demand. Dopamine is administered to increase BP and cardiac output. 3. Incorrect: Position upright to promote optimal ventilation by reducing venous return and lessen pulmonary edema. These are signs of shock: BP of 90/50, a weak, thready pulse at 108/min, cool, clammy skin, and confusion. So what needs to be done for the client in shock? 1. True - Client's in shock can start having dysrhythmias so cardiac monitoring is appropriate. 2. True - Shock will lead to decreased perfusion of vital organs such as the kidneys. Poor perfusion to the kidneys for 20 minutes can lead to renal failure. UOP above 30 mL/hour is needed. 3. False - With hypovolemic shock you could lie the client supine. But this is cardiogenic shock. Decrease the workload of the heart by sitting the client up. 4. True- The more the client moves, the more oxygen is required. The heart is not working well due to an MI. 5. True. Dopamine (Intropin) is used to treat certain conditions, such as low pressure, that occur when you are in shock, which may be caused by heart attack, trauma, surgery, heart failure, kidney failure, and other serious medical conditions. Dopamine produces positive chronotropic and inotropic effects on the myocardium, resulting in increased heart rate and cardiac contractility. This is accomplished directly by exerting an agonist action on beta-adrenoceptors and indirectly by causing release of norepinephrine from storage sites in sympathetic nerve endings.

Which teaching points should the nurse include when preparing the school-age child for heart surgery? 1. Discuss postoperative discomfort and interventions. 2. Show unfamiliar equipment. 3. Explain that an endotracheal tube will be needed. 4. Let the child hear the sounds of an ECG monitor. 5. Answer questions about surgery using words at the child's level of understanding.

1., 2., 4. & 5. Correct: Yes, let the child talk about their feelings, fears and discomforts. Explain in simple terms what the child will see and experience the day of surgery. Let the child visit the surgical area to see and touch equipment. All of these interventions will help decrease anxiety during the pre and postoperative periods and reduces fear of the unknown. 3. Incorrect. Do not use big words or give too much detail of events of which the child will not be aware. School-age children (5 to 12 years) are capable of concrete, logical reasoning and are gaining an increased understanding of cause and effect. They are better able to cooperate with treatment because they can think before they act. They have an increased awareness of body parts and body function and are able to understand the steps in a surgical process. Offer a simple explanation of what part of the body the operation will affect. Body outlines, pictures, or dolls may be helpful. Option 3 is false, because endotracheal tube is complex and not in layman terms.

What discharge teaching should the nurse include to the parent of an adolescent who has a mild concussion? 1. Concussion symptoms may last anywhere from hours and days to weeks and months. 2. Return to the emergency department for worsening headache. 3. Monitor for increased intracranial pressure. 4. Avoid physical activities until released from care. 5. Awaken the client every two hours.

1., 2., 4., & 5. Correct. This injury will result in symptoms that may last anywhere from hours and days to potentially weeks and months. Contact the primary healthcare provider or the Emergency Department if the client has repeated vomiting, severe or worsening headache, severe or worsening dizziness, or any worsening symptom that alarms client or family. Avoid physical activities (sports, gym, and exercise) and reduce cognitive demands (reading, texting, computer use, video games, etc). The brain is responsible for managing physical and cognitive functions of the body; therefore, it is important to decrease any activity that increases symptoms. Awaken every two hours to check level of consciousness. 3. Incorrect. A lay person would not know the signs/symptoms of increased ICP.

A school nurse is planning to teach kindergarten students about oral health. Which points should the nurse include? 1. Do not drink soft drinks between meals. 2. Eat raw vegetables to help keep teeth clean. 3. Brush teeth twice a day with toothpaste that does not have fluoride. 4. Use a pea size amount of toothpaste. 5. Floss teeth daily.

1., 2., 4., & 5. Correct: Avoid sweet foods and drinks between meals. Take them in moderation at meals. Eat coarse, fibrous foods, cleansing foods, such as fresh fruits and raw vegetables. If unable to brush after a meal, vigorously rinse mouth with water. Teeth should be flossed daily. 3. Incorrect: Brush the teeth thoroughly with toothpaste that has fluoride. Bacteria feed on sugar and produce acid waste, which erodes the tooth to create a cavity—so one of the best ways to keep to keep teeth healthy is to limit intake of sugary foods and drinks. Soda, Gatorade, sticky candy, and other sweets increase risk. Limit sugary snacks and drinks to mealtimes and brushing soon after eating. Crunchy fresh fruits and veggies like apples, carrots, and celery are better options because they help scrub plaque from teeth as you eat. Once a child gets the first tooth, brush gently with a soft-bristled toothbrush twice daily for 2 minutes or as needed, with the goal of removing plaque from each tooth. Flossing is also necessary to reach the spaces between teeth and should be done as soon as two teeth touching each other. Because the toothpaste has fluoride, only a small amount, the size of a pea, is needed. The toothpaste should be spit out of the mouth rather than swallowed.

The nurse is caring for a client post heart transplant who is being discharged on cyclosporine and azathioprine. Which precautions would be important for the nurse to teach the client? 1. Avoid crowds. 2. Do not obtain live vaccinations. 3. Drink at least 3 liters of fluids per day and watch the urine for sediment. 4. Use a soft-bristled brush to clean your teeth. 5. Advise to use contraceptive measures during treatment.

1., 2., 4., & 5. Correct: Both cyclosporine and azathioprine are immunosuppressants. Clients should be taught to protect themselves from sources of infection. Vaccinations are not given to immunocompromised clients. Avoidance of crowds will decrease the client's chance of contact with infections, especially those spread by droplets. As a general rule, significantly immunosuppressed clients should not receive live vaccines. Cyclosporine may cause growth of extra tissue in your gums so use a dentist regularly. These drugs are teratogenic. Clients should avoid pregnancy while on these medications. 3. Incorrect: Drinking 3 liters of fluids per day will not prevent renal impairment.

An alcoholic client has agreed to take disulfiram 250 mg PO daily. The nurse recognizes that education has been successful when the client makes which statements? 1. "If I decide to stop taking disulfiram, I should not ingest any alcohol for at least 2 weeks or I will have a reaction." 2. "I must read labels carefully so that I know that alcohol is not an ingredient." 3. "I am allowed to eat chili made with beer since the alcohol evaporates from the chili with prolonged cooking." 4. "This medication is not a cure. I still need to attend therapy sessions." 5. "I should avoid eating a lot of chocolate while on this medication."

1., 2., 4., & 5. Correct: Disulfiram works by reacting with alcohol to produce negative side effect which may last up to two weeks after discontinuation of the drug.The client should not consume any alcohol including hidden alcohol such as mouthwash and cough syrups. Disulfiram is not a cure for alcoholism. It is used in combination with supportive care and psychotherapy. Disulfiram can increase the side effects of caffeine, so avoid chocolate and other caffeine containing substances. 3. Incorrect: Not even a small amount of alcohol can be ingested. This includes sauces and foods made with alcohol vinegar and vanilla extract. Meat holds on to alcohol, so chili with beer in the sauce should not be consumed. Additionally, do not use after shave, cough mixtures, or rubbing alcohol.

What nursing interventions should a nurse initiate for a client diagnosed with pyelonephritis? 1. Monitor urine for dark, cloudy, foul smelling urine. 2. Place client on intake and output monitoring. 3. Decrease fluid intake to 1 liter/day. 4. Advise client that urine may change color with administration of nitrofurantoin. 5. Monitor for hypotension, tachycardia, fever.

1., 2., 4., & 5. Correct: With pyelonephritis urine will be dark, cloudy and foul smelling due to the bacteria. Anytime a client has a renal problem, that client should be placed on I&O. Nitrofurantoin, an antibiotic, will turn the urine brown. Monitor for septic shock, a complication of pyelonephritis. S/S include hypotension, tachycardia, and fever. 3. Incorrect: Fluid intake should be increased to 2-3 liters/day unless contraindicated. Option 1 is true. Cloudy or milky urine is a sign of a urinary tract infection, which may also cause a bad smell. Milky urine may also be caused by bacteria, crystals, fat, white or red blood cells, or mucus in the urine. Option 2 is true. Pyelonephritis is a kidney problem. Any renal problem needs to have intake and output measurements monitored. Option 3 is false. The kidneys need to be flushed out to get rid of the bacteria. Fluid intake should be increased to 2-3 liters/day unless contraindicated. Option 4 is true. Nitrofurantoin, an antibiotic, will turn the urine brown. Option 5 is true. Shock is a serious complication of pyelonephritis so the nurse must monitor for this.

A nurse manager notices that unit nurses consistently forget to ask clients to rate their pain level on a scale of 0-10. What strategies could the nurse manager initiate to improve performance? 1. Provide "just in time" posters outlining the critical importance and steps in pain assessment. 2. Conduct brief in-services for each shift. 3. Write nurses up when pain level scale is not utilized. 4. Ensure that a complete and clear performance standard exists. 5. Assess nurses' reasons for not using pain level scale.

1., 2., 4., 5. Correct: If nurses have been provided the knowledge and performed the skill before, but the opportunity to perform is presented infrequently, a different type of education is required. This may take the form of "just in time" tools such as posters or guidelines outlining the critical steps in performing the skill. Brief in-services, videos, or DVDs available on the unit may also be effective in providing on the spot refreshers. Ensuring that performance standards exist, are clear and complete, and that they are readily available to staff is essential. Of course, nurses must have read the standards and understand them. The first step in correcting a performance gap is to understand what the difference is between the behavior being exhibited and what the expectations are. Always assess why staff are doing or not doing what is needed for clients. There may be a lack of knowledge or there may be a sense of non-importance. Perhaps a process is not working properly. So assessment is first. 3. Incorrect. This is not the most effective way of improving performance as it is considered punitive. If the above listed strategies are not effective, formal reporting of the behavior may be necessary. Option 3, right? Yes. I just told you that performance improvement is not to be a punitive process, so you wouldn't want to begin by writing nurses up for not using the pain level scale. Now look at Option 1. The goal is to implement different strategies to increase appropriate performance. So will posters remind nurses to use a pain scale? Sure it will. So this option is true. Option 2. Teach. Yes, the nurse manager can teach staff about the importance of rating pain objectively. Option 4. Put the performance standard in writing. Make it a policy. Yes this is true. Option 5. Assessing why a pain scale is not being used in the first place is an excellent way to begin the process. Gather data to guide how to improve performance. True.

What symptoms of meningeal irritation would the nurse anticipate when performing an assessment on a newly admitted client with a diagnosis of bacterial meningitis? 1. Positive Kernig's sign 2. Positive Brudzinski's sign 3. Presence of Babinski's reflex 4. Photophobia 5. Severe headache 6. Nuchal rigidity

1., 2., 4., 5., & 6 Correct: Brudzinski's sign is the involuntary lifting of the legs when the neck is passively flexed (head is lifted off the examining surface). Kernig's sign is positive when the thigh is bent at the hip and knee at 90 degree angles and attempts to extend the knee are painful, resulting in resistance. Both of these signs are thought to indicate meningeal irritation. These seem to be caused when the motor roots become irritated as they pass through inflamed meninges, and the roots are brought under tension. Photophobia (sensitivity to bright light), severe, unrelenting headache, and nuchal rigidity (stiff neck) are all believed to be due to irritation of the meninges. 3. Incorrect: Babinski reflex is a normal reflex in infants up to age 2, but is a pathological reaction in adults. It is often indicative of severe damage to the central nervous system but is not indicative of meningeal irritation.

A client who is at high risk for developing a stroke has been advised to follow a Mediterranean type diet by the primary healthcare provider. Which food choices, if selected by the client, would indicate to the nurse that the client understands this diet. 1. Grilled eggplant 2. Purple grape juice 3. Bacon 4. Cashews 5. Skim milk 6. Salmon

1., 2., 4., 5., & 6. Correct: It is reasonable to counsel clients to follow a Mediterranean-type diet over a low-fat diet. The Mediterranean type diet emphasizes vegetables, fruits, and whole grains and includes low fat dairy products, poultry, fish, legumes, and nuts. It limits intake of sweets and red meats. 3. Incorrect: Substitute fish and poultry for red meat. When eaten, make sure it's lean and keep portions small (about the size of a deck of cards). Also avoid sausage, bacon and other high-fat meats. The Mediterranean diet is based largely on plant based. In addition to fruits, whole grains, olive oil, cheese, yogurt and fish, the Mediterranean diet places a major emphasis on vegetables. Emphasis is placed on fruits and vegetables, whole grains, legumes and nuts. Butter should be replaced with healthy fats such as olive oil and canola oil. Herbs and spices instead of salt is used to flavor foods. Red meat is limited to no more than a few times a month and replaced with fish and poultry at least twice a week. Drinking red wine in moderation is acceptable or purple grape juice. Getting plenty of exercise is emphasized as well. Nuts are high in fat, but most of the fat is not saturated. Because nuts are high in calories, they should not be eaten in large amounts — generally no more than a handful a day. Avoid candied or honey-roasted and heavily salted nuts. Fatty fish such as mackerel, lake trout, herring, sardines, albacore tuna and salmon are rich sources of omega-3 fatty acids. Limit high fat dairy products such as whole or 2 percent milk, cheese and ice cream. Switch to skim milk, fat free yogurt and low-fat cheese. A glass of wine at dinner is acceptable if approved by the client's primary healthcare provider. If the client doesn't drink alcohol, purple grape juice may be an alternative. Eggplant, which is one of the few purple vegetables, is a good addition to the Mediterranean diet. This vegetable can be cut into thin slices and used in place of noodles when making lasagna or grill the eggplant slices and top them with fresh tomatoes and a drizzle of olive oil.

Which signs and symptoms does the nurse expect to see in a client admitted to the medical unit with Parkinson's disease? You answered this question Incorrectly 1. Blank affect. 2. Decreased ability to swing arms. 3. Waddling gait. 4. Walking on toes. 5. Pill-rolling tremor. 6. Stiff muscles.

1., 2., 5., & 6. Correct: Classic characteristics of Parkinson's disease include a blank facial expression, forward tilt in the posture, slow/slurred speech, tremor, and a short shuffling gait. These symptoms also are manifested by a decreased ability to swing the arms and stiff muscles. 3. Incorrect: This is a sign of Duchenne Muscular Dystrophy. The client with Parkinson's disease has a shuffling gait. 4. Incorrect: This is a sign of Duchenne Muscular Dystrophy. The client with Parkinson's disease has a shuffling gait. Parkinson's disease is a progressive disorder of the nervous system that affects movement. It develops slowly and symptoms may be barely noticeable. For instance, the client may have a tremor of just one hand. Stiffness or slowing of movement may be noted. In the early stages, the client's face may show little or no expression, or their arms may not swing when walking. Their speech may become soft or slurred. These symptoms worsen as the condition progresses over time. So now you can look at the options and see some of the signs and symptoms, can't you? Yes. From this, did you say options 1, 2, and 6 are correct? Good. Let's look at the other options to se which others may be correct as well! Option 3. Waddling gait. False. This is a sign of Duchenne Muscular Dystrophy. The client with Parkinson's disease has a shuffling gait. Option 4. Walking on toes. This is false as well and is a sign of Duchenne Muscular Dystrophy. Option 5. Pill-rolling tremor. Well, we said that the client has a slight tremor, and that tremor looks like they are rolling a pill or something small in their hand. So, this is correct. So the correct options are 1, 2, 5, and 6. Got it? Great!

A nurse is in the mall when a shopper who suddenly becomes non-responsive. Obtaining an available AED, the nurse would initiate what emergency interventions? 1. Clear everyone before shock. 2. Turn on the machine. 3. Initiate shock immediately. 4. Place pads on client's chest. 5. Await arrival of paramedics. 6. Shave client's chest.

1., 2.,& 4. Correct: It is always critical to be certain rescuers are "clear" before providing a shock; that is, not physically in contact with the client or any equipment that is touching the client. It is also important to verify the machine is turned on prior to use. Additionally, the electrode pads need to be properly applied in the correct location on the client's chest, as depicted on the AED and the pads, in order for the machine to analyze the client's rhythm and determine action. 3. Incorrect: A shock is only delivered when the machine determines it is necessary to do so. 5. Incorrect: The purpose of the AED is to provide the public a means of providing emergency care to those experiencing a cardiovascular emergency while awaiting the arrival of qualified medical personnel. 6. Incorrect: Shaving would waste time and risk injury to the client.

Which tasks would be appropriate for the nurse to delegate to an unlicensed assistive personnel (UAP)? You answered this question Correctly 1. Prepare a client's room for return from surgery. 2. Observe for pain relief in a client after receiving acetaminophen with codeine. 3. Assist a client with perineal care after having diarrhea. 4. Clean nares around a client's nasogasttric (NG) tube. 5. Pour a can of tube feeding into a client's percutaneous endoscopic gastrostomy (PEG).

1., 3, & 4. Correct. These are appropriate tasks for an UAP to complete. The UAP can provide hygiene needs to a client such as perineal care and cleaning of the nares. Also, making a surgical bed for the client returning from surgery is a basic procedure. 2. Incorrect. The UAP cannot assess or evaluate or even monitor the effectiveness of pain medication. That is what you are asking the UAP to do here. The client has received a narcotic and you have asked the UAP to evaluate the effectiveness of the medication. 5. Incorrect. Administering tube feeding into a PEG tube is beyond the scope of practice for the UAP. This is a procedure which requires a licensed personnel. Catheter placement must be confirmed, client identity checked, tube site flushed with water or sterile water and flow rate determined. Before we look at this question, I want you to remember this: The RN cannot delegate assessment, evaluation, plan of care development or teaching to an LPN or UAP. And don't forget to follow the 5 rights of delegation: Right task, right person, right circumstance, right direction, right supervision. What can the UAP do? The UAP can assist the stable client with activities of daily living and can collect routine data, such as vital signs, and I&O. So let's look at the options now. Option 1. Is a special skill needed to prepare a client's room for return from surgery? No. Making a surgical bed is a basic procedure that the UAP has the skill to complete. The UAP can also place and set up any necessary equipment needed for the client, such as a vital sign machine, suction equipment, etc. Option 2. The UAP cannot assess, evaluate or even monitor the effectiveness of pain medication. That is what you are asking the UAP to do here. The client has received a narcotic and you have asked the UAP to evaluate the effectiveness of the medication. The UAP does not have the knowledge or training to complete this task. Option 3. The UAP can provide hygiene needs to a client like perineal care. Remember the UAP can help with activities of daily living. Option 4. The UAP can provide hygiene needs to a client like cleaning of the nares. This is part of daily hygiene. Option 5. Administering tube feeding into a PEG tube is beyond the scope of practice for the UAP. This is a procedure which requires a licensed personnel. Catheter placement must be confirmed, client identity checked, tube site flushed with water or sterile water and flow rate determined. The UAP is not allowed to perform these observations and implement this procedure. They made it sound like you were just pouring a can of tube feeding in, simple right! Don't let simple words talk you into selecting the wrong answer.

Which clients should the nurse recommend receive the human papillomavirus (HPV) vaccine? 1. Twelve year old male. 2. Nine year old female. 3. Twenty-five year old bisexual male. 4. Twenty-two year old female with compromised immune system. 5. Twenty-six year old male who has not received the HPV vaccine.

1., 3, & 4. Correct: The HPV vaccine is recommended for preteen boys and girls at age 11 or 12 so they are protected before ever being exposed to the virus. Young women can get HPV vaccine through age 26, and young men can get vaccinated through age 21. The HPV vaccine is recommended for any man who has sex with men through age 26 and for men with compromised immune systems through age 26 if they did not get HPV vaccine when they were younger. The HPV vaccine is recommended for men and women with compromised immune systems through age 26. 2. Incorrect: The HPV vaccine is recommended for preteen boys and girls at age 11 or 12 so they are protected before ever being exposed to the virus. Young women can get HPV vaccine through age 26, and young men can get vaccinated through age 21. 5. Incorrect: Catch up vaccines are recommended for males through age 21 and for females through age 26 if they did not get vaccinated when they were younger.

A pediatric nurse is teaching a group of new parents about what to expect regarding their infants eyes and vision. What points should the nurse include? 1. At 4 weeks of age, the infant should be able to gaze at objects. 2. Infants should have tears by the age of 1 month. 3. Visual acuity is about 20/300 at 4 months of age. 4. During the first 2 months of life, infant's eyes may appear to be crossed. 5. Depth perception begins around the 5th month of age.

1., 3, 4, & 5. These statements are correct. Babies learn to see over a period of time, much like they learn to walk and talk. They are not born with all the visual abilities they need in life. The ability to focus their eyes, move them accurately, and use them together as a team must be learned. Also, they need to learn how to use the visual information the eyes send to their brain in order to understand the world around them and interact with it appropriately. At birth, babies have not yet developed the ability to easily tell the difference between two targets or move their eyes between the two images. Their primary focus is on objects 8 to 10 inches from their face or the distance to parent's face. During the first months of life, the eyes start working together and vision rapidly improves. Eye-hand coordination begins to develop as the infant starts tracking moving objects with his or her eyes and reaching for them. By eight weeks, babies begin to more easily focus their eyes on the faces of a parent or other person near them. For the first two months of life, an infant's eyes are not well coordinated and may appear to wander or to be crossed. This is usually normal. However, if an eye appears to turn in or out constantly, an evaluation is warranted. Babies should begin to follow moving objects with their eyes and reach for things at around three months of age. When taking photos, the parent will begin to notice the baby blinking at the flash. Visual acuity is still in the 20/200 to 20/400 range. Infants are beginning to recognize familiar people, and by 3 months they should be reaching for things. By 4 months the child's eyes should be working together. He or she should begin to follow objects and people, recognize familiar objects, as well as watch parents' faces when being talked to.

Which assessment findings would the nurse expect when assessing a client for dementia? 1. Slow progressive deterioration of cognitive functioning 2. Decreased level of consciousness 3. Personality changes 4. Difficulty paying attention 5. Suicidal thoughts and sadness

1., 3. & 4. Correct: Dementia is characterized by a slow onset of symptoms over months to years. Dementia progresses to noticeable changes in personality. Dementia progresses to noticeable changes in attention span. 2. Incorrect: Dementia is progressive deterioration of cognitive functioning with no change in consciousness. 5. Incorrect: Sadness and suicidal thoughts are signs of depression. You must know about dementia and the assessment findings in a client with dementia. This is a straight forward question and determines your knowledge of dementia. Option 1: Yes. There is a slow onset and progression occurs over time. Dementia causes memory loss and loss of other mental abilities. This affects cognitive functioning! Option 2: No. There is no change in consciousness. Consciousness is the state of being awake and aware of one's surroundings. Option 3: Good choice. The client with dementia has personality changes and may become more irritable, less motivated, easily upset and less talkative. Option 4: Yes. Attention span and concentration will decrease. The client will have difficulty staying focused. Option 5: No. This client may have frequent mood swings and increased anxiety. Also in latter stages may become paranoid. But suicidal thoughts and sadness are not part of dementia.

A nurse is planning a teaching session for a group of clients diagnosed with irritable bowel syndrome. What points should the nurse include to help the clients control symptom flare-ups? 1. If you are constipated, try to make sure you have breakfast. 2. Avoid low fat foods. 3. If you think a certain food is a problem, try cutting it out of your diet for about 12 weeks. 4. Drinks containing caffeine are likely to contribute to symptoms. 5. Foods such as broccoli and cabbage are good sources of fiber.

1., 3. & 4. Correct: If you are constipated, try to make sure you eat breakfast, as this is the meal that is most likely to stimulate the colon and give you a bowel movement. If you think a certain food is a problem, try cutting it out of your diet for about 12 weeks. (If you suspect more than one, cut out one at a time so you know which one causes you problems.) If there's no change, go back to eating it. The foods most likely to cause problems are: Insoluble (cereal) fiber; Coffee/caffeine; Chocolate; Nuts 2. Incorrect: Avoid meals that over-stimulate the gut, like large meals or high fat foods. 5. Incorrect: Broccoli and cabbage are common gas-producing foods that can cause abdominal distention and flatulence.

A client with diabetes mellitus has a newly prescribed insulin pump. Which statements made by the client indicate understanding of an insulin pump? 1. "I will attach the pump to my waistband or wear it in the pocket of my pants." 2. "I can eat whatever I want as long as I cover the calories with sufficient insulin." 3. "I may take my insulin pump off when I exercise." 4. "I need to check my blood glucose level several times a day." 5. "I have to change the catheter at the end of the pump every week."

1., 3. & 4. Correct: Insulin pumps are externally worn on clothing or in a pocket. Pumps can be easily disconnected for limited periods such as for showering, exercise, and sexual activity. Clients must assess their blood glucose level several times daily. 2. Incorrect: Clients are prescribed a specific caloric intake and insulin regimen, and maintaining as much consistency as possible in the amount of calories and carbohydrates is essential. 5. Incorrect: The needle or catheter attached at the end is changed at least every three days.

Following vaginal birth, a neonate has a large area of diffuse swelling over the left occiput that crosses the sagittal suture line. When discussing this finding with the neonate's parents, which statements by a nurse are accurate? 1. "No treatment will be required to resolve swelling." 2. "Due to the swelling, hyperbilirubinemia may occur." 3. "The swelling lies above the periosteum that covers the skull bone." 4. "Pressure on the fetal head before delivery caused the swelling." 5. "Your infant has a collection of blood under the skull bone."

1., 3. & 4. Correct: Swelling over the left occiput that crosses the sagittal suture (caput succedaneum) requires no treatment, is swelling that overlies the periosteum, and is caused by pressure on the fetal head before delivery. 2. Incorrect: Caput succedaneum is a collection of serosanguineous fluid, not blood. Hyperbilirubinemia is associated with cephalohematoma. 5. Incorrect: Caput succedaneum is a collection of serosanguineous fluid, not blood. Cephalohematoma is a collection of blood under the skull bones and does not cross the suture lines.

The family of an elderly woman is concerned that their mother is not getting restful sleep. As a result, the family members' sleep is disturbed. Which questions would be important for the nurse to ask? 1. Has there been any change in your mother's state of health? 2. Can family members take naps during the day? 3. Does she take routine diuretics? 4. Has there been an increase in noise levels? 5. Can the family take turns in managing the mother's sleep problems?

1., 3. & 4. Correct: There may be a physical reason for the difficulty sleeping, perhaps pain or presence of an infection. Diuretics should be scheduled early in the day so as not to interfere with sleep. Perhaps there has been a change in medication schedule. Changes in the sleep environment, such as an additional TV in the home or other noise, may impact sleep. 2. Incorrect: This may be necessary; however, the nurse is working toward helping the mother of the family to sleep better. 5. Incorrect: The family may need to do this over time; however, the focus is to help the mother of the family to sleep better.

A nurse is educating the family of a client in the middle stages of Alzheimer's disease how to encourage independence during meals. What points should the nurse include? 1. Serve meal in a quiet environment 2. Give 30 minutes to eat 3. Serve finger foods 4. Serve one dish at a time 5. Do not worry about neatness

1., 3., 4., & 5. Correct: Limit distractions by serving meals in quiet surroundings, away from the television and other distractions. Too many foods at once may be overwhelming. Simplify by serving one dish at a time. For example, mashed potatoes followed by meat. Serve finger foods, which are foods easy to pick up to eat. Do not worry about neatness. Let the person feed self as much as possible. Consider plates with suction, built-up rims and no spill glasses to allow users to more easily place food on their utensils. 2. Incorrect: Give the person plenty of time to eat. Remind client to chew and swallow carefully. Keep in mind that it may take an hour or longer to finish eating.

An intubated client admitted to the intensive care unit appears anxious and fearful of the equipment in the room. The nurse observes this and takes the time to explain each piece of equipment and its role in providing care to the client. How does this action demonstrate client advocacy? 1. Providing information to the client. 2. Promoting client compliance. 3. Providing emotional support. 4. Ensuring the client's wishes for treatment are followed. 5. Fostering a sense of security.

1., 3. & 5. Correct: Client advocacy has been described in many different ways and involves many things such as assistance in gaining needed healthcare, assuring quality of care, protection of client's rights, and simply serving as a mediator between the client and the healthcare system as a whole. Client advocacy involves regular communication in which the nurse explains what is being done or likely to happen, reasons for tests or procedures, and simplifying medical terminology into words that can be easily understood. Emotional support is also an aspect of client advocacy that the nurse should employ. The nurse acts as a client advocate by providing information to the client to alleviate fear of the unfamiliar equipment and by fostering a sense of security. 2. Incorrect: This question addressing client advocacy is not related to client compliance. Client compliance may improve if the nurse served as an appropriate client advocate. However, promotion of compliance is not a basic part of advocacy. 4. Incorrect: This question addressing client advocacy is not related to client's healthcare treatment wishes. This would be related to the client's advance directive. Client advocacy has been described in many different ways and involves many things such as assistance in gaining needed healthcare, assuring quality of care, protection of client's rights, and simply serving as a mediator between the client and the healthcare system as a whole. Look at option 1. True or False? True. Client advocacy involves regular communication in which the nurse explains what is being done or likely to happen, reasons for tests or procedures, and simplifying medical terminology into words that can be easily understood. Option 2, promoting compliance. False. Promotion of client compliance is not a basic part of advocacy. However, client compliance may improve if the nurse served as an appropriate client advocate. What about option 3, providing emotional support? This is true. Emotional support is also an aspect of client advocacy that the nurse should employ. Option 4? False. This question is not asking about advance directives which is related to client's healthcare treatment wishes. Ok, option 5, fostering a sense of security? True. The nurse acts as a client advocate by providing information to the client to alleviate fear of the unfamiliar equipment and by fostering a sense of security.

The nurse is caring for a client hospitalized with dissociative amnesia. Which nursing interventions are appropriate for this client? 1. Obtain client likes and dislikes from family members. 2. Expose the client with data regarding the forgotten past. 3. Expose client to stimuli that was a happy memory of the past. 4. Hypnotize the client to help restoration of memory. 5. Ensure client safety.

1., 3. & 5. Correct: Considering likes and dislikes may help the client to remember. Using information to expose the client to stimuli that were happy memories may help the client remember. The client's disorder may lead to inattention to safety. Think safety first! 2. Incorrect: Do not expose the client to data regarding the forgotten past. Clients who are exposed to painful information from which the amnesia is providing protection may decompensate even further into a psychotic state. Dissociative amnesia is marked by an inability to recall important personal information, often traumatic or stressful in nature. 4. Incorrect: This is not a nursing function. Hypnosis is not in the nurse's scope of practice. First, think safety. Then, think of nursing interventions that will help relieve symptoms. Option 1: Do it. Determining the client's likes and dislikes will help the nurse plan client care. Also may help the client begin to remember and process the painful memories. Then the client can develop new coping skills. Option 2: No. The amnesia is protecting the client from memories that are traumatic. The memories can resurface after being triggered. Nursing interventions must focus on helping the client express and process painful memories in appropriate ways. This is not the way. Option 3: Yes! You want the client to remember happy times. This is therapeutic! Option 4: Oh no! The stem asks for a nursing intervention. Do not choose this option. Hypnosis may be a treatment for dissociative amnesia but is not a nursing intervention. Read the stem carefully. Option 5: Excellent. A priority is client safety. This disorder may be so distressing and cause behavior problems and confusion. An appropriate nursing intervention is client safety.

A client requires external radiation therapy. The nurse knows external radiation may cause which problems? 1. Pancytopenia 2. Leukocytosis 3. Erythema 4. Fever 5. Fatigue

1., 3. & 5. Correct: Effects of radiation therapy include, but are not limited to pancytopenia (marked decrease in the number of RBCs, WBCs and platelets), erythema (redness of the skin), and fatigue. 2. Incorrect: Leukocytosis is an increase in WBCs. External radiation causes pancytopenia which is a decrease in the number of blood cells including WBCs. 4. Incorrect: Fever is not typically seen with external radiation.

A nurse is planning care for a laboring client who is about to be started on oxytocin. What interventions should the nurse include in this plan of care? 1. Piggy back oxytocin into main IV fluid. 2. Monitor for early decelerations. 3. Discontinue if contractions last longer than 60 seconds. 4. Maintain one on one care. 5. Check fetal heart tones hourly

1., 3., & 4. Correct: The oxytocin is piggy backed into the main IV fluid, so when the nurse discontinues the medication, the main IV fluid is quickly resumed. Contractions should be at a rate of 1 every 2-3 minutes with each lasting no more than 60 seconds. Hyperstimulation of the uterus can occur and result in fetal distress. One on one care is needed since complications such as fetal distress and uterine rupture can occur. 2. Incorrect: External continuous fetal monitoring should begin prior to oxytocin administration. A reactive fetal heart rate tracing should be obtained over 30 minutes. 5. Incorrect: Continuous fetal monitoring is must be maintained during oxytocin administration to fetus is not experiencing distress in utero with contractions.

A nurse enters the operating room (OR) with artificial fingernails in place. What should the charge nurse explain to the nurse? 1. Pathogenic bacteria can be found on the fingertips of those who wear artificial fingernails. 2. Artificial fingernails are allowed to be worn in the OR. 3. Fungal growth can occur under the artificial fingernail, thus increasing the risk of surgical site infection to the client. 4. A more vigorous scrub is required if artificial fingernails are worn. 5. Long fingernails and artificial fingernails increase microbial load on the hands.

1., 3. & 5. Correct: The variety and amount of pathogenic bacteria cultured from the fingertips of those wearing artificial fingernails is greater than from those with natural nails, both before and after handwashing. Fungal growth occurs frequently under artificial fingernails because moisture gets trapped between the natural fingernail and artificial fingernail, providing a medium for growth. Natural nails should be less than 1/4 inch (6.35 mm) long. 2. Incorrect: Artificial fingernails are not allowed in the OR. They are known to harbor gram-negative microorganisms and fungus. 4. Incorrect: A vigorous scrub of the fingernails does not decrease the moisture under the nails. Chipped nail polish, or if worn longer than 4 days, should be removed due to increased likelihood of harboring bacteria. Remember client safety is always a priority. This question is clarifying whether artificial fingernails can be worn in the operating room. So let's look at the options. Option 1 is a true statement. Pathogenic is a medical term that describes viruses, bacteria, and other types of germs that can cause some kind of disease. These bacteria are commonly found under artificial nails. Option 2 is false. Because artificial fingernails harbor pathogenic bacteria, they are not allowed in the surgical suite. Option 3 is true. Pathogenic bacteria cause disease, so fungal and bacterial growth under artificial fingernails can contribute to infection of the client with a surgical wound. Option 4 is false. A vigorous scrub will not decrease the bacterial and fungal count under the fingernails enough to prevent the spread of infection. Option 5 is true. Both long, natural nails and artificial fingernails increase the risk of infection. fingernails should be no longer than 1/4 inch (6.35 mm) long.

The women's health charge nurse is making assignments for the next shift. The unit is short one staff member and will receive a nurse from the medical surgical unit. Which clients should be assigned to the medical surgical nurse? 1. Total abdominal hysterectomy (TAH). 2. Client post C-section to be discharged home. 3. Breast Reduction. 4. Vaginal delivery of fetal demise. 5. 28 week gestation of bed rest. 6. Bladder suspension with anterior and posterior repair.

1., 3. and 6. Correct: When a nurse is pulled to another unit, always assign them like a brand new nurse. A client with a TAH, Breast reduction or bladder suspension require basic post-operative care. These are within the scope of knowledge of a brand new nurse with medical-surgical knowledge. 2. Incorrect: This client is going to require specific discharge teachings related to her care, medication and care of the newborn. Specialized training is necessary here. 4. Incorrect: The nurse caring for this client needs to have skills related to postpartum care as well as psychological care of this type of loss. A pulled nurse from the medical-surgical floor will not be prepared to assist this client with all her needs. 5. Incorrect: This client is on bedrest for a reason and is hospitalized for a reason. Skilled assessment to identify change in status or denote impending complications is essential. This is not appropriate for the pulled medical-surgical nurse. The key to this question is the fact that, although the nurse is an experienced medical-surgical nurse, the nurse is not experienced in gynecological nursing. The client who could receive care or a treatment on a medical-surgical floor should be assigned to the nurse. So, let's look at the client options we have been provided. Option 1 is a client who had a total abdominal hysterectomy. Can the medical surgical nurse take care of a surgical client? Yes. Look at option 2. A client who had a c-section to be discharged. This client is going to require specific discharge teachings related to her care, medication and care of the newborn. Specialized training is necessary here. So do not assign the medical-surgical nurse to this client. What about option 3. The client post breast reduction. Is any specialized knowledge needed to take care of this post op client beyond what a medical-surgical nurse would know? No. So this nurse could care for this client. Option 4 is a client who lost her baby after vaginal delivery. The nurse caring for this client needs to have skills related to postpartum care as well as psychological care of this type of loss. A pulled nurse from the medical-surgical floor will not be prepared to assist this client with all her needs. Option 5 is a client who is at 28 weeks gestation who is on bedrest. What do you think? No. This client is on bedrest for a reason and is hospitalized for a reason. Skilled assessment to identify change in status or denote impending complications is essential. This is not appropriate for the pulled medical-surgical nurse. What about option 6? Client who had a bladder suspension. Is specialized knowledge needed that only an OB/GYN nurse should know how to care for? No. Bladder suspension requires basic post-operative care.

The nurse is planning to teach a group of young women who want to become pregnant. What information should be included as recommendations to increase the chances of having a healthy baby? 1. Take 400 micrograms of folic acid every day. 2. Limit alcohol to 1 glass per day. 3. Avoid smoking. 4. Take the flu vaccine during flu season. 5. Start prenatal care by 3 months of pregnancy

1., 3., & 4. Correct: Folic acid is a B vitamin. If a woman has enough folic acid in her body at least a month before and during pregnancy, it can help prevent neural tube defects. Smoking can lead to premature birth, cleft lip or cleft palate, and infant death. The flu shot given during pregnancy has been shown to protect mom and baby (up to 6 months old) from flu. 2. Incorrect: When a woman drinks alcohol, so does her unborn baby. This can cause the baby to be born with fetal alcohol spectrum disorder. 5. Incorrect: A woman should be certain to see her healthcare provider when planning pregnancy and start prenatal care as soon as she thinks she is pregnant.

The nurse is planning discharge teaching for the family of a 6 month old client with heart failure. is in the hospital. Which instructions about feeding should the nurse include in discharge teaching? 1. Feed when baby wakes up 2. Let the baby cry so you know the baby is hungry 3. Report to the primary healthcare provide if baby sweats during feedings 4. Feed large thickened feeding every 3 hours to increase calorie intake 5. Feed when baby is well rested 6. Use a special cardiac nipple with a small opening

1., 3., & 5. Correct. Feeding when baby awakens and when well rested will decrease workload of the heart. If baby starts to sweat, then baby is having to work too hard for feeding. 2. Incorrect: Would increase the workload of the heart. You want to minimize crying, feed before crying. 4. Incorrect: Small frequent feedings are best to increase caloric intake without overstimulating the baby.

A new nurse is documenting in a client's electronic record. Which documentation would the charge nurse evaluate as appropriate documentation by the nurse? 1. Forty year old admitted with diagnosis of cholecystitis to room 410 for surgical services. 2. Appears to be having abdominal discomfort. 3. Permit signed for laparoscopic cholecystectomy after discussing procedure with surgeon. 4. Pre-op diazepam 10.0 mg given po 5. Transferred to surgical suite per stretcher with side rails up, in stable condition.

1., 3., & 5. Correct. These are written correctly. The first entry provides the age, provides the diagnosis, room number, and plan for care. this gives a "snap shot" of the admission. Option 3 is documentation of informed consent for surgery. It states that the consent is signed, the surgery to be performed, and very importantly, that the client is consenting to surgery after the surgeon discussed the procedure. Option 5 appropriately documents a transfer. It presents where the client was transferred, how they were transported, and the condition upon their transfer. 2. Incorrect. "Appears" is subjective. Pain should be assessed in an objective manner, such as by using a pain scale. The nurse should not use subjective documentation of the client's pain. 4. Incorrect. Do not use trailing zeros after a decimal point. Always lead a decimal point with a zero (0.5). These are safety issues. Trailing zeros are identified on the Joint Commission on the Accreditation of Hospitals Organization (JCAHO) "Do Not Use" list. The placement of a zero after a decimal point could lead to the inadvertent administration of a medication ten times the prescribed dose if the decimal point was not noted or could not be seen. Nurses should always follow JCAHO standards and guideline for documentation and nursing care.

Which of the following should the nurse teach regarding nutrition for a client with celiac disease? 1. Gluten is a protein found in wheat and oats. 2. A gluten intolerant person can eat foods that are made with barley or rye. 3. Fruits can be eaten on a gluten free diet. 4. Gluten causes inflammation of the large intestines of people with celiac disease. 5. Accidentally eating a product containing gluten may result in abdominal pain and diarrhea.

1., 3., & 5. Correct: A gluten-free diet is used in the treatment of celiac disease. Gluten is a protein found in barley, oats, rye, and wheat. All products containing these grains are to be avoided. Rice and corn may be used. Fruits, vegetables, nuts, diary products and meats not prepared with gluten containing ingredients can be eaten. Accidentally ingesting food with gluten may result in abdominal pain and diarrhea. 2. Incorrect: The main starchy foods that a person can eat are made with rice, corn, potatoes, quinoa, and Tapioca. The gluten intolerant clients can not eat barley and rye. Gastrointestinal pain and diarrhea may occur. 4. Incorrect: Gluten causes inflammation in the small intestines of people with celiac disease. Eating a gluten-free diet helps people control their signs and symptoms and prevent complications.

A nurse is planning to provide education to a client wishing to breastfeed. What instructions should the nurse include when teaching this client? 1. Apply warm compresses to breast just prior to breastfeeding. 2. Establish a routine for breastfeeding. 3. Massage breasts during feeding. 4. Wear well-fitting bra continuously for first 24 hours after birth. 5. Wash hands before breastfeeding.

1., 3., & 5. Correct: Applying warm compresses or taking a warm shower prior to breastfeeding will help the let-down reflex. Massaging breasts during feeding can help with emptying. Emphasize the importance of hand hygiene prior to breastfeeding to prevent infection. 2. Incorrect: Allow newborns to nurse on demand. Allow newborns to feed 15-20 minutes per breast or until the breast softens. Begin the next feeding session on the breast that was not completely emptied. 4. Wear well fitting bra continuously for at least 72 hours after birth to avoid milk stasis. Option 1 is true. Warm compresses will help with the let-down of milk. Option 2 is false. Newborns should be fed on demand as some feed every 1.5 hours. Option 3 is true. Massaging the breast will help to express the milk. Option 4 is false. A well fitting bra should be worn continuously for at least 72 hours. Option 5 is true. Hand washing prevents infection.

A client wishing to stop smoking receives a prescription for bupropion from the healthcare provider. What educational points should the nurse include regarding this medication? 1. This medication can cause a false positive drug screening test. 2. Alcohol intake should be limited to two drinks per day. 3. Nicotine gum may be prescribed in addition to bupropion. 4. An increased interest in sexual activity occurs while taking this medication. 5. Smoking can continue for 1 week after starting this medication.

1., 3., & 5. Correct: Bupropion is an antidepressant medication prescribed for major depressive disorders and seasonal affective disorder. Bupropion is used to help people stop smoking by reducing cravings and other withdrawal effects. This medication can cause a false positive drug screening test so the client should inform the laboratory personnel that the client is taking bupropion. Nicotine patches or gum may be prescribed to further support smoking cessation treatment as bupropion is a nicotine free prescription. The client can continue to smoke for about 1 week after starting the medicine. A date to quit smoking during the second week of treatment is determined. The recommended length of bupropion therapy is seven to twelve weeks. 2. Incorrect: Drinking alcohol may increase risk of seizures. If the client drinks alcohol regularly, they should talk with the primary healthcare provider before changing the amount of alcohol intake. Bupropion can cause seizures in people who drink a lot of alcohol and then suddenly quit drinking when they start using the medication. 4. Incorrect: A common side effect is a decreased interest in sexual activity.

A nurse is teaching a client who has frequent urinary tract infections how to prevent future infections. What statement by the client would indicate to the nurse that treatment has been successful? 1. "I will go to the bathroom as soon as the urge to void hits me." 2. "It is important for me to drink five to six 8 ounce glasses of water every day." 3. "I should eat foods such as plums and prunes to increase the acidity of my urine." 4. "Nylon underwear should be worn when I am free from infection." 5. "When I clean after voiding, I will discard toilet paper after each swipe."

1., 3., & 5. Correct: Holding urine can lead to stasis of urine and increasing the risk for infection. Foods such as eggs, cheese, meat and poultry, whole grains, cranberries, plums and prunes, and tomatoes tend to increase the acidity of urine. Acidic urine is less likely to allow for bacterial growth. Discarding toilet paper after each swipe will decrease exposure and accidental introduction of bacteria into the urinary meatus. 2. Incorrect: Emphasize the importance of drinking eight to ten 8 ounce glasses per day. Water helps flush bacteria from the urinary tract. 4. Incorrect: Cotton underwear is recommended. The natural fibers work to wick moisture away from the skin which discourages yeast growth.

The nurse is caring for a client that requires lifting. What techniques should the nurse utilize to prevent injury to self and potentially the client? 1. Tighten stomach muscles. 2. Keep the knees straight. 3. Keep weight to be lifted close to body. 4. Bend at the waist. 5. Avoid twisting the body.

1., 3., & 5. Correct: Tightening the stomach muscles provides stability for the movement. Keeping the weight close to the body provides additional support and reduces the risk of a stretching type injury. When the body is in alignment, it is considered to be balanced. Therefore, twisting motions cause the body to be off balanced and make the nurse more susceptible to injury. 2. Incorrect: The trunk should be kept straight, and the knees should be bent. This provides an upright posture to maximize stability and reduce risk of injury. 4. Incorrect: The trunk should be kept straight, and the knees should be bent. This provides an upright posture to maximize stability and reduce risk of injury. Bending at the knees helps prevent back injuries. Remember client safety is always a priority. This question is asking about proper lifting techniques. So let's look at the options. Option 1: The core abdominal muscles will support your abdomen and your back when the nurse is lifting. True The nurse should tighten their abdominal muscles prior to lifting to decrease the possibility of straining the abdominal and back muscles. Tightening the abdominal muscles as the lift begins will result in the abdominal cavity sharing a greater portion of the the weight bearing than the back muscles. This will reduce the strain to the back muscles. Option 2: This is a false option. The trunk should be kept straight and the knees should be bent. This provides an upright posture to maximize stability and reduce risk of injury. The legs muscles are flexed to support the trunk during lifting which will decrease the strain to the lower back. Option 3: Keeping the weight close to the body provides additional support and reduces the risk of a stretching type injury. True This will reduce stress to the back by preventing the lower back from being the central point of support while lifting. Option 4: This is a false option. The trunk should be kept straight and the knees should be bent. This provides an upright posture to maximize stability and reduce risk of injury. The nurse should bend at the knees which helps to prevent back injuries. The lower back should be kept in it's usual arched position. Option 5: When the body is in alignment, it is considered to be balanced. True Therefore, twisting motions cause the body to be off balanced and make the nurse more susceptible to injury. The feet should pivot in the direction that the object is moved. This will eliminate the need to twist the weight.

The nurse is assisting a new mother with breastfeeding her newborn baby. The mother verbalizes concern that the baby is not getting adequate milk. Which observations by the nurse indicate adequate fluid intake? 1. Birth weight regained in 14 days 2. Fontanels soft and depressed 3. Pulse rate of 135/min 4. Six to eight wet diapers a day 5. Baby appears satisfied after feedings

1., 3., 4. & 5. Correct: Are all indicators of adequate fluid intake in a newborn. Gaining weight, a heart rate between 70 to 190 beats per minute (BMP), six to eight wet diapers a day and periods of contentment after feedings alternate with periods of wakefulness indicate adequate breast feeding. 2. Incorrect: Fontanels should be soft, firm and flat. A depressed or sunken fontanel may indicate dehydration. Dehydration is one of the major causes of sunken fontanels.

The nurse is preparing to speak to a group of clients at the community center about influenza. Which risk factors for influenza complications would be included in the session? 1. Age over 65 years. 2. History of grand mal seizures 3. Diabetes 4. Renal disease 5. Clients who reside in a nursing home.

1., 3., 4. & 5. Correct: Clients who are over the age of 65, have diabetes, have renal disease, or who reside in a nursing home are all at risk for post-influenza complications. 2. Incorrect: A client who has grand mal seizures would not put the client at risk for flu complications. If the client has the other risk factors, then flu complications are more likely. This question is asking who is ask the greatest risk of developing complications from having the flu. Remember, the very young, the very old, and people with chronic diseases are more likely to have complications from the flu. So look at the options we are provided. What do you think? Did you say Options 1., 3., 4. & 5? Yes! Good for you. Clients who are over the age of 65, have diabetes, have renal disease, or who reside in a nursing home are all at risk for post-influenza complications. Their immunity is not as strong as others. Option 2. This is false. A history of grand mal seizures alone does not put a person at increased risk for complications from the flu. The key is History of. Maybe it has been years since the last seizure. If the client has the other risk factors, then flu complications are more likely.

The family of a bedfast 80 year old is providing care in the home. Which reports by the family indicate adequate understanding of interventions that will reduce the risk for skin breakdown? 1. I make sure that the sheets and the foam pad in the chair stay dry. 2. I will not encourage my parent to turn in the bed at night. 3. The perineal area should be kept dry and clean. 4. My parent eats 2 meals per day and drinks a supplement. 5. I may reposition my parent more than every 2 hours, if their preception of pressure is intact,

1., 3., 4. & 5. Correct: Keeping moisture from the skin is important for reducing the risk of skin breakdown. Keeping the client dry after using a bedpan is important to maintain healthy skin. As long as the intake of food is adequate, no further action is needed with nutrition. The client who is aware of sensations of pressure on the body has less risk of skin breakdown than those that have lost sensation. 2. Incorrect: If the client is not repositioned at night, the resulting pressure on one site may lead to skin breakdown, even when the sensations of pressure are intact. A client in a state of sleep would not be as likely to respond to sensations of pressure appropriately, so the family would need to do this for her. Before we review the options, let's look at the question. The key words in a question should be identified. The key words in this question are bedfast, 80 year old, understanding, interventions, and risk for skin breakdown. This is a select all question so there will be 2 or more options correct. Also each option stands alone with the question. After reviewing the question, look at each option and identify if it is true or false. Remember client safety is always a priority. This question is talking about interventions to prevent skin breakdown. So let's look at the options. Option 1 is true. The sheets and the foam pad should remain dry. The skin should not be exposed to a wet surface for a extended length of time. Keeping moisture from the skin is important for reducing the risk of skin breakdown. Option 2 is false. If the client is not repositioned at night, the resulting pressure on one site may lead to skin breakdown, even when the sensations of pressure are intact. A client in a state of sleep would not be as likely to respond to sensations of pressure appropriately, so the family would need to do this for her. Option 3 is true. Keeping moisture from the skin is important for reducing the risk of skin breakdown. The perineal area should cleansed and dried after each voiding and bowel movement. Option 4 is true. The nutritional status of the client should be evaluated. Appropriate levels of macronutrients (carbohydrates, proteins, and lipids) and micronutrients (vitamins and minerals) must be consumed to provide the for proper skin health. The client appears to consuming adequate food. Option 5 is true. The client who is aware of sensations of pressure on the body has less risk of skin breakdown than those that have lost sensation. If the client can tell their family, if they want to turn move.

The employee health nurse is designing a health promotion plan for a group of workers who have neck and back strain symptoms and repetitive movement pain from long periods of computer work. Which interventions should be included in the plan to reduce these symptoms? 1. Suggest that the workers place the keyboard and mouse close to the body. 2. Adjust computer screen to below eye level. 3. Drop and roll shoulders periodically. 4. Type with forearms parallel to the floor. 5. Keep elbows at the side when typing.

1., 3., 4. & 5. Correct: Keeping the keyboard and mouse close keeps the body in a more neutral position. Dropping the shoulders puts them in a relaxed position. As tension increases, the shoulders tend to rise. This position keeps the neck and back from being stretched and keeps the body in a more neutral position. This allows the shoulders to stay in a more neutral position. 2. Incorrect: The computer screen should be at eye level so that the neck does not become strained as easily. So what will decrease back and neck pain and repetitive movement pain. Body parts need to be maintained in their neutral position. Option 1 is true. Outstretched arms will put more pressure on the shoulders and back. Option 2 is false. If you constantly look down, more stress is being placed on the neck and back. The neck is not in its neutral position. Option 3 is true. Tension causes shoulders to rise, putting more stress on the neck and back. Option 4 is true. If you type with the hands higher than the elbows, then you are causing stress to the wrist. Option 5 is true. Elbows extended or abducted will increase stress on the arms and wrist.

The nurse recognizes which manifestations as signs of community-acquired pneumonia? 1. Cough 2. Decreased respiratory rate 3. Fever 4. Myalgia 5. Pleuritic chest pain

1., 3., 4. & 5. Correct: Signs of community-acquired pneumonia include cough, crackles, egophony, tactile fremitus, fever, dyspnea, sputum production, myalgias, and pleuritic chest pain. A client with an infection (particularly pneumonia) will exhibit these symptoms. 2. Incorrect: Decreased respiratory rate is not a sign of community-acquired pneumonia; respiratory rate increases with fever and dyspnea. Well, if you know the signs and symptoms of pneumonia, then you have it made with this question. Remember, With Select All That Apply questions, look at each option as a true/false statement. Match what is being asked in the stem with the options. If the stem is asking for a true response, then the true options should be selected. And, With Select All That Apply questions, there will be two or more correct options. So which options are true? Did you say options 1., 3., 4. & 5? Yes, these are the signs of community-acquired pneumonia. With pneumonia the client will have a cough, crackles, egophony, tactile fremitus, fever, dyspnea, sputum production, myalgias, and pleuritic chest pain. Why is Option 2 false? Decreased respiratory rate is not a sign of pneumonia; the respiratory rate increases with fever and dyspnea.

The nurse is assisting with a client who will receive electroconvulsive therapy (ECT). The anesthesiologist administers succinylcholine chloride intravenously. What adverse effects should the nurse monitor for post procedure? 1. Malignant hyperthermia 2. Hypokalemia 3. Apnea 4. Tetany 5. Arrhythmias

1., 3., 4. & 5. Correct: Succinylcholine is a paralytic used to relax the muscles to prevent severe muscle contractions during the seizure, thereby reducing the possibility of fracture or dislocated bones. Adverse effects include malignant hyperthermia, apnea, and arrhythmias. It causes paralysis of the muscles of the face and those used to breath, so monitoring for apnea is very important. Tetany, spasms or stiffness in the jaw would be adverse effects and can indicate malignant hyperthermia. 2. Incorrect: Hyperkalemia can occur. Succinylcholine is a depolarizing muscle relaxant which means during prolonged muscle depolarization, the muscle may release large amounts of potassium into the blood.

The nurse is preparing a client for transport to the radiology department for a left lung tissue biopsy. Which actions should the nurse make certain have been completed? 1. The consent form is signed. 2. The operative site is prepped with a razor. 3. The most recent lab work is on the chart. 4. Any preoperative medication is given as prescribed. 5. Person performing the procedure has marked the site.

1., 3., 4. & 5. Correct: The nurse should ensure that the consent form is signed, the lab work is in order, and any prescribed preoperative medication is given. The operative site is marked by the person who is ultimately accountable for the procedure and will be present when the procedure is performed. 2. Incorrect: The site should be prepped with clippers as opposed to a razor, which can cause injury to the client. The goal of preoperative skin preparation is to decrease bacteria without injuring the skin. Remember client safety is always a priority. This question is identifying nursing interventions to be completed prior to the left lung tissue biopsy. So let's look at the options. Option 1 The nurse should identify that the consent form has been signed. True The client has the legal right to sign a consent form prior to any invasive procedure such as a lung biopsy. Option 2 is false. The site should be prepped with clippers as opposed to a razor, which can cause injury to the client. The goal of preoperative skin preparation is to decrease bacteria without injuring the skin. Option 3 The most recent lab work should be posted on the chart for review by the healthcare providers. True The client's physical status should be evaluated prior to the biopsy. The client's lab reports are a vital part the physical assessment. Option 4 The nurse should review the chart to determine if preoperative medication has been ordered. True Many medications are administered during the perioperative period. The last dose if ordered is usually taken with a sip of water within 2 hours prior to the procedure. Also other medications may be ordered that are specific to the client's procedure and the client's diagnoses. Option 5 The operative site is marked by the person who is ultimately accountable for the procedure and will be present when the procedure is performed. True

The crisis line nurse answers a call from a client who is voicing intent to commit suicide. The client tells the nurse, "I am sitting here with a bottle of pain killers in my hand." What is the nurse's most appropriate response? 1. "I want to help you to resolve the problem." 2. "You should drive yourself to the emergency room." 3. "You did the right thing by calling." 4. "I want you to stay on the phone with me." 5. "Have another person call 911 for an ambulance."

1., 3., 4. & 5. Correct: The nurse wants to establish a positive relationship with the client as quickly as possible. The nurse wants to recognize positive qualities. Keeping the client on the phone may prevent the client from taking the pain killers. The crisis line nurse keeps the person on the line as long as possible as this is most important. Losing contact is a threat to the client's safety. This client is planning action with access to the plan. Emergency personnel should be called. 2. Incorrect: The client has a plan and the means available for suicide. The client does not need to drive to the emergency room. The nurse keeps the client on the phone as she activates the 911 call. In this select all that apply question, you should read each option as a true or false statement. Remember to take all suicide plans by the client seriously. Keep the client safe! Option 1: Great choice! The nurse offers to help. By calling the crisis line, the client is reaching out for help to prevent carrying out the suicide plan. Option 2: No way! This client is in great emotional pain and does not need to drive to the emergency department. This is unsafe! Option 3: Yes. The nurse is supporting the client in their call for help. This statement is supportive of the client. Option 4: For sure! Keeping the client on the phone will keep the client from carrying out the suicide plan. This allows time for emergency help to arrive. This is a very important nurse response. Option 5: A must! 911 call and emergency personnel are needed. This client has definite suicide plans and the means (Bottle of pain medication) to carry out the suicide plan.

An occupational health nurse is planning to teach a group of manufacturing workers how to prevent back injuries. What teaching points should the nurse plan to include? 1. Wear comfortable, low-heeled shoes. 2. When sitting, keep knees slightly lower than the hips. 3. Avoid movements that require spinal flexion with straight legs. 4. Squarely face the direction of anticipated movement. 5. Pivot to turn while holding an object.

1., 3., 4., & 5. Correct: Comfortable, low heeled shoes provide good foot support and reduce the risk of slipping, stumbling, or turning your ankle. Flexion of the spine with the legs straight (toe-touches, sit-ups) will injure the back. Avoid twisting of the back by squarely facing the direction of movement. Move toward or away from your center of gravity. Pivoting is a technique in which the body is turned in a way that avoids twisting of the spine. 2. Incorrect: When sitting, keep knees slightly higher than the hips.

A client with distended and tortuous veins along the inner aspects of both legs asks the nurse how to decrease the development of these veins. What should the nurse advise? 1. Exercise 2. Follow a low protein diet 3. Wear low heeled shoes 4. Elevate legs above heart several times per day 5. Do not cross legs

1., 3., 4., & 5. Correct: These are varicose veins. Get moving. Walking is a great way to encourage blood circulation to the legs. Low-heeled shoes work calf muscles more, which is better for veins. To improve circulation in legs, take several short breaks daily to elevate legs above the level of the heart. Do not cross legs as it decreases circulation distally. 2. Incorrect: Low sodium diet will prevent swelling caused from water retention. A diet low in protein will not decrease the development of these veins. The key is to keep swelling down so that pressure on the veins is reduced.

Which health promotion instructions should the nurse provide to a client diagnosed with cirrhosis? 1. Use a shower chair when performing hygiene. 2. Limit alcohol intake. 3. Stop any activity that causes dizziness. 4. Calculate daily sodium intake. 5. Proper hand hygiene.

1., 3., 4., & 5. Correct: Using a shower chair while showering and performing hygiene will help to save energy. Stop any activity that causes chest pain, a marked increase in shortness of breath, dizziness, or extreme fatigue or weakness. High sodium promotes fluid volume excess. The client should maintain a low sodium intake. Proper hand hygiene prevents infection. 2. Incorrect: The client must stop drinking alcohol to halt the progression of cirrhosis. The primary functions of the liver are: Bile production and excretion; Excretion of bilirubin, cholesterol, hormones, and drugs; Metabolism of fats, proteins, and carbohydrates; Enzyme activation; Storage of glycogen, vitamins, and minerals; Synthesis of plasma proteins, such as albumin, and clotting factors; Blood detoxification and purification. Option 1 is true. Think about it. If the liver detoxifies the body of waste products such as ammonia, then these levels increase when the liver is not functioning properly. So the client is tired. Sitting will conserve energy. Option 2 is false. Chronic alcoholism is the leading cause of cirrhosis in the United States. Drinking too much alcohol can cause the liver to swell, which over time can lead to cirrhosis. Option 3 is true. Where does ascites fluid come from? The vascular space, right? Yes. So dizziness may be an indication of decreased vascular volume. Option 4 is true. Limiting salt in the diet will help to prevent or reduce fluid buildup of ascites. Option 5 is true. Teaching proper hand hygiene to clients is always the proper thing to do. Clients with a chronic disease are at risk for infection so proper hand hygiene is appropriate.

A client diagnosed with hypothyroidism has been taking levothyroxine in increasing doses over the past week. Which findings, if present, would indicate to the nurse that the drug dosage is too high? 1. Irritability 2. Weight gain 3. Tachycardia 4. Tremors 5. Headache 6. Bradycardia

1., 3., 4., & 5. Correct: When a nurse administers levothyroxine, there is an expected therapeutic response of increase in energy, improved affect, improved gastric motility, weight loss, and less sensitivity to cold. If the levothyroxine dose is too high, the client may experience an tachycardia, dysrhythmias, tremors, and a headache. When the levothyroxine level is too high, the symptoms are the same as hyperthyroidism. 2. Incorrect: Weight gain is a symptom of the decrease level of the thyroid hormones, T3 and/or T4. This is a symptom of hypothyroidism. 6. Incorrect: Bradycardia is a symptom of hypothyroidism. This is a result of a decrease in the thyroid hormones, T3 and/or T4 is a s/s of hypothyroidism.

Which finding in fetal heart rate during a non-stress test would indicate to the nurse that a potential problem for the fetus may exist? 1. Increases 30 beats per minute for 20 seconds with fetal movement. 2. Increases 8 beats per minute for 10 seconds with fetal movement. 3. Remains unchanged with maternal movement. 4. Increases 5 beats per minute for 30 seconds with maternal movement.

2. Correct. A non-reactive test is when the FHR accelerates less than 15 beats per minute above baseline. This may indicate fetal compromise. 1. Incorrect. This would be a reactive test. This is characterized by acceleration of fetal heart rate of more than 15 beats per minute above baseline, lasting for 15 seconds or more. 3. Incorrect. This test does not look at fetal heart rate with maternal movement. 4. Incorrect. This test does not look at fetal heart rate with maternal movement.

The nurse, caring for a client diagnosed with Alzheimer's Disease (AD), notices the client becoming agitated. What nursing strategies would be appropriate for the nurse to initiate? 1. Provide a snack for the client. 2. Tell the client to stop the unwanted behavior. 3. Take client for a walk. 4. Ask the client to sweep the floor. 5. Inform the client that restraints will be used if behavior continues. 6. Turn on the client's favorite music.

1., 3., 4., & 6. Correct: Nursing strategies that address difficult behavior include redirection, distraction, and reassurance as provided by these correct interventions. 2. Incorrect: These behaviors are often unpredictable and not intentional. Do not challenge the client, use redirection, distraction, and reassurance. 5. Incorrect: When dealing with a difficult client, do not threaten to restrain the client or call the primary healthcare provider. A calming family member can be asked to stay with the client until the client becomes calmer. Behavioral problems occur in about 90% of client's diagnosed with Alzheimer's. These include repetitiveness, delusions, hallucinations, agitation, aggression, altered sleeping patterns, wandering, and resisting care. These behaviors are often unpredictable and may challenge caregivers. Caregivers need to be aware that these behaviors are not intentional. Nursing strategies that address difficult behavior include redirection, distraction, and reassurance. Option 1: Good choice. This nursing action will serve as a distraction and redirection. This changes the client's focus and helps decrease agitation. The AD client also may be malnourished and underweight and benefit from a healthy snack. Option 2: No. The client with AD may not be able to control their behavior. This is a behavioral change that comes with the disease. Option 3: Yes! Taking the client for a walk involves exercise and change of environment. This serves as redirection and distraction. Option 4: Another good choice. Redirection and distraction are helpful in controlling agitation. Sweeping the floor gives the AD client a purposeful activity. Option 5: Oh no - wrong! Stay away from restraints! This is a very last resort. Also, this is threatening the client and would most likely increase agitation. Reassurance involves communicating to the client that he or she will be protected from harm, danger, and embarrassment. Reassurance is an appropriate nursing strategy for AD. Option 6: Yes! Use of songs, poems, music, massage, and favorite object can soothe the client.

What nursing interventions should the nurse initiate in a client who experiences sundowning? 1. Limit naps. 2. Encourage TV watching in the evening. 3. Create a calm, quiet environment. 4. Open window blinds during the day. 5. Leave the lights on at night. 6. Maintain a routine.

1., 3., 4., & 6. Correct: Sundowning occurs when the client becomes more confused and agitated in the late afternoon or evening. Behaviors commonly seen include agitation, aggressiveness, wandering, resistance to redirection, and increased verbal activity such as yelling. Limit naps because too much daytime napping may interfere with sleeping at night. Light therapy may reduce agitation and confusion so open the blinds. Caregivers should remain calm and avoid confrontation. Routine helps the client feel secure. 2. Incorrect: Watching television for this client may lead to restlessness, agitation, and confusion. Calming and more restful activities are better for the evening. 5. Incorrect: Lights should be on during the day but turned off at night (except for low lighting or nightlights so the client can see).

Which discussion points should a nurse plan to include when teaching a group of college students on prevention of sexually transmitted infections (STI)? 1. Safe sex practices 2. Routine human immunodeficiency virus (HIV) testing 3. Proper use of birth control pills 4. Sexual abstinence 5. Vaccinations for STIs

1., 4. & 5. Correct: All of these topics should be included when discussing prevention of STIs. Safe sex practices include proper use of condoms. Abstinence is the best way to prevent STIs. Vaccines are available for some STIs such as human papillomavirus vaccine (HPV). 2. Incorrect: Routine HIV testing is not a way to prevent HIV or other STIs. It will provide early diagnosis. The best course of action is to prevent occurrence. 3. Incorrect: Birth control pills help prevent unplanned pregnancy. STIs can still be contracted if proper safe sex techniques are not implemented. Before we review the options, let's look at the question. The key words in a question should be identified. The key words in this question are teaching, college students, prevention, and sexually transmitted infections (STI) . This is a select all question so there will be 2 or more options correct. Also each option stands alone with the question. After reviewing the question, look at each option and identify if it is true or false. Remember client safety is always a priority. The question is asking for teaching points to prevent STIs. So let's look at the options. Option 1 is true. Safe sex practices will include proper use of contraceptives, sexual attitudes, and sexual anatomy and physiology, Option 2 is false. Routine HIV testing is not a way to prevent HIV or other STIs. It will provide early diagnosis. The best course of action is to prevent occurrence. Option 3 is false. Birth control pills help prevent unplanned pregnancy. STIs can still be contracted if proper safe sex techniques are not implemented. Option 4 is true. Abstinence is the best way to prevent STIs. Option 5 is true. Vaccines are available for some STIs such as human papillomavirus vaccine (HPV)..

What nursing interventions should the nurse include when planning care for a client admitted with Guillain-Barre' Syndrome? 1. Monitor for contractures. 2. Place prone for 30 minutes, 4 times per day. 3. Provide therapeutic massage for pain relief. 4. Teach range of motion exercises. 5. Provide high protein meals 3 times a day. 6. Refer to physical therapist.

1., 3., 4., & 6. Correct: This client will have progressive weakness and paralysis. Contractures and pressure ulcers need to be prevented through ROM exercises and frequent turning. Muscle spasms and pain can be relieved by therapeutic massage, imagery, diversion, and pain medication. 2. Incorrect: The client will need to be repositioned every 2 hours to prevent pressure sores and pneumonia and atelectasis. Elevate the head of the bed to help with lung expansion. Prone will interfere with lung expansion ability. 5. Incorrect: Encourage small, but frequent meals that are both well-balanced and nourishing. With this disease we know that there is progressive muscle weakness, cramping and paralysis. So interventions should focus on the hazards of immobility, pain, and maintaining the airway. Option 1 is true. The client develops paralysis so contractures can occur. Contractures can be prevented by frequent ROM exercises and proper alignment. Option 2 is false. This client has decreased respiratory excursion due to muscle weakness. Placing the client prone will further inhibit adequate breathing. Option 3 is true. The main premise behind incorporating massage therapy for this disease is that massage can influence the afferent neural pathways by manually sending signals back to the brain so that the client's pain decreases. Option 4 is true. Early in the acute phase clients benefit from daily ROM exercises and proper positioning to prevent muscle shortening and joint contractures. Option 5 is false. These clients often have difficulty with dysphagia and may need more frequent, high calorie meals as well as enteral or parenteral feedings. Option 6 is true. Estimates suggest that approximately 40% of clients who are hospitalized with GBS require inpatient rehabilitation. The goals of the therapy programs are to reduce functional deficits and to target impairments and disabilities resulting from GBS.

A client who needs to have a stool specimen for an occult blood test is instructed by the nurse to avoid which substances two hours prior to testing? 1. Liver 2. Tomato 3. Ibuprofen 4. Sardines 5. Ascorbic acid

1., 3., 4., 5. Correct: The following foods can cause a false-positive reading: rare meats, liver, poultry, turnips, broccoli, cauliflower, melons, salmon, sardines, and horseradish. Medications altering the test include aspirin, ibuprofen, ascorbic acid, indomethacin, colchicines, corticosteroids, cancer chemotherapeutic agents, and anticoagulants. Ingestion of vitamin rich foods can cause a false negative result. 2. Incorrect: A tomato is not on the food list for false-positive reading and do not have to be avoided. The test taker must be knowledgeable about diagnostic tests. This question is asking about what to avoid 2 hours prior to a occult blood test. So let's look at the options. Option 1 is true. Liver ingested within 2 hours of the test can cause a false-positive reading of the occult blood test. Option 2 is false. A tomato is not on the food list that will cause a false positive reading if eaten within 2 hours prior to a occult blood test Option 3 is true. Ibuprofen is on the medication list that will cause a false reading if taken within 2 hours to a occult blood test. Option 4 is true. Sardines ingested within 2 hours of the occult blood test will cause a false positive reading. Option 5 is true. Ascorbic Acid is on the medication list that will cause a false reading if taken 2 hours prior to the occult blood test.

Which assessment finding in a client 5 hours post open cholecystectomy would require the nurse to notify the surgeon? 1. Absent bowel sounds. 2. Jackson Pratt drain has 90 mL of blood. 3. Urinary output of 180 mL since return from surgery. 4. Client report of abdominal pain of 8/10.

2. Correct. An open cholecystectomy will usually result in the placement of a drain. The drainage should be green (bile). Blood is a problem and needs immediate intervention. 1. Incorrect. It is not uncommon for bowel sounds to be absent after abdominal surgery. This client is only 5 hours postoperative. The client needs to remain NPO until bowel sounds return. 3. Incorrect. The urine output is greater than than 30 mL/hour which is an acceptable amount. There is not a baseline to compare to, so greater 30mL/hour is not abnormal. 4. Incorrect. Pain for this client is an expected finding 5 hours after surgery.

A nurse is planning to discuss steps that senior citizens can take to keep the brain healthy. What should the nurse include? 1. Memorize poetry. 2. Eat foods low in Omega 3, fatty acids. 3. Brush teeth with nondominant hand. 4. Do crossword puzzles. 5. Learn a new language. 6. Volunteer.

1., 3., 4., 5., & 6. Correct: All of these activities exercise the brain and increase mental functioning. 2. Incorrect: Brain-boosting food is any food high in Omega 3 fatty acids, which has been linked to a lower risk of dementia and improved focus and memory. Although there is no known definitive way to prevent Alzheimer's disease, steps can be taken to keep the brain healthy. Growing evidence indicates that people can reduce their risk of cognitive decline by adopting key lifestyle habits. When possible, combine these habits to achieve maximum benefit for the brain and body. To keep the brain active, learn something new. Take up a language, an instrument, memorize poetry. A good, healthy diet will improve all areas of our health, but there are many studies and an increasing amount of evidence suggesting that certain foods slow mental decline. Brain-boosting food is any food high in Omega 3 fatty acids, which has been linked to a lower risk of dementia and improved focus and memory. Brain games, puzzles and brainteasers help create new associations between different parts of the brain, which keeps it sharp. Challenge the brain by doing normal activities with the non-dominant hand like brushing your teeth or combing your hair. Research shows that volunteering can lower stress levels and increase mental functioning. It also adds to a person's wellbeing and overall health. Social activity keeps our minds sharp. This is especially true later in life, when aging takes its toll on memory and other complex neurological processes.

What foods should the nurse teach a client who has been diagnosed with iron deficiency anemia to increase in the diet? 1. Chickpeas 2. Milk 3. Oysters 4. Raisins 5. Spinach 6. Tuna

1., 3., 4., 5., & 6. Correct: All of these are high in iron. 2. Incorrect: Milk is not high in iron and slows the absorption of iron. Clients with iron deficiency need foods high in iron. So what foods are high in iron? Red meat, pork and poultry. Seafood. Beans. Dark green leafy vegetables, such as spinach. Dried fruit, such as raisins and apricots. Iron-fortified cereals, breads and pastas. Peas. Do you see any of those options? Yes. Option 1 - peas Option 3 - seafood Option 4 - dried fruit Option 5 - dark green leafy vegetable Option 6 - seafood Option 2, milk is false. Milk slows the absorption of iron.

A nurse is performing eye care for an unconscious client. Which interventions should the nurse include? 1. Administer moist compresses to cover eyes every 2 hours. 2. Clean eyes with saline and cotton balls, wiping from outer to inner canthus. 3. Use a new cotton ball for each cleansing wipe. 4. Instill artificial tears into the lower eye lids as prescribed. 5. Protect the eyes with a protective shield. 6. Monitor eyes for redness and exudate.

1., 3., 4., 5., & 6. Correct: All of these interventions are appropriate for eye care of the comatose client. These actions prevent infection, keep eyes moist, and protect the eye from injury. 2. Incorrect: Clean the eyes with saline solution and cotton balls. Wipe from the inner to outer canthus. This prevents debris from being washed into the nasolacrimal duct.

The nurse is teaching comfort measures to a postpartum client with an episiotomy and external hemorrhoids. Which teaching points should the nurse include? 1. Apply ice to perineum for first 12 hours. 2. Take sitz baths at temperature of 107.6°-111.2°F (42-44°C). 3. Use witch hazel compresses on rectal areas for hemorrhoids. 4. Take ibuprofen for pain. 5. Apply topical anesthetics to perineal area. 6. Avoid sexual intercourse until episiotomy has healed.

1., 3., 4., 5., & 6. Correct: Ice causes vasoconstriction and is most effective if applied soon after the birth to prevent edema and to numb the area. Chemical ice packs or clean gloves filled with ice may be used during the first 12 hours after a vaginal birth. Witch hazel contains chemicals called tannins. When applied directly to the skin, witch hazel might help reduce swelling and help repair broken skin. Analgesics such as acetaminophen and nonsteroidal anti inflammatory drugs (NSAIDs) such as ibuprofen frequently are prescribed to provide relief for mild to moderate discomfort. Topical anesthetic may be used as needed to decrease surface discomfort and allow more comfortable ambulation. Sexual intercourse prior to healing of the episiotomy may contribute to further perineal damage. 2. Incorrect: This temperature is too hot and can damage the injured tissue. The sitz bath should be at a temperature of 100-104°F (38-40°C).​

A community health nurse is planning to discuss how to prevent pesticide ingestion at a local health fair. What should the nurse include in this teaching session? 1. Discard the outer leaves of lettuce. 2. Wash fruits and vegetables with dish soap. 3. Buy organic produce. 4. Peel fruits prior to eating. 5. Dry produce thoroughly with disposable paper towels after washing. 6. Use a scrub brush when washing fresh fruits and vegetables.

1., 3., 4., 5., & 6. Correct: The outer leaves of green, leafy vegetables, such as lettuce and cabbage, should be discarded as pesticide residue likely remains there. Another great idea to reduce overall exposure to pesticides is to buy organic or unsprayed produce. If you can't buy organic, peel fruits and vegetables prior to eating. Washing your fruits and veggies is not enough if you want to reduce the pesticide load you expose yourself to, as it is very important to thoroughly dry them with disposable paper towels as well. This will remove all the remaining pesticide residue and make the produce safer to eat. A scrub brush is very effective in cleaning the crevices and areas around the stem. 2. Incorrect: One of the most common mistakes people make in their attempt to remove all pesticide residue from their produce is that they wash their fruits and vegetables with soap or, even worse, dish soap. Never use detergents, special rinses or soaps of any kind, as this will only do more harm than good. Unless the soap is entirely made of natural and organic materials, it tends to contain harmful compounds that easily penetrate the skin of the fruits, thus doing more harm than the actual pesticides after you ingest them. Simply wash with tap water. Before we review the options, let's look at the question. The key words in a question should be identified. The key words in this question are community health nurse, prevent pesticide ingestion, health fair, and teaching session. This is a select all question so there will be 2 or more options correct. Also each option stands alone with the question. After reviewing the question, look at each option and identify if it is true or false. Remember client safety is always a priority. The question is asking about a teaching session to prevent pesticide ingestion. So let's look at the options. Option 1 is true. The outer leaves of green, leafy vegetables such as lettuce, and cabbage should be discarded as pesticide residue likely remains there. Option 2 is false. One of the most common mistakes people make in their attempt to remove all pesticide residue from their produce is that they wash their fruits and vegetables with dish soap. Unless the soap is entirely made of natural and organic materials, it tends to contain harmful compounds that easily penetrate the skin of the fruits. Simply wash with tap water. Option 3 is true. Another great idea to reduce overall exposure to pesticides is to buy organic foods. Organic foods are grown without the use of pesticides or synthetic fertilizers. Option 4 is true. If you can't buy organic, peel fruit and vegetables prior to eating. Option 5 is true. Washing your fruits and veggies is not enough if you want to reduce the pesticide load you expose yourself to, as it is very important to thoroughly dry them with disposable paper towels as well. This will remove all the remaining pesticide residue and make the produce safer to eat. Option 6 is true. A scrub brush is very effective in cleaning the crevices and areas around the stem. The scrub brush should not break the skin of the produce.

Which task would be appropriate for the nurse to assign the unlicensed assistive personnel (UAP)? 1. Assess any pressure ulcers noted on clients. 2. Report if any client indicates pain. 3. Monitor amount of chest tube drainage. 4. Demonstrate coughing and deep breathing exercises to post-op clients.

2. Correct. It is within the scope of practice for the UAP to ask the client if they are experiencing pain. The nurse will then assess the pain. The nurse can delegate, assess, develop a plan of care and evaluate. 1. Incorrect. This is an RN task. The UAP does not have the appropriate education to assess a pressure ulcer. This is not within their scope of practice. 3. Incorrect. The UAP cannot assess or evaluate. This is an RN task. Monitoring the amount of chest tube drainage is an appropriate action for the nurse. The UAP cannot monitor the amount of chest tube drainage. 4. Incorrect. The UAP cannot teach. This is an RN task. The nurse cannot delegate teaching or demonstrating to the UAP. This is the responsibility of the RN.

Which interventions should the nurse include in the plan of care for a client who has been admitted with a head injury? 1. Pad side rails. 2. Place hips in flexed position for 15 minutes every 4 hours. 3. Elevate head of bed 35 degrees. 4. Maintain neck in neutral position. 5. Cluster nursing activities. 6. Maintain a quiet environment.

1., 3., 4., and 6. Correct: The client with a head injury is at risk for seizures. Padding the side rails is a safety precaution. Elevate the HOB 30-45 degrees to facilitate venous drainage and reduce ICP. Maintain the client's head midline to facilitate blood flow. A quiet environment is necessary to keep the client calm. An increase in environmental stimuli can increase ICP. 2. Incorrect: Do not allow pronounced neck or hip flexion as ICP will increase. Maintain HOB at 30-45 degrees and body in neutral position to avoid an increase in ICP. 5. Incorrect: Clustering nursing activities will increase ICP. Activities should be spaced out. Remember, the client needs a quiet environment. Option 1 - Pad the side rails. Is this a good safety precaution? Yes, because the client is at risk for seizures. A head injury disrupts the pathways of the brain resulting in a seizure. During the seizure the electrical fields in the brain are overloaded. Option 2 - Place hips in a flexed position. Will this help the client? NO. False. Flexion is bad. Flexion will increase intracranial pressure. We want the client's body in a neutral position. Option 3 - Elevate the HOB. Is this good for the client? Absolutely True. Elevating the HOB will decrease ICP. A supine position will increase ICP and we don't want that. Option 4 - Maintain neck in neutral position? What do you think? This is true. The purpose of the jugular veins is to facilitate venous drainage. This decreases ICP. If the client's head is flopped over to one side then you want get good blood flow from the carotid artery to the brain and you will not get good venous drainage from the brain. Option 5, Cluster nursing activities. Well in nursing school we want to cluster nursing activities most of the time to allow the client more rest time between interruptions. However, if we do that for this client, what could happen? Increasing ICP. So here we want a quiet environment, a calm client, and spaced out interventions. We are trying to keep the ICP from rising. Option 6, maintain a quiet environment. Well we just said we are trying to decrease ICP so we are not in the business of stimulating this client. This statement is true.

A client diagnosed with gout has received instruction on maintaining a low-purine diet. Which statements, if made by the client, would indicate to the nurse that teaching was successful? 1. "I will eliminate foods from my diet that contain 150 mg or more of purine per serving." 2. "Rather than drinking a glass of wine, I should drink a glass of beer." 3. "Losing weight can help reduce the uric acid levels in my blood." 4. "Potatoes, rice, and barley are high in purine and should be eliminated from my diet." 5. "Vegetables that should be limited to 2 times/week include cauliflower, spinach, and mushrooms." 6. "Increasing fluid intake to 8-10 cups/day will help to eliminate purines through my urine."

1., 3., 5., & 6. Correct: Foods that contain 150 mg or more of purine are considered high purine foods and should be eliminated from the diet. Weight loss has been shown to improve insulin resistance, and therefore reduce uric acid levels in the blood. Vegetables that have high purine content include cauliflower, spinach, peas, asparagus, and mushrooms. These should be limited to no more than 2 times per week. Ensuring a sufficient fluid intake helps to reduce the risk of crystals forming in joints. Keeping hydrated and avoiding dehydration can lessen this risk and help to prevent gout attacks. 2. Incorrect: Alcohol and caffeinated beverages - These cause increased dehydration and interfere with uric acid elimination. The metabolism of alcohol in your body is thought to increase uric acid production, and alcohol contributes to dehydration. Beer is associated with an increased risk of gout and recurring attacks, as are distilled liquors to some extent. The effect of wine is not as well understood. 4. Incorrect: Potatoes, rice, barley, noodles, and pastas are low in purine and can contribute to the 4 or more servings of starches needed per day.

A client has been admitted to the medical unit after sustaining a stroke. The admitting nurse initiates a nursing diagnosis of unilateral neglect related to a decrease in visual field and hemianopia from cerebrovascular problems as evidenced by consistent inattention to stimuli on the affected side. What nursing interventions should the nurse initiate for this client? 1. Instruct client to scan from left to right to visualize the entire environment. 2. Encourage client to practice exercises independently. 3. Position bed in room so that individuals approach the client on the unaffected side. 4. Apply splints to achieve stability of affected joints. 5. Touch unaffected shoulder when initiating conversation with client. 6. Position personal items within view on the unaffected side.

1., 3., 5., & 6. Correct: Instructing the client to scan from left to right will help the client to visualize the entire environment. The client has to be reminded to do this since only one side of the client's visual field is working. By positioning the bed so that individuals approach the client from the unaffected side and by touching the client on the unaffected shoulder, the client is not surprised or frightened when realizing someone is in the room. Placing personal items where the client can see them will allow the client to use the material. Then gradually move personal items and activity to the affected side as the client demonstrates an ability to compensate for neglect. 2. Incorrect: Practicing exercises independently focuses on impaired physical mobility rather than unilateral neglect. 4. Incorrect: Applying splints to affected joints focuses on impaired physical mobility rather than unilateral neglect. Hemianopia is a decreased vision or blindness in half the visual field, usually on one side of the vertical midline. The most common causes of this damage are stroke, brain tumor, and trauma. So interventions should be focused on helping the client to see their environment, as well as items and individuals within their environment. As the client is able to compensate for the neglect, then the nurse can gradually move personal items and activity to the affected side.

Which tasks are most appropriate for the hospice nurse to delegate to an unlicensed assistive personnel (UAP)? 1. Bathe the client. 2. Provide spiritual support 3. Listen to the client reminisce. 4. Administer routine medications. 5. Weigh the client. 6. Take vital signs

1., 3., 5., & 6. Correct: The UAP can bathe, listen to the client remininsce, weigh, and take the vital signs. These are within the scope of practice of the UAP. These assignments are routine and revolve around activities of daily living. 2. Incorrect: The task of providing spiritual support could best be delegated to the pastor or chaplain. 4. Incorrect: The nurse can not delegate routine medication administration to the UAP. This is not within the UAPs scope of practice. This is an LPN or RN responsibility. In order to answer this question appropriately, you must understand what a UAP can and cannot do. The UAP can assist clients with activities of daily living. They can also do routine care tasks, such as daily weights, vital signs, I&Os. Knowing this, let's look at the options. What did you chose? I hope you chose options 1, 3, 5, and 6. Look at option 2. Why is it wrong. Who would be the best person to provide spiritual support? A pastor or chaplain. What about option 4, administer medications. No. This is beyond the scope of practice for the UAP. An LPN or RN must do this.

The nurse is teaching a client who is at risk for developing a stroke. What primary prevention strategies should the nurse include? 1. Promote a diet rich in fruits and vegetables. 2. Provide instruction on benefits of carotid endarterectomy. 3. Limit sodium intake to 2 grams/day. 4. Engage in low intensity exercise once a week. 5. Avoid tobacco products. 6. Decrease alcohol consumption to two drinks per day.

1., 3., 5., & 6. Correct: These strategies are considered primary prevention strategies that can decrease the risk of developing a stroke. 2. Incorrect: This would be considered secondary prevention: early diagnosis and treatment to prevent stroke. 4. Incorrect: The client needs 3-4 sessions per week of moderate-vigorous intensity aerobic physical exercise to reduce stroke risk factors. Session should last an average of 40 minutes. Moderate intensity exercise is typically defined as sufficient to break a sweat or noticeably raise heart rate (e.g. walking briskly, using an exercise bicycle). Vigorous intensity exercise includes activities such as jogging.

The nurse is caring for a client admitted with heart failure. Which prescriptions would necessitate that the nurse seek clarification from the primary healthcare provider? 1. Furosemide 20.0 mg p.o. daily 2. Rosuvastatin 5 mg p.o hs 3. Digoxin 0.125 mg IVP every 8 hours for three doses 4. Folic acid 1 mg daily 5. Heparin 1000 IU subcutaneously daily

1., 4. & 5. Correct: It is inappropriate to have a trailing zero after a decimal point for doses expressed in whole numbers. It can be mistaken as 200 if the decimal point is not seen. The folic acid order lacks a route, thus needs clarification. The Heparin order should be written as Heparin 1,000 units subcutaneously daily. Use commas for dosing units at or above 1,000 or use words such as one thousand to improve readability. Use units rather than IU (International units) as this can be mistaken as IV or 10. 2. Incorrect: This medication order is written correctly. 3. Incorrect: This medication order is written correctly. When you first read this question, you probably thought, "What medications should not be given to this client?" Right? Yes, that is what I initially thought. But look at the options. Some of these prescriptions are written incorrectly and could cause a medication error if not clarified with the primary healthcare provider. This is a safety issue. So select the options that you would need to clarify with the primary healthcare provider. Option 1: Yes. You better clarify this one! It is inappropriate to have a trailing zero after a decimal point for doses expressed in whole numbers. It can be mistaken as 200 if the decimal point is not seen. Option 2: No clarification needed here. This is a statin medication that is written correctly. Option 3: No need to clarify this digoxin prescription. It is written correctly. Option 4: Yes, clarification is needed with this prescription. The folic acid order lacks a route, thus needs clarification. Option 5: Yes, you better clarify this one. The Heparin order should be written as Heparin 1,000 units subcutaneously daily. Use commas for dosing units at or above 1,000 or use words such as one thousand to improve readability. Use units rather than IU (International units) as this can be mistaken as IV or 10.

Which medications, if prescribed to a client, should indicate to a nurse that retention of CO2 is a possibility? 1. Narcotics 2. Diuretics 3. Glucocorticoid steroids 4. Antiemetics 5. Hypnotics

1., 4. & 5. Correct: Narcotics sedate and decrease the respiratory rate, which increases CO2 retention. Always monitor respiratory rate. Some antiemetics (such as promethazine) are very sedating and will decrease the respiratory rate while increasing CO2 retention. Sleeping pills can cause sedation to the point of hypoventilation, which leads to CO2 retention. Always monitor respiratory rate. 2. Incorrect: Diuretics do not affect breathing patterns. 3. Incorrect: Steroids do not affect breathing patterns.

The nurse is caring for a client on the surgical unit. Which prescriptions could the nurse safely administer to the client? 1. Chlordiazepoxide 10 mg p.o. q 4h p.r.n. for agitation 2. Regular insulin 10 U stat 3. MS 2 mg IVP every 2 hours as needed for pain 4. Cefepime 1 gram IVPB every 8 hours 5. Diphenhydramine 25 mg p.o. hour of sleep for three nights

1., 4. & 5. Correct: These medication prescriptions are correctly written following approved Joint Commission abbreviations. 2. Incorrect: The "U" can be mistaken for "0" (zero), the number "4" (four) or "cc". Units should be written out completely. 3. Incorrect: MS can mean morphine sulfate or magnesium sulfate. Write "morphine sulfate". Write "magnesium sulfate". You should have learned in your fundamentals course what abbreviations are approved by Joint Commission to use when medications are prescribed or documentation is done in the nurse's notes. Let's see what you remember. Option 1 is true. There is nothing wrong with how this prescription is written. It is a complete prescription with medication, dose, route, time, and reason since for as needed use. Abbreviations used are acceptable. Option 2 is false. The "U" can be mistaken for "0" (zero), the number "4" (four) or "cc". Units should be written out completely. Think safety here. What would happen if you gave 100 units or 10 cc's. Hypoglycemia!!! Killer! Option 3 is also false. MS can mean morphine sulfate or magnesium sulfate. Write "morphine sulfate". Write "magnesium sulfate". Option 4 is true. This antibiotic prescription is correctly written with appropriate abbreviations. Option 5? This one is true as well.

Which observations should the home health nurse discuss with the parents of a two year old regarding potential safety threats in the home? 1. Security gates at the stairs. 2. Cleaning supplies under sink cabinet. 3. No blinds on windows. 4. Use of space heaters. 5. Water heater temperature 140°F (60°C) 6. Use of tablecloths

2, 4, 5 & 6 Correct: Cleaning supplies should be placed high away from child's reach. Cabinets should have childproof locks. Space heaters need to be checked every year prior to use. Additionally, small children can be burned by space heaters if they get too close. A guard should be applied. Water heaters should be set at no higher than 120°F (48°C). Burns may occur with a 6 second exposure to 140°F water temperature. Children can pull on table cloths and spill hot food or break dishes which could lead to injury. 1. Incorrect: Placing security gates at the stairs will prevent falls. 3. Incorrect: This is not a concern for the child. If there are blinds, the string should be out of the child's reach. Remember client safety is always a priority. This question is asking about safety threats for a 2 year old at home. Identify key words in the stem that indicate negative polarity, such as potential safety threats. This word indicates negative polarity and the question being asked is looking for what is false. So let's look at the options. Option 1 is false. Children falling is one of the top 5 causes of pediatric unintentional injury death. Placing security gates at the stairs will prevent falls. Option 2 is true. Cleaning supplies should be placed high away from child's reach. Cabinets should have childproof locks. Option 3 is true. This is not a concern for the child, since there are not blinds on the windows. If there are blinds, the string should be out of the child's reach. The string could get around the child's neck and strangle them. Option 4 true. Water heaters should be set at no higher than 120°F (48°C). Burns may occur with a 6 second exposure to 140° degree water temperature. Option 5 is true. Table clothes are about the eye level of children. They can pull on table clothes and spill hot food, or break dishes which could lead to injury.

A 65 year old client is admitted for management of dehydration with an IV infusion of LR @ 125 mL/hr. What assessment findings would be of concern to the nurse? 1. Anxiety 2. BP 136/80 3. CVP 5 mmHg 4. Crackles noted right posterior lung field 5. S3 heart sound

1., 4. & 5. Correct: Volume overload is an adverse effect of IV therapy in the elderly. Anxiety is an early sign of hypoxia due to FVE. Crackles to the bases are an early sign of fluid volume excess (FVE). S3 heart sounds are also an indication of FVE. 2. Incorrect: This blood pressure is not considered hypertension in this age group. Blood pressure of >140/90 is cause for concern in this age group. Also, one BP is not cause for concern. In assessing for FVE, it is important to compare to the client's baseline. 3. Incorrect: Normal CVP is 2-6 mmHg. A CVP reading of 5mmHg does not indicate FVE. Did you say options 1, 4, and 5? Good for you. When you are in a FVE, fluid can back up into the lungs so the client may have hypoxia. The first sign of hypoxia is anxiety, and restlessness. Do you think the client will have wet lung sounds? Yes. What is an S3 heart sound? The third heart sound (S3), also known as the "ventricular gallop," occurs just after S2 when the mitral valve opens, allowing passive filling of the left ventricle. The S3 sound is actually produced by the large amount of blood striking a very compliant left ventricle. Now look at option 2, BP of 136/80. Is this a BP to worry about? No, not when there is not other BPs to compare this one to. It does not indicate FVE or FVD. What about option 3, a CVP of 5 mmHg? What is a normal CVP? 2-6 mmHg, so this is a normal reading.

The nurse is caring for a client diagnosed with major depression post electroconvulsive therapy (ECT). What nursing interventions should be included in this immediate post-treatment period? 1. Monitor vital signs every hour for eight hours. 2. Position the client on their side. 3. Stay with the client until fully awake. 4. Provide flexibility in scheduling routine activities. 5. Encourage the client to ambulate in room and hall.

2. & 3. Correct: Positioning on the side will prevent aspiration. Stay with the client until they are fully awake, oriented, and able to perform self-care activities without assistance. Safety is priority. 1. Incorrect: Pulse, respirations, and blood pressure should be monitored every 15 minutes for the first hour. Vital signs every hour are too long immediately post-treatment. 4. Incorrect: The client needs a highly structured schedule of routine activities in order to minimize confusion. Also, immediately post-treatment is too soon to address routine activities. 5. Incorrect: The client should remain in bed during the immediate post-treatment period. The client needs to be fully awake prior to ambulation.

A client with renal failure has returned to the unit post kidney transplant. Which postoperative interventions should the nurse provide? 1. Administer furosemide. 2. Maintain fluid replacement at 150 ml per hour for 8 hours. 3. Measure abdominal girth every 24 hours. 4. Weigh daily. 5. Measure urine output every 30 - 60 minutes.

1., 4., & 5. Correct: Diuretics are administered to promote postoperative diuresis. Daily weights are done to make sure there is not rapid weight gain which is a pertinent of fluid retention. Careful and frequent assessment of UOP helps determine fluid balance and transplant function. Oliguria is an early sign of acute tubular necrosis and should be detected as soon as possible post-op. 2. Incorrect: Fluid replacement is generally calculated to replace urine output over the previous 30-60 minutes, milliliter for milliliter. The stem of the question does not give you enough information about the client to know that this rate of fluid replacement is safe. 3. Incorrect: Signs of hemorrhage after this surgery include swelling at the op site, increased abdominal girth, signs of shock and decreased level of consciousness. Assessment for signs of hemorrhage should be done much more frequently than every 24 hours.

What actions would be appropriate for a nurse who is administering ear drops to a six year old child? 1. Position supine with affected ear up. 2. Administer ear drops immediately upon removing from the refrigerator. 3. Open ear canal by drawing back on the pinna and slightly downward. 4. Allow prescribed number of drops to fall along inside of ear and flow into ear by gravity. 5. Have client remain supine for several minutes.

1., 4., & 5. Correct: Supine with affected ear up allows for proper administration of medication. Never attempt to put drops directly on the eardrum. Administer along inside of ear so that drops flow by gravity into ear. Remaining supine for several minutes permits the fluid to be absorbed. 2. Incorrect: If medication is not instilled at room temperature, the client may experience vertigo, dizziness, pain, and nausea. Additionally, cold ear drops cause discomfort. 3. Incorrect: This is the method for a child less than 3 years of age. For older than 3 years, open canal of ear by drawing back on the pinna and slightly upward.

Which tasks should the charge nurse complete at the end of the shift before leaving for the day? 1. Talk to each nurse about concerns related to assigned clients. 2. Call the family of a client suffering from dementia to discuss long term care placement. 3. Briefly assess every client. 4. Complete a client assignment sheet for the oncoming staff. 5. Receive report from the emergency department (ED) on a new client.

1., 4., & 5. Correct: Talking to the nurses about client concerns and completing the client assignment sheet for oncoming staff will provide for a thorough shift change report. It is crucial that the oncoming staff have an opportunity to voice any concerns regarding assignments and clarify any information provided.This proper exchange of information and concerns helps to ensure the safety of clients, provides continuity of care, and possibly prevents problems that might arise if these concerns had not been addressed. Taking the report from the ED could be delayed but is a courtesy to the ED and will provide information about the client that will be useful in making assignments for the next shift. 2. Incorrect: This will take some time and would be best accomplished by sitting with the family to discuss options. Doing this at the end of the shift could prevent completion of the client assignment sheet for the next shift and possibly create unnecessary overtime for the charge nurse. 3. Incorrect: The charge nurse does not have to assess every client. This will take a lot of time, and the charge nurse can get the information needed from the nurses caring for the clients in order to make appropriate client assignments for the next shift.

Which interventions would the nurse initiate in a 1 year old with Sickle Cell Crisis? 1. IV hydration 2. PCA pump for pain relief 3. Exercise 4. Analgesics 5. Antibiotics 6. Strict neutropenic precautions

1., 4., & 5. Correct: The client with Sickle Cell Crisis is dehydrated. Increasing hydration will improve ability of RBCs flow through the vascular system. Analgesics are needed for pain control. Antibiotics are needed to fight infection. 2. Incorrect: False, the client is not old enough to manage a PCA pump 3. Incorrect: False, during a crisis we want to promote rest to limit oxygen needs 6. Incorrect: False, sickle cell clients aren't neutropenic from sickle cell, and in order to pick an answer like this for a sickle cell client, the nurse would need to know what the client ANC (absolute neutrophil count) is. Y'all what are neutrophils? They are part of the WBC count, so if a client is on neutropenic precautions their neutrophil count is low and we are worried about them getting an infection. The client in this scenario has sickle cell, so the problem is the RBCs, we would be looking f

A nurse has completed education on safe sexual practices to a group of college students. Which comments by the students would indicate that education has been successful? 1. "The best way to prevent HIV is to abstain from sex." 2. "Contraceptives should contain spermicide N-9." 3. "Douching is recommended after intercourse." 4. "Drinking too much alcohol can increase the risk exposure to sexually transmitted disease (STDs)." 5. "If my partner will not use a condom, I will."

1., 4., & 5. Correct: These are correct statements. The best way to prevent HIV or STIs is to abstain from sex. If one decides to have sex, know your HIV status and your partners. Use condoms, male or female. Practice monogamy. Limit sexual partners. Use protection for all kinds of sexual contact. Get screened for sexually transmitted infections (STIs). Don't abuse drugs or alcohol, which are linked to sexual risk taking. 2. Incorrect: Spermicide N-9 will lower chances of pregnancy, but not HIV and other STIs. Spermicide N-9 actually makes your risk of HIV infection higher, caused by irritation of the vagina, which can make it easier for HIV to go into the body. 3. Incorrect: Douching removes some of the normal bacteria in the vagina that protects you from infection. This can increase your risk of getting HIV. The clue is "safe" sexual practice, and "successful" education. The test taker should look for comments that indicate knowledge of safe sexual practice. Option 1 is true. Abstaining from sex is the safest sexual practice. Option 2 is false. Spermicides do not kill HIV or STI viruses. Option 3 is false. Douching does not eliminate HIV or STI viruses. Option 4 is true. Alcohol can lead to risky sexual behaviors. Option 5 is true. Condoms are an effective method of decreasing the risk of contracting HIV or STI infections.

The nurse educator has provided education to newly hired emergency department nurses regarding mandatory reporting laws. Which suspected instances provided by the new nurses indicates to the nurse educator that education was effective? 1. Financial abuse of an elder 2. Negligence of a colleague 3. Spousal abuse denied by the victim 4. Gunshot victim 5. Client diagnosed with Gonorrhea 6. Client diagnosed with West Nile virus

1., 4., 5., & 6. Correct: Federal and state laws require that certain individuals, particularly those who work in health care with the elderly, with children, and other vulnerable populations, have an affirmative duty to report to a specified state agency when violence occurs against those populations. This includes physical, mental, and financial abuse. Gunshots and knife injuries are reportable to law enforcement. Certain communicable diseases such as gonorrhea and West Nile virus are reportable to the CDC. 2. Incorrect: Suspected negligence of a colleague is not in the realm of mandatory reporting to authorities, but the nurse should discuss with the supervisor. 3. Incorrect: A spouse is not considered a vulnerable person so it is not required by law to report. You should encourage the spouse to report the abuse but you, as the nurse, are not bound by law to do so.

A medical surgical nurse has been floated to the pediatric unit to assist during a staffing shortage. Which clients would be most appropriate for the float nurse? 1. A 10 year old in sickle cell crisis. 2. A 6 month old in a croup tent. 3. A 4 month old with bronchiolitis. 4. A 2 year old with cleft palate repair. 5. A 8 year old with Crohn's disease. 6. A 4 year old with acute asthma.

1., 5. & 6. Correct:The nurse has been floated to a pediatric unit, which not only has a special client population but also specific disease processes that are rarely encountered on an adult medical surgical unit. Client assignments should be based on both the developmental age/needs of the client and the disease process. A 10 year old sickle cell client is appropriate because school age children are more compliant and adapt easier to hospitalization than other groups; additionally, the float nurse may have had clients on the medical surgical unit with sickle cell disease. The same is true for an 8 year old with Crohn's disease: this would definitely be a safe assignment for a medical surgical nurse. Although the four year old is in the pre school group, usually parents of young children remain at the bedside to provide emotional support. Also, this nurse would have assessed and cared for asthmatics in the adult population previously. 2. Incorrect: Croup is a term used to refer to a variety of respiratory problems in children or infants. This infant's condition was serious enough to necessitate hospitalization with the use of a croup tent for oxygen and humidification. Therefore, the client's respiratory status will require close specialized monitoring by a pediatric nurse. This assignment would be unsafe for a medical surgical float nurse. 3. Incorrect: Not only is this client an infant, but the issue is a respiratory illness specific to pediatrics. Infants can deteriorate quickly, requiring specialized assessment and intervention techniques. A float nurse who is not experienced in pediatric assessments would not be appropriate for this client. 4. Incorrect: While the client's young age could prove challenging, it is the repair of a cleft palate that makes this an unsafe assignment for the float nurse. This is a specific surgery with potential airway issues, requiring particular assessment expertise not usually performed by a medical surgical nurse. This client should be assigned to a pediatric nurse.

The nurse is caring for a client on the cardiac unit. Which assessments are most important for the nurse to perform prior to the administration of diltiazem? 1. Note the rate and character of the apical pulse. 2. Ausculate the anterior and posterior breath sounds. 3. Check the morning results of serum calcium. 4. Review the last 24 hour urine output. 5. Monitor blood pressure. 6. Assess for chest pain.

1., 5., & 6. Correct: Diltiazem is a calcium channel blocker. It works by relaxing the muscles of the heart and blood vessels. Monitor blood pressure and pulse before and frequently during administration of diltiazem, as it causes systemic vasodilation and suppresses arrhythmias. Diltiazem is used to treat angina, so the nurse should assess for anginal pain. 2. Incorrect: Breath sounds need to be assessed to monitor for signs of heart failure, this would be a complication after diltiazem administration. Breath sounds are not necessarily assessed just prior to administration. 3. Incorrect: Diltiazem is a calcium channel blocker, but the total serum calcium concentration is not affected by it. Calcium channel blockers affect the flow of calcium into muscle cells. 4. Incorrect: A decrease in output would be an indicator of heart failure, which is a complication of diltiazem administration. This would be assessed after giving the medication.

The nurse educates a client that the prescribed medication indomethacin is used to manage which symptoms? 1. Pain 2. Inflammation 3. Fever 4. Cough 5. Urticaria

1.,2., & 3. Correct: Indomethacin is a non-steroidal anti-inflammatory agent used to treat pain, inflammation, and fever. 4. Incorrect: Indomethacin does not have any cough suppressant actions. 5. Incorrect: Urticaria is a side-effect associated with indomethacin use. So what is indomethacin? It is a non-steroidal anti-inflammatory agent. So what symptoms does it manage? Did you say: Option 1? Yes. Indomethacin is used to treat pain. by reducing hormones that cause pain in the body. Option 2? Yes, true. Indomethacin is used to treat inflammation by reducing hormones that cause inflammation in the body. What do you think about option 3? True. Indomethacin prevents prostaglandin synthesis (prostaglandin elevates body temperature and makes nerve endings more sensitive to pain transmission). Option 4. No, this is false. Indomethacin does not have any cough suppressant actions. Option 5. False. Urticaria is a side-effect associated with indomethacin use, so we would not give it to manage urticaria.

What discharge instructions should the nurse include for a client following a transsphenoidal hypophysectomy? You answered this question 1. Sleep with head of bed at 35 degrees. 2. Notify the primary healthcare provider for an increased urinary output. 3. Brush the teeth three times a day followed by rinsing with a commercial mouthwash. 4. Nasal packing will need to be removed in 48 hours. 5. Use a humidifier in the room. 6. Stay on a clear liquid diet for two days.

1.,2., 5. Correct. Sleeping with the head of the bed elevated will promote drainage of cerebrospinal fluid. An increased UOP could indicate diabetes insipidus, an adverse reaction to this surgical procedure. Humidified air prevents drying of nasal passages. 3. Incorrect. Because the incision for this surgery is just above the gumline, the client should not brush the front teeth. Oral care should be performed with a sponge until the incision heals. 4. Incorrect. There is no nasal packing. The incision is located just above the gumline of the upper teeth. 6. Incorrect. The client can eat a regular diet. Option 1 HOB elevated when sleeping. What would happen if the bed was flat? Increased pressure right? Yes. We don't want pressure on the suture line do we? No. So this is true. Option 2. Notify the PHCP for increased UOP. Could this be a complication of this surgery? Yes, because of Diabetes Inspidus. Not enough ADH, so the client could go into shock! Option 3. Because the incision for this surgery is just above the gumline, the client should not brush the front teeth. Oral care should be performed with a sponge until the incision heals.​ Option 4. There is no nasal packing. The incision is located just above the gumline of the upper teeth. Option 5. True. Humidified air prevents drying of nasal passages. Option 6 False. The client can eat a regular diet.

Which interventions are appropriate for the nurse to identify for a client admitted to the psychiatric unit for management of anorexia nervosa? You answered this question Incorrectly 1. Weigh daily. 2. Allow only 20 minutes of exercise daily. 3. Allow the client to bargain for privileges as long as the client eats. 4. Stay with the client during the established time for meals. 5. Maintain visual observation for 1 hour following meals.

1.,4. & 5. Correct: Weigh daily, immediately upon rising and following morning void, using same scale and clothes if possible. The established time for meals is usually 30 minutes. This takes the focus off of food and eating and provides the client with attention and reinforcement. The hour following meals may be used to discard food stashed from tray or to engage in self-induced vomiting. 2. Incorrect: The client will work with their primary healthcare provider to create a controlled exercise program. This is usually done once healthy eating habits and some weight gain is achieved. See the word only in this option and think incorrect. 3. Incorrect: Do not argue or bargain with the client who is resistant to treatment. Be matter of fact about which behaviors are unacceptable and how privileges will be restricted for noncompliance. The person who is denying a problem and who also has a weak ego will use manipulation to achieve control. Think nursing interventions. And how to manage anorexia nervosa. Option 1: A must! Weight loss programs recommend weighing weekly. But wait! This client with anorexia nervosa is seriously underweight. Weight gain is a life saving measure. The nurse needs to get a daily weight on this client in the psychiatric unit. And remember, if hospitalized, the anorexia nervosa is severe and causing acute symptoms in the client. Option 2: No. This inpatient client may be on restricted activity! A very individualized exercise plan prescribed by the primary healthcare provider is needed as the client improves. Option 3: No way. Bargaining is a nontherapeutic technique. The client will learn acceptable and unacceptable behaviors and how to obtain privileges. Option 4: Yes! Much can be observed by the nurse during meal time. The nurse can intervene appropriately during this time with positive reinforcement. Option 5: Oh yes! The anorexic client may engage in binge eating followed by self-induced vomiting. The client may use the hour after meals to induce vomiting due to client's fear of weight gain.

A client weighing 140 pounds (63.64 kg) has been admitted to the telemetry unit with a diagnosis of Class III pulmonary hypertension. The primary healthcare provider prescribes digoxin. How many micrograms should the nurse administer now? Round to the whole number.

140 pounds / 2.2 kg = 63.6363636 63.6363636 x 15 micrograms = 954.545455 954.545455 / 2 = 477.272727 or 477 micrograms now The following 2 doses would be *239 micrograms*.

A client has developed preeclampsia at 30 weeks' gestation. The nurse is instructing the client on an appropriate diet for preeclampsia. The nurse knows the teaching was successful when the client selects what menus? (Select All That Apply). 1. Caesar salad with feta cheese 2. Grilled cheese with tomatoes 3. Chipped ham on a croissant roll 4. Hot dog with a glass of soda pop 5. Chicken sandwich on wheat toast

2 and 5. CORRECT: A high protein, calcium rich diet is most important for the preeclampsia client who is losing protein in urine. Grilled cheese is an excellent selection for lunch, especially since it contains tomato slices, which adds another level of nourishment and vitamins. Additionally, a chicken sandwich, particularly on whole wheat toast, is very appropriate for this preeclampsia client. 1. INCORRECT: Caesar dressing is made with raw eggs, exposing the client to the potential for salmonella. Pregnant woman should not eat raw foods, including eggs, fish or meat. Additionally, feta cheese is a 'soft cheese', exposing the client to another bacterium known as listeria. Although a salad could be a good choice, this particular salad is not healthy. 3. INCORRECT: The need to restrict salt is not a priority for preeclampsia clients, but chipped ham is a processed meat containing less protein than other meats and increasing the risk for contracting listeria. The croissant roll is made of refined white flour and sugar. The client would benefit more from whole grain products. 4. INCORRECT: Processed meats, such as hot dogs, are not the healthiest choice for the client, as they increase the risk for listeria. Also, a client with preeclampsia should avoid alcohol, caffeine, and refined sugar to help control the blood pressure. The glass of soda pop is not a healthy selection.

The nurse is caring for a client following gastric bypass surgery. The client reports dizziness, sweating and palpitations after eating meals. The nurse would recommend which actions to alleviate these symptoms? 1. Increase liquids with food. 2. Reduce intake of carbohydrates. 3. Eat small, frequent meals daily. 4. Sit semi-recumbent for meals. 5. Remain upright for one hour after eating.

2, 3, & 4. Correct: The symptoms described indicate the client is experiencing dumping syndrome, an adverse response following gastric or bariatric surgery. Clients may also experience tachycardia, nausea or cramping with the intake of food due to surgical restructuring of the gastrointestinal tract. Because this will be a lifetime issue, the nurse must teach the client to adjust eating habits and patterns. Reduction of carbohydrates will help decrease the problem since carbohydrates speed through the digestive track too quickly. Eating smaller, more frequent meals in a semi-recumbent position will further slow food through the digestive track and eliminate most of the uncomfortable symptoms. 1. Incorrect: Increasing liquids while eating will speed food processing and increase the side effects. Clients are instructed to eliminate all fluids during meals. In some cases, clients may also need to eliminate fluids for one hour before and immediately after meals in order to control symptoms and slow the progress of food through the digestive tract. 5. Incorrect: Sitting up after a meal is counterproductive, since this will increase the speed of food through the digestive track. Therefore, clients are encouraged to lie down on the left side following meals to slow the processing of food.

A child diagnosed with gastroenteritis is being given fluids in the emergency room for severe dehydration. Prior to discharge, the nurse instructs the mother how to prepare a BRATT diet. The nurse knows the teaching was successful when the mother selects what foods for the child? 1. Raisins 2. Bananas 3. Apples 4. Toast 5. Rice 6. Tea

2, 4, 5 and 6. CORRECT: The Bratt diet is useful for children following any type of gastroenteritis which included nausea, diarrhea or severe vomiting. This bland diet is used in the first 24 hours to allow the gut to rest and readjust slowly to foods that are low protein, low fat and low fiber. The BRATT diet is for short term use only and consists of bananas, rice, apple sauce, toast and tea. 1. INCORRECT: Although raisins are normally a natural source of healthy fruit, they have too much fiber for an irritated gastric tract. They are not part of the BRAT diet. 3. INCORRECT: Apples are high in fiber and natural sucrose, which is not appropriate for a child with severe gastroenteritis. However, apple sauce is part of the BRAT diet and is an excellent source of nutrition without stressing a weakened gastrointestinal system.

After reinforcing dietary teaching to a client diagnosed with Crohn's Disease, the nurse recognizes client understanding when the client selects which low-residue foods? 1. Broccoli 2. Oatmeal 3. Green peas 4. Spaghetti 5. Cantaloupe 6. Raisins

2, 4,& 5 Correct: A low residue diet is recommended for clients with inflammatory bowel diseases such as Crohn's Disease, diverticulitis or Ulcerative Colitis. This special diet is designed to decrease fiber in order to limit bowel peristalsis while still including nutritional elements for clients. Cooked oatmeal or pasta are both good choices as well as fruits with no skin and little pulp. Cantaloupe is an excellent choice, since it is a great source of nutrients but has little pulp. 1. Incorrect: Broccoli is a very fibrous vegetable that causes excessive peristalsis, even when cooked. This will create excessive gas and increase discomfort for clients. 3. Incorrect: Green or yellow peas are rough, fibrous vegetables that will cause gas and cramping for most clients with inflammatory bowel diseases, even if cooked. This vegetable is discouraged along with beans, lentils, seeds and nuts. 6. Incorrect: Raisins are high in fiber as are other dried fruits such as prunes. Even cooked raisins tend to increase peristalsis which will lead to cramping and excessive bowel movements daily. Clients also need to avoid most fresh fruits with skin, pulp or seeds.

What instructions should the nurse include when teaching a mother, whose newborn has hyperbilirubinemia, regarding phototherapy and its effects? 1. Breastfeeding should be discontinued until phototherapy is completed. 2. Feed newborn at least every 2-4 hours. 3. Make sure the newborn's eyes are closed when applying eye patches. 4. Keep the baby quiet and swaddled. 5. Report immediately if the urine becomes dark in color.

2. & 3. Correct: Providing adequate breast milk or formula by feeding at least every 2-4 hours is key in preventing and treating jaundice because it promotes elimination of the bilirubin in the stools and urine. The infant should be monitored for signs of dehydration, including decreased skin turgor and decreased urinary output. Dehydration often results from decreased intake, phototherapy, and diarrhea. When applying the eye patches, the newborn's eyes should be closed to avoid causing a corneal abrasion. 1. Incorrect: Breast feeding is encouraged and is an important part of meeting both the nutritional and emotional needs of the newborn. 4. Incorrect: The infant's clothing is removed to allow maximum exposure of the skin to the phototherapy. The genitalia should be covered. 5. Incorrect: The caregiver should be taught to expect the infant's stools to be green and the urine dark because of photo degradation products (breakdown of bilirubin for excretion).

What interventions should the nurse include in the care plan of a client admitted with Guillain-Barre syndrome? 1. Assess for descending paralysis. 2. Keep a sterile tracheostomy at the bedside. 3. Monitor for heart rate above 120/min. 4. Maintain in side-lying, supine position. 5. Have client perform active range of motion (ROM) every 2 hours while awake.

2. & 3. Correct: This client is at risk for respiratory paralysis as the disease progresses. An emergency tracheostomy may need to be performed so the nurse should watch out for imminent signs of respiratory failure. Signs include heart rate that is more than 120 bpm or lower than 70 bpm and respiratory rate of more than 30 bpm. The nurse should assess for signs of respiratory distress and prepare for intubation if needed. 1. Incorrect: Ascending paralysis should be assessed for with this disease. Paralysis begins in the lower extremities and moves upward. 4. Incorrect: The client should be assisted to a position with head of bed elevated for full chest excursion. 5. Incorrect: The nurse should perform passive range of motion exercises. Active exercise should be avoided during the acute phase as the client is easily fatigued and muscles are weak. Passive ROM stimulates circulation, improves muscle tone and increases joint mobilization.

Who often performs the responsibilities of a case manager? 1. Physical therapist 2. Social worker 3. Dietitian nutritionist 4. Nurse 5. Unlicensed assistive personnel

2. & 4. Correct: A client's case manager can be a nurse, social worker, or other appropriate professional. Case management is a cross-disciplinary practice. It's function is to advocate for the client. 1. Incorrect: The physical therapist focuses on one area which is the client's ability to move and perform functional activities in their daily lives. The physical therapist would not be the client's case manager. 3. Incorrect: The dietitian nutritionist focuses on one area which is human nutrition and the regulation of diet. The dietitian nutritionist would not be the client's case manager. 5. Incorrect: The unlicensed assistive personnel does not have the education and/or training for case management.

What information would be included when a disaster relief nurse counsels parents of young clients who have experienced a disaster? 1. Act as if things are normal. 2. Understand young children may exhibit separation fears and clinging. 3. Sedate the client until the crisis is resolved. 4. Understand nightmares and sleep disturbances may occur in young children. 5. Refrain from talking about the disaster.

2. & 4. Correct: Following a disaster, children exhibit a range of emotional and physiological reactions including separation, fear, and sleep issues. They may also appear confused, passive, fearful, and have somatic symptoms. They have difficulty talking about the event or identifying feelings. 1. Incorrect: Acting as if nothing happened is a nontheraputic parental response. The parents should recognize and acknowledge the child's feelings. 3. Incorrect: Sedation may be an emergency need, but more therapeutic responses are quickly warranted. 5. Incorrect: Refraining from talking about the disaster would be nontherapeutic. Again, allowing the child to discuss their feelings will assist the child in working through their fear and worry. Option 1 does not explore the client's feelings. This option ignores feelings. Option 3 says to give medication! Stay away from medication as long as possible on NCLEX! This option ignores feelings as well. Option 5 ignores the client's feelings. So 1, 3, and 5 are false. Options 2 and 4 are both true statements.

In which situations should the nurse notify the primary healthcare provider of a medication incident? 1. Every occurance. 2. Client is harmed or dies. 3. Medication incident is a near miss. 4. Nurse administers an incorrect dosage. 5. Client questions the medication color.

2. & 4. Correct: The primary healthcare provider should be notified if harm is brought to the client or death occurs as a result of the medication incident. The primary healthcare provider should be notified if the nurse administers an incorrect dosage to the client, and an incident report needs to be completed in this situation. 1. Incorrect: The primary healthcare provider should be notified if harm is brought to the client but not for all events with medications. An incident report should be completed so the hospital can track incident patterns for quality improvement. 3. Incorrect: Near misses do not need to be reported to the primary healthcare provider. Following the rights of medication administration every time ensures medication error prevention. 5. Incorrect: The nurse should answer questions regarding medication color. Depending on the manufacturer, the shape and color of the medication can vary.

The nurse would make which recommendations when conducting community health teaching about obesity to a group of adolescents? 1. Limit TV viewing and video game playing to 4 hours a day 2. At least 60 minutes of moderate-intensity activity daily 3. Exercise should be structured 4. A strict diet should be followed avoiding all junk food and drinking water only 5. Set a goal of at least 11,000 to 13,000 steps each day

2. & 5. Correct: 60 minutes of moderate-intensity physical activity 7 days a week. Girls should take a least 13,000 steps daily and boys should take 11,000 steps daily. 1. Incorrect: TV viewing and video game playing should be 2 or fewer hours each day. 3. Incorrect: Exercise does not need to be structured. 4. Incorrect: If the diet is too restrictive, it is likely to fail. 1. Look at each option as True or False. 2. Option 1 is false. The American Academy of Pediatrics (AAP) recommends TV viewing and video game playing be limited to 2 hours a day. 3. Option 2 is true. The American Heart Association (AHA) recommends at least 60 minutes of moderate-intensity physical activity 7 days per week. 4. Option 3 is false. Exercise does not necessarily have to be structured. For example, it might consist of family walks, bike riding, skateboarding, or swimming. Even table tennis requires more activity than watching television. 5. Option 4 is false. Encourage good eating habits such as avoiding junk food and drinking water in place of sodas. If the diet is too restrictive, and it had none of the child's favorite foods, it is likely to fail. Focus on making small but permanent changes. 6. Option 5 is true. The AAP recommends boys take at least 11,000 steps daily and girls take at least 13,000 steps daily. Parents could buy a pedometer so the child can track the number of steps.

A client has experienced a cerebrovascular accident (CVA) which resulted in left homonymous hemianopia. Based on this fact, what measures will the nurse include in the client's initial plan of care? 1. Approach the client from his left side. 2. Place the client's meal on the right side of the over bed table. 3. Request a consult for an ophthalmologist. 4. Stand directly in front of the client when addressing. 5. Have client look at the left side of the body.

2. & 5. Correct: Homonymous hemianopia is blindness in half of the visual field. The client has lost half of the visual field in the left side of both eyes. To avoid startling the client who has lost vision in half of their visual field and so the client can better view the food, the nurse should approach the client from the right side. Neglect of the left side can occur because the left side is out of the visual field. Encourage the client to intentionally look at the left side of the body to avoid neglect. 1. Incorrect: Approaching the client from the left side is inappropriate. The client cannot see in the left half of the visual field and could be startled. 3. Incorrect: An ophthalmologist cannot fix this problem. The problem is due to damage in the central nervous system, not in the eyes. 4. Incorrect: Standing in front of the client does not address the client's visual field deficit. This would give the client an altered view of the nurse since the left half of the visual field is affected. In order to get this answer correct you must know the definition of homonymous hemianopia. Homonymous hemianopia is blindness in half of the visual field. This client has lost half of the visual field in the left side of both eyes. So the client cannot see things on the left side. Remember safety and prevention of neglect of the affected side. If you cannot see one side of the body, the brain forgets about it. Option 1. Do we want to approach the client from the side the client can see you coming? No, so this is false. Option 2. Place the client's meal on the right side of the over bed table. True. You want the client to be able to see the food, right? right. Option 3. Request an ophthalmologist consult. False. Can this problem be fixed. No it is not an eye problem but rather a central nervous system problem. Option 4, stand directly in front of the client. False. You would need to stand on the unaffected side, in this case the right side, so that the client can see you. Option 5, have the client look at the left side of the body. This allows the brain to connect with the left side of the body. If the client cannot see parts of the body, then the brain will not connect with it.

The nurse is preparing to discharge a client home from the hospital. Which statement made by the client indicates to the nurse that instructions about antibiotic administration have been successful? 1. "I will take the antibiotic until I feel better but save some to take in case the infection returns." 2. "I should follow the instructions on the label." 3. "I need to double the dose for two days so I will get better." 4. "I should double the dose the next time the antibiotic is due after missing a dose." 5. "I will finish all of my antibiotic medication."

2. & 5. Correct: Instruct the client to follow the instructions and finish the entire prescription. Failure to do so could lead to resistance or relapse. 1. Incorrect: Not taking the whole prescription can lead to resistance of the organism and cause a relapse of the infection. 3. Incorrect: Medication doses should never be doubled. The prescription should be taken as instructed per the primary healthcare provider. 4. Incorrect: Medication doses should never be doubled, even if one dose is missed.

Which tasks can the nurse delegate to the unlicensed assistive personnel (UAP)? 1. Reporting lab results to the client 2. Measuring intake and output 3. Discontinuing an IV 4. Discussing client condition with the client's spouse 5. Performing oral hygiene for an older client

2. & 5. Correct: Measurement of intake and output and oral hygiene for the older client are tasks that the UAP can perform, and these tasks may be delegated. 1. Incorrect: Reporting of lab results should be accomplished by the nurse who has the knowledge to interpret results. This is not appropriate for the UAP and must be done by a licensed nurse. 3. Incorrect: Removal of the IV requires assessment skills that the unlicensed assistive personnel does not have. 4. Incorrect: Discussion of client's condition should be done by the nurse with the client's permission. To answer this question correctly, you must know what the nurse can safely delegate to the UAP. Remember, activities of daily living and routine tasks on stable clients such as vital signs, daily weights, and intake and output measurements. So with this in mind, let's look at the each option as a true/false question. Option 1: Reporting lab results to the client. This is false. First of all, the UAP has no reason to even look at lab results, so this would be a violation of HIPPA. The UAP cannot explain lab results. So, this is the task of the nurse. Option 2. True. The UAP can measure intake and output. This is part of the UAPs training. Option 3. False. Discontinuing an IV is beyond the scope of practice for the UAP. It has nothing to do with ADL and is not an uncomplicated, routine task. Option 4. What do you think? False. Again this is a HIPPA violation. The UAP should not have access to this information and discussing the client's condition is out of the scope of practice / education of the UAP. Option 5. Oral care? True. Isn't this part of our activity of daily living? Yes.

The nurse notices that a client's bedside privacy curtain has been left partially open during the client's bath. Which are appropriate actions for the nurse to take in order to ensure the client's right to privacy? 1. Inform the client that the curtain was left partially open. 2. Close the privacy curtain to protect the client's right to privacy. 3. Since the client did not notice the open privacy curtain no action is necessary. 4. Only a few visitors are on the unit at this time so no action is necessary. 5. Instruct the nurse giving the client's bath about the open curtain and need for privacy.

2. & 5. Correct: The curtain should be closed as soon as the opening is noticed to protect the client's right to privacy. Also the nurse giving the bath should be made aware of the partially open curtain and the need to maintain privacy for the client. 1. Incorrect: Informing the client that the curtain was open is an embarrassment to the client and may lead to a lack of trust in additional care that may be provided. It does not help solve the privacy issue, which the nurse has the responsibility to protect. 3. Incorrect: The "No action is necessary" option is incorrect because the client has the right to privacy. Nurses should take client's rights seriously and carry out needed measures to assure these rights are protected, even when the client is unaware of the situation. 4. Incorrect: The "No action is necessary" option is incorrect because the client has the right to privacy. It is irrelevant how many people may have been in close proximity. The nurse should take the client's rights to privacy very seriously and ensure measures are taken to protect this right. This question takes us back to the basics. Clients have a right to privacy. How do you maintain client privacy when bathing? You want to make sure that the client's door is closed, the curtain is drawn, and that the client is covered as much as possible when providing a bath. Place a sign on the door instructing people to knock prior to entering room. So let's see what options protect the client's privacy. Option 1. How does informing the client about the breach in privacy ensure the client's right to privacy? It doesn't. Option 2. Closing the curtain will protect the client's privacy. This is an action that will protect the client. Option 3. No action will always be wrong. You aren't going to leave the curtain open. Option 4. This is a "no action" statement as well. Wrong. Option 5. It is the RNs role to teach. Teach clients, families, staff.

The client's primary healthcare provider orders a blood transfusion for a client whose hemoglobin level is 5.0 mg/dL (3.103 mmol/L). The nurse informs the client that blood will be drawn for a type and cross-match prior to the blood transfusion. The client avoids eye contact with the nurse and states, "I am a Jehovah's Witness. I thought that was on my chart." The nurse demonstrates the role of client advocate by which response to the client? 1. "Your hemoglobin is very low. I can notify your primary healthcare provider to discuss with you how important it is for you to receive the blood." 2. "I will place that information in your medical record. You have the right to refuse treatment which conflicts with your beliefs." 3. "Your primary healthcare provider ordered this blood transfusion because your hemoglobin is low." 4. "Why do Jehovah's Witnesses choose not to receive blood transfusions?" 5. "Would you like to speak with your primary healthcare provider about other treatment options?"

2. & 5. Correct: The nurse as client advocate supports the client's beliefs and treatment wishes. Reaffirming the client's beliefs and treatment wishes is important to promote a sense of trust and demonstrate respect for the individual. The information that the client is Jehovah's Witness who refuses blood products should be placed prominently in the medical record per the facility's policy to avoid further confusion. Providing reasonable healthcare treatment choices that do not conflict with the client's beliefs is the responsibility of the multi-disciplinary healthcare team. 1. Incorrect: This response disregards the client's wishes and religious beliefs. It is appropriate for the client to be made aware of the low hemoglobin level, but the client's wishes and religious beliefs should be respected without being told that blood should be received. 3. Incorrect: Again, this response disregards the client's wishes and religious beliefs. This comment could belittle the client's wishes and beliefs by making the primary healthcare provider the "authority" or focus here. 4. Incorrect: Questions reasons for religious beliefs and does not focus on client's needs. This response could place the client in a defensive situation. Clients should not be asked to explain or defend personal or religious beliefs.

The nurse receives report about a client who is termed "a drug seeker". The nurse giving report states that the client does not need the pain medication and is just asking for medication because the client is "hooked on it." After receiving report, what actions should the nurse take? 1. Consult with the primary healthcare provider. 2. Assess the client. 3. Increase gradually the time between pain medication. 4. Encourage the client to wait longer before requesting the medication. 5. Utilize a pain scale to determine level of pain.

2. & 5. Correct: The nurse should carefully assess the client. The nurse must serve as an advocate for the client. A pain scale is used to determine level of pain. 1. Incorrect: The nurse must assess the client before consulting with the primary healthcare provider about the medication. 3. Incorrect: This action assumes that the client does not have pain, which does not take into consideration what is wrong with the client. 4. Incorrect: This action assumes that the client is a "drug seeker". The nurse must carefully assess the client. Back to the Basics here! Pain assessment is considered the fifth vital sign. And remember that pain is what the client says it is. Nurses should not be judgmental and stereotype clients based on the nurse's own beliefs. The nurse coming on duty must carefully assess the client independently and without bias. Look at option 1. Is there a need to call the primary healthcare provider at this point? No. The nurse needs to assess the client. Which is what option 2 says to do. So option 1 is false and option 2 is true. What about option 3? Gradually increase the time between pain medication. Doing this may actually make it harder to relieve the client's pain. Pain relief works best when pain is not to the highest level of severity. Pain medication around the clock during the acute phase of pain is the treatment of choice for adequate pain relief and smaller doses are usually adequate when provided this way. Option 4: This option is very similar to option 3 which we just said was false. Now option 5: Use a pain scale. True. We know that this is a way to objectively evaluate pain.

An Asian client, who cannot speak or comprehend English, is brought to the emergency department by family. One family member is able to understand simple sentences of English. How would the nurse best explain how to obtain a clean catch urine to the client? 1. Have the family member repeat the nurse's explanation to the client. 2. Contact Social Services to find an authorized interpreter. 3. Use simple hand motions to explain the procedure to the client. 4. Draw a diagram to demonstrate the use of the sterile cup when obtaining the specimen.

2. CORRECT. Hospitals must have a means of communicating with a variety of non-English speaking clients, as well as deaf clients. It is vital to have interpreters that are capable of translating medical terms or instructions correctly and also to relay the client's specific concerns back to medical staff. Because of the importance of accuracy, only trained and qualified interpreters should be used when communicating with those who do not understand English. 1.INCORRECT. Although the family is present and may be able to translate information from the nurse to the client, the accuracy of that information cannot be guaranteed since the family member has a limited understanding of English and no medical knowledge. It is vital that the procedure be clearly explained to the client, since test results will be affected if the specimen is not correctly collected. 3. INCORRECT. The use of hand motions does not ensure that the client will understand the procedure correctly, and in this particular situation, demonstrating some of the steps could be embarrassing to this client. Hand motions do not provide a means for the client to ask questions about anything that is not clearly understood. 4. INCORRECT. Although there may be circumstances in which a nurse can use alternative methods of communication, such as picture boards or hand gestures, this situation calls for clear instructions on the proper method for obtaining a clean catch urine so that test results are accurate. Additionally, a picture does not allow the client to confirm understanding of the process.

A client who is gravida 2 para 1 is visiting the obstetric clinic for a checkup. The first delivery was a cesarean for failure to progress and the client indicates a desire for a vaginal delivery this time. The nurse knows the most important factor in determining the possibility of a vaginal birth after cesarean (VBAC) is what? 1. The length and difficulty of the previous labor. 2. The type of incision used for the cesarean. 3. The position of the fetus before delivery. 4. Total number of pregnancies desired.

2. CORRECT: A VBAC is often requested by a client for a number of reasons. There is less pain after delivery with a shorter recovery period and less chance of infection. A VBAC also can potentially increase the number of pregnancies possible, since cesarean sections dramatically limit the number of children. The main factor that determines whether the client could safely have a VBAC is the type of uterine incision made for the previous C-section. Those who have had a low, transverse incision are candidates for trial of labor after cesarean (TOLAC). 1. INCORRECT: Obviously, in this situation, the client had experienced failure to progress in the previous pregnancy, necessitating the need for a C-section. The length and difficulty of the previously attempted birth would have affected the decision to have the first C-section, but would not impact the current choice to attempt a VBAC. 3. INCORRECT: The position of the fetus prior to delivery does not have a significant bearing on the decision to have a VBAC. Clients who intend to try a vaginal birth are very closely monitored prior to labor. If the obstetrician determines by ultra sound that the fetus is incorrectly position, there is the possibility of "turning" the fetus prior to the onset of labor. Only in extreme circumstances would the fetal position prevent vaginal birth. 4. INCORRECT: The factor that is most important in determining the chance for a VBAC is not the number of pregnancies desired, but rather the position of the uterine incision made with the last C-section. The ability to actually have a successful VBAC may affect the total number of children desired.

While preparing to hang a bag of chemotherapy the nurse accidently pulls the tubing apart, spilling the solution onto the floor. After clamping the tubing, what is the nurse's immediate action? 1. Use disposable towels to clean up the liquid. 2. Obtain spill kit specific to this type of solution. 3. Complete an incident report for supervisor. 4. Call housekeeping to help clean up the floor.

2. CORRECT: Chemotherapy spill kits are pre-packaged supplies specific to the type of cytotoxic drugs used and are kept in close proximity to the location the chemo is administered. These kits vary slightly but all follow the basic guidelines. Individuals cleaning up the spill must be completely covered head to toe to prevent any contact with the drug. This includes inhalation. (Notice that this option contains the word solution, which also appears in the question.) 1. INCORRECT: Disposable towels are not acceptable to clean up a chemotherapy spill. Although these towels are absorbent for kitchens and bathrooms, only special absorbent pads can be used to clean up cytotoxic drugs. 3. INCORRECT: While it is true that the nurse will need to complete an incident report regarding the chemo spill, it is certainly not the nurse's immediate action. Focusing on staff and client safety is first. 4. INCORRECT: The responsibility for cleaning up cytotoxic drugs is for the nursing staff involved at the time. Special training and knowledge is required to handle this issue, and fewer individuals exposed would be much safer.

A primigravida client at 35 weeks gestation has been diagnosed with human papillomavirus (HPV). The nurse knows that the most important information to discuss with this client is what? 1. The infant will not be able to breast feed. 2. The mother will need frequent follow up Pap smears. 3. The fetus will need to be delivered by C-section. 4. The mother must start metronidazole immediately.

2. CORRECT: HPV is a sexually transmitted viral infection that can cause genital warts or even precancerous lesions. This virus is spread by direct contact with infected mucous membranes, and is transmitted through sexual contact. Although HPV generally clears itself through the human immune system, clients diagnosed with this infection are recommended to have a follow-up Pap smear every six months for the first year, particularly if infected with HPV 16 or HPV 18. 1. INCORRECT: The risk of transmitting HPV in breast milk is extremely minimal. Research has shown that the miniscule amounts of HPV which could be transmitted do not outweigh the benefits of allowing the infant to breastfeed. 3. INCORRECT: The chance of transmitting HPV during vaginal birth is small. Even in the presence of non-cancerous genital warts, the greatest concern is whether the birth canal is blocked by these growths. The existence of warts does not mean the client will automatically need a cesarean section. 4. INCORRECT: The primary healthcare provider will treat the mother and all sexual partners for the HPV, usually with a medication such as metronidazole. However, this information is not the most important topic for the nurse to discuss with the client at this time.

A child diagnosed with acute lymphocytic leukemia (ALL) is receiving vincristine sulfate during the induction phase of chemotherapy. What client side effect should the nurse report immediately to the primary healthcare provider? 1. Anemia 2. Paresthesia 3. Nosebleeds 4. Alopecia

2. CORRECT: Paresthesia is an uncommon but serious reaction to chemotherapy, particularly vinca alkaloids like vincristine sulfate. The abnormal tingling or "pins and needles" sensation is caused by pressure or damage to peripheral nerves which may include both motor and sensory sensations. The nurse should immediately notify the primary healthcare provider of this critical side effect of vincristine therapy. 1. INCORRECT: Anemia is an expected side effect of many types of chemotherapy, including vincristine sulfate. Chemotherapy drugs attack rapidly dividing cells, including those that create red blood cells. Anemia contributes to fatigue, shortness of breath and lack of energy. This should definitely be evaluated, but is not the most urgent concern for the nurse at this time. 3. INCORRECT: Vincristine sulfate has many side effects, including depletion of platelets which are responsible for blood clotting. When platelets are depleted, the client can experience nose bleeds, bruising, or bleeding gums. While this is of concern, it is not the side effect which needs reported immediately to the doctor. 4. INCORRECT: Alopecia is a very common, expected side effect of chemotherapy. Damage to hair follicles commonly occurs after the first two treatments. While this side effect can impact the client psychologically, it is not an issue that the nurse must immediately report.

A preschooler has been hospitalized for observation. The unlicensed assistive personnel (UAP) offers to sit with the child and asks the nurse to suggest an appropriate activity. The nurse knows the best activity choice for a preschool child is what? 1. Children's television show 2. Small stacking blocks 3. A checker board game 4. Children's card game

2. CORRECT: Preschool children, ages 3 to 5, are in the Erikson stage of "initiative versus guilt" where the learning goals involve exploration and manipulation of the environment. Motor skills are developing and playing is used to increase self-esteem through imagination and creativity. Stacking small blocks to build structures or create creatures is definitely appropriate for this age group and can easily be done on the bedside table. 1. INCORRECT: A preschool child does not have the patience to sit through an entire television show. Additionally, this would not address the developmental needs of this age group, which focuses on creative activities such as coloring, painting, playdough or building blocks. Even hospitalized children must have their developmental needs addressed. Television may appeal more to adolescents. 3. INCORRECT: Table games like checkers are more appropriate for school age children, who tend to like group activities, particularly with peers of the same age and sex. Playing checkers is too tedious and inactive for a young preschooler. 4. INCORRECT: Card games, even those designed for children, are generally too boring for youngsters. Preschool children prefer activities which require imagination and activity with others. Dressing up in clothing, riding bikes or other physical games are good for engaging this age group. In the hospital, creativity can be encouraged with drawing, chalk, or playdough.

A client in the third trimester of pregnancy arrives at the emergency room reporting general illness. The client is noted to have a blood glucose level of 390 mg/dL and is diagnosed with gestational diabetes. The primary healthcare provider orders 30 units of NPH subcutaneous stat. What is the nurse's priority action? 1. Administer the dose of insulin immediately. 2. Question the type of insulin ordered. 3. Insert an IV for an insulin infusion. 4. Question the dose of the insulin.

2. CORRECT: The client's blood glucose is extremely high and needs to be quickly reduced. The order given by the primary healthcare provider is for 30 units of NPH insulin, a long-acting insulin whose onset is about 1 ½ hours. That is too long to wait to start reducing this elevated glucose. This client should have been ordered regular insulin. otice that the client was ordered NPH insulin. Remember that NPH is long-acting insulin with an onset of about 1 ½ hours. That is too long to wait to start bringing down such an elevated insulin. The nurse's first priority should be to question the type of insulin ordered for the client. 1. INCORRECT: While this client should indeed receive insulin immediately to start reducing the blood glucose, there is a problem with the order that the nurse must address before implementing. 3. INCORRECT: Even though the primary healthcare provider has not ordered an insulin drip, an IV would be an important intervention with a pregnant female whose blood sugar is very high. However, there is another problem that takes priority first. 4. INCORRECT: Thirty units of insulin is not an unreasonable dose for a blood glucose level of 390 mg/dL. However, the amount of insulin is not the problem here. There is another issue that is of greater concern for the nurse.

Question 38: A community health nurse is presenting a seminar to teen parents on the topic of infant safety. What priority topic presented by the nurse represents the leading cause of injury or death among infants? 1. Monitoring the infant for unexpected food allergies. 2. Placing the infant in rear-facing, approved car seat. 3. Never propping bottle to feed when infant is alone. 4. Positioning infant supine when sleeping or napping.

2. CORRECT: The leading cause of death among infants under the age of one year is motor vehicle accidents. When instructing first time or young parents, it is vital to teach the need to have the infant snuggly restrained in an appropriately sized, approved infant car seat in the back seat and rear-facing. 1. INCORRECT: While discussing the signs or symptoms of undiagnosed food allergies is an important topic for new parents, this is not the most vital information the nurse could present to the teen parents. 3. INCORRECT: An infant should never be left unattended while feeding, and propping a bottle could lead to aspiration or respiratory distress. This is a dangerous practice that needs to be discussed by the nurse; however, there is another topic that is more urgent. 4. INCORRECT: The research studies to date indicate the safest sleeping position for newborns and infants is supine, not prone. Positioning is always a nursing concern, and teaching new parents about the potential for sudden infant death syndrome (SIDS) would be crucial. However, another topic presents more important information.

A preeclampsia client is being treated with magnesium sulfate. The nursing assessment shows a respiratory rate of 10 with deep tendon reflexes of 0. What is the nurse's priority action? 1. Place client in Trendelenburg position and apply oxygen. 2. Stop magnesium and prepare to give calcium gluconate. 3. Ask another nurse to verify the deep tendon reflexes. 4. Prepare client for an emergency cesarean section.

2. CORRECT: The nurse's findings indicate the client's central nervous system has been overly depressed, with a respiratory rate of 10 and absent deep tendon reflexes. The nurse's priority intervention is to stop the magnesium, which is the factor cause of the problem, and prepare to reverse the situation with calcium gluconate. 1. INCORRECT: Placing a client head down, in Trendelenburg position, is used for treating shock. No information is presented indicating shock. Also, there is no information about oxygen saturation levels that would indicate the need for oxygen. 3. INCORRECT: It is not unusual for one nurse to ask another nurse to confirm abnormal findings; however, in this situation, it would be considered a delay of treatment and transfers care of the client to someone else. The nurse's priority action must focus directly on the client. 4. INCORRECT: The purpose of administering magnesium sulfate is to prevent seizures and decrease the blood pressure in a preeclampsia client. There is no information in the question that indicates that either the client or the fetus is in distress, requiring an emergency section. Option 1: Oh no! We never place a pregnant female in a Trendelenburg positon. Lying supine places pressure against the vena cava and diminishes blood supply to the placenta. Also, this position would shift the weight of the fetus up against the mother's diaphragm, making breathing very difficult and respirations are already down to 10. Oxygen needs are based on saturation levels, which are not provided in this question. Therefore, neither of these nursing actions would benefit the client. Option 2: Awesome! You have noticed the two important clues that indicate the mother's central nervous system has been depressed too much by the magnesium sulfate. A respiratory rate of 10 is low. But more importantly, deep tendon reflexes are no longer present. Without deep tendon reflexes (DTRS), a client's system cannot respond appropriately, including protecting the airway. This situation has been caused by the magnesium sulfate, which must be turned off immediately. The nurse should be prepared to administer the antidote of calcium gluconate to correct the problem. Option 3: No way. First, this option transfers care of the client to someone else, which is not a safe action on the NCLEX. Secondly, though it seems logical that a nurse might want to verify abnormal findings, this is considered a delay in appropriate treatment for the client. There is a more appropriate action the nurse should implement at this time. Option 4: Definitely not. Although this mother is being treated with magnesium sulfate for preeclampsia, the issue is neither maternal nor fetal distress, which is the only justification for an emergency cesarean section. The information provided indicates the mother's central nervous system is extremely depressed, making the mother too unstable for surgery. Look at the clues again and think what might help stabilize the mother.

An emergency room nurse must evaluate a toddler with a possible broken arm sustained in a fall. The child is crying in pain while the mother attempts to provide comfort. What action by the nurse is most appropriate initially? 1. Gently move fingers of injured arm. 2. Observe skin color and appearance of injured arm. 3. Auscultate bilateral breath sounds. 4. Obtain a tympanic temperature.

2. CORRECT: The nursing process begins with assessment and data collection. It is particularly important when caring for a child in pain to observe before touching. The nurse can observe the color and appearance of the injured arm, gathering some valuable data before proceeding to more painful aspects of the examination. Other aspects of the nursing assessment, including vitals, may decrease cooperation because of added discomfort. 1. INCORRECT: When dealing with a child, the more painful aspects of the assessment should wait until last, since it will become more difficult to get needed cooperation. Instead of actually moving the fingers, the nurse should ask the child to wiggle the fingers. Although the nurse will at some point need to gently palpate that arm to determine skin temperature, anything that may increase discomfort should be kept for last. 3. INCORRECT: Auscultation of lung sounds is part of a standard nursing assessment. However, in this particular situation, the child is crying which is a good indicator of respiratory status. It also makes it more difficult to actually assess the lungs. The focus at this point should be on the injured arm. 4. INCORRECT: Vitals will definitely be important, and a tympanic temperature is a non-invasive process that will be easier for this distraught toddler. However, depending on the child's degree of pain and fear, even something as simple as a tympanic temperature may cause distress. This is not the nurse's first action.

A client admitted for debridement of a leg wound has been diagnosed with vancomycin-resistant enterococci (VRE). What is the nurse's priority action? 1. Place with another client in contact isolation for methicillin-resistant staphylococcus aureus (MRSA). 2. Move the client to a private room with contact precautions. 3. Alert staff to use masks, goggles and gown to provide care. 4. Notify family members to gown and glove before entering room.

2. CORRECT: VRE normally lives in healthy intestines, but in an immunocompromised client, it can be found in a number of locations, including urinary tract, intestines, blood or wounds. A client diagnosed with VRE must be placed alone in contact isolation for the entire hospital stay, or until there are three negative cultures of that area, each one week apart. 1. INCORRECT: The location of this client's VRE is not noted, and it cannot be assumed that it is in the leg wound. A client with VRE must be placed in a private room. This client definitely cannot be placed in the same room as a client with MRSA just because both require contact isolation. Cross-contamination may occur between the two clients. 3. INCORRECT: VRE requires contact precautions. Anyone entering the room must first apply a gown and gloves. Before exiting the room, the gown and gloves are removed, discarded inside the room and hands washed prior to leaving. Goggles and mask are not required in this situation. 4. INCORRECT: Anyone entering the room will need to follow the contact isolation protocols, including staff, visitors and family. A gown and gloves must be used when entering the room, then removed and discarded prior to leaving the room. However, this is not the nurse's priority.

A new nurse is preparing to give a medication to a nine month old client. After checking a drug reference book, crushing the tablet and mixing it into 3 ounces of applesauce, the new nurse proceeds to the client's room. What priority action should the supervising nurse take? 1. Tell the new nurse to recheck the drug reference book before administering the medication. 2. Suggest the new nurse reconsider the client's developmental needs. 3. Check the prescription order and the client dose. 4. Observe the new nurse administer the medication.

2. Correct. Mixing medication with applesauce is appropriate in some circumstances, but the volume of 3 ounces is excessive for a nine month old. The nurse will want to make sure the client gets all of the medication. Additionally, applesauce may or may not have been introduced into the diet, and it is inappropriate to introduce a new food during an illness. 1. Incorrect. There is nothing in the stem about a problem with the medication dose or route. The drug reference book does not provide guidelines for meeting developmental needs when administering the medication. This is something that the nurse must look up if uncertain about developmental tasks. 3. Incorrect. There is nothing in the stem about a problem with the medication dose or route. Once the medication has been mixed in applesauce, the supervising nurse would not be able to compare the dose to the prescribed amount. Therefore, this would not be an appropriate action. It would not address the developmental task that is the underlying issue here. 4. Incorrect. This is an appropriate action. However, it is not the priority. The new nurse should be competent in medication administration but is needing guidance with the developmental considerations related to medication to a nine month old.

A pediatric nurse notes a "chubby" toddler who is pale. According to the parent, the toddler is easily fatigued. Based on this data, what initial question should the nurse ask the parent? 1. "How much weight has your child gained in the past month?" 2. "How much milk does your toddler drink in a day?" 3. "How many hours does your toddler sleep within a 24 hour period?" 4. "Do you give your child vitamins every day?"

2. Correct. Paleness and easily fatigued are common signs of anemia. Drinking large amounts of milk puts the child at increased risk for iron deficiency anemia. Breast milk and cow's milk are poor sources of iron. Milk also inhibits absorption of iron. 1. Incorrect. This is not the most important question to ask first. The nurse needs to know how much milk the toddler is consuming as it will decrease iron consumption. 3. Incorrect. This is not the most important question to ask first. The nurse needs to know how much milk the toddler is consuming as it will decrease iron consumption. 4. Incorrect. This is not the most important question to ask first. The nurse needs to know how much milk the toddler is consuming as it will decrease iron consumption. Also, the nurse needs to ask specifically about iron supplements. Anemia caused by a low iron level is the most common form of anemia. The body gets iron through certain foods. It also reuses iron from old red blood cells. A diet that does not have enough iron is the most common cause. During periods of rapid growth, even more iron is needed. Babies are born with iron stored in their bodies. Because they grow rapidly, infants and toddlers need to absorb a lot of iron each day. Iron deficiency anemia most commonly affects babies 9 through 24 months old. Breastfed babies need less iron because iron is absorbed better when it is in breast milk. Formula with iron added (iron fortified) also provides enough iron. Infants younger than 12 months who drink cow's milk rather than breast milk or iron-fortified formula are more likely to have anemia. Cow's milk leads to anemia because it: has less iron, causes small amounts of blood loss from the intestines, makes it harder for the body to absorb iron, and children older than 12 months who drink too much cow's milk may also have anemia if they do not eat enough other healthy foods that have iron.

Which prescription should the nurse question when a client is receiving spironolactone 25 mg by mouth daily? You answered this question Correctly 1. Digoxin 0.125 mg by IVP daily 2. Potassium chloride 40 mEq orally t.i.d. 3. Cimetadine 200 mg IVPB q6h 4. Metoprolol 100 mg p.o. daily

2. Correct: Do not give potassium supplements, salt substitutes, or angiotensin-converting enzyme inhibitors to clients taking potassium sparing diuretics because these drugs can increase the risk of developing high to extremely high blood potassium levels. 1. Incorrect: This medication does not adversely interact with potassium sparing diuretics; however, the nurse should be on the alert for digoxin toxicity with hyper or hypokalemia. 3. Incorrect: Cimetadine is a H2 receptor antagonist indicated for ulcers and GI complaints. It does not adversely interact with potassium sparing diuretics. 4. Incorrect: This medication is a beta blocker, which may be given in addition to a diuretic for hypertension control.

A client scheduled for electroshock therapy becomes anxious prior to the initial treatment and refuses the procedure. What is the nurse's priority at this time? You answered this question 1. Administer pre-op sedation to help the client relax. 2. Notify the primary healthcare provider of the client's refusal. 3. Remind the client that the consent is already signed. 4. Ask the family to help convince the client to re-consider

2. Correct. The client has withdrawn consent for the procedure; therefore, the primary healthcare provider should be informed immediately to cancel the treatment. The primary healthcare provider may wish to speak with the client, but the client can legally refuse any procedure at any time. 1. Incorrect. Pre-op sedation is considered part of the procedure for which the client has withdrawn consent. Giving this medication would violate the client's right to refuse treatment and could be considered assault by the nurse. 3. Incorrect. Signing a consent form indicates that the primary healthcare provider has informed the client of all potential risks of the procedure. The client's signature represents an acknowledgement and understanding of that explanation. It is not an iron-clad contract agreeing to have the procedure. 4. Incorrect. Involving family members to try to convince the client to have this procedure would be unethical and could be considered coercion. Depression does not make the client incompetent to make decisions about healthcare options.

The family of a client recently placed on antipsychotic medications for the treatment of schizophrenia calls the nursing hot line and reports that the client's temperature is 105.1ºF (40.6ºC), and that the client's muscles are stiff. What should the nurse tell the family? 1. Continue to monitor for signs and symptoms of infection. 2. Transport the client to the emergency room. 3. The signs and symptoms will subside within a day or so. 4. They should call the primary healthcare provider tomorrow.

2. Correct. The client may be experiencing neuroleptic malignant syndrome, a potentially life threatening adverse reaction. Symptoms include high fever, unstable blood pressure and myoglobinemia. The client should be taken to the ER. 1. Incorrect. The client may be developing neuroleptic malignant syndrome. This high fever is not associated with infection. Immediate treatment is necessary. 3. Incorrect. The symptoms will progress and may lead to death. Remember, do not delay treatment. 4. Incorrect. The symptoms will progress and the client may die without treatment. Do not delay treatment!

Which infant in the newborn nursery requires an immediate intervention by the nurse? 1. Four hours old, who has passed a small meconium stool. 2. Three hours old, who is having tremors. 3. Two hours old, who has several episodes of apnea lasting 10 seconds. 4. One hour old, who has acrocyanosis.

2. Correct. Tremors and jitteriness indicate hypoxia which requires immediate intervention by the nurse. 1. Incorrect. Meconium is usually passed within 24 hours. This does not require intervention by the nurse. 3. Incorrect. Apnea of 5-15 seconds is not unusual and decreases with time. This does not require intervention by the nurse. 4. Incorrect. Acrocyanosis in the newborn is due to premature peripheral circulation that will improve. This does not require intervention by the nurse.

Which client should the nurse assign to a room closest to the nurse's station? You answered this question Correctly 1. A multigravida admitted with a new diagnosis of gestational diabetes 2. A primigravida admitted with a diagnosis of placenta previa 3. A primigravida admitted with a diagnosis of complete abortion 4. A pregestational diabetic admitted for glycemic conteol

2. Correct: A client with a diagnosis of placenta previa is at high risk for bleeding and must be monitored closely. Placenta previa is a complication of pregnancy in which the placenta is either partially or wholly inserted in the lower uterine wall and blocks the cervix. It is the leading cause of antepartum hemorrhage. Clients with this complication will have to have a C-section to prevent harm to the mother and fetus from bleeding. 1. Incorrect: This client's primary needs are monitoring and education. While important to educate this client to ensure the health of the mother and fetus, this does not take priority over monitoring a client that is at risk for hemorrhage. 3. Incorrect: All the products of conception are expelled with a complete abortion, and she is at low risk for hemorrhage. 4. Incorrect: This client's primary needs are monitoring and education and are not priority over a client that is at risk for hemorrhage.

The nurse is teaching a community health class for cancer prevention and screening. Which individual does the nurse recognize as having the highest risk for colon cancer? 1. Diagnosed with irritable bowel syndrome 2. Has a family history of colon polyps 3. Diagnosed with cirrhosis of the liver 4. Has a history of colon surgery

2. Correct: A family history of colon polyps and/or colon cancer is the greatest risk factor for development of colon cancer. Other factors include increasing age and a low fiber diet of processed foods. 1. Incorrect: Irritable bowel syndrome is not a risk factor for developing cancer of colon. 3. Incorrect: Cirrhosis of the liver is not a risk for cancer of colon. 4. Incorrect: Having had colon surgery does not increase a person's risk for developing cancer of colon.

The nurse assesses a client post thyroidectomy for complications by performing which assessment? 1. Accucheck 2. Chovostek's 3. Ballottement 4. Ice water colonic

2. Correct: A positive Chovostek's and Trousseau's is indicative of tetany and low calcium. This can occur when a couple of parathyroids are accidently removed when the thyroid is removed. 1. Incorrect: Accucheck assesses for blood glucose levels, which is not the problem post thyroidectomy. 3. Incorrect: This assessment technique is used in examining the abdomen when ascites is present. It is done by palpating the abdomen to detect excessive amounts of fluid (ascites). 4. Incorrect: If you have never heard of it, no one else has either. The phrase implies using ice water to cleanse the colon and this would never be a good thing, especially for someone post thyroidectomy that would be intolerant to extremes in temperature.

The nurse has been working on a health plan promoting increased physical activity for a sedentary client. Which client outcome would indicate that the interventions were successful? 1. Walks 10 minutes per day periodically. 2. Establishes a routine of 30 minutes of brisk walking three days per week. 3. Reports there is not enough time for exercise. 4. Reports walking daily for about two weeks out of the last three months.

2. Correct: A successful outcome would indicate a change in behavior. A routine of walking three times per week indicates behavioral change. Therefore, the interventions were successful. 1. Incorrect: Periodic walking does not demonstrate a positive outcome for the plan. Perhaps barriers exist that keep the client from a routine. Further assessment would be needed. 3. Incorrect: The client's report indicates perceived barriers to action. The client needs help to identify small blocks of time. 4. Incorrect: The behavior changed for a brief period; however, the new behavior did not continue. Further evaluation should be performed.

The nurse is educating a group of parents about the importance of immunizing their daughters against the human papillomavirus (HPV) in an effort to prevent the development of which cancer? 1. Breast 2. Cervical 3. Ovarian 4. Uterine

2. Correct: A vaccine that offers protection from the virus responsible for most cases of cervical cancer is the latest addition to the official childhood immunization schedule. The HPV vaccine is recommended for boys and girls at age 11 or 12 so they are protected before ever being exposed to the virus. 1. Incorrect: This vaccine does not offer protection for breast cancer. HPV does not increase the risk for breast cancer. 3. Incorrect: This vaccine does not offer protection for ovarian cancer. HPV does not increase the risk for ovarian cancer. 4. Incorrect: This vaccine does not offer protection for uterine cancer. HPV does not increase the risk for uterine cancer.

The nurse is teaching a group of high school students about car accident prevention. Who would the nurse include as the highest risk for a motor vehicle crash (MVC)? 1. Males who have just turned 19 years of age. 2. Drivers who have recently acquired a driver's license. 3. A group of students that carpool to the senior prom. 4. Female students who drive to weekly football games.

2. Correct: According to the Centers for Disease Control and Prevention(CDC), crash risk is particularly high during the first year that teenagers are eligible to drive. Though teenagers who are 19 years old, carpooling to the senior prom, and driving to weekly football games are also at risk for an MVC, they are not the highest-risk teenage group. 1. Incorrect: The risk for all teens is higher than any other driving group, however, by the age of 19, the teen has generally been driving for several years and the statistical risk of having an accident drops. 3. Incorrect: Carpooling to the senior prom does not by itself increase a teen's risk for having a wreck. Driving while under the influence of alcohol will, so a designated driver is key. 4. Incorrect: Driving to a football game weekly does not by itself increase a teen's risk for having an accident. The female gender does not increase the risk of MVC.

Following a motor vehicle accident, a client is brought to the emergency room with shallow, labored respirations. The client is intubated and placed on a ventilator. What is the nurse's priority action immediately after the intubation? 1. Suction to clear all secretions 2. Listen for bilateral breath sounds 3. Secure the endotracheal tube 4. Obtain x-ray to verify tube placement

2. Correct: All actions are important but assessment is the nurse's immediate priority. Clear and equal bilateral breath sounds along with equal chest wall movement would confirm that the endotracheal tube has been correctly inserted into trachea. 1.Incorrect: Although suctioning after intubation is an appropriate action, the nursing process requires assessment first. Clearing secretions will be more effective if the endotracheal tube is actually in the correct location in the lungs. 3.Incorrect: Prior to securing the endo tube to the client's mouth/face, it is more important to verify that tube is in the correct location within the lungs. If the tube is not placed properly, the client will not be adequately ventilator, and the tape would have to be removed for reinsertion. 4.Incorrect: Follow up chest x-rays are needed to verify that the endotracheal tube has been properly placed in the lungs. However, this action is not an immediate priority for the nurse. Assessment is the nursing priority.

A 68 year old client was admitted two days ago to a long term care facility. The client has chronic kidney disease, coronary artery disease and chronic obstructive pulmonary disease. Oxygen 2 L/min by nasal cannula is being administered. Assistance is needed with activities of daily living. The primary healthcare provider visits today and writes new prescriptions. Who is the best person for the charge nurse to delegate carrying out these prescriptions? *Exhibit (prescription)* Sputum for culture and sensitivity Incentive spirometry every 2 hours while awake Monitor SaO2 every 4 hours Levofloxacin 250 mg by mouth every 8 hours 1. Unlicensed assistive personnel (UAP) 2. LPN/LVN 3. RN 4. Charge Nurse

2. Correct: All the nursing responsibilities associated with the primary healthcare provider's prescriptions are within the scope of practice of the LPN/LVN. The UAP cannot carry out all of the prescriptions. The charge nurse should not delegate to the RN those things that the LPN can do. So the best person to delegate these responsibilities to is the LPN. 1. Incorrect: Giving medications is out of the scope of practice of the UAP, but can be carried out by the LPN. 3. Incorrect: All the nursing responsibilities associated with the primary healthcare provider's prescriptions are within the scope of practice of the LPN/LVN. Therefore, the responsibilities can be delegated to the LPN and free the RN to do other tasks that are outside the scope of practice for the LPN. 4. Incorrect: The charge nurse is responsible for all client care during the shift, so carrying out these prescriptions is not the best use of time and resources available to the charge nurse since the LPN can perform these things within the scope of practice.

The charge nurse in a psychiatric facility is assigning morning tasks to an unlicensed assistive personnel (UAP). What task should the nurse instruct the UAP to complete first? 1. Accompany client off unit to smoking area. 2. Obtain a morning weight on anorexic client. 3. Assist a client who is depressed to get out of bed. 4. Prepare the day room for group breakfast.

2. Correct: An accurate daily weight is obtained each morning at the same time, on the same scale, in the same clothing. The accuracy of this procedure is particularly critical for the anorexic client and should be performed prior to breakfast. The nurse will also remind the UAP to be particularly vigilant of the client attempting to alter the scale reading, perhaps by hiding an object in a bathrobe pocket. 1. Incorrect: Clients who smoke often request an early morning cigarette, prior to breakfast, and must be accompanied by a member of the staff during that time. Although many facilities are non-smoking, older clients who do smoke are provided with a specific location to do so, but this is not a priority at this time. 3. Incorrect: It is important to help clients who are depressed to participate in daily routines, such as eating breakfast in a group setting. Based on the degree of depression, many clients may also require assistance to even get out of bed and dress. The UAP will need to complete this task before breakfast but it is not the first priority of the morning. 4. Incorrect: It will be important to get the day room ready for group breakfast. However, preparing the dayroom will likely take quite a bit of time and there is a more important task that needs completed prior to breakfast preparations.

The nurse is caring for a hypertensive client who has been taking a loop diuretic while hospitalized. Upon discharge, the nurse must teach the client about the need for adequate electrolyte intake through foods and/or dietary supplements. Which foods should the nurse suggest to the client? 1. Cereals and breads 2. Avocados and apricots 3. Table salt and spinach 4. Blueberries and strawberries

2. Correct: Avocados, apricots, milk, fruit juices, bananas and cantaloupe are good sources of potassium. Loop diuretics deplete the electrolyte potassium. 1. Incorrect: Cereals and breads are good sources of B vitamins. Since the client is losing potassium they need foods that are high in potassium. Cereals and breads are not high in potassium. 3. Incorrect: Table salt and spinach are good sources of sodium, but the hypertensive client usually should limit intake of sodium. The client is taking a potassium depleting diuretic and needs potassium rich foods. Spinach is high in potassium but the table salt makes this option incorrect. 4. Incorrect: Blueberries and strawberries both are relatively low in potassium. Clients on loop diuretics are losing potassium and need to consume foods high in potassium.

The community health nurse is developing a presentation for adolescents on dealing with gun violence in school. What initial action should the nurse take? 1. Design a booklet for school districts on handling aggression. 2. Survey students to determine attitudes towards weapons. 3. Provide information on anger management to grade schools. 4. Investigate existing safety procedures in the schools.

2. Correct: Based on teaching/learning theory, the most important initial step is to determine if the client (in this case, a group of adolescents) is receptive to learning. Motivation is vital to successful learning. By first surveying student attitudes, the nurse gathers the data needed to prepare an age appropriate presentation that is more likely to be successful. 1. Incorrect: In this option, the nurse is implementing an action without collecting appropriate data. In addition, a booklet, while useful, is not what the nurse is to prepare. The correct issue is gun violence, not general aggression. 3. Incorrect: While it is true that anger management can influence violence, particularly at a young age, it is not the issue in this question. The idea of presenting information at the grade school level is logical but this nurse is to prepare an adolescent-based presentation. 4. Incorrect: This option does discuss collecting data; however, the information being collected does not address the topic in the question, which is gun violence. Safety procedures in schools could focus on many diverse concerns other than the issue of gun violence.

During a physical assessment of a client who was started on haloperidol 5 days ago, the nurse notes restlessness, muscle weakness, drooling, and a shuffling gait. What should be the nurse's first action? 1. Hold the next haloperidol dose. 2. Administer the prn benztropine mesylate. 3. Notify the primary healthcare provider to discontinue the haloperidol. 4. Draw a blood sample for drug level.

2. Correct: Benztropine mesylate is an anticholinergic that counteracts the extrapyramidal symptoms (EPS) seen with the use of haloperidol. 1. Incorrect: Holding a single dose of haloperidol does not correct the extrapyramidal symptoms. 3. Incorrect: The primary healthcare provider has prescribed benztropine mesylate to combat the side effects of the haloperidol. There is no need to notify the primary healthcare provider, which will delay treatment. 4. Incorrect: The client is showing extrapyramidal symptoms associated with haloperidol therapy. Benztropine mesylate is an anticholinergic agent that can be used to treat the extrapyramidal effects that may be seen as a side effect of haloperidol therapy.

A client is prescribed phenobarbital to control seizures. Which medication prescribed for the client would the nurse recognize interacts with phenobarbital? 1. Lovastatin 2. Loratadine 3. Lansoprazole 4. Lactulose

2. Correct: Both of these drugs can cause CNS depression. There is a drug to drug interaction between antiseizure medications and antihistamines. Loratadine is the only medication in the answer options that can cause CNS depression. 1. Incorrect: Lovastatin is indicated for the treatment of increased cholesterol and triglyceride levels. There is no drug to drug interaction that exists between phenobarbital and lovastatin. 3. Incorrect: Lansoprazole is a proton-pump inhibitor indicated for the treatment of stomach ulcers and GI complaints. There is no drug to drug interaction that exists between phenobarbital and lansoprazole. 4. Incorrect: Lactulose is an ammonia reducer and laxative. It is indicated for the treatment of constipation and to decrease the ammonia level in the treatment of client's with liver disease. There is no drug to drug interaction that exists between phenobarbital and lactulose.

How would a tendency toward stereotyping and countertransference affect the nurse's ability to complete a client's cultural assessment? 1. Facilitate the care planning process 2. Promote decisions based on the nurses value system 3. Utilize an open honest approach while responding to the client's concerns 4. Develop an unbiased approach to care.

2. Correct: Both stereotyping and countertransference will decrease the nurse's sensitivity to the client's needs and the culture they represent. Nurses who impose these values upon clients will make decisions based on their attitudes, values and beliefs and not those of the culturally different client. 1. Incorrect: Both stereotyping and countertransference also interfere with the treatment process you can't base your care plan on your general views toward a client's culture. Care plans, must be individualized and not based on stereotypes. 3. Incorrect: The nurse will make automatic responses based on preconceived ideas and expectations. The nurse is unable to be open and honest about client concerns. Remember, stereotyped behavior is based on the assumption that all people in a similar cultural, racial or ethical group think and act alike. 4. Incorrect: The nurse's need to maintain an unbiased care is important because the client's needs remain unmet. Value clarification by the nurse will assist in preventing stereotyping and countertransference to other clients. The nurse will never have an unbiased approach to care for clients unless the nurse understands and removes unhealthy values affecting the assessment process.

When caring for a client on extended bedrest, which intervention should the nurse implement to decrease the risk of contractures? 1. Use a large pillow to support the head and shoulders. 2. Properly reposition every 2 hours. 3. Use a knee gatch to place knees at a 30 degree angle. 4. Place a trochanter roll along the inner aspect of each thigh.

2. Correct: Properly repositioning every 2 hours is the best way to prevent contracture. 1. Incorrect: A small rather than large pillow should be used to prevent neck flexion contracture. A small pillow should be used under their head and shoulders. 3. Incorrect: Avoid the use of knee gatch and pillows under the knees for extended times to avoid knee contractures. Knee gatch can be used but the position will need to be changed every 2 hours. 4. Incorrect: A trochanter roll is used on the outside of the thigh to prevent external rotation of the hips. This technique will not prevent contractures.

Which assessment finding identified in a client diagnosed with Guillain-Barre Syndrome would indicate that the nurse needs to notify the primary healthcare provider? 1. Vital lung capacity of 900 mL. 2. Breathlessness while talking. 3. Heart rate of 98 beats per minute. 4. Respiratory rate of 24 breaths per minute.

2. Correct: Breathlessness while talking indicates respiratory fatigue. Preparation for intubation needs to be made. 1. Incorrect: If the vital lung capacity drops below 800 mL, mechanical ventilation is warranted. 3. Incorrect: Imminent signs of respiratory failure include a heart rate greater than 120 beats per minute or less than 70 beats per minute. 4. Incorrect: Imminent signs of respiratory failure include a respiratory rate greater than 30 breaths per minute. All four options are looking at the respiratory system. So you must decide which respiratory option indicates respiratory difficulty by the client. Option 3 is a normal heart rate, so it can be eliminated. Look at option 1. Vital capacity is the maximum amount of oxygen a person can expel from the lungs after a maximum inhalation. We worry about a vital lung capacity of less than 800 mL. The normal adult respiratory rate is 12-20 breaths per minute. So options 2 and 4 are not normal. But which one is most life-threatening? Breathlessness while talking. This indicates respiratory fatigue and places the client at high risk for respiratory failure and need for intubation.

The nurse is discharging a client who had a kidney transplant and the primary healthcare provider has prescribed mycophenolate. Which nursing instruction is priority regarding this medication? You answered this question Correctly 1. Take the medication with food 2. Notify primary healthcare provider at first signs of an infection 3. Nausea, vomiting, and diarrhea are common side effects 4. Use sunscreen when planning to be outdoors

2. Correct: Calling the primary healthcare provider at the first signs of an infection is priority because mycophenolate is an immunosuppressant. It diminishes the body's ability to identify and eliminate pathogens. 1. Incorrect: It is recommended the client take the medication on an empty stomach but the drug may be taken with food if the client experiences stomach upset. This is not the priority teaching point. 3. Incorrect: These side effects may be experienced but this is not the priority teaching point. 4. Incorrect: Sun exposure should be avoided and clients should be advised to use sunscreen because it can make the client more prone to sunburn but this is not the priority teaching point. The clue in this question is "priority" and "kidney transplant." Mycophenolate is an immunosuppressant prescribed to prevent organ rejection. Immunosuppressant drugs diminish the body's ability to identify and eliminate infection. Therefore, infections can be masked. Infections are the major cause of death for kidney transplant recipients.

A female client receiving chemotherapy for breast cancer reports vomiting, stomatitis, and a 10 pound weight loss over the past month. The primary healthcare provider orders an antiemetic and daily mouthwashes. When the home care nurse evaluates the client one week later, what change described by the client would best indicate improvement? 1. Eating three meals daily. 2. Weight gain of two pounds. 3. No further mouth pain. 4. Improved skin turgor.

2. Correct: Chemotherapy typically causes gastrointestinal disturbances severe enough to interfere with a client's ability to eat or absorb nutrients. A ten pound weight loss over one month is significant but expected because of the reported vomiting and stomatitis. A weight gain of two pounds in a week would be the best specific indicator of improvement. 1. Incorrect: The ability to eat three meals daily does not mean that the client is actually absorbing those nutrients successfully. This option suggests that the antiemetic is working well, but there is not enough evidence to demonstrate significant client improvement. 3. Incorrect: The client's denial of any further mouth pain signifies that the mouthwashes have decreased mouth inflammation and stomatitis. While this is a positive change in the client's condition, it is not the best evidence noted by the nurse. 4. Incorrect: Skin turgor specifies the hydration status of a client. Since this client had previously been vomiting, improved skin turgor would indicate the antiemetic is working well and the client is able to retain fluids. While this is a positive change, it is not the most significant indicator of client improvement. Option 1: Sounds good, but it's not the best answer. For a client who had been vomiting, being able to eat a meal is great. But just because this client is eating three times a day does not mean that nutrients are being absorbed, or even that the client is keeping the food down! Also, the clues in the question do not indicate how many meals a day the client was eating prior to the new medication orders! Never assume! Option 2: Fantastic! You got the right one! Remember that the client had lost 10 pounds the previous month, most likely from vomiting and a sore mouth preventing the ability to eat. Now this client has gained two pounds in one week, indicating that nutrients are being absorbed and retained by the body. You can document weight gain on a scale, and that is the BEST, most definitive proof that the client is improving. Option 3: Well, this is certainly an improvement in the client's status, but not the best one. It is true that the client had mouth pain (stomatitis) a week ago and was losing weight. Obviously, the new order for mouthwashes has been successful since there is no further report of mouth pain. Though this improvement contributes to the fact that the client can now eat (option 1) and has no pain (option 3), this is still not the change we are looking for. Think "specific proof"! Option 4: Did you notice that this response is a bit vague? "Improved" is one of those words that should lead to more questions: improved over what and how much better? What was it like before? Can you compare what it was before treatment with what is presented now? The answer to all those questions is no. Although this option shows improvement, there is no way to measure how significant that might be. Too vague.

What information would the nurse include when participating in community health training about sexually transmitted infections? 1. Clients are screened for chlamydial infection and/or gonorrhea only if the client is experiencing cervical discharge, dyspareunia, and dysuria. 2. Women with chlamydial infection or gonorrhea are likely to be asymptomatic. 3. In many instances, chlamydia infection and/or gonorrhea will go away without intervention. 4. It is only necessary for females to be treated for chlamydial infections and/or gonorrhea due to the potential damage to a female's reproductive system.

2. Correct: Chlamydial infections of the cervix and gonorrhea often produce no symptoms. 1. Incorrect: All sexually active women aged 25 and younger should be screened annually​. 3. Incorrect: If left untreated, chlamydial infections and gonorrhea can result in serious complications​. 4. Incorrect: Partners do need to be treated​. Look at each option as True or False. Option 1 is false. Nearly two-thirds of all sexually transmitted infections (STIs) occur in people younger than 25 years. Screening does not depend on client exhibiting S/S. Option 2 is true. Because many STIs have few or no initial symptoms, an infected person can transmit the infection unknowingly. Up to 90% of women infected with gonorrhea or chlamydial infections have no symptoms. Option 3 is false. Serious complications can happen when clamydial infection and/or gonorrhea are not treated/ Some STIs may be easily treated and others require more complicated treatment. Option 4 is false. Males and females must be treated. Both chlamydia and gonorrhea are reportable diseases.

A distraught client arrives at a mental health crisis center following a house fire that also took the life of a young family member. The nurse knows what action is most important when initiating crisis intervention for this client? 1. Assist the client to verbalize feelings of grief. 2. Assess client for any suicidal behaviors. 3. Admit client to general mental health unit. 4. Assign client to a grief counseli

2. Correct: Client safety is always the nurse's priority concern where no other life threatening issues exist. A distraught client in crisis from such overwhelming events does not always think or act clearly. The loss of home combined with the death of a family member places the client at potential risk for suicide. Because the client has presented to the mental health crisis center, the nurse must assume the worst and assess for unexpected responses. 1. Incorrect: While it is true that encouraging the client to verbalize feelings is therapeutic in a crisis, that is not the most important initial action by the nurse at this time. Recall the nursing process when considering an irrational action. 3. Incorrect: Arriving at a mental health crisis center does not automatically require admission to the hospital. This client is overwhelmed by circumstances which include the death of a family member; however, ideally the client may respond to counselling or medications without the need for inpatient care. 4. Incorrect: Following evaluation by a primary healthcare provider, this client will definitely receive counseling, perhaps both individualized and in a support group for those under extreme duress. However, this is not the initial concern for the nurse.

The nurse delegated feeding of a client to the unlicensed assistive personnel (UAP). Two hours after other trays were picked up from the rooms, the nurse notes that the client's untouched tray is still at the bedside. What should the nurse do first? 1. Feed the client after warming the food. 2. Speak to the UAP to determine what happened with the feeding. 3. Pick up the tray and tell the UAP that they didn't do a good job. 4. Provide a between meal supplement to the client.

2. Correct: Communication is important in delegation, as is follow-up. There may be a good reason that the tray was not served. The key word in the stem is first. The other options may be correct but are not the best first action. 1. Incorrect: The client does need to have food; however, there is another action that should be performed first. The reason for the UAP not feeding the client needs to be determined. 3. Incorrect: The nurse retains the responsibility for the delegated task. The nurse should not assume that the UAP just did not do their job, but needs to ascertain the reason for not feeding the client. 4. Incorrect: The concern here is the client being fed their meal. Speak to the UAP first and then decide if a between meal supplement is needed.

A nurse asked the charge nurse on the psychiatric unit, "Why did you ask that client to explain the meaning of 'It's raining cats and dogs?'" What is the charge nurse's best response? 1. "I was attempting to get the client to admit to being afraid of cats and dogs." 2. "I am assessing the concreteness of the client's form of thought." 3. "Phrases like this one will help the client improve their abstract thinking ability." 4. "Concrete thinking is a higher form of thinking and means that the client's form of thought is improving."

2. Correct: Concreteness, or literal interpretation of the environment, represents a regression to an earlier level of cognitive development. Abstract thinking is very difficult. The client with schizophrenia would have great difficulty describing the abstract meaning of "It's raining cats and dogs". 1. Incorrect: Asking the meaning of "It's raining cats and dogs" has nothing to do with fear of the animals. 3. Incorrect: The purpose of asking the meaning of "It's raining cats and dogs" is to assess concreteness of the client's form of thought. Explaining the meaning of this phrase will not improve abstract thinking. 4. Incorrect: Abstract thinking is a higher level of thinking. Abstract thinking is the ability to conceptualize ideas.

A client enters the post-anesthesia care unit with a three way indwelling urinary catheter that has a continuous irrigation of normal saline infusing. The urine in the indwelling urinary catheter bag, is dark red. Which action should the nurse take first? 1. Chart the drainage color and amount. 2. Increase the flow rate of the irrigation solution until the urine is a light pink. 3. Notify the primary healthcare provider of the dark red drainage. 4. Pull traction on the indwelling tubing and tape the indwelling tubing to the client's leg.

2. Correct: Continuous bladder irrigation is used following surgery to ensure that the bladder remains clear of blood clots.The nurse would need to increase the irrigation rate until the urine becomes light pink. 1. Incorrect: If the urine is not diluted, the client could form clots in the urine that could obstruct the urine flow. Charting the drainage color and amount would not address the issue. 3. Incorrect: Dark red color to the urine would warrant an increase in irrigation. There is no need to call the primary healthcare provider. If the color of urine doesn't clear or the vital sign show signs of shock (increased heart rate and decreased blood pressure) then notifying the primary healthcare provider would be needed. 4. Incorrect: This is the intervention that would be carried out if the client is hemorrhaging from the prostate. The balloon on the catheter would be used to apply pressure to the prostate and decrease bleeding. If there was more evidence of hemorrhage such as a decrease in blood pressure or increase in heart rate this type traction would be initiated. There is not enough evidence of hemorrhage at this point to initiate traction, therefore, more assessments should be performed.

A newborn is admitted to the nursery with a diagnosis of rule out cytomegalovirus (CMV). Which of the following RNs should not be assigned to this baby? 1. A nurse just back from maternity leave. 2. A nurse who is 10 weeks pregnant. 3. A nurse who is breastfeeding her 4 month old. 4. A nurse who is on hormone replacement therapy.

2. Correct: Cytomegalovirus is a viral infection that can be devastating to a fetus, especially in the first trimester. Assigning this nurse to the newborn with CMV would put her unborn baby at high risk for life-long defects and even death. 1. Incorrect: Most adults have already been exposed to the virus and are not at risk for adverse effects of the infection. Even though this nurse just had a baby, there is no risk of her transmitting this virus to her child. 3. Incorrect: Most adults have already been exposed to the virus and are not at risk for adverse effects of the infection. Even though this nurse just had a baby, there is no risk of her transmitting this virus to her child. 4. Incorrect: Most adults have already been exposed to the virus and are not at risk for adverse effects of the infection. Hormone replacement does not affect the immune system and, therefore, this nurse is not at risk for infection from CMV exposure.

The nurse is assessing a client who was admitted to the inpatient psychiatric unit five days ago for exacerbation of psychotic symptoms, as evidenced by delusions of grandeur. Which type of client remarks indicate continued delusions of grandeur? 1. Comments with fear as a theme. 2. Comments with a theme of being grand or powerful. 3. Comments related to missing body organs. 4. Comments related to being under someone else's control.

2. Correct: Delusions of grandeur include thoughts that the person has exaggerated power or importance. Clients experiencing these feeling believe they are a deity, have special powers, rare abilities or hidden talents. They often feel they should be praised and publicly recognized for these powers. 1. Incorrect: Such comments would indicate delusions of persecution. These delusions occur when a client falsely believes they are being conspired against by others, being spied on, or anything that invokes fear in the events of their daily life. 3. Incorrect: These comments indicate somatic delusions in which the client experiences a false belief that relates to body functions and/or physical appearance. 4. Incorrect: These comments indicate delusions of control or influence in which there is a false belief that an eternal being, group, or energy is capable of controlling their thoughts and influencing their behavior.

The nurse is providing care to a 5 year old client who has been experiencing moderate pain. Which intervention is appropriate for the nurse to use with this client? 1. Encourage the client to talk about the pain. 2. Provide distraction by turning on the TV. 3. Contact the primary healthcare provider for a pain medication prescription. 4. Request that the parents leave the room.

2. Correct: Distraction is a good technique to use with the toddler/preschooler. Other distractions might be to read a book, or look at pictures. Heat and cold therapies should also be considered. 1. Incorrect: The client at this age does not have the cognitive abilities to discuss pain other than to say that he/she has pain and to tell where it is. They can rate their pain at age 5-8 but describing or qualifying pain occurs at age 10 and older. 3. Incorrect: Distraction and other techniques should be used before pain medication. If there is something you can do to fix the problem, do that first. 4. Incorrect: Separation from the parents could cause more anxiety for the child. Parents should be allowed to stay with the client unless they are hindering safe care.

The son of a client diagnosed with Alzheimer's Disease who is listed as a person who has access to the client's health information asks the nurse why his father has been prescribed donepezil. What response should the nurse make? 1. "Depression is often treated with this medication." 2. "This medication is used to treat confusion." 3. "Behavioral problems are diminished when the client receives this medication." 4. "This medication will address sleep disturbances."

2. Correct: Donepezil is a cholinesterase inhibitor. It improves the function of nerve cells in the brain. It works by preventing the breakdown of acetylcholine. People with dementia usually have lower levels of this chemical, which is important for the processes of memory, thinking, and reasoning. Donepezil is used to treat mild to moderate dementia caused by Alzheimer's disease. 1. Incorrect: Common antidepressant medications used for treating depression related to Alzheimer's are the selective serotonin reuptake inhibitors (SSRIs). 3. Incorrect: Antipsychotics and Benzodiazepines are used for behavioral problems such as agitation, physical aggression, and disinhibition. 4. Incorrect: Zolpidem is the most common prescription used to help with sleep disturbance found in the client diagnosed with Alzheimer's Disease. Although drug therapy is available for Alzheimer's disease, these drugs do not cure or reverse the progression of the disease. Drugs help many people, but not for very long and not very well. The use of drugs may lead to a modest decrease in the rate of decline of cognitive function. However, the drugs have no effect on overall disease progression. One way Alzheimer's harms the brain is by decreasing levels of acetylcholine, a chemical messenger that's important for alertness, memory, thought and judgment. Cholinesterase inhibitors are a type of drug that boosts the amount of acetylcholine available to nerve cells by preventing its breakdown in the brain. Cholinesterase inhibitors can't reverse Alzheimer's disease, and they don't stop the underlying destruction of nerve cells. Because dwindling brain cells produce less acetylcholine as the disease progresses, these medications eventually lose effectiveness. Donepezil is the only Alzheimer's drug approved to treat all stages of the disease. It's taken once a day as a pill. It's usually well-tolerated, with side effects occurring in about 20 percent of people who take it.

A client received a severe burn to the right hand. When dressing the wound, it is important for the nurse to do what? 1. Apply a wet to dry dressing for debridement. 2. Wrap each digit individually to prevent webbing. 3. Open blisters to allow drainage prior to dressing. 4. Allow the client to do as much of the dressing change as possible.

2. Correct: Each finger must be wrapped individually to prevent webbing. If not done appropriately the client could develop contractures and lose functional use of the hand. 1. Incorrect: No debridement is needed if dressing changes are done as ordered. 3. Incorrect: Blisters should be left intact so as not to create an open wound and an environment for infection to easily start. 4. Incorrect: This is not appropriate at this time and is not the most important option for the nurse to do to properly care for the wound and enhance healing.

The nurse on an inpatient psychiatric unit has been assigned to care for a group of clients. Which client should receive priority during morning round assessment? 1. 40 year old woman who is being discharged today. 2. 80 year old man with suicidal thinking. 3. 45 year old man who has suicidal thinking. 4. 50 year old woman with history of acute panic attacks.

2. Correct: Elderly males have the highest risk of suicide in the US. This is the client that is priority and should be assessed first. 1. Incorrect: This client normally would require less intense attention. If this client is being discharged today then they are considered stable. 3. Incorrect: This client should be checked frequently; however, another client listed is at higher risk. This client would be assessed second but the elderly male is at a higher risk. 4. Incorrect: Panic attacks are uncomfortable and the nurse should stay with the client; however, there is no reason to think that the client is currently in distress. The client has a history of acute panic attacks. There is no indication that the client is currently experiencing a panic attack.

A client admitted to the psychiatric unit is diagnosed with depression. What is the nurse's best response? 1. I understand what you are feeling. I have been left by someone I loved before. 2. You feel upset and unhappy by the loss of your significant other? It is ok to cry. 3. Don't worry. You will feel better once we start giving you medication for depression. 4. Crying isn't going to help anything. Let's talk about your past medical history now.

2. Correct: Empathy is the ability to see beyond outward behavior and to understand the situation from the client's point of view. Therapeutic language is necessary for this client and this acknowledges the clients feelings, restates for clarification, and allows the clients expression of feelings in a trusting environment. 1. Incorrect: This shows sympathy rather than empathy. With sympathy the nurse shares what the client is feeling and experiences a need to alleviate distress. This also takes the focus away from the client, and puts the attention on the nurse, which is self-centered and nontherapeutic. 3. Incorrect: This is giving false reassurance to a client that is sharing pertinent information related to the diagnosis and treatment plan. 4. Incorrect: This ignores the client's feelings by passing judgment and then changes the subject without solving the issue at hand.

The nurse administers chemotherapeutic drugs to a client with breast cancer. Where should the nurse dispose of the medication vials? 1. In a puncture-resistant biohazard container 2. In a chemotherapy sharps container 3. In a biohazard waste container 4. In a chemical container

2. Correct: Empty vials and sharps such as needles and syringes used in delivering chemotherapy agents should be disposed of in a chemotherapy sharps container. These waste containers are designed to protect workers from injuries and are disposed of by incineration at regulated medical waste facilities. 1. Incorrect: Hazardous, drug-contaminated sharps should not be placed in red biohazard containers that are used for infectious wastes, since these are often autoclaved or microwaved. 3. Incorrect: Biohazard waste containers are not designed for sharps and can cause injuries. 4. Incorrect: Chemical containers are not designed for sharps and can cause injuries.

After obtaining vital signs, which prescribed medication should the nurse hold when caring for a client on the cardiac unit? *Exhibit* T - 98 ° (36.7°) P - 74 R - 20 BP - 88/50 1. Rosuvastatin 2. Enalapril 3. Digoxin 4. Clopidogrel

2. Correct: Enalapril is an angiotensin converting enzyme (ACE) inhibitor. An ACE inhibitor will lower the client's blood pressure. The blood pressure in the stem's exhibit is low. Lowering the client's blood pressure more could have a negative effect on the client's condition. 1. Incorrect: Rosuvastatin is a lipid lowering medication. The client's blood pressure has no bearing on whether or not to administer the medication. 3. Incorrect: Digoxin is an antiarrhythmic/inotropic agent. It will slow the heart rate and increase the force of myocardial contraction. This action could actually increase the blood pressure. 4. Incorrect: Clopidogrel is an antiplatelet agent. The client's blood pressure would not have a bearing on whether or not to administer the medication.

A client reports to the nurse, "I just do not feel well. Something is wrong." The client's vital signs are BP: 130/88, HR: 102, RR: 28. What should the nurse do next? 1. Administer PRN anxiolytic. 2. Connect to oxygen saturation monitor. 3. Reassure the client that everything is okay. 4. Instruct on relaxation technique.

2. Correct: Everything is pointing toward hypoxia. Look at HR and RR. This data is telling you that the client is restless and has tachycardia...think hypoxia FIRST when you see these 2 symptoms. 1. Incorrect: Anxiolytic medications are used to treat anxiety, however, in a client with hypoxia, this would decrease the respiratory rate so much that respiratory arrest could occur. Don't be a killer nurse. 3. Incorrect: This would be incorrect because everything is not okay. Remember, you must pick an answer that fixes the problem. This choice ignores it. 4. Incorrect: This will not fix the problem and is unsafe. The client is hypoxic, and this would make you a killer nurse.

A home health nurse is educating a female client about home care considerations for intermittent catheterization. Which statement by the client would let the nurse know that the client understands what has been taught? 1. "After insertion, I will tape the tubing to my lower abdomen." 2. "I will wash the rubber catheter thoroughly with soap and water after use." 3. "It is important that I keep the drainage bag below the level of my bladder." 4. "Catheterization should be done hourly."

2. Correct: For intermittent catheterization in the home, the client should follow clean technique. Wash rubber catheters thoroughly with soap and water after use, then dry and store in a clean place. 1. Incorrect: There is no drainage bag for intermittent catheterization. If there was an indwelling catheter, it would be secured to the woman's upper thigh. 3. Incorrect: With intermittent catheterization, there is no drainage bag. This would be an incorrect comment if made by the client. 4. Incorrect: Intermittent catheterization should be done first thing in the morning and just before going to bed at night. In most cases, self catheterization should be done every 4 to 6 hours. The client may need to self catheterize more frequently if oral intake of fluids has increased.

Question 34: A client has recently been diagnosed with rheumatoid arthritis. The nurse anticipates which class of pharmacologic agents will likely be a part of the client's treatment regimen? 1. Mitotic inhibitors 2. Systemic glucocorticoids 3. Antifungals 4. Anticoagulants

2. Correct: Glucocorticoids (steroids) are an appropriate pharmacologic treatment for rheumatoid arthritis. Other treatment options include the use of NSAIDs, biologic and nonbiologic DMARDs (methotrexate and others). Remember, all the other problems associated with the use of steroids. 1. Incorrect: Mitotic inhibitors are a class of chemotherapeutic agents and are not indicated for the treatment of rheumatoid arthritis. Medications in this class include plant alkaloids (vincristine) and taxanes (paclitaxel). 3. Incorrect: Antifungals are not indicated for the treatment of rheumatoid arthritis. Rheumatoid arthritis is an autoimmune disease, not associated with a fungal disorders. 4. Incorrect: Anticoagulants are indicated for the treatment and prevention of thrombolytic disease and are not indicated for the treatment of rheumatoid arthritis. Salicylate (aspirin), an antiplatelets, may be used as an anti-inflammatory agent.

Which meal option should the client diagnosed with gout select? 1. Tuna salad on bed of lettuce, apple slices, coffee 2. Vegetable soup, whole wheat toast, skim milk 3. Roast beef with gravy sandwich, baked chips, diet coke 4. Spinach salad with chick peas and asparagus, apple, tea

2. Correct: Gout is pain and inflammation that occurs when too much uric acid crystallizes and deposits in the joints. This is a good choose as it is low in purine and fat. Purines are broken down into uric acid. A diet rich in purines can raise uric acid levels. Meat and seafood increase the risk of gout. Dairy products may lower risk for gout. 1. Incorrect: The client should not eat tuna, which is high in purine. 3. Incorrect: Gravy is a high purine food and should be avoided. Also avoid artificial sweeteners. 4. Incorrect: Although spinach, and asparagus can be consumed in moderation, they still contain purines, so it is not as good of a choice as the vegetable soup, toast and skim milk.

A client being treated in the intensive care unit following methamphetamine intoxication states, "Snakes are crawling all over the room, get me out of here!" How does the nurse document this assessment finding? 1. Delusions 2. Hallucinations 3. Flashbacks 4. Depersonalization

2. Correct: Hallucinations are false sensory perceptions not associated with real external stimuli. When the client begins to respond to a stimuli that is not visible to the nurse, this is a hallucination. 1. Incorrect: Delusions are false personal beliefs that are inconsistent with the person's intelligence or cultural background. These beliefs are not consistent with reality. Often the client will either feel all powerful or have extreme unrealistic fears. 3. Incorrect: Flashbacks are a spontaneous recurrence of the hallucinogenic state without ingestion of the drug. These can occur months after the drug has been taken. 4. Incorrect: Depersonalization can occur, but it is the observation of oneself having an experience. The client may report feelings of being an outside observer of their own thoughts or body with a sense of loss of control. This is sometimes described as an out-of-body experience for the client.

Which assessment finding by the nurse is likely to indicate an increased level of stress in a client? 1. Weight at normal level. 2. Daily experience of headaches and other body aches. 3. Use of the problem solving method to handle daily annoyances. 4. Reports of increased creativity in the job situation.

2. Correct: Headaches and other body aches may indicate increased levels of stress. Body aches may even be an indicator of depression. Restlessness and fatigue may also indicate increased levels of stress. 1. Incorrect: Normal weight is not an indicator of increased stress. If stress levels are increased, most people experience weight loss or a gain. 3. Incorrect: This does not indicate an increased level of stress. Use of the problem solving method is an effective way to deal with stress. 4. Incorrect: This does not indicate an increased level of stress. Creativity is usually decreased when stress levels are increased.

A client with hemophilia has been scheduled for extraction of wisdom teeth. The nurse anticipates that the client will receive what priority intervention before this procedure? 1. Prophylactic antibiotics 2. A unit of cryoprecipitate 3. Packed red blood cells 4. Fresh frozen plasma

2. Correct: Hemophilia is a heredity disease characterized by a deficiency of specific clotting factors, including Factor VIII, Factor XIII, and fibrinogen. Clients with hemophilia are given cryoprecipitate prophylactically prior to invasive procedures to replace these deficient factors and prevent hemorrhaging. 1. Incorrect: The priority concern is the potential for bleeding during the procedure. Although infection is always a concern and antibiotics may be considered, it is not the main issue for this client. 3. Incorrect: Packed red blood cells (PRBC's) mean the liquid portion of the blood has been removed so only the cells are infused. PRBC 's are generally administered in the face of severe hemorrhaging or very low hemoglobin and hematocrit. Bleeding is the main concern for this client, but packed red blood cells would not be the correct intervention prior to the procedure. 4. Incorrect: Although hemophilia affects the blood's coagulation ability, fresh frozen plasma (FFP) is not the correct intervention. FFP is generally used in situations such as massive hemorrhaging, severe anemia, cardiac bypass, or DIC. Another prophylactic intervention would be considered priority for the hemophilia client.

Which prevention measure should the nurse include when instructing a client on avoidance of otitis externa? 1. Gently cleaning the ear canal with a cotton tipped applicator daily. 2. Use of astringent drops after bathing. 3. Taking preventative antibiotics prior to swimming in lakes or ponds 4. Routine use of nasal saline to clear the sinuses and eustachian tubes.

2. Correct: Prevention and avoidance measures for otitis externa include thorough ear canal drying and use of acidifying or astringent drops after swimming or bathing. 1. Incorrect: Clients should be taught to NEVER stick objects, including cotton tipped applicators, into the ear canal. This could result in rupture of the tympanic membrane. 3. Incorrect: Taking preventative antibiotics is unnecessary and increases the risk for antibiotic resistance. 4. Incorrect: Use of saline may be useful in keeping the sinuses cleared thereby reducing accumulation in the eustachian tubes. While this might be beneficial for otitis media, it would serve no benefit in otitis externa.

The nurse has just received a client from the special procedures lab for a liver biopsy. What is the position of choice for this client post procedure? 1. Fowler's 2. Right side 3. Left side 4. Prone

2. Correct: How do you stop bleeding from a puncture site? With pressure, right? Yes. So where is the liver? In the right upper abdomen under the rib cage. So position the client on the right side so that pressure is applied to the liver's puncture site. Then apply pressure with a sand bag or rolled up towel. This will help to stop bleeding. 1. Incorrect: This will not help control the bleeding. Pressure needs to be applied to the liver, so we want the liver coming forward toward the abdominal wall and pressure to be applied with a sand bag or rolled up towel. 3. Incorrect: The liver is on the right, not the left. Without the liver next to the abdominal wall, pressure cannot be exerted on the liver's puncture site. 4. Incorrect: We don't turn client onto abdomen. You will not be able to assess for bleeding with the client in this position.

The nurse is assigned to care for a client with the diagnosis of schizophrenia. The client tells the nurse, "I am having trouble tuning out the voices." What is the nurse's best response to this statement? 1. "There is nothing to help with this problem." 2. "You might hum when the voices are so troublesome." 3. "You should ask your primary healthcare provider to increase your medication." 4. "Wear earplugs to block out the voices."

2. Correct: Humming or listening to music may help to decrease the intrusive voices. This increases time spent in reality based activities and decreases preoccupation with delusional and hallucinatory experiences. 1. Incorrect: There are things that the client may do, such as humming or listening to an iPod. Telling the client that there is nothing to help them is not therapeutic. 3. Incorrect: The medication may need to be adjusted, but further assessment is needed. Remember, stay away from drugs as long as possible on the NCLEX. 4. Incorrect: Earplugs suggest blocking external stimuli; hallucinations are internal voices. Earplugs will not help internal voices and saying this could make the client think that the nurse hears the voices. Remember, The nurse is not supposed to go along with the hallucinations.

A client diagnosed with systemic lupus erythematosus (SLE) has been started on hydroxychloroquine sulfate to decrease joint pain and swelling. What statement by the client indicates to the nurse the medication teaching has been effective? 1. "I will be prone to infections while on this medication." 2. "I need to see my eye doctor at least once every year." 3. "I might develop a red rash on my nose and cheeks." 4. "I can stop this medicine after my symptoms are gone."

2. Correct: Hydroxychloroquine sulfate(Plaquenil) is in the category of DMARDs (disease modifying anti-rheumatic drug) and was originally developed to treat or prevent malaria. When taken once or twice daily, this medication reduces swelling and joint pain while also decreasing skin problems in Lupus clients. Though there are relatively few side effects, the most serious is retinal toxicity which requires treatment by an ophthalmologist. It is imperative for clients on this medication to have an eye examination every 6 to 12 months. 1. Incorrect: This medication is an antimalarial which has been shown to decrease pain from arthralgia in clients with SLE. Minimal side effects are generally limited to gastrointestinal disturbances such as nausea or diarrhea. This medication does not increase the client's risk of infection at all. 3. Incorrect: Clients with SLE frequently develop the classic red "butterfly rash" across the nose and cheeks which becomes worse when exposed to the sunlight. This symptom occurs because of the disease process and is not related to any medications the client may be taking. 4. Incorrect: There are several categories of medications used to treat SLE; however, none of them should be stopped suddenly. The disappearance of symptoms generally indicates the medication regime is working well, and the client should never suddenly discontinue any medicine unless instructed to do so. Abruptly stopping this drug increases the risk of an exacerbation of symptoms such as nephritis or vasculitis.

An elderly client is admitted to the floor with vomiting and diarrhea for three days. The client is receiving IV fluids at 200 mL/hr via pump. What would be the priority nursing action? 1. Obtaining Intake and Output 2. Frequent lung assessments 3. Vital signs every shift 4. Monitoring the IV site for infiltration

2. Correct: IV fluids at 200 mL/hr is a rapid infusion rate. The elderly adult is at risk for circulatory overload and should be closely monitored during rapid infusion rates. Lung assessments are important in detecting fluid overload. The client may experience shortness of breath and moist crackles on auscultation. 1. Incorrect: I and O are important, but less priority than lung assessment in the elderly client. 3. Incorrect: Vital signs should probably be more frequent than every shift on the elderly client with dehydration. 4. Incorrect: The IV site should be monitored for infiltration, but will not be priority over lung assessment in the elderly client.

The nurse is caring for a client taking lithium. Which comment by the client indicates lack of understanding of the therapeutic regimen? 1. "I must keep my sodium intake steady over time. " 2. "If I miss a dose of lithium, I should make it up with the next dose." 3. "I must check with my primary healthcare provider before changing my diet for weight loss." 4. "I must keep my exercise routine the same or discuss with my primary healthcare provider. "

2. Correct: If a client misses a dose of lithium, the client should take the next dose as prescribed without doubling it. If the client adds the missed dose, toxicity may occur. If sodium intake is reduced or the body is depleted of its normal sodium (due to sweating, fever, diuresis), lithium is reabsorbed by the kidneys, increasing the possibility of toxicity. 1. Incorrect: This comment indicates understanding. The client should keep sodium levels the same over time as lithium and sodium are both excreted by the kidney. 3. Incorrect: This comment indicates understanding. Food intake should remain constant. Therapeutic levels should be monitored closely while the client is losing weight. Sodium reduction can lead to lithium reabsorption in the body causing toxicity. 4. Incorrect: This comment indicates that the client does understand the treatment regimen. Any changes that would change the concentration of the drug in the bloodstream should be discussed with the primary healthcare provider. Activities that cause excess sodium loss, such as heavy exertion, exercise in hot weather, or saunas should be avoided.

What should the nurse who is educating about the most common initial visual changes associated with glaucoma inform the client? 1. Central vision is lost. 2. Progressive tunnel vision occurs. 3. Sudden flashes of light in the eyes. 4. Eye floaters begin to be noticed.

2. Correct: If glaucoma is not diagnosed and treated early, an individual starts to lose peripheral vision, or the area of vision outside the central field of sight. People who have glaucoma experience tunnel vision and cannot see objects to the side, near the head, or by their feet. Central vision can be lost later if the disease progresses. 1. Incorrect: Central vision loss is the classic visual disturbance for macular degeneration but peripheral vision is usually maintained. 3. Incorrect: Individuals experiencing retinal detachment may have sudden flashes of light in the affected eye, but this is not an initial visual change related to glaucoma. 4. Incorrect: Eye floaters are more common in eye disorders such as retinal detachment or may occur associated with the aging process.

The nurse cares for a client who is scheduled for an upper GI series. The nurse teaches the client about the test. Which statement by the client indicates an understanding of the nurse's teaching? 1. I'll have to take a strong laxative the morning of the test. 2. I'll have to drink contrast while x-rays are taken. 3. I'll have a CT scan after I'm injected with a radiopaque contrast dye. 4. I'll have an instrument passed through my mouth to my stomach.

2. Correct: In an upper GI series (sometimes called a barium swallow test), the client swallows barium contrast while x-rays are taken. 1. Incorrect: Laxatives are taken the night before a colonoscopy to ensure stool is cleared from the colon. Waiting to take the laxative the morning of the test would be ineffective and uncomfortable for the client. 3. Incorrect: Radiopaque dye injected before a CT (computed tomography) scan is not part of a GI series. This would be a totally different diagnostic test from the upper GI. 4. Incorrect: In a gastroscopy (sometimes called a gastric endoscopy), a scope is passed through the mouth to the stomach to visualize the inner lining of the upper GI tract.

Which menu selection by the client diagnosed with cholelithiasis indicates to the nurse that teaching of proper diet was understood? 1. Fried chicken, rice and gravy, broccoli and cheese, custard pie 2. Grilled pork chops in peach sauce, baked sweet potato, sherbet 3. Oven roasted bbq ribs, baked beans, tomato slices, ice cream 4. Pasta topped with boiled shrimp and butter sauce, salad, bread pudding

2. Correct: In cholelithiasis, the bile becomes super saturated with cholesterol. This leads to precipitation of cholesterol which presents as gall stones. A client with cholelithiasis should avoid foods high in fat. Foods high in fat include any fried foods, cheeses, milk, custard, cream, ice cream, pies, and cakes, red meats, baked beans. 1. Incorrect: This diet of fried food is high in cholesterol. Foods high in fat should be avoided. 3. Incorrect: This meal seems to be prepared in a healthy manner with being oven roasted; however, the ribs are high in fat (cholesterol). Ice cream is also high in fat content. 4. Incorrect: Butter sauce and bread pudding are high in fat. Broiled shrimp is a seemingly healthy choice; however, butter sauce and bread pudding are high in fat content.

A client is undergoing outpatient psychiatric treatment for somatization disorder. Prior to the beginning of group therapy, the client tells the nurse, "I keep having headaches that are killing me! This has never happened to me before." What is the nurse's best response to this client? 1. You need to sit down, because we need to start the group session now. 2. I will notify the primary healthcare provider about your headaches, after the group session. 3. I guess we can discuss your pain now. Group therapy will have to start later. 4. Your headaches are not real, so ignore them. Go on into therapy so we can start.

2. Correct: Initially, the nurse would fulfill the client's urgent dependency needs, but gradually withdraw attention to physical symptoms. Minimize time given to response to physical complaints. Gradual withdrawal of positive reinforcement will discourage repetition of maladaptive behavior. However, all new symptoms should be reported to ensure physician assessment of the complaint. 1. Incorrect: This is a nontherapeutic response. The client's feelings and concerns should not be denied. This will increase the anxiety level of the client. Do not totally ignore the client's complaint. 3. Incorrect: By postponing the group session the nurse is reinforcing the clients somatization disorder. The group session should start on time. 4. Incorrect: The pain is real to the client. This response is not therapeutic communication. The direct ignoring of the client's complaint will increase their anxiety level.

An occupational health nurse is reviewing the current medications of a client who has recently been prescribed propranolol for hypertension. Which current medication taken with propranolol by the client should be of concern to the nurse? 1. Cyanocobalamin 2. Melatonin 3. Cetirizine 4. Esomeprazole

2. Correct: Melatonin is a manmade form of the hormone that is key in regulation your body's internal clock. It is often used in treating sleep disorders. Melatonin can raise blood pressure in people who are taking beta blockers to control blood pressure. Avoid using it in conjuction with propanolol or any other beta blockers. 1. Incorrect: There are no known interactions between propranolol and Vitamin B12 (cyanocobalamin). Vitamin B12 is one of the essential vitamins and can be found in meat, fish and dairy. 3. Incorrect: There are no known interactions between propranolol and Zyrtec (cetirinzine). Cetirinzine is an antihistamine used to treat cold or allergy symptoms. This medication may cause severe drowsiness. 4. Incorrect: There are no known interactions between propranolol and Nexium (esomeprazole). Esomeprazole is a proton inhibitor that decreases stomach acid, and remember, it is not used for immediate relief of heartburn symptoms.

What is the primary electrolyte that the nurse should be aware to monitor for in a client who is receiving an insulin infusion? 1. Sodium 2. Potassium 3. Calcium 4. Phosphorus

2. Correct: Insulin causes movement of potassium into the cells, which can lead to a severe reduction in serum potassium if not regulated appropriately. A severe decrease in serum potassium could be fatal. 1. Incorrect: Although insulin has been shown to increase sodium reabsorption in the kidneys, the change is not as rapid and not as life threatening as the change in potassium can be. 3. Incorrect: A significant change in the calcium level is not anticipated with the insulin infusion. 4. Incorrect: A significant change in the phosphorous level is not anticipated with the insulin infusion.

The nurse notices that the primary healthcare provider, who has been looking at a client's morning laboratory results, walked away from the computer work station without logging out of the system, leaving the page of client medical information visible on the computer screen. What is the most appropriate action by the nurse? 1. Log the primary healthcare provider off the facility's health information system. 2. Minimize the screen so that the client information is no longer visible, and then ask the primary healthcare provider if the computer can be logged out. 3. Do not interfer since the primary healthcare provider is responsible for this information. 4. Read the health information that the primary healthcare provider left visible on the computer screen to see if the document was completed.

2. Correct: It is appropriate to minimize or hide the screen so that the information is no longer visible, then inquire whether the user will be returning to the computer work station. The nurse is held responsible to ensure that client information on a computer screen remains confidential. Legislation legally protects a client's right to privacy and confidentiality of personal health information. 1. Incorrect: Simply logging the other person off the computer system could be a correct option if that person cannot be found, but it is professional to ask if they will be returning and safeguard client's personal health records. 3. Incorrect: The nurse needs to take action to protect client's confidential health information. Professionally remind the primary healthcare provider that they did not log out and left client information visible to unauthorized persons. 4. Incorrect: It is better to ask primary healthcare provider. The nurse can not accurately determine if the primary healthcare provider is finished.

Immediately following a below-the-knee amputation (BKA), the nurse positions the client to prevent complications. What intervention related to position of the residual limb is a priority at this time? 1. Flat on the bed 2. Elevate foot of the bed 3. In a position of comfort 4. In a dependent position

2. Correct: It is normal to experience post-operative swelling after a BKA. Immediately after surgery, the foot of the bed should be elevated to reduce swelling. An ACE compression bandage will be used to reduce swelling and prevent hemorrhage. The other positions would not be as appropriate since swelling is an issue after a below-the-knee amputation. 1. Incorrect: Flat on the bed will not relieve swelling. Post-operatively for a BKA, hemorrhage and swelling are the biggest concerns immediately following surgery. 3. Incorrect: Position of comfort may increase swelling. Immediately following a BKA, elevating the foot of the bed and the ACE compression wrap are used to present hemorrhage and swelling. Positioning for comfort is not appropriate. 4. Incorrect: Placing in a dependent position will increase swelling. Swelling post-operative is a normal occurrence and elevating the foot of the bed and the use of an ACE wrap will help prevent swelling.

How should a nurse prepare to administer a Measles, Mumps, Rubella (MMR) vaccination to a 6 year old child? 1. 3 mL syringe with 23 gauge, 1" needle for IM injection 2. Use a 25 gauge, ¾" needle for subcutaneous (Sub-Q) injection. 3. Prime intranasal spray for administration. 4. Tuberculin (TB) syringe with 28 gauge, 3/8" needle for intradermal injection.

2. Correct: MMR is given Sub-Q. Subcutaneous injections are administered in the fat layer, underneath the skin. When administering SQ injections use a 23-25 gauge needle, needle length for infants (1- 12 months) is 5/8", children 12 months and older 5/8" - ¾". 1. Incorrect: MMR is given Sub-Q. An intramuscular injection is used to deliver medication deep into the muscles. 3. Incorrect: MMR is given Sub-Q. The intranasal spray is given by nasal delivery route. 4. Incorrect: MMR is given Sub-Q. Intradermal injections are given into the dermis, just below the epidermis and commonly used for tuberculin and allergy tests.

A postpartum client is receiving methylergonovine maleate 0.2 mg by mouth three times a day. What is most important for the nurse to monitor with this client? 1. Dizziness 2. Hypertension 3. Nausea and vomiting 4. Headache

2. Correct: Methylergonovine affects smooth muscle of a woman's uterus. It improves muscle tone and strength. It is used after childbirth to help deliver the placenta. Cardiovascular side effects have included palpitations, hypertension, hypotension, acute myocardial infarction, transient chest pains, arterial spasm (coronary and peripheral), bradycardia, and tachycardia. These need to be reported to the primary healthcare provider. 1. Incorrect: Dizziness is a rare side effect and not as life threatening as hypertension. 3. Incorrect: Nausea and vomiting are minor signs and symptoms and not as life threatening as hypertension. 4. Incorrect: Headache is a minor symptom and not as life threatening as hypertension.

The emergency department nurse is assigned to care for four pediatric clients with varying symptoms. Which client should the nurse examine first? 1. 12 year old reporting a severe headache 2. 6 month old with respiratory rate of 68/min while sleeping 3. 2 year old with a broken arm who is crying and appears in pain 4. 8 year old with cellulitis of the left leg and an elevated body temperature

2. Correct: Normal respiratory rate for a 6 month old is 30-50 breaths a minute. A 6 month old who is sleeping is not exerting themselves, and the respiratory rate should be within normal limits. A rate of 68 should alert the nurse to a problem that needs to be addressed. 1. Incorrect: This client does not prioritize higher than one experiencing a respiratory problem. 3. Incorrect: This client's findings are expected behavioral responses to a broken bone. Remember that pain does not prioritize higher than a respiratory problem. Pain never killed anyone! 4. Incorrect: Elevated body temperature is an expected physiological response to cellulitis. This problem does not prioritize higher than a respiratory problem.

A client diagnosed with advanced cirrhosis is admitted with dehydration and elevated ammonia levels. While discussing dietary issues, the client requests larger portions of meat with meals. Which response by the nurse provides the most accurate information to the client? 1. I will ask the dietician to add more meat with dinner. 2. Protein must be limited because of elevated ammonia levels. 3. You need to drink more fluids because of your dehydration. 4. We can ask for between meal snacks with more carbohydrates.

2. Correct: Normally, protein is broken down into ammonia, which the liver converts into urea, and the kidneys then easily excrete. However, in a diseased liver, this conversion is not possible, and ammonia continues to build up in the body, ultimately affecting the brain. The nurse would be aware that additional protein would be harmful for this client. 1. Incorrect: Increasing meat at mealtimes would be detrimental to the client's health. When protein is taken into the body, a healthy liver will convert this into urea that is then excreted by the kidneys. However, this client's impaired liver is not able to make that conversion; therefore, the ammonia levels would continue to increase. The nurse can discuss with the client other foods that might safely be added to meals. 3. Incorrect: While it is true this client is dehydrated, the issue is that the client wants to increase the amount of meat at mealtimes. This response does not address the client's request nor does it provide any teaching that would help the client once discharged. 4. Incorrect: Although this response indicates that the nurse is focusing on the client's issue with food, this reply does not address the request for more meat with meals. This would be the appropriate opportunity to educate the client on the need to limit daily protein in the diet.

The emergency department called the labor and delivery unit to give report on a 24 year old primigravida at term, having contractions every 5-8 minutes. The unit is very busy, and all the RNs are with other clients. What action by the charge nurse would be most appropriate? You answered this question 1. Request that the emergency department hold the client until one of the RNs is available to do the initial assessment. 2. Instruct the LPN/VN to obtain initial vital signs and connect the client to a fetal monitor, then report this data to the charge nurse. 3. Assign an LPN/VN to complete the nursing history and an initial obstetric assessment on this client. 4. Inform one of the RNs that a client is coming from the ED and that a nursing history should be completed as soon as possible.

2. Correct: Obtaining vital signs and placing clients on electronic fetal monitors are within the scope of practice of LPN/VN. 1. Incorrect: The ED is not staffed to care for a client in labor. The client should be transferred to the labor and delivery unit. The change nurse would then make the appropriate nurse assignment. 3. Incorrect: LPN/VNs are not qualified to perform the initial assessments. 4. Incorrect: At least, baseline data should be obtained on this client (vital signs, fetal heart and contraction patterns). Someone must assume care of the client and the LPN/VN can obtain the vital signs and connect the client to the fetal monitor.

The nursing staff have not been able to control the outbursts of a violent adult client. The primary healthcare provider prescribes physical restraints to be applied for the next 8 hours. What is the nurse's best action? 1. Apply the restraints for the 8 hours, with a trial release every 2 hours. 2. Explain to the primary healthcare provider that the prescription will have to be reissued in 4 hours. 3. Refuse to place the client in restraints unless the primary healthcare provider gets a permit signed from the family. 4. Apply the restraints, and observe the client hourly.

2. Correct: Orders for restraints or seclusion must be reissued by a primary healthcare provider every 4 hours for adults age 18 and older, every 2 hours for children and adolescents ages 9-17, and every hour for children less than 9 years. 1. Incorrect: Orders for restraints or seclusion must be reissued by a primary healthcare provider every 4 hours for adults age 18 and older, every 2 hours for children and adolescents ages 9-17, and every hour for children less than 9 years. 3. Incorrect: A permit is not needed if the client is a risk to self or others. The primary healthcare provider must write a prescription for restraints. 4. Incorrect: Clients in restraints or seclusion must be observed and assessed every 10-15 minutes with regard to circulation, respiration, nutrition, hydration, and elimination.

The primary healthcare provider has prescribed hydromorphone 2 mg intravenously (IV) every 4 hours as needed for pain. When should the nurse plan to administer the medication to the client? 1. Only when requested. 2. Prior to onset of intense pain. 3. With reports of acute pain lasting for at least one hour. 4. Continuously every 4 hours to keep the client pain free.

2. Correct: Pain is best managed before acute pain has developed. If the client waits until the pain is intense, the pain medication may not work as effectively or not at all. 1. Incorrect: Clients sometimes need pharmacologic treatment for pain even if not requested. Nurses should monitor the client for physical signs of pain. Vital sign changes and facial grimacing may be signs of pain. The word "only" is too limiting. 3. Incorrect: Clients should be treated for pain before acute pain develops when possible. The client should be educated to report pain prior to experiencing it for at least one hour. 4. Incorrect: The order is as needed, not continuously. Also, the goal of being pain free may be unrealistic. The nurse wants to keep the client's pain at a tolerable level. Always measure pain on a pain scale such as 0-10.

A client is admitted to the LDR from the emergency department at 34 weeks gestation with profuse, painless, bright red vaginal bleeding. The priority action by the nurse is to prepare for which procedure? 1. Sterile vaginal exam 2. Ultrasound exam 3. Amniocentesis 4. Contraction stress test

2. Correct: Painless, bright red vaginal bleeding is a sign of a placenta previa. Ultrasound can confirm this diagnosis with minimal risk to the mother and her fetus. This is the safest action for this client and best for fixing the problem. 1. Incorrect: If the placenta is over the cervix, a finger can go right through the placenta and cause hemorrhage and fetal death so vaginal exams would be absolutely contraindicated. 3. Incorrect: Amniocentesis is done for genetic analysis or to determine fetal lung maturity when delivery is likely. It is preferable to delay delivery until the fetus is term. It would not be safe to puncture the abdomen of a client that is already hemorrhaging. 4. Incorrect: Contractions can cause further detachment of the placenta from the cervix, which would also cause hemorrhage.

A nurse notes redness, warmth, and pain at a client's intravenous (IV) insertion site. What does the nurse suspect? 1. Colonization 2. Phlebitis 3. Infectious disease 4. Bacteremia

2. Correct: Phlebitis refers to inflammation of a vein and it can be caused by any insult to the blood vessel wall, impaired venous flow, or coagulation abnormality.Clinical evidence includes redness, heat and pain. These signs and symptoms show that the client is experiencing a localized inflammation such as phebitis. 1. Incorrect: Colonization is used to describe microorganisms present without host interference or interaction. There is an absence of tissue invasion or damage. 3. Incorrect: Infectious disease is the state in which the infected host displays a decline in wellness due to the infection. Clinical signs and symptoms may or may not be present. 4. Incorrect: Bacteremia is determined by presence of bacteria in the bloodstream. Bacteremia can lead to sepsis and signs and symptoms such as fever, hypothermia, tachycardia, tachypnea and inadequate blood flow to internal organs. So that leaves option 2. Phlebitis refers to inflammation of a vein and it can be caused by any insult to the blood vessel wall, impaired venous flow, or coagulation abnormality.Clinical evidence includes redness, heat and pain. These signs and symptoms show that the client is experiencing a localized inflammation such as phebitis. Look at option 1. Colonization is used to describe microorganisms present without host interference or interaction. There is an absence of tissue invasion or damage. Is there an absence of tissue damage in this stem? No, we see that there is redness, warmth, and pain at the IV insertion site. So option 1 is false. Option 3 is infectious disease. Infectious disease is the state in which the infected host displays a decline in wellness due to the infection. Clinical signs and symptoms may or may not be present. Is there any indication in the stem that the client is not well due to the signs and symptoms provided? No. Option 4 is bacteremia. Bacteremia is determined by presence of bacteria in the bloodstream. Bacteremia can lead to sepsis and signs and symptoms such as fever, hypothermia, tachycardia, tachypnea and inadequate blood flow to internal organs. There are no s/s identified in the stem that would indicate bacteremia.

The nurse is teaching a client about the use of a cane. Which is the correct cane technique? 1. Place the cane on weaker side of the body to support the weaker leg. Using the cane for support, the client should step forward with strong leg, and then move the weaker leg and cane forward to the strong leg. 2. Place the cane on stronger side of body. Place cane forward 6 to 10 inches while client stands with body weight divided between both legs. The weaker leg is advanced to cane, with body weight divided between strong leg and the cane. 3. Place cane on weaker side of body. The cane is placed forward 6 to 10 inches while the client stands with body weight divided between both legs. The weaker leg is advanced to the cane, with the body weight divided between the strong leg and the cane. 4. Place cane on stronger side of body to help support weaker leg. Using cane for support, step forward with the strong leg and then move the weaker leg and the cane forward to the strong leg.

2. Correct: Place the cane on the stronger side of the body. The cane is placed forward 6 to 10 inches while the client advances the weak leg at the same time. The body weight is divided between the strong leg and the cane. 1. Incorrect: The cane should be on the stronger side of the body to create a wider base for balance as the client advances the strong leg and must use the weaker leg for support with the cane. If the cane is placed on the weaker side of the body, this would create a narrower base for support and balance and increase the risk of falling. 3. Incorrect: The cane should be on the stronger side of the body to create a wider base for balance as the client advances the strong leg and must use the weaker leg for support with the cane. If the cane is placed on the weaker side of the body, this would create a narrower base for support and balance and increase the risk of falling. 4. Incorrect: The cane should be on the stronger side of the body to create a wider base for balance as the client advances the strong leg and must use the weaker leg for support with the cane. If the cane is placed on the weaker side of the body, this would create a narrower base for support and balance and increase the risk of falling.

The nurse observes an unlicensed assistive personnel (UAP) performing AM care for a client with a plaster leg cast applied 12 hours ago. Which action by the UAP should the nurse intervene? 1. Lifting the affected leg with the palms of the hands 2. Covering the affected leg with a blanket to avoid chills 3. Placing plastic over the entire cast prior to bathing 4. Elevating the casted leg on two pillows

2. Correct: Plaster cast take 24-48 hours to completely dry. During this time they release heat. The new cast should not be covered so that heat from the cast can evaporate. If the heat is not allowed to dissipate, drying will take longer. 1. Incorrect: Plaster cast take 24-48 hours to completely dry. During this time the cast should be handled carefully as to avoid indentations. Handling the cast carefully with the palms and not the fingertips will prevent indentations. Indentations in the cast could cause skin breakdown inside the cast. 3. Incorrect: Yes, will keep cast from getting wet. Plaster casts should never get wet. The plaster cast does not hold up well in water. Wet casts can also irritate the skin underneath. 4. Incorrect: Yes, may elevate the leg. Fractures are prone to swelling. Elevating fractures while casted is a common occurrence. Elevation prevents swelling.

A client with a diagnosis of endocarditis and a new peripherally inserted cential catheter (PICC) line has been discharged home to receive daily intravenous antibiotics for six more weeks. The home health nurse is making an assessment visit today. What instruction by the nurse is most important initially? 1. Take antibiotics before dental procedures. 2. Brush and floss teeth at least twice daily. 3. Report any flu like symptoms immediately. 4. Include rest periods throughout the day.

2. Correct: Poor dental hygiene is one of the chief causes of endocarditis in adults, leading to growth of vegetation on heart valves, emboli, strokes, or even death. Instructions on proper oral care is considered primary or preventative teaching and encourages the client to take an active role in personal health care. Decreasing mouth bacteria or disease will decrease the potential for a reoccurrence of endocarditis. 1. Incorrect: Although primary healthcare providers may order antibiotics prior to a dental visit, it depends on what procedure the dentist is going to perform. Invasive mouth procedures where bleeding is likely generally require pre-visit antibiotics. However, this is not the most important information by the nurse initially. 3. Incorrect: Flu like symptoms are an indication of a possible exacerbation or reoccurrence of endocarditis. The client would be instructed to report such signs as fever, chills, malaise, or night sweats immediately to the primary healthcare provider. While it is important for the client to understand what to report, preventative measures are more important at this time. 4. Incorrect: Infection within the heart is very serious and, despite aggressive treatment, may have lasting effects on the client's cardiovascular system. Decreasing the workload of the heart during treatment and recovery time would certainly assist with the healing process. However, the need for frequent rest periods throughout the day is determined by a variety of factors, such as the client's age and morbidity factors, general health, amount of damage to the heart, and response to antibiotics. Rest is not the most important instruction the nurse must present initially.

A client has been admitted to the labor and delivery unit with a diagnosis of preeclampsia. During afternoon rounds, which assessment finding by the nurse should be reported to the primary healthcare provider immediately? 1. Deep tendon reflexes of plus three. 2. Urine output of 80 mL over four hours. 3. Respiratory rate of 24 breaths/minute. 4. Severe headache with blurred vision.

2. Correct: Preeclampsia is a condition in which the client's blood pressure is consistently elevated, with a systolic greater than 140 mm Hg and a diastolic above 90 mm Hg. The greatest main concern is decreased perfusion to the placenta, endangering mother and fetus, potentially accompanied by seizures, kidney or liver failure. This client has had only 80 mL of urine in four hours, indicating an output less than the minimum required of 30 mL per hour. This indication of possible kidney failure should be reported to the primary healthcare provider immediately. 1. Incorrect: Deep tendon reflexes (DTR'S) range from 0 to +5 and are used to assess the neurologic integrity of the body. Normal reflexes for the body range around +2 but become elevated in preeclampsia. The possibility of seizures increases as DTR's increase over the normal range. This symptom is serious but expected in a client with preeclampsia. The nurse should continue monitoring this. 3.Incorrect: As blood pressure increases in the preeclampsic client, both respirations and heart rate would also begin to elevate. The client may display excessive swelling of hands and feet, occasionally accompanied by facial swelling. Although a respiratory rate of 24 is a bit elevated, it is nothing the nurse needs to report immediately. 4. Incorrect: The combination of increased blood pressure and swelling in preeclampsia frequently results in severe headaches and blurred vision. If the blood pressure reaches life-threatening levels, clients have been known to develop blindness because of retinal response to the decreased body perfusion. Although headache and blurred vision are serious symptoms, this is not completely unexpected and therefore does not need to be reported to the primary healthcare provider immediately.

The charge nurse is observing a nurse perform a dressing change on a client with a Stage III pressure ulcer. What observation by the charge nurse would indicate a need for further teaching? 1. Irrigates the pressure ulcer with 50 mL normal saline. 2. Irrigates the pressure ulcer with half-strength hydrogen peroxide. 3. Packs the wound with sterile gauze soaked in normal saline. 4. Applies a hydrocolloid dressing over the wound after cleansing.

2. Correct: Pressure ulcers should not be cleaned with substances that are cytotoxic such as hydrogen peroxide, betadine or Dakin's solution. Cytotoxic means toxic to cells, or cell-killing. Any agent or process that kills cells. These solutions can kill or damage cells, especially fibroblasts. Dakin's solution is a type of hydrochlorite solution. It is made from bleach that has been diluted and treated to decrease irritation. Chlorine is the active ingredient in Dakin's solution. 1. Incorrect: Normal saline is an appropriate solution and is used to clean pressure ulcers. This does not kill or damage cells. 3. Incorrect: Normal saline is an appropriate solution and pressure ulcers may be packed with sterile gauze. This helps remove necrotic tissue. 4. Incorrect: The wound should be covered with an appropriate dressing after cleaning. Hydrocolloid dressings support healing in clean granulating wounds and autolytically debride necrotic wounds. Hydrocolloid dressings are occlusive, so they provide a moist healing environment, autolytic debridement, and insulation. First remember, do no harm. Which option would cause harm to this client? Will normal saline cause harm or further injury to the client? No. So if the nurse irrigated the ulcer with normal saline, it would not harm the client. So you can eliminate option 1. Look at option 3. This option uses normal saline as well, but adds a sterile gauze to the wound. Will this harm the client? No. What about option 4? No, this type dressing will not harm the client or cause further damage to tissue. But option 2 can damage tissue. Hydrogen peroxide kills cells and healthy tissue. So the option that is false is option 2.

A clinic nurse completed teaching the parents of a 9 month old baby how to prevent otitis media infections in their baby. Which statement by the parents indicates to the nurse that further teaching is necessary? 1. "Our baby should sit up for feedings." 2. "It is fine to prop up a juice bottle for our baby to drink at night." 3. "Since our baby has ear tubes, ear plugs should be worn when swimming." 4. "We need to keep our baby away from people who are smoking."

2. Correct: Propping up a bottle can contribute to otitis media and dental caries with a propped bottle, the liquid pools in the back of the mouth and can back up through the eustachian tube. Bacteria may then enter through the tube and cause an ear infection. 1. Incorrect: This is a correct response. Reflux of milk up the eustachian tubes is less likely in the vertical or semi-vertical position during feedings. 3. Incorrect: This is a correct statement. Parents should keep bath water and shampoo water out of the ear, if possible. Swimming without earplugs poses a slightly increased risk of infection. 4. Incorrect: This is a correct statement. Second hand smoking increases the risk of persistent middle ear effusion by enhancing attachment of the pathogens that cause otitis to the middle ear space, prolonging the inflammatory response and impeding drainage through the eustachian tube.

Which nursing intervention represents secondary prevention level? 1. Teaching the effects of alcohol to elementary school children. 2. Providing care for abused women in a shelter. 3. Leading a group of adolescents in drug rehabilitation. 4. Ensuring medication compliance in a client with schizophrenia.

2. Correct: Providing care for clients in a shelter for abused women indicates that a problem has been identified and is being monitored to prevent the problem from getting worse. The focus of secondary prevention is early detection, use of referral services, and rapid initiation of treatment to stop the progress of the disease. 1. Incorrect: This is primary prevention which is aimed at reducing the incidence of mental or physical disorders within the population. 3. Incorrect: This is tertiary prevention which is designed to restore self-suffering and to limit complications and disabilities associated with a disease state, such as substance abuse or mental illness. 4. Incorrect: This is tertiary prevention which is designed to restore self-suffering and to limit complications and disabilities associated with a disease state, such as substance abuse or mental illness.

A traumatized soldier goes to the infirmary after being told he almost died in a gun battle. He tells the nurse, "I do not remember any of the details of this event. What is wrong with me?" What is the nurse's best response? 1. "I understand you are upset, but you will have to go back to your unit sooner or later." 2. "You are repressing this event because it was frightening and painful for you." 3. "In my professional opinion, you are trying to undo what happened in the battle." 4. "You are splitting from the bad you, so that the good you survives."

2. Correct: Repression is the unconscious blocking from awareness an event or memory of something that is threatening or painful. It is the mind's way of forgetting or experiencing temporary amnesia until it can cope with an overwhelming circumstance. The nurse's response is concise and honest for a client that needs a trusting therapeutic relationship after a traumatic event. 1. Incorrect: The nurse is being aggressive and judgmental which is inappropriate and not therapeutic for the client. 3. Incorrect: Undoing is canceling out a behavior or trying to make amends. This is not a correct assessment of what the client has reported to the nurse. 4. Incorrect: Splitting occurs when a person cannot stand the thought that someone might have both good and bad aspects, so they polarize their view of that person as someone who is "all good" or "all bad.

At a monthly staff meeting in a long-term care facility, the charge nurse requests staff input to create new activities for the clients. An RN has been assigned to gather information for staff consideration. What method would provide the RN with the best data for this project? 1. Ask clients' families which activities they would like to have available. 2. Research professional articles for guidelines to activities in long-term care. 3. Have clients peruse a variety of games and select what interests them. 4. Contact other facilities to inquire what types of programs they provide.

2. Correct: Research based criteria generally have a high rate of success because the testing has been completed under controlled circumstances and are practice based. 1. Incorrect: Although it would be acceptable to speak with the clients' families, this would not provide the most complete data for the project. Families would not likely understand pertinent considerations such as cost of supplies, number of staff required to assist, or clients' ability to participate. 3. Incorrect: While encouraging client input does allow for some independence, multiple choices can be overwhelming for elderly clients. There would also be an unnecessary expense in purchasing and providing multiple choices for the clients. 4. Incorrect: Most facilities individualize activities based on clientele, funding, and even location. Activities that work in one long term care facilities may not be appropriate for another facility.

The nurse is working on health promotion plans for a small group of school-aged children who are at risk for obesity. Which baseline data would support the risk for obesity? 1. Spends one hour playing sports or swimming daily. 2. Spends at least two hours watching TV after dinner each day. 3. Assists mom in preparing low carb snacks for the family. 4. Participates in the marching band at school.

2. Correct: Sedentary activities, such as watching television, playing video games and using a computer to surf the internet or engage with friends can also contribute to obesity and cardiovascular health problems in later life. 1. Incorrect: The more active the child is, the less likely he is to be overweight. Activity for at least one or more hours per day should be encouraged. 3. Incorrect: Children who are exposed to healthy snacks are less likely to be overweight and are more likely to choose healthy snacks. 4. Incorrect: The marching band is an excellent source of exercise for the child. This information does not support the risk for obesity.

When providing care to a client diagnosed with pheochromocytoma, which actions could the nurse safely delegate to the unlicensed nursing personnel (UAP)? 1. Explain the purpose of the vanillylmandelic acid test. 2. Remove caffeinated beverages from the client's meal tray. 3. Remind client not to smoke. 4. Tell the client to limit activity. 5. Monitor hydration status.

2., & 3. Correct: The UAP can follow directions about removing items from a client's meal tray. The UAP can provide simple instruction reminders after the nurse has provided teaching. 1. Incorrect: The UAP cannot teach. 4. Incorrect: This is teaching and cannot be done by the UAP. Further explanation by the nurse would need to be included as well. 5. Incorrect: The UAP cannot evaluate, which would include monitoring hydration status. The UAP could collect data but cannot monitor or evaluate.

The client arrives in the emergency department with crushing substernal chest pain radiating down the left arm. Which measure should the nurse initiate first? 1. Attach to a cardiac monitor 2. Administer oxygen at 2 L/nasal cannula 3. Start an intravenous (IV) line of D5W to keep open 4. Draw blood for troponin level

2. Correct: So what should the nurse be worried about? That the client is having an MI? Yes. A crushing substernal chest pain radiating down the left arm is classic for an MI. So what option can help the client? Oxygen administration to get more oxygen to the heart muscle. 1. Incorrect: Looking and watching is what you are doing if you select this option. Will attaching the client to the cardiac monitor first help the client? No. Give the client oxygen first. 3. Incorrect: Getting an IV line is good so that cardiac medications can be given, but help the client first by starting the O2. If you wait to provide oxygen until after starting the IV, the client may be waiting a while and heart muscle will be dying. 4. Incorrect: Yes, the lab will be there shortly, but get started with O2 while you wait.

Which comment made by a new nurse regarding sodium polystyrene sulfonate indicates to the charge nurse that the new nurse understands the effects of this medication? 1. "Sodium is exchanged for potassium in the blood." 2. "Fluids will need to be encouraged after administration." 3. "This medication will increase potassium and decrease sodium." 4. "Sodium polystyrene sulfate is only given as an enema."

2. Correct: Sodium polysterene sulfonate (kayexalate) is used to treat hyperkalemia, and it works by helping your body get rid of the extra potassium by exchanging sodium ions for potassium ions in the intestines. Sodium level increases after administration and this increase causes some dehydration. Pushing fluids will offset the dehydration. 1. Incorrect: This is an incorrect statement by the new nurse. Sodium is exchanged for potassium in the GI tract, and the majority of the exchange occurs in the large intestine where potassium ions are excreted in larger amounts. 3. Incorrect: Potassium will decrease and sodium will increase. Remember, this medication is used for hyperkalemia. 4. Incorrect: Sodium polystyrene sulfonate can be given as a liquid by mouth, through a stomach feeding tube, or as a rectal enema.

A nurse is working with community officials to decrease the incidence of violence in the community. Which primary preventive measures might the nurse suggest? 1. Provide a safe haven for victims of violence. 2. Provide educational programs about types of violence. 3. Form a neighborhood watch program. 4. Develop a media campaign identifying risk factors of potential abuse. 5. Provide for the immediate removal of a victim of violence from the home.

2., 3. & 4. Correct: These are all appropriate interventions for the nurse to suggest to the community. The key is prevention. The nurse is teaching ways to prevent violence before it occurs. Primary prevention is true prevention. Primary prevention aims to prevent disease or injury before it ever occurs. This is done by preventing exposures to hazards that cause disease or injury, altering unhealthy or unsafe behaviors that can lead to disease or injury, and increasing resistance to disease or injury should exposure occur. Examples include: legislation and enforcement to ban or control the use of hazardous products (e.g. asbestos) or to mandate safe and healthy practices (e.g. use of seatbelts and bike helmets); education about healthy and safe habits (e.g. eating well, exercising regularly, not smoking); immunization against infectious diseases. 1. Incorrect: This is a true statement but is not a preventive measure. This does not prevent violence from occurring; it is an intervention to decrease the chance of future violence making it tertiary prevention. Tertiary prevention aims to soften the impact of an ongoing illness or injury that has lasting effects. This is done by helping people manage long-term, often-complex health problems and injuries (e.g. chronic diseases, permanent impairments) in order to improve as much as possible their ability to function, their quality of life and their life expectancy. Examples include: cardiac or stroke rehabilitation programs, chronic disease management programs (e.g. for diabetes, arthritis, depression, etc.); support groups that allow members to share strategies for living well; vocational rehabilitation programs to retrain workers for new jobs when they have recovered as much as possible.​ 5. Incorrect: This is not a primary preventive measure but a secondary preventive measure. Removing the victim is not preventing primary violence but additional violence. Secondary prevention aims to reduce the impact of a disease or injury that has already occurred. This is done by detecting and treating disease or injury as soon as possible to halt or slow its progress, encouraging personal strategies to prevent re-injury or recurrence, and implementing programs to return people to their original health and function to prevent long-term problems. Examples include: regular exams and screening tests to detect disease in its earliest stages (e.g. mammograms to detect breast cancer); daily, low-dose aspirins and/or diet and exercise programs to prevent further heart attacks or strokes; suitably modified work so injured or ill workers can return safely to their jobs.​​

Which nursing intervention should the nurse implement when administering a medication through a nasogastric (NG) tube? 1. Place the client in a high-Fowler's position for medication administration. 2. Flush the tubing between administering medications 3. Turn the client onto their left side after medication administration. 4. Mix the medication directly into the tube feeding

2. Correct: The NG tube should be flushed with appropriate facility approved amount of fluid between medications. The amount of the flushing solution should be added to the intake amount. 1. Incorrect: Semi-Fowler's position is the position of choice for administering tube feedings. This position helps prevent aspiration and promotes digestion. The volume of fluid administered with medication administration is usually much smaller than with tube feedings, so high-Fowler's is not required. 3. Incorrect: The left side position slows gastric emptying, which could lead to aspiration. The right side is the position that best promotes gastric emptying. 4. Incorrect: Do not mix medications in the enteral feeding solution. The tube feeding rate may be prescribed at different rates or the tube feeding can be held for a designated time. The proper administration of the medication could not be determined.

A client diagnosed with bipolar mania was prescribed lithium carbonate 2000 mg daily two months ago. What is the nurse's best action? *Exhibit (lab results)* *lab results* Sodium - 143 mEq/L (143 mmol/L) Potassium - 4.5 mEq/L (4.5 mmol/L) Magnesium - 1.9 mEq/L (0.8 mmol/L) Serum Lithium - 1.8 mEq/L 1. Record the lab results in the chart and recheck in one month. 2. Inform the primary healthcare provider that the lithium level is too high. 3. Notify the primary healthcare provider because the sodium level is too high. 4. Let the primary healthcare provider know that the magnesium level is too low.

2. Correct: The appropriate serum lithium level for acute mania is 1.0 to 1.5 mEq/L. For maintenance it is 0.6 to 1.2 mEq/L. Levels exceeding 1.5 to 2.5 mEq/L begin to produce toxicity. 1. Incorrect: All lab results should be documented; however, the lithium needs to be reported so that the dose can be adjusted. 3. Incorrect: The sodium level is normal: 135-145 mEq/L (135-145 mmol/l). 4. Incorrect: The magnesium level is normal: 1.3 - 2.1 mEq/L (0.65-1.05 mmol/l).

A client diagnosed with bipolar mania was prescribed lithium carbonate 2000 mg daily two months ago. What is the nurse's best action? *Exhibit* Sodium - 143 mEq/L (143 mmol/L) Potassium - 4.5 mEq/L (4.5 mmol/L) Magnesium - 1.9 mEq/L (0.8 mmol/L) Serum Lithium - 1.8 mEq/L 1. Record the lab results in the chart and recheck in one month. 2. Inform the primary healthcare provider that the Lithium level is too high. 3. Notify the primary healthcare provider because the Sodium level is too high. 4. Let the primary healthcare provider know that the Magnesium level is too low.

2. Correct: The appropriate serum lithium level for acute mania is 1.0 to 1.5 mEq/L. For maintenance it is 0.6 to 1.2 mEq/L. Levels exceeding 1.5 to 2.5 mEq/L begin to produce toxicity. Acute lithium toxicity symptoms are dizziness, vomiting, coma, hand tremors, lack of coordination of arms and legs and uncontrollable eye movement. 1. Incorrect: All lab results should be documented, however, the lithium needs to be reported so that the dose can be adjusted. 3. Incorrect: The sodium level is normal: 135-145 mEq/L (135-145 mmol/l). 4. Incorrect: The magnesium level is normal: 1.3 - 2.1 mEq/L (0.65-1.05 mmol/l).

A new nurse has a prescription to insert a feeding tube. The new nurse has never performed the procedure, but learned how to do it while in nursing school. What would be the best action by this new nurse? 1. Ask to observe another nurse perform the procedure. 2. Look up how to perform the procedure in the policy and procedure manual. 3. Tell the charge nurse that someone else will have to insert the feeding tube. 4. Insert the feeding tube as learned in nursing school.

2. Correct: The best action for the nurse, is to look up how the procedure is done in the agency policy and procedure manual. The nurse could then discuss the procedure with a fellow nurse and ask them to observe during the procedure. 1. Incorrect: This is passive and will not give the new nurse the experience they need. The best action would be to look up how to do the procedure. Then the new nurse could discuss with another nurse, and have that nurse observe the insertion of the feeding tube by the new nurse. 3. Incorrect: This is not the best option. The new nurse needs to learn how to insert a feeding tube. This will not help the new nurse learn. Actually doing the procedure after checking the policy and procedures manual will give the new nurse the experience they need. 4. Incorrect: Nursing school knowledge is needed but also check agency policy and procedure manuals. Then the new nurse can discuss the procedure with a fellow nurse and ask the them to observe the feeding tube insertion.

The following clients arrive to the emergency department (ED) at the same time. The triage nurse gives priority to which client? 1. A client with a possible fracture of the tibia 45 minutes ago. 2. A client with left hemiparesis and aphasia beginning 1 hour ago. 3. A client smelling of alcohol and reporting of severe abdominal pain. 4. A client involved in a motor vehicle accident (MVA) with a possible fractured pelvis.

2. Correct: The client who is started experiencing hemiparesis and aphasia 1 hour ago is likely having a stroke. The window for treatment with fibrolytics is 3 hours, thus taking priority over the other clients. Time is brain! 1. Incorrect: This client has a possible fracture of the tibia. This is not a large bone, which would be at risk for hemorrhage. Splinting and ice packs could be used until after seeing the client having a stroke. 3. Incorrect: With this client, you would worry about pancreatitis. This client needs to be seen soon but not prior to the client having a stroke. 4. Incorrect: The MVA client could have bleeding from a fractured pelvis. This client is high on the admit list, but after the client having a stroke.

A nurse wants to find out a better way to perform oral care on unresponsive clients. What is the best first action for the nurse to take in order to achieve this goal? 1. Try different methods of oral care on unresponsive clients to see what works best. 2. Discuss the issue with the leader of the "best practices" committee. 3. Read all the current literature related to oral care on unresponsive clients. 4. Ask the primary healthcare provider to suggest the best oral care procedure.

2. Correct: The best first action for the nurse is to identify a problem, and follow up with the appropriate person. An experienced person who can research "best practice" regarding the issue is needed. The best practice committee works to improve clinical practice based on current research 1. Incorrect: This is doing research, which requires the research process be implemented, including appropriate approval. The best practice committee utilizes current research in their recommendations. 3. Incorrect: This will take a lot of time and is best initiated from the "best practice" committee. The nurse could definitely be part of the committee. But the evidence-based care leaders are trained to help nurses through the proper process of evidence based research. 4. Incorrect: This is a nursing responsibility and the best practice committee is the best place to begin. The primary healthcare provider may have suggestions but this is not the best first action.

Four clients arrive at the emergency department. Which client should the nurse triage as the highest priority for care? 1. Adult with severe upper gastric pain. 2. Child with stridor and excessive drooling. 3. Adult with an open fracture to the right radius. 4. Child with fever of 103ºF (39.44 °C) and blood streaked sputum.

2. Correct: The child with stridor and excessive drooling is in respiratory distress from epiglottis. Epiglottis is a potential life-threatening condition and should be seen first. This client is exhibiting signs of respiratory distress. 1. Incorrect: Pain important, but not before airway. This client is not exhibiting any life threatening symptoms. The severe pain should be assessed but not prior to an airway complication. Pain never killed anyone. 3. Incorrect: The open fracture needs to be evaluated as soon as possible due to the potential of compression syndrome and infection. The child however has an airway complication which takes priority over this client. 4. Incorrect: Fever and blood streaked sputum significant, but not before airway. Children with an axillary fever should be examined. The client has blood streaked sputum as well. But the priority client is still the child with the life threatening airway obstruction.

The telemetry unit nurse is assessing a newly admitted client following a fall at home. The client has been diagnosed with a left sided cerebrovascular accident (CVA), including aphasia, and a sprained wrist. What is the most effective method the nurse could use to assess the client's pain? 1. Monitor vital signs for elevations. 2. Observe client's non verbal behaviors. 3. Assess sleeping position client chooses. 4. Ask client to point to the pain rating scale.

2. Correct: The client has had a left sided stroke which damages the left hemisphere of the brain. Although the question does not specify whether this is receptive or expressive aphasia, the client may have great difficulty identifying the location or amount of pain. Because of the client's difficulty in communicating at this time, the nurse must rely on non-verbal cues such as facial expressions, vocalizations (moaning, crying) or client attention to the injured portion of the body (massaging or holding the painful area). 1. Incorrect: Despite the fact that vital signs often become elevated in the presence of pain, this is not a reliable indicator, particularly since the vital signs could be impacted by the recent CVA. Additionally, tolerance to pain varies, and changing vitals would not provide the most accurate data about the severity or even the location of the client's pain. 3. Incorrect: This method is the least reliable approach in evaluating client discomfort. Waiting for the client to fall sleep delays effective treatment, assuming the client is able to rest at all while experiencing pain. Also, the nurse is assuming that the client would be able to position self in a manner indicating what area is most painful. The impact of a stroke in the left brain might prevent the client from accurately locating or identifying the exact painful area. 4. Incorrect: The client has had left hemisphere damage to the brain with resulting aphasia. There is no data provided in the question regarding the category of aphasia; therefore, the nurse would be aware the client may not be able to indicate the correct location or severity of pain, even if utilizing the smiling face picture scale. This client also may not be able to understand instructions on the use of the scale, or to self evaluate the actual level of pain.

Which assigned client should the nurse see first? 1. Diagnosed with urinary tract infection 2 days ago who is to be discharged. 2. Admitted last night with a diagnosis of severe pneumonia. 3. 45 year old who had a hernia repair 24 hours ago. 4. Scheduled for an endoscopy in two hours.

2. Correct: The client with severe pneumonia is at greatest risk for respiratory difficulty and should be seen first. Clients with severe pneumonia may develop the following complications: bacteremia, septic shock, lung abscesses, pleural effusion, empyema, pleurisy, renal failure, and respiratory failure. 1. Incorrect: The client who is being discharged is considered to be stable. A client who was diagnosed with a urinary tract infection is considered to be stable. This client is not exhibiting signs of potential airway complications. 3. Incorrect: This postoperative client of 24 hours is considered stable. The client's age of 45 also does not suggest that the client was a surgical risk. 4. Incorrect: The client admitted for an endoscopy is considered to be stable at this point. There is no data listed to support the client needing to be assessed first.

The client expresses concern to the nurse about the ability to provide self-care and perform activities of daily living at discharge. Which member of the healthcare team should the nurse contact to provide information and assist the client with resources for an effective discharge plan? 1. Primary healthcare provider 2. Case manager 3. Physical therapist 4. Occupational therapist

2. Correct: The client's case manager should be contacted regarding the order for pending discharge from the healthcare facility. The case manager coordinates care and provides the client with information and resources for an individualized discharge plan. 1. Incorrect: The primary healthcare provider does not assume the case management role in the acute care facility setting, and generally does not coordinate the discharge planning process. 3. Incorrect: The physical therapist is a member of the multidisciplinary team and might help evaluate, but does not coordinate discharge planning. They are not responsible for case management and coordination of overall client care for discharge from the facility. 4. Incorrect: The occupational therapist is a member of the multidisciplinary team and might help evaluate, but does not coordinate discharge planning. They are not responsible for case management and coordination of overall client care for discharge from the facility.

The nurse is caring for a client admitted to rule out myocardial infarction. The nurse has administered sublingual nitroglycerin. What time frame should the nurse expect the earliest onset of effectiveness? 1. 15 seconds 2. 3 minutes 3. 5 minutes 4. 15 minutes

2. Correct: The onset of action for nitroglycerin sublingual is 1 to 3 minutes. So the effectiveness can be assessed 3 minutes after the drug is administered. 1. Incorrect: This time frame is too short for the onset of action of nitroglycerin given sublingual. 3. Incorrect: Sublingual doses of nitroglycerin can be repeated every 5 minutes. The drug would start to be effective before 5 minutes. 4. Incorrect: Fifteen minutes would be to long to wait to assess the effectiveness of nitroglycerin sublingual, in a client suspected of a myocardial infarction.

A client's Aunt calls the nurse's station to check on the status of her niece. What should the nurse do? 1. Tell the Aunt to call the nurse supervisor for the information. 2. Inform the Aunt that she is not in a position to "need to know". 3. Tell the Aunt that the client is being discharged. 4. Provide an update on the client's condition.

2. Correct: The client's rights to privacy are priority in this situation. The nurse should simply state that information about a client can not be given to someone who is not in the position to "need to know". The nurse should explain that this is part of the HIPAA guidelines that the nurse must adhere to. The client can authorize the release of information to certain individuals. 1. Incorrect:The aunt does not have authorization or a "need to know" about the client's condition and the nurse supervisor can not provide this information. 3. Incorrect:The staff nurse should not give any information about the client to the Aunt. This would be a breech of privacy for the client and violate HIPAA. 4.Incorrect:The staff nurse should simply state that she cannot give any information about a client to someone who is not in the position of "need to know" and has been authorized to receive this information.

A client receiving electro-convulsive therapy (ECT) tells the nurse, "I don't know if I can take another treatment." What is the nurse's best response? You answered this question 1. "Remember to focus on the fact that you will be fine after you complete all of your treatments." 2. "The therapy must be difficult for you at times. How do you feel about your progress at this point?" 3. "Hang in there. It's for your own good and times will get better." 4. "What makes you say that? You know it will make you well."

2. Correct: The correct answer allows the client to continue discussing feelings and redirects the client to thoughts of progress and effectiveness of the treatment. Acknowledge the client's feelings and then asking an open-ended question are both appropriate a therapeutic communication techniques. 1. Incorrect: This response gives false reassurance which is inappropriate and negates the therapeutic trusting relationship between the nurse and the client. 3.Incorrect: This response is a trite expression or cliché which minimizes the importance of the client's feelings. This is also a close-ended statement that does not allow for any further expression of feelings by the client. 4.Incorrect: This response is demanding an explanation for the client's thoughts, feelings, or events. It makes the client have to be defensive.

A schizophrenic client tells the nurse, "The President of the United States just told me to leave the hospital immediately because a spy is on the way to tap into the secret information in my brain." What is the nurse's best response? 1. The voice you heard is because of your illness and will go away in time. 2. I know you think the President of the United States is talking to you, but I do not see the President. We are the only ones here. 3. I find it hard to believe that you have talked to the President of the United States. This is not the White House! 4. I think the primary healthcare provider needs to increase your medication dose, since you are still hearing voices.

2. Correct: The correct answer is to present reality. When a client has a misperception of the environment, the nurse defines reality or indicates his or her perception of the situation to the client. This delusion is called "thought withdrawal". It is the belief that thoughts have been removed from one's mind by an outside agency. 1.Incorrect: This response gives reassurance. In fact, it is false reassurance because the voices may not completely go away. Remember that the client believes what they think is real. You are not going to help them by blaming it on their illness. 3.Incorrect: This response is disagreeing. Never challenge the client or belittle the client with your response. 4.Incorrect: This response gives an opinion and does not bring reality into the conversation. You need to report the hearing of voices, to the primary healthcare provider, but this is not therapeutic for the client.

An elderly client is admitted to the outpatient unit with anemia and is receiving a blood transfusion. What is the nurse's priority assessment? 1. Monitor for peripheral edema. 2. Assess breath sounds. 3. Keep bedrails up at all times. 4. Monitor hemoglobin every 6 hours.

2. Correct: The elderly client receiving a blood transfusion is at greater risk for fluid volume overload. The nurse should recognize that the very old and the very young are at increased risk for fluid volume overload which could manifest as wet breath sounds. 1. Incorrect: Monitoring for edema is valuable in assessing for fluid volume overload, but the priority is the lung assessment. 3. Incorrect: Safety is important, especially with an elderly anemic client, but monitoring for fluid volume excess is the priority. Physiologic needs prioritize higher than safety needs. 4. Incorrect: Monitoring hemoglobin is important but not necessary every 6 hours.

The nurse manager is performing a chart audit for clients who were restrained. For which client would the side rails in the up position be considered a restraint? 1. The client who requests that the rails be placed in the up position. 2. The client who is confused and wanders about the unit. 3. The client who is ambulatory and places the side rails up without staff assistance. 4. The client who asks the family to place all the rails up before leaving.

2. Correct: The intent of the side rails in the up position is to limit movement; therefore, they are considered a restraint. The nurse cannot restrain or limit a client's movement without a primary healthcare provider prescription. 1. Incorrect: The client may request that side rails be raised at any time. 3. Incorrect: The ambulatory client can put his/her own side rails up if that increases feelings of security. 4. Incorrect: The family may place the rails up at the request of the client. That action would not be considered a restraint.

The nurse is teaching a client regarding herbal therapy. What is the main goal of herbal therapy? 1. To treat a specific disease or symptom by taking prescription medications. 2. To restore balance within the body by supporting the client's self-healing ability. 3. To avoid the use of toxic chemicals within the body. 4. To incorporate Eastern healing practices into Western medicine.

2. Correct: The main goal of herbal therapy is to restore balance within the body by supporting the client's self-healing ability. When teaching clients, the main goal should always be included. 1. Incorrect: The main goal of drug therapy is the treatment of a specific disease or symptom. Herbal therapy should not treat diseases. They are for support only. 3. Incorrect: The main goal of herbal therapy is to restore balance and support healing. Many times herbal therapy is considered less toxic but the question is asking for the main goal of herbal therapy. 4. Incorrect: Not the main goal of herbal therapy. This is not the main goal of herbal therapy. The main goal is to restore balance within the body by supporting the client's self-healing ability.

A client has been trying to implement a low fat diet for prevention of heart disease and enhancement of weight loss. He further reports that his wife shows her love by preparing rich foods and pastries. Which action should the nurse make? 1. Suggest that the client prepare all meals at home. 2. Schedule a meeting with husband and wife to discuss diet and health. 3. Suggest that the client limit intake to one serving of each food at meals. 4. Ask the client to give his wife a cookbook with low fat recipes.

2. Correct: The meeting with the wife and husband together may help to gain the support of the wife. She may not realize that meal preparation is actually serving as a barrier to successful change. Also, the importance of the opinions and behaviors of the wife are important to the client as he tries to engage in long-term behavioral change. 1. Incorrect: This intervention may actually increase barriers to change because the wife's feeling and support are necessary to maintain long-term change. 3. Incorrect: While this practice may reduce the intake of fat, the issue of spousal support should be addressed. 4. Incorrect: Open discussion with the wife about the need for low-fat meals is essential.

The nurse has observed that the client on the skilled nursing unit has been consuming fewer calories over the past three days. There has been no other change in the client's condition. Which intervention is most important for the nurse to initiate? 1. Suggest that the family seek an appointment with the primary healthcare provider. 2. Ask the dietician to visit the client and discuss food preferences. 3. Note any weight loss over the next month. 4. Continue to monitor intake over the next couple of weeks

2. Correct: The nurse is using the expertise of other team members by requesting that the dietician visit the client. This is the most important measures to address the client's nutritional needs. The problem may be that the client simply does not like the foods that have been served and the dietician is the best one to address these issues. 1. Incorrect: An appointment with the primary healthcare provider may not be necessary. It is the best to first utilize available team members such as the dietician. The nurse would then notify the primary healthcare provider of any pertinent findings. 3. Incorrect: To simply monitor weight loss for a month would not be an appropriate intervention. There could be significant weight loss within a month. This is much too long to wait before taking measures to ascertain the reason for the client consuming fewer calories. 4. Incorrect: The nurse should monitor intake and weight over the next couple of weeks; however, there is a more immediate action that is appropriate. The nurse takes action by asking the dietician to see the client.

A client who is of the Jehovah's Witness faith presents to the emergency department following a traffic accident. The primary healthcare provider orders a type and cross-match for this client. It is determined that the client will benefit from two units of blood. What should the nurse do? 1. Prepare the client for the administration of blood. 2. Explain to the primary healthcare provider that the client's faith prohibits blood transfusions. 3. Explain to the client that the blood transfusions are needed for return to health. 4. Try to convince the client to accept the transfusions.

2. Correct: The nurse must serve as the client's advocate. This client's religion prohibits blood transfusions. 1. Incorrect: This action is in opposition to the client's religious beliefs. 3. Incorrect: The client has made the decision based on religious beliefs. 4. Incorrect: The client has the right to make decisions autonomously and in congruence with religious beliefs.

A client diagnosed with Alzheimer's disease becomes agitated and combative when the nurse approaches to perform a shift assessment. What would be the most appropriate first action for the nurse to take? 1. Obtain assistance to restrain the client. 2. Talk quietly to the client. 3. Administer haloperidol. 4. Leave until the family can calm the client down.

2. Correct: The nurse needs to present a calm manner and speak quietly to the client. This will convey trust and decrease tension and stress in the client. 1. Incorrect: Restraints are a last resort and can make the client more agitated. 3. Incorrect: The use of positive nursing actions can reduce the use of chemical (drug therapy) restraints. 4. Incorrect: Do not pick an answer that transfers the client away from the nurse's care. The nurse needs to present a calm manner and speak quietly to the client. This will convey trust and decrease tension and stress in the client. When dealing with the difficult client, do not threaten to restrain the client or call the primary healthcare provider. Reassurance involves communicating to the client that he or she will be protected from harm, danger, and embarrassment. These behaviors are often the client's way of responding to a precipitating factor, such as pain, frustration, temperature extremes, or anxiety. When these behaviors become problematic, the nurse must plan interventions carefully. Initially assess the client's physical status. Check for changes in vital signs, urinary and bowel patterns, and pain that could account for behavioral problems. Then assess the environment to identify factors that may trigger behavior disruptions. Extremes in temperature or excessive noise may lead to behavior changes. When the client is agitated by the environment, either move the client or remove the stimulus.

The nurse is caring for a client who has the diagnosis of schizophrenia. The nurse enters the room to administer the morning dose of the prescribed antipsychotic medication. The client is drooling and has extreme muscular rigidity. After assessing the client for adequate respiratory effort, what is the nurse's priority? 1. Elevate HOB and give the medication as prescribed. 2. Hold the medication and call the primary healthcare provider. 3. Report the behaviors to the on-coming shift. 4. Hold the medication, and check the vital signs.

2. Correct: The nurse should hold the medication, and report the symptoms to the primary healthcare provider. The client may be experiencing neuroleptic malignant syndrome. 1. Incorrect: The client is experiencing symptoms of possible neuroleptic malignant syndrome. The nurse should not give another dose of the medication without consultating with the primary healthcare provider. 4. Incorrect: The client may be experiencing neuroleptic malignant syndrome. It is important to notify the primary healthcare provider immediately. 3. Incorrect: The symptoms that the client has are very serious and should be reported to the primary healthcare provider immediately.

The nurse is caring for a client with a fibula fracture. The primary healthcare provider makes rounds and writes prescriptions. What is the nurse's best action? *Exhibit* MSO4 8 mg IM now Advance diet as tolerated Hgb and Hct in AM 1. Check the prescription prior to sending it to the pharmacy. 2. Clarify the prescription with the primary healthcare provider. 3. Notify the pharmacy that the prescription is needed immediately. 4. Gather the supplies needed for an injection.

2. Correct: The nurse should notify the primary healthcare provider, because MSO4 is an unapproved abbreviation that presents safety concerns. MSO4 is the abbreviation for morphine sulfate. MgSO4​ ​is the abbreviation for magnesium sulfate. Notifying the primary healthcare provider to clarify the prescription will prevent a medication error. 1. Incorrect: The prescription should not be sent to the pharmacy until after it is clarified with the primary healthcare provider. The Institute for Safe Medication Practices (ISMP) and The Joint Commission (TJC) recommend using the complete names for morphine and magnesium to eliminate confusion. 3. Incorrect: MSO4 is not an approved abbreviation. Before notifying the pharmacy, make sure you know what the prescription is for. The complete drug name should be written out. 4. Incorrect: You might be making a medication error if you assume you know what you are giving. Always seek clarification when in doubt.

The client is undergoing progressive ambulation on the third day after a myocardial infarction. Which clinical manifestation would indicate to the nurse that the client should not be advanced to the next level? 1. Facial flushing 2. Reports shortness of breath 3. Heart rate increase of 10 beats/min. 4. Systolic blood pressure increase of 10 mm Hg

2. Correct: The onset of shortness of breath could be an indicator that the client should not advance to the next level. The client should be instructed to stop and rest if chest pain or shortness of breath occurs. While in a rehabilitation program, it is imperative to give the client very specific guidelines for physical activity so overexertion will not occur. 1. Incorrect: Facial flushing is not life-threatening. The client can advance to the next level. 3. Incorrect: An increase in heart rate of 10 beats a minute is an expected finding with physical activity. This would not prevent the client from advancing to the next level. 4. Incorrect: An increase in systolic BP is an expected finding with physical activity.

A client with diabetes has a history of ignoring the primary healthcare provider's prescription for daily medication management of the illness. The client has been working toward a health promotion goal of increased adherence to prescribed medication regimen. Which outcome suggests that the client has met the health promotion goal? 1. Client has lost five pounds. 2. Client takes medication as prescribed. 3. Client has been hospitalized twice for complications of diabetes. 4. Client walks one mile per day.

2. Correct: The outcome directly addresses medication adherence, the major focus of the health promotion plan. 1. Incorrect: This is a positive outcome; however, the focus is on medication adherence. 3. Incorrect: This outcome would indicate possible non-adherence to the medication regimen. 4. Incorrect: While this is a positive outcome for anyone's health, the focus is medication adherence.

A client scheduled for a bronchoscopy and possible lung biopsy tells the nurse, "I don't know what a bronchoscopy is." Which nursing intervention should the nurse implement? 1. Explain the bronchoscopy procedure to the client and inform the client of the risks, benefits, and treatment alternatives. 2. Immediately inform the primary healthcare provider that the client requests additional information about the bronchoscopy procedure. 3. Give the client an information pamphlet on the bronchoscopy procedure, and tell the client to sign the consent after reading the pamphlet. 4. Instruct the client to sign the informed consent form. The primary healthcare provider will answer any additional questions right before the procedure is performed.

2. Correct: The primary healthcare provider performing the procedure should explain the risks and benefits, recovery time, and reasonable alternatives, as well as the consequences of refusing treatment. 1. Incorrect: The nurse can explain the bronchoscopy procedure and expectations to the client, but the nurse is not performing the bronchoscopy. 3. Incorrect: Providing an information pamphlet to the client may be beneficial, but this should never be substituted for the primary healthcare provider's communication with the client. 4. Incorrect: Questions should be answered by the primary healthcare provider before the client signs the consent form.

A pregnant client has been receiving daily heparin injections for a history of deep vein thrombosis (DVTs) during pregnancy. Which laboratory test result should be immediately reported to the primary healthcare provider? 1. PT of 13 seconds 2. PTT of 22 seconds 3. INR of 1.0 4. Hemoglobin of 11 g/dL (6.8266 mmol/L)

2. Correct: The test that monitors the efficacy of heparin is the PTT. The normal range for a PTT is 30-40 seconds, but desired outcome of heparin therapy is PTT of 1.5-2.5 times the control without signs of hemorrhage. This client's PTT is below therapeutic range so it is not preventing DVT formation. The dose of heparin will need to be increased. 1. Incorrect. PT monitors the efficacy of warfarin, which is contraindicated in pregnancy because it crosses the placenta which means the fetus would be receiving the medication. PT is measured in seconds. Most of the time, results are given as what is called INR (international normalized ratio). If a client is not taking blood thinning medicines, such as warfarin, the normal range for PT results is 11 to 13.5 seconds. Normal value ranges may vary slightly among different laboratories. Some labs use different measurements or test different samples. 3. Incorrect: INR monitors the efficacy of warfarin, which is contraindicated in pregnancy because it crosses the placenta which means the fetus would be receiving the medication. (Normal INR of 0.8 to 1.1.) If the client is taking warfarin to prevent blood clots, the primary healthcare provider will most likely choose to keep the INR between 2.0 and 3.0. 4. Incorrect: A hemoglobin of 11 g/dL (6.8266 mmol/L)​ is adequate in pregnancy. In pregnancy, there is an increase in plasma volume of the blood in order to help supply oxygen and nutrients to mother and baby. There can be a 20% increase in the total number of red blood cells, but the amount of plasma increases even more causing dilution of those red cells in the body. A hemoglobin level of pregnancy can naturally lower to 10.5 gm/dL (6.5163 mmol/L) representing a normal anemia of pregnancy.

What does a non-stress test tell the nurse about a pregnant client? 1. That the baby is going to be a boy or girl 2. The baby is doing well and the placenta is providing enough oxygen at this time 3. That the baby's heart is healthy and there are no birth defects 4. That the mother is strong enough to undergo vaginal delivery

2. Correct: Yes, the non-stress test identifies whether an increase in the fetal heart rate (FHR) occurs when the fetus moves, indicating adequate oxygenation, a healthy neural pathway from the fetal central nervous system to the fetal heart and the ability of the fetal heart to respond to stimuli. 1. Incorrect: No, the sex is not determined by this test. 3. Incorrect: No, we can't determine birth defects from a non-stress test. 4. Incorrect: No, we can't determine if the mother is strong enough to undergo vaginal delivery from a non-stress test.

The nurse is assigned a group of clients. For which client would the use of acetaminophen pose a higher risk? 1. 42 year old female who abuses cocaine. 2. 54 year old male who abuses alcohol. 3. 23 year old female who has asthma. 4. 34 year old male with sickle cell anemia.

2. Correct: The use of acetaminophen poses a higher risk for the client who abuses alcohol due to its interaction with the liver. Clients should be educated to be cautious if using acetaminophen due to the hepatotoxicity that can occur with liver dysfunction and failure. 1. Incorrect: Clients who use cocaine do not carry a higher risk of hepatotoxicity with acetaminophen use. 3. Incorrect: Clients who have a history of asthma do not carry a higher risk of hepatotoxicity with acetaminophen use. 4. Incorrect: Pain management should follow the "analgesic ladder" recommended by the World Health Organization for the treatment of cancer-related pain. The choice of analgesic and the dosage should be based on the severity of pain in the individual client. The ladder starts with nonsteroidal anti-inflammatory drugs (NSAIDs) or acetaminophen for mild-to-moderate pain. Because clients with sickle cell disease have varying degrees of hepatic impairment, acetaminophen may be contraindicated. So, the alcoholic client is at greatest risk.

The medical surgical nurse is admitting a client diagnosed with deep vein thrombosis (DVT) of the right leg. The client suddenly begins to report shortness of breath. Which additional early signs/symptoms indicative of a complication would the nurse need to report to the primary healthcare provider immediately? 1. Tachycardia with tachypnea. 2. Restlessness and dizziness. 3. Pain in the lower right leg. 4. A positive Homan's sign.

2. Correct: The worst complication of a DVT is the potential for a pulmonary embolism, resulting when part of the blood clot breaks free and travels to the lungs. This life-threatening complication presents with symptoms of hypoxia, including restlessness, agitation, or dizziness. The client may also develop chest pain, depending on the size of the clot. 1. Incorrect: While these symptoms may require further assessment, the question does not provide any parameters for vital signs. Individually, tachycardia and tachypnea could be attributed to pain, anxiety, or even hospitalization. There is not enough information provided to necessitate an immediate call to the Primary healthcare provider. 3. Incorrect: Pain in the affected extremity is not an unexpected finding with this diagnosis, although the nurse would need to further assess and evaluate the level and location of the pain in relation to the blood clot. This symptom is not surprising and would not require immediately alerting the primary healthcare provider. 4. Incorrect: The Homan's sign was a method formerly used to assess for the presence of a DVT and was performed by dorsiflexing the foot of the affected leg in an effort to elicit pain. However, this technique has proven to be unreliable and is no longer part of the assessment process.

A home care nurse is preparing to perform venipuncture on a client to draw blood. As the nurse gathers supplies, the client begins to experience palpitations, trembling, nausea, shortness of breath and a feeling of losing control. What is the client most likely experiencing? 1. Hyperventilation 2. Panic disorder 3. Somatization 4. Conversion disorder

2. Correct: These are all signs of panic disorder. Additional s/s include: sweating, feeling of choking, chest discomfort, abdominal distress, dizziness, lightheadedness, faintness, feelings of unreality or being detached from self, fear of losing control, fear of dying, Paresthesias, chills or hot flashes. 1. Incorrect: A client experiencing hyperventilation would exhibit rapid respiratory rate, and tingling of lips and/or hands. Hyperventilation may occur with a panic attack but the best answer is option 2. 3. Incorrect: Somatization is the process by which psychological needs or stress are expressed in the form of physical symptoms. These reports of signs and symptoms are usually several years in duration. 4. Incorrect: Conversion disorder is a psychological disorder with symptoms or deficits affecting motor or sensory function that mimic a neurological or general medical disease.

The nurse is evaluating a client for compliance to the prescribed diabetic program by checking recent lab results. Based on the lab data, what should the nurse conclude regarding the client? *Exhibit (lab work)* *Fasting Blood Glucose* 90 mg/dL (4.995 mmol/L) *Hemoglobin A1C* 6.5% 1. At risk for developing hypoglycemia. 2. Demonstrating good control of blood glucose. 3. At risk for developing Somogyi phenomenon. 4. Demonstrating signs of insulin resistance.

2. Correct: These are normal lab values indicating good control of blood glucose. 1. Incorrect: The client is not at risk for developing hypoglycemia. Fasting glucose is currently normal. 3. Incorrect: Somogyi pnenomenon occurs when there is a rebound hyperglycemia. The client is given too much insulin and their blood glucose level drops. The body attempts to compensate and releases hormones (epinephrine) that causes glycogenolysis. This leads to a rebound increase in the blood glucose levels. The insulin dose for this client should be decreased. Dawn's phenomena occurs when there is an increase in the blood glucose in the early morning hours due to release of growth hormone and cortisol. The client's insulin dose should be increased. 4. Incorrect: There are no signs of insulin resistance demonstrated from these normal values. A fasting blood sugar less than 100 mg/dL (5.6 mmol/L) is normal. So the client's lab result of 90 mg/dL (4.995 mmol/L) is normal. The goal for a client with diabetes is to have a hemoglobin A1c less than 7%. This client's hgb A1c is 6.5%. So which option matches these results? Option 2.

The nurse is caring for a client taking enoxaparin. Which group of symptoms should be reported to the primary healthcare provider? 1. AST of 12 U/L and ALT 20 U/L 2. Hematocrit of 46% decreased to 35% and blood pressure decreases from 122/78 to 108/54 3. Ecchymosis around the abdominal subcutaneous injection site and platelet count of 200,000. 4. Hemoglobin of 14.5 g/dL (2.3 mmol/L) increased to 16 g/dL (2.5 mmol/L) and increased erythemia of oral mucus membranes.

2. Correct: These values indicate a drop in hematocrit and drop in blood pressure. Both of these could represent bleeding. These would be important to report to the primary healthcare provider. 1. Incorrect: The nurse would need to watch and report any signs of liver complications due to the use of enoxaparin. The AST and ALT are two liver enzyme values that would increase with liver complications. These two values represent normal AST (8-40 U/L) and ALT (10-30 U/L) values. 3. Incorrect: Bruising (ecchymosis) at the injection site is a frequent occurrence with administration of enoxaparin. This platelet count is within the normal range. 4. Incorrect: The Hgb and color of oral mucus membranes indicate an increase in Hgb. This would not indicate bleeding.

Which action by two unlicensed nursing personnel (UAPs), while moving the client back up in bed, would require intervention by the nurse? 1. Lowers the side rails closest to them. 2. Places hands under client's axilla. 3. Lowers the head of bed. 4. Raises the height of the bed.

2. Correct: This action is not appropriate and requires intervention by the nurse. This could damage the brachial plexus nerves under the axilla. Use a draw sheet to prevent this from occurring. 1. Incorrect: This is a correct action. The UAPs will need to lower the side rails closest to them to safely move the client up in bed. Not lowering the rails could injury the UAPs back. 3. Incorrect: This action is correct. Moving the client upward with the head of the bed raised works against gravity, requires more force and can cause back strain. 4. Incorrect: This action is appropriate and would not require intervention by the nurse. Raising the height of the bed brings the client close to the UAPs center of gravity and decreases the chance of back injury.

The client with bi-polar disorder is parading around the common areas of the psychiatric unit in a sexually suggestive manner. The client then sits on the lap of one of the young male clients. What should the nurse do? 1. Tell the client that the behavior is inappropriate. 2. Accompany the client to the TV room on the unit. 3. Allow the male client to handle the situation. 4. Continue with the unit routine.

2. Correct: This behavior must be interrupted, as the rights of other clients are being jeopardized. The other clients are being exploited by the manic client. Stop the behavior by going with them to another area. Many people with bipolar disorder don't recognize the extreme changes in their moods and the effects these changes have on their lives and others. You must stop them and remove them from the situation. 1. Incorrect: This client is not able to make accurate decisions in this current state, so telling the client that this behavior is inappropriate is not likely to stop it. Hypersexual behavior is often a warning sign of a manic episode. Evaluate the clients medications because the mood stabilizing medications like lithium or valprate (Depakote®​) should prevent these symptoms. 3. Incorrect: The male client has a right to a safe care environment. He should not be exploited by staff or clients. Asking a fellow client to handle an appropriate situation will never be right on the NCLEX. 4. Incorrect: The behavior must be interrupted in order to maintain the rights of other clients and to maintain the dignity of the client who is in a manic state. Ignoring the symptoms and the disruption will not fix the problem. As a nurse you always want to fix the problem.

A nurse has provided postpartum discharge instructions to a client who had a cesarean section. What statement by the client would indicate to the nurse that further teaching is necessary? 1. "I will relax and contract my pelvic floor muscles 10 times, eight times a day." 2. "Driving is permitted in one week if I am pain free." 3. "Lifting anything heavier than my baby is not advised." 4. "I will not cross my legs while sitting."

2. Correct: This is an incorrect statement. C-sections require a much longer recovery. The client will not have the abdominal muscles to press down on the brake pedal in an emergency. Therefore, new moms who had C-sections should wait until after the three week postpartum appointment to drive. 1. Incorrect: This would be a correct statement. The client should do Kegel exercises to prevent incontinence and strengthen the pelvic muscles. 3. Incorrect: This is a correct statement. Lifting heavy objects can cause bleeding. New moms are not able to lift anything more than the baby's weight. 4 Incorrect: This is a correct statement. Avoid sitting for prolonged periods of time with legs crossed to prevent thrombophlebitis.

The nurse is caring for a primipara client at 27 weeks gestation. Which client learning need should the nurse identify as priority at this stage of pregnancy? 1. Appropriate nutrition 2. Signs of preterm labor 3. Fetal teratogens 4. Newborn care

2. Correct: This client is entering the third trimester when the risk of preterm labor and delivery are highest. Women who are aware of the consequences of preterm birth may be more likely to take action to prevent it. Signs and symptoms of preterm labor should be recognized and reported immediately to the primary healthcare provider. 1. Incorrect: Appropriate nutrition is a learning need of the first trimester. Nurses should provide nutritional education to women who are considering pregnancy. Nutritional needs should be discussed early in the pregnancy to ensure that essential nutrients are provided and harmful foods are avoided. 3. Incorrect: Appropriate learning need of the first trimester. Ideally, prevention of exposure to harmful influences begins before conception because all major organ systems develop early in pregnancy, often before a woman realizes that she is even pregnant. 4. Incorrect: While this is an important topic for the last trimester, physiological needs have priority. Teaching the woman signs of preterm labor should be done before newborn care teaching.

A 16 year old female student is escorted to the school nurse after fainting in gym class. The student tells the nurse, "I just got weak from running." Upon examination, the nurse notes poor skin turgor, dry mucous membranes, and erosion of tooth enamel from her front teeth. Height is 5'4" (162.56 cm) and weight is 110 lbs (50 kg). The student reports muscle pain in the legs. Based on this data, what should the nurse suspect? 1. Anorexia Nervosa 2. Bulimia Nervosa 3. Obesity 4. Physical violence

2. Correct: This client is exhibiting the signs of bulimia nervosa. Additionally, the client will binge on excess calories, and then purge through vomiting and the use of laxatives, diuretics, and enemas. Weight fluctuates: usually within normal limits or slightly under or slightly overweight. Tears in the esophageal and gastric mucosa can occur. Due to vomiting, tooth enamel can erode. 1. Incorrect: Gross distortion of body image, refusal to eat, grossly underweight and malnourished. Characteristics of anorexia nervosa. The client is at the low end of the weight range for her height, but not underweight. 3. Incorrect: Obesity occurs from eating more than the body needs. Weight is more than 20% over expected body weight. They do not purge. This client's BMI of 18.9 is normal, not obese. 4. Incorrect: There is no indication that this is physical violence. The client may report headaches, dizziness and accidents such as falls. However, this client does not have any signs and symptoms of battering such as bruises, scars or burns.

The nurse is administering medication to an elderly client who has no visitors. The client takes the pills, and, as the client hands the medication cup back to the nurse, grabs onto the nurse's hand tightly. What is the most logical rationale for the client's action? 1. Confusion and disorientation. 2. Scared and lonely and grabs the nurse's hand for comfort. 3. Would like to talk with the nurse. 4. Would like to reminisce with the nurse.

2. Correct: This elderly client with no visitors is most likely scared and lonely. The touch of the nurse's hand is comforting for the client. 1. Incorrect: There is no indication of confusion or disorientation. 3. Incorrect: Grabbing the nurse's hand indicates more than just a desire to talk. This is indicative of needing comfort and personal touch. 4. Incorrect: There is no indication of a desire to reminisce from the information in the question.

An 18 month old is admitted to the unit with a diagnosis of pertussis. The mother asks the nurse, "How did my child get this disease? I didn't think anyone got that anymore." What is the appropriate response by the nurse? 1. "Pertussis is a common childhood disease since there is no vaccine." 2. "Since not all children are immunized against pertussis, the disease has reemerged." 3. "Your baby got this disease because you didn't have your child immunized." 4. "Since your child is already sick, let's just focus on getting well."

2. Correct: This is a correct statement. Therapeutic communication means providing information that will help clients make better choices. Not all parents have had their children immunized against pertussis, so this disease is being seen in clients again. DPaT should be given at 2, 4 and 6 months of age. A booster is given at 15-18 months old and then at 4-6 years old. 1. Incorrect: This is not true. There is a vaccine. DPaT should be given at 2, 4, and 6 months of age. A booster is given at 15-18 months old and then at 4-6 years old. 3. Incorrect: Don't be confrontational. This puts the mother on the defensive. This is not therapeutic communication. Giving one's own opinion, evaluating, moralizing or implying one's values by using words such as "nice" "bad" "right" "wrong" "should" and "ought". "You shouldn't do that. It is wrong".​​ Everyone who does not get immunized gets the disease. 4. Incorrect: Do not change the subject. This does not address the mother's concern. Changing the subject, or introducing new topic inappropriately, can create anxiety. The nurse needs to address the mother's question of how the child contracted the disease.

A client who has been prescribed zolpidem for insomnia has received medication education. Which statement by the client indicates to the nurse that education was successful? There is a high potential for tolerance with this medication." 2. "I may do things in my sleep that I will not remember the next day." 3. "Daytime drowsiness is rare when taking this medication." 4. "The most common side effects of this medication are confusion and a bitter aftertaste."

2. Correct: This is a true statement. The client may sleep drive, make phone calls, prepare food while asleep and have no memory of the activity. 1. Incorrect: This is a schedule 4 substance. There is a low potential for tolerance, dependence, or abuse with this medication. 3. Incorrect: Daytime drowsiness and dizziness are common side effects. 4. Incorrect: Daytime drowsiness and dizziness are the most common side effects. Bitter aftertaste does not occur with this medication. Zolpidem is a sedative, also called a hypnotic. It affects chemicals in the brain that may be unbalanced in people with sleep problems (insomnia). Zolpidem may impair the client's thinking or reactions. The cleint may still feel sleepy the morning after taking this medicine, especially if taking the extended-release tablet. Wait at least 4 hours or until fully awake before doing anything that requires being awake and alert. Some people using this medicine have engaged in activity such as driving, eating, walking, making phone calls, or having sex and later having no memory of the activity.

The nurse enters a client's room to administer morning medications and notes that the client is praying aloud. What would be the nurse's best action? 1. Interrupt the client to administer the medications. 2. Wait quietly until the prayer is finished. 3. Join the client for the prayer. 4. Ask the client if you can provide a directed prayer.

2. Correct: This is the best action by the nurse as this is a private spiritual moment for the client. Prayer is a self-care strategy that provides comfort, increases hope, and promotes healing and psychological well-being. The nurse could either leave and return later or wait quietly for the client to finish. 1. Incorrect: Administering the medications can wait until the client finishes the prayer. 3. Incorrect: Do not assume that the client wants others to join in the prayer. This is a private moment for the client. 4. Incorrect: Do not assume that the client wants others to join in the prayer. Don't interrupt the client while praying.

The nurse overhears this client responding on the phone when their boss asks them to work an extra night shift. Which statement by the client demonstrates assertive communication? 1. "I know you are joking! I have already worked an extra night shift." 2. "I do not want to work an extra night shift. I have already worked an extra shift this week." 3. "Umm, well, okay. I guess I will work an extra night shift." 4. "Okay, I'll work an extra night shift." Then they say to another client. "The nerve of my boss to ask me to work another extra shift."

2. Correct: This is an example of assertive communication, the best response. Assertiveness is asking for what one wants or acting to get what one wants in a way that respects the rights and feelings of other people. 1. Incorrect: This response is aggressive behavior. This response is delivered in a forceful manner. 3. Incorrect: This response is nonassertive. This statement is giving into the boss, even though the client really doesn't want to work. Keywords are "umm, well and okay" 4. Incorrect: This response is Passive-Aggressive. It is the indirect expression of anger.

A client has received discharge education post extracapsular cataract surgery. Which statement made by the client indicates to the nurse that further teaching is needed? 1. "A protective eye patch will be needed for 24 hours." 2. "I will notify my primary heathcare provider for any amount of discharge, redness or scratchy feeling because these symptoms are abnormal." 3. "I will clean the surgical eye with a clean tissue, wiping once from the inner aspect of the closed eye to the outer eye." 4. "When sleeping, I will avoid lying on the same side of my affected eye."

2. Correct: This is an incorrect statement by the client. Slight morning discharge, some redness, and a scratchy feeling may be expected for a few days." Clients are instructed to report any pain that is unrelieved, redness around the eye, nausea or vomiting to the primary healthcare provider. 1. Incorrect: This is a true statement and does not require intervention. Following surgery, the eye is covered with a patch and a metal or plastic shield for protection from light and trauma. 3. Incorrect: This is the correct way to clean the surgical eye. Cleaning from the inner to outer canthus avoids entrance of microorganisms into the lacrimal duct. 4. Incorrect: This is a correct action. Increased intraocular pressure needs to be avoided. Clients are instructed to avoid sleeping on the operative side.

The nurse has been educating a client diagnosed with general anxiety disorder (GAD). Which statement by the client indicates the need for further education? 1. "I will avoid caffeine from now on." 2. "When I feel anxious I will increase my breathing to get more oxygen to my brain." 3. "I will go for a brisk walk when I begin to feel anxious." 4. "I will keep a diary of anxiety attacks to determine what triggers them."

2. Correct: This is an incorrect statement. The client needs to slow breathing down with deep-breathing exercises. An increase in respirations can lead to respiratory alkalosis. 1. Incorrect: Caffeine can increase panic and anxiety in clients whom suffer with GAD. Caffeine is a stimulant and can produce symptoms like those of anxiety. This statement means that teaching has been effective. 3. Incorrect: Physical activities discharge excess energy in a healthful manner. Exercise produces endorphins, which promotes a sense of well-being. This statement means test teaching has been effective. 4. Incorrect: Recognition of precipitating factors is the first step in teaching a person to interrupt escalation of anxiety. Also, identifying stressors promotes future change. This statement means that teaching has been effective.

After applying oxygen using bi-nasal prongs to a client who is having chest pain, the nurse should implement which intervention? 1. Have the client take slow deep breaths in through the mouth and out through the nose. 2. Post signs on the client's door and in the client's room indicating that oxygen is in use . 3. Apply Vaseline petroleum to both nares and 2 x 2 gauze around the oxygen tubing at the client's ears. 4. Encourage the client to hyperextend the neck, take a few deep breaths and cough.

2. Correct: This is an oxygen therapy safety precaution that the nurse should implement after applying oxygen. It is also the only correct and safe option in the question. 1. Incorrect: The bi-nasal prongs would mean that the oxygen is going in through the nose. Breathing deeply through the mouth and out through the nose would not increase oxygenation for a client having chest pain and would disrupt the flow of oxygen through the nose. 3. Incorrect: The nurse should avoid using petroleum products where oxygen is in use because they are flammable. 4. Incorrect: These client actions have nothing to do with oxygen administration and would cause more distress to the client with chest pain.

A home health nurse is caring for a Mexican-American client who has been discharged from the hospital post myocardial infarction. While the nurse is at the house, a curandero is also at the home at the request of family members. What is the best action of the nurse? 1. Leave, and return once the curandero has left. 2. Discuss the plan of care with the client, family, and curandero. 3. Ask the curandero to leave so that the client can be assessed. 4. Explain to the family that the curandero is not a reliable healthcare option.

2. Correct: This is the best course of action for the nurse. The health and healing of a client come from many components, including spirituality, religion, folk remedies, alternative therapies, and modern medicine. Unless something is harmful to the client, it is best to incorporate all components into the care of the person. 1. Incorrect: Leaving will not allow the nurse to discuss care of the client with all members of the healthcare team and family. This is a good time to learn about the curandero, health beliefs, etc. 3. Incorrect: The client and family have requested the curandero. Asking him to leave would be insulting. The nurse would not develop a good rapport with the client this way. 4. Incorrect: This does not take into account the client's beliefs in health, wellness, and illness. The nurse should work to incorporate folk medicine from the curandero as long as it will not harm the client.

After discontinuing a peripherally inserted central line (PICC), it is most important for the nurse to record which information? 1. How the client tolerated the procedure. 2. The length and intactness of the central line catheter. 3. The amount of fluid left in the IV solution container. 4. That a dressing was applied to the insertion site.

2. Correct: This is the most important information that needs to be documented. This information would be important in determining if a potential safety issue/complication could occur as a result of the PICC line being removed or a portion of the line breaking off before removal. 1. Incorrect: This is not the most important information that needs to be documented. There are no client safety issues with charting the client's tolerance of the procedure. 3. Incorrect: This would be charted so the intake and output could be calculated. This is not the most important data that needs to be documented related to the removal of the PICC line. 4. Incorrect: This would need to be documented because a dressing is applied to the insertion site after removal. However, this is not the most important data that would need to be documented after this procedure.

A client with tuberculosis (TB) has been coming to the health department for directly observed therapy (DOT) for the past month. Today, the client states, "I don't think I need to come back anymore. I am feeling much better now." What should the nurse tell the client? 1. "You have taken your medication long enough so, the primary healthcare provider should discontinue it today." 2. "If you stop taking your medication now, your disease could become resistant to this medication, making it harder for you to be cured." 3. "I will be required to have you arrested if you do not come back for further treatment." 4. "Just let us decide when you should stop taking the medication."

2. Correct: This is true regarding TB treatment. The Medication has to be taken for the entire course. The minimal length of time for therapy is 3 months. 1. Incorrect: Treatment usually lasts 4-7 months. If the medication regimen is not strictly and continuously followed, the disease may become drug-resistant. It is not the nurse's place to determine when enough medication has been taken. 3. Incorrect: The nurse needs to discuss the reason for continuing to take the medication. This step is premature and intimidating. Also, threatening to have the client arrested will not likely maintain a good patient-nurse relationship. 4. Incorrect: This statement is non-therapeutic and dismissive of the client. This does not address the client's statement of thinking they have had enough medicine and should stop.

A client who must use crutches, is being taught by the nurse how to perform a three-point gait. What information should the nurse provide? 1. Move right crutch forward, then left foot. Next move left crutch forward, then right foot. 2. Move both crutches forward without bearing weight on the affected leg, then move the unaffected leg forward. 3. Move left crutch and right foot forward together, then move the right crutch and left foot forward together. 4. Move both crutches ahead together, then lift body weight by the arms and swing both legs to the crutches.

2. Correct: This method is correct for the three-point gait. Client has to bear weight on the unaffected foot and both crutches. The affected leg does not touch the ground. 1. Incorrect: This is the four-point alternate gait. This type of gait is used commonly when =both legs are weakened. 3. Incorrect: This is the two-point alternate gait. Two point requires at least partial weight bearing on each foot. 4. Incorrect: This is the swing-to gait. This gait is indicated for individuals with limited use of lower extremities and trunk instability.

The driver of a motor vehicle was driving while intoxicated with a friend in the passenger seat. Both clients are admitted to the Intensive Care Unit. The nurse is caring for the driver of the vehicle who states, "I'm so scared. What if the car accident is my fault and my friend dies?" What is the most appropriate response from the nurse? 1. "I wouldn't worry about that; everything will be all right." 2. "You are worried that you may be responsible for your friend's condition?" 3. "How come you were drinking and driving?" 4. "Let's not talk about that right now."

2. Correct: This type of therapeutic communication is called restating. By repeating the client's statement, the nurse expresses an understanding of what the client said. Restating also gives the client the chance to clarify or continue his thoughts. 1. Incorrect: This statement gives the client false reassurance. It devalues the client's feelings and may discourage the client from expressing further feelings. 3. Incorrect: This statement is probing and causes the client to feel defensive. 4. Incorrect: This statement rejects the client's ideas and feelings; this may lead to decreased interaction with the nurse due to fear of future rejection.

A client with asthma uses a corticoid inhaler. What teaching should the nurse provide to decrease the risk of an oral fungal infection? 1. Lessen the exposure of the oral mucosa to the ICS by exhaling rapidly. 2. Rinse the mouth completely and brush teeth following the use of the ICS. 3. Use alcohol based mouth rinses with ICS. 4. Drink water prior to using the ICS.

2. Correct: Thrush, is an oral fungal infection, which is one of the most common side effects of ICS. Up to 1/3 of all clients on ICS develop this infection. Rinsing and brushing helps to remove the medication residual from the oral mucosa and upper pharyngeal area. 1. Incorrect: This is not appropriate because exhaling rapidly would result in a loss of the medication and reduce the effectiveness. 3. Incorrect: This is not accurate because alcohol based mouth rinses have not been shown to reduce the risk of thrush. Alcohol based mouth wash can be drying to the oral mucosa. 4. Incorrect: Drinking water, prior to using the ICS is not an effective means of preventing thrush.

An unresponsive client begins to vomit. What intervention by the nurse would have the highest priority? 1. Suction the client's mouth. 2. Turn the client onto their side. 3. Apply oxygen per face mask. 4. Insert an oropharyngeal airway.

2. Correct: To prevent aspiration the first thing to do is turn the client onto their side. Leaving the client in the supine position will allow vomitus to get into the lungs when the client breathes. 1. Incorrect: Suctioning can be done after you turn the client onto the side. They could aspirate while you get the suction equipment put together to suction. 3. Incorrect: Applying oxygen will not prevent aspiration. Turn onto side. 4. Incorrect: The first thing to do is to turn to the side to protect the airway. Don't waste time. The oropharyngeal airway will not prevent aspiration. It is used to maintain an open airway.

A new mother asks the clinic nurse why her baby should receive recommended vaccinations. What is the best response by the nurse concerning vaccinations? 1. "Vaccinations give antibodies to your baby to protect them from disease." 2. "Vaccinations will help your baby produce antibodies against disease causing organisms." 3. "Federal law requires that your baby receive recommended vaccinations." 4. "There is no reason not to vaccinate your baby since only mild, uncomfortable reactions can occur."

2. Correct: Vaccines are suspensions of antigen preparations intended to produce a human immune response to protect the person from future encounters with the organism. 1. Incorrect: Vaccines will cause the body to produce antibodies. Vaccines give possible immunity to the baby. 3. Incorrect: Vaccines are required for admittance into public school. If a child is homeschooled, the parent may not have the child vaccinated. 4. Incorrect: It is true that the vaccination may cause a mild reaction, but this is not the best response. This answer does not address the mother's question.

A client is curious about visible appearance changes related to menopause. What menopausal changes, in general, would the nurse explain to the client? 1. Bone loss and fractures. 2. Loss of muscle mass. 3. Improved skin turgor and elasticity. 4. A reduction in waist size.

2. Correct: Visible changes associated with menopause include loss of muscle mass, increased fat tissue leading to thicker waist, dryness of the skin and vagina, hot flashes, sleep abnormalities, and mood changes. 1. Incorrect: Bone loss is dependent on bone mass, weight-bearing exercise, and nutrition. Some bone loss may occur, but may not lead to fractures. 3. Incorrect: A decrease in turgor and elasticity may occur as we grow older. 4. Incorrect: There is increased fat tissue with an increase in waist size.

A client with a history of adrenal insufficiency is placed on fludrocortisone. Which value is most important for the nurse to monitor? 1. Magnesium 2. Weight 3. Pain 4. Glucose

2. Correct: Weight is monitored daily to assess for sudden increases which would indicate fluid retention. Fludrocortisone is a man made glococorticoid and is used to treat low gloucocorticoid levels caused by diseases of the adrenal gland. Glucocorticoids are important in maintaining salt and water balance in the body and normalizing blood pressure. 1. Incorrect: No, monitor for lowered serum potassium instead of magnesium because fludrocortisone causes the body to retain sodium, and excrete calcium and potassium. 3. Incorrect: Adrenal insufficiency and steroid therapy are not precursors of pain. 4. Incorrect: Glucose may increase as a result of steroid therapy as glucocorticoids inhibit insulin. But, weight is the critical value to monitor for dosing, as treatment may be discontinued with a sudden weight increase.

The nurse is caring for a client on the post surgical unit. What should the nurse teach the client about short term treatment of post op pain? 1. There are no concerns about addiction from pain medications following surgery. 2. Pain control following surgery rarely results in addiction. 3. The opioid medications typically result in addiction. 4. The primary healthcare provider will not prescribe an addictive medication.

2. Correct: When a person is in acute pain following surgery, the risk of addiction to pain medication is rare. The key is to provide the medication over a short period of time to get the client past the initial pain of surgery. Remember the client will be ambulating early. Ambulation and nonpharmaceutical comfort measures should also be provided by the nurse to decrease the need for narcotics as client recovery continues. 1. Incorrect: There are slight concerns about addiction with administration of opioids; however, it is usually not a concern for the majority of post op clients with short term use in the hospital. The nurse should use alternative methods for providing relief as well. Guided imagery, massage, gradual ambulation, are just a few examples. 3. Incorrect: Use of opioids may result in addiction; however, research shows that only a small percentage of the population is prone to addiction. The goal of postoperative pain management is to relieve pain while keeping side effects to a minimum. This is often best accomplished with a multimodal approach. 4. Incorrect: Opioids are potentially addictive; however, they serve a very useful purpose in the treatment of short-term post-op pain.

In the office for a yearly physical examination, a 30-year-old client reports that the client and husband used to be very happy before the children were born. Now the client is struggling with the current situation. What should the nurse understand about this situation? 1. The client is probably having an extramarital affair. 2. The developmental task at this stage is adjusting to the needs of more than two family members. 3. A relative or close friend should be consulted for help so the client can pursue activities outside the home. 4. The client should be referred to a psychotherapist for evaluation and care.

2. Correct: When children are born or adopted into a family, the established couple must adjust to supporting the physical and emotional needs of the additional family member. Additionally, the couple is engaged in developing an attachment with the child(ren) and coping with energy depletion and lack of privacy. These requirements may lead to a sense of unhappiness and frustration on the part of one or both parents. 1. Incorrect: The answer choice that the client is having an extramarital affair is inappropriate. There is no evidence to support this assumption. 3. Incorrect: Although receiving assistance from family and friends is a good option, it is not the best answer choice. This family has to learn to adjust to being a family. 4. Incorrect: The client's feelings are normal and do not require that the client be referred to a psychotherapist.

Over which locations does the nurse auscultate breath sounds? 1. Trachea and lateral areas of thoracic cage 2. Anterior and posterior aspects of all lung fields 3. The mid-section as well as the lateral section of the lungs 4. The mid-clavicular to mid-axillary lines comparing side to side

2. Correct: Yes, you must go side to side (lateral aspects) and also listen to the front and back (anterior and posterior). 1. Incorrect: Does not include anterior and posterior lung fields. Do not auscultate over trachea. 3. Incorrect: Does not include anterior and posterior lung fields. 4. Incorrect: Does not include the posterior lung fields. If possible, auscultation of the chest should be done with the client in the seated position. The diaphragm of the stethoscope should be used. The stethoscope should be placed against the client's bare skin. The examination should include listening to the anterior chest, the midaxillary region, and the posterior chest. The posterior chest should be examined from the apex to the base of the chest. The breath sounds should be assessed during both quiet and deep breathing. A full breath should be auscultated in each location. The examiner should listen for the pitch, intensity, duration, and distribution of breath sounds, as well as note any abnormal or adventitious sounds.

A nurse is teaching a group of women about human papillomavirus (HPV). What should the nurse tell the women that human papillomavirus puts women at risk for? 1. Human immunodeficiency virus 2. Cervical cancer 3. Hepatitis B 4. Cirrhosis

2. Correct: Women who have had human papillomavirus are at increased risk for developing cervical cancer. 1. Incorrect: HPV does not increase the risk of developing HIV. HPV increases the risk for developing cervical cancer. 3. Incorrect: HPV does not contribute to Hepatitis B. HPV increases the risk for developing cervical cancer. 4. Incorrect: HPV does not contribute to Cirrhosis. HPV increases the risk for developing cervical cancer.

A child is being discharged home following a bone marrow transplant. When providing discharge instructions to the parents, what information is most important for the nurse to include? 1. Clean toothbrush weekly with alcohol. 2. Avoid eating raw fruits and vegetables. 3. Drink bottled water the day. 4. Apply heating pad to bruised areas of the skin.

2. Correct:The greatest risk to clients following a transplant is the chance of infection from any source since the client is severely immune-compromised for an extended period of time. There are numerous precautions necessary to avoid bacteria, but one area of concern is food storage, preparation, and consumption. Raw fruits with no skin to peel, such as strawberries, and raw vegetables like broccoli and cauliflower, present a serious risk for bacterial contamination and should not be consumed by new transplant clients. 1. Incorrect: Precise mouth care is vital following a bone marrow transplant; however, rinsing a toothbrush in alcohol is unsafe. Any residual alcohol would cause irritation and trauma to gum tissue, placing the client at risk for mouth inflammation and infection. Clients are instructed to brush teeth twice daily with a soft bristle brush, using a fluoride toothpaste. Some clients are instructed to soak the toothbrush once weekly in a special bleach solution, then rinse in hot water, while others need to replace the toothbrush weekly, based on lab test results. 3. Incorrect: Standing water of any type quickly builds up bacteria, including flower vases and vaporizers. Although bottled water may seem a safe choice, after that bottle is opened, bacteria begins to quickly build up, even if the bottle is recapped. Any water standing more than 15 minutes is considered old and must be thrown out. 4. Incorrect: With bone marrow transplant clients, it will be months before the body begins to stabilize and produce normal blood cells. Bruising and low platelet counts are to be expected for a period of time. When clients develop bruising, the approved treatment is cold compresses or ice packs applied for 15 minutes a couple times per day, and never a heating pad. Additionally, the healthcare provider should be notified so that a current platelet count can be obtained.

A teenager leaves class in the middle of an exam to go to the school nurse's office. The student reports difficulty sleeping for several days, increasing nervousness, irritability, and palpitations. The nurse notes flushing of the skin, and an irregular heartbeat. What would be the best question for the nurse to ask this client? 1. "Do you feel this way because you are afraid that you are failing the exam?" 2. "Have you been drinking energy drinks while studying for your exam?" 3. "What drugs are you taking?" 4. "Do you want me to call your mother?"

2. Incorrect: Caffeine is a stimulant used to keep people awake and increase energy. It is found in many OTC medications and in many soft drinks and energy drinks. The student is exhibiting all the signs/symptoms of ingesting too much caffeine. 1. Incorrect: This is not addressing the problem. These are physical symptoms of a problem. 3. Incorrect: This is confrontational and will put the student on the defensive. Caffeine is not generally thought of as a drug by clients, so might not even be considered in the first place. 4. Incorrect: This might be done later, but this question does not help to determine what is going on with the student.

The nurse is planning care for a client admitted with a diagnosis of new onset myasthenia gravis. Which nursing interventions should the nurse include in order to decrease the risk of aspiration? 1. Provide thin liquids such as water with meals. 2. Offer small bites of food. 3. Allow client to rest between each bite of food. 4. Offer small meals in the morning and larger meals in the evening. 5. Position client upright with head tilted slightly back when eating. 6. Provide meals 30 minutes before administration of cholinesterase inhibitor medication.

2., & 3. Correct: Offer the client small bites and instruct to chew well, eat slowly, swallow after each bite, and swallow frequently. Allow the client to rest while chewing and in between bites to restore strength. 1. Incorrect: Provide highly viscous foods and thickened liquids that are easy to chew and swallow. Thin liquids are more likely to cause aspiration. 4. Incorrect: Offer large meals in the morning and small meals in the evening. The client is more fatigued as the day progresses, so a smaller meal is best in the evening. 5. Incorrect: Position the client upright with head slightly forward when eating and drinking, using compensatory maneuvers (chin tuck, head turn) as necessary. 6. Incorrect: Adjust the client's eating schedule to optimize medication efficacy. Typically, meals should be taken during periods of optimal strength (such as during the earlier part of the day, 30 minutes after administration of cholinesterase inhibitor medications, or after rest periods).

Where should a nurse place the stethoscope when auscultating heart sounds? 1. First intercostal space left of the sternum to hear sounds from the pulmonic valve area. 2. Fourth intercostal space to the left of the sternum to hear sounds from the tricuspid area. 3. Second intercostal space to the right of the sternum to hear sounds from the aortic valve area. 4. Fifth intercostal space left side of sternum to hear sounds from the mitral area. 5. Apex of the heart to hear the loudest 2nd heart sound (S2).

2., & 3. Correct: These are correct locations to listen to heart sounds. 1. Incorrect: Second intercostal space left of the sternum to hear sounds from the pulmonic valve 4. Incorrect: The fifth intercostal space in the midclavicular line is where you will hear sounds in the mitral area. 5. Incorrect: This is where you will hear the loudest 1st heart sound (S1). Listen at the base to hear S2 the loudest. Option 1 is false. This option should say 2nd intercostal space. Option 2 is true. Option 3 is true. Option 4 is false. The correct location is 5th intercostal space midclavicular line. Option 5 is false. The first heart sound is best heard at this location.

The nurse is caring for a client with multiple episodes of diarrhea and suspected Clostridium Difficile (C. diff). Which interventions should be included in the plan of care? 1. Institute contact precautions only after confirmation of stool culture. 2. Instituting contact precautions for all who enter the client's room 3. Using alcohol based foam for hand hygiene. 4. Dedicating equipment for use only in the client's room. 5. Requesting antidiarrheal medication for the client.

2., & 4. Correct: Contact isolation will be needed to prevent the spread of infection. Also the electronic equipment for vital signs must not be used in the room. The client will need a disposable stethoscope, BP cuff and thermometer dedicated for use in that patient room. 1. Incorrect: Precautions should be instituted and a stool sample sent for any client with persistent diarrhea. Isolation should be in place with suspected c. diff. 3. Incorrect: Soap and water must be used to clean the hands. Alcohol based foams do not have enough alcohol in them to destroy the c diff spores. 5. Incorrect: Medications to stop diarrhea will not be prescribed with c. diff. because they cause even further irritation.

The nurse is providing discharge teaching for a family of a 2 year old going home on digoxin. Which teaching should the nurse include? 1. Use a 5 mL syringe to draw up the medication so the numbers can be seen easily 2. Give 1 hour before or 2 hours after feedings 3. Add the medication to the bottle so it is easier to administer 4. Check the apical pulse before giving medication 5. Store the medication in the refrigerator

2., & 4. Correct: Digoxin should be given on an empty stomach for better absorption. The apical pulse should be checked prior to administering digoxin. 1. Incorrect: Remember rarely do we give more than 1mL to an infant, it should be drawn up in the smallest available syringe (1mL syringe), you are increasing the chance of a medication error using the larger syringe. 3. Incorrect: NEVER add medication to a bottle, if the infant doesn't eat the entire bottle you have no idea how much medication was actually administered. 5. Incorrect: Digoxin is stored at room temperature.

An unlicensed assistive personnel (UAP) reports to the charge nurse that a postoperative client's 8AM blood pressure is 200/104 and the oxygen saturation reading is 86%. What actions would be appropriate for the charge nurse to delegate? 1. Tell the LPN to assess for shortness of breath and evidence of tissue prefusion. 2. Have the LPN reinforce the use of relaxation techniques. 3. Ask the LPN to draw arterial blood gas levels. 4. Instruct the RN to administer the prescribed dose of labetalol hydrochloride IV. 5. Instruct the UAP to call the primary healthcare provider and notify of change in client's condition.

2., & 4. Correct: The LPN can reinforce teaching. The client's BP is elevated and using relaxation techniques along with the medication that is being administered may help to decrease the client's BP. Labetalol is beneficial in this situation because of its rapid onset of action (approximately 5 minutes). The charge nurse delegates this to the RN because it would be outside the scope of practice of the LPN and not in the role of the UAP to administer IV medication. 1. Incorrect: The LPN cannot assess, evaluate or teach. These are the roles of the RN and are outside the scope of practice of the LPN. 3. Incorrect: Drawing ABGs from an artery is out of the scope of practice of the LPN. 5. Incorrect. It is not in the role of the UAP to notify the primary healthcare provider of changes in the client's condition. The UAP could not receive additional prescriptions should the primary healthcare provider desire to add or change prescriptions based on the client's change in condition. In this question the charge nurse is delegating tasks to someone else. It is a general question but does not specify who to delegate to. So look at each option alone as a true or false statement. Remember that the RN is responsible for assessment, evaluation, planning care, and teaching. Knowing that, let's look at the options. What option can you immediately eliminate? Option 1 because we know the LPN cannot assess. Look at option 2. This options asks the LPN to reinforce use of relaxation techniques. Can the LPN reinforce teaching? Yes they can. So this is true. Option 3. Can the LPN draw arterial blood gasses? No, this is beyond the scope of the PN. Sticking the artery is a high risk procedure. Option 4 Can an RN give IV labetalol (a beta blocker)? Yes Option 5. Can the UAP call the primary healthcare provider? No, this is beyond the scope of the UAPs practice.

Which client is legally able to sign a consent for surgery? 1. An 86 year old client who is disoriented. 2. A 62 year old client who speaks only Spanish. 3. A 41 year old client who just received midazolam. 4. A 17 year old client needing an emergency appendectomy whose parents cannot be contacted. 5. A 44 year old with schizophrenia who is hallucinating.

2., & 4. Correct: The Spanish speaking client should have a trained medical interpreter, either in person,by telephone, or by video conference, but the client can still sign the consent. The 17 year old client is considered a minor, however, since the parents are not available, the emergency exception rule, known as "implied consent" would be followed. The primary healthcare provider must document the nature of the emergency, the reason why immediate treatment is required, and the attempts to obtain consent from the minors parents or legal guardian. 1. Incorrect: The 86 year old client who is disoriented is not considered capable of making an informed decision. 3. Incorrect: Midazolam is a benzodiazepine administered for preoperative sedation/amnesia. For a consent to be legally valid, the consent must be signed prior to being administered preoperative medication or other mind-altering medications. 5. Incorrect: This client with schizophrenia who is hallucinating does not have the ability at this time to understand explanations, understand risks and benefits, and communicate a decision based on that understanding.

A nurse has taught a group of teenage girls about breast self-awareness. Which statements by the teens would indicate to the nurse that teaching was effective? 1. "I should have a clinical breast exam every 5 years starting at the age of 18." 2. "Doing a monthly breast self-exam will help me learn what is normal for me." 3. "It is important to know my maternal health history." 4. "Signs I should not ignore include dimpling of the skin, and nipple discharge." 5. "Self-breast exam should be done a few days before my menstrual cycle begins."

2., & 4. Correct: The purpose of breast self-exam is to determine what is normal. This will allow the client to recognize when there is a change in breast tissue. S/S of breast cancer includes dimpling of the skin, nipple discharge, tenderness, change in appearance, retracted nipple, hard lump and itchy or scaly skin. 1. Incorrect: Clinical breast exams are recommended every 3 years starting at age 20, and every year starting at age 40. 3. Incorrect: Talk to both sides of the family to learn about your family health history. 5. Incorrect: The breast self-exam should be done after the menstrual cycle (day 7-12) for a better exam. The breasts will be too tender just prior to the period. Option 1 is false. This statement should read every 3 years starting at age 20. Option 2 is true. You have to know what is normal in order to determine when something is abnormal. Option 3 is false. Knowledge about breast health is needed from both sides of the client's family. Option 4 is true. The signs listed could be indicative of breast cancer. Option 5 is false. Day 7-12 is the best time to perform the self-breast exam.

What characteristics would indicate to the obstetrical nurse that a client is experiencing false labor? 1. Cervical dilation noted. 2. Contractions decrease with sleep. 3. Bloody show noted. 4. Contraction intensity increases with walking. 5. Contractions felt in abdomen above umbilicus.

2., & 5. Correct: False labor or Braxton Hicks contractions are mild, irregular frequency, and intermittent; decrease in frequency, duration, and intensity with walking or position changes; often stop with sleep or comfort measures such as oral hydration or emptying of the bladder. False labor contractions are typically felt as a tightening or pulling sensation of the top of the uterus. In contrast, true labor contractions are more commonly felt in the lower back and gradually sweep around to the lower abdomen. 1. Incorrect: True labor includes progressive effacement and dilation. False labor does not significantly change the cervix in effacement or dilation. 3. Incorrect: Effacement and dilation cause expulsion of the mucus plug, rupturing the small cervical capillaries in the process. False labor does not cause effacement or dilation; therefore, there will be no bloody show. 4. Incorrect: True labor contractions tend to increase with walking. False labor contractions do not change or may decrease with activity (such as walking).

The client with ulcerative colitis calls the clinic and reports increasing abdominal pain and increased frequency of loose stools. The client asks the nurse to clarify foods that can be eaten with ulcerative colitis. What foods should the nurse suggest? 1. Dried beans 2. Fish 3. Apples 4. Yogurt 5. Scrambled eggs

2., & 5. Correct: Fish and scrambled eggs are both high in protein and low in fiber. Foods high in fiber are irritating to the GI tract and should be avoided. A food diary is needed to determine triggers for flare-ups. 1. Incorrect: Fiber in the beans will increase motility. 3. Incorrect: Fiber in apple will increase motility. 4. Incorrect: Dairy products should be avoided in times of flare-ups as dairy is often a cause of flare ups. Ulcerative colitis is a type of inflammatory bowel disease (IBD) that causes inflammation and ulcers in the lining of the large intestine, or colon, and rectum. It can cause pain, bloating, and diarrhea during flare-ups, and often few or no symptoms between episodes.There's no single diet that will help everyone with UC. The condition changes over time, so the client will need to be flexible. The key is to find what works. To stay organized, the client should keep a food diary to what was eaten and drank and how it made them feel, both good and bad. When preparing meals the client should remember that a well-balanced diet is high in protein, whole grains, and fresh produce. Some people follow a low-residue diet or low-fiber diet every so often, getting about 10-15 grams of fiber a day. That can help them go to the bathroom less often.

The nurse is caring for a client diagnosed with Addison's disease. Which finding would indicate to the nurse that a client has received excessive mineralocorticoid replacement? 1. Oily skin 2. Weight gain of 4 pounds in one week 3. Loss of muscle mass in extremities 4. Blood glucose of 58 mg/dL 5. Serum potassium of 3.2 mEq

2., & 5. Correct: Remember that aldosterone is a mineralocorticoid, which causes the client to retain sodium and water. Retaining sodium and water will cause the client's weight to increase. Also remember, any sudden gain in weight is due to water retention. Too much aldosterone makes you retain too much sodium and water and lose potassium. Normal potassium is 3.5-5.0 mEq/L, so a lowering of potassium could indicate high levels of aldosterone. 1. Incorrect: Oily skin would be seen with an increase in sex hormones such as testosterone and estrogen. Oily skin is not common with mineralocorticoids like aldosterone. 3. Incorrect: Too many glucocorticoids will cause the breakdown of protein and fat but muscular weakness and increased fatigue is seen with too little mineralocorticoids. 4. Incorrect: Too many glucocorticoids will inhibit insulin, causing the serum blood glucose level to go up. Normal blood glucose is 70-110.

After a thoracotomy, which interventions will the nurse initiate to reduce the risk of acute respiratory distress? 1. Allow 4 hours of rest between deep breathing and coughing exercises. 2. Splint the incision during deep breathing and coughing exercises. 3. Have the client drink a glass of water before coughing. 4. Perform percussion and vibration every 2 hours. 5. Promote incentive spirometer use several times per hour while awake.

2., & 5. Correct: Splinting helps with the ability to control pain and produce an effective cough. Incentive spirometry encourages deep inspiratory efforts, which are more effective in re-expanding alveoli than forceful expiratory efforts. 1. Incorrect: They need to cough more often than every 4 hours. It is the best when this is done every 2 hours. 3. Incorrect: It takes longer than a few minutes to liquefy secretions and, if the stomach is full, vomiting may occur which would put the client at risk for aspiration. 4. Incorrect: After the surgery, we do not want to percuss and vibrate the incision. Besides being extremely painful, this could potentially disrupt the suture line.

The parents of a 2 year old child, diagnosed with autism spectrum disorder (ASD), ask the nurse what led the primary healthcare provider to diagnose this disorder for their child. What behaviors will the nurse indicate as signs of ASD? 1. Delusions 2. Twisting 3. Preoccupation with objects 4. Delayed speech 5. Changes are easily tolerated.

2., 3. & 4. Correct: All are behaviors seen in children with ASD. Additionally, they often do not form interpersonal relationships with others or play well with others. They are usually not socially responsive with eye contact and facial expressions. The language characteristics may be delayed, totally absent, echolalia, unusual vocalizations, immature grammatical structures or idiosyncratic words. Their motor behaviors may include poor coordination, tiptoe walking, peculiar hand movements such as flapping and clapping and stereotypical body movements of rocking, dipping, swaying or spinning. 1. Incorrect: Delusions and hallucinations are not characteristic of ASD. These are seen more in the adult. 5. Incorrect: Changes are met with resistance with ASD. Changes in daily routines or in the child's environment can cause catastrophic reactions. First, think about the child with ASD. Then determine which of the options are behaviors expected with ASD. Option 1: No. Delusions are a symptom of many mental disorders including schizophrenia but are not a behavior indicating a sign of ASD. Option 2: Yes! Twisting is a behavior pattern of ASD. The child performs repetitive movements and moves constantly. Option 3: Good one! The child with ASD is fascinated with the details of an object. The objects may include the wheels on a toy truck. Option 4: Yes, correct. The child with ASD has communication impairments which include delayed speech and they may lose language skills. They might say "I" when they mean "you" or vice versa. Option 5: Oh, no! Even minor changes in routine can be very upsetting! Children with ASD thrive on routine and any change in routine may lead to a tantrum.

A client is transported to the emergency department following a 20 foot fall from a ski lift. The nurse records initial assessment findings on the chart. Based on that data, what actions should the nurse implement immediately? *exhibit (vitals)* BP 90/40; HR 125; RR 30 and labored; + jugular venous distention (JVD) with subcutaneous emphysema noted to right shoulder area. 1. Apply occlusive dressing to chest. 2. Initiate large gauge IV line. 3. Prepare for chest tube placement. 4. Administer high flow oxygen. 5. Position client on right side.

2., 3. & 4. Correct: Based on the assessment data recorded by the nurse, the client most likely has a tension pneumothorax secondary to blunt force trauma from the fall. Immediate actions must focus on preventing tracheal deviation and a fatal outcome. The need for intravenous fluids and medications in any trauma requires at least one large bore IV line or more. This client will need immediate chest tube placement to relieve increasing intrathoracic pressure. While preparing the client for this procedure, high-flow oxygen should be administered via nonrebreather mask because of the client's respiratory distress. 1. Incorrect: There is no indication in the question of an open chest wound, or that a dressing is needed. The occlusive chest dressing will be placed over the insertion site of the chest tube after placement is completed. 5. Incorrect: This trauma client will be secured to a back board, most likely with a cervical collar in place, until x-rays confirm there has not been a cervical spine injury. Placing the client on the right side is counterproductive and in fact could further impair respiratory efforts. 2. Option 1: No need for this. Nowhere in the question is there a reference to any open chest wound from the fall. The use of an occlusive dressing will only be necessary after chest tube placement, to cover the insertion site. 3. Option 2: Very good. This client will actually need several large bore intravenous lines, as do most trauma clients. Basic IV fluids will be needed as well as other medications such as pain meds, antibiotics, or sedation. In some cases, blood may need to be administered; therefore, large gauge IV lines are important. 4. Option 3: Definitely! The client's symptoms indicate a tension pneumothorax, which can be rapidly fatal if the pressure is not relieved immediately. Although a large gauge needle may initially be inserted into the 2nd intercostal space to allow release of pressure, the primary healthcare provider will need to insert a chest tube to prevent death. 5. Option 4: Another good choice. The issue is that the client is in respiratory distress secondary to a tension pneumothorax. Even though this client needs a chest tube placed to relieve the intra-thoracic pressure, the present respiratory distress indicates the need for oxygen which should be administered at 15 liters via a non rebreather mask. 6. Option 5: No way. First, remember that this is a trauma client, most likely secured to a back board with a cervical collar in place. That makes it pretty tough to place the client on the side! Secondly, this client is experiencing respiratory distress with JVD. Positioning on either side will further impede breathing.

The nurse has been working with an attractive teenage girl regarding appropriate nutrition. Which statement by the teenager would support a disturbed body image and the need for education on adequate nutrition? 1. "I am happy my weight is within normal limits. " 2. "I can never exercise enough to lose those saddle bags." 3. "I can always work a little harder on school work and hobbies." 4. "I try to eat only two meals a day to keep my weight down." 5. "I have been trying to include more fruits and vegetables in my diet."

2., 3. & 4. Correct: Compulsive exercising may indicate an eating disorder or a risk for developing one. Perfectionism in school, sports, and hobbies may indicate low self-esteem, which is reflected in eating disorders. Compulsive adherence to routines for weight loss or control may indicate a risk for developing an eating disorder. 1. Incorrect: This would indicate a healthy self-concept. 5. Incorrect: Choosing to eat more fruits and vegetables is a positive, healthy promotion strategy for good nutrition. Read each answer option with the question for continuity of content and context. The question is looking for statements by the client that support a disturbed body image and/or a need for education on nutrition. Does option 1 support a disturbed body image? No, satisfaction with a normal weight demonstrates a positive body image. Does option 2 support a disturbed body image? Yes, she sees saddle bags that she cannot get rid of. What about option 3? Yes, this statement reflects low self-esteem that can lead to eating disorders. Option 4? Yes, Clients should eat three meals a day. Skipping meals is not the healthy way to lose or maintain weight. What about option 5? No, this does not support a disturbed body image or the need for education on adequate nutrition. So the answer to what supports a disturbed body image and need for education on adequate nutrition are options 2, 3, and 4.

As a member of the emergency preparedness planning team at the hospital, which action should the nurse encourage the team to implement? 1. Developing a response plan for each potential disaster. 2. Providing education to employees on the response plan. 3. Practicing the response plan on a regular basis. 4. Evaluating the hospital's level of preparedness. 5. Every hospital should prepare for all the same emergencies.

2., 3. & 4. Correct: Developing a single response plan, educating individuals to the specifics of the response plan, and practicing the plan and evaluating the facility's level of preparedness are effective means of implementing emergency preparedness. The basic principle of emergency preparedness are the same for all types of disasters. Only the response interventions vary to address the specific needs of the situation. 1. Incorrect: One good response plan should be developed rather than multiple plans. This will ensure adequate understanding of the plan and decrease confusion of roles that could occur with multiple plans. 5. Incorrect: Consideration must be given to the proximity of chemical plants, nuclear facilities, schools and areas where large groups gather.

As a member of the emergency preparedness planning team at the hospital, which actions should the nurse encourage the team to implement? 1. Developing a response plan for every potential disaster. 2. Providing education to employees on the response plan. 3. Practicing the response plan on a regular basis. 4. Evaluating the hospital's level of preparedness. 5. Assigning all client care duties to the Nursing Supervisor.

2., 3. & 4. Correct: Developing a single response plan, educating individuals to the specifics of the response plan, and practicing the plan and evaluating the facility's level of preparedness are effective means of implementing emergency preparedness. The basic principles of emergency preparedness are the same for all types of disasters. Only the response interventions vary to address the specific needs of the situation. 1. Incorrect: One good response plan, not multiple plans, should be developed. This will ensure adequate understanding of the plan and decrease confusion of roles that could occur with multiple plans. There is no feasible way for the hospital to have a response plan for every potential disaster. 5. Incorrect: All client care duties cannot safely be assigned to one caregiver. The nursing supervisor needs the help of other staff to carry out nursing care for the clients in the hospital.

A client with a history of myasthenia gravis (MG) has been discharged from the hospital following a thymectomy. When teaching the client how to prevent complications, the home care nurse emphasizes what daily actions are most important? 1. Include daily weight lifting exercises. 2. Practice stress reduction techniques. 3. Complete chores early in the day. 4. Take medications on time and prior to meals. 5. Eat three large meals daily.

2., 3. & 4. Correct: Myasthenia gravis is a chronic autoimmune disorder characterized by progressive muscle weakening and chronic fatigue. Clients become weaker throughout the day, contributing to the potential for complications. Stress reduction techniques are important since stress can contribute to a myasthenic crisis, a severe respiratory emergency. Daily tasks, including ADL's, should be completed early in the day when the client has the most energy. Medications for MG, including neostigmine and pyridostigmine, must be taken on time and prior to meals. 1. Incorrect: Clients with myasthenia gravis are instructed to include gentle daily exercise combined with periods of rest throughout the day. Weight lifting would be too strenuous and would quickly tire this client, possibly leading to a myasthenia crisis. 5. Incorrect: Because of the difficulty in chewing or swallowing, multiple small meals throughout the day are safer and more beneficial to a client with myasthenia gravis. Medications are timed in relation to meals, so consistent but smaller meals would be more beneficial for the client.

The staff nurse is caring for a 3-month old client receiving potassium IV therapy. Which actions indicate to the charge nurse that the staff nurse understands IV management? 1. Uses a 15 gtt factor drip chamber when changing the IV tubing. 2. Applies elbow restraints to prevent dislodgement of the IV catheter. 3. Checks the IV site for blood return hourly. 4. Instructs unlicensed assistive personnel (UAP) to count drip rate hourly. 5. Attaches a volume-controlled IV administration set to IV bag prior to beginning IV therapy

2., 3. & 5. Correct: Young children and infants usually must be restrained to some degree to prevent accidental dislodging of the needle. Elbow restraints can prevent an infant with a scalp IV from rubbing or touching the IV site. When a foot, leg, or arm is used, limb motion must be limited. IV potassium is an irritant. When the fluid being infused is a known irritant or vesicant, the nurse should check the IV site for blood return and possible infiltration hourly. Infants and young children have a narrow range of normal fluid volume, and the risk for fluid overload is great, especially in an infant. Always use a volume-controlled IV administration set with an infant or small child. These sets hold no more than 100-150 mL of fluid, so the maximum amount that could accidentally be infused is limited. 1. Incorrect: Always use microdrip tubing which is a 60 gtt chamber. Microdrip chambers are used for children and for clients who can not tolerate a fast infusion rate or large volumes. 4. Incorrect: This intervention is beyond the scope of a UAP. The UAP may assist with activities of daily living and bedside care under the supervision of a registered nurse or other healthcare professional. The nurse is responsible for monitoring the IV flow rate.

A nurse is participating in a community health fair for middle aged individuals. Which points should the nurse stress for decreasing the risk of stroke? You answered this question 1. Reduce dietary intake of unsaturated fat. 2. Swim or walk most days of the week. 3. Treat obstructive sleep apnea, if present. 4. Drink five or more 8 ounce glasses of water daily. 5. Decrease smoking to less than ½ pack a day.

2., 3., & 4. Correct: Aerobic or cardio exercise reduces risk of stroke in many ways. Exercise can lower blood pressure, increase HDL, lower cholesterol, and improve overall health of blood vessels and heart. Exercise helps in weight loss, controlling diabetes and reduces stress. Obstructive sleep apnea can decrease oxygen to the brain and place you at increased risk of stroke. Water helps to thin blood, which in turn makes it less likely to form clots. Drink water throughout the day, rather than all at once. 1. Incorrect: When choosing fats, pick unsaturated fat over saturated or trans fat. Saturated fat raises total blood cholesterol levels and low density lipoprotein. 5. Incorrect: Tobacco use is a major preventable risk factor for stroke and heart disease. Even if a person has smoked for years, the risk of stroke can be reduced by quitting all smoking.

A client diagnosed with new onset atrial fibrillation has been prescribed dabigatran. What should the nurse teach this client? 1. Place medication in a weekly pill organizer so that medication is not forgotten. 2. Do not take with clopidogrel. 3. Dabigatran decreases the risk of stroke associated with atrial fibrillation. 4. Take this medication with food. 5. aPTT and INR levels will be drawn monthly.

2., 3., & 4. Correct: Do not take dabigatran with any other anticoagulants, including clopidogrel due to increased bleeding risk. Dabigatran decreases the risk of stroke and systemic embolism in clients with atrial fibrillation that is not associated with a cardiac valve problem. Take this medication with food to decrease gastric side effects such as dyspepsia and gastritis. Proton pump inhibitors and histamine 2 recepter blockers may also decrease gastric side effects. 1. Incorrect: After container is opened, medication should be used within 30 days. It is sensitive to moisture and should not be stored in weekly pill organizers. To maintain efficacy, keep medication in manufacturer- supplied bottle. 5. Incorrect: This medication does not require monitoring of aPTT and INR levels. However, the client should be informed about the risk of bleeding and to monitor for signs of bleeding.

What interventions should the nurse initiate while caring for a client who has a cooling blanket in place? 1. Assess temperature every hour. 2. Perform comparison check with another thermometer periodically. 3. Assess client skin condition hourly. 4. Turn blanket off when temperature is at goal temperature. 5. Observe for signs of chilling.

2., 3., & 5. Correct. Perform comparison check with another thermometer periodically to ensure there is no problem associated with equipment failure. For cooling treatments, extended periods of cooling can cause areas of decreased perfusion, skin burns, and tissue injury. Chilling can increase metabolism and body needs. 1. Incorrect: Check the client's temperature every 15 minutes. If the client is cooled too quickly, chilling, increased metabolism, and adverse reactions may occur. 4. Incorrect: The blanket will not immediately return to room temperature and will continue to cool the client even after it is turned off. Turning it off shortly before the goal temperature is achieved will prevent altering the client's core temperature beyond the desired outcome. So the key here is to remember safety. We don't want to drop the temperature too rapidly or cause shivering. Shivering increases body metabolism and elevates body temperature. Option 1, assess temperature hourly. What do you think? That is a long time. Could the temperature drop rapidly within that time? Yes. So this is false. Monitor vital signs and neurologic status every 5 minutes until the desired body temperature is reached and then every 15 minutes until temperature is stable or as prescribed. Option 2, Perform comparison check with another thermometer periodically. Remember safety. What if the rectal thermometer probe is not accurate? So compare temp results with another thermometer. True statement. Option 3, assess client skin condition hourly. True. Observe for color changes in skin, lips, and nail beds and for edema, induration, inflammation, pain and sensory impairment. If they occur, discontinue the procedure and notify the primary healthcare provider. Option 4, Turn blanket off when temperature is at goal temperature. This is false. If you wait until the temperature goal is reached to turn it off, the client's temperature will continue to drop because the blanket will still be cool. Body temperature can fall as much as 5º F or 2.8º C after the machine is turned off. Option 5, observe for chilling. True. Shivering increases body metabolism and elevates body temperature.

What interventions should the nurse plan to implement when admitting a client diagnosed with measles? 1. Admit to a semi-private room with a client diagnosed with tuberculosis (TB). 2. Place a surgical mask on the client when transferring to x-ray. 3. Initiate airborne precautions. 4. Wear surgical mask when entering the client's room. 5. Assign a nurse who has received the measles vaccine to take care of this client.

2., 3., & 5. Correct: If the client must leave the room, a surgical mask should be worn to prevent transmission to others. Measles can be transmitted via contact, droplet, and airborne methods, so airborne precautions are needed. Healthcare providers who are not immune to measles should not care for a client with measles. 1. Incorrect: The client should be placed in a private room with negative air pressure when airborne precautions are necessary. TB and measles are not like illnesses and should not be placed in the same room. 4. Incorrect: A particulate or N95 respirators should be worn by staff entering the room of a client on airborne precautions. N95 respirators filter particles that you may inhale. A surgical mask prevents the spread of particles during exhalations.

The nurse is teaching a client, recovering from a myocardial infarction (MI), about the prescribed diet of low sodium, low saturated fat, and low cholesterol. Which statements, if made by the client, would indicate to the nurse that teaching has been successful? You answered this question 1. "I should drink fruit juices rather than soft drinks." 2. "A good snack to eat would be unsalted popcorn." 3. "When making homemade tomato sauce, I should not add salt." 4. "I should use 2% milk when cooking." 5. "There is no restriction on egg white consumption."

2., 3., & 5. Correct: Popcorn without salt is a healthy snack choice for clients on a low sodium, low saturated fat, and low cholesterol diet. Homemade tomato sauce can be made without adding salt. The American Heart Association no longer makes recommendations on how many egg yolks to eat or not to eat. A good, general guideline is to eat no more than 1 egg yolk a day, up to 5 total a week. There is no restriction on egg whites (including those used in baking and cooking). 1. Incorrect: Consume whole vegetables and fruits rather than fruit juices. Fruit juices have added sugars and lack fiber. 4. Incorrect: Use skim or 1% milk rather than 2% or whole milk. Clients who have had an MI are advanced to a low sodium, low saturated fat, and low cholesterol diet. Fat intake should be about 30% of calories, with most coming from mono and polyunsaturated fats. Red meat, egg yolks, and whole milk products are major sources of saturated fat and cholesterol and should be reduced or eliminated. Clients with CAD are encouraged to take omega 3 fatty acid supplements. Recommended daily intake of cholesterol is less than 200 mg/day. Popcorn without salt is a healthy snack choice for clients on a low sodium, low saturated fat, and low cholesterol diet. Popcorn is a food coming from whole grain corn, which is encouraged on this diet. Other foods made of whole wheat, oats/oatmeal, rye, barley, brown rice and wild rice are good choices. Homemade tomato sauce can be made without adding salt. The American Heart Association no longer makes recommendations on how many egg yolks to eat or not to eat. A good, general guideline is to eat no more than 1 egg yolk a day, up to 5 total a week. There is no restriction on egg whites (including those used in baking and cooking).

The nurse is cleaning and dressing a foot ulcer of a diabetic client. Which actions are appropriate? 1. Uses a clean basin and washcloth to clean the ulcer. 2. Wears sterile gloves to clean the ulcer. 3. Cleans ulcer with normal saline. 4. Warms saline bottle in microwave for 1 minute. 5. Cleans ulcer in a full circle, beginning in the center and working toward the outside.

2., 3., & 5. Correct: The nurse needs to wear sterile gloves when cleaning the wound. Normal saline solution is the preferred cleansing agent because, as an isotonic solution, it doesn't interfere with the normal healing process. Gently clean the wound in a full or half circle, beginning in the center and working toward the outside. 1. Incorrect: Sterile supplies should be used with this procedure because the client is at risk for infection and gangrene. Gauze and salve should be used instead of a wash cloth. 4. Incorrect: Before you start, make sure the cleansing solution is at room temperature. Do not heat in the microwave. It could scald the client! Option 1 is false. Because the client has diabetes, the client is at risk for infection and gangrene. The nurse needs to wear sterile gloves when cleaning the wound. Sterile supplies should be used with this procedure. Gauze and prescribed cleaning solution should be used instead of a wash cloth. Option 2 is true. The nurse needs to wear sterile gloves when cleaning the wound. The sterile gloves are to protect the client from acquiring an infection in the foot ulcer. Option 3 is true. Normal saline solution is the preferred cleansing agent because as an isotonic solution, it doesn't interfere with the normal healing process. Option 4 is false. Remember...Client safety is always a priority. It is important for the cleansing solution to be at room temperature. Do not heat in the microwave. It could burn the client! Option 5 is ​true. Gently clean the wound in a full or half circle, beginning in the center and working toward the outside. A wound is to be cleaned from the least contaminated area such as from the wound to the surrounding skin.

Which assignment by the charge nurse would be most appropriate for a general pediatric nurse being reassigned to the hematology/oncology pediatric unit? 1. Child dying with leukemia who has been on the hematology/oncology unit for two weeks. 2. Teenager with sickle cell disease in for pain management. 3. Child admitted following a bicycle accident that has idiopathic thrombocytopenic purpura (ITP). 4. New admit scheduled for bone marrow transplant. 5. Child diagnosed with leukemia admitted for stomatitis.

2., 3., & 5. Correct: The nurse should be given an assignment similar to the type of clients and skill level the nurse is accustomed to on the general pediatric unit. Therefore, the choices should be these three clients. Even though one of the clients has leukemia, the child is being treated for stomatitis, not the leukemia. Sickle cell clients are frequently cared for on general pediatric units. The reassigned nurse has the knowledge and skills needed to meet the clients needs for pain management and treatment for the sickle cell disease. The general pediatric nurse should be competent in caring for children with low platelet counts, so the child with ITP could be assigned to this nurse. The nurse would be familiar with bleeding precautions, monitoring for bleeding, and associated care. 1. Incorrect: This client is dying with leukemia and needs consistency in the staff assigned to care for them. Although the general pediatric nurse could competently care for a dying child, the focus should be on the client. This child needs and deserves consistent care and care by those that are familiar to this child. 4. Incorrect: A child who is to receive a bone marrow transplant would not be the best assignment, since the nurse must have special preparation and an understanding of the protocol with a bone marrow transplant client. This is not something that a general pediatric nurse would typically do. Therefore, this client would need to be cared for by the nurses on the hematology/oncology unit who has this special training and/or knowledge.

The nurse is planning care for the prevention of skin breakdown in a client diagnosed with a stroke. What intervention is important for the nurse to include? 1. Massage reddened skin areas located over bony prominences. 2. Place pillows under lower extremities to raise heels off the bed. 3. Position client on paralyzed side for one hour. 4. Apply emollients to dry skin. 5. Place a gel seat cushion on the wheelchair seat. 6. Shift client weight every two hours while sitting in a wheelchair.

2., 4., & 5. Correct: These interventions will decrease the risk of skin breakdown by eliminating sustained pressure to areas at greatest risk of breakdown. 1. Incorrect: Do not massage the damaged area because this may cause additional damage. 3. Incorrect: This is way too long. The client should only be on their paralyzed side for 30 minutes. 6. Incorrect: This is way too long. Skin breakdown can result within this period of time. The client's weight should be shifted within the wheelchair every 15-20 minutes.

A client who has Parkinson's disease has a new prescription for benztropine. What should the nurse include when teaching the client and spouse about this medication? 1. This medication blocks dopamine in the brain to decrease tremors and muscle stiffness. 2. Notify your primary healthcare provider if you develop urinary retention. 3. Benztropine can reduce the ability to sweat, so do not become overheated. 4. No lab tests are needed while taking this medication. 5. Sit up or stand up slowly to prevent lightheadedness.

2., 3., & 5. Correct: Urinary retention is a side effect of benztropine. Benztropine can reduce the ability to sweat and cause the body to overheat. Do not become overheated in hot weather or while you are being active because heatstroke may occur. Benztropine may cause dizziness, lightheadedness, or fainting. Alcohol, hot weather, exercise, or fever may increase these effects. To prevent these negative effects, sit up or stand slowly, especially in the morning. Sit or lie down at the first sign of any of these effects. 1. Incorrect: Benztropine is an anticholinergic. It works by decreasing the effects of acetylcholine, a chemical in the brain. This results in decreased tremors or muscle stiffness, and helps improve walking ability for clients with Parkinson's disease. 4. Incorrect: Lab tests, including liver function, kidney function, lung function, blood pressure, fasting blood glucose, and blood cholesterol, may be performed while using benztropine. These tests may be used to monitor the client's condition or check for side effects.

An elderly client living in a long-term care facility fell 8 hours ago causing a laceration on the occipital area of the skull and steri-strips were applied for closure. Which signs/symptoms would indicate to the nurse that the client should be transferred to the emergency department? 1. Purposeful movement. 2. Sudden emotional outbursts. 3. Client report of blurred vision. 4. Pupils equal, react to light, and accommodation. 5. Bright red blood oozing from the wound. 6. Headache unrelieved by acetaminophen.

2., 3., & 6. Correct: Signs/symptoms of increased ICP include: excessive sleepiness, inattention, difficulty concentrating, impaired memory, faulty judgment, depression, irritability, emotional outbursts, disturbed sleep, diminished libido, difficulty switching between two tasks, and slowed thinking. Abnormalities in vision and extraocular movements occur in the early stages of increased ICP. A headache that is unrelieved by acetaminophen would warrant further investigation. 1. Incorrect: This is a normal response and does not warrant further investigation. 4. Incorrect: This is a normal response and does not warrant further investigation. 5. Incorrect: The scalp is very vascular and oozing would be expected. Apply pressure to stop bleeding. An elderly client has fallen and hit the back of the head hard enough to cause a laceration. What is the number 1 thing you should be worried about? Increased intracranial pressure, right? Right. Now that you know what you are worried about, you know you are looking for signs and symptoms that would indicate an increasing ICP. Option 1, purposeful movement. That is a good thing. We want the client to have purposeful movements. So this is not an indication of ICP. Option 2, sudden emotional outbursts. True. Mood and emotional changes can be an indication of brain injury. Option 3, reports of blurred vision. True. Vision changes indicate pressure on the optic nerve. Option 4. PEARLA. Normal. So this is false. Option 5, bleeding from the laceration does not mean that the client has ICP. Scalp wounds can bleed a lot because the scalp is very vascular. Option 6, headache unrelieved by acetaminophen. True. A worsening headache may mean ICP.

What should a nurse teach a group of teenage boys who admit to using smokeless tobacco? 1. Smokeless tobacco increases risk for lung cancer. 2. Inspect mouth frequently for lesions. 3. White patches in mouth should be reported to healthcare provider. 4. Risk for stomach cancer can be decreased by not swallowing smokeless tobacco juice. 5. Report decreased saliva to primary healthcare provider. 6. Smoking cessation.

2., 3., & 6. Correct: The mouth should be inspected frequently for painless lesions that do not heal. This may be a sign of oral cancer and should be reported to the primary heathcare provider. White patches (leukoplakia) is a sign of potential oral cancer as well. Nicotine is addictive and is found in smokeless tobacco. Clients using smokeless tobacco can benefit from smoking cessation information/classes. 1. Incorrect: Use of smokeless tobacco increases the risk developing of esophageal cancer, cancers of the mouth, throat, cheek, gums, lips, tongue, pancreatic cancer, stomach cancer, kidney cancer. 4. Incorrect: This is an incorrect statement. Some amount of tobacco juice will be swallowed and can lead to esophagus and stomach cancer. 5. Incorrect: Decreased saliva is not associated with oral cancer.

The home care nurse, working with an infant in the home, is concerned about the infant developing diaper rash from wearing cloth diapers. Which strategies should the nurse teach to the parents to prevent skin irritation? 1. Change diapers every four hours. 2. Wash diapers with hypoallergenic detergent. 3. Rinse diapers twice when washing. 4. Apply a protective ointment to diaper area with each diaper change. 5. Check infant at least hourly for wet or soiled diapers.

2., 3., 4. & 5. Correct: Hypoallergenic detergent will remove skin bacteria as well as urine from the diaper. Detergents can be irritating to the skin and may cause dryness; therefore, adequate rinsing is important. Double rinse the diapers in cold water to remove traces of chemicals and soap. A protective ointment is even more important to use with cloth diapers, as they do not have the same wicking properties of the disposable diapers. Frequently checking the diaper for wetness and soiling will limit the contact time for urine or feces to be in contact with the skin. Whether using cloth diapers, disposables or both kinds, always change the baby as soon as possible after wetting or soiling the diaper to keep the bottom as clean and dry as possible. 1. Incorrect: The child should be checked more frequently to prevent irritation to the skin from soiling. At least every 2 hours is recommended.

The nurse is talking with the mom of a preschooler at the well-child visit. The mom reports that her 3 year old has a lot of energy and sleeps 9 hours per night. What assessment questions should the nurse ask in response to this comment? 1. Nothing, as this is normal for preschoolers. 2. Does your child take naps during the day? 3. Does your child wake up spontaneously or do you wake her? 4. Does your child appear rested upon awakening? 5. Does your child have trouble settling down for sleep?

2., 3., 4. & 5. Correct: Preschoolers typically require 11 - 13 hours of sleep per day. The child may be supplementing nighttime sleep with long naps. It is important to determine if the child has to be awakened after nine hours or if the child awakens spontaneously. The child may have to be awakened due to mom's work schedule. The adequacy of rest should be determined, as the child is sleeping less than is typical. The nurse should determine if the child has difficulty falling asleep. If so, perhaps more restful nighttime rituals should be implemented. 1. Incorrect: Preschoolers typically require 11-13 hours of sleep per day. Nine hours is not enough.

The nurse should assess for what signs of toxicity in a child who is admitted with salicylate overdose? 1. Hypoventilation 2. Vomiting 3. Tinnitus 4. Diaphoresis 5. Dehydration 6. Hypothermia

2., 3., 4., & 5. Correct: Nausea and vomiting are the most common toxic effects. This can be caused by CNS toxicity or by direct damage to the gastric mucosa. Salicylates can be neurotoxic, and this is manifested by ringing in the ears. Ototoxicity can also lead to hearing loss. Diaphoresis results in the early phase of toxicity. Serious dehydration can result from insensible losses due to hyperventilation and fever, as well as active losses due to vomiting. 1. Incorrect: The first phase of salicylate toxicity is characterized by hyperventilation due to stimulation of the respiratory center in the brain. This is a key feature of salicylate toxicity. 6. Incorrect: Hyperpyrexia is an indication of severe toxicity, especially in younger children.

What signs or symptoms should the nurse assess for when monitoring a client who has a brain injury? 1. Increased pulse 2. Rhinorrhea 3. BP 150/60 4. Papilledema 5. Projectile vomiting

2., 3., 4. & 5. Correct: Rhinorrhea is an important symptom that is associated with leakage of cerebrospinal fluid (CSF) The pulse pressure of 150/60 is 90 {greater than 40 is a sign of increased intracranial pressure (ICP)}. Signs of increased intracranial pressure also include papilledema, elevated systolic pressure, widened pulse pressure, decreased pulse, and slow respirations. Projectile vomiting is classically associated with increased ICP. 1. Incorrect: The major focus of a client with a brain injury is increased ICP, which is associated with bradycardia. Your client has a brain injury. What is your major concern? Increasing intracranial pressure, right? Yes. Do you know the signs/symptoms of IICP? The most common symptoms of an ICP are headache, papilledema, blurred vision, vomiting, changes in behavior, decreasing level of consciousness, weakness or problems moving or talking. Cushing's Triad is an important, but late sign of raised intracranial pressure. Cushing's Triad is a set of three primary signs: 1. A change in respirations, often irregular and deep, such as cheyne stokes 2. A widening pulse pressure (the difference between the Systolic and the Diastolic BP) 3. Bradycardia. What options match what we know about s/s of IICP? Did you say options 3, 4, and 5? Good. The pulse pressure of 150/60 is 90 {greater than 40 is a sign of increased intracranial pressure (ICP)}. Signs of increased intracranial pressure also include papilledema, elevated systolic pressure, widened pulse pressure, decreased pulse, and slow respirations. Projectile vomiting is classically associated with increased ICP. What about option 2: Rhinorrhea? This is true. CSF rhinorrhea is a rare but potentially devastating condition that can lead to significant morbidity and mortality for the patient. Disruption of the barriers between the sinonasal cavity and the anterior and middle cranial fossae is the underlying factor leading to the discharge of CSF into the nasal cavity. So that leaves option 1: Increased pulse. This is false. Bradycardia is an indication of ICP.

What should a nurse include when preparing to educate a female client on how to prevent recurrent cystitis? 1. Drink at least eight, 4 ounce glasses of water per day. 2. Urinate as soon as the urge occurs. 3. Avoid irritating perineum with harsh soap. 4. Empty your bladder post coitus. 5. Avoid use of a diaphragm.

2., 3., 4. & 5. Correct: The client should void as soon as the urge occurs and completely empty the bladder. The client should not use excessive soap or harsh soap to minimize irritation of the urethra. Women should void immediately after sexual intercourse. This helps prevent bacteria from moving into the urethra. Pressure on the urethra may cause irritation and urinary tract infections in women who use diaphragms. 1. Incorrect: A client with normal renal function and who does not have heart or kidney disease needs to drink 2200 to 2700 mL of fluid daily. Increasing fluid intake helps flush out solutes or particles that collect in the urinary system.

Which signs, if observed in a child, should a clinic nurse associate with Kawasaki disease? 1. Productive cough 2. Strawberry tongue 3. High and persistent fever 4. Enlarged cervical lymph nodes 5. Erythema of the palms and soles

2., 3., 4. & 5. Correct: The nurse should recognize strawberry tongue, high and persistent fever, enlarged cervical lymph nodes, and redness of the palms of the hands and soles of the feet as signs of Kawasaki disease. Kawasaki disease is an autoimmune disease in which the medium-sized blood vessels throughout the body become inflamed. Many organ systems, mainly those including the blood vessels, skin, mucous membranes, and lymph nodes, are affected. The most prominent signs are a high and persistent fever that is not responsive to normal treatment with acetaminophen or ibuprofen, extreme irritability, and the presence of a "strawberry tongue" caused by necrotizing microvasculitis. 1. Incorrect: Coughing is not a typical sign of Kawasaki disease.

A primary healthcare provider has prescribed restraints for a violent client. Which measures would the nurse provide as proper interventions for this client? 1. Observe the client in restraints every hour. 2. Ensure that circulation to extremities is not compromised. 3. Assist client with needs related to nutrition and elimination. 4. Provide help with personal hygiene. 5. Conduct a staff debriefing after the incident.

2., 3., 4. & 5. Correct: These are correct interventions for safety when a violent client requires restraints. When applying restraints you do not want the restraint so tight that extremity circulation is diminished. The client must still be provided with proper nutrition, hydration, and allowed to go to the restroom. If the client is restrained, the client will need help with basic care and comfort measures. It is essential that staff debrief after such an event to see what might have been done differently to de-escalate the client's violence. 1. Incorrect: The client in restraints should be observed every 15 minutes. Safety of the client is extremely important. Physical needs, such as food and toileting, should also be addressed.

The nurse is planning discharge teaching for a client with thrombocytopenia. Which should the nurse include? 1. Floss between teeth daily. 2. Eat soft foods. 3. Take docusate sodium daily to prevent straining 4. Wear well fitted shoes while ambulating. 5. Apply a cool compress to site with any soft tissue trauma.

2., 3., 4. & 5. Correct: Thrombocytopenia is a deficiency of platelets, and platelets help your blood clot which stops bleeding. Hard food can cause bleeding as it passes through the esophagus and can cause gums to bleed. A stool softener should be taken daily to prevent a hard stool. Straining and hard stools can lead to tissue trauma and bleeding from the rectum. Well fitted shoes can prevent injury while ambulating. Cool compress will prevent hematoma formation and stop bleeding. 1. Incorrect: This client should not floss the teeth, as bleeding can result. Soft bristle toothbrush will be needed also to prevent injury to gums.

The nurse is making a home assessment for the purpose of preventing injury for a visually impaired elderly client who also has diabetes. Which findings are important for the nurse to include in this assessment? 1. Episodes of mild anxiety 2. Rugs secured to the floor 3. Adequate lighting 4. Functional eye glasses 5. Client is wearing well-fitting closed toe shoes

2., 3., 4. & 5. Correct: Throw rugs may cause falls, as the client may trip. Diabetes may also impact sensation to know there is a rug in place. Adequate lighting is important so that the client does not bump into furniture or miss a step when navigating stairs. The visually impaired client needs functioning glasses to maximize sight and safety within the home. Diabetic clients do not need open toed shoes, as injury may occur to the foot and the client may not actually be aware of it. Also, wearing sandals is a risk to the person who is visually impaired and elderly, as the shoe may come off unexpectedly. 1. Incorrect. While depression is common and may result in self harm for elderly clients, anxiety is not likely to result in injury. Depression assessment should be performed on all elderly clients.

A nurse is helping a client to maintain normal voiding habits while recovering from a cesarean section. What methods should the nurse initiate? 1. Have the client recline slightly while using bedside commode. 2. Encourage the client to push over the pubic area with hands. 3. Suggest the client read or listen to music. 4. Pour warm water over the perineum. 5. Stay and talk with client while waiting for urge to void.

2., 3., 4. Correct: Encourage the client to push over the pubic area with the hands or lean forward to increase intraabdominal pressure and external pressure on the bladder. Reading or listening to music will help to decrease anxiety and tension. Pouring warm water over the perineum promotes muscle relaxation. 1. Incorrect: Assist the client to a normal position for voiding. For males, standing. For females, squatting or leaning slightly forward when sitting. These positions enhance movement of urine through the tract by gravity. 5. Incorrect: Provide privacy. Many people cannot void in the presence of another person.

What should a nurse include when teaching a client diagnosed with shigellosis regarding how to prevent the spread of the infection to others? 1. Wash hands three times a day with alcohol. 2. Do not return to work until authorized by local health department. 3. Do not prepare food for others while you are sick. 4. Avoid swimming until fully recovered. 5. No sex until several days after diarrhea has stopped.

2., 3., 4., & 5. Correct. These are correct actions to prevent the spread of infection. Shigella germs are present in the stools of infected persons while they have diarrhea and for up to a week or two after the diarrhea has gone away. Shigella is very contagious; exposure to even a tiny amount of contaminated fecal matter—too small to see-- can cause infection. Transmission of Shigella occurs when people put something in their mouths or swallow something that has come into contact with stool of a person infected with Shigella. 1. Incorrect. Wash your hands with soap carefully and frequently, especially after using the toilet. Option 1 says to wash hands three times a day with alcohol.​ This is false. Soap and water is needed, not alcohol. And is three times a day sufficient? No. Hands should be wash after going to the bathroom and prior to eating. Option 2: Do not return to work until authorized by local health department. This is true. Shigella is present in the stools of infected persons while they have diarrhea and for up to a week or two after the diarrhea has gone away. It is very contagious; exposure to even a tiny amount of contaminated fecal matter—too small to see-- can cause infection. Option 3: Do not prepare food for others while sick. This is true. Transmission of Shigella occurs when people put something in their mouths or swallow something that has come into contact with stool of a person infected with Shigella. Option 4: Avoid swimming until fully recovered. What do you think? True. Swallowing recreational (for example lake or river water while swimming) or drinking water that was contaminated by infected fecal matter can transmit this disease. Option 5: No sex until several days after diarrhea has stopped. True. Avoid sexual activity with those who have diarrhea or who recently recovered from diarrhea. It only takes a small amount of fecal material (not seen by the eye) to transmit the disease.

The nurse is caring for a trauma client who is receiving a unit of whole blood. The client begins to experience lower back pain. What actions should the nurse take? 1. Administer diphenhydramine. 2. Collect a urine specimen. 3. Stop the transfusion. 4. Take the client's vital signs. 5. Change the IV tubing

2., 3., 4., & 5. Correct: Assume the worst, and stop the transfusion first, then continue with the assessment. Low back pain is a sign of an acute hemolytic reaction. This is the most dangerous and potentially life-threatening type of transfusion reaction. It occurs when the donor blood is incompatible with that of the recipient. Get lab tests such as a urinalysis to check for presence of hemoglobin, which indicates hemolytic reaction. Take vital signs. Change IV tubing to remove all blood and maintain the IV line with normal saline solution, with new IV tubing, at a slow rate. 1. Incorrect: Diphenhydramine is indicated for an allergic reaction to the blood component being transfused. It is not indicated for a hemolytic reaction.

What dietary information should the nurse provide to a client diagnosed with Celiac disease? 1. "The most cost effective way to follow the lactose free diet is to eat more fruits and vegetables." 2. "Creamed based canned soups are a source of hidden wheat." 3. "You can eat foods containing fax, corn, or rice." 4. "Avoid foods and beverages that contain malt." 5. "Do not eat traditional wheat products such as pasta."

2., 3., 4., & 5. Correct: Soups and sauces are one of the biggest sources of hidden gluten, as many companies use wheat as a thickener. It is always a good idea to read the label of any pre-prepared or canned soups and sauces, paying special attention to those that are cream based. Grains that are naturally gluten free include rice, corn, potato, quinoa, kasha, flax, and nut flours. Malt flavoring or extract, which contains gluten may be found in cornflakes and puffed rice cereal. It is also found in beers, ales, and malt vinegars. As a rule, traditional wheat products such as pastas, breads, crackers, and other baked goods are not gluten-free. However, there are many gluten-free options available that use alternative flours and grains. 1. Incorrect: The client who has Celiac disease is prescribed a gluten free diet rather than a lactose free diet.

A nurse receives a client in the post anesthesia care unit following application of a long leg cast to the left leg due to a fractured tibia and fibula. Which interventions should the nurse initiate? 1. Elevate foot of bed 30 degrees. 2. Palpate bilateral pedal pulses. 3. Apply ice packs to fracture site. 4. Mark break through bleeding. 5. Assess client's ability to move

2., 3., 4., & 5. Correct: The priority nursing assessment focuses on any intervention that maintains good circulation to the extremity and prevents complications that can impair mobility. This must include checking distal pulses in both legs to compare the strength of the pulse on both the affected and unaffected side. The nurse should also decrease swelling and risk of compartment syndrome by applying ice to fracture site, assess for bleeding, and check for tingling, coldness, numbness, and ability to move toes; in other words - neurovascular/sensation checks. 1. Incorrect: The affected leg should be elevated, but not both. The nurse should place the affected leg on a pillow and not raise the foot of the bed since this would raise both extremities.

What interventions should the nurse include when planning care for a client post heart transplant? 1. Place on airborne precautions. 2. Instruct visitors to wash hands prior to entering the room. 3. Maintain strict aseptic technique. 4. Initiate pulmonary hygiene measures. 5. Provide for early ambulation.

2., 3., 4., & 5. Correct: The transplant recipient is at high risk for infection due to the suppression of the body's normal defense mechanisms. All of these interventions decrease the incidence of the client developing an infection. The heart transplant client is prescribed medications to reduce the risk of organ rejection by inhibiting or suppressing the immune system. Handwashing is the main defense against infection. Pulmonary hygiene measures are are implemented to maintain open airways and prevent respiratory infections. The pulmonary measures can include oral hygiene, deep breathing exercises, mucus-controlling agents, and intermittent positive-pressure breathing. Pulmonary hygiene helps to decrease the development of pneumonia. Early ambulation helps increase general strength and lung expansion. Also ambulation increases circulation,peristalisis,and joint mobility, Emotionally ambulation improves self-esteem and feelings of independence. 1. Incorrect: The client needs to be protected from everyone else so a private room and protective isolation are needed. All persons entering the room must wash hands well and wear a mask and gloves. The client must wear a mask when leaving the room.

A nurse is planning an educational session on fluticasone/salmeterol for a group of clients who have been prescribed this medication. What teaching points should the nurse include? 1. Swallow the capsule when having an acute asthma episode. 2. Rinse mouth after medication administration to decrease infection. 3. Take this medication every day, even on days when breathing fine. 4. Administer by HandiHaler DPI, twice daily. 5. Carry a rescue inhaler, such as albuterol, when leaving home.

2., 3., 4., & 5. Correct: This medication contains a steroid which can increase the risk of oropharyngeal fungal infections. Rinsing will also decrease mouth and throat irritation. Medication should be taken every day as directed, even on days when client feels they are breathing better. This is a preventative medication not a rescue medication. This medication is administered by an inhaler. It is not given orally. A rescue inhaler, such as albuterol, is needed when the client leaves home. Fluticasone/salmeterol is not a rescue inhaler but for long term control and maintenance treatment for the prevention of bronchospasm and airway inflammation associated with asthma, chronic bronchitis, and COPD. 1. Incorrect: Medication must be taken with an inhaler. Capsules and tablets are not to be swallowed. Teach client how to use a DPI for medication administration.

The nurse is teaching a newly diagnosed diabetic client about self-injection of insulin. Which statement made by the client indicates to the nurse that teaching has been effective? 1. "The abdominal site is best because it is closest to the pancreas." 2. "I can reach my thigh the best, so I will use different areas of the same thigh." 3. "By rotating the sites within one area, my chances of having tissue changes are less." 4. "If I change injection sites from the thigh to the arm, the rate of absorption will be different." 5. "I should inject at least 1-2 inches away from the last injection site."

2., 3., 4., & 5. Correct: To promote consistency in insulin absorption, the client should systematically rotate injection sites within an anatomic area to prevent lipodystrophy. Four main areas for injection are the abdomen, upper arms, thighs and hips. The client should try not to use the exact same site more than once in 2 to 3 weeks. If insulin is injected where there is more fat underneath the skin, insulin may be absorbed more slowly. Also, insulin should not be injected into the limb that will be exercised; absorption will be faster, increasing risk of hypoglycemia. The client should avoid using the exact same site more than once in 2 to 3 weeks. 1. Incorrect: The diabetic client should rotate sites within the same area before moving to a new area. This will assist in preventing lipodystrophy. Use of the abdominal site has nothing to do with being close to the pancreas. The abdomen is the preferred site because it provides the most rapid insulin absorption.

A nurse is participating in a cancer risk screening program. Which signs/symptoms would indicate to the nurse that a client needs further investigation? 1. Unexplained weight gain of 10 pounds 2. Leukoplakia 3. Prolonged hoarseness 4. Hematuria 5. Persistent abdominal bloating

2., 3., 4., & 5. Correct: White patches inside the mouth or white spots on the tongue may be leukoplakia, which is a pre-cancerous area that is caused by frequent irritation. It is often caused by smoking or other tobacco use. People who smoke pipes or use oral or spit tobacco are at high risk for leukoplakia. If untreated, it can become mouth cancer. A cough that does not go away and prolonged hoarseness may be a sign of cancer. Hematuria may be a sign of bladder or kidney cancer and needs further investigation. Although women may experience bloating with changes in the menstrual cycle, constant bloating should be investigated to rule out ovarian cancer. 1. Incorrect: Unexplained loss of weight or loss of appetite may indicate some types of cancer. Weight gain is not typically associated with cancer.

Which interventions should the nurse include for a client with sickle cell crisis who is experiencing pain? 1. Apply cold compresses to affected joints. 2. Massage affected areas gently. 3. Support and elevate swollen joints. 4. Monitor pain level by looking for BP, respiratory, and heart rate elevation. 5. Place client on Nothing By Mouth (NPO) status. 6. Administer Normal Saline (NS) at 125 mL/hour.

2., 3., 4., & 6. Correct: Apply local massage gently to affected areas to help reduce muscle tension. Massage also warms the area and promotes vasodilation. Supporting and elevating affected joints will decrease swelling, thus decreasing pain. Physiological manifestations of vital signs aid in evaluation of pain and effectiveness since pain is unique to every person. The nurse should also assess pain with an objective scale by having the client rate the pain 1-10.​ The NS provides hydration and is appropriate in sickle cell crisis. ​Hydrating the client makes it easier for the abnormally formed RBCs to pass through vessels. 1. Incorrect: Apply warm, moist compresses to affected joints or other painful areas. Avoid use of ice or cold compresses. Warmth causes vasodilation and increases circulation to hypoxic areas. Cold causes vasoconstriction and compounds the crisis. 5. Incorrect: The sickle cell client should not be kept from eating or drinking. Hydration is an important component of therapy. Dehydration promotes sickling process.

Which signs/symptoms should the nurse assess for when caring for a client diagnosed with bulimia nervosa? 1. Increased thirst 2. Muscle cramps 3. Blurred vision 4. Tingling of lips 5. Constipation

2., 4., 5. Correct: The typical abnormalities associated with bulimia are hypokalemia and metabolic alkalosis because of the binging and purging process. This leads to muscle cramps, weakness, fatigue, constipation, and arrhythmias are all symptoms of this electrolyte and acid-base imbalance. Hypokalemia leads to metabolic alkalosis. 1. Incorrect: Increased thirst is a sign of hyperglycemia and would not be the concern with someone that is purging. This client would be more likely to be hypoglycemic instead. 3. Incorrect: Blurred vision is a sign of hyperglycemia because of the effect of too much glucose in the small vessels of the eye. Microvascular damage is one of the biggest concerns with hyperglycemia; the bulemic client would be hypoglycemic.

Which signs/symptoms noted by the nurse would support a client history of chronic emphysema? 1. Atelectasis. 2. Increased anteroposterior (AP) diameter. 3. Breathlessness. 4. Use of accessory muscles with respiration. 5. Leans backwards to breathe. 6. Clubbing of fingernails

2., 3., 4., & 6. Correct: Emphysema is described as a permanent hyperinflation of lung beyond the bronchioles with destruction of alveolar walls. Airway resistance is increased, especially on expiration. Inspection reveals dyspnea on exertion, barrel chest (anteroposterior diameter is equal to transverse diameter), tachypnea, and use of accessory muscles with respiration. Clubbing of fingernails is due to chronically decreased oxygen levels. 1. Incorrect: Atelectasis is collapse of alveolar lung tissue, and findings reflect presence of a small, airless lung. This condition is caused by complete obstruction of a draining bronchus by a tumor, thick secretions, or an aspirated foreign body, or by compression of lung. 5. Incorrect: Client tends to lean forward (orthopnea) and uses accessory muscles of respiration to breathe. Look at the options now. Option 1: Atelectasis. Atelectasis is the collapse of part or, much less commonly, all of a lung. So this is false. Option 2: Increased AP diameter. This is true. It occurs because of chronically hyperinflated lungs and loss of lung elasticity. Option 3: Breathlessness is true. We just said that emphysema destroys the alveoli, making it progressively difficult to breath. ​ Look at option 4, use of accessory muscles to breathe. True. Accessory muscles of respiration are typically only used under conditions of high metabolic demand (e.g. exercise) or respiratory dysfunction (e.g. an asthma attack, or emphysema). Option 5: Lean backwards to breathe. This is false. This client tends to lean forward (orthopnea) and uses accessory muscles of respiration to breathe. Now option 6. Clubbing of fingernails. True. This is due to chronically decreased oxygen levels. Think about it, the alveoli are destroyed so gas exchange is impaired. Oxygen levels decrease.

What assessments would be appropriate for the school nurse to perform related to school safety practices and emergency preparedness? 1. Teach about gun control laws. 2. Observe for gaps or changes in levels of sidewalks. 3. Identify which students have special healthcare needs. 4. Locate all entrances and exits to buildings. 5. Identify threats and hazards in the school and surrounding community. 6. Perform a check of all fire extinguishers.

2., 3., 4., 5., & 6. Correct: One of the first things that a school nurse should do is to assess where an accident might happen. Observing for gaps or changes in the level of sidewalks is an example of this assessment. The school nurse should assess for special healthcare needs in the event that the school enters a time of extended lockdown. Some students would require attention during the time of lockdown, such as diabetics who could not wait to receive insulin or have food available. All entrances to the schools must be identified to know where a potential entry for intruder might could occur. Some access points may need to be changed to reduce risk to students. Becoming familiar with all exits is crucial to planning timely and safe evacuation of students if needed. The school nurse can draw upon a wealth of information that exists regarding threats or events that have occurred in the past at the school or in the local community in order to plan for possible future events. Fire extinguishers should be checked on a regular schedule for assessment of access, date of expiration, and functionality. 1. Incorrect: Teaching about laws on gun control is not an assessment, but rather an intervention that can be done. Teaching is not the initial step of the nursing process. Assessment comes first.

The nurse is preparing to educate a group of clients on how to decrease the risk of developing recurrent renal calculi. What topics should the nurse include? 1. High-purine foods to consume 2. Discuss diuretic use to prevent urinary stasis 3. Straining urine with each void 4. Maintaining a daily water intake of at least 2 liters 5. Foods low in calcium

2., 4., & 5. Correct: Diuretics are often used to prevent urinary stasis and further calculus formation. Thiazides decrease calcium excretion into the urine. Daily fluid intake should be 2-3 liters per day to ensure good renal function. Most stones are calcium stones, so decrease calcium in the diet to reduce the chance of calcium stones. 1. Incorrect: Clients with a history of renal calculi should be on a low-purine diet. When purines are digested, they produce a waste product called uric acid, which needs to be decreased in clients with renal calculi and with history of gout. 3. Incorrect: Straining is only necessary for the client with acute signs/symptoms of renal calculi. Remember client safety is always a priority. This question is focusing on decreasing risk factors for renal calculi. So let's look at the options. Option 1 is false. Don't consume a high-purine diet. Foods high in purines are organ meats, Other high purine meats will include bacon,beef, pork and lamb. Also beer, anchovies, sardines, herring, mackerel and scallops are high in purine. Clients with a history of renal calculi should be on a low-purine diet. When purines are digested, they produce a waste product called uric acid, which needs to be decreased in clients with renal calculi. Straining is only necessary for the client with acute signs/symptoms of renal calculi. Option 2 is true. Diuretics are often used to prevent urinary stasis and further calculus formation. Thiazides decrease calcium excretion into the urine. Option 3 is false. Unless the client has been diagnosed with a renal calculi, there is no reason to strain the urine. Straining is only necessary for the client with acute signs/symptoms of renal calculi. Option 4 is true. When the client drinks at least 2 liters of water per day the client is also producing more urine. This process continues to flush the kidneys and decreases the chance of renal calculi forming. Option 5 is true. Most stones are calcium stones, so decreasing the calcium in the diet will decrease the risk of developing renal calculi.

The nurse is caring for a client who has aphasia. What interventions should the nurse include in the plan of care to improve communication with this client? 1. Increase speaking volume and tone. 2. Present one thought at a time. 3. Use and encourage use of gestures. 4. Do not push communication if client is tired. 5. Give client time to generate a response. 6. Ask questions that can be answered with "Yes" or "No".

2., 3., 4., 5., & 6. Correct: These interventions will help to improve communication. Don't overwhelm the client with multiple thoughts or pushing communication on a tired or anxious client. Gestures such as pointing to something or asking the client to point to what they want will help to increase communication. Using a communication board for the client to point to commonly needed things is helpful. Give extra time to comprehend and respond to communication. Keep questions easy to answer and communication simple. 1. Incorrect: Don't yell at the client. The problem is not inability to hear. Asphasia is loss of ability to understand or express speech caused by brain injury. Nursing interventions that support communication include communicating frequently and meaningfully; allowing time for the client to comprehend and answer; using simple, short sentences; using visual cues; structuring conversation that permits simple answers by the client; and praising the client honestly for improvements in speech. The stroke client may be easily overwhelmed by verbal stimuli. Give extra time to comprehend and respond to communication. A picture board may be helpful.

A community health nurse is presenting a seminar to a group of senior citizens on ways to reduce the risks of peripheral artery disease (PAD). What topics should the nurse include? 1. Anti-embolic stockings 2. Smoking cessation 3. Moderate exercise 4. Application of heat 5. Low cholesterol diet 6. Decrease blood pressure

2., 3., 5. & 6. Correct: Senior clients are at increased risk for peripheral artery disease for a variety of reasons, though many erroneously believe that this process is an unavoidable part of the aging process. Educating clients on preventative activities will help reduce incidence of atherosclerosis and improved mobility along with quality of life. Smoking is a major risk factor in developing PAD by contributing to arterial constriction. Clients can increase collateral circulation with a moderate exercise program of at least 30 minutes three times a week. A low cholesterol, heart healthy diet with more fruits and vegetables helps reduce cholesterol while decreasing blood pressure, both important goals towards controlling PAD. 1. Incorrect: Increasing arterial blood flow is important in the prevention or management of peripheral artery disease; however, anti-embolic stockings are designed to improve venous return in clients with decreased mobility. The use of these stockings would actually hinder arterial flow in lower extremities. 4. Incorrect: Clients with PAD often complain of cold extremities secondary to decreased arterial blood flow. But the application of heat such as use of a heating pad is unsafe and is always contraindicated in the elderly with PAD. Inability to sense temperature extremes may result in serious burns to lower extremities. Additionally, clients with PAD do not heal as well from injuries or wounds.

The nurse recognizes that treatment has been successful in resolving fluid volume excess based on which assessment findings? 1. Continued lethargy 2. Heart rate 112/min 3. Decreasing shortness of breath 4. BP 114/78 5. Increased thirst

3. & 4. Correct: Urinary output should increase with decreasing shortness of breath as hydration is corrected, and BP should be normal. 1. Incorrect: Level of consciousness (LOC) should improve with perfusion to the brain. 2. Incorrect: Heart rate should decrease if hydration is corrected. 5. Incorrect: Thirst level should be decreased if hydration is corrected.

What interventions should the nurse include when teaching a client how to prevent and treat fungal infections of the feet? 1. Apply cornstarch to the feet after bathing. 2. Put terbinafine hydrochloride cream 1% on affected areas twice a day for two weeks. 3. Wear socks at all times until infection has cleared up. 4. Wash feet daily with soap and water. 5. Wear shower sandals when showering in public places. 6. Wear shoes that allow the feet to breathe.

2., 4. 5, & 6. Correct: Athlete's foot is treated with topical antifungal in most cases. Severe cases may require oral drugs. The feet must be washed daily with soap and water and dried thoroughly since the fungus thrives in moist environments.Steps to prevent athlete's foot include wearing shower sandals in public showering areas and wearing shoes that allow the feet to breathe. 1. Incorrect: Clients with fungal skin infections should avoid the use of cornstarch. The carbohydrates in cornstarch may provide nutrition to fungal infections and should be avoided. 3. Incorrect: Allow feet to have exposure to the air. The feet must be kept clean and dry since fungus thrives in moist environments. Keeping the feet covered all the time causes a dark, moist environment for the fungus to thrive. This question is asking about preventing and treating fungal infections. So let's look at the options. Option 1 is false. Clients with fungal skin infections should avoid the use of cornstarch. The carbohydrates in cornstarch may provide nutrition to fungal infections and should be avoided. The client can apply a powder specifically for feet is approved. Option 2 is true. Terbinafine hydrochloride cream 1%, an topical antifungal, is prescribed for the treatment of fungal infections of the feet. Option 3 is false. Allow feet to have exposure to the air. The feet must be kept clean and dry since fungus thrives in moist environments. Keeping the feet covered all the time causes a dark, moist environment for the fungus to thrive. Option 4 is true. The feet must be washed daily with soap and water and dried thoroughly since the fungus thrives in moist environments. Option 5 is true. Steps to prevent athlete's feet include wearing shower sandals in public showering areas. The showers in public areas may continue to stay moist after someone has showered. A moist floor is a good medium for the growth of fungus. Also other people who showered may currently have a fungal infection on their feet. Option 6 is true. Wearing shoes that allow the feet to "breathe" will decrease the feet from perspiring. Moisture on the sole of the feet and between the toes is a good medium for the growth of fungus.

The nurse is reviewing the Centers for Disease Control and Prevention's (CDC's) guidelines for immunization recommendations with a group of parents whose children are preparing to attend college in the fall. Which immunization recommendations should the nurse include? 1. Rotavirus 2. Meningococcal 3. Herpes zoster 4. Seasonal influenza 5. Human papilloma virus

2., 4., & 5. Correct: Meningococcal vaccine protects against bacterial meningitis and is recommended for students entering college. Influenza vaccine is recommended annually for protection against the viruses predicted to be most common for the season. Human papilloma virus vaccine is recommended for protection against the virus which causes cervical and anal cancers. 1. Incorrect: Rotavirus vaccine is recommended during infancy. Rotavirus is the most common cause of diarrheal disease among infants and children. 3. Incorrect: Herpes zoster vaccine is recommended for adults, over the age of 60 to reduce the risk of getting shingles. The clues in this question are "preparing to attend college" and "immunization recommendations." Rotavirus is the leading cause of vomiting and diarrhea among children. It is not generally recommended beyond 8 months of age. Meningococcal vaccine is recommended for all adolescents. Since protection wanes, a booster dose is recommended at age 16 or so during the ages that they are at highest risk. Herpes zoster vaccine (shingles) is recommended for people 60 years-old and older. The annual seasonal Influenza vaccine is recommended for everyone age 6 months and older by the end of October, if possible. Human papilloma virus (HPV) is a 3 dose series vaccine recommended for all 11 or 12 year-olds. Those who do not start or finish can get it through age 26 for females and 21 for males.

A nurse is teaching a group of expectant parents about epidural anesthesia. What information should the nurse include? 1. Contraindications for an epidural include a previous cesarean section. 2. Post procedure position should be side lying. 3. Headache is a post procedure side effect. 4. The major complication is hypotension. 5. Usually administered at 3-4 cm dilation.

2., 4., & 5. Correct: The client should not lie supine but should position self in a side-lying position. This will prevent compression on the vena cava. The major complication of epidural anesthesia is hypotension and supine position increases the risk. If this occurs, a bolus with 1000 mL of NS or LR to increase blood pressure by increasing vascular volume. Epidurals are usually placed during stage 1 at 3-4 cm dilation. 1. Incorrect: Previous C-sections do not eliminate the ability to have an epidural. Epidurals are commonly utilized for anesthesia during a cesarean birth. 3. Incorrect: A sterile guide needle and a small epidural catheter is placed between the spinal cord and the outer membrane. There is usually no headache since the needle does not enter the spinal column but rather the epidural space.

A nurse is educating several unlicensed assistive personnel (UAP) about a dietary prescription for clear liquids. Which selections by the UAP indicate to the nurse an understanding of a clear liquid diet? 1. Vanilla custard 2. Lemon jello 3. Tomato juice 4. Sprite 5. Banana popsicle

2., 4., & 5. Correct: These are considered clear liquids. You can see through them. The banana popsicle and lemon jello in a liquid state can be seen through. 1. Incorrect: This would be on a full liquid diet. A full liquid diet allows clear liquids along with thin hot cereals, strained cream soups, juices, milkshakes, custard, pudding and liquid nutritional supplements. 3. Incorrect: This would be on a full liquid diet. A full liquid diet allows clear liquids along with thin hot cereals, strained cream soups, juices, milkshakes, custard, pudding and liquid nutritional supplements.

Question: An elderly client who lives alone is being discharged home following a total hip replacement. The home care nurse is completing a safety assessment of the home environment prior to the client's arrival. Which conditions would require modifications to ensure client safety? 1. Wall-to-wall carpeting 2. Entrance throw rugs 3. Downstairs bathroom 4. Rail-free porch stairs 5. Step stool in kitchen

2., 4., & 5. Correct: Throw rugs are loose fall hazards that should be removed or tacked down to prevent tripping, particularly for a client whose mobility is impaired by hip surgery. Both inside and outside stairs should have hand rails to provide stability when in use. The presence of a step stool in the kitchen indicates that some items in the cupboards are out of reach for the client. Rearranging frequently used items to within the client's reach would be much safer than using a step stool. 1. Incorrect: Wall to wall carpeting does not generally present safety issues since it is attached to the floor on all four sides and does not move even if the client were shuffling on the surface of the carpet or using a walker. 3. Incorrect: The existence of a downstairs bathroom is a positive feature that alleviates the need for the client to climb stairs frequently during the day. Stairs are challenging for elderly adults, plus this client also has limited mobility following a total hip replacement.

An elderly client who lives alone is being discharged home following a total hip replacement. The home care nurse is completing a safety assessment of the home environment prior to the client's arrival. Which conditions would require modifications to ensure client safety? 1. Wall-to-wall carpeting 2. Entrance throw rugs 3. Downstairs bathroom 4. Rail-free porch stairs 5. Step stool in kitchen

2., 4., & 5. Correct: Throw rugs are loose fall hazards that should be removed or tacked down to prevent tripping, particularly for a client whose mobility is impaired by hip surgery. Both inside and outside stairs should have hand rails to provide stability when in use. The presence of a step stool in the kitchen indicates that some items in the cupboards are out of reach for the client. Rearranging frequently used items to within the client's reach would be much safer than using a step stool. 1. Incorrect: Wall to wall carpeting does not generally present safety issues since it is attached to the floor on all four sides and does not move even if the client were shuffling on the surface of the carpet or using a walker. 3. Incorrect: The existence of a downstairs bathroom is a positive feature that alleviates the need for the client to climb stairs frequently during the day. Stairs are challenging for elderly adults, plus this client also has limited mobility following a total hip replacement. Remember client safety is always a priority. This question is asking about which home modifications should be implemented after a safety assessment of the home. This is question is focusing on areas of the home environment that need to be changed. So let's look at the options. Option 1 is false. Wall to wall carpeting does not generally present safety issues since it is attached to the floor on all four sides and does not move even if the client were shuffling on the surface of the carpet or using a walker. Option 2 is true. The presence of the throw rugs is identified as a negative safety assessment. Throw rugs are loose fall hazards that should be removed or tacked down to prevent tripping, particularly for a client whose mobility is impaired by hip surgery. Option 3 is false. The existence of a downstairs bathroom is a positive feature that alleviates the need for the client to climb stairs frequently during the day. The client needs to avoid flexion of the hip. Option 4 is true. Both inside and outside stairs should have hand rails to provide stability when in use. The presence of the rail free porch stairs is identified as a negative safety assessment. Option 5 is true. Because the the client could fall off a stool, the stool in the kitchen is identified as a negative safety assessment. Rearranging frequently used items to be within the client's reach would be much safer than using a step stool.

The nurse is caring for a burn client 48 hours after the burn occurred. What would be the nurse's priority assessment? 1. Measure the abdominal girth. 2. Administer pain medication. 3. Auscultate the lungs every 2 hours. 4. Inspect the burn for infection.

3 Correct: After 48 hours, the fluid in the interstitial spaces will begin to shift back into the vascular space and can lead to fluid volume excess. Excess fluid can back up into the lungs, so auscultation of the lungs takes priority. Remember: Airway, breathing, then circulation. 1. Incorrect: No indication of need to measure abdominal girth. Fluid is now shifting out of the tissue and abdominal cavity back to the vascular space. Worry about fluid volume excess now. 2. Incorrect: Not priority over pulmonary function. Pain is a priority from the client's perspective, but remember pain never killed anyone. However, fluid in the lungs will! Lung assessment takes priority. 4. Incorrect: Not priority over pulmonary function. Again, we want to inspect for infection, but it is not the priority over assessing the lungs.

A nurse is planning to teach a group of adult males in their 40's about health care promotion recommendations. Which recommendations should the nurse include? 1. Do bi-annual skin self-exam to check for new moles or changes in moles. 2. Comprehensive eye exam every 5 years starting at age 45. 3. Limit alcohol intake to no more than two drinks per day. 4. Yearly physical exam from a health care provider. 5. Get at least 30 minutes of moderate physical exercise on most days of the week.

3, & 5. Correct: If a client must drink alcohol, they should do so only in moderation. For men, that means up to two drinks a day for men age 65 or younger and one drink a day for men over age 65. The risk of various types of cancer, such as liver cancer, appears to increase with the amount of alcohol ingested and the length of time that one has regularly been drinking. Too much alcohol can also raise blood pressure. Physical exercise can go a long way toward managing stress and controlling weight. Controlling stress and obesity can decrease the risk of many health risks such as heart disease, diabetes, and stroke. 1. Incorrect: Do monthly skin self-exam to check for new moles or changes in moles. 2. Incorrect: Comprehensive eye exam every 2 years is recommended. Changes in vision is a relatively common problem for people in their 40s. They may find that glasses are needed for the first time in their life. They may need glasses to see at a distance or for reading. 4. Incorrect: Physical exam every 2-3 years when no health issues exist including height, weight, and BMI. Routine blood tests, urinalysis and mental health screening is conducted at this time.

The previous shift nurse reported to the oncoming nurse a suspicion that a client's central line has developed a fibrin sheath. Which prescription does the nurse anticipate the healthcare provider will prescribe? 1. Heparin 2. Enoxaparin 3. Alteplase 4. acetylsalicylic acid

3. Correct: If a catheter becomes partially blocked due to a fibrin sheath or loses its blood return, a fibrinolytic is typically prescribed. Currently, alteplase is the preferred thrombolytic to treat thrombotic occlusions. 1. Incorrect: Systemic anticoagulation with heparin for treatment of a fibrin sheath has not been proven to be beneficial. 2. Incorrect: Enoxaparin is a low dose molecular heparin and is not beneficial in treating a fibrin sheath. 4. Incorrect: One of the effects of acetylsalicylic acid is its inhibition of platelet aggregation. However, these blood thinning effects are not beneficial in treatment of a fibrin sheath.

Which signs and symptoms will the nurse include when teaching a client about indicators of recurrent nephrotic syndrome? 1. Dysuria 2. Hematuria 3. Foamy urine 4. Periorbital edema 5. Weight loss

3. & 4. Correct: Foamy urine, which may be caused by excess protein in the urine, is seen with nephrotic syndrome. Swelling (edema), particularly around the eyes (periorbital) and in the ankles and feet, is a symptom. 1. Incorrect: Dysuria would be a symptom of disorders such as kidney stone or UTI, rather than nephrotic syndrome. 2. Incorrect: Proteinuria rather than hematuria is seen. 5. Incorrect: Weight gain is seen with renal disorders due to poor renal function and increased fluid volume. Well before you can teach about nephrotic syndrome you have to know something about it, don't you? Yes. So what is it? Nephrotic syndrome is a kidney disorder that causes the body to excrete too much protein in the urine because of big holes that occur in the glomerulus, the filtering portion of the kidneys. Signs and symptoms of nephrotic syndrome include severe edema (anasarca), periorbital edema; foamy urine, which may be caused by excess protein in the urine; and weight gain due to excess fluid retention. So which options match what we know about nephrotic syndrome? Did you say options 3 and 4? Good job! Look at option 1. Dysuria would be a symptom of disorders such as kidney stone or UTI, rather than nephrotic syndrome. Option 2 is hematuria, which would be seen with infection. Option 4 is false. We said weight gain would occur, not weight loss.

A client is being discharged with halo traction. What should the nurse teach about home care of this traction? 1. Showering is permitted. 2. Apply baby powder under the halo vest to prevent irritation. 3. Never pull on any part of the halo traction. 4. Clean around pins at least twice a day using sterile technique. 5. Driving is allowed after discharge.

3. & 4. Correct: Never pull on any part of the halo traction. It can damage or loosen the traction. Pin care is done to prevent infection. Clean around pins at least twice daily with sterile q-tip applicator. Use a new sterile q-tip for each pin site to decrease contamination from one pin site to another. Do not use ointments or antiseptics unless prescribed. 1. Incorrect: Client should never attempt a shower since there is no reliable way to keep vest liner dry. Take sponge baths or sit in a bathtub with about 2-3 inches of water. Use towels or plastic to keep vest from getting wet. 2. Incorrect: Do not use soaps, creams, lotions or powders beneath the vest as these may irritate the skin. 5. Incorrect: Absolutely no operating a motor vehicle until the primary healthcare provider allows this activity: field of vision and movement is narrowed and/or diminished. The client is an impaired driver and could cause an accident.

The nurse is caring for a client with body dysmorphic disorder. The client tells the nurse, "My ugly ears make everyone sick!" Which defense mechanism is this client utilizing? 1. Sublimation 2. Somatization 3. Symbolism 4. Projection 5. Conversion

3. & 4. Correct: Symbolism and projection are the correct answers. Symbolism is the unconscious process by which one object or idea comes to represent another. The client's ears symbolize everything distasteful and unacceptable to her/him. Projection is the unconscious process by which an individual attributes one's own intolerable wishes, emotions, or motivations to another person. The client states that others are horrified by the ears, but actually the client is horrified by them. 1. Incorrect: Sublimation is the unconscious process of substituting acceptable impulses for unacceptable ones. Sublimation is not shown in this scenario. 2. Incorrect: Somatization has significant functional impairment as recurrent medical symptoms appear, but no physical cause can be identified. This is not shown in this scenario. 5. Incorrect: Conversion is marked by deficits in voluntary motor or sensory function. Conversion is not shown in this scenario. You remember defense mechanisms. Now in the question stem the client is using at least 2 defense mechanisms. This is a select all that apply question and two or more will be correct. Option 1: No. Sublimation is substituting acceptable impulses for unacceptable impulses. With symbolism and this client scenario presented in the stem, the "ears" symbolize "unacceptable". Option 2: No. There are no indications of somatization in this question stem. Option 3: Great choice! The client's ears symbolize everything distasteful and unacceptable. The clues in the question stem - ugly ears. Option 4: Yes! Again the clues are in the question stem - make everyone sick. The client is unconsciously projecting unacceptable emotions in others. Option 5: No. There are no deficits in voluntary function. This client has no symbolic physical symptom such as pain, paralysis, or loss of sight.

Which conditions in children and/or adolescents should a nurse identify as a potential cause of metabolic alkalosis? 1. Anemia 2. Renal failure 3. Pyloric stenosis 4. Bulimia nervosa 5. Aspirin abuse

3. & 4. Correct: The nurse should identify pyloric stenosis and bulimia nervosa as potential causes of metabolic alkalosis. Children diagnosed with pyloric stenosis or bulimia nervosa experience loss of stomach acid from excessive vomiting. The loss of stomach acid (decrease in hydrogen ions) will eventually lead to an increase in blood pH (alkalosis). 1. Incorrect: Any condition associated with a fall in PaO2 below 55 or with decreased oxygen delivery to the tissues, increases ventilation, causing respiratory alkalosis (anemia, high altitude). 2. Incorrect: Renal failure results in metabolic acidosis, not metabolic alkalosis. 5. Incorrect: Aspirin (acetylsalicylic acid) abuse or chronic overuse results in metabolic acidosis, not metabolic alkalosis.

The nurse is assigned five clients on a medical floor. When planning care, the nurse recognizes which clients to be at greatest risk for ineffective oral hygiene? 1. A client who has just had knee surgery taking opioids for pain. 2. A right handed client who had a stroke affecting the right hemisphere of the brain. 3. A client with breast cancer who is experiencing severe nausea and vomiting after chemotherapy. 4. An elderly client experiencing loss of appetite. 5. A client who takes phenytoin for partial seizures.

3. & 5. Correct: A client with severe nausea and vomiting after chemotherapy is at an increased risk for ineffective oral hygiene problems due to vomiting, decreased oral intake, and the effects of the chemotherapy on the oral mucosa. Phenytoin causes gingival overgrowth, swelling and bleeding of the gums. This can make oral hygiene more difficult. 1. Incorrect: This client can perform oral hygiene with minimal assistance. Knee surgery and opioid pain medication do not interfere with oral hygiene. 2. Incorrect: Movement for one side of the body is controlled by the opposite side of the brain. If stroke affects the right side of the brain, then you will have trouble with the left side of your body. Since this client is right handed and his left side is affected, the client can perform oral hygiene. 4. Incorrect: This client can perform oral hygiene with minimal assistance. There is no information in this option that would put this client at risk for ineffective oral hygiene.

The RN, LPN, and unlicensed assistive personnel (UAP) are providing care for clients on the nursing unit. Which tasks could be completed only by the RN? 1. Administration of routine medications. 2. Dressing changes. 3. Assessment of newly admitted clients. 4. Calling the primary healthcare provider about lab results. 5. Teaching the diabetic client foot care.

3. & 5. Correct: Admission assessments and teaching must be performed by the RN. The nursing process, along with teaching are outside the scope of practice of the LPN. These are tasks that must be performed by the RN. The LPN can reinforce teaching. 1. Incorrect: Medication administration is within the LPN scope of practice and can be completed by the LPN. 2. Incorrect: Dressing changes may be delegated to the LPN as this is within the LPN scope of practice. 4. Incorrect: The LPN may call lab results to the primary healthcare provider because this is within the scope of practice for the LPN. If any additional prescriptions are required, the LPN can take these prescriptions over the phone. So Here is the Deal: The RN cannot delegate assessment, evaluation, plan of care development or teaching to an LPN or UAP. So these things must be done by the RN. Also, keep this in mind: Follow the 5 rights of delegation: Right task, right person, right circumstance, right direction, right supervision. Basically, you have to know what an LPN and the UAP can and cannot do. So let's look at the options we have been provided. Option 1: Administer routine medications. Is the RN the only person who can administer ROUTINE medications? No. The LPN can do this. So this is false. Option 2: Dressing changes. The LPN is taught to do dressing changes in school so this is part of their scope of practice. So this to is false. Option 3: Assessment of newly admitted clients. Who does the assessment? The RN. And we don't know anything about the new admit. This client might be unstable. So the RN should do this assessment. Now FYI, the LPN can collect some of the assessment data for the RN, but the RN is responsible for the assessment. Option 4: Calling lab results to the PHCP. Are RNs the only ones who can do this? No, the LPN can call the PHCP and can take prescriptions from the PHCP. So this is false. Option 5: Teaching the diabetic client foot care. Who teaches? The RN is responsible for teaching. So this is true.

The nurse is caring for a heart failure client taking spironolactone. Which snack choices would indicate to the nurse that the client understands proper dietary choices while on this medication? 1. Bananas 2. Cheese and crackers 3. Apples 4. Sweet potatoes 5. Grapes

3. & 5. Correct: Apples and grapes are low in sodium and potassium. Spironolactone is a potassium sparing diuretic. The client with heart failure needs to limit sodium and potassium. 1. Incorrect: The action of spironolactone is to inhibit the reabsorption of sodium in the kidney while saving potassium. It is a diuretic so the client will lose water. Bananas are high in potassium. 2. Incorrect: Cheese and crackers are high in sodium. Spironolactone is given to lower BP and decrease fluid. Foods high in sodium should be limited. 4. Incorrect: The action of spironolactone is to inhibit the reabsorption of sodium in the kidney while saving potassium. It is a diuretic so the client will lose water. Sweet potatoes are high in potassium. Option 1. False. Bananas are high in? Potassium. You knew that. Option 2. False. Cheese and crackers are high in sodium. Spironolactone is given to lower BP and decrease fluid. Foods high in sodium should be limited. Option 3. True. Apples are low in sodium and potassium. Option 4. False. Sweet potatoes are high in potassium. Option 5. True. Grapes are low in sodium and potassium.

A nurse walks into the medication area of a long-term care facility and sees a colleague taking a pill from a resident's supply of narcotics. The nurse says, "Please don't say anything. I need my job and I have a migraine." What actions should the nurse take? 1. Reassure the colleague that she won't tell this time. 2. Insist that the colleague get some help. 3. Report what was seen to the supervisor. 4. Send the colleague home. 5. Follow procedure to return medication to the resident's supply.

3. & 5. Correct: The nurse should follow the procedure to return the narcotic, and then the nurse should report the observation to the supervisor. The nurse must serve as client advocate by reporting a nurse who may be impaired. 1. Incorrect: This may be the first observation; however, it is unlikely that it is the first incidence. The impaired nurse must be reported. You are responsible to the clients on the unit, not to the staff member. 2. Incorrect: The supervisor is the one to provide information on obtaining help. The hospital or long term care facility will have a policy for the supervisor to follow. Usually this policy also includes rehabilitation. 4. Incorrect: The nurse should leave if she is taking narcotics. The supervisor will be the one to send the nurse home. The supervisor needs to determine if the degree of impairment would interfere with the ability to drive home safely. So in this question we have a nurse who is taking a client's narcotic medication. What actions should the nurse take? Let's look at the options. Option 1. So are you going to cover for the nurse and not tell anyone? No, this will not only hurt the nurse in the long run, but could lead to client harm. It's a safety issue, isn't it? Yes. Option 2. What about insisting the nurse get some help? Well, the nurse may say ok, but then never seek help. The person caught will generally do or say anything to keep the authorities from finding out. As a colleague you can offer support, but don't go there alone. Follow the policy of the institution and state. Option 3. Report what is seen to the supervisor? Yes. Get it out of your hands and into the hands of the person responsible for instituting the policy for substance abuse by staff. Option 4. Send the nurse home? Not your call. Notify the supervisor to deal with the situation. The nurse should leave if she/he is taking narcotics. The supervisor will be the one to send the nurse home. The supervisor needs to determine if the degree of impairment would interfere with the ability to drive home safely. Option 5. The nurse should follow the procedure to return the narcotic. Remember to think safety. Safety for the nurse. Safety for the clients on the unit. The nurse must serve as client advocate by reporting a nurse who may be impaired.

The nurse is admitting an adolescent reporting severe depression and amenorrhea. What additional assessment findings by the nurse would suggest the client may develop anorexia nervosa? You answered this question 1. Tight fitting clothes 2. Oily, elastic skin 3. Brittle, dry nails 4. Gingival infections 5. Low blood pressure

3. & 5. Correct: This client is reporting symptoms consistent with anorexia nervosa, a serious and potentially life-threatening eating disorder that develops secondary to the type of family or social stress experienced in adolescence. In addition to severe depression and amenorrhea, the nurse has identified brittle, dry nails, and a low blood pressure secondary to weight loss as additional indications of anorexia nervosa. 1. Incorrect: Despite the fact that anorexic clients experience severe weight loss, they continue to view themselves as heavy and generally wear loose fitting clothing to hide what they perceive as an overweight body. 2. Incorrect: Because of skeletal muscle atrophy and poor nutritional intake, anorexic clients display sallow, dry skin with brittle nails and hair. Oily, non-elastic skin would not be noted in a client with anorexia nervosa. 4. Incorrect: Gingival infections and dental caries are typical of clients with bulimia, another eating disorder in which stomach acid from frequent vomiting causes gum infections or dental caries. This is not common in anorexics. Option 1: Definitely not. These clients have low self-esteem and believe that they are fat. So rather than showing off a thin figure, they tend to wear loose or ill fitting clothing in order to hide their body. Over sized clothing is usually a clue, since they obviously appear thin. Option 2: Nope! Because nutrition is poor, an anorexic individual experiences skeletal muscle atrophy secondary to loss of body fat. Oily skin would not occur in these dehydrated clients. Elastic skin is just another way to describe skin turgor. Hydrated, healthy skin has good elasticity and bounces back when pinched. Nonelastic skin would refer to poor skin turgor, which often accompanies malnourishment along with sallow, but not oily, skin. Option 3: Yes, this is a positive indication of anorexia! Poor nutrition will lead to dry skin, brittle nails, and even a loss of hair. Because these clients are so pre-occupied with buying and preparing "healthy" foods, they often do not notice other symptoms. They are obsessively focused on what they believe is an overweight body rather than the appearance of their hair and nails. Option 4: This is not a symptom of anorexia, but rather indicates another eating disorder - bulimia. In that eating disorder, the clients tend to binge on food and then induce vomiting out of guilt to get rid of the calories ingested. Because of this frequent vomiting, stomach acid washes over the teeth and gums, causing infections and cavities. Since there is not a dramatic weight issue, this eating disorder may not be diagnosed for years. Option 5: Good choice. The anorexic client has many physical manifestations typical of poor nutrition, including weight loss and electrolyte imbalance. They also have a very low blood pressure secondary to lack of food and fluids, constant dieting, and rigorous exercise. These clients generally die from cardiac arrhythmias.

The nurse has initiated instruction for an 11 year old child newly diagnosed with diabetes mellitus. The child indicates anxiety about the need for daily insulin injections. What nursing action would best address this issue? 1. Tell the child it only hurts for a moment. 2. Have the parents administer the shots. 3. Show the child how to give self-injections. 4. Provide toy syringe for the client to play with.

3. CORRECT: A school age child needs a sense of achievement and control of the situation. Because diabetes will be a life-long disease, it is important for the child to begin learning about self-care which includes daily insulin injections. Age eleven is not too young to begin administering self-injections. 1. INCORRECT: This is a false statement, considering the fact that pain perception varies. Minimizing the amount of potential discomfort will instill distrust in the child, decreasing compliance with the health regimen. 2. INCORRECT: While parents may administer injections for much younger children, school aged children are capable of becoming independent with all aspects of diabetes. Additionally, an 11 year old client needs to develop a sense of mastery and achievement to accomplish this stage successfully. 4. INCORRECT: The client is too old for "pretend play" with imitation syringes. That process is more appropriate for a preschool child. It would be beneficial to allow this child to handle regular syringes without a needle initially, and then add all the necessary equipment when the client feels more comfortable handling everything.

A nurse is attempting to assess lung sounds on a 3 year old with a history of asthma. The nurse knows the best method to encourage the child to take a deep breath is what? 1. Allow the child to blow out a lighted candle. 2. Encourage child to blow bubbles from a wand. 3. Teach child to blow cotton balls off the table. 4. Instruct child on using an incentive spirometer.

3. CORRECT: Assessing lung sounds requires a client to inhale and exhale while the nurse auscultates. The most efficient method is to have the child participate in a game that requires breathing in and out. Blowing a cotton ball across a table is an appropriate activity which can be easily understood by a 3 year old child and mimics a game that will encourage participation. 1. INCORRECT: An open flame, even on a small candle, is a safety violation and contraindicated in all types of health facilities. Additionally, since the child will need to breathe in and out multiple times, it would be both risky and inconvenient to keep relighting a candle. 2. INCORRECT: While it might be easier, and even enjoyable, for the child try to blow bubbles from a bubble wand, consider the safety aspect. Bubbles are made from soap, which would make floors slippery. Additionally, children usually want to chase bubbles, and having the child remain still for auscultation would be a challenge. 4. INCORRECT: The purpose of using an incentive spirometer is to encourage clients to cough and deep breath. Instructing a 3 year old on the spirometer would be challenging and does not help the nurse to assess lung sounds.

A client is seeing the obstetrician for a monthly checkup at 35 weeks. Vitals are within normal limits but the nurse notes bilateral pedal edema. What statement by the nurse provides the most appropriate information to the client? 1. "Do not use any salt for your food." 2. "Cut your daily fluid intake in half." 3. "Sit and elevate feet above your heart." 4. "I'll request a prescription for a diuretic."

3. CORRECT: In the third trimester of pregnancy, it is common for clients to complain about edema of hands and feet, most common at the end of the day. The client should sit for short periods of time, with feet elevated above the level of the heart, to decrease the edema. 1. INCORRECT: The type of edema the nurse has noted is not caused by salt intake, but rather is caused by walking or standing too long during the day. Removing sodium from the diet will not affect the amount of edema the client develops. 2. INCORRECT: Pregnant females need to stay well hydrated, particularly in the third trimester, to avoid complications such as Braxton Hicks contractions. Cutting daily fluid intake has no effect on the presence of edema, but may cause other problems. 4. INCORRECT: Staying well hydrated is important in all stages of pregnancy, particularly the third trimester. Unless the mother has cardiac issues, a diuretic would be contraindicated.

An adolescent is being instructed on the proper way to use crutches following knee surgery. The nurse knows that teaching has been successful when the client makes what statement? 1. "The weight of the crutches should be on my shoulders." 2. "It's ok to lean against the crutches if I am standing still." 3. "If going up the stairs, my non-operative leg goes up first." 4. "When sitting down, first lean crutches against the wall."

3. CORRECT: It is evident the client has understood the nurse's instructions with this statement. When going up stairs with crutches, the unaffected (non-operative) leg goes up first. The strong leg bears the body weight and therefore provides a solid base while the client lifts up the weaker leg. 1. INCORRECT: The correct position for crutches should be about 2 inches below the axilla and never directly up into the arm pit. Body weight is carried by the hands on the hand-grips, which means the force to push comes from the forearms and biceps, not the shoulders. Bearing weight in the axilla can cause severe damage to the axillary nerves and muscles. 2. INCORRECT: Crutches are an ambulatory aid only and, if not used correctly, can actually be a safety hazard. Even if the client is standing still, it is unsafe to use crutches as a leaning support. Additionally, even resting on the crutches can cause trauma to the axillary area. 4. INCORRECT: When sitting in a chair, the client should back up to the chair until the back of the knees gently touches the chair seat. The client should then reach back and grab the arm of the chair while holding both crutches in the other hand. Once seated, the client can then lean the crutches upright nearby. They will balance better if stood upside down.

Which task would be appropriate for the nurse to assign to the unlicensed assistive personnel (UAP)? 1. Check the bladder for distension in the client who had a indwelling catheter removed 4 hours ago. 2. Obtain BP of client with syncope in the lying, sitting, and standing positions. 3. Prepare a sitz bath for a postpartum client. 4. Monitor for grimacing in the client who has had a stroke.

3. Correct. The UAP can assist clients with hygiene care, so it is within the scope of practice for the UAP to assist a client with a sitz bath for the postpartum client. 1. Incorrect. This is not within the scope of practice for the UAP. The nurse must assess and evaluate.Checking the bladder for distension is an assessment that requires the nurse's attention. 2. Incorrect. This client is not stable if having episodes of syncope that could be related to orthostatic hypotension. Since the client is not stable, the UAP should not obtain the client's BP. The nurse should assess the client. 4. Incorrect. The nurse cannot delegate an assessment or evaluation task to the UAP. This is beyond the scope of practice for the UAP.

The nursing supervisor is reviewing several instances in which restraints have been used. The nurse is aware the only acceptable use of restraints is what? 1. An elderly male had a chest restraint applied after crawling over bed rails several times. 2. An Alzheimer client's room door is closed to prevent wandering during shift change. 3. A confused client with a closed head injury had hand mitts applied after pulling out IV 4. A dementia client with sundowners is placed in Geri-chair with lap belt at nurse's station.

3. CORRECT: Restraints are considered a last resort when caring for a client, whether soft cloth or chemical restraints. The most acceptable use is to prevent a client from harming self or others. In this instance, a confused client has previously pulled out an ordered IV Therefore, the use of hand mitts is the most appropriate, least-restrictive method to prevent the client from further self-harm. 1. INCORRECT: There are several problems here. The client had side rails up, which are considered a form of restraint and in many facilities are no longer permitted. By applying a chest restraint, the client has been restrained twice. Just because a client is elderly does not mean restraints are needed. This restraint is not acceptable. The nurse should provide regular toileting periods and determine why this client is climbing out of bed. 2. INCORRECT: Closing a client into a room is overly restrictive and unsafe. This Alzheimer's client needs to be observed and closing the room door prevents visual access. Additionally, closing the door may violate fire safety codes in certain facilities. At shift change, when staff is occupied with report, special arrangements should be made so that the client can be observed and not restrained. 4. INCORRECT: Depending on the facility, placing a client upright at night, using a Geri-chair and a lap belt is overly restrictive. A client with dementia is challenging, particularly in the presence of sundowner syndrome. However, keeping a client upright all night, belted into a chair for the purpose of observation, is neither safe nor healthy for the client.

The nurse is presenting an informational seminar to the parents of grade school children about developmentally appropriate behaviors. What activity would the nurse discuss as expected school age behavior? 1. Worrying about clothing and appearance. 2. Playing quietly with play dough or sand. 3. Collecting and classifying rocks or leaves. 4. Participating in a rock band at the school.

3. CORRECT: School age children, between ages 6 and 12, represent Erikson's stage of "industry versus inferiority" which focuses on developing a sense of self-competency thru creative learning and problem-solving. These children enjoy cooperative play involving games, including collecting, sorting and classifying objects. 1. INCORRECT: Concern about appearance and clothing is more appropriate for the adolescence group, ages 13 to 20 which experience "identity versus role confusion". These individuals are slowly becoming emancipated while examining and creating a sense of sexual identity. 2. INCORRECT: Solitary play, particularly with textured substances like clay, dirt, or sand, begins among children in the infancy group at ages of 6 to 12 months. In this "trust versus mistrust" stage, increasing mobility and eye hand coordination leads to curiosity and exploration of new textures. 4. INCORRECT: Joining a band is more appropriate in the adolescent group where the pressure to belong and achieve is enormous. This activity is not usually associated with school age children, who may enjoy group activities, but with peers of the same age and sex.

The labor and delivery charge nurse is making staff assignments, including a new nurse. What client is most appropriate for the new nurse? 1. A gravida 3 para 2 in active phase of stage one, expecting twins. 2. A gravida 2 para 0 at 41 weeks gestation, awaiting induction. 3. A primigravida in active phase of stage one, waiting for epidural. 4. A 12-hour post Cesarean section needing assistance to ambulate.

3. CORRECT: The primigravida presents many basic and diverse skills that would be very educational for the new nurse. This is the most appropriate client and will provide a good experience in basic labor and delivery procedures. 1. INCORRECT: While this may seem like an interesting case, there is the potential for several problems. A third pregnancy generally proceeds faster, and this client is expecting multiple births. This case can quickly become too complicated for a new nurse. 2. INCORRECT: Although this may seem like an interesting case for the new nurse, induction of labor can lead to many problems which could be too complicated for this new nurse. This client requires close monitoring during the induction and would not be the best choice here. 4. INCORRECT: Ambulating a post-C-section for the first time would be within the level of competency for a new nurse. However, there is very little educational value in this assignment and it is important to provide learning opportunities for this new nurse.

A preschool child has returned from surgery following the extraction of multiple teeth and is crying loudly. The nurse is aware that the best method to perform the post-operative assessment is what? 1. Start by medicating the child for pain. 2. Offer the child cold water to drink. 3. Examine the most painful part last. 4. Verify the patency of the IV site.

3. CORRECT: The process of completing a post-operative assessment must be modified when evaluating children, particularly at the preschool age. The combination of fear, pain, and confusion after anesthesia could lead to lack of cooperation. The best method to approach this client is to leave the most painful part till last, which in this case would be examining the mouth. 1. INCORRECT: Though it is understandable this client will need medication for pain, a nursing assessment must be completed first. Administering pain medication would alter responses or symptoms the nurse must evaluate in order to spot early signs of complications. 2. INCORRECT: Following any surgery, the post-operative assessment is the nurse's priority when the child arrives back on the floor. While the child may possibly want a drink, the nurse must first determine whether the gag reflex is intact. 4. INCORRECT: Assessing the patency of the I.V. site is part of a complete nursing assessment following surgery. However, this does not focus on the overall method for assessing a child in pain.

A client diagnosed with human immunodeficiency virus (HIV) is to be sent home today. The nurse has initiated discharge instruction on the proper handling of blood and body fluid at home. The nurse knows the teaching is successful when the client makes what statement? 1. "As long as it's my home, I can use normal cleaning methods." 2. "I must scrub with hot, soapy water and allow it to air dry." 3. "I should clean area with a 10% mixture of bleach and water." 4. "I must sterilize with isopropyl alcohol and rinse with ammonia."

3. CORRECT: The proper method to clean spills of blood or body fluids at home is to use a 10% solution of household bleach, which means 9 parts of water to 1 part bleach. It is recommended to leave the bleach solution on the contaminated area for 10 to 20 minutes, and then rinse with hot water. Any towels or cloths used to clean the area should be double bagged and discarded. 1. INCORRECT: This demonstrates a false sense of security. Even in a home environment, visitors and family could become contaminated with the HIV virus. Microscopic amounts of blood or body fluids could contaminate others, and therefore proper cleaning methods must be followed even at home. 2. INCORRECT: Hot, soapy water will not kill the HIV virus on hard surfaces, regardless of the type of soap or the temperature of the water. Additionally, air drying will not decrease the virulence of the virus. 4. INCORRECT: Isopropyl alcohol, or rubbing alcohol, does not inactivate the HIV virus, even if rinsed with ammonia. Also, mixing household chemical cleaners, such as bleach with ammonia, can create dangerous fumes that are toxic to humans.

A housekeeper has been called to the medical-surgical unit to complete several tasks. Which of the following tasks by the housekeeper has priority? 1. Replace the full sharps container in the medication room. 2. Clean room of discharged client who was isolated with MRSA. 3. Wipe up spilled coffee in the family waiting room. 4. Repair a malfunctioning curtain around a client's bed.

3. CORRECT: When considering multiple safety issues, the priority is the situation which puts the greatest number of individuals at risk. Liquid on a floor is a fall hazard to anyone in that vicinity. A family waiting room has dozens of visitors a day, including adults, children, clergy, other staff and possibly other clients. The floor needs to be clean and dry to prevent injury. 1. INCORRECT: The only individuals affected in this situation would be those staff personnel authorized to be in the medication room. In addition to the housekeeper, nursing staff can also change sharps containers. Therefore, even a nurse could replace the filled containers if need be. This action is not the first priority. 2. INCORRECT: Cleaning an isolation room is a time-consuming process. Waiting until more important tasks are completed will not put anyone at risk since the room cannot be used until cleaned. Another task has priority first. 4. INCORRECT: The curtains that hang around a client's bed are for the purpose of privacy. Even a "malfunctioning curtain", which could be anything from torn fabric to broken hooks, does not pose a hazard. Although the client may not have complete privacy, this problem would not affect other clients. There is another issue that affects many individuals.

A 70 year old client was admitted to the unit during the night shift with chronic hypertension. At 0830, the unlicensed nursing assistant (UAP) reports that the client's blood pressure is 198/94 mm Hg. What would be the best action for the charge nurse to delegate at this time? 1. Ask the UAP to put the client back in bed immediately. 2. Tell the UAP to take the BP in the opposite arm in 15 minutes. 3. Have the LPN/LVN administer the 0900 furosemide and enalapril now. 4. Ask the LPN/LVN to assess the client for pain.

3. Correct. Administering the client's blood pressure medicines are aimed at correcting the hypertension. The therapeutic action of furosemide is diuresis which will lower the blood pressure. Enalapril is an angiotensin converting enzyme (ACE) that treats hypertension. These medications can be administered within 30 minutes of 0900. 1. Incorrect. Assisting the client back to bed is appropriate, but does not address the problem of lowering the client's BP. Administration of furosemide and enalapril will benefit the client with hypertension. 2. Incorrect. Retaking the BP in the opposite arm is within the scope of practice of an UAP, but does not address the problem of lowering the client's blood pressure. Additionally this should be completed prior to 15 minutes time. The priority is to get the BP down by giving the prescribed medications for hypertension. 4. Incorrect. The LPN can ask the client if they have chest pain. The client does have a BP of 198/94 which could lead to chest pain. The priority is to get the BP down to decrease the risk of complications associated with hypertension, such as MI, and stroke.

A client is to undergo an endoscopy in the client's room. The gastroenterologist gives a verbal prescription to the general floor nurse to prepare and administer propofol 10 mL slow IVP until sedation is achieved. What action should the nurse take? 1. Administer the propofol as prescribed. 2. Draw up the propofol and give it to the gastroenterologist to administer. 3. Inform the gastroenterologist that giving propofol is outside the nurse's scope of practice. 4. Request the gastrointerologist write the prescription.

3. Correct. Propofol administration is outside the scope of practice for general floor nurses. The gastroenterologist cannot monitor the client adequately while performing the procedure. A nurse anesthetist or anesthesiologist should be present. 1. Incorrect. Propofol administration is outside the scope of practice for general floor nurses. 2. Incorrect. The nurse should not draw up a medication and hand it to someone else to administer. Additionally, propofol should not be administered on a general unit without an anesthesiologist, or nurse anesthetist in attendance. 4. Incorrect. Prescriptions should be written rather than given verbally. However, the RN cannot administer propofol.

The nurse is teaching a client diagnosed with salmonellosis about how to decrease the transmission to others. Which statement by the nurse would require follow up? 1. "I will wash my hands after feeding pets." 2. "I will use a meat thermometer to cook food to safe temperature." 3. "I will clean my hands with water before handling food." 4. "I will use disposable dishes until infection free."

3. Correct. Salmonellosis is caused by the bacteria salmonella. Hands should be washed with soap and warm water. Only washing with water is not correct and requires the nurse to do further teaching with the client. 1. Incorrect: This statement indicates the patient understands teaching. The client should wash hands after contact with animals, their food or treats, and their living environment. 2. Incorrect: A meat thermometer should be used to ensure foods are cooked properly. Undercooked meat and unpasteurized milk is a source for the organism. 4. Incorrect: Disposable dishes help prevent the spread of infection. It prevents the organism from being transferred on dirty dishes.

A nurse has received report on a client to be admitted from the surgical suite following an unexpected amputation of the right arm because of a tractor accident. Which action by the nurse would best help the client upon arrival to the unit? 1. Notify hospital social services about adaptive equipment needs. 2. Prepare to change the dressing so the client can see the stump. 3. Ask client's family and hospital chaplain to be present in room. 4. Advise dietary that client will need food precut in small pieces.

3. Correct. This client will be awake from surgery to face the unexpected amputation of an arm, which has long-term physical, psychological, emotional and financial implication. Even clients facing a scheduled limb removal experience distress, anger or depression. Anticipating that the client will need a great deal of emotional support, the nurse is aware that having family and/or the hospital chaplain present after surgery may help the client cope with the bad news. 1. Incorrect. Losing an extremity involves relearning how to complete ADL's in an alternative manner and usually requires using adaptive equipment. This client may be introduced to specialty equipment prior to discharge, and social services will arrange for any home care needs. However, this action is not of primary concern at the time of the clients arrival in the unit. 2. Incorrect. The loss of a limb, particularly unexpectedly, can overwhelm a client and result in feelings of shock, anger, or even denial. Clients can experience a range of emotional responses based on age, beliefs, values or social support. One common behavior among new amputees is the hesitancy to look at the stump. Forcing the client to visualize the wound before being psychologically ready can hinder or delay adaptation. Additionally, the surgeon generally removes the original surgical dressing. 4. Incorrect. This client will need to learn many adaptive skills once the surgical site has healed. However, having food precut into small pieces diminishes self-esteem and discourages client independence, which is important to recovery. An occupational therapist will be consulted regarding special eating utensils and techniques but usually there is little alteration in the preparation of food.

Based on expected growth and development for a 7 month old infant, what would the nurse anticipate that the mother would report at the infant's well-baby visit? 1. Has slight head lag when pulled to sitting position. 2. Walks holding onto furniture. 3. Able to sit, leaning forward on both hands. 4. Has neat pincer grasp.

3. Correct: A 7 month old is not expected to be able to sit fully unsupported but is able to sit by leaning forward on both hands. 1. Incorrect: No head lag should be seen when pulled to a sitting position. Head lag should end around 5 months of age. 2. Incorrect: The 7 month old is expected to be able to bear full weight on feet but generally does not walk holding onto furniture until around 11 months of age. 4. Incorrect: A neat pincer grasp does not usually develop until around 11 months of age. A 7 month old would only be expected to rake small objects with the fingers.

Which lab value on a client who is one day postpartum should the nurse report to the primary healthcare provider immediately? 1. Hemoglobin of 11 g/dL (110 g/L) (6.8266 mmol/L) 2. White Blood Cell count of 22,000 mm3 3. Hematocrit of 18% 4. Serum glucose of 80 mg/dL (4.44 mmol/L)

3. Correct: A hematocrit in postpartum women can drop as low as 20% (0.2) and not require transfusion in the absence of symptoms of hypovolemia. A hematocrit of 18% and lower should be reported even in the absence of dizziness, lightheadedness, shortness of breath with exertion, and syncope. 1. Incorrect: A hemoglobin of 11 g/dl (110 g/L) (6.8266 mmol/L) is considered to be normal for pregnancy and postpartum. 2. Incorrect: It is not unusual for a postpartum woman to have a WBC up to 25,000 mm3 without infection because of the healing process of the reproductive system. 4. Incorrect: Serum glucose of 80 m/dL (4.44 mmol/L) is within the normal range of glycemic control.

A nurse is caring for a client who had a total hip replacement 2 days ago. What assessment finding would be a priority concern for the nurse? 1. Small amount of red drainage on the surgical dressing. 2. Continues to report pain in hip when being repositioned. 3. Temperature of 101.8°F (38.7°C). 4. Slight swelling in the leg on the affected side.

3. Correct: A low grade fever is normal following hip surgery but a temperature of 101.8ºF (38.7ºC) two days postoperatively is higher than the expected slight increase and should be a priority concern. The development of an infection is one of the major complications for clients following hip surgery. Therefore, fever that persists above 101ºF that is accompanied by chills, diaphoresis, or increasing drainage and odor from the incision should be reported. 1. Incorrect: A small amount of red, bloody drainage on the dressing is expected as part of the normal healing process. 2. Incorrect: Some pain during repositioning after hip surgery is normal and can generally be managed with analgesics. 4. Incorrect: Swelling in the operative leg is a normal part of the postoperative process after hip surgery. Normal swelling is lessened in the morning but tends to re-accumulate throughout the day. This can be minimized by elevating the client's legs or having the client lie down for approximately 45 minutes during the day.

While the postpartum nurse was in report, four clients called the nurse's station for assistance. Which client should the nurse see first? 1. Client with three dime sized clots on her perineal pad. 2. Breastfeeding client who is reporting uterine cramping. 3. Client reporting blood running down legs upon standing. 4. Client who had an epidural and is now reporting a headache.

3. Correct: A new nurse should assess this client first because we are worried about hemorrhage. If the fundus is boggy, a fundal massage will need to be done. Assess vital signs for hemorrhage. 1. Incorrect: Clots smaller than a silver dollar are normal. However, do not ignore any bleeding. Always assess the client with any signs of bleeding to determine that the problem is significant. 2. Incorrect: Breastfeeding causes the release of endogenous oxytocin from the pituitary, which causes the uterus to contract. When the uterus contracts, the client may call this discomfort, cramping. This is a normal process necessary for the uterus to return to normal. 4. Incorrect: A post epidural headache can be an indication of inadvertent puncture of the dural membrane. This client will need to be positioned prone, push fluids, given caffeine and may need a blood patch to seal the dural leak.

After drawing up insulin for subcutaneous administration, the nurse receives a return phone call from a primary healthcare provider who wants to give prescription orders on a new admit. The nurse asks a new nurse to administer the insulin dose. What action should the new nurse take? 1. Administer the insulin dose to the client. 2. Consult with the charge nurse about administering the insulin dose to the client. 3. Tell the nurse that whoever draws up the medication has to administer that medication. 4. Offer to take the call from the primary healthcare provider so the nurse can administer the insulin.

3. Correct: A nurse can only administer medication that has been drawn up by that nurse. It is not acceptable practice to administer a medication drawn up by another nurse. 1. Incorrect: The nurse who gives this medication does not really know what was drawn up. It could be the wrong medication, the wrong dose, the wrong time. A nurse can only administer medication that has been drawn up by that nurse. 2. Incorrect: There is no need to consult the charge nurse because the new nurse should not administer the medication that has been drawn up by another nurse. 4. Incorrect: The nurse should first take the return phone call from the primary healthcare provider and then administer the insulin yourself.

Donepezil has been prescribed to a client with cognitive impairment. Which statement by the family member indicates understanding of the nurse's instructions on this medication? 1. This medicine will control agitation and aggression. 2. This medication should be given at bedtime since it is for insomnia. 3. Notify the primary healthcare provider if the client is vomiting coffee ground material. 4. This drug is given as needed for confusion.

3. Correct: A rare but very serious side effect that can occur: black stools, vomit that looks like coffee grounds, severe stomach/abdominal pain. Notify the primary healthcare provider immediately. 1. Incorrect: An antipsychotic medication such as risperidone is used for agitation, aggression, hallucinations, thought disturbances, and wandering. Donepezil helps to decrease the symptoms of dementia (impairment of memory, judgment, abstract thinking and personality changes) in client's with Alzheimer disease. 2. Incorrect: Donepezil should be given in the evening just before bedtime, however, it is not for insomnia. Sedative/hypnotics such as zolpidem and temzaepam are given for insomnia. 4. Incorrect: Donepezil should be given regularly in order to get the most benefit from it. Do not stop taking it or increase the dosage unless the primary healthcare provider changes the dose. It may take a few weeks before the full benefit of this drug takes effect.

A client asks, "I would like to view my medical records." Which response made by the nurse is most appropriate? 1. You will first need to contact your primary healthcare provider. 2. You may view your electronic health records on a weekly basis. 3. You have the right to view the medical records that pertain to your care. 4. You want to view your medical records?

3. Correct: According to the Client's Bill of Rights, the client has the right to view medical records pertaining to the client's care and to have those records explained if necessary. 1. Incorrect: The client may contact medical records and does not need to first contact the primary healthcare provider. 2. Incorrect: The electronic health record can be made available to the client when requested. 4. Incorrect: This is an open ended question, but the client may view this as challenging their desire to view the medical records.

The nurse is caring for a client who is receiving weekly infusions of Factor VIII for Hemophilia. What assessment finding by the nurse related to the client's skin is indicative of a therapeutic response? 1. An absence of jaundice 2. The presence of petechiae 3. A reduction of bruising 4. A capillary refill time of < 3 seconds

3. Correct: A reduction in bruising indicates an increase in circulating coagulating substances in the blood of hemophilia clients. Factor VIII is the clotting factor that is deficient in hemophilia clients. Administration of Factor VIII in these clients would result in a reduction of bleeding episodes and the s/s associated with them. 1. Incorrect: Jaundice is an indicator of elevated unconjugated bilirubin levels. Elevated unconjugated bilirubin levels are seen with liver disease and/or rapid destruction of RBCs. 2. Incorrect: Petechiae are commonly seen with thrombocytopenia and can be an indicator of decreased clotting capability of the blood. The treatment goal with hemophilia is to increase the blood's ability to clot. 4. Incorrect: Capillary refill is an indicator of tissue perfusion and not the blood's ability to clot.

The nurse is planning a teaching session with the family members of a client diagnosed with moderate Alzheimer's disease. Which topic is most important for the nurse to discuss? 1. Encouraging dependence on family members 2. Performing passive range of motion 3. Providing a safe environment 4. Monitoring vital signs every 8 hours

3. Correct: A safe home environment is a priority. When you see a safety answer always consider it. This client has a memory deficit and may wander away, leave food on the stove cooking or burn themselves with hot water. Safety is a priority. 1. Incorrect: It is important to promote independence in self-care activities to promote dignity and autonomy. The client cannot make decisions alone but the family can give the client choices to pick from., Never promote dependency. 2. Incorrect: Active range of motion and regular exercise are encouraged, but this is not the most important topic. The stem does not mention that the client is mobility impaired. Walking is usually intact until late stages of Alzheimer's. 4. Incorrect: Focus is on cognitive and behavioral symptoms. V/S would be monitored as needed. This client is at home with family and nothing indicates the need to take the client's vital signs three times a day.

The client has been diagnosed with cutaneous anthrax in a cut on the right hand. What measure should be implemented by the nurse to prevent further spread of the disease? 1. Wear mask only. 2. There are no precautions necessary. 3. Standard precautions. 4. Limit interactions with client.

3. Correct: Cutaneous anthrax is not spread person-to-person. However, it can be spread to others in rare events if the wound is draining. Standard precautions should protect the individual. 1. Incorrect: Inhalation is not the mode of transmission for cutaneous anthrax; standard precautions should be used with every client. 2. Incorrect: Standard precautions should be used in the care of every client. 4. Incorrect: Clients need personal care and assistance throughout the treatment of the infection. Stay away from answers that ask you to limit interactions with the client.

The nurse on a large surgical unit needs to evaluate several clients returning from procedures. Which client should the nurse assess first? 1. Lumbar puncture reporting a headache. 2. Cystogram reporting burning on urination. 3. Thoracentesis reporting shortness of breath. 4. Cardiac catherization with a decreased pedal pulse below insertion site.

3. Correct: A thoracentesis is performed to remove fluid from the pleural cavity and improve the client's respiratory status. This client should report an improved respiratory, not shortness of breath. The worst complication following a thoracentesis is a possible pneumothorax; therefore, the nurse should assess this client first. 1. Incorrect: A lumbar puncture involves removing cerebral spinal fluid from the subarachnoid space to diagnose specific diseases or the presence of bacteria. Headache following this procedure is a potential side effect and would not be the priority concern for the nurse. 2. Incorrect: The purpose of a cystogram is to examine the inside of the bladder to confirm the presence or absence of abnormalities, or even obtain a biopsy. Because a scope is inserted through the urethra for this procedure, the client may experience burning or frequency immediately following this test. Although this will require assessment, this client is not the priority at this time. 4. Incorrect: A slightly decreased pedal pulse to the affected extremity is not unusual following cardiac catherization. This invasive procedure results in some edema to the vessel used for the procedure but assessing only one pedal pulse does not provide sufficient data to verify a complication.

Following surgery, a client has an indwelling urinary catheter attached to a collection bag. The nurse empties the collection bag at 0900. At the change of shift at 1500, the collection bag contains 100 mL of urine. The system has no obstructions to urinary flow. What would be the nurse's most appropriate initial response? 1. Elevate the head of the client's bed. 2. Start giving the client 8 ounces of oral fluid per hour. 3. Check circulation and take the vital signs of the client. 4. Continue monitoring, because this is an expected finding.

3. Correct: A urine output (U/O) of 100 mL over a 6 hour period is dangerously low. This client could be experiencing hypovolemic shock. In clients who are "shocky", the kidneys stop making urine to try to hold on to what little volume the body has left. The nurse is checking the vital signs for low BP and increased HR, indicators of hypovolemic shock. Also, when the urine output is this low, the client is at risk for renal failure. 1. Incorrect: Elevating the head of the client's bed is a good choice when the client is having difficulty breathing, but not here. Raising the HOB will cause the BP to drop lower. Clients in shock should be supine. 2. Incorrect: Normally, pushing fluids is a good choice if the urine output were low. 100 mL over six hours requires more aggressive treatment to combat shock. 4. Incorrect: This is not an expected finding. Urine output less than 240 mL in an eight hour time frame should alert the nurse to a serious problem such as shock.

How closely monitored is access to a facility's health information system? 1. No monitoring; the system is password protected. 2. Monitored intermittently. 3. Monitored closely and constantly for inappropriate use. 4. Monitored daily and sporadically.

3. Correct: Access to a health care facility's computerized health information system is monitored closely and constantly. Records of each healthcare team member's time and date of access, as well as the information that was accessed, are kept by the information technology services department. Access can be suspended, restricted, or revoked for unauthorized or inappropriate use. 1. Incorrect: This is like doing nothing. Healthcare providers must be diligent about maintaining confidentiality, which includes the use of technology that contains confidential client information. 2. Incorrect: Intermittent monitoring is not adequate. Access should be monitored closely and constantly. A breach of confidentiality could occur if intermittent monitoring was done. 4. Incorrect: Access should be monitored closely and constantly. Sporadically and once daily is not adequate for protecting client confidentiatlity.

The nurse is caring for a diabetic client. The client's glucose level at 0700 is 265. What is the nurse's best action? *Exhibit* 40 units NPH insulin every AM Regular Insulin per Sliding Scale both AC and HS Sliding Scale: Blood glucose < 200: 0 units Blood glucose 200-249: 2 units Blood glucose 250-299: 4 units Blood glucose 300-349: 6 units Blood glucose 350-399: 8 units Blood glucose 400 or >: Call primary healthcare provider 1. Hold the NPH and regular insulin 2. Give 8 units of regular insulin and hold the NPH 3. Give the NPH and 4 units of regular insulin 4. Give 40 units of NPH and hold the regular insulin

3. Correct: According to the prescription and sliding scale, the client will need 40 units of NPH and 4 units of regular insulin for a glucose level of 250-299. 1. Incorrect: According to the prescription and sliding scale, the client will need 40 units of NPH and 4 units of regular insulin for a glucose level of 250-299. 2. Incorrect: According to the prescription and sliding scale, the client will need 40 units of NPH and 4 units of regular insulin for a glucose level of 250-299. 4. Incorrect: According to the prescription and sliding scale, the client will need 40 units of NPH and 4 units of regular insulin for a glucose level of 250-299.

A client who underwent a laparoscopic cholecystectomy is being discharged from an outpatient surgical center. Which statement by the client shows the nurse that discharge teaching has been effective? 1. I will need to eat a low fat diet since I no longer have a gallbladder. 2. I can expect drainage from the incisions for a few days. 3. I may have some mild pain from the procedure. 4. I should plan to limit my activities and not return to work for several weeks.

3. Correct: After a laparoscopic procedure the client can expect to have some mild pain. Severe pain, however, would indicate a problem. 1. Incorrect: The client can resume their usual diet. The liver will produce enough bile to digest fats. The gallbladder stores bile. Without the gallbladder, the bile just drains from the liver. 2. Incorrect: The client should not have drainage from the incisions. There are 2-3 small incisions on the abdomen that do not normally have drainage. 4. Incorrect: The client can return to normal activities in 2 to 3 days. This is not considered a major surgical procedure with a large abdominal incision. Recover time is much shorter, allowing the client to return to normal activities sooner.

A client diagnosed with hypothyroidism has received dietary education from the nurse. Which snack selection chosen by the client would indicate that education has been successful? 1. Cup of almonds 2. Cheese and crackers 3. Popcorn 4. Sweet potato fries

3. Correct: Hypothyroidism clients tend to have constipation due to decreased motility of the GI tract and need increased fiber and fluid intake. Popcorn is high in fiber. 1. Incorrect: People with hypothyroidism have a slow metabolism and do not need high protein but a well balanced diet. Almonds are high in protein. 2. Incorrect: Cheese and crackers are high in sodium. This client is at risk for CAD, so sodium should be limited. 4. Incorrect: This client does not need high potassium, which fried sweet potatoes have. The high potassium dietary approaches to stop hypertension (DASH) diet is only for healthy clients with hypertension.

What is the most effective method of stroke prevention that the nurse should teach to the public? 1. Administering platelet inhibitors to prevent clot formation. 2. Undergoing transluminal angioplasty to open a stenosed artery and improve blood flow. 3. Maintaining normal weight, exercising, and controlling comorbid conditions. 4. Administering tissue plasminogen activator (tPA).

3. Correct: Although administering platelet inhibitors, tPA, and undergoing transluminal angioplasty may improve cerebral blood flow, the goals of stroke prevention include health promotion for the healthy individual and education and management of modifiable risk factors to prevent a stroke. Health promotion focuses on a healthy diet, weight control, regular excercising, no smoking, limited alcohol consumption and routine health assessments. 1. Incorrect: This is not the most effective method to prevent a stroke. Anti platelet drugs are usually the chosen treatment to prevent stroke in clients who have had a TIA. 2. Incorrect: This is not the most effective method to prevent a stroke. Transluminal angioplasty is an invasive procedure to improve blood flow. 4. Incorrect: This is not the most effective method to prevent a stroke. tPA is administered IV to reestablish blood flow through a blocked artery in a client with acute onset of ischemic stroke symptoms.

An LPN/VN has been floated to the emergency room following a chemical plant explosion. What task would be best to assign to the LPN/VN? 1. Identify and assess each incoming client. 2. Triage and assign color-coded tags to each client. 3. Gather and apply dressings to open wounds. 4. Initiate oxygen and IV lines as needed.

3. Correct: An LPN/VN's scope of practice includes tasks such as wound care. Covering open wounds will help to decrease bacterial exposure until the registered nurse or primary healthcare provider can assess and treat each wound. If the LPN notes any serious bleeding situations, it would need reported immediately to the RN. 1. Incorrect: Although it will be crucial to identify each incoming client, the LPN/VN's scope of practice does not include assessment. That task would require an RN or primary healthcare provider. 2. Incorrect: In a mass casualty situation, triage allows the nurse or primary healthcare provider to quickly determine which clients are critical versus those stable enough to wait. Because this involves assessment, an LPN/VN would not be assigned this task. 4. Incorrect: Initiating intravenous lines is not within the scope of the LPN/VN. Additionally, the decision to apply oxygen involves assessment of the respiratory system, which also is not within the LPN/VN's scope of practice.

A client who has had a stroke presents with lethargy, facial droop, and slurred speech. The client has a history of gastroesophageal reflux disease (GERD). From this history, what does the nurse recognize as an increased risk for this client? 1. Diminished colonic motility 2. Esophageal hemorrhage 3. Aspiration pneumonia 4. Stress ulcers

3. Correct: Anyone who has had a stroke is at risk for aspiration, especially with a history of reflux disease. It is important to remember that the stomach is full of acid. When aspiration of this acid occurs, it causes irritation to the lung tissue. The client can develop a severe pneumonitis. That's what could kill the client, so this answer takes priority. 1. Incorrect: Diminished colonic motility may become a problem, but aspiration pneumonia is more acute. Remember airway, breathing, and circulation will take priority. 2. Incorrect: Esophageal hemorrhage is seen with esophageal varices, not reflux disease. 4. Incorrect: GERD is not associated with increased risk for stress ulcers, but GERD can also lead to strictures and/or precancerous lesions called Barrett's esophagus.

Which action should the nurse take for a client who is of the Roman Catholic faith? 1. Notifying dietary that all food is required to be kosher. 2. Administering last rites to the client if death is imminent. 3. Ensuring there is no meat served with meals on Fridays during Lent. 4. Positioning the dying client's bed facing Mecca (east).

3. Correct: Avoiding eating meat on Fridays during Lent is a practice of those of the Catholic faith; this action demonstrates cultural sensitivity and spiritual support. 1. Incorrect: Kosher food is required in Judaism. Kosher diet is based on a section of Jewish law which identifies which foods can be eaten. 2. Incorrect: A priest, not the nurse would administer last rites of the sick. Only a priest has been trained to celebrate the Sacrament of Reconciliation and the Anointing of the sick (last rites). 4. Incorrect: Persons of the Muslim (Islam) faith who are dying want their body turned to Mecca (east). The body or the heels should be positioned to the Mecca (east).

A concerned mother is asking the nurse about activities that would be best for her child who has been diagnosed with asthma. In order to minimize the risk of exercise induced asthma, which activity would be best for the nurse to suggest? 1. Track 2. Basketball 3. Baseball 4. Soccer

3. Correct: Baseball is an activity that is considered "asthma friendly". It requires short, intermittent periods of exertion and is therefore tolerated better by children with asthma. 1. Incorrect: This activity requires extended periods of exertion and is not considered "asthma friendly" and often not tolerated by clients with asthma. 2. Incorrect: This activity requires extended periods of exertion and is not considered "asthma friendly" and often not tolerated by clients with asthma. 4. Incorrect: This activity requires extended periods of exertion and is not considered "asthma friendly" and often not tolerated by clients with asthma. Asthma is a condition in which the airways narrow and swell and produce extra mucus. This can make breathing difficult and trigger coughing, wheezing ,and shortness of breath. Being active and playing sports is an especially good idea if a client has asthma. Why? Because it can help the lungs get stronger so they work better. Some sports are less likely to bother a person's asthma. Golf and yoga are less likely to trigger flare-ups, and so are sports like baseball, football, and gymnastics. In some sports, you need to keep going for a long time. They include cycling, long-distance running, soccer, and basketball. Cold, dry air can also make symptoms worse, so cross-country skiing and ice hockey might be difficult for people with asthma.

Which medication does the nurse expect will help decrease tremors in a client diagnosed with hyperthyroidism? 1. Steroids 2. Anticonvulsants 3. Beta blockers 4. Iodine compounds

3. Correct: Beta blockers help anxiety and tremors. Beta blockers reduce the effects of adrenaline in the body and help decrease anxiety. In times of stress and emergency the adrenal gland produces adrenaline that acts on various organs in the body to enable us to deal with the situation. For example, the heart beats faster due to adrenaline. In order for adrenaline to be able to do this, various organs have beta receptors to accept the adrenaline and use it to behave differently in times of stress. Beta blockers block these receptors. They stop various organs in the body from accepting adrenaline. Taking them means the heart does less work generally and doesn't get over-worked in times of stress. One of the main symptoms of anxiety is a speeding heart which is part of the fight-or-flight response. In times of danger our body produces adrenaline to stop the heart from beating faster makes us feel calmer. Taking beta blockers for anxiety also makes us feel less shaky. The energy boost to our muscles (from the increased supply of blood and oxygen) which makes us feel 'jittery' and 'on-edge' doesn't happen without a fast heartbeat. 1. Incorrect: Steroids influence the body system in several ways, but they are used mostly for their strong anti-inflammatory effects and in conditions that are related to the immune system function such as arthritis, colitis (ulcerative colitis, and Crohn's disease), asthma, bronchitis. Steroids are used to treat systemic lupus, severe psoriasis, leukemia, lymphomas, idiopathic thrombocytopenic purpura, and autoimmune hemolytic anemia. These corticosteroids also are used to suppress the immune system and prevent rejection in people who have undergone organ transplant as well as many other conditions. 2. Incorrect: Anticonvulsants are used to normalize the electrical activity in the brain which in turn reduces the risk of seizures. But anticonvulsants have also been shown to work on mood disorders such as depression or mania. Anticonvulsants help increase the naturally occurring nerve calming chemical known as GABA while decreasing the nerve exciting chemical known as glutamate. Tremors can actually be a side effect of anticonvulsants. 4. Incorrect: Iodine compounds decrease the production of thyroid hormones in the treatment of hyperthyroidism. It does not have an effect on tremors.

Which task would be appropriate for the charge nurse to assign to a LPN/VN? 1. Assessing a client who was just admitted to the unit. 2. Administering morphine IV push to a two day post-op client. 3. Bolus feeding a client who has a gastrostomy tube. 4. Reinserting a PICC line that a client accidentally pulled out.

3. Correct: Bolus feeding by way of a gastrostomy tube would be the best assignment for the LPN/VN. This is a nursing action that can be performed by the LPN/VN and does not require verification nor a co-signature by the RN. 1. Incorrect: The LPN/VN can collect data on a new client, but the RN must verify the information and co-sign the assessment. New admits require initial observation and data collection. From this, the RN must evaluate the information and formulate priorities of care. 2. Incorrect: Administering morphine IVP is out of the scope of practice for the LPN/VN. Therefore, this task should not be assigned to the LPN/VN. 4. Incorrect: Reinserting a PICC line is out of the realm of practice for an LPN/VN. Therefore, this task should not be assigned to the LPN/VN.

A client with the diagnosis of mild anxiety asks the nurse why the primary healthcare provider switched medications from lorazepam to buspirone. What should the nurse tell the client? 1. "Lorazepam takes longer to start working than buspirone so the primary healthcare provider decided to switch medications." 2. "Buspirone can be stopped quickly if neccessary." 3. "Buspirone does not depress the central nervous system like lorazepam does, so you should not have as much sedation." 4. "You need to ask your primary healthcare provider why the medication was changed from lorazepam to buspirone."

3. Correct: Buspirone does not depress the CNS system and is believed to produce the desired effects through interaction with serotonin, dopamine, and other neurotransmitter receptors. 1. Incorrect: Buspirone takes 1-2 weeks to take effect and can take up to 4-6 weeks to achieve full clinical benefits. Lorazepam is a benzodiazepine and begins to work within a few hours to 1-2 days. 2. Incorrect: The client should not stop taking any antianxiety medications abruptly. Serious withdrawal symptoms can occur: depression, insomnia, anxiety, abdominal and muscle cramps, tremors, vomiting, sweating, convulsions, delirium. 4. Incorrect: The nurse should be able to discuss medication administration with the primary healthcare provider.

A 70 year-old client reports not sleeping well at night, having trouble staying asleep, and awakening about 4:00 a.m. What should the nurse teach the client about sleep patterns in the elderly? 1. Don't worry about a few hours of lost sleep. 2. Elders need as much sleep as younger adults. 3. Caffeine and some medications may interfere with sleep. 4. Elders sleep more than younger adults.

3. Correct: Caffeine and some medications may interfere with sleep. 1. Incorrect. The client is concerned about the sleep problem, and the nurse should address the client's concerns. Sleep disturbances can also indicate depression. This option is denying their concerns. 2. Incorrect. Elders actually require less sleep because they are less active. Elderly do not need as much sleep. 4. Incorrect. Elders are likely to have more disturbed sleep. They usually do not need more sleep.

The nurse working in a pediatrician's office is teaching a couple with small children about proper medication administration for children. What statement by the couple would indicate that further teaching is needed? 1. We should carefully measure elixir medication with the provided dropper. 2. Our children should not watch us take medicine. 3. We tell our children the medicine is candy so they will take it without a fuss. 4. Even though medicine comes in a childproof container, we will put medication out of reach.

3. Correct: Calling medication "candy" is inappropriate and misleading to the child. Children may take medication to eat as candy if they have access to it.. 1. Incorrect: This is a correct statement by the parents. Medication should be measured closely because too much or too little might cause harm to the child. 2. Incorrect: Taking medicine in front of children is not recommended, as children often try to imitate adult behavior. 4. Incorrect: All medication should be placed out of the reach of children.

A client with a history of eczema has been admitted with cellulitis of the left forearm. Which admission order should the nurse question immediately? 1. Start IV of normal saline at 100 mL per hour. 2. Keep left arm elevated on pillow at all times. 3. Apply ice packs to affected area every shift. 4. Ibuprophen 800 mg po every 6 hours prn pain.

3. Correct: Cellulitis is a bacterial skin infection resulting in warm, redden and edematous tissue, sometimes accompanied by fever and chills. Swelling in the affected area impedes blood flow and increases pain. In order to decrease the edema, warm, moist compresses are used to stimulate circulation and speed reabsorption of the fluid within the tissue. This order should be questioned immediately. 1. Incorrect: An infection serious enough to require hospitalization indicates this client is either septic or may need intravenous antibiotics. Fluids are a primary treatment for sepsis along with bedrest and antibiotics. A basic solution of normal saline at 100/mL per hour would be an appropriate order for this client. 2. Incorrect: The swelling characteristic in cellulitis in painful and diminishes circulation to the area. Elevation on one or two pillows at all times helps to improve blood flow so that healing can take place. In some facilities, clients are provided with a wedge shaped cushion that provides better support of the affected area. This order is appropriate. 4. Incorrect: Ibuprophen provides relief from both pain and inflammation associated with cellulitis. A dose of 800 milligrams by mouth every 6 hours as needed for pain would be appropriate for this client. This is not an order the nurse should question.

A nurse is caring for a client who has chest pain. Which statement made by the client leads the nurse to suspect angina instead of a myocardial infarction(MI)? 1. I became dizzy when I stood up. 2. I was nauseated and began vomiting. 3. The pain started in my chest and stopped after I sat down. 4. The pain was not relieved after taking 3 nitroglycerine tablets.

3. Correct: Chest pain brought on by exercise and stopped with rest is the hallmark of angina. If it were an MI, the pain would continue even with rest or position changes. 1. Incorrect: This indicates orthostatic hypotension which is not definitive for angina or MI. 2. Incorrect: Vomiting is a symptom of an MI not angina and is a bad sign related to the acute pain from the MI. This type of pain stimulates the vagus nerve, which causes the heart rate, BP and cardiac output to decrease and this is never good with a heart client. 4. Incorrect: This is the picture of MI, not angina

A client with a history of intolerance to fatty foods is admitted to the hospital with a sudden onset of severe right upper quadrant pain radiating to the right shoulder. What should be included in the nurse's initial focused assessment of this client? 1. "Do you have pain in the middle of your stomach that is relieved by vomiting?" 2. "Have you noticed any red splotches on your skin?" 3. "Please describe your bowel habits and stool." 4. "Tell me how often you eat high fat meals."

3. Correct: Clay colored stools are a sign of biliary obstruction and are due to lack of bile in the stool. Bile adds a darker color to the stool. Asking the client to describe stool is open ended and will give the nurse more detail. 1. Incorrect: Epigastric pain relieved by vomiting is found with clients who suffer from peptic ulcers. 2. Incorrect: Spider angiomas are seen in clients with liver disease 4. Incorrect: This does not relate to the client's pain and will not obtain needed information about the client's current condition.

Which task would be appropriate for the Labor, Delivery, Recovery, Postpartum (LDRP) charge nurse to assign to an LPN/VN? You answered this question 1. Administering IV pain medication to a client three days postopertive cesarean section. 2. Drawing a trough vancomycin level on a client 3 days postpartum with bilaterial mastitis. 3. Reinforce how to perform perineal care to a primipara who is four hours postpartum. 4. Drawing routine admission labs on a client admitted in final stages of labor.

3. Correct: Client teaching may be reinforced by an LPN/VN on a stable client. 1. Incorrect: Administering IV pain medications is out of the scope of practice of LPN/VN. 2. Incorrect: Drawing lab work on a client with severe infection and only 3 days postpartum is an unstable client and needs care from the RN. 4. Incorrect: Drawing routine admission labs on a client in final stages of labor would be inappropriate because the client is potentially unstable and needs experienced LDRP nursing care.

The nurse is passing morning medication on a busy medical-surgical unit and has been delayed in completing rounds. When re-evaluating how to distribute the remaining scheduled medications, which client would the nurse consider at greatest risk if medications are late? You answered this question 1. The client with congestive heart failure receiving digoxin. 2. The client with epilepsy scheduled to receive phenytoin. 3. The client with myasthenia gravis on pyridostigmine. 4. The client with hypertension due for daily nifedipine.

3. Correct: Clients diagnosed with myasthenia gravis (MG) need a precise medication administration schedule since drugs such as pyridostigmine have a very short half-life. These medications are used to help decrease the weakness typical in MG clients, and the meds are only effective for 4 to 6 hours. It would be especially imperative to deliver the morning dose on time so that the client would not experience weakness during breakfast, increasing the risk for aspiration. 1. Incorrect: Digoxin is a cardiac glycoside which increases the contractility of the heart muscle while decreasing pulse rate, thus making the heart work more efficiently. This medication is generally given once daily and as such does not present dire consequences to the client if the dose is slightly later than usual. 2. Incorrect: The anti-seizure medication phenytoin is generally given once daily and has a very long half-life. Even if this medication is scheduled twice daily, the client is not likely to suffer any complications if the morning dose is late. 4. Incorrect: Nifedipine is a calcium channel blocker used to prevent daily angina episodes. Although these meds can be administered twice daily, this client is to receive just a daily dose and would not experience any complications from a slightly delayed morning dose.

Which nursing action takes priority once a term infant has delivered vaginally? 1. Apply identification bands 2. Apply eye ointment 3. Dry the baby 4. Obtain footprints

3. Correct: Cold stress is the biggest danger to a newborn. A newborn is wet, and evaporation will rapidly cool the baby, which can cause hypoglycemia and respiratory distress. The stimulus of drying the skin also promotes vigorous crying and lung expansion in most healthy infants. 1. Incorrect: A task that needs to be done before the baby leaves the delivery room, but is not immediate priority. 2. Incorrect: Eye prophylaxis may be delayed until the end of the first hour after birth without adverse effects. Because the ointment may temporarily blur the infant's vision, parents may wish to delay treatment for a short time during initial bonding. 4. Incorrect: A task that needs to be accomplished before the baby leaves the delivery room, but is not immediate priority.

A child presents to the school nurse with left knee pain after suffering a fall on the playground. Which action should the nurse initiate? 1. Instruct the child to extend the affected knee 2. Perform range of motion exercise on both knees 3. Compare the appearance of the left knee to the right knee 4. Have the child soak the affected knee in warm water

3. Correct: Comparing the appearance of the left knee to the right knee is the least invasive assessment and allows the nurse to assess if there is a change in the appearance of the affected knee to the unaffected knee. 1. Incorrect: The extent of the injury is not known until after an assessment is done. Remember the nursing process here. Assess first. Extending the affected knee may cause further damage. 2. Incorrect: You don't want the child to move the extremity prior to assess for broken bones. Range of motion exercises may cause further damage to the affected knee. 4. Incorrect: Soaking the affected knee in warm water will not help the nurse assess whether or not an injury occurred. This child has been injured and the nurse does not know the extent of the injury. So assess first. Options 1, 2, and 4 are interventions. Option 3 is assessment, making it the unique option. Additionally, what comes first in the nursing process? Assessment.

A client is sedated. His wife asks the nurse about her husband's test results. The client does not have a healthcare proxy or durable power of attorney executed at this time. How should the nurse respond in compliance with HIPAA (Health Insurance Portability and Accountability Act) regulations regarding the confidentiality of the sedated client's health information? 1. I can't give you those results. You should ask his primary healthcare provider the next time that he comes in to examine your husband. 2. Those test results are confidential, but since you are his wife I can give them to you. Let me look them up in the computer system. 3. The health information of all clients is confidential and is protected by law. Those test results cannot be released without the consent of the client in order to protect the client's right to choose who receives health information. 4. Your husband is only lightly sedated. I can wake him up and ask him if it is all right to release these test results to you.

3. Correct: Each client's health information is confidential and protected by law. The nurse should inform the client's wife of this fact, and explain the rationale for health information confidentiality. Family members are often offended or angry upon learning that health information cannot be released to them without the client's consent , but healthcare employees are bound by law to confidentiality. 1. Incorrect: The wife is not automatically able to receive personal health information about her husband.The husband has to list the wife as a person who can receive personal health information. The Healthcare employees, including primary healthcare providers, are bound by law to keep health information confidential without the client's consent. 2. Incorrect: Healthcare employees, including primary healthcare providers, are bound by law to keep health information confidential without the client's consent. 4. Incorrect: A client who has received sedative medications cannot give legal consent, as these medications alter a client's level of consciousness and impair the ability to make informed decisions.

Which statement by the spouse of a client diagnosed with Alzheimer's indicates to the nurse that the spouse is dealing appropriately with stressors? 1. "I am in charge of every aspect of the care provided." 2. "I do not expect our children who live out of town to help." 3. "I keep a list of small tasks ready for people who ask me if they can help." 4. "I only go to my primary healthcare provider when I am sick."

3. Correct: Encourage caregivers to say "yes" when someone offers assistance. It's smart to have a list ready of small tasks that others could easily take care of, such as picking up groceries or driving the person to an appointment. 1. Incorrect: The caregiver should be willing to surrender some control. Delegating is one thing. Trying to control every aspect of care is another. People will be less likely to help if the caregiver micromanages, or insists on doing things their way. 2. Incorrect: The caregiver should spread the responsibility. Get family members involved as much as possible. Even someone who lives far away can help. Encourage the caregiver to divide up caregiving tasks. One person can take care of medical responsibilities, another with finances and bills, and another with groceries and errands. 4. Incorrect: Encourage the caregiver to stay healthy by keeping on top of primary healthcare provider visits. They should not skip annual routine, checkups, or medical appointments.

The nurse is caring for a client undergoing electroconvulsive therapy (ECT) for major depression. What is the nurse's most important intervention during the treatment? 1. Monitor vital signs and cardiac functioning. 2. Provide support to the client's arms and legs. 3. Provide suctioning as needed. 4. Place electrodes on temples.

3. Correct: Ensuring patency of the airway is the nurse's first priority. The client should also be NPO for 6-8 hours prior to the procedure. 1. Incorrect: This is an intervention, but does not come before airway. Vital signs do need to be monitored but the client's breathing is a higher priority. 2. Incorrect: This is an intervention, but does not come before airway. Support the extremities due to the seizure activity but highest priority remains airway. 4. Incorrect: This is done prior to initiation of the procedure to deliver the electrical stimulation. The question asks for the most important intervention during the treatment.

What should the nurse tell a 68 year old client who states that they have started experiencing tremors? 1. "This is nothing to worry about and is common with aging." 2. "You should increase your intake of potassium." 3. "We need to let your primary health care provider know because it may indicate a problem." 4. "Have someone check your blood pressure the next time you experience tremors."

3. Correct: Fine tremors are the first symptom reported in 70% of client's diagnosed with Parkinson's Disease. 1. Incorrect: Tremors are not a normal age change. 2. Incorrect: Tremors may indicate a problem. 4. Incorrect: Tremors may indicate early onset Parkinson's Disease. 1. Look at each option as True or False. 2. Option 1 is false. Tremors are not a normal part of the aging process. 3. Option 2 is false. The cardinal signs of Parkinson's Disease are tremor, rigidity, and bradykinesia. 4. Option 3 is true. Slow unilateral resting tremors are present in the majority of clients at the time of diagnosis of Parkinson's Disease. 5. Option 4 is false. Parkinson's Disease is diagnosed clinically from a client's history and the presence of two of four manifestations: tremors, rigidity, bradykinesia, and postural changes.

A high school nurse is assessing multiple students reporting general flu-like symptoms. Which additional symptoms reported by a student would prompt the nurse to immediately call an ambulance? 1. Blurred vision and Trousseau's sign. 2. Vomiting and a Murphy's sign. 3. Sensitivity to light and Kernig's sign. 4. Fever and a Chvostek's sign.

3. Correct: Flu-like symptoms, sensitivity to light and a positive Kernig's sign, often accompanied by neck stiffness, suggests possible meningitis, a serious inflammation of the brain and meninges. Kernig's sign is elicited by placing the client in a supine position with one leg flexed up toward the abdomen. The lower leg is then gently extended upward, producing severe pain in the presence of meningitis. It is important to isolate the student until the type of meningitis is diagnosed. Because bacterial meningitis is contagious, the student should be transported to the emergency room immediately to initiate definitive diagnosis and treatment. 1. Incorrect: Blurred vision in the presence of flu-like symptoms could indicate a number of non-life threatening factors, including dehydration or fatigue. Trousseau's sign is used to assess for tetany with suspected hypocalcemia. It is elicited by applying and inflating a blood pressure cuff while observing for hand spasms. These symptoms would not require immediate transport to the hospital. 2. Incorrect: Although vomiting is unusual in cases of the flu, a Murphy's sign is an indication of possible gall bladder inflammation. Placing fingers under the right rib cage and asking the client to take a deep breath causes intense pain that indicates diseased gall bladder. The presence of either of these signs would not require urgent transport to a hospital. 4. Incorrect: Fever is not uncommon for those with the flu. However, Chvostek's sign is an assessment tool used to determine possible hypocalcemia. To elicit this sign, the client's cheek is tapped gently, which causes muscle spasms on the opposite cheek. Neither of these symptoms would require immediate transport to the hospital.

The nurse is caring for a client with a closed head injury. Three days after admission, urinary output for 8 hours was 1800 mL. In response to this data, what would be the appropriate nursing action? 1. Hydrate the client with 500 mL of IV fluid in the next hour. 2. Monitor BUN and creatinine. 3. Check urine specific gravity. 4. Recognize this as a side effect of dexamethasone.

3. Correct: For any client with a head injury and abnormally high urinary output, the nurse knows the client is at risk for ADH (anti-diuretic hormone) problems. The pituitary gland is located in the brain. ADH is produced in the pituitary gland. In head injured clients, ADH can get messed up. If the client does not have enough ADH large volumes of water will be lost in the urine. The name of this disease is diabetes insipidus (DI). Large volume losses place the client at risk for shock. The nurse knows to further investigate the problem by checking a urine specific gravity. For clients in DI, the urine specific gravity will be very, very low because they are losing so much water. When you see the letters DI, think of the "D" for diuresis and think SHOCK first. 1. Incorrect: Administration of 500 mL of fluid over one hour is possible if the client were in shock. The stem of the question, however, does not indicate this client is in shock. 2. Incorrect: Monitoring BUN and creatinine does not help identify diabetes insipidus. 4. Incorrect: Decadron can cause fluid retention, not increased urinary output.

What information should be included when a nurse is teaching a group of college students about the transmission of hepatitis B and human immunodeficiency virus (HIV)? You answered this question 1. HIV is transmitted via toilet seats whereas hepatitis B is not. 2. HIV is transmitted by sexual contact whereas hepatitis B is not. 3. Hepatitis B is more readily transmitted via needle sticks than HIV. 4. Neither virus is transmitted via body fluids.

3. Correct: Hepatitis B virus (HBV) and HIV can be transmitted in similar ways, but hepatitis B is more infectious. Studies show hepatitis B is more readily transmitted via needle sticks than HIV. More than 1 million people currently have HIV in the United States. Hepatitis B is 50-100 times more infectious then HIV. 1. Incorrect: Neither virus is transmitted via toilet seats. Both are spread by contact with infected body fluids such as blood, semen and vaginal fluid, or from a mother to her baby during pregnancy or delivery. 2. Incorrect: Both hepatitis B and HIV are transmitted via body fluids through sexual contact. Therefore, condoms should be used during sexual contact. Using a latex condom reduces the chances of hepatitis B and HIV being passed on during sex. Syringes and other injecting drug equipment should never be shared. 4. Incorrect: Both hepatitis B and HIV are transmitted via body fluids through sexual contact. Standard precautions should be implemented for both HIV and hepatitis B. The CDC recommends HBV vaccination for people who are at risk for or living with HIV, including men who have sex with men (MSM); people who inject drugs; household contacts and sex partners of people who have HBV; people with multiple sex partners; anyone with a sexually transmitted infection; people with diabetes; and health care and public safety workers who may be exposed to blood on the job. Look at options 2 and 4. Option 2 says that HIV is transmitted by sexual contact, but option 4 says that neither virus is transmitted by body fluids. So one of these options must be wrong. If you know that at least one or both of these diseases is transmitted via body fluid then you can automatically eliminate option 4. Look at option 2 now. We know that HIV is transmitted sexually. That's true. But what about Hepatitis B? It is also transmitted sexually, so option 2 is wrong. Look at option 1. Both of these disease will not live outside of body fluid, so cannot live on a clean toilet seat. Keep in mind that dried blood can have the disease, however. That leaves option 3. Even if you did not know that Hepatitis B is more readily transmitted via needle sticks than HIV, you should be able to easily eliminate the other options.

A client in the manic phase of bipolar disorder is constantly walking around the day room and refuses to sit down to eat the spaghetti and meatballs sent by the kitchen. Which food should the nurse request from dietary? 1. Carrots and apples 2. Donuts 3. Pepperoni pizza sticks 4. Strawberry pastry

3. Correct: High protein, high calorie, nutritious finger foods are required when the client will not sit down to eat. This client needs food they can eat "on the go" because they are burning more calories in this phase of bipolar disorder. 1. Incorrect: Although nutritious, these foods are not high calorie or high protein. 2. Incorrect: Donuts are high in calories but do not have high nutritional valve. 4. Incorrect: Pasties are also high in calories but do not have high nutritional valve. They are also not very easy to eat "on the go"

A client in active labor has an epidural catheter inserted for management of pain. Which finding should the nurse report to the primary health care provider? 1. Early decelerations 2. Fetal heart rate (FHR) 160/min 3. Blood pressure 90/62 4. Temperature of 99.6° F (37.5° C).

3. Correct: Hypotension is an adverse effect of epidural analgesia due to vasodilation. Maternal hypotension reduces blood supply to the placenta, decreasing fetal oxygen supply. Immediate intervention is required. 1. Incorrect: Early decelerations are not associated with fetal compromise and require no intervention. 2. Incorrect: The fetal heart rate is within normal range. The normal fetal heart rate (FHR) averages from 110 to 160 beats per minute (bpm) for a full term baby. 4. Incorrect: Maternal fever after epidural analgesia is common, but this is a slight elevation in temperature which is not life threatening.

A float nurse arrives on the unit to assist in the care of clients for the shift. During report, the charge nurse notes that the float nurse appears disheveled, flushed, and is trembling slightly while drinking coffee. Based on this information,what should the charge nurse do? 1. Ask the float nurse, "Have you been drinking?" 2. Assign the float nurse to the least acute clients. 3. Notify the nursing supervisor of the observations. 4. Notify the board of nursing (BON) that the float nurse is an alcoholic.

3. Correct: If suspicious behavior occurs, it is important to keep careful, objective records. Confrontation should occur in the presence of a supervisor or other nurse and should include the offer of assistance in seeking treatment. This can prevent harm to client's and save the nurse's career or life. 1. Incorrect: If alcohol or drug dependency is suspected, confrontation will result in hostility and denial. The charge nurse should not lecture, scold or argue with the float nurse. 2. Incorrect: This response overlooks a potentially severe problem. Nurses dependent on drugs or alcohol can harm clients. The nurse should not be assigned to provide care if impairment is suspected. Patient safety must remain the priority. 4. Incorrect: If a report is made to the BON, it should be a factual documentation of specific events and actions, not a statement of impairment. The report should contain consequences. Each state BON differs in that also some have treatment programs they administer themselves.

A client prescribed oral iron medication is reporting nausea after administration. What should the nurse teach the client to decrease this symptom? 1. Take the iron with a class of milk. 2. Eat bran cereal immediately after ingesting iron. 3. Drink orange juice with the iron medication. 4. Take docusate sodium at bedtime.

3. Correct: Iron is best absorbed on an empty stomach, however, if nausea and vomiting occur, drink orange juice with the iron. It will help decrease nausea and vomiting, and will enhance absorption of the iron. 1. Incorrect: Do not take iron with milk, calcium and antacids as they bind with iron to decrease amount delivered to the body. The client should wait at least two hours after having these before taking iron supplement. 2. Incorrect: Foods that affect absorption and should not be eaten at the same time as iron is taken include: high fiber foods such as whole grains, bran, and raw vegetables. 4. Incorrect: Docusate sodium is a stool softener used to treat constipation which can occur with iron intake. But it does not help nausea.

A client has been admitted with advanced cirrhosis. The nurse's assessment reveals an abdominal girth increase of 5 inches (12.7 cm) and a weight increase of 6 lbs. (2.72 kg) since yesterday's measurements. Based on this data, what would be the nurse's priority assessment? 1. Stool for occult blood 2. Ammonia blood level 3. Blood pressure 4. Level of consciousness

3. Correct: In ascites, the client is in FVD and we worry about shock. If my blood pressure drops, I will have decreased perfusion of my vital organs. Poor perfusion leads to organ damage and failure. 1. Incorrect: We are worried about bleeding because the liver is sick but not first priority in the case. 2. Incorrect: Ammonia level would indicate a worsening liver, but not a priority here. 4. Incorrect: LOC does tell us perfusion but may be affected by other factors as well. BP will tell us the most about shock and that is what I am worried about here.

Which action would the nurse need to perform to increase stability while initiating a client transfer? 1. Lift with the back. 2. Put on a back belt. 3. Spread feet to width of the shoulders. 4. Lean forward slightly.

3. Correct: In order to increase stability, the nurse will need to increase the base of support. This can be done by spreading the legs to the width of the shoulders. 1. Incorrect: Do not use your back to do heavy lifting. They are not your strongest muscles. Use your legs. 2. Incorrect: A back belt will not increase the base of support. 4. Incorrect: The nurse should not lean forward or backward. The ears, shoulders, hips and feet should be aligned. Keep a wide base of support. The feet should be shoulder-width apart, with one foot slightly ahead of the other (karate stance). Squat down, bending at the hips and knees only. If needed, put one knee to the floor and other knee in front of you, bent at a right angle (half kneeling). Keep good posture. Look straight ahead, and keep back straight, chest out, and shoulders back. This helps keep upper back straight while having a slight arch in lower back. Slowly lift by straightening hips and knees. Keep back straight, and don't twist while lifting. Hold the load as close to the body as possible, at the level of the belly button. Use the feet to change direction, taking small steps. Lead with the hips as you change direction. Keep shoulders in line with hips while moving. Set down the load carefully, squatting with the knees and hips only.

A client receiving palliative care is reporting constipation. What intervention should the palliative care nurse provide first? 1. Increase foods high in fiber. 2. Administer an enema 3. Increase fluid intake 4. Administer docusate sodium

3. Correct: Increase fluid intake is correct. Dehydration is one of the most common causes of constipation. Fluids keep your stool soft and easy to pass. 1. Incorrect: Fiber should be increased. This is true but water is the first intervention that should be implemented. Fiber will increase bulk and help with passage of stool but fluids should be first. 2. Incorrect: Administering an enema would not be the first thing to try for constipation. Least invasive first. Avoid medicines as long as possible. 4. Incorrect: Docusate sodium is colace and a stool softener, although appropriate avoid medicines as long as possible.

The nurse caring for a client who had a transurethral resection of the prostate (TURP) would increase the flow of the continuous bladder irrigation for which assessment data? 1. The drainage is continuous but slow. 2. The drainage is cloudy and dark yellow. 3. The drainage is bright red. 4. No drainage of urine or irrigation solution is noted.

3. Correct: Indicates blood and increasing the flow helps flush the catheter. 1. Incorrect: Continuous irrigation causes continuous drainage. 2. Incorrect: The color is noted and color from pink to amber is expected. 4. Incorrect: Indicates a possible obstruction. 1. Look at each option as True or False. 2. Option 1 is false. Continuous drainage is expected with continuous irrigation. The drainage bag is monitored for bloody urine. 3. Option 2 is false. Urine is carefully noted and documented. A change in color from pink to amber indicates reduced bleeding. 4. Option 3 is true. The irrigation is used to prevent clot formation. Increasing the rate of solution helps flush the catheter. 5. Option 4 is false. Gentle irrigation of the catheter may be prescribed to remove any obstructing clots. Increasing the flow would be contraindicated when there is no return of urine and irrigating solution.

Which statement by a client diagnosed with infectious mononucleosis indicates to the nurse that education has been successful? 1. "I should let my primary healthcare provider know if I start having pain in the side of my stomach" 2. "I can return to my normal activities in 5 days." 3. "I will not let others drink from my glass." 4. "My immediate family needs to get vaccinated against mononucleosis."

3. Correct: Infectious mononucleosis, caused by the Epstein-Barr virus is transmitted by saliva and intimate physical contact like kissing, sharing of utensils, and eating/drinking after others. 1. Incorrect: The client should observe for left upper quadrant abdominal pain radiating to the left scapula as this is an indicator of splenic rupture, a complication of infectious mononucleosis. 2. Incorrect: This is too soon. Most people get better in 2 to 4 weeks; however, some people may feel fatigued for several more weeks. Occasionally, the symptoms of infectious mononucleosis can last for 6 months or longer. 4. Incorrect: There is no vaccine to protect against infectious mononucleosis. The best way of the Epstein-Barr virus is to eliminate contact with oral secretions.

The labor nurse is assessing a client admitted in preterm labor. Which client finding would require a social service consult? 1. Very quiet and avoids eye contact. 2. Reports that she is not married. 3. Has injuries in various stages of healing. 4. Reports frequent arguments with her partner.

3. Correct: Injuries in various stages of healing indicate a pattern of abuse. Abuse not only harms the mother, but also increases the risk of fetal harm or death and preterm delivery. 1. Incorrect: While these signs may indicate abuse, cultural differences may explain her demeanor. 2. Incorrect: Single parenthood is not an indicator for referral to social services. 4. Incorrect: Frequent arguments with her partner are not an indicator for referral to social services. This could also be from other stressors in the client's life, emotional mood swings from hormone changes, or other factors unrelated to the pregnancy.

A nurse is preparing to administer an insulin infusion to a client. The nurse calculates the infusion pump setting as 9 mL/hr. What should the nurse do next? 1. Administer the calculated medication dosage. 2. Call the primary healthcare provider to clarify the dosage. 3. Ask another nurse to calculate the dosage. 4. Notify pharmacy of the pump setting for the calculated dosage.

3. Correct: Insulin is a high alert drug and must be double checked by another nurse before it is administered. High alert drugs that could have significant side effects if administered improperly. 1. Incorrect: Insulin is a high alert drug and must be double checked by another nurse before it is administered. High alert drugs that could have significant side effects if administered improperly. 2. Incorrect: Calling the primary healthcare provider is inappropriate. The nurses are trained to properly calculate this drug calculation problem. 4. Incorrect: The nurse will calculate the infusion rate and then have a second nurse verify the rate. There is no reason to notify pharmacy.

A new admit arrives to the nursing unit with one thousand dollars in cash. What would be the best action by the nurse to safeguard the client's money? 1. Insist the money go home with the client's visitor. 2. Place the money in the client's bedside table drawer. 3. Put itemized cash in envelope and place in hospital safe. 4. Lock money up in narcotic cabinet with client's identity and room number.

3. Correct: It is best to have two witnesses (preferably hospital staff) sign the inventory list. The best action by the nurse would be to itemize the valuables, place in an envelope with the record of the inventory, and have it put in the hospital safe. If you fail to properly safeguard the client's property, the trust of the healthcare team for medical care can also be lost. Liability waivers should be signed if for whatever reason, the valuables must remain at the hospital. 1. Incorrect: This is not the best option. The visitor may not be the best person to send the money with. The client also has the right to refuse. Sending the money home with someone else does not safeguard the client's valuables and puts the nursing unit at risk for liability if a liability waiver has not been signed. 2. Incorrect: This is not a safe option. Anyone could retrieve the money. This would be considered careless actions by the nurse and could cause a lack of trust in the entire healthcare team. 4. Incorrect: This is not a safe option. Anyone with access could retrieve the money. Although it is in a locked area, it does not need to be placed with narcotics where the cabinet would be accessed by multiple people. This would still be considered a failure to properly safeguard the client's valuables.

The nurse is caring for a client in the Emergency Department (ED) who reports a migraine headache unrelieved by over the counter medications. This is the 4th visit to the ED for this problem in 6 weeks. What is the priority nursing intervention? 1. Refer the client to their primary healthcare provider in the morning. 2. Make the client an appointment with the chronic pain clinic. 3. Rate the client's pain using the pain scale used in the ED. 4. Perform a visual acuity test.

3. Correct: Just because a client is a frequent visitor to the emergency department reporting migraines does not mean that the client is addicted to narcotics or that the client is not really experiencing the pain. Pain is what the client says it is and assessment is priority. 1. Incorrect: This is delay of treatment and does not address the pain. The nurse should have the client rate the pain in order to become objective data. 2. Incorrect: This is the primary healthcare provider's decision and also indicates you think the pain is not real. Assessment by the nurse and primary healthcare provide are warranted. Don't delay treatment. 4. Incorrect: Assessment of the eyes could be an option since eye strain can lead to headaches. Rating their pain would be the priority assessment however.

The nurse is teaching a pregnant teenage client about resources available through the health department. The client says, "I am not sure that I want to have this baby. What do you think about an abortion?" What should the nurse say? 1. What does the baby's father think about an abortion? 2. I know this is a difficult decision. 3. What are your thoughts about abortion? 4. There are many options other than abortion.

3. Correct: It is important that the client talk about her thoughts regarding abortion. The nurse must be careful to protect the client's right to autonomy without imposing personal values onto the client if not solicited. However, the nurse should talk with the client and explore her fears, feelings, and available options. Once the client's choice is made, the client can be referred to the appropriate services. 1. Incorrect: This is asking about another person's opinions and not assisting the client to formulate their opinion. Decisions that are made based on other's thoughts and opinions may lead to regret or guilt later. 2. Incorrect: The nurse is responding with sympathy and not assisting the client exploring their feelings. Since the client specifically asked the nurse about feelings related to abortion, the nurse should answer the client in a way that gets the client to explore her own feelings, thoughts, and concerns. 4. Incorrect: To move to other options immediately discounts the client's right to autonomy and the need to discuss this very personal issue with the nurse. The nurse should use the opportunity to first explore her concerns, feelings and thoughts. It is appropriate to discuss available options with her in order for her to make her own decisions.

After making initial assessment rounds on assigned clients in the morning, the RN tells the charge nurse that the clients are too difficult. The RN requests reassigning at least one of the clients to another nurse. What is the best response by the charge nurse? 1. Offer to take one of the clients. 2. Notify the nursing supervisor of the situation. 3. Ask the RN why the assignment is too heavy. 4. Explain to the RN that all the nurses have the same number of clients.

3. Correct: It would be best to explore the reason the RN thinks the assignment is too heavy. The charge nurse needs additional information to make a decision. This will allow the charge nurse to analyze the situation to make a better decision as to whether the assignment should be changed. 1. Incorrect: Volunteering to take a client would add more work to the charge nurse when this might not be necessary. The charge nurse's best response is to first obtain the needed information to make the best decision. 2. Incorrect: The charge nurse should first obtained the needed information and then decide whether to notify the nursing supervisor. The situation should be explored before bringing the supervisor in on the situation. 4. Incorrect: It is important to hear what the nurse is saying and not to dismiss the request by refusing to reassign the clients. Something new could have occurred with the clients, making the assignments too heavy. The charge nurse might not have realized all the responsibilities of taking this team of clients. Client assignments are based on client acuity and nurses do not necessarily have the same number of clients.

The first day of a pregnant client's last menstrual cycle was October 20th. What does the nurse calculate as the client's expected date of confinement? 1. June 27 2. July 20 3. July 27 4. August 27

3. Correct: Naegele's rule is often used to establish the expected date of delivery. This method involves subtracting 3 months from the date of the last normal mentrual period (LMP) began, adding 7 days and then correcting the year if appropriate. So July 27th is correct. 1. Incorrect: This is an incorrect calculation of Naegele's Rule. 2. Incorrect: This is an incorrect calculation of Naegele's Rule. 4. Incorrect: This is an incorrect calculation of Naegele's Rule.

The nurse is monitoring the healing of a full-thickness wound to a client's right thigh. The wound has a small amount of blood during the wet to dry dressing change. What action should the nurse initiate next? 1. Notify the primary healthcare provider. 2. Obtain wound culture. 3. Document the findings. 4. Remove dressing and leave open to air.

3. Correct: Look at the clues: full thickness wound, small amount of blood, wet to dry dressing. With a full thickness wound there is destruction of the epidermis, dermis, and subcutaneous tissues going down to the bone. ​So you would expect to see a small amount of blood or drainage wouldn't you? Yes. This is expected. Simply document this normal finding. 1. Incorrect: Is there really anything to worry about in this situation? No, so you do not need to notify healthcare provider. Now, with most questions on NCLEX there is something to worry about but just not with this one. 2. Incorrect: No, bleeding is not a sign of infection which is what you would be worried about if you got a wound culture. 4. Incorrect: Probably not, just a sign of blood flow in healing wound. Wet to dry dressing helps to debride the wound. So if you remove the dressing will debridement occur? No.

A female client has used medroxyprogesterone acetate injections for birth control for several years. For the past 6 months, attempts to become pregnant have been unsuccessful. What instruction should the nurse provide to the client? 1. Be seen in the fertility clinic by a primary healthcare provider who specializes in this problem. 2. Have a sperm count performed on the client's partner. 3. Be aware that ovulation may not occur for many months after using medroxyprogesterone acetate. 4. Ensure proper nutrition, rest, and establish an exercise program.

3. Correct: Medroxyprogesterone acetate is an injectable progestin that prevents ovulation for 14 weeks (although injections should be scheduled every 12 weeks). After discontinuing injections, it may take approximately 9 to 10 months to reestablish normal ovulation and menstruation. 1. Incorrect: A fertility workup for the client and her partner may be warranted after adequate time to reestablish ovulation has passed. Fertility is not expected to return until approximately 9 to 10 months and this couple has only been attempting a pregnancy for 6 months. 2. Incorrect: A sperm count on the client's partner may be warranted after adequate time to reestablish ovulation has passed. 4. Incorrect: Good nutrition, rest, and exercise are important for all individuals, but does not apply to this client's concerns.

The nurse is caring for an elderly client who is approaching death and expressing intense despair and anxiety. Based on Erikson's theory, the nurse recognizes that this client's despair and anxiety would most likely be based on what? 1. An inappropriate desire for youthfulness and staying young. 2. The decision to never marry. 3. The lack of a sense of wholeness, purpose, and a life well lived. 4. The fear of experiencing a painful death.

3. Correct: Older adults who view their lives as purposeful and full have an increased ability to view death as a meaningful part of life. 1. Incorrect: As people age they lose physical function and don't look as youthful as they once did. But the stage the elder adult faces is despair and anxiety regarding the life lived. 2. Incorrect: Individual choice that may or may not lead to satisfaction. 4. Incorrect: Conversely, older adults who view their lives as meaningless and full of lost opportunities view their approaching death with despair and conflict.

Which goal is the most important for the nurse to address for a client admitted to the cardiac rehabilitation unit? 1. Reduction of anxiety 2. Referral to community resources 3. Identification of lifestyle changes 4. Verbalization of energy-conservation techniques

3. Correct: On admission, the best starting point is to survey what is good and what needs to be changed. 1. Incorrect: No, people need some anxiety to change. 2. Incorrect: Not yet.This may be done, but it is not the most important thing right now. 4. Incorrect: For cardiac rehab we want to exercise, not conserve, at this point. Conserving energy is for times of hypoxia or angina.

Which goal is the most important for the nurse to address for a client admitted to the cardiac rehabilitation unit? 1. Reduction of anxiety 2. Referral to community resources 3. Identification of lifestyle changes 4. Verbalization of energy-conservation techniques

3. Correct: On admission, the best starting point is to survey what is good and what needs to be changed. 1. Incorrect: No, people need some anxiety to change. 2. Incorrect: Not yet.This may be done, but it is not the most important thing right now. 4. Incorrect: For cardiac rehab we want to exercise, not conserve, at this point. Conserving energy is for times of hypoxia or angina.

A client is admitted to the hospital reporting chills, fatigue and left lower leg pain for nearly a week. During initial assessment, the nurse notes wide-spread swelling and redness of left ankle in addition to a fever of 103.5° F (39.72° C). Which admission order should the nurse implement first? 1. Perform sterile wound care to lower leg. 2. Start I.V. for administration of antibiotics. 3. Place client on bedrest with left leg elevated. 4. Draw blood for serial cultures and lab work.

3. Correct: Osteomyelitis is a serious inflammation of bone tissue caused when bacteria or fungus has entered the body through an open wound, an infected prosthetic, or even animal bite. Symptoms include fever, chills, nausea, and fatigue with decreased mobility in the affected extremity. The client can quickly become septic as the illness spreads through the system. Bedrest along with massive doses of antibiotics are necessary to prevent the spread of the infection, resulting in possible bone death or even amputation. 1. Incorrect: Since the nurse is in the process of admitting this client, wound care is not a priority action. There are more urgent orders to be implemented in order to stabilize the client. 2. Incorrect: Intravenous antibiotics are generally prescribed for up to six weeks, and the client may need a PICC line to continue antibiotic therapy in the home setting. While starting an I.V. line for antibiotic administration is important, this is not the most crucial first action. 4. Incorrect: Lab tests can provide valuable diagnostic information about clients with osteomyelitis. The Healthcare provider would most likely order a complete blood count (CBC) and sediment rate, expecting elevations in both. Blood cultures would also confirm whether the infection has become systemic. However, a venipuncture can wait until a more important action has been completed.

A client is admitted with an acute episode of diverticulitis. What symptom would the nurse promptly report to the primary healthcare provider? 1. Midabdominal pain radiating to the shoulder 2. Nausea and vomiting periodically for several hours 3. Abdominal rigidity with pain in the left lower quadrant 4. Elimination pattern of constipation alternating with diarrhea

3. Correct: Pain in the lower left quadrant with abdominal rigidity indicates the client is experiencing a perforated diverticuli and is a medical emergency. Abdominal rigidity indicates either perforation or internal bleeding. Both of these symptoms are considered an "acute abdomen" and are emergencies. 1. Incorrect: Midabdominal pain radiating to the shoulder is a common s/s for a client with cholecystitis but is not a medical emergency. 2. Incorrect: Nausea and vomiting periodically for several hours is often seen with diverticulitis but is not a medical emergency. 4. Incorrect: Elimination pattern of constipation alternating with diarrhea indicates a partial bowel obstruction and may require further investigation, but this is not a medical emergency.

Which side effect of vincristine should the nurse immediately report to the primary healthcare provider? 1. Nausea 2. Fatigue 3. Paresthesia 4. Anorexia

3. Correct: Paresthesia is a side effect of some chemotherapeutic medications and if it occurs, the primary healthcare provider needs to modify the dosage or discontinue. 1. Incorrect: Nausea and vomiting are common side effects of many chemotherapeutic medications. 2. Incorrect: Fatigue is a common side effect of many chemotherapeutic medications. 4. Incorrect: Anorexia is a common side effect of many chemotherapeutic medications. 1. Look at each option as True or False. 2. Options 1,2, and 3 are false. Nausea, vomiting, fatigue, and anorexia are common side effects of chemotherapeutic medications. The nurse can assist with these side effects by planning for periods of rest, giving antiemetics as ordered, and encouraging high protein and high calorie small meals. The most serious dose limiting adverse effect is related to nervous system toxicity. Symptoms include numbness and tingling in the limbs, muscular weakness, loss of neural reflexes, and pain.

The primary healthcare provider prescribed phenytoin for a client with grand mal seizures. What intervention would the nurse plan for the client's care? 1. Offer the client frequent high calorie snacks. 2. Check the apical pulse before each dose. 3. Perform or assist with oral hygiene every shift. 4. Give the medication 30 minutes prior to meal.

3. Correct: Phenytoin is an anticonvulsant. It works by slowing down impulses in the brain that cause seizures. A major side effect is gingival hyperplasia. Oral hygiene is important for decreasing this complication while the client is on phenytoin. 1. Incorrect: Weight gain or loss are not typically a concern with phenytoin. 2. Incorrect: The apical pulse is checked with digoxin, not phenytoin. 4. Incorrect: The medication often causes gastric distress and may need to be taken with a meal, not before.

The nurse is caring for a preoperative client who received intravenous lorazepam 5 minutes ago and is now requesting to void. What is the appropriate nursing action? 1. Ask the unlicensed assistive personnel to assist the client to the bathroom. 2. Insert a indwelling urinary catheter since the client is going to surgery. 3. Place the client on a bedpan. 4. Allow the client to go to the bathroom.

3. Correct: Placing the client on a bedpan is the safest and least invasive choice. Lorazepam can cause drowsiness and the client should not be allowed to ambulate. 1. Incorrect: The client does not need to get up after receiving lorazepam because it can cause drowsiness. The client might fall. Think safety. 2. Incorrect: Not all surgical clients require a indwelling urinary catheter. This is not the least invasive choice. 4. Incorrect: Lorazepam can cause drowsiness and the client should not be allowed to ambulate. The risks of falls, especially alone, is too great.

The nurse is caring for a female client who is at risk for renal failure. The nurse has completed the initial assessment of the most recent lab results so that any concerns can be reported to the primary healthcare provider. Which assessment finding warrants further action? 1. Hemoglobin of 12 g/dl (120 g/L) 2. Hematocrit of 38% (0.38) 3. Potassium levels of 5.2mEq/L (5.2 mmol/L) 4. BUN of 15 mg/dl. (5.35 mmol/L)

3. Correct: Potassium is excreted primarily through the kidneys. When the kidneys are not working, potassium is being held. The normal value is 3.5 - 5.0 mEq/L (3.5 - 5.0 mmol/L). This potassium level is high and warrants further action. 1. Incorrect: This hemoglobin level is within normal limits for the female client. Normal hemoglobin values are 11.7-15.5g/dL (117-155g/L). 2. Incorrect: This hematocrit value is within the normal range for a female client. Normal hematocrit values are 35-45% (0.35-0.47). 4. Incorrect: This BUN level is within normal limits. Normal BUN levels are 6-20mg/dL (2.1-7.1mmol/L). Just because this client is in renal failure would not indicate a need to select this option.

A client comes into the women's clinic with amenorrhea, breast tenderness, and urinary frequency. Which term should the nurse use to describe these signs/symptoms of pregnancy? 1. Probable 2. Positive 3. Presumptive 4. Early

3. Correct: Presumptive signs and symptoms suggestive of pregnancy that may also indicate another condition. They occur early and are more subjective than other signs. The presumptive signs are amenorrhea, nausea and vomiting, frequent urination,and fatigue. 1. Incorrect: Probable signs are objective findings that can be documented by the examiner. The probable signs are more certain than presumptive signs but are not definitive. They include elevation of basal body temperature, breast tenderness and swelling, chloasma, linea nigra, chadwick sign, abdominal enlargement, softening of the cervix, ballotability of the uterus, quickening, and a positive pregnancy test result. 2. Incorrect: Positive signs are signs only attributable to the presence of a fetus. Three unmistakable signs of pregnancy are fetal heart tones heard on auscultation, fetal skeleton seen on x-ray or ultrasonogram, and fetal parts felt on palpation. 4. Incorrect: Early is not one of the three classifications indicating pregnancy.

During a health fair, a client asks the nurse about the methods used to detect prostate cancer. What should the nurse tell the client about the detection process? 1. Abdominal x-rays to detect the presence of lesions and masses. 2. A serum calcium test to detect elevated levels, which may indicate bone metastasis. 3. Digital rectal exam (DRE) and prostate-specific antigen (PSA) test to evaluate the prostate. 4. A magnetic resonance image (MRI) study to detect tumors and other abnormal growths.

3. Correct: Prostate cancer is the second most common type of cancer and the second leading cause of cancer death in men. Early detection improves outcome. DRE and PSA should be offered annually beginning at age 50 to men who have a life expectancy of at least 10 years and at age 45 in high-risk groups. The DRE estimates the size, symmetry, and consistency of the prostate gland while the PSA measures for elevated levels consistent with prostatic pathology, although not necessarily cancer. Declining PSA levels are useful in determining efficacy of treatment for prostate cancer. 1. Incorrect: Radiologic studies are not screening tools for this disease. 2. Incorrect: Hypercalcemia may indicate cancerous bone involvement, but it's not a screening tool. 4. Incorrect: MRI is a diagnostic tool, not a screening tool.

The home health nurse is caring for an elderly client who lives with an adult child. The client's child is divorced, works full-time, and is responsible for caring for two young children. Recently, the client has become incontinent of urine. Which stressor on the caregiver may increase the risk for abuse of this elderly client? 1. Care of young children 2. Being divorced 3. Recent increased care demands 4. Loneliness of the adult child

3. Correct: Recently increased care demands place a greater strain on the time and money required to provide care. The changing level of demands may increase the risk of abuse. 1. Incorrect: The adult child has been successfully managing the children and the elderly client up to this point. The physiological changes of incontinence for the client and increased care required for this is the most significant risk factor that could cause abuse. 2. Incorrect: The divorce is not a recently added stressor so is not a current change or stressor that would trigger the risk for abuse. 4. Incorrect: There is no mention of loneliness as a possible stressor in this scenario. This would be reading into the question and assuming incorrect data.

When preparing to administer the client a dose of intravenous (IV) antibiotics, the nurse notes that the IV pump cord is frayed with wiring visible. What priority action should the nurse take? 1. Notify maintenance to come and check the pump immediately. 2. Continue to use the IV pump and fill out an equipment maintenance request. 3. Obtain a replacement pump. 4. Tag the equipment for maintenance.

3. Correct: Removing potentially hazardous equipment is priority. Continued use of a faulty IV pump could lead to client endangerment such as electrical shock or fire. 1. Incorrect: Maintenance should be notified, but after equipment is removed from client care and properly tagged. Do not leave potentially hazardous equipment in patient's reach. 2. Incorrect: The equipment maintenance request should be filled out but after the equipment is removed from client care. The nurse needs to get a properly working IV pump to administer the antibiotics. 4. Incorrect: This should occur after it has been removed from the client's room. Patient safety is always the priority.

The nurse is caring for a client who is unresponsive during a postictal state. Which position is correct for this client? 1. Orthopneic 2. Dorsal recumbent 3. Sims' 4. Reverse trendelenburg

3. Correct: Sim's is a semi prone position where the client is halfway between lateral and prone positions. Often used for enemas or other examinations of the perianal area. Sim's is used for unconscious client's because it facilitates drainage from the mouth and prevents aspiration. 1. Incorrect: Orthopneic position places the client in a sitting position with arms resting on an overbed table. It allows maximum expansion of the chest. This would not be a safe position for an unresponsive client. 2. Incorrect: Dorsal recumbent is a back lying position where the shoulders are slightly elevated on pillows. it is used after surgeries and anesthetics. 4. Incorrect: Reverse trendelenburg is where the body the body is completely straight but the head is elevated and the feet are down. This position helps with gastroesophageal reflux disease, snoring, and with some surgeries.

A young adult client frequently engages in high risk behaviors, including driving at high speeds, using alcohol in excess, and engaging in high risk sexual behaviors. Which problem is priority for the nurse to assess? 1. Antisocial personality traits causing the disregard for life. 2. Impaired judgment caused by arrested psychological maturation. 3. Unconscious suicidal thoughts. 4. Unhealthy grieving.

3. Correct: Since all the behaviors could lead to death, these are considered indicators of self-destructive behaviors. The resulting conflict of weakness and strengths can produce negative emotions. These emotions can be manifested in risky behaviors. 1. Incorrect: Disregard for life and the needs of others is most often recognized in personality disorders. There is no indication of this in the stem of the question. 2. Incorrect: The underlying cause of arrested psychological maturation is past trauma or disturbances in childhood or adolescence and seen to be more specifically related to disruption or problematic parent-child relationships in the early developmental stages. It may occur from causes such as rejection, incest, molestation, emotional abuse or physical abuse. Arrested psychological maturation is when a person may be changing physically according to their age, but has not progressed emotionally past their teenage years. There is no indication of arrested psychological maturation in the question even though they are making unwise choices. 4. Incorrect: There is no indication in this question of a loss that would have precipitated grieving.

A client is seen in the clinic for recurrent, unexplained, vague stomach pain over the past 5 years. Esophagogastroduodenoscopy (EGD), colonoscopy, gallbladder ultrasound, and lab results have revealed no physical reason for the pain. The client tells the nurse, "the pain is so bad sometimes that I can't function!" What disorder is this client likely experiencing? 1. Conversion disorder 2. Pseudocyesis 3. Somatization disorder 4. Dysmorphic disorder

3. Correct: Somatization disorder is a syndrome of multiple somatic symptoms that cannot be explained medically and are associated with psychosocial distress and long-term seeking of assistance from healthcare professionals. Symptoms are vague, dramatized, or exaggerated in presentation. The disorder impairs social, occupational and other forms of functioning. 1. Incorrect: Conversion disorder is a loss of or change in body function resulting from a psychological conflict, the physical symptoms of which cannot be explained by any known medical disorder. This disorder affects voluntary motor or sensory functioning suggestive of a neurological disease. 2. Incorrect: Pseudocyesis is false pregnancy that may represent a strong desire to be pregnant. The client has nearly all the usual signs and symptoms of pregnancy such as enlarged abdomen, weight gain, cessation of menses and morning sickness.. 4. Incorrect: Dysmorphic disorder is characterized by the exaggerated belief that the body is deformed or defective in some way. Most common complaints are slight flaws of face or head, such as thinning hair, acne, wrin

A female client considers using spermicidal agents because she wants both birth control and protection from sexually transmitted infections (STIs). What information should the nurse provide the client about spermicidal agents? 1. Effectively reduces vaginal fungal infections such as Candida albicans. 2. Eliminates bacterial and viral sexually transmitted infections. 3. Most effective when used in conjunction with barrier methods, such as a diaphragm. 4. Causes few side effects.

3. Correct: Spermicidal agents have an approximately 25% failure rate in preventing pregnancy. These agents kill sperm by destroying the protective surface of sperm and preventing metabolic activities necessary for survival. 1. Incorrect: They do not kill fungi such as Candida albicans, even in high concentrations. 4. Incorrect: Spermicidal agents are used only when sexual intercourse is expected, but side effects include vaginal and penile irritation, lesions, and ulcerations due to the detergent effect. Disruption of normal protective vaginal flora results in an increased risk of opportunistic vaginal infections and urinary tract infections. 2. Incorrect: Spermicidal agents do not eliminate bacterial and viral STIs.

Which assignment would be most appropriate for the nurse to delegate to an unlicensed assistive personnel (UAP)? 1. Obtaining a sterile urine specimen from a Foley catheter. 2. Inserting an in-and-out catheter on a client postpartum. 3. Taking vital signs on a client 12 hours postpartum. 4. Removing a Foley catheter on a client postpartum.

3. Correct: Taking vital signs is within the scope of practice for the UAP, but the nurse is responsible for evaluating the vital signs. 1. Incorrect: Invasive procedures are not appropriate tasks for UAP (obtaining sterile specimen from Foley catheter). 2. Incorrect: Invasive procedures are not appropriate tasks for UAP (inserting catheter). 4. Incorrect: Invasive procedures are not appropriate tasks for UAP (removing foley catheter).

Which activity by the unlicensed assisted personnel (UAP) assisting a client with Parkinson's disease would require intervention by the nurse? 1. Assisting the client with ambulating to the bathroom and back to bed 2. Reminding the client not to look down while walking 3. Performing bathing and oral care for the client 4. Encouraging the client to feed self

3. Correct: The UAP should encourage the client to be as independent as possible. The nurse should intervene and teach UAP about client performing as much care as possible to encourage independence. 1. Incorrect: The UAP should assist the client when ambulating. This would not require intervention. 2. Incorrect: The UAP should remind the client to watch the horizon and not look down. This would not require intervention. 4. Incorrect: The UAP is encouraging independence. This is appropriate intervention and would not require intervention. 1. Look as each option as True or False. 2. Option 1 is the correct intervention for the UAP to do. With Parkinson's disease, a person's gait is unstable, which may result in falls. 3. Option 2 is the correct intervention for the UAP to do. Clients with Parkinson's disease need to concentrate on walking erect and watching the horizon as well as using a wide based gait. Clients have a tendency to have a shuffling gait and lean forward. 4. Option 3 is not what the UAP should be doing. One of the goals for a client with Parkinson's disease is to include maintaining independence in ADLs. 5. Option 4 is what the UAP should do as this encourages maintaining independence in ADLs.

A 12 year old female, with a history of juvenile rheumatoid arthritis, is being admitted for re-evaluation. The child reported these symptoms for the last week: temperature of 102.9ºF/39.4ºC at 4:00 pm every day, increased pain in joints, loss of appetite, and fatigue. What would be an appropriate room assignment by the charge nurse? 1. Private room only. 2. Rooming with a 12 year old male in skeletal traction due to a fractured femur. 3. Rooming with a 10 year old female that has been admitted for sickle cell disease. 4. Rooming with a 14 month old female that has been admitted for orthopedic surgery.

3. Correct: The appropriate answer is to room her with the 10 year old being worked up for sickle cell disease. This is an acceptable age/sex to pair as roommates. Each has a chronic illness and this allows them to see how another person with limitations adjusts. 1. Incorrect: It is not necessary for this child to be in a private room. The fever at a particular time of the day is a symptom of juvenile rheumatoid arthritis and does not mean an infection. 2. Incorrect: It would be inappropriate to room her with a 12 year old male due to opposite sex and age. 4. Incorrect: The 12 year old who is in p

A client admitted with biliary atresia has just arrived on the pediatric unit. The unit is very busy and the other RNs are busy with other clients at this moment. What action by the charge nurse would be most appropriate? 1. Instruct the unlicensed assistive personnel (UAP) to obtain clients vital signs and a weight. 2. Assign an LPN/VN to perform the initial nursing history and physical assessment. 3. Have an LPN/VN perform collect data on the client and report results to RN. 4. Inform one of the RNs that a new client is on the floor and that a nursing history should be completed as soon as possible.

3. Correct: The best answer is to have the LPN/VN collect intial data on the client and report it to the RN. The RN can evaluate data and initiate the priorities of care. Assessment on the new client should be completed by an RN within eight hours of arriving on the unit. It is acceptable to let the LPN/VN initiate the process. It would be best if a licensed person did a brief initial assessment on the child instead of the UAP. 1. Incorrect: Initial assessment is priority and must be done by RNs or a licensed person assigned by the charge nurse. The charge nurse is accountable for the initial assessment and must take the data and evaluate it and set the plan of care. 2. Incorrect: The RN can assign this takes but is accountable to verify the information and sign off on the data. The RN cannot delegate the assessment process but can obtain data collection assistance from the LPN/VN. 4. Incorrect: Initial assessment is priority and must be done first. The charge nurse cannot ignore a newly admitted client. Anew admission is always considered unstable and requires an initial observation and data collection by a licensed personnel. As soon as information is collected, the RN must evaluate it and set the priority to obtain the entire nursing history and initial plan of care.

The community health nurse plans to educate a client diagnosed with tuberculosis (TB) how to avoid spreading the disease to others. What should the nurse include when educating this client? 1. Wear a N95 respirator when around family at home. 2. Have adult family members get the TB vaccine. 3. Complete TB medication regimen. 4. Live at a sanatorium until cured of TB.

3. Correct: The best way to prevent the spread of TB is by completing the medication regimen. TB bacteria die very slowly, and so the drugs have to be taken for quite a few months. Even when a client starts to feel better they can still have bacteria alive in their body. So the person needs to keep taking the TB treatment until all the bacteria are dead. All the drugs must be taken for the entire period of TB treatment. If the entire treatment is not taken then the bacteria may not all be killed. They may then become resistant to the TB drugs which then don't work. 1. Incorrect: This is not required. It is expensive and must be fitted to each family member. Cloth or disposable masks can be used. Also, the client should cover mouth when coughing or sneezing, as TB is spread through the air. 2. Incorrect: The vaccine, BCG, works against a severe type of TB that affects young children but it is not effective against the type of bacteria found in adults. That severe type of TB is called military TB, is rare in the US, so the vaccine isn't used on young children here. 4. Incorrect: This used to be common treatment protocol, but no longer.

What should the nurse do after administering a chemotherapeutic drug intravenously (IV) to a client in the outpatient infusion unit? 1. Hang a 250 mL normal saline (NS) bag to flush the IV line. 2. Wear shoe covers during disposal of the drug. 3. Place the IV bag and tubing into a chemotherapy waste container. 4. Remove personal protective equipment (PPE) and dispose in a biohazardous container.

3. Correct: The disposable items such as the IV bag and tubing should remain intact and be disposed of in a securely sealed chemotherapy waste container. Tubing should never be disconnected from an IV bag containing a hazardous drug because of the risk of splashing. 1. Incorrect: The IV line does not have to be flushed with that much fluid. Usually, a central venous access device (CVAD) is flushed with 10 mL NS and a heparin solution if appropriate for the client and the device . For a peripheral line 2-5 mL NS is sufficient. 2. Incorrect: Shoe covers are not needed. Shoe covers are used when there is a chance of walking in contamination such as blood spilled on floor. 4. Incorrect: PPE (including double gloves, goggles, and protective gown) should be worn for all activities associated with chemotherapeutic drug administration. PPE used in chemotherapy drug administration should be disposed of in chemotherapy—not biohazardous—waste receptacle.

A nurse is caring for a client on the second day after a thoracotomy. The client reports incisional pain. The nurse assesses the client and evaluates the vital signs. Based on the data documented in the chart, what action should the nurse take first? *Exhibit (nursing notes and vital signs)* *Nursing notes* Client reports incisional pain as 8/10. Wound is clean and dry, without redness, edema, or drainage. Shallow respirations noted at 24/min. Adventitious lung sounds noted in bilateral bases. *vital signs* Oral temperature 100º F/37.8ºC Heart rate 92/min and regular BP 130/80 Respirations 24/min 1. Have client cough and deep breathe. 2. Administer acetaminophen for fever 3. Administer the prescribed analgesic 4. Assist the client to ambulate.

3. Correct: The client described in this question is post thoracotomy. With ANY post-op client, the number one concern, especially as a brand new nurse, is preventing pneumonia. A thoracotomy is very painful and the client is unlikely to breathe deep unless the pain is relieved. Temperature of 100º F/37.8ºC, HR 92, respirations 24, bilateral crackles (indicating atelectasis) all reveal this client is heading for pneumonia. 1. Incorrect: Coughing and deep breathing exercises are exactly what the client needs, but the client will not cough and deep breathe if it hurts. Give pain medication first. 2. Incorrect: Acetaminophen is not potent enough to relieve pain. The goal is to "fix the problem". The problem is that the client is not properly deep breathing due to pain. 4. Incorrect: Assisting the client to ambulate is a good idea, but the nurse has to fix the problem, and the problem is that the client is not deep breathing.

A client diagnosed with major depression has been admitted to a psychiatric facility for medication management. During nighttime rounds, an LPN/VN notes the client is not in bed. Which behavior by the client should the LPN/VN report to the RN immediately? 1. Sitting in a chair crying. 2. Reports inability to sleep. 3. Rearranging furniture. 4. Pacing around the room.

3. Correct: The client has been diagnosed with major depression, but is actively moving furniture around the room. While major depression is characterized by low energy, fatigue and lack of interest. Moving furniture indicates the client is displaying manic behavior which should be reported to the RN immediately. 1. Incorrect: Crying would be an expected behavior for clients diagnosed with major depression, along with decreased energy, irritability, and disinterest in most activities. Finding this client sitting in a chair and crying would not be an unusual behavior and would not need reported to the RN. 2. Incorrect: Major depression encompasses a variety of symptoms, such as a feeling of helplessness and hopelessness, despair and sleep disturbances that range from constant sleep to not sleeping at all. The inability to sleep would not be unusual for this client and would not need to be reported by the LPN/VN. 4. Incorrect: Pacing is one of a variety of behaviors displayed by clients diagnosed with major depression, which may also include irritability, sleep disturbances and lack of focus. While pacing the room should be monitored, there is another activity that is of more concern.

A nurse is caring for a client who was brought into the ED with a gunshot wound to the chest. There is an occlusive dressing in place and the client is receiving high flow oxygen. The nurse notes a deviated trachea, asymmetrical chest wall movement and decreased breath sounds bilaterally. What action should the nurse take first? 1. Elevate the head of the bed. 2. Initiate CPR. 3. Remove the occlusive dressing. 4. Notify the primary healthcare provider.

3. Correct: The client has developed a tension pneumothorax as evidenced by these signs/symptoms. This developed as a result of the placement of an occlusive dressing over the chest wound. By removing the occlusive dressing the pressure pushing to the opposite side of the chest should stop. Dressings over "sucking chest wounds" should be taped down on 3 sides only to allow air to escape but not re-enter. A needle decompression may be required as an emergency measure. 1. Incorrect: Although elevating the HOB typically helps ease the effort of respirations, in this case, it will not fix the tension pneumothorax. 2. Incorrect: It is not necessary to start CPR at this point. The client has not arrested. The client needs emergency relief measures such as removal of dressing and possible needle decompression to prevent further deterioration and possible cardiopulmonary arrest. 4. Incorrect: Call the primary healthcare provider after removing the occlusive dressing. This is an emergency situation. Attempts to resolve the issue are crucial to prevent further deterioration of the client's condition.

While suctioning a client's endotracheal (ET) tube, the nurse notes that the client's heart rate has gone from 78 to 44. The nurse stops suctioning the ET tube. What is the nurse's best action? 1. Deflate the ET tube cuff. 2. Have the client cough several times in a row. 3. Oxygenate the client with 100% oxygen. 4. Notify the primary healthcare provider.

3. Correct: The drop in pulse rate indicates acute hypoxia, which can be caused by suctioning. The nurse should stop suctioning and oxygenate with 100% oxygen. 1. Incorrect: There is no indication to deflate the ET cuff. Routine ET cuff deflation is not recommended. 2. Incorrect: This is a vagal maneuver that can be done to increase parasympathetic tone and decrease the conduction of the electrical impulses to the heart, usually done for treatment of supraventricular tachycardias. 4. Incorrect: The primary healthcare provider does not have to be called unless the client does not respond to oxygen.

A client is scheduled for surgery today. As the nurse prepares the pre-op medication, the client says, "I have changed my mind. I don't want to go through with the surgery." What should the nurse do first? 1. Convince the client to proceed with the plans for surgery. 2. Notify the surgery department to cancel surgery. 3. Notify the primary healthcare provider of the client's decision. 4. Suggest that the client talk over the decision with family members.

3. Correct: The client has the right to make decisions about their care. The primary healthcare provider should be notified about the client's decision to not have the surgery. 1. Incorrect: The nurse should not try to convince the client into having the surgery. If the nurse tries to have the client do something they do not want to do, the client is being coerced to make a decision by the nurse. This breaches the client's rights. 2. Incorrect: The primary healthcare provider should be notified prior to the surgery department being notified. The primary healthcare provider will need to review the client's plan of care with the client. 4. Incorrect: The client has informed the nurse of their decision. The nurse should not suggest any further action related to the client's decision. The client has the right to make decisions autonomously.

The nurse is working on an in-patient psychiatric unit. The nursing care plan includes teaching a client about assertiveness. The client has a long history of being manipulated by the employer and spouse. What is the best rationale for including assertiveness training in this client's treatment plan? 1. All clients should have assertiveness skills. 2. The client has low self-esteem. 3. The client is being taught self-advocacy. 4. No client deserves to be manipulated by an employer.

3. Correct: The client is being taught assertiveness and thus self-advocacy is learning to speak up for yourself and one's needs. The nursing role includes advocacy. This client will be discharged soon and needs improved skills in assertiveness for the word place and home environment. 1. Incorrect: This maybe true; however, it does not serve the best rationale for this client. Specifically this client needs assertiveness to assist their return to employment and home environment. 2. Incorrect: This statement is maybe true; however, it does not serve as an accurate rationale assertiveness training. 4. Incorrect: No the person should not be manipulated by an employer; however, the rationale in this question is to teach the client self advocacy through assertiveness training.

The nurse is caring for a client post hysterectomy. Based on data obtained from the nurse's notes, what should be the nurse's initial response? *Exhibit (nursing notes)* *0900:* Client received to post-anesthesia unit after hysterectomy. Easily aroused. BP 128/72, respirations 18, heart rate 90, skin warm and dry. *1100:* Lethargic, Vital signs are BP 100/68, respirations 24, heart rate 102, skin cool and moist. 1. Retake the vital signs. 2. Administer the ordered dopamine to maintain a blood pressure of 110 systolic. 3. Increase the IV rate of the lactated ringer's solution. 4. Raise the head of the bed to 30 degrees.

3. Correct: The client is exhibiting signs of hypovolemic shock. Increasing the IV rate of an isotonic solution will help to increase the blood pressure. 1. Incorrect: This delays treatment. The client is exhibiting signs of hypovolemic shock. Taking the vital sounds again will not be as important as administering volume. Remember, "less volume, less pressure". 2. Incorrect: Dopamine will help with hypotension, low cardiac output and poor perfusion of vital organs. This drug will not be used until there is persistent hypotension after adequate volume. Increasing the IV fluid rate would be the initial response. 4. Incorrect: Elevating head of bed would not increase B/P. If someone gets hypotensive, head down (Trendelenburg), would maintain blood flow to the head. Trendelenburg would be the correct answer if the client's blood pressure continues to drop.

The nurse is planning an activity for the client who has a diagnosis of paranoid schizophrenia. Which activity would be most appropriate for the client? 1. A game of Scrabble with peers 2. A group game of basketball. 3. An individual art project. 4. A card game with the nurse.

3. Correct: The client is likely to be most comfortable with solitary activities. When the client is extremely distrustful of others, solitary activities are best. Activities that demand concentration keep the client's attention on reality and minimize hallucinatory and delusional preoccupation. 1. Incorrect: The client is paranoid; therefore, he would not be comfortable within a group. Noisy environments may be perceived as threatening. 2. Incorrect: The client is likely to be very suspicious of the other players, thereby increasing their own anxiety level. The noisy basketball game may be too threatening for the client. Physical games are not the best choice for the paranoid schizophrenic client. 4. Incorrect: As trust builds with the nurse, this may be an appropriate activity, but there is a better answer. The "most" appropriate is an individual art project. The second best answer would be a card game with the nurse.

The nurse is caring for a client who is severely depressed and has an extremely low energy level. The client answers questions by using one or two words, and makes no eye contact. Which intervention is most appropriate for this client? 1. Ask the client to go to the group session with you. 2. Remind the client to interact with the nurse today. 3. Sit with the client and make no demands. 4. Allow the client to decide when to talk with the nurse.

3. Correct: The client is severely depressed and does not wish to have one on one interaction. Sitting with the client without demands demonstrates that the client is worthy of your time. The silence may also encourage the client to talk with you. 1. Incorrect: The client's energy level is low, so the client would not respond positively to this request. Depressed clients may speak slowly and have slowed comprehension. Group therapy would not be appropriate at this time. 2. Incorrect: The client may not have adequate energy for spontaneous interaction today. Also, reminding the client to interact is not therapeutic. The client may view this as the nurse thinking they are worthless. 4. Incorrect: When clients are extremely depressed, they cannot make decisions independently. Extreme fatigue interferes with social activities and relationships.

When planning post procedure care for a client who is having a barium enema, what must the nurse include? 1. Cardiac monitoring for potential arrhythmias 2. Monitoring urinary output 3. Administration of a laxative or enema after the procedure 4. Reordering the client's diet

3. Correct: The client must expel the barium post procedure. If the barium is not eliminated, it can harden in the colon and cause an obstruction. 1. Incorrect: It is not standard practice to place clients on a cardiac monitor after a barium enema. 2. Incorrect: Monitoring urine output has nothing to do with this procedure and does not answer the specific question related to this diagnostic procedure. 4. Incorrect: Reordering the client's diet is important but is not as life-threatening as a bowel obstruction.

Which comment made by a client scheduled for a lumbar laminectomy and discectomy indicates to the nurse that the client needs further teaching? 1. After the incision is healed, I can go for daily walks. 2. By the time I am discharged, my back and leg pain will be better. 3. I can turn by myself after surgery, but I will need help to get out of bed. 4. The staff will frequently check my feet and legs for feeling and movement.

3. Correct: The client must log roll with assistance. The spine must be kept in proper alignment to allow the area time to rest and heal. The nurse should reinforce this information with the client. 1. Incorrect: After the incision heals, it is acceptable practice to go for daily walks so this is an accurate understanding of what the client can do after the surgery. 2. Incorrect: Successful laminectomies and discectomies will relieve back and leg pain so this is accurate, also. 4. Incorrect: The nurse knows it is very important to perform neurovascular checks after ANY orthopedic surgery. Any changes from their baseline should be reported to the primary healthcare provider immediately. This indicates successful teaching.

A client is transported to the emergency department by the police following a sexual assault. What is the nurse's priority intervention? 1. Instruct the client to remove all of her clothes so they can be bagged as evidence. 2. Ask the client to describe what happened . 3. Tell the client she is safe here. 4. Perform a rape kit in order to preserve the evidence .

3. Correct: The client who has been sexually assaulted often experiences great fears and must be reassured of her safety. She may also be overwhelmed with self-doubt and self blame. This statement will instill trust. Remember, safety first! The most helpful things a nurse can do is listen and let the victim talk. 1. Incorrect: This needs to be done, but you must first build trust and establish a feeling of safety with the client. A woman who feels understood is no longer alone. She then feels more in control of the situation to remove her clothes. 2. Incorrect: This may be done, but you must initially build trust and establish with the client that she is safe. Non-judgmental listening provides an avenue for catharsis that the client needs to begin healing. A detailed account may need to be given for legal follow-up. A caring nurse, as a client advocate, may help to lessen the trauma of evidence collection. 4. Incorrect: This needs to be done. But you must build trust and establish the client feels safe initially. The nurse must maintain a nonjudgmental attitude and provide nonjudgmental care before the rape kit can be obtained.

The charge nurse has received report from the emergency department about a client diagnosed with Cushing's disease being admitted to the unit. Which client in a semi-private room would be appropriate for the charge nurse to have this client share? 1. Client who has leukemia. 2. Client diagnosed with gastroenteritis. 3. Client who has a fractured hip. 4. Client diagnosed with bronchitis.

3. Correct: The client with Cushing's disease could go in the room with the client who has a fractured hip, as this client does not have an infection. 1. Incorrect: Both of these clients are immunocompromised and should not share a room with each other. 2. Incorrect: The client with gastroenteritis poses a risk of infection to the client with Cushing's disease because this client is immunosuppressed. 4. Incorrect: The client with bronchitis poses a risk of infection to the client with Cushing's disease.

The client, who recently started college, tells the nurse, "I am having trouble studying for my tests. Every time I try to study, my mind begins to wander." What is the nurse's best response? 1. "Stop making excuses and make a study schedule you will follow." 2. "I wouldn't worry. You are smart enough to pass college." 3. "You are having difficulty concentrating?" 4. "What do you mean you can't study?"

3. Correct: The correct answer demonstrates the therapeutic communication technique of "restating". The main idea is to let the client know whether or not an expressed statement has been understood and gives him or her the chance to continue or clarify if necessary. 1. Incorrect: This response is disapproving and gives advice. It is not therapeutic. The nurse does not know the client is making excuses and should not assume. 2. Incorrect: This response is giving reassurance. This statement seems like the nurse is somewhat "blowing off" the client. It does not address this issue. 4. Incorrect: This response is belittling. This statement does not address the client's problem. It also implies that the client should not have problems studying.

The nurse is preparing to discharge four clients from the unit. Which client is most likely to warrant a referral to other agencies or community outreach programs? 1. 45 year-old client who had nasal surgery. 2. 50 year-old client postop mastectomy. 3. 72 year-old client with diabetes and obesity. 4. 80 year-old client with a diagnosis of delirium caused by dehydration.

3. Correct: The elderly client with diabetes and obesity is likely to need referrals at the time of discharge, whether to physical therapy, home health, weight loss program or other agencies. 1. Incorrect: There is no need to think that this client who is 45 years old will need assistance after nasal surgery. 2. Incorrect: This 50 year-old client postop mastectomy is mobile and does not need referral to other agencies or community outreach programs. 4. Incorrect: Delirium is an acute illness that resolves once the physical stressor is eliminated. In this case the delirium was caused by dehydration that has been corrected if the client is ready for discharge. There is no indication that the client is in need of post-hospital care.

After artificial rupture of membranes (AROM), the baseline fetal heart rate tracking begins to show sharp decreases with a rapid recovery with and between contractions. Which of the following actions by the RN has priority? 1. Position the client on her left side 2. Increase the IV fluid rate 3. Place the client in the knee-chest position 4. Administer oxygen per tight face mask

3. Correct: The fetal heart pattern is that of repetitive deep variable decelerations. This pattern is likely due to a prolapsed umbilical cord after AROM. The priority intervention is to relieve the pressure on the cord from being trapped between the presenting part and the pelvis. This can be accomplished by manual pressure on the presenting part, placing the client in Trendelenburg position, or placing her in the knee-chest position. 1. Incorrect: This intervention will improve placental perfusion, but will not relieve compression of a prolapsed cord. If the cord is compressed, it doesn't matter how well perfused the placenta is because the oxygen cannot reach the baby. Late decels and low BP would be an indicator that we need to increase uterine perfusion by positioning on left side. 2. Incorrect: This intervention will not improve placental perfusion. IV fluids will not relieve compression of a prolapsed cord. 4. Incorrect: This intervention will improve placental perfusion, but will not relieve compression of a prolapsed cord. If the cord is compressed, it doesn't matter how well perfused the placenta is because the oxygen cannot reach the baby.

A hospice nurse is assessing a client reporting chronic pain (5/10 on the pain scale). In addition to the primary healthcare provider and the nurse, what member of the care team will assist in providing comfort therapies for this client? 1. Physical therapist 2. Nutritionist 3. Massage therapist 4. Occupational therapist

3. Correct: The massage therapist provides alternative therapies that complement the medical pain control therapies being provided by the primary healthcare provider and the nurse. 1. Incorrect: A physical therapist is trained to improve and restore mobility. Due to the client's terminal status short term reduction of the pain is needed. 2. Incorrect: The nutritionist's main focus is on the client's nutritional status. This will not affect the pain level of the client. 4. Incorrect: The occupational therapist's main focus is to improve the client's ability to perform activities of daily living and work skills. Due to the terminal status of this client, pain management is the goal for this client.

A nurse is providing discharge teaching to a client who has had a cystectomy and formation of an ileal conduit. What client statement indicates that teaching was successful? 1. I should restrict my fluid intake to decrease the need to empty the drainage bag. 2. I will change my appliance daily to prevent skin excoriation from the leakage of urine. 3. I will change my drainage bag whenever it is leaking, giving special attention to my skin around the bag. 4. I will restrict going to events outside the home because leakage is common and embarrassing.

3. Correct: The goal is to prevent skin irritation by changing the bag regularly and using proper equipment to prevent leakage. The client with an ileal conduit (urinary diversion) must be educated appropriately to ensure that self-care abilities are complete and safe. As long as the bag is not leaking and no skin breakdown is evident, changing the appliance bag only has to be done as needed, not daily. 1. Incorrect: Clients with ileal conduits are encouraged to increase fluid intake, as opposed to restricting it, to maintain adequate urine flow for prevention of urinary tract infections (UTI). 2. Incorrect: Changing the apparatus daily is too often and will cause trauma and skin breakdown. 4. Incorrect: Clients with ileal conduits are encouraged to maintain an active and normal lifestyle. People with ileal conduits have a generally low incidence of complications and high client satisfaction level due to the ease of care and minimal lifestyle changes.

The nurse is caring for a client following a transurethral resection of the prostate (TURP). The client has a 3 way irrigation catheter in place. Which observation would indicate the need to slow the irrigation? 1. Clots in urine 2. Bladder pressure 3. Clear urine 4. Bladder spasms

3. Correct: The irrigation is regulated so that the urine is free of clots and slightly pink tinged. When it becomes clear after surgery, the fluid is going too fast and not clearing any blood clots effectively. 1. Incorrect: The irrigation should be increased if you see clots in order to keep the catheter patent. 2. Incorrect: Bladder pressure may mean that the indwelling urinary catheter is obstructed. Either increase flow or manually irrigate catheter to ensure patency and no retention of fluid in the bladder. 4. Incorrect: Bladder spasms occur with clots so you do not want to slow the irrigation if this happens. This would indicate the need for increasing the irrigation fluid rate.

A client has just found out that she is pregnant and asks the nurse, "When is my baby due?" The client's last menstrual period began March 3. What date will the nurse calculate as the expected date of confinement? 1. December 3 2. December 7 3. December 10 4. December 13

3. Correct: The most common method of determining the expected date of confinement is by Nagele's rule. To use this method begin with the first day of the last menstrual period, add seven days, subtract 3 months and add one year. So the expected date of confinement for this client would be December 10. 1. Incorrect: To use Nagele's rule begin with the first day of the last menstrual period, add seven days, subtract 3 months and add one year. So the expected date of confinement for this client would be December 10. 2. Incorrect: To use Nagele's rule begin with the first day of the last menstrual period, add seven days, subtract 3 months and add one year. So the expected date of confinement for this client would be December 10. 4. Incorrect: To use Nagele's rule begin with the first day of the last menstrual period, add seven days, subtract 3 months and add one year. So the expected date of confinement for this client would be December 10.

A client who is ventilator dependent is scheduled to be discharged home. What is the most critical assessment for the nurse case manager to make? 1. Financial stability for home health care. 2. Long-term home care needs. 3. Safe home environment. 4. Home medical equipment needed.

3. Correct: The most critical assessment is to make sure that the client is going home to a safe environment. Then the other assessments could be made. Without a safe environment the client does not need to go home. Information about electrical wiring, back-up power, hygiene and infection control needs all provide a safe environment for this client. 1. Incorrect: This is not the most critical assessment and can be done after making certain the client will be safe. Remember Maslow's Hierarchy of Needs. After you determine needed resources (#4) then financial stability would be next. 2. Incorrect: Long term goals are very important but we are worried about short term needs right now. Remember in a priority question all options are plausible but only one is critical now. 4. Incorrect: Once the environment is considered safe for the needed or required care of the client, then the needed equipment would be next.

Following a lumbar puncture, the client reports a headache on a pain scale of 8 out of 10. What priority action should the nurse perform? 1. Instruct the client to drink at least 8 ounces of water. 2. Close room blinds to darken the environment. 3. Assist the client into a supine position in bed. 4. Notify primary healthcare provider of client's complaints.

3. Correct: The most frequent cause of headache following a lumbar puncture is loss of, or leaking, of cerebrospinal fluid from the puncture site. Positioning a client is an important nursing responsibility, particularly in this situation since the supine position could help to stop any leaking. Following this, the nurse will pursue additional actions as ordered by the primary healthcare provider, which may include increasing fluids or even a blood patch. 1. Incorrect: Although increasing fluids may help clients under specific circumstances, it is not the priority action in this situation. Additionally, the primary healthcare provider may order IV fluids rather than PO fluids. 2. Incorrect: A darkened room can be beneficial for clients with severe migraine headaches, but would not be useful to this client. Headaches following a lumbar puncture are caused by the loss of cerebrospinal fluid and would not respond to a quiet, dark environment. 4. Incorrect: Although the primary healthcare provider should indeed be notified of this situation, the nurse's priority action should first focus on stabilizing the client by addressing the cause of this problem and positioning the client.

A frightened client comes to the nurses' station during the night and reports hearing the voice of the devil speaking to them. Which response by the nurse is priority? 1. "Could you have overheard the staff talking at the desk?" 2. "I will get you some medication for anxiety." 3. "What did the voice tell you? " 4. "You do not have to worry about this. You are safe."

3. Correct: The most important thing the nurse needs to find out is what the voice was telling the client. This is a safety issue. The nurse needs to know if the voice was telling the client to harm themselves or others. 1. Incorrect: In this question, this is not the priority response. This is voicing doubt and also presenting reality. This response could come later in the interaction. 2. Incorrect: This is changing the subject, which is non-therapeutic. The nurse needs to do something prior to giving medicine. 4. Incorrect: This is giving reassurance, which can be a non-therapeutic response. However, it could be used later in the interaction if the nurse finds out the client is safe. This statement does not address the voice heard by the client.

The nurse evaluates an electrocardiogram (EKG) and notices a U-wave. The nurse suspects that this occurrence is caused by which electrolyte imbalance? 1. Hypermagnesemia 2. Hypocalcemia 3. Hypokalemia 4. Hyponatremia

3. Correct: The most serious complications of hypokalemia are cardiac changes. Hypokalemia impairs myocardial conduction and prolongs ventricular repolarization. This can be seen by a prominent U-wave (a positive deflection following the T-wave on the EKG). The U-wave is not totally unique to hypokalemia, but its presence is a signal for the clinician to check the serum potassium level. 1.Incorrect: Remember hypermagnesemia results in the client having a sedated appearance, decreased deep tendon reflexes, decreased level of consciousness, decreased respiratory rate, and ultimately cardiac arrest. 2. Incorrect: In hypocalcemia, this client is not sedated and will have an increased nerve excitability, tetany, appearance of Tousseau's, and Chvostek's sign. Cardiac manifestations include Vtach. 4. Incorrect: Hyponatremia results in neurological symptoms: confusion, irritability, and ultimately coma.

What action should the nurse take when testing a client's near vision? 1. Have client read a Snellen chart from 20 feet away. 2. Have client read Ishihara plates at 30 inches (75 cm). 3. Have client read a newspaper at 14 inches (36 cm). 4. Have client alternate gaze from a near object to a distant object.

3. Correct: The nurse can get a general idea of near visual acuity by asking the client to read from a newspaper. The newspaper should be held 14 inches from the eyes. This exam can also be done with the Jaeger chart containing a few short lines or paragraphs of printed text. The size of the print gradually gets smaller. The client is asked to hold the card about 14 in. (36 cm) from the face and read aloud the paragraph containing the smallest print he/she can comfortably read. Both eyes are tested together, with and without corrective lenses. This test is routinely done after age 40, because near vision tends to decline as one ages (presbyopia). 1. Incorrect: The Snellen chart is used to test distant vision. To test distance vision, individuals stand 20 feet from the Snellen eye chart, cover one eye, read aloud the smallest line they can clearly see, and then repeat this process with the other eye. After performing an eye test, a person's visual acuity is written as a fraction. Normal vision is defined as 20/20 visual acuity, which means at 20 feet away from the eye chart, the person is able to read the line that most human beings with normal vision can read at 20 feet away.​ 2. Incorrect: Ishihara plates are used to assess color blindness, and are not used to test near vision. ishihara-transformation 4. Incorrect: Having the client alternate their gaze from a near object to a distant object tests for accommodation. It does not test for near vision.

A client admitted to the Coronary Care Unit (CCU) following a myocardial infarction (MI) expresses fear of the equipment and noise in the busy unit. What is the most therapeutic response by the nurse? 1. "Everyone gets scared here at first." 2. "Why are you afraid of equipment?" 3. "This all seems frightening to you." 4. "You won't have to be here very long."

3. Correct: The nurse is making a statement that reflects back the feeling of fear expressed by the client. This therapeutic communication tool acknowledges that the nurse has heard the client while providing an open-ended approach which will allow the client to continue to communicate emotions. Encouraging the client to continue to express feelings is important. 1. Incorrect: This is a belittling response in which the nurse is focusing on everyone, rather that this specific client. It is a closed, non-therapeutic reply that discourages further interaction with the client and does not allow for further expression of feelings. 2. Incorrect: When a client expresses emotions, asking why demands an explanation that is neither necessary nor therapeutic. In most cases, clients may not be able to provide any explanation and the need to do so further restricts the potential for open communication. 4. Incorrect: Instead of focusing on the client's feelings, the nurse has changed the topic and blocked the potential for the client to communicate further. Therapeutic communication should provide open-ended opportunities in which the clients can freely express concerns.

A charge nurse is caring for clients when a new admit arrives on the unit. What action by the charge nurse is most appropriate? 1. Instruct the unlicensed assistive personnel (UAP) to complete emptying the catheter bag, and assess the new admission. 2. Send the UAP to take VS on the new admit and begin the history until she can get there. 3. Assign a nurse on the floor to initiate the assessment process. 4. Ask the unit secretary to make the client and family comfortable until she can complete her present task.

3. Correct: The nurse is the only one who can assess. 1. Incorrect: The UAP can empty the urinary catheter bag, but can not assess the client. 2. Incorrect: It is out of the scope of practice for a UAP to complete any portion of the admission assessment. 4. Incorrect: The unit secretary can welcome the client, but the admission assessment must be completed by an RN.

What is the nurse's priority when preparing a client for a paracentesis? 1. Place client in the prone position. 2. Position the client supine with right arm behind the head. 3. Ask the client to empty bladder. 4. Obtain client's vital signs immediately prior to the procedure.

3. Correct: The nurse knows this is a lower abdominal puncture, and the bladder should be empty to avoid puncturing the bladder. 1. Incorrect: The prone position would completely obstruct the area where the procedure is done. The optimal position for paracentesis is HOB up to allow fluid to pool in one spot for paracentesis. 2. Incorrect: The optimal position for paracentesis is HOB up to allow the fluid to pool in one spot for paracentesis. Supine would put the client at risk for the bladder getting punctured during the procedure. 4. Incorrect: Obtaining a set of vital signs immediately prior to the procedure is important as well, but remember, pick the answer that is most life threatening when answering priority questions. Puncturing the bladder is more life threatening than obtaining vital signs.

The nurse manager on a medical-surgical unit receives official notification that staff overtime must be decreased as a cost-saving measure. In order to reorganize staffing, the nurse manager should initiate which action first? 1. Announce the new changes at the monthly staff meeting. 2. Ask for any staff objections to rearranging work hours. 3. Invite staff to contribute ideas on scheduling changes. 4. Explain administration is demanding a decreased overtime.

3. Correct: The nurse manager is aware that open communication with staff is vital to increase workplace satisfaction and staff retention. One important aspect is encouraging the flow of ideas between management and staff members. Open communication and brainstorming sessions in which staff can freely share thoughts or ideas creates a positive work environment while helping decrease dissatisfaction. 1. Incorrect: While it is true that the nurse manager is ultimately responsible for implementing and announcing new schedule changes, doing so without any staff input can create discontent in the work environment. When staff do not feel vested in any new process, there is a sense of underappreciation. This perceived lack of control can create distrust and frustration among personnel, ultimately impacting client care. 2. Incorrect: Although this action appears to be opening lines of communication, the nurse manager is actually fostering animosity in a situation where the outcome is already pre-determined. Allowing staff to vent is acceptable but the nurse manager should focus on constructive methods of adjustment to the impending mandated changes. 4. Incorrect: The nurse manager is aware that health care facilities often face both political and financial issues that impact staff and clients simultaneously. The responsibility of the nurse manager is to implement change in a positive manner, while assisting staff adaptation even to unpopular modifications. Assigning blame for the changes to administration will not help staff adjust.

A client from a long-term care facility arrives in the emergency department by ambulance with altered level of consciousness. The primary healthcare provider instructs the respiratory therapist to prepare for intubation. The nurse discovers a Do Not Resuscitate (DNR) bracelet on the client's wrist during the initial assessment. Which immediate action should the nurse take to advocate appropriately for this client? 1. Assist the respiratory therapist to prepare the client for immediate intubation. 2. Attempt to contact the client's family. 3. Notify the primary healthcare provider immediately of the client's DNR bracelet. 4. Notify the charge nurse immediately of the client's DNR bracelet.

3. Correct: The nurse should immediately notify the primary healthcare provider upon discovering the client's DNR bracelet. The DNR bracelet is an indicator that the client or their healthcare surrogate decision maker wants the client's wishes be known regarding healthcare treatment and resuscitation. 1. Incorrect: Ignoring the DNR bracelet and assisting the respiratory therapist to prepare for immediate intubation is incorrect because the client has a DNR notification on their person and should not be intubated. 2. Incorrect: Reaching the client's family allows the family to be with the client and to provide additional health history, but this should be done after notifying the primary healthcare provider. 4. Incorrect: Notifying the charge nurse of the client's DNR bracelet is not priority. The primary healthcare provider must be notified first.

The circulating nurse prepares the sterile field in the operating room (OR). Fifteen minutes later, the nurse is informed the surgery will be delayed for 20 minutes because the surgeon is working at another hospital. Which is the best action for the nurse to take? 1. Cover the sterile field with a sterile drape until the surgery is about to begin. 2. Close and tape the OR doors so that no one may enter. 3. Monitor the sterile field while awaiting the surgeon. 4. Tear down the sterile field until the surgeon arrives in the OR.

3. Correct: The nurse should monitor the sterile field while awaiting the surgeon. This means keeping the sterile field in your site. 1. Incorrect: Sterile fields should not be covered. Although there are no research studies to support or discount the practice, removing a table cover may result in a part of the cover that was below the table level being drawn above the table level or air currents drawing microorganisms from a nonsterile area to the sterile field. It is important to continuously monitor all sterile areas for possible contamination.​ 2. Incorrect: There is no specified amount of time designated that a room can remain open and not used and still be considered sterile. The sterility of an open sterile field is event-related. An open sterile field requires continuous visual observation. Direct observation increases the likelihood of detecting a breach in sterility.​ 4. Incorrect: It is unnecessary to tear down the sterile field as the delay is minimal. This is also an added cost to discard materials and redo the sterile field when it has not been contaminated. Sterile fields should be prepared as close as possible to the time of use. The potential for contamination increases with time because dust and other particles present in the ambient environment settle on horizontal surfaces over time. Particulate matter can be stirred up by movement of personnel when opening the room and also can settle on opened sterile supplies.

Which arterial blood gas (ABG) results would the nurse expect on a client who has overdosed on aspirin (ASA)? 1. pH 7.54, PaCO2 41, PaO2 63, SaO2 91, HCO3 36 2. pH 7.24, PaCO2 37, PaO2 83, SaO2 95, HCO3 18 3. pH 7.49, PaCO2 30, PaO2 68, SaO2 92, HCO3 28 4. pH 7.12, PaCO2 28, PaO2 72, SaO2 93, HCO3 10

3. Correct: This ABG result indicates respiratory alkalosis. Aspirin stimulates the respiratory center and causes an increase in respiratory rate and depth. This causes respiratory alkalosis by blowing off CO2 and causing the pH to increase. Losing CO2 (acid) makes the client more alkalotic, which is reflected with an increased pH, decreased PaCO​2 and increased HCO​3. 1. Incorrect: This ABG result indicates metabolic alkalosis. The pH is high, PaCO​2 is normal and HCO​3 is high. Normal pH is 7.35-7.45, normal PaCO​2 is 35-45, normal HCO​3 is 22-26. 2. Incorrect: The client with an aspirin overdose will have a respiratory acid/base imbalance. 4. Incorrect: This ABG also indicates metabolic acidosis. The problem in the stem would result in a respiratory problem.

The nurse is caring for a client who presents to the mental health unit following a violent altercation with the spouse. The client has numerous bruises on the face, chest, and back. There is one laceration where spouse "came at me" with a knife. At this time, what is most likely to be the mood of the perpetrator in this situation? 1. Extreme anger 2. Anxiety 3. Kindness 4. Irritability

3. Correct: The perpetrator has completed the acute battering phase and has now likely entered the honeymoon phase with extreme kindness and acts of love. The attacker is now calm after the tension has been released. You may witness remorseful and apologetic behaviors like bringing gifts and promises of love. 1. Incorrect: The anger phase is likely over after the attacker has beaten the victim. This anger building stage is called tension building stage and is characterized by minor incidents like pushing, shoving and verbal abuse. During this time the abused spouse may accept the abuse for fear of it getting worse so the abuser rationalizes that the behavior is acceptable. The abuser may even turn to alcohol and drugs to curb the anger. The extreme anger exhibited during the acute battering stage. The abuser releases the built-up anger and tension by brutal and uncontrollable beatings. After the beating the client is calm and described as "in shock" or have have amnesia of the event. You also see extreme anger in the escalation/de-escalation stage. 2. Incorrect: The tension or anxiety would be felt during the tension-building phase. This anger building stage is called tension building stage and is characterized by minor incidents like pushing, shoving and verbal abuse. During this time the abused spouse may accept the abuse for fear of it getting worse so the abuser rationalizes that the behavior is acceptable. The abuser may even turn to alcohol and drugs to curb the anger. The extreme anger exhibited during the acute battering stage. The abuser releases the built-up anger and tension by brutal and uncontrollable beatings. After the beating the client is calm and described as "in shock" or have have amnesia of the event. You also see extreme anger in the escalation/de-escalation stage. 4. Incorrect: Irritability would be demonstrated during the tension-building phase.

A nurse is caring for a client admitted to the hospital for a total hip replacement. In preparing the post-operative plan of care for this client, the nurse recognizes which goal as the highest priority? 1. Prevent complications of shock. 2. Prevent dislocation of prosthesis. 3. Prevent respiratory complications. 4. Prevent skin breakdown.

3. Correct: The postoperative client with a total hip replacement is at risk for thromboembolism and fat emboli which can travel to the lungs and cause respiratory distress. Without proper turning, coughing, and deep breathing, pneumonia and atelectasis may occur. So preventing respiratory complications is high on the priority list. Remember the ABCs - airway, breathing, then circulation. Preventing respiratory complications is the highest priority because of the possibility of sudden death from the complications of deep vein thrombosis and pulmonary embolism. 1. Incorrect: This client is at risk for hemorrhage and/or hematoma formation related to surgical trauma to blood vessels (the hip is a very vascular area) and use of anticoagulants or antiplatelet agents before and after surgery. So the nurse will need to monitor for shock caused by loss of volume. The nurse should monitor drains, wound dressings, and intake and output. But remember, Airway and Breathing take priority. 2. Incorrect: Dislocation of the prosthesis is another complication to worry about. It will cause pain and possible deformity and is very important, but airway is the priority. Dislocation of the hip prosthesis is related to weakness of the hip muscles, improper positioning or movement of the operative extremity, and/or noncompliance with weight-bearing limitations. 4. Incorrect: The client is at risk for skin breakdown if not turned and repositioned properly or ambulated as soon as prescribed. However, Airway is still the priority for this client.

A client with psychosis, tells another client, "You are so adorabogalishus." Which form of thought process should the nurse document this client as having? 1. Magical thinking 2. Tangentiality 3. Neologism 4. Perseveration

3. Correct: The psychotic person invents new words, or neologisms, that are meaningless to others but have symbolic meaning to the psychotic person. Remember, do not use the invented word. "Adorabogalishus" is not a real word. 1. Incorrect: With magical thinking, the person believes that their thoughts or behaviors have control over specific situations or people. The client believes that thinking something can make it happen. 2. Incorrect: With tangentiality, the person never really gets to the point of the communication. Unrelated topics are introduced, and the original discussion is lost. The client goes off the topic which can destroy interpersonal communications. 4. Incorrect: The person who exhibits perseveration persistently repeats the same thought, phrase or motor response to different questions. This is associated with brain damage.

The ICU nurse is caring for a client with massive head injuries. The nurse notices that the client's respirations have a rhythmic increase and decrease of rate and depth and include brief periods of apnea. How would the nurse document this respiratory pattern? 1. Apneusis 2. Ataxic 3. Cheyne-Stokes 4. Cluster

3. Correct: The respiratory pattern described is Cheyne-Stokes. A client with massive head injuries is at risk for this breathing pattern due to an injury with the cerebal hemispheres. 1. Incorrect: Apneusis is characterized by a sustained inspiratory effort. It does not typically have a period of apnea. 2. Incorrect: Ataxic respirations have an irregular, random pattern of deep and shallow respirations with irregular apneic periods. The irregularity of it differentiates ataxic respirations from Cheyne-Stokes respirations. 4. Incorrect: Cluster breathing is characterized by a closely grouped series of gasps followed by a period of apnea. There is no rhythmic increase and decrease as in Cheyne-Stokes respirations.

After shift report, which client should the nurse see first? 1. Eight year old that is in skeletal traction. 2. Six year old who is 5 hours postop appendectomy. 3. Unattended two year old admitted for a sleep study. 4. Four year old cerebral palsy child with a tracheostomy admitted for urinary tract infection (UTI).

3. Correct: The unattended child should be checked first to make sure he/she is safe and having no complications. A child this age is entirely dependent on someone else. Safety is priority here. 1. Incorrect: An eight year old in skeletal traction does not take priority over unattended 2 year old.. 2. Incorrect: A six year old who is 5 hr post appendectomy should be seen, but not as immediate as an unattended 2 year old. 4. Incorrect: This client has UTI and not acutely ill at this time. The major clues are age and unattended. This child's safety is the reason the child takes priority.

One hour after administering pyridostigmine, the nurse notes increased salivation, lacrimation, and urination in the client. What initial action should the nurse take? 1. Administer a second dose of pyridostigmine. 2. Place client in side lying position. 3. Notify the primary healthcare provider. 4. Prepare for intubation and mechanical ventilation.

3. Correct: These are signs and symptoms of cholinergic crisis. The client can get increasingly worse. The primary healthcare provider can prescribe atropine as treatment of overdose. 1. Incorrect: Giving an additional dose of pyridostigmine will make the client worse. 2. Incorrect: For better respiratory effort the client should be placed in a semi fowler's position. 4. Incorrect: This can be done after notifying the primary healthcare provider.

A client buzzes the nurses' station to report chest pain. The nurse looks at the client's cardiac rhythm strip, then hurries into the client's room to find the client unresponsive and without a pulse. What initial action should the nurse take? 1. Administer Epinephrine 1mg IV push. 2. Begin cardiopulmonary resuscitation (CPR) for 2 minutes. 3. Defibrillate at 120 joules. 4. Insert supraglottic airway device.

3. Correct: This client had a witnessed arrest with pulseless ventricular tachycardia. The first thing that the nurse should do is to defibrillate the client at 120-200 joules. Subsequent dosing is increased in a stepwise fashion as needed. 1. Incorrect: Epinephrine, a vasopressor is used to produce vasoconstriction and to raise the BP. It is used for the treatment of Ventricular fibrillation and pulseless ventricular tachycardia for its vasoconstrictive effects. It is not the first action however. If defibrillations do not work, then CPR is done for two minutes prior to administration of epinephrine. 2. Incorrect: Defibrillation should be done immediately unless it is not readily available. Then CPR is performed until the defibrillator arrives. 4. Incorrect: Consider an advanced airway after defibrillations fail to revert the client to Normal Sinus Rhythm. First you must be able to identify the strip. There are 5 steps to rhythm analysis. Normal Sinus Rhythm is the only rhythm when each of the five steps of rhythm analysis are "normal". All other rhythms you will analyze will have at least one of the 5-steps presenting an abnormality. Is the rhythm regular? Is the heart rate range between 60 - 100 bpm? Are the P waves upright and uniform in appearance (in Lead II), with a P wave for each QRS complex? Does the PR interval measure in the normal range of 0.12 - 0.20 second (and measure the same each time)? Is the QRS complex similar beat-to-beat and measure between 0.06 - 0.10 second (and measure the same each time)? So what do we see when we look at this strip? Rhythm: Regular Atrial rate: None Ventricular Rate: 200 bpm P wave: none PR interval: none QRS: 0.12-0.14 second Interpretation: Ventricular tachycardia So, the client is in ventricular tachycardia without a pulse. This means there is no cardiac output due to extreme heart rates and lack of coordinated atrial contraction (loss of "atrial kick"). This leads to hypotension, collapse, and acute cardiac failure. Decreased cardiac output may result in decreased myocardial perfusion with degeneration to Ventricular Fibrillation.

A child with a radial fracture reports itching below the casted area. What is the appropriate nursing action to relieve itching? 1. Allow the child to use a Q-tip to scratch the area. 2. Assess the fingers and areas above the cast to identify areas of irritation. 3. Apply an ice pack for 10-15 minutes. 4. Explain to the child that itching is an indication the fracture is healing.

3. Correct: This will change the sensation. Normally the answer is use a cool blow dryer, but they wanted to see if you would be flexible with what you know. Use an ice pack that will not get the cast wet. 1. Incorrect: A Q-tip is soft, trying to make you feel like this is okay to put into a cast. 2. Incorrect: How does assessing the fingers decrease itching? It doesn't. 4. Incorrect: How does explaining the pathophysiology help decrease itching? It doesn't.

The nurse just received an arterial blood gas (ABG) report that shows a borderline high PCO2 on a client who had chest surgery. What should be the priority nursing intervention? 1. Tell the client to breathe faster. 2. Medicate for pain and ambulate. 3. Have client use the incentive spirometer. 4. Prepare to administer bicarbonate to buffer.

3. Correct: This client had chest surgery and the pCO2 is high. What are you worried about? Hypoventilation. Yes, the client is probably hurting due to the incision and does not want to take deep breaths. In order to get rid of the excess CO2 the client needs to turn, cough, and deep breath. Incentive spirometry can be provided to assist the client with this effort. 1. Incorrect: Breathing faster will only work for a few minutes. The problem is the client needs to breathe deeper to get more oxygen to the tissue and more CO2 out of the lungs. Hyperventilating will lead to respiratory alkalosis. 2. Incorrect: No more sedation! The client is not breathing enough. Walking would be okay. This client needs to take deep breaths. 4. Incorrect: No, we want the client to blow off the CO2. Bircarb will make the pH happy for a short period of time but will not correct the problem. The problem is shallow respirations, so fix the problem.

The nurse is caring for four clients. Which client should the nurse see first? You answered this question 1. The client hospitalized with dehydration related to diarrhea. 2. The seizure client who is currently in the postictal phase. 3. The post-op client who received Morphine 4 mg IV 15 minutes ago. 4. The client who is due pre-op medication now.

3. Correct: This client is at risk for respiratory depression caused by morphine and should be assessed. Remember airway, breathing and circulation (ABCs). Decreased or suppressed respiration are priority. 1. Incorrect: Dehydration can produce postural hypotension, fever, confusion, agitation and if it develops quickly or is severe, coma and seizure may occur. Decreased respiratory rate would be priority. 2. Incorrect: Postictal is the phase after the seizure where they are drowsy, lethargic, and possibly asleep. Make sure the client is safe and in the recovery position. Client would need to be seen soon, but again, decreased respirations takes priority. 4. Incorrect: Pain never killed anyone. Decreased or suppressed respirations would be priority.

A client with recurrent angina and hypertension has been started on new medications. When reviewing the admission forms, the nurse should immediately question which prescription? *Exhibit* Spironolactone 50 mg. PO once daily Metoprolol 25 mg. PO once daily Diltiazem 120 mg. PO once daily Potassium 10 meq PO once daily 2 gm. sodium diet 1. 2 gm sodium diet 2. Metoprolol 25 mg PO once daily 3. Potassium 10 meq PO once daily 4. Diltiazem 120 mg PO once daily

3. Correct: This client is being treated for recurrent angina with hypertension. The admission prescription includes spironolactone daily, which is a potassium-sparing diuretic; therefore, the client should NOT be taking a daily dose of potassium. 1. Incorrect: A 2 gram sodium diet is considered a low salt diet, which would be appropriate for a client with hypertension. Excessive dietary salt leads to water retention and increased blood pressure. This prescription is appropriate for the client and does not need to be questioned. 2. Incorrect: Metoprolol is a beta-blocker used to decrease preload, which will also decrease pulse and blood pressure. The dose is appropriate for this client and does not need to be questioned. 4. Incorrect: Diltiazem is a calcium channel blocker which vasodilates the arterial system and reduces recurrent angina by decreasing afterload. Additionally, calcium channel blockers help to decrease blood pressure. This medication and dose are appropriate for this client.

A client has been taking tranylcypromine for approximately two weeks. The client is visiting the nurse at the local mental health center for follow up and group therapy. Which client comment indicates a lack of understanding of the medication that could result in a medical emergency? 1. I know that I must take this medication until my primary healthcare provider tells me to stop. 2. It is frustrating to have to follow dietary restrictions. 3. I am getting a cold, and I am going to take some over the counter cold medicine. 4. I am going to have broccoli salad and roasted turkey for lunch today.

3. Correct: This is an MAOI medication. OTC cold medications could result in hypertensive crisis when combined with the monoamine oxidase inhibitor. Warnings are placed on cold preparations and other medicines that are not to be taken with the MAOIs. Be cautious about adding over the counter medications and possible drug interactions. 1. Incorrect: This is a true statement. The primary healthcare provider's orders should be followed. This comment would not result in a medical emergency. 2. Incorrect: Foods high in tyramine should be avoided while taking tranyclpromine, and dietary restrictions must be followed. This comment indicates correct understanding of this medication. 4. Incorrect: This dietary choice demonstrates adequate understanding of the medication and dietary restrictions required.

A client diagnosed with lung cancer is told that the client only has about 6 months to live. The spouse tells the nurse, "I pray every night that God will give me more time with my loved one." Which Kübler-Ross stage of grief does the nurse recognize the spouse to be exhibiting? 1. Anger 2. Acceptance 3. Bargaining 4. Depression

3. Correct: This is the 3rd stage of grief. At this stage, the individual attempts to strike a bargain with God for a second chance, or for more time. The person acknowledges the loss, or impending loss, but holds out hope for additional alternatives. 1. Incorrect: The 2nd stage of grief is when reality sets in. Feelings include sadness, guilt, shame, helplessness, and hopelessness. Self-blame or blaming of others may lead to feelings of anger toward self and others. 2. Incorrect: The fifth and last stage of grief. At this time, the person has worked through the behaviors associated with the other stages and either accepts or is resigned to the loss. Anxiety decreases, and methods of coping with the loss have been established. 4. Incorrect: The 4th stage of grief. The individual mourns that which has been or will be lost. This is a very painful stage, during which the person must confront feelings associated with having lost someone or something of value.

The nurse is caring for an oncology client with a WBC-5.5 x 103 /mm3, Hgb-12g/dL, PLT-90 x 103 /mm3. Which measure should be instituted? 1. Protective isolation 2. Oxygen therapy 3. Bleeding precautions 4. Strict intake and output

3. Correct: Yes. That is the only value that is not a normal level, and it is way too low, so this client is at risk for bleeding. Bleeding precautions are the appropriate intervention. A normal platelet count ranges from 150,000 to 450,000 platelets per microliter of blood. Having more than 450,000 platelets is a condition called thrombocytosis; having less than 150,000 is known as thrombocytopenia. 1. Incorrect: The WBC is okay. An average normal range is between 4,500 and 10,000 white blood cells per microliter (mcL). Leukopenia is the medical term used to describe a low WBC count. 2. Incorrect: There is no indication of hypoxia in stem, and the Hgb is normal, so the client is not anemic. A low hemoglobin count is generally defined as less than 13.5 grams of hemoglobin per deciliter (135 grams per liter) of blood for men and less than 12 grams per deciliter (120 grams per liter) for women. 4. Incorrect: There is no indication for I & O measurement. I&O should be done with clients who have a fluid volume, cardiac, or renal problem.

A client who has a history of major depression is in the emergency department. Which statement would demonstrate a risk for suicide or self-directed injury? 1. "I can't do anything right anymore." 2. "I am not sure what to do anymore." 3. "I just cannot take this loneliness anymore." 4. "No one cares about me."

3. Correct: This statement indicates that the person cannot tolerate the current situation. The client is at risk for harm to themselves. 1. Incorrect: The client has a negative outlook about themselves. This statement indicates low self-esteem. The client is not exhibiting suicidal tendencies. 2. Incorrect: The client is having difficulty making choices. This statement indicates indecisiveness, which is a symptom of depression. Indecisiveness is not a risk for suicide or self-directed injury. 4. Incorrect: This statement indicates possible social isolation and low self-esteem. The client maybe physically separated from people or the perception of being isolated from others. The client does not exhibit a loss of hope that is connected with suicide.

What would be the nurse's priority for a child who has arrived at the emergency department after sustaining a severe burn? 1. Start intravenous fluids. 2. Provide pain relief. 3. Establish airway. 4. Place an indwelling catheter.

3. Correct: This stem does not tell you where the child's burns are, however, you are told that the burns are severe. So assume the worse. What are you most worried about the child losing? Yes, the airway. So we want to assess, establish, and maintain an airway. 1. Incorrect: Not before airway. This child will need IV fluid resuscitation within the first 24 hours. But if you can only pick one action to complete first, it better be to make sure the airway is patent. Then you can start the IV or delegate the task to someone else. 2. Incorrect: Give pain med after starting the IV, not before airway. The best pain relief method for a severe burn is going to be through the IV route. But we must make sure the airway is patent first. 4. Incorrect: Give the pain medication before placing the indwelling catheter but not before establishing the airway. Intake and output will need to be closely monitored in the client who is severely burned. But again, it will need to be done after establishing a patent airway.

The nurse is teaching crutch walking to a client with a fractured lower leg with a non weight bearing cast. Which crutch gait would be most appropriate for the nurse to teach? 1. Swing through 2. Two point 3. Three point 4. Four point alternating

3. Correct: Three point gait. All of the weight bearing is done by the unaffected leg and the crutches. The injured leg does not touch the ground during the performance of this gait. This is most appropriate for the client with a lower leg cast. 1. Incorrect: The swing through alternating gait would require some form of weight bearing on the fractured leg. This would not be an acceptable form of crutch walking for this client. 2. Incorrect: The two point alternating gait would require some form of weight bearing on the fractured leg. This would not be an acceptable form of crutch walking for this client. 4. Incorrect: The four point alternating gait would require some form of weight bearing on the fractured leg. This would not be an acceptable form of crutch walking for this client.

Which action should the nurse recommend to parents so that their home will be safer for a toddler? 1. Place the child in the center of an adult-sized bed when napping. 2. Buckle the child into the high chair if parents leave the room during a meal. 3. Anchor top-heavy furniture or fish tanks so that they cannot be pulled over. 4. Allow the toddler to explore stairs in the home if supervised.

3. Correct: Top-heavy furniture, TVs, and fish tanks can be pulled over by the toddler, especially if the child is trying to reach something on top of them. 1. Incorrect: The safest place for the toddler to nap or sleep is in a crib. The toddler may easily fall from an adult-sized bed. 2. Incorrect: The toddler should never be left unsupervised in a highchair. It can tip if the child tries to climb out, or the child may push against something and fall. 4. Incorrect: Stairs in the home present a risk for falls and accidents for the toddler. Safety gates should be in place, and the adults should hold the toddler's hand when navigating the stairs.

The nurse is evaluating care provided by an unlicensed assistive personnel (UAP). Which action should the nurse interrupt the UAP from performing? 1. Draining the colostomy bag on a client with diarrhea. 2. Performing passive range of motion (ROM) on the client with right sided paralysis. 3. Placing the traction weights on the bed to transfer the client to x-ray. 4. Discarding the first urine voided by the client starting a 24 hour urine test.

3. Correct: Traction should never be relieved without a primary healthcare provider's prescription. It can result in muscle spasm and tissue damage. This client could be transferred with traction still maintained. 1. Incorrect: A colostomy client with diarrhea will have a lot of drainage requiring frequent emptying of the colostomy bag. Draining of the bag is a routine toileting procedure for the colostomy client and.is within the scope of practice for the UAP. 2. Incorrect: Passive ROM is performed with paralysis and can be delegated to the UAP. Each ROM movement should be repeated 5 times during the session. 4. Incorrect: The first void of a 24 hour urine is discarded and can be delegated to the UAP. The nurse would then start the 24 hour urine once the 1st void has been discarded. The nurse also needs to be aware of the color and amount of urine voided.

A school nurse is caring for a child who fell on the playground. Upon examination of the child, the nurse notes multiple bruises in various stages of healing. What is the nurse's initial intervention? 1. Ask the parents who hit the child on the back. 2. Notify the child's primary healthcare provider. 3. Contact the Department of Health and Human Services. 4. Document the findings and observe the child over the next week.

3. Correct: Unless there is a policy to direct otherwise, the nurse who suspects child abuse is obligated to report it to the Department of Health and Human Services (DHS). 1. Incorrect: This is confrontational and will warn the parents that you suspect abuse. This may lead to greater harm for the child. 2. Incorrect: Unless there is a policy to direct otherwise, the nurse who suspects child abuse is obligated to report it to the Department of Health and Human Services. DHS, rather than the primary healthcare provider can intervene to maintain the child's safety. 4. Incorrect: This is delaying care. If the child is being abused, not reporting it could lead to serious injury or even death.

The nurse is caring for a client diagnosed with deep vein thrombosis, who has been treated with intravenous heparin for one week. The primary healthcare provider plans to change the medication from heparin IV to warfarin sodium by mouth. The nurse understands which approach would be appropriate? 1. Begin the warfarin sodium and stop the heparin simultaneously. 2. Stop the heparin 24 hours, then begin the warfarin sodium. 3. Begin the warfarin sodium before stopping the heparin. 4. Stop the heparin, wait for the coagulation studies to reach the control value, and begin the warfarin sodium.

3. Correct: Warfarin sodium is initiated while the client remains on heparin. This is done so that the client remains adequately anticoagulated during the transition from IV heparin to warfarin sodium. The onset of action of warfarin sodium is 36 hours to 3 days. 1. Incorrect: Warfarin sodium's onset of action is 36 hours to 3 days. If heparin were stopped and warfarin sodium initiated there would be a lag time when the client would be inadequately anticoagulated and at an increased risk for clotting. 2. Incorrect: Warfarin sodium's onset of action is 36 hours to 3 days. Stopping heparin 24 hours before administering warfarin sodium would cause a lag time and increased risk of clotting. 4. Incorrect: Waiting for coagulation studies before administering warfarin sodium would cause a lag time and put the client at increased risk for clotting. Additionally, heparin and warfarin are measured by different clotting lab tests. The aPTT can measure the effectiveness of heparin. The PT and INR can be used to measure the effectiveness of warfarin sodium.

The nurse is assessing a pregnant client returning for her first, one month check-up. The client has normal vital signs, blood count, and urinalysis, but has gained 6 pounds (2.7 kg). What is the most important assessment at this time? 1. Blood glucose level 2. Ankles for edema 3. Twenty-four hour diet recall 4. Confirmation of last menstrual period

3. Correct: What is she eating? how much? Are the calories healthy? This is too much weight. 1. Incorrect: Weight gain: think eating or fluid, not blood sugar 2. Incorrect: First month: this weight gain is from excessive eating, snacking, etc. 4. Incorrect: Not an indication that the date is wrong This question is asking for the "most" important assessment. What are we worried about? PIH or eating too much and gaining weight? Well the BP is normal. So can we eliminate PIH as a possible problem? Yes. Option 4 can be eliminated. We know the client is pregnant. Look at the clues in this stem. "Pregnant" "Normal vital signs" "Gained 6 pounds", "assessment". So what option relates to gaining 6 pounds in one month and is an assessment? Does glucose have anything to do with gaining weight? No, so eliminate that one. Now, we are left with options 2 and 3. The BP and labs are normal, so this does not point to PIH. What is the best assessment for determining weight gain? Option 3: the 24 hour diet recall.

While performing wound care to a donor skin graft site, the nurse notes some scabbing around the edges and dark collection of blood. What is the nurse's next action? 1. Leave the scabbing area alone and apply extra ointment. 2. Notify the primary healthcare provider. 3. Gently remove the debris and re-dress the wound. 4. Apply skin softening lotion for 3 hours and then re-dress.

3. Correct: What likes to live in the scabs and dried blood? Bacteria. That is why it is important to remove the debris to prevent infection. 1. Incorrect: This is not appropriate because bacteria is in the scabbing area and ointment would trap it, enhance reproduction of the germs, and increase infection. 2. Incorrect: There is no need to notify primary healthcare provider at this time. This is not the best option for the nurse to fix the problem. 4. Incorrect: We don't put lotion in the wound because this would cause infection of the wound.

Post epidural anesthesia, a laboring client's blood pressure drops to 92/42. Which intervention by the nurse takes priority? 1. Elevate the head of the bed 2. Administer oxygen by face mask at 40% 3. Position client side-lying 4. Begin dopamine 5 mcg/kg/min

3. Correct: When you turn them on their side, this relieves pressure on the vena cava and the BP will go UP. 1. Incorrect: This will drop the pressure more. 2. Incorrect: O2 doesn't bring up the BP. 4. Incorrect: Stay away from drugs as long as you can.

The nurse is performing sterile wound care for partial thickness burns on a client's lower right leg. Prior to initiating this procedure, what action should the nurse complete first? 1. Position client upright with right leg elevated. 2. Obtain wound culture before cleaning wound. 3. Assess current pain level and medicate. 4. Encourage client to verbalize concerns.

3. Correct: Wound care on burns is a painful process, particularly with partial thickness burns (formerly referred to as second degree) because nerve endings are intact and exposed. Pre-medicating is a priority action, since pain medication can take up to 30 minutes to activate within the body. Clients are more cooperative and heal faster when pain is well controlled. 1. Incorrect: Proper visualization during wound care is vital, as is client comfort during the procedure. However, completion of this process does not require the client to be in an upright position. In fact, that may be counter productive at this time. Additionally, whether the right leg needs elevated depends on the size or location of the burn on the right leg, and that information has not been provided in the question. 2. Incorrect: While it is true that any wound culture must be obtained prior to cleaning the affected area, this action is not presently the nurse's first priority. Consider the nursing process and choose another option. 4. Incorrect: Therapeutic communication is an on going process during any client interaction, particularly when the nurse needs to explain an upcoming procedure. Allowing the client to express fears, verbalize concerns or ask questions enhances cooperation. Although this exchange of information is occurring throughout this period of time, the nurse has another priority action that should be completed first.

The nurse is caring for a client with a history of Gastroesopeageal Reflux (GERD). Based on the assessment, what is the priority intervention? *Exhibit (nursing notes)* Lung sounds diminished on the left side posteriorly with crackles noted bilaterally. Frequent, moist cough. Temp of 103º F (39.45ºC) 1. Administer ibuprofen 2. Establish IV access 3. Obtain oxygen saturation 4. Chest x-ray

3. Correct: Y'all, what do you think is wrong with this client? Did you say.............................Pneumonia? Yes, these are classic things we see with Pneumonia, so you better be checking oxygenation. Is a client with GERD at risk for pneumonia? Yes, because the acid that is refluxing can be aspirated into the lungs. 1. Incorrect: Oxygenation takes priority here. 2. Incorrect: The nurse needs to check oxygenation first. 4. Incorrect: Oxygenation takes priority.

Which assessment by the nurse indicates a tension pneumothorax? 1. Sudden hypertension and bradycardia 2. Productive cough with yellow mucus 3. Tracheal deviation and dyspnea 4. Sudden development of profuse hemoptysis and weakness

3. Correct: Yes, as pleural pressure on the affected side increases mediastinal displacement occurs with resultant respiratory and cardiovascular compromise. Symptoms of tension pneumothorax include dyspnea, chest pain radiating to the shoulder, tracheal deviation, decreased or absent breath sounds on the affected side, neck vein distention and cyanosis. 1. Incorrect: Hypoxia causes tachycardia rather than bradycardia. The client would more likely to be hypotensive due to decreased cardiac output. 2. Incorrect: Yellow mucus indicates infection, such as from pneumonia. This does not indicate a tension pneumothorax. 4. Incorrect: Profuse hemoptysis and weakness may indicate a serious condition such as a ruptured vessel, but it is not an indication of a mediastinum shift.

What would the nurse include when teaching a client newly prescribed timolol maleate eyedrops for glaucoma? 1. The medication works by causing the pupils to constrict 2. The medication will dilate the canals of Schlemm 3. This medication decreases the production of aqueous humor 4. The medication improves ciliary muscle contraction

3. Correct: timolol does decrease aqueous humor formation; therefore decreasing IOP 1. Incorrect: timolol does not constrict pupils​ 2. Incorrect: timolol does not dilate the canals of Schlemm​ 4. Incorrect: timolol does not cause ciliary muscle contraction​ 1. Look at each option as True or False. 2. Option 1 in false. Miotics are medications that cause pupillary constriction such as pilocarpine. The action of miotics are to increase aqueous fluid outflow by contracting ciliary muscle and causing miosis (constriction of the pupil) and opening of the trabecular network. 3. Option 2 is false. The canal of Schlemn may be widened by laser trabeculoplasty to promote outflow of aqueous humor and decrease IOP. 4. Option 3 is true. Timolol maleate is a beta-blocker. Beta-blockers decrease aqueous humor production. 5. Option 4 is false. Ciliary muscle contraction is affected by cholinerigcs causing an increase in the outflow of aqueous humor through a larger opening between the iris and the trabecular meshwork.

The nurse is caring for a client diagnosed with pneumonia. The primary healthcare provider has prescribed erythromycin. What teaching points should the nurse plan to teach the client regarding this medication? 1. Crush the medication if unable to swallow capsule. 2. Take erythromycin 1 hour after eating. 3. Report clay-colored stools. 4. Do not take erythromycin with grapefruit juice. 5. Keep capsules in bathroom cabinet.

3., & 4. Correct: The client should be taught signs and symptoms of liver problems such as nausea, increased stomach pain, itching, tired feeling, loss of appetite, dark urine, clay-colored stools, or jaundice. Grapefruit juice can interfere with absorption of this medication. 1. Incorrect: Do not crush, chew, or break a delayed release capsule or tablet. Swallow it whole. 2. Incorrect: This medication should be taken 1 hour before or 2 hours after a meal. 5. Incorrect: Keep at room temperature, away from excess heat and moisture (not in bathroom).

The nurse manager of an Alzheimer's unit as completed inservice education to new nursing staff regarding guidelines for dealing with dementia. Which identified guidelines by the new nursing staff indicates to the nurse manager that education was successful? 1. Use a firm touch to guide the client to a different location when needed. 2. Be persistent when getting the client to do something. 3. Provide simple directions using gestures or pictures. 4. Do not argue with the client. 5. Play memory games to decrease dementia. 6. Require participation in daily activities.

3., & 4. Correct: When a person is confused and has dementia, we need to communicate in a simple manner. Provide simple directions or instructions, short sentences, and gestures. Use pictures. Do not give instructions on multiple things. Do not argue, criticize, or correct the client. This can increase anxiety, agitation, and anger. 1. Incorrect: Use a gentle touch rather than a firm touch with these clients. You do not want to be confrontational or evoke fear in the client. 2. Incorrect: Be flexible. If one approach does not work, try another. 5. Incorrect: Avoid questions or topics that require extensive thought, memory, or words. This can increase anxiety, frustration, and agitation. 6. Incorrect: Do not require or force participation in activities or events. This can increase anxiety, frustration, and agitation. Education on appropriate nursing actions with dementia clients is very important especially for new nursing staff. Knowledge of dementia is necessary. Behavioral problems occur in the majority of dementia client's and nurses need appropriate responses. Option 1: No. First the key word in this option is firm. The client with dementia may be fearful and firm touch may cause increased fear. Use a gentle touch. Option 2: No. Persistence will not work with dementia clients. Flexibility is much better as it allows the nurse to modify actions as needed. Option 3: Excellent choice. Directions must be simple as this client has decreased cognitive functioning. Also the use of gestures and pictures may greatly increase their understanding and thus their willingness to follow directions. Option 4: For sure. Arguing is a nontherapeutic communication technique for any client! This will not work with dementia clients and worse, may likely increase behavioral problems. Option 5: Sadly, no. Dementia is caused by severe changes in the brain. The memory games will be very frustrating to the cleint and bring about no positive change in memory. Option 6: Not a good choice. The key word in this option is require. Yes, the nurse wants to offer and encourage participation in activities as a positive means of redirection and distraction. The word "require" makes this option wrong.

What should the nurse include when teaching a client diagnosed with Grave's disease who is scheduled to receive radioactive iodine? 1. Stay 6 feet from people for 2 weeks. 2. This medication is given intravenously as a one-time dose. 3. Radioactive iodine will leave the body in urine and saliva within a few days. 4. You cannot receive radioactive iodine if you are pregnant. 5. Radioactive iodine is absorbed by the parathyroid glands.

3., & 4. Correct: Within a few days after treatment, the radioactive iodine will leave the body in urine and saliva. If the client is pregnant, she should not receive radioactive iodine treatment. This kind of treatment can damage the fetus's thyroid gland or expose the fetus to radioactivity. Women should wait a year before conceiving if they have been treated with radioactive iodine. 1. Incorrect: Stay away from babies for 1 week and do not kiss anyone for 1 week. 2. Incorrect: Radioactive iodine is given in a capsule or liquid form. One dose is usually all that is needed. 5. Incorrect: Radioactive iodine is absorbed by the thyroid gland. It destroys the thyroid. So now the client becomes hypothyroid.

Which tasks should the nurse delegate to the unlicensed assistive personnel (UAP)? 1. Demonstrate post operative exercises. 2. Reposition the transcutaneous electrical nerve stimulation (TENS) unit. 3. Empty the indwelling catheter bag. 4. Assist a client with position change every 2 hours. 5. Apply anti-embolism stockings.

3., 4. & 5. Correct. It is within the role of an UAP to empty the indwelling catheter bag, assist with position change and apply anti-embolism stockings. The nurse should confirm that these tasks have been done, but they are safe to delegate to the UAP. 1. Incorrect. This is a task for the RN and involves teaching and evaluation of effectiveness. 2. Incorrect. The physical therapist is the best team member to manage the TENS unit since this is a pain control device that affects nerves and muscles. What can an UAP do? The UAP can perform tasks related to activities of daily living and routine tasks. Remember, the RN cannot delegate assessment, evaluation, plan of care development or teaching to an LPN or UAP. And make sure to follow the 5 rights of delegation: Right task, right person, right circumstance, right direction, right supervision. Option 1 says to have the client demonstrate post op exercises. Can the UAP evaluate whether these are done correctly? No. This is part of the nursing process that cannot be delegated: Evaluation. Option 2 wants you to delegate repositioning of a TENS unit to the UAP. Is this routine? No. Does it having anything to do with ADL? No. False. Option 3 says empty the indwelling catheter bag. Can the UAP measure urine? Yes, the UAP is trained to do this. Option 4: Help the client change positions. Can the UAP assist with doing this? Yes. Option 5: Apply anti-embolism stockings? Yes, the UAP is trained to do this.The nurse should confirm that these tasks have been done, but they are safe to delegate to the UAP.

A client tells a clinic nurse of plans to travel to Europe by plane. What tips should the nurse provide the client regarding prevention of clot formation? 1. Do not cross legs longer than 15 minutes at a time. 2. Get up and move around the plane every 4 hours. 3. Wear compression stockings while traveling. 4. Frequently move legs while sitting. 5. Avoid coffee while traveling.

3., 4. & 5. Correct: Compression stockings put gentle pressure on the leg muscles. Studies in healthy people have shown that wearing compression stockings minimizes the risk for developing DVTs after long flights. It is important for passengers to keep moving their legs to help the blood flow, even when waiting in the airport terminal. Alcohol and coffee contribute to dehydration, which can lead to thickened blood and increased risk for DVT. 1. Incorrect: Do not cross legs at all. 2. Incorrect: The client should get up and move around at least every 2 hours. When walking, the muscles of the legs squeeze the veins and move blood to the heart.

Which tasks can the RN delegate to an unlicensed assistive personnel (UAP) when caring for a client who has had a stroke and is being rehabilitated? 1. Assess a client's ability to swallow. 2. Develop a plan of care for hygiene needs. 3. Assist the client using a walker. 4. Calculating the intake and output. 5. Encourage and assist the client with the use of a hairbrush on the affected side. 6. Teach the family about the need to prevent pressure ulcers.

3., 4. & 5. Correct: It is within the role of the UAP to assist a client with a walker. The UAP can collect and calculate intake and output. When assisting the client with hygiene needs, the UAP can promote strengthening of the affected side by encouraging and assisting the client to place the hairbrush in the hand of the affected side. 1. Incorrect: The UAP cannot assess a client. Assessment is not in role of the UAP and is also outside the scope of practice for LPN's. Assessments must be performed by the RN. 2. Incorrect: Although the UAP can provide care and assistance related to hygiene needs, the planning of care cannot be delegated to the UAP. This is part of the nursing process and must be performed by the RN. 6. Incorrect: Teaching cannot be delegated to the UAP and is also outside the scope of practice for the LPN. Teaching is a responsibility and in the scope of practice for the RN.

A client admitted to the psychiatric unit after a suicide attempt is placed on suicide precautions. Which nursing interventions would be appropriate? 1. Assign the client to a private room away from nurses station. 2. Make rounds to assess the client at regular intervals. 3. Secure a promise that the client will seek out staff when feeling suicidal. 4. Closely supervise the client during meals. 5. Formulate a no harm contract for the client to sign.

3., 4. & 5. Correct: Remove harmful objects from the client's access, such as sharp objects, straps, belts, ties, glass items, and alcohol. Close supervision is necessary during meals. Increased feelings of self-worth may be experienced when the client feels accepted unconditionally regardless of thoughts or behavior. 1. Incorrect: Clients should not be left alone for long periods of time. A private room close to the nurses' station is best. Do not allow clients to spend too much time alone in their rooms. 2. Incorrect: Make rounds frequently, at least every 15 minutes. Do not become predictable with rounds. Predictability allows the client to learn your routine and potentially harm themselves during the time they know you will not be in there.

What independent nursing interventions should the nurse include when planning care for a client who is in a fluid volume excess (FVE)? 1. Monitor Central venous pressure (CVP) 2. Administer diuretic 3. Monitor for orthopnea 4. Raise head of bed (HOB) to 45 degrees 5. Elevate edematous extremities

3., 4. & 5. Correct: These are independent nursing actions that will increase venous return and decrease edema. Also the nurse should assess for crackles, changes in respiratory pattern, shortness of breath (SOB), orthopnea. 1. Incorrect: This is a collaborative intervention. 2. Incorrect: This is a dependent intervention. Dependent nursing interventions are those that require an order from other health care professionals. So which options can the nurse initiate without a prescription? What about option 1? Monitor Central venous pressure (CVP). False. This would be considered an interdependent nursing intervention. The primary healthcare provider would have to place the CVP line in order for CVP monitoring to be done. Option 2? Administer a diuretic? False. This is a dependent intervention as a prescription is needed. Option 3? Monitor for orthopnea? Does the nurse need a prescription to do this? No. Does the nurse need to collaborate with anyone to do this? No. So this is a True statement. Option 4? Elevate the HOB? This is true. The nurse can do this independently. A prescription is not needed. Option 5? Elevate edematous extremities? True. No prescription needed here!

The nurse is caring for a client diagnosed with heart failure who has developed pulmonary edema. Which findings best indicate that the client's medications are having a therapeutic effect? *Exhibit (assessment & prescription)* *assessment* Blood pressure (BP) of 190/122, apical heart rate of 134, respiratory rate (RR) of 34 and urine output of 28 mL per hour. *prescription* Digoxin 0.125 mg IV now Furosemide 40mg IV now 1. Respiratory rate of 34/min 2. Blood pressure 90/50 3. Urine output of 100 mL over the last hour 4. Apical heart rate of 88/min 5. Blood pressure of 160/90

3., 4. & 5. Correct: With pulmonary edema the client will have a rapid, irregular heart beat. The heart rate has decreased to within normal range. The blood pressure has decreased, and the urine output has increased. 1. Incorrect. The respiratory rate is still too high, so the pulmonary edema has not resolved. 2. Incorrect. The blood pressure is too low. We worry when the blood pressure gets to 90 systolic, or 30mmHg off the baseline.

The nurse is caring for a client admitted with an episode of bleeding esophogeal varices. What should the nurse monitor for after administering propranolol to this client? 1. Increased systolic BP 2. Hypokalemia 3. Bradycardia 4. Wheezing 5. Decreased hematemesis

3., 4., & 5. Correct: Propranolol is a beta blocker that affects the heart and circulation. It is used in the treatment of high blood pressure, irregular heartbeats and in the prevention of angina and headaches. This medication works by blocking epinephrine and reduces heart rate, blood pressure and strain on the heart. Decreasing the heart rate should decrease bleeding. Wheezing is an adverse reaction from propranolol and should be monitored for after administration. A decreased in heart rate and blood pressure will help to decrease bleeding. Hematemesis is vomiting blood. 1. Incorrect: Blood pressure is the force of blood flow against the walls of your arteries. Propranolol should decrease blood pressure, thus decreasing bleeding. 2. Incorrect: Beta blockers inhibit renin release which can decrease the release of aldosterone. We should monitor for hyperkalemia, rather than hypokalemia.

The nurse is working with clients who are diagnosed with eating disorders. Which eating disorder would the nurse expect to cause the greatest fluctuation in serum potassium levels? Dysthymic disorder Anorexia nervosa Binge eating disorder Bulimia nervosa

Bulimia nervosa Hypokalemia can be caused by overuse of laxatives and by prolonged fasting and starvation. But the greatest fluctuation in potassium levels is associated with bulimia, due to the purging process that causes dehydration and potassium loss. Low potassium levels can cause weakness, abdominal cramping and irregular heart rhythms. Dysthymic disorder is associated with poor appetite or overeating.

What signs/symptoms would the nurse expect to assess in a client diagnosed with Guillain-Barre' Syndrome? 1. Opisthotonos 2. Seizures 3. Paresthesia 4. Hemiplegia 5. Hypotonia 6. Muscle aches

3., 5., & 6. Correct: Guillain-Barre' Syndrome is an acute, rapidly progressing, and potentially fatal form of polyneuritis. It is characterized by ascending, symmetric paralysis affecting the cranial and peripheral nerves. Signs and symptoms include paresthesia, hypotonia, areflexia, muscle aches, cramps, orthostatic hypotension, hypertension, bradycardia, facial flushing, facial weakness, dysphagia, and respiratory distress. 1. Incorrect: Opisthotonos is extreme arching of the back and retraction of the neck. This is seen with tetanus, not with Guillain-Barre' Syndrome. 2. Incorrect: Seizures can be associated with many neuromuscular problems but are not typical with Guillain-Barre' syndrome. Look for seizures with such problems as increasing ICP, infections of the brain, high fever, epilepsy. 4. Incorrect: Hemiplegia, paralysis on one side of the body, is not seen. There is a symmetric paralysis starting in the lower extremities and ascending through the body. In other words, weakness begins in the feet and progresses upward. The client gets better in reverse order.

The nurse is caring for a client that is 3 days post tonsillectomy and reports a 2 pound (0.91 kg) weight loss, lethargy, and frequent swallowing. What is the nurse's priority assessment? 1. Urinary Output 2. Daily weight 3. Heart rate 4. Breath sounds

3.Correct: This client is in a FVD. The weight has dropped. Fluid is moving out of the vascular space and the client is getting worse so what could happen to my heart rate... drop! And then I am not going to perfuse adequately and I could kill my brain, kidneys..............all my vital organs. Right? My body will be able to compensate for awhile, but when my HR starts to drop I know my client is getting worse. 1. Incorrect: SURE, you are going to watch the urinary output and the daily weight. Breath sounds are very important as well and one of the first assessments we make, BUT if I can only do one of these assessments, I better take the heart rate because that is the one that says SHOCK 2. Incorrect: SURE, you are going to watch the urinary output and the daily weight. Breath sounds are very important as well and one of the first assessments we make, BUT if I can only do one of these assessments, I better take the heart rate because that is the one that says SHOCK 4. Incorrect: SURE, you are going to watch the urinary output and the daily weight. Breath sounds are very important as well and one of the first assessments we make, BUT if I can only do one of these assessments, I better take the heart rate because that is the one that says SHOCK

The nurse is caring for a client receiving an intravenous infusion of normal saline that is prescribed at 150 milliliters per hour. Using a tubing that has a drop factor of 15, how many drops per minute should the nurse deliver? Round answer to the nearest whole number.

38 Divide 150 by 60 minutes to equal 2.5. Multiplying 2.5 by the drip factor of 15 equals 37.5. Since partial drops cannot be counted, always round to nearest whole number, which is 38.

Labetalol has been prescribed for a client in the emergency room. Prior to administering this medication, what assessment should the nurse perform? 1. Listen to the client's breath sounds. 2. Check the client's temperature. 3. Monitor for peripheral edema. 4. Auscultate the apical pulse rate.

4 Correct: The therapeutic effect of labetalol, which is a beta blocker, is to lower the blood pressure and decrease the heart rate. Apical pulse should be assessed for 1 full minute. If pulse is less than 60 the medication is held and the healthcare provider should be notified. 1. Incorrect: Indirectly a beta blocker could affect breath sounds but assessing breath sounds is not as important as taking the client's apical pulse. Beta blocks should be used cautiously in clients with a history of COPD or asthma these could cause airways to constrict. 2. Incorrect: Labetalol does not affect the client's temperature. This is not a side effect of labetalol. 3. Incorrect: Indirectly a beta blocker could affect the amount of peripheral edema, however, assessing for peripheral edema is not as important as taking the client's apical pulse.

The nurse asks if the client has an advance directive. The client responds by saying, "What is an advance directive?" What is the nurse's best response to the client's question? 1. Specifies your wishes regarding your personal effects and finances should you become unable to make decisions. 2. Specifies your wishes regarding healthcare and your finances should you become incapacitated. 3. Similar to a will, it specifies your wishes for burial should you die during hospitalization. 4. Specifies your wishes regarding healthcare and treatment options should you become incapacitated. 5. The person signing the advanced directive must be competent.

4. & 5. Correct: An advance directive is a legal document prepared by a competent individual that specifies what treatments, if any, the client desires should the client become incapacitated or unable to make informed healthcare decisions in the future. The person signing the advance directive must be able to understand and agree with the document. 1. Incorrect: An advance directive does not address client personal effects and finances. These might be included in a last will and testament but are not part of an advance directive. 2. Incorrect: An advance directive does not address client personal effects and finances. These might be included in a last will and testament but are not part of an advance directive. 3. Incorrect: An advance directive does not address burial wishes. So what exactly is an advance directive? Advance directives are legal documents that allow you to spell out your decisions about end-of-life care ahead of time. They give you a way to tell your wishes to family, friends, and health care professionals and to avoid confusion later on. Based on this definition let's see which options fit. Option 1: That's false. Specifying what a person wants to happen to personal belongings and finances after death is made known through a last will and testament, not an advanced directive. Option 2: This is false as well. Financial issues are not covered in an advanced directive. That would be done through a Power of Attorney which provides the authority to act for another person in specified or all legal or financial matters. Option 3: This is false. An advance directive does not specify wishes after death. Option 4. This is pretty close to the definition of advance directive, isn't it? Yes, so this is true. Option 5. This is true as well. At the time a person signs an advance directive, or any legal document, that person must be competent.

The emergency room nurse is assessing a client reporting severe abdominal pain for several hours prior to arrival at the hospital. Assessment findings include slight mottling of the lower extremities and a pulsating mass near the umbilicus. Which actions should the nurse implement immediately? 1. Position client on the left side. 2. Apply warm blankets to legs. 3. Administer I.M. pain medication. 4. Alert the operating room staff. 5. Notify the primary healthcare provider. 6. Palpate mass to determine size.

4. & 5. Correct: The client's symptoms indicate the presence of an aortic abdominal aneurysm that may be dissecting (rupturing) at this time. This is a life-threatening emergency and the client will need urgent surgery to survive. The nurse should immediately notify the healthcare provider and alert the operating room staff of impending surgery. 1. Incorrect: These are the classic symptoms of a dissecting abdominal aneurysm, a life-threatening situation requiring immediate surgery. Positioning the client on either side is contraindicated as that action may cause further internal bleeding, complete rupture of the aneurysm, or death. 2. Incorrect: Mottling of lower extremities accompanied by severe abdominal pain suggests a dissecting abdominal aneurysm. The discoloration of lower extremities indicates compromised circulation secondary to interrupted blood flow because of the aneurysm. This client would not benefit from warm blankets but rather needs immediate surgery to survive. 3. Incorrect: Pain medications in general are not administered until an exact diagnosis is confirmed, since relieving pain would mask those signs or symptoms needed to verify the problem. While the client may be given medications at some point, this is not the life-saving action the nurse must take immediately. 6. Incorrect: The client's symptoms are suggestive of a dissecting abdominal aneurysm, a life-threatening emergency requiring immediate surgical intervention. It is never acceptable for the nurse to palpate an abdominal mass, particularly a pulsating mass, since this would likely cause complete rupture of the blood vessel and immediate death.

A medical-surgical LPN has been sent to a short-staffed pediatric unit. The charge nurse knows what client would be most appropriate for this LPN? 1. 3 month old child with nonorganic failure to thrive. 2. 14 year old with exacerbation of cystic fibrosis. 3. 5 year old newly admitted with epiglottitis. 4. 10 year old with type 1 diabetes mellitus.

4. CORRECT. A medical-surgical LPN would likely have seen and cared for diabetics on the floor, including checking fingerstick blood sugars and injecting insulin. A 10 year old school-age child would also be more cooperative, making it easier for the LPN to interact with that client. 1. INCORRECT. This client is only 3 months old, which would require specialized skills to evaluate developmental needs. Additionally, nonorganic failure to thrive is a serious situation in which the infant is not getting appropriate nutrition. There could be economic factors, resulting from a lack of food or poor-quality breast milk. Parental beliefs or negligence could also contribute to the situation; therefore, an RN should be assigned to this infant. 2. INCORRECT. Although this client is an adolescent, an exacerbation of cystic fibrosis would require careful and frequent respiratory assessments with possible chest physiotherapy. This client would be more appropriate for an RN. 3. INCORRECT. A new admission is not appropriate for a nurse sent from the medical surgical unit to the pediatric unit, particularly an LPN, because of the need for initial and frequent assessments. Epiglottitis is a respiratory illness that also impacts the airway. This child should be assigned to an RN.

A child with acute lymphocytic leukemia (ALL) is receiving chemotherapy through a single lumen Groshong catheter. During the infusion, the child complains of nausea and vomits. The primary healthcare provider has ordered an ondansetron IV. The nurse is aware the most appropriate method to administer this medication is what? 1. Ask primary healthcare provider for an oral antiemetic. 2. Give antiemetic as a piggyback with the chemotherapy. 3. Wait until chemotherapy is complete to infuse antiemetic. 4. Stop chemo temporarily and flush line to give the antiemetic.

4. CORRECT: A Groshong catheter is implanted when other venous access sites are no longer useable. The child has begun to react to the chemo and needs medicated now. Because this implanted device has only one lumen, the nurse must stop the chemo infusion temporarily, flush the port, administer the ondansetron, flush again and restart the chemotherapy infusion. 1. INCORRECT: Because this client is vomiting, changing the medication to the oral route would not be effective. The medication takes longer to work if given orally, which means the client may vomit again before the medication activates, losing part of the dose. 2. INCORRECT: Chemotherapy infusions should not be mixed with other categories of drugs, such as an antiemetic, because of the possibility of drug interactions. Certain chemical mixtures could also cause precipitates to form in the tubing, which is dangerous to the child. 3. INCORRECT: The child is experiencing nausea and vomiting at this time. Waiting to give the antiemetic until after the chemotherapy is completed causes the child to suffer needlessly. The nurse should take action immediately to alleviate symptoms.

A 20 year old client has been admitted to the hospital with a diagnosis of preeclampsia. The charge nurse has only semiprivate rooms available. What roommate would be most appropriate for the new admission? 1. An adolescent primigravida with many visitors. 2. A 25 year old post induction for fetal demise. 3. A 35 year old awaiting discharge after a total abdominal hysterectomy (TAH). 4. A 30 year old post dilation and curettage (D&C) who enjoys knitting.

4. CORRECT: A client with preeclampsia will be experiencing stress and elevated blood pressure. There is a risk of seizures, and therefore a calm, relaxed environment would provide the most therapeutic setting for the client. The 30 year old client is ideal because knitting is a quiet activity. Additionally, a D&C is a relatively uncomplicated procedure and this client will most likely soon be discharged, leaving the preeclampsia client alone in that room. 1. INCORRECT: Although the client is close in age to the adolescent, the teenaged primigravida has many young visitors which would create noise or confusion in the environment. Since this client is suffering from preeclampsia, a quiet environment is necessary to prevent other complications such as seizures. This adolescent would not be the best roommate. 2. INCORRECT: The client is admitted with a diagnosis of preeclampsia, which means elevated blood pressure, edema and the possibility of seizures. A quiet calm environment would be crucial for this client. The 25 year old client is close in age; however, that client has experienced a fetal demise and delivery of that fetus. There will most likely be grieving, multiple family members, and tension in that environment which would not be helpful to the client with preeclampsia. 3. INCORRECT: Though there is a large age difference, that issue does not impact whether this client would be an appropriate roommate. A client with preeclampsia needs a restful, calm environment to prevent further complications. Depending on the reason for the total abdominal hysterectomy (TAH), this client may require special teaching, referrals for further care and treatment, or emotional support for an unexpected diagnosis. The charge nurse knows this may be too hectic of an environment for the client with preeclampsia.

Which finding should take priority when the nurse is assessing the skin of a client diagnosed with diabetes? 1. Vitiligo of the chest. 2. Scleroderma to scapula and posterior neck region. 3. Redness of face and upper chest. 4. Small abrasion on great toe.

4. Correct: Skin breakdown on the foot is priority. Healing is likely to be impaired and the abrasion can be an entry point for microorganisms. There maybe other risk factors observed in the assessment; however, this finding should receive priority. 1. Incorrect: Vitiligo is a skin problem commonly associated with type I diabetes. The melanin containing cells are destroyed, resulting in patches of discolored skin. Vitiligo poses no harm to the client. 2. Incorrect: Scleroderma affects people with type 2 diabetes causing thickening of the skin to the upper back and neck. Scleroderma poses no harm to the client. 3. Incorrect: Redness should be noted and reasons found; however, this is not the priority finding. Redness/flushing can be due to many conditions but poses no obvious harm to the client.

During morning report, the nurse learns that a client's call bell is not working and maintenance cannot do repairs until tomorrow. The nurse is aware that the safest temporary method for the client to signal staff is what? 1. Provide a hand-held bell for client to ring. 2. Ask family to stay with client to alert staff. 3. Tell client to call out loudly to the staff. 4. Have staff visit client's room every 15 minutes.

4. CORRECT: It is vital for clients to be able to contact or alert staff for needs and concerns. The safest method is for the staff to check on the client at specified intervals. This will help alleviate client concerns about being able to signal the staff while ensuring that someone actually observes the client. 1. INCORRECT: While a hand-held bell could be an option, it is not reliable. The client could easily push it onto the floor, or it could become tangled in the linens. Additionally, depending on the noise level of the unit, a hand bell could either disturb other clients or not be heard by staff. 2. INCORRECT: It is not the responsibility of the family to sit with the client 24/7 just because the hospital has non-working equipment. Not only would this be an imposition, it violates most visiting policies and places the burden on the family. 3. INCORRECT: Having a client call out to staff is both inefficient and unsafe. Assuming the client's voice is even loud enough to be heard, it is unlikely that the verbalizations of one client could be distinguished from others that may call out because of dementia or normal nighttime utterances. This is not safe.

The pediatric nurse is assessing a child following an appendectomy. What is the nurse's main priority following surgery? 1. Obtain vital signs every four hours. 2. Assess the need for pain medication. 3. Tally intake and output every eight hours. 4. Auscultate lung sounds every four hours.

4. CORRECT: No matter what type of surgery, recall that the effects of anesthesia and possibly intubation can lead to complications, particularly in children. The potential for atelectasis and pneumonia follows surgery unless the client is encouraged to cough and deep breathe. Auscultating lung sounds frequently post-op is crucial. 1. INCORRECT: Although vital signs are important, initially the nurse should check vitals every half hour to one hour. Despite the frequency, another assessment is even more important. 2. INCORRECT: It is crucial to medicate a post-operative client; however, pain medications should never be administered until after the initial assessment as pain medication will alter important symptoms the nurse needs to determine any complications. 3. INCORRECT: Standard intake and output is tallied once a shift, or every eight hours. Though this information is vital to determine hydration and function of the kidneys, it is not the nurse's main priority.

A toddler with a history of chronic otitis media has been admitted for a bilateral myringotomy. The nurse is discussing how to prevent future ear infections with the parents. The nurse knows that what comment by the parents is the most likely contributing factor to the child's frequent otitis media? 1. "The baby still takes a bottle of milk at bedtime." 2. "We put cotton in the baby's ears at bath time." 3. "We had to change the formula to soy milk." 4. "We smoke outside when the baby is awake."

4. CORRECT: Research from the Centers for Disease Control and Prevention (CDC) has shown that living in a smoke-filled environment leads to frequent otitis media in both children and adults. This statement by the parents indicates that smoking is done outside only when the child is awake. Smoking inside, even occasionally, causes smoke to build up in fabrics and on surfaces, exposing the child all day to secondhand smoke. This will greatly contribute to chronic otitis media. 1. INCORRECT: Children who go to sleep while sucking on a bottle have an increased risk for several problems, including otitis media and aspiration. The fluid from the bottle can seep into the middle or inner ear via the Eustachian tube, increasing the risk of bacteria to the middle ear. However, the parents did not actually say the child was drinking while lying down in the crib. This issue can be addressed later. 2. INCORRECT: Water entering or becoming trapped in the middle ear can definitely contribute to chronic otitis media. Pediatricians often recommend that parents use ear plugs, which are safer than cotton. This statement shows that teaching is needed but this is not the main factor leading to recurrent ear infections. 3. INCORRECT: Many children develop an allergy to milk, requiring a change to soy milk. But the symptoms of a food allergy are usually gastric in nature, such as abdominal pain, gas, and bloating. A milk allergy is unlikely to contribute to cause ear infections.

A client scheduled for an amniocentesis expresses concerns about the procedure to the nurse, despite having signed the consent form. What statement by the nurse would be most appropriate for the client? You answered this question Incorrectly 1. "Don't worry, it's a very simple procedure." 2. "You have already signed the consent form." 3. "I will tell the doctor you need to talk more." 4. "Can you tell me what most concerns you?"

4. CORRECT: The client is obviously having second thoughts and needs further clarification or discussion. Even though a consent form was signed, the client has the legal right to withdraw that consent at any time. This open-ended question by the nurse is an appropriate approach to encourage the client to express concerns, allowing the nurse to gather further information and formulate a suitable plan to proceed. 1. INCORRECT: A nurse should not use the words "don't worry" to a client. Doing so dismisses both the client's feelings and the right to request further information. This is not a "very simple procedure", and has definite, serious complications. This comment by the nurse does not employ any appropriate therapeutic communication techniques. 2. INCORRECT: This comment by the nurse is not true, since a client can withdraw consent for a procedure at any time, including just before the actual procedure. The client is expressing concerns about the amniocentesis now. "Patient Rights" always assure clients the ability to question any and all proposed treatments at any time. 3. INCORRECT: Since the client needs more information, it would be important to notify the primary healthcare provider. However, it is not appropriate on the NCLEX to transfer care of the client to someone else initially. The client is anxious and worried; therefore, the nurse should use therapeutic communication techniques to encourage the client to talk.

Following a home birth, the mother brings the 2 day old baby to the emergency room for an evaluation. The mother is concerned about an edematous, dark blue discoloration on the right side of the infant's head that does not cross suture lines. What statement by the nurse to the mother is most appropriate? 1. "We will have to do an immediate CAT scan." 2. "No need to worry, this is perfectly normal." 3. "Did you have a really long or difficult labor?" 4. "This common occurrence will resolve in 6 weeks."

4. CORRECT: The infant has a cephalohematoma which is hemorrhaging between the skull and the periosteum, usually secondary to a prolonged second stage of labor. This occurrence is not unusual and generally resolves within 2 to 6 weeks. 1. INCORRECT: The fact that the discoloration does not cross the suture line indicates this is a non-lethal cephalohematoma, with slight hemorrhaging under the skull. A CAT scan would not be necessary. 2. INCORRECT: Denying the client's feelings shows poor communication skills by the nurse. Despite indicating the discoloration is normal, the nurse needs to share further details and educate the mother, which would help to alleviate fear and concern. 3. INCORRECT: While this question may be considered part of the assessment phase for the nurse, it does not address the mother's concern about the injury on the infant's head. Option 1: Definitely not. The nurse observes that this is a non-critical cephalohematoma, which is hemorrhaging between the skull and periosteum. While the primary healthcare provider may want to monitor this situation until it resolves, there is no need for a CAT scan. Option 2: No! Although this is an accurate statement, this comment by the nurse dismisses the mother's concerns and does not provide appropriate education about this cephalohematoma. Whether this was a planned or unexpected home birth, the mother needs instructions and reassurance, including when the infant's skull will return to normal. Option 3: Close but not quite. This is a very good question, since a cephalohematoma often occurs because of a prolonged second stage of labor. The nurse may gather further information that may be of benefit to the mother in the future if another home birth is planned. However, despite being a good question, this does not address the mother's concerns about the appearance of infant's skull. Option 4: Outstanding! The nurse has stated two basic facts to initiate therapeutic communication with the mother. The infant has a cephalohematoma, which is a hemorrhaging between the skull and periosteum from a difficult or prolonged labor. This swelling will generally resolve within several weeks to several months without the need for medical intervention. The only potential problem may be a slight jaundice in the child's appearance over that time, for which the primary healthcare provider may check their bilirubin level.

A preschool child has been rushed to the emergency room after ingesting an undetermined amount of chewable baby aspirin. The nurse immediately initiates what priority action? 1. Inject subcutaneous dose of vitamin K. 2. Induce vomiting with ipecac. 3. Initiate large bore IV line. 4. Insert a nasogastric tube.

4. CORRECT: The most urgent need in an overdose situation is to neutralize or inactivate the drug and/or poison. Activated charcoal is the treatment of choice for aspirin. In a child this young, the only way to instill the charcoal is via NG tube, which will also decrease the chance of aspiration. 1. INCORRECT: Vitamin K is used as an antidote to reverse the effects of excessive warfarin. Although Vitamin K is used to decrease bleeding, it is not an appropriate or effective intervention for baby aspirin. 2. INCORRECT: Inducing vomiting with syrup of ipecac is no longer considered an acceptable intervention for poisoning. Inducing vomiting increases the chance for aspiration and electrolyte imbalances. 3. INCORRECT: Although an IV site will be important for fluid resuscitation, initiating an intravenous line is not the nurse's first priority. Option 1: No indeed. While it is true that vitamin K can be used in a bleeding situation, it is not appropriate for baby aspirin. An injection of vitamin K is administered sub-Q for an excessively high pro-time/INR in clients taking warfarin. This antidote is not effective with aspirin. Option 2: Definitely not. Historically, poison control centers were told to recommend syrup of ipecac to induce vomiting when children ingested inappropriate substances or medications. The use of this syrup is no longer advised due to the chance of aspiration. Also, some parents were using ipecac inappropriately, such as when a caustic product was ingested. Vomiting should never be induced in that situation. Option 3: Close but not quite. This client will definitely need a large bore intravenous line for volume resuscitation. Knowing that aspirin absorbs in an acidic environment, this child will be given fluids that include sodium bicarbonate to induce alkaline diuresis. In addition, the child may need replacement potassium because of the excessive diuresis. But an IV is not the first action. The nurse must select an action that addresses the aspirin. Option 4: Excellent choice! The first goal of treatment is to rapidly inactivate or absorb the aspirin before there is a very serious, or fatal, outcome. However, a preschool child is not likely to cooperate in drinking liquid charcoal. Because time is of the essence, inserting an NG tube to instill the charcoal is the best approach.

The nurse is presenting a seminar to expectant teen parents regarding infant car seat safety. What statement from a teen parent indicates to the nurse that teaching was successful? 1. "It's okay to place the car seat up front as long as it faces backwards." 2. "The baby has to stay rear facing until at least 40 pounds or 40 inches." 3. "Regular seat belts can be used if the child does not like the booster seat." 4. "An infant must stay in the backseat, facing backward, till at least a year old."

4. CORRECT: The nurse is looking for a statement that indicates the teen parents understand the proper use of infant car seats. Although there are some variations from state to state, the National Safety Council advises that infants should be in a rear-facing car seat in the back seat of a vehicle until at least age one year. This comment indicates the parents understand the teaching clearly. 1. INCORRECT: An infant or child car seat can never be placed in the front seat at any time, regardless of what direction it may face. Further teaching is definitely indicated. 2. INCORRECT: A child of 40 pounds or forty inches is of pre-school age, usually around 3 to 4 years old. This is too old for a rear-facing car seat. The issue of height and weight is more useful when determining whether a child can safely move from a car seat to a booster seat. The parents did not understand the instruction. 3. INCORRECT: The choice of booster seat versus regular car seat belts is not based on whether the child likes, or is comfortable, in using either type of restraints. The most accepted guideline for child safety is that children under the age of 8 years old should be in either a child's car seat or booster seat. Further teaching is needed.

Two cognitively impaired siblings are clients in the same hospital room. During rounds, the nurse notes they have removed identification bracelets. Because of similar appearance, the nurse is unable to identify the correct client for blood work. The nurse is aware that the most reliable method for identifying these clients is what? 1. Draw blood to type and crossmatch and compare with chart. 2. Call the primary healthcare provider to identify each client. 3. Ask nurses on the next shift to try to identify the clients. 4. Notify family to come in and identify clients in person.

4. CORRECT: The only way to definitely identify a client with no identification bracelet is to have immediate family verify the client in person. When the family member arrives and verifies the client, the hospital must apply a new ID bracelet in the presence of the family for added security. 1. INCORRECT: Even typing and cross-matching to determine the blood type does not guarantee a correct identification. Additionally, both clients may have the same type blood since they are siblings. 2. INCORRECT: A primary healthcare provider would not necessarily be able to identify a specific client. Having hundreds of clients would make it more difficult to remember individuals correctly. It is unlikely that the healthcare provider could correctly indicate which client needed blood work. 3. INCORRECT: This is the least effective approach to learn identification. Certainly, nurses spend more time with clients than other healthcare individuals, but asking another nurse to make this type of identification is still extremely risky and unreliable.

The nurse discovers that a client was given the wrong medication. After verifying the client is stable, an incident report is completed. The nurse is aware that the proper disposition of the report is what? 1. Send a copy of the report to the primary healthcare provider. 2. Notify the State Board of Nursing about the incident report. 3. Document that a report was completed on the client's chart. 4. Give the report to the hospital's risk management team.

4. CORRECT: The purpose of an incident report is to describe and document a particular event, injury, medication error, or other occurrence that affects a client or staff member. This report is then sent directly to the hospitals risk management team for the express purpose of developing a plan or protocols to prevent a repeat occurrence. 1. INCORRECT: Although the primary healthcare provider will need to be informed of the medication error and the client's current status, including vitals, a copy of the incident report is never provided. 2. INCORRECT: The State Board of Nursing is rarely notified about medication errors or the existence of an incident report. 3. INCORRECT: The information documented on the main chart includes the client's current status and assessment specifics. It should also be documented that the primary healthcare provider was notified. However, there should not be any mention of the incident report on the client's chart.

The mother of a hospitalized 4 year old is upset because the child has become incontinent while in the hospital. The nurse explains that the best way to handle this regressive behavior is what? 1. Remind the child to ring the call bell to use the bathroom. 2. Withhold privileges until the behavior ceases. 3. Use nighttime diapers so the child can stay asleep. 4. Realize regressive behavior is a normal stress response.

4. CORRECT: The stress of hospitalization can be overwhelming for both the child and family. Fear, pain, disrupted routines and separation anxiety can combine together to trigger regressive behavior in children. Becoming incontinent while hospitalized, especially at night, is not unusual for a preschool child. 1. INCORRECT: While a four year old is capable of understanding simple commands, such as ringing the call bell, the stress of hospitalization combined with possible restriction of independence can interfere with the child's cooperative behavior. 2. INCORRECT: Punitive measures are inappropriate for an action that is unintentional and most likely embarrassing to the child. Regressive behaviors are not purposeful, but rather are a response to the stress of hospitalization. 3. INCORRECT: A four-year-old has most likely been bathroom trained for awhile. Returning to wearing diapers, or even 'pull-ups', at night would be embarrassing and even verge on punitive behavior. Despite wanting the child to have a restful night's sleep, it is more appropriate to assist the child to the bathroom at night than to resort to diapers.

The nurse has received morning report on several new pediatric clients. What client should the nurse assess first? 1. A child with exacerbation of cystic fibrosis and PaO2 of 93%. 2. A child with rheumatic fever and an oral temperature of 100°F (37.7°C). 3. A child with a sickle cell crisis reporting pain of 4 out of 10. 4. A child with closed head injury and a urine specific gravity of 1.004.

4. CORRECT: This client is definitely the nurse's priority. The client's specific gravity is much lower than normal values of 1.010 to 1.030. Because this client has a closed head injury, the nurse suspects the child may be developing diabetes insipidus, placing the child at risk for hypovolemic shock. This client is the nurse's priority assessment. 1. INCORRECT: Although this child has an exacerbation of cystic fibrosis and will need a respiratory assessment, the current PaO2 of 93% is within normal limits. This client is not the nurse's priority. 2. INCORRECT: A client with rheumatic fever can be expected to have a fever in addition to other symptoms. An oral temperature of 100°F (37.7°C) is not particularly high, so the nurse can see this child later. 3. INCORRECT: An exacerbation of sickle cell anemia is indeed painful, and the child is reporting a pain level of 4 on a 1 to 10 scale. While pain evaluation and medication will be needed, this client is not the nurse's first priority.

An unlicensed assistive personnel (UAP) is asked to transfer a client with left hemiplegia from the bed to a wheelchair. The nurse tells the UAP the safest approach for this transfer is what method? 1. Lift client from edge of bed, supporting under arms and pivot to chair. 2. Utilize a slide board to transfer client from bed to the wheelchair. 3. Apply an ambulation belt around client's waist and pull into the chair. 4. Use a mechanical lift to move client from the bed into the wheelchair.

4. CORRECT: When transferring a large or physically impaired client out of bed to either a litter or wheelchair, safety for both staff and client is most important. The UAP should use a mechanical lift, first rolling the client onto the sling, attaching the lift loops, and allowing the machine to do the work of lifting the client. This provides a safe, gentle lift for the client and protects the UAP from injury. 1. INCORRECT: This is not appropriate for a client with hemiplegia. Because the client has no sensation or control over the left side, managing that extra dead weight will be placed on the UAP. Trying to lift the client under the arms and pivot into the chair is extremely risky, putting both the client and UAP in danger of being injured. 2. INCORRECT: A slide board is utilized when a client is lying flat and needs transferred between two flat surfaces, such as bed to litter. A slide board requires several people to utilize safely, and is not appropriate from bed to chair. 3. INCORRECT: An ambulation belt is a wide, flexible belt of heavy cotton webbing that is placed around a client's waist to assist when moving from a standing position, whether walking or sitting in chair. In order to correctly utilize this belt, the client needs to be able to stand. This client has hemiplegia and would not be able to stand independently.

A client has been admitted for evaluation of severe anxiety and new onset panic attacks following the loss of a spouse. Which client factor would the nurse consider most important in developing a plan of care? 1. Available support system 2. Perception of the situation 3. Desire to return to work 4. Coping mechanisms

4. Correct. The plan of care for a client in crisis involves a complex combination of factors to achieve a positive outcome. However, the most important consideration is the client's own coping skills. Treatment and subsequent recovery is more successful when the client has the coping skills and is able to participate in the recovery process. 1. Incorrect. Although a good support system is crucial during any psychiatric or emotional crisis, this is not the most important aspect of a client's plan of care. The priority is the client. Available support systems is not the priority when developing a plan of care. 2. Incorrect. The client's own perception of the problem can enhance or detract from a successful outcome; however, there is another facet that is more critical to a client's positive outcome. The client's coping mechanisms can affect their perception of the situation. 3. Incorrect. Having a goal, such as returning to employment, is important to the client's recovery, but by itself is not enough to ensure a positive outcome for a client. Returning to work is not the priority with new onset panic attacks.

The nurse is providing care to a client who has returned to the long-term facility following cataract surgery. Which finding would indicate a possible complication? 1. Slightly swollen eyelid 2. Slight discomfort of the eye 3. "Bloodshot" appearance of the eye 4. Extreme pain in the eye

4. Correct. The postoperative cataract client usually experiences little to no pain, and it can be managed with mild analgesics. If the pain is severe, there may be an increase in intraocular pressure, hemorrhage, or infection, and the surgeon should be notified. 1. Incorrect. Slight swelling of the eyelid is considered a normal finding following cataract surgery. 2. Incorrect. The postoperative cataract client usually experiences little to no discomfort following surgery. This is a normal finding. 3. Incorrect. Slight redness is an expected finding. Pay attention to the word "slight". Increased redness is cause for concern. Compare it to the non-operative eye.

The nurse is caring for a client who has been intubated and placed on a ventilator. The nurse hears the ventilator alarm and enters the client's room to find the high pressure alarm sounding. The client is very agitated with a respiratory rate of 40; arterial line BP 98/44; oxygen saturation 82%; cardiac monitor sinus tachycardia at 138. What action should the nurse take first? 1. Turn off alarm, then check ventilator settings. 2. Increase FiO2 setting to 100%. 3. Hyperventilate client, then suction ET tube. 4. Auscultate lung sounds.

4. Correct. When an alarm sounds, the first action by the nurse should be to assess the client. In this situation, assessment of lung sounds, chest movement, and respiratory effort should indicate which respiratory complication the client may be experiencing. Depending on the assessment findings, the other actions may be necessary. Excessive positive pressure can result in lung complications, including a pneumothorax. This could quickly progress to a tension pneumothorax. Therefore, the nurse should consider any sudden changes in oxygen saturations and signs of respiratory distress as life threatening. Immediate assessment of the client is warranted with actions taken based on the findings. 1. Incorrect. Depending on the assessment findings, this action may be necessary. Check the ventilator after checking the client. The ventilator is checked following the client assessment to assure that it is working properly and that the settings are appropriate. 2. Incorrect. Depending on the assessment findings, this action may be necessary. The goal of mechanical ventilation is optimal gas exchange by maintaining oxygen delivery and alveolar ventilation. The lowered oxygen saturation could be the result of the underlying illness, but since there was an abrupt change, mechanical factors should be considered as well. The nurse would need to institute other measures to promote gas exchange in addition to consideration of increasing the fraction of inspired oxygen. 3. Incorrect. Depending on the assessment findings, this action may be necessary. Ventilation use increases the production of secretions, regardless of the initial reason for ventilating support. The client must be assessed first for the presence of secretions by auscultating the lungs bilaterally. If excessive secretions are present, suctioning should be performed with caution to prevent damage to the airway mucosa.

The nurse makes selections from the hospital menu for a client who is confused and suspicious of others. Which menu choice is best? 1. Ham and vegetable casserole 2. Cheese and crackers 3. Caffeine free tea 4. Packaged sugar free Jell-O

4. Correct: A client who is suspicious of others needs foods that are packaged and can see them opened. 1. Incorrect: A client who is suspicious of others needs to be able to identify the ingredients in the food that is being eaten. A casserole contains many ingredients and the client may fear that something has been added to the food. 2. Incorrect: Finger foods are best for clients that are manic. 3. Incorrect: Drinks and foods with no caffeine are okay for the confused and suspicious client but this menu choice is not the best choice from the list here.

How would a case manager best describe a clinical pathway to nursing students? You answered this question Correctly 1. A decision-making flowchart that uses the if/then method to address client responses to treatment. 2. A set of practice guidelines developed by a professional medical organization such as the American College of Surgeons. 3. A standardized set of preprinted primary healthcare provider prescriptions for client care, which expedite the prescription process and can be customized to individual clients. 4. A set of client care guidelines based on a specific client diagnosis, which provides an overview of the multidisciplinary plan of care.

4. Correct: A clinical pathway is a set of multi-disciplinary client care guidelines for a specific diagnosis or condition. It can be used to guide the plan of care and to identify deviations from the plan of care. These clinical pathways reduce the degree of variation in clinical practice, improves outcomes, and promote organized and effective client care based on evidenced based practice. Clinical pathways are different from algorithms, practice guidelines, and protocols because they incorporate a multidisciplinary team approach and focus on coordination and quality of care. 1. Incorrect: A decision-making flowchart that uses the if/then method is the definition of an algorithm. The algorithm direction changes based on the information gained at each level of the algorithm, so decisions for actions will be different. 2. Incorrect: A set of practice guidelines developed by professional medical organizations is the definition of a practice guideline. These guideline assist in decision making about appropriate healthcare for specific clinical situations but are not fixed protocols that are designed to be followed in an exact manner. They are recommendations for consideration. The practice guidelines are specific to practice areas rather than having a multidisciplinary approach. 3. Incorrect: A standardized set of preprinted primary healthcare provider prescriptions. These preprinted prescriptions are available for immediate access and use with clients, include commonly prescribed interventions, and reduces oversight of interventions by having a standardized format. Other advantages have also been identified for the use of preprinted prescriptions.

Which nursing intervention should the nurse include when caring for a client with Alzheimer's disease being admitted to a long term care facility? 1. Offer multiple environmental stimuli at the same time to provide distraction. 2. Encourage the client to participate in activities such as board games. 3. Restrain the client in a chair to prevent falls when sundowning occurs. 4. Involve the client in supervised walking as a routine.

4. Correct: A regular routine and physical activity help client's with Alzheimer's disease maintain abilities for a longer period of time. Physical activities promote strength, agility and balance. The client's walking should be supervised for client safety issues. 1. Incorrect: Environmental stimuli should be limited with clients with Alzheimer's Disease. The client can become agitated and/or more disoriented with an increase in environmental stimuli. 2. Incorrect: Board games would not be appropriate due to the client's cognitive and memory impairment. Board games require complex cognitive actions. 3. Incorrect: Restraints should be avoided because they increase agitation. The client may become agitated by the restriction of he restraints. Also the client may perceive the restraints as a threat. Option 1 is false. A calm, predictable environment helps people with dementia interpret their surroundings and activities. Over reaction can occur to excessive stimulation. Option 2 is false. People with dementia are encouraged to participate in simple activities such as walking and caring for plants. Option 3 is false. Wandering behavior can often be reduced by gentle persuasions, or distraction. Restraints should be avoided because they increase agitation. Option 4 is true. Physical activity is demonstrated to slow some of the cognitive decline of Alzheimer's Disease. A regular routine helps minimize confusion and disorientation.

A postpartum client who is 2 hours post vaginal delivery remains on a oxytocin infusion for bleeding. Upon examination, the nurse determines that the client's fundus is boggy and soft. What is the priority nursing intervention? 1. Ambulate in the room 2. Perform crede' exercises 3. Reassess the fundus in 30 minutes. 4. Massage the fundus.

4. Correct: If the fundus is boggy and soft, massaging the fundus until firm will increase uterine tone and decrease bleeding. This is the only option that will fix the problem. 1. Incorrect: Ambulation will not fix a boggy fundus and would not be safe. 2. Incorrect: Crede' exercises are for bladder tone. Although urinary retention will prevent uterine contraction, the appropriate nursing intervention in the case of a full bladder is to have the client empty her bladder or to catheterize her if she is unable to void. 3. Incorrect: Postponing care could make the bleeding worse. This is delaying care.

Which client should the labor nurse see first? 1. Primigravida on IV magnesium sulfate with deep tendon reflexes of 2+ 2. Multigravida on po terbutaline with a pulse rate of 110/per minute 3. Primigravida on IV oxytocin with contractions every 3-4 minutes 4. Multigravida on po methyldopa with a blood pressure of 142/90.

4. Correct: A systolic blood pressure of ≥ 140 mmHg or a diastolic BP of ≥ 90 mmHg indicates hypertension. This client is already on methyldopa, which is an antihypertensive medication. Her hypertension is worsening and may compromise fetal well being. 1. Incorrect: + DTRs are normal. Clinical signs of safe dosage of magnesium sulfate include normal deep tendon reflexes. Adverse effects include depressed reflexes. 2. Incorrect: Maternal tachycardia (up to 120 bpm) is expected when on this medication. Terbutaline is a beta adrenergic agonist could have significant cardiovascular effects. 3. Incorrect: The desired contraction pattern with oxytocin is 3 in 10 minutes. A contraction every 3-4 minutes would equal 3 contractions in 10 minutes. The dosage of the oxytocin is individualized until the desired contraction rate is achieved.

Following a hemorrhagic stroke, a client had a craniotomy with insertion of a ventriculostomy. Upon arrival in the ICU, the nurse's initial readings indicate an increase in intracranial pressure (ICP). What is the nurse's priority action? 1. Position client on the right side. 2. Call the primary healthcare provider. 3. Lower the head of the bed immediately. 4. Hyperventilate client with a bag valve mask.

4. Correct: A ventriculostomy is a temporary drain placed in the brain to remove excess cerebral spinal fluid in order to decrease intracranial pressure. Because the client's ICP readings are increasing, the nurse's initial action is to try to reduce that pressure by hyperventilating the client with a bag valve mask, also called an Ambu bag or manual resuscitator. This lowers cerebral CO2 levels, causing vasoconstriction which temporarily decreasing blood flow and reducing pressure within the brain. 1. Incorrect: Clients experiencing increased intracranial pressure must be positioned in a neutral position, head midline and slightly elevated, generally with sandbags or immobilizers on either side of the skull to maintain that position. This allows for optimal drainage of cerebral spinal fluid (CSF) from the ventriculostomy. 2. Incorrect: The primary healthcare provider or surgeon will indeed need to be notified. However, the nurse's initial action is always focused on stabilizing the client if possible. In this case, the nurse can intervene prior to calling the primary healthcare provider. 3. Incorrect: Lowering the head of the bed is contraindicated following brain surgery since it increases blood flow to the brain, thus increasing intracranial pressure. The ideal position is head slightly elevated at 30 to 45 degrees with head immobilized in the midline position to improve drainage of CSF.

Which statement by a client would indicate to the nurse that education about alendronate has been successful? 1. "It is recommended that I recline for 15 minutes after taking my medication." 2. "Food should be eaten immediately after taking alendronate." 3. "My medication tablet should be chewed for rapid absorption." 4. "I should drink a full 8 ounce glass of water with my medication."

4. Correct: Alendronate is a biophosphonate drug used in the treatment of osteoporosis and other bone diseases. The client should take each tablet in the morning with a full glass of water (6-8 ounces or 180-240 ml) at least 30 to 60 minutes before the first food, beverage or medication of the day, to increase absorption. 1. Incorrect: After taking alendronate, the client should remain upright (sitting or standing) for 30-60 minutes. The client should not lie down until after eating. These actions help to decrease the likelihood of esophageal and GI associated side effects. 2. Incorrect: The client should wait at least 30-60 minutes before eating, drinking or taking any other medication, to increase absorption. 3. Incorrect: The client should not chew the medication tablet, mouth ulcers can occur when alendronate is chewed or dissolved in the mouth.

Multiple clients have arrived at the emergency room following an earthquake in which a building collapsed. Which client should the nurse designate as priority one? 1. A client with a partial amputation of the right lower leg 2. A client with neck pain and clear fluid dripping from nose 3. A client with decreased sensation to both lower extremities 4. A client with asymmetrical chest movement and tracheal deviation

4. Correct: Although all the clients are critical, this individual is at greatest risk of dying first. Asymmetrical chest movement and tracheal deviation are indications of a tension pneumothorax, requiring immediate placement of a chest tube to prevent death. The basic ABC's indicate that airway then breathing are the priority concerns of life. 1. Incorrect: A partial amputation would be very serious and involve excessive blood loss. The focus would be controlling bleeding and preservation of the limb. However, when considering prioritization, the ABC's indicate breathing comes before bleeding. This client is not the first priority. 2. Incorrect: This client's assessment indicates a probable basilar skull fracture; therefore, the chief concern would be to monitor for increased intracranial pressure. An intracranial pressure monitor (ventriculostomy) may need to be surgically inserted, and the client will need to be instructed not to blow the nose. Despite the life threatening issues here, this client is not the priority focus at the moment. 3. Incorrect: These assessment findings suggest a possible spinal cord injury with compression of the cord and will require either x-rays or an MRI to confirm. Although at risk of developing spinal shock, this client is not the most critical at this time.

An alcoholic client was admitted to the medical unit with substance-withdrawal delirium. Two days later, the client decides to leave the hospital against medical advice. What is the priority nursing intervention at this time? 1. Hide the client's clothes so that he cannot leave. 2. Administer the ordered sedative. 3. Place restraints on the client. 4. Determine why the client wants to leave.

4. Correct: Always assess why the client wishes to leave first. This will provide an opportunity to attempt to fix the problem and possibly revise the client's decision. 1., 2. & 3. Incorrect: Confining a client against his or her wishes, except in an emergency situation, may be considered false imprisonment. Actions that may invoke these charges include: locking an individual in a room, taking a person's clothes for the purposes of detainment against his or her will, and retaining in mechanical restraints a competent voluntary client who demands to be released.

An LPN/VN is reinforcing instructions for the spouse of a home care client recently diagnosed with Alzheimer's Disease. The LPN/VN acknowledges that previous teaching was successful when the spouse makes what statement? 1. "Activities that provide stimulation will help to reorient my spouse". 2. "With medications and therapy, my spouse will begin to improve". 3. "Keeping the rooms dark and quiet will be calming for my spouse". 4. "As the disease progresses, I need to review safety issues at home".

4. Correct: Alzheimer's Disease is a progressive disorder characterized by declining mental abilities severe enough to interfere with activities of daily living. Clients demonstrate a deterioration in memory, language, judgement and behaviors. An LPNVN cannot initiate teaching but can re-evaluate that learning has been successful following the initial teaching by an RN. The client's spouse indicates understanding the importance of safety as the disease progresses, demonstrating to the LPN/VN the previous teaching was successful. 1. Incorrect: Stimulating visual or auditory activities generally increase confusion and agitation in clients with Alzheimer's Disease because they cannot process the input fast enough. This statement by the spouse would indicate to the LPN/VN that initial teaching was not successful. 2. Incorrect: Alzheimer's Disease is progressive until the ultimate outcome of death. Although medications have been developed that may slow the progress of the disease, Alzheimer's is not reversible. This response by the spouse indicates that prior teaching was not effective, and further instruction is needed. 3. Incorrect: While a stimulating environment is not beneficial to the Alzheimer's client, a darkened environment can similarly create confusion and agitation. The effect, referred to as Sundowner's Syndrome, is characterized by an increase in hallucinations, confusion or agitation that occurs at night. Keeping rooms darkened would create the impression of nighttime, likely increasing client's anxiety or agitation. This statement by the spouse would alert the LPN/VN that learning did not occur and that teaching needs reinforced.

The nurse asks if the client has an advance directive. The client responds by saying, "I have heard of advance directives, but I do not have one. What is an advance directive?" Which response by the nurse is appropriate? 1. Specifies your wishes regarding your personal effects and finances should you become unable to make decisions. 2. Specifies your wishes regarding healthcare and your finances should you become incapacitated. 3. Similar to a will, it specifies your wishes for burial should you die during hospitalization. 4. A form of a living will. It specifies your wishes regarding healthcare and treatment options should you become incapacitated.

4. Correct: An advance directive is a legal document prepared by a competent individual that specifies what treatments, if any, the client desires should the client become incapacitated or unable to make informed healthcare decisions in the future. The document includes wishes regarding resuscitation measures, withdrawing treatment and life support, and end-of-life care. 1. Incorrect: An advance directive does not address client personal effects and finances. These might be included in a last will and testament, but are not part of an advance directive. 2. Incorrect: An advance directive does not address client personal effects and finances. These might be included in a last will and testament, but are not part of an advance directive. 3. Incorrect: An advance directive does not address burial wishes.

The charge nurse is orienting a new nurse to the pediatric unit. Which teaching related to assessment is appropriate? 1. One assessment should be done daily on each client by the charge nurse. 2. An assessment should be done daily on each client at the beginning of the shift. 3. Assessments of clients should be updated as the nurse provides care to clients. 4. Assessments of clients should be done at the beginning of the shift and updated as nursing care is provided.

4. Correct: Assessment is ongoing; however, for each shift a baseline assessment should be done so the nurse can verify or make judgment regarding other findings throughout the 24 hour day. It is best to get the baseline as soon as possible once the shift begins, and update or reevaluate during the shift. This option actually incorporates the other 3 options making it the correct option. 1. Incorrect: Must include ongoing updated assessments, not just one assessment. These can be done by the assigned RN, not the charge nurse. 2. Incorrect: Must include ongoing updated assessments, not just at beginning of shift. 3. Incorrect: Must include initial beginning of shift assessment.

The nurse is teaching a community education class on alternative therapies. Which alternative therapy that uses substances found in nature should the nurse include? 1. Energy therapies. 2. Mind-body interventions. 3. Body-based methods. 4. Biologically-based therapies.

4. Correct: Biologically-based therapies use substances found in nature such as herbs, foods, and vitamins. 1. Incorrect: Energy therapies use energy fields. Substances found in nature are biologically-based therapies. 2. Incorrect: Mind-body interventions use the mind to help affect the function of the body. Substances found in nature are biologically-based therapies. 3. Incorrect: Body-based methods use movement of the body. Substances found in nature are biologically-based therapies.

The nurse is teaching a client with chronic obstructive pulmonary disease (COPD) about nutrition and maintaining body weight. Which instruction is most important for this client? 1. Do postural drainage just before meals. 2. Consume fluids only at meal times. 3. Prepare meals high in carbohydrates. 4. Plan rest periods before and after meals.

4. Correct: Both ingestion and digestion require a great deal of energy expenditure for clients. Resting prior to eating helps decrease dyspnea, allowing the client to complete an entire meal. Relaxing afterwards compensates for the increased blood flow sent to the gastrointestinal system during digestion, again minimizing respiratory effort. Frequent rest periods throughout the daily are vital for COPD clients. 1. Incorrect: Postural drainage techniques help COPD clients loosen and expel excessive mucus that builds up from inflammation within the lung tissue. Because of specific therapy positions, including those in which the client faces head down, there is the chance of wheezing or vomiting. This can lead to aspiration or infection. The best time to complete chest therapies is at least an hour before or two hours after a meal. 2. Incorrect: Fluids help to thin excessive mucous secretions typical in chronic obstructive pulmonary disease. Also, if the client is using oxygen, the mucous membranes will quickly dry out. COPD clients are encouraged to drink at least 64 ounces of caffeine free liquids throughout the day, rather than just at mealtime. 3. Incorrect: Because dyspnea interferes with eating, weight loss and malnutrition are areas of concern for clients with COPD. Small frequent meals high in protein are important to maintain nutrition and improve the immune system.

An infant has been prescribed Bryant's traction for a diagnosis of developmental dislocated hips (DDH). At what degree of hip flexion should the nurse maintain the infant's hip for proper traction alignment? 1. 15 2. 30 3. 45 4. 90

4. Correct: Bryant's traction is used for DDH. The child's body and the weights are used as tension to keep the end of the femur in the hip socket. Traction helps position the top of the femur into the hip socket correctly. This is accomplished with 90 degrees of hip flexion. 1. Incorrect: Fifteen degrees of flexion is not adequate to keep the femur end in the hip socket. 2. Incorrect: Thirty degrees of flexion is not adequate to keep the femur end in the hip socket. 3. Incorrect: Forty-five degrees of flexion is not adequate to keep the femur end in the hip socket.

A client with cirrhosis is being treated with bumetanide 1 mg daily for the management of ascites. What should the nurse assess for related to the effects of this medication? 1. Hyperbilirubinemia 2. Hypercalcemia 3. Hypoaldosteronism 4. Hypokalemia

4. Correct: Bumetanide is a K+ depleting diuretic. Potassium is lost primarily through the kidneys; therefore, when the urine output increases with the use of a diuretic, more potassium can be lost and the client is at risk for hypokalemia. 1. Incorrect: Hyperbilirubinemia may be present with cirrhosis but is not related to the effects of bumetanide. Don't let the presence of ascites convince you that this question is asking about a liver problem. 2. Incorrect: Although loop diuretics, such as bumetanide, can cause a slight increase in the excretion of calcium, which would lead to hypocalcemia, compensatory mechanisms generally are able to keep the calcium levels within normal range. 3. Incorrect: Bumetanide does not cause hypoaldosteronism. This is an aldosterone problem.

The nurse admits a child with a history of cystic fibrosis (CF) with vomiting for 3 days, headache, and unusual behavior. What does the nurse anticipate the lab values will show? 1. Hypernatremia 2. Hypercalcemia 3. Hypocalcemia 4. Hyponatremia

4. Correct: CF kids are always losing sodium! That's why mom will often say they taste "salty" when she kisses them and why the sweat chloride test is diagnostic. They are looking for sodium chloride when they do the test. So when they are sick their risk for becoming hyponatremic goes up even more because they are stressed. And we know the brain doesn't like for our Na to be messed up. Did you pick out these Key words: Vomiting x3 days, headache, and unusual behavior. 1. Incorrect: CF kids are always losing sodium! Also, when you vomit, you lose all electrolytes. 2. Incorrect: CF kids are always losing sodium! Calcium is not the problem here. 3. Incorrect: CF kids are always losing sodium! Calcium is not the problem here.

Which client should the nurse recognize as being at greatest risk for the development of cancer? 1. Smoker for 30 plus years 2. Body builder taking steroids and using tanning salons 3. Newborn with multiple birth defects 4. Older individual with acquired immunodeficiency syndrome

4. Correct: Cancer has a high incidence in the immune deficiency client and in the older adult with both of these risk factors together, this one is the highest risk for cancer. 1. Incorrect: Although smoking is a known environmental carcinogen, this one risk factor alone is not the highest risk. 2. Incorrect: These are known environmental carcinogens, but do not rank as highly as aging and immune deficiency. 3. Incorrect: Birth defects are not a risk factor for cancer.

The postanesthesia care unit has received several postoperative clients. While encouraging the clients to cough and deep breath, the nurse realizes that coughing poses the greatest risk to which client? 1. A female with an abdominal hysterectomy 2. A male who had a right upper lobectomy 3. An adolescent with an open appendectomy 4. An elderly client who had cataract removal

4. Correct: Cataract removal involves replacing the eye's lens with an artificial lens that is permanently sutured into place. Coughing would increase intraocular pressure in this client and risk dislodging the lens and eye sutures. The nurse needs to monitor and prevent additional potentially harmful actions such as sneezing, vomiting, bending over, or straining. 1. Incorrect: Coughing is a potential complication following any surgery in which there are external sutures. However, a priority post op action is to encourage the client to cough and deep breath to prevent possible atelectasis; therefore, the nurse must instruct this client to splint the lower abdomen prior to taking deep breaths and coughing. This client is not at the highest risk. 2. Incorrect: Any type of thoracic or cardiac surgery places the client at high risk for pneumonia or atelectasis. Deep breathing is particularly important for this client. The nurse will definitely need to encourage this client to cough and deep breath while assisting with splinting the surgical incision. This client would be at a greater risk by NOT coughing. 3. Incorrect: An open appendectomy indicates this was not done laparoscopically, but rather that this client will have abdominal sutures. Despite this fact, the client must be encouraged to cough and deep breathe to prevent complications. The nurse will instruct the client to splint the lower abdomen to protect the suture line while coughing and deep breathing.

A pregnant woman who has just been admitted to the labor and delivery room states that her "water just broke". What should the nurse do immediately? 1. Confirm that fluid is amniotic fluid with a pH test strip 2. Obtain maternal vital signs 3. Observe amniotic fluid color 4. Check fetal heart rate (FHR) pattern

4. Correct: Check the FHR immediately following the rupture of membranes. Changes in FHR pattern such as bradycardia or variable decelerations may indicate prolapsed umbilical cord. 1. Incorrect: The first thing the nurse should do is check FHR pattern. Changes in the FHR, such as bradycardia or variable decelerations could indicate prolapsed cord. Interruption of the blood flow through the cord interferes with oxygenation and is potentially fatal. 2. Incorrect: FHR is the priority as a change could indicate prolapsed cord. If the umbilical cord slips downward after the membranes rupture. it can become compressed which would be indicated by changes in the FHR, not the maternal vital signs. 3. Incorrect: The nurse should observe the amniotic fluid color after checking the FHR.

What should the chemotherapy infusion nurse recognize as the major barrier of chemotherapy success in treating cancer clients? 1. Inadequate knowledge of the side effects of chemotherapy 2. Difficulty obtaining an IV access 3. The development of alopecia 4. Toxicity to normal tissues

4. Correct: Chemotherapy is toxic to both cancerous and non-cancerous cells. Widespread destruction of non-concancerous "normal" cells can limit the use of chemotherapeutic agents. 1. Incorrect: Inadequate knowledge can be addressed and is not considered a major barrier for chemotherapy treatment. 2. Incorrect: Implantable ports are most often used for chemotherapy administration and eliminate the difficulty of obtaining a repeated peripheral IV site. 3. Incorrect: Alopecia is an adverse effect of chemotherapy but does not affect the success of chemotherapeutic agents.

The nurse is caring for a client diagnosed with schizophrenia who is admitted to the hospital for possible bowel obstruction. The client has a nasogastric tube (NG) and reports pain 8/10. What is the priority nursing action? You answered this question Correctly 1. Decrease the stimuli and observe frequently. 2. Administer the prn sedative. 3. Call the primary healthcare provider immediately. 4. Administer the prn pain medication.

4. Correct: Small bowel obstruction has a clinical manifestation of crampy pain that is wave like and colicky due to persistent peristalsis above and below the blockage. Nursing care of the patient includes pain management. 1. Incorrect: Decreasing the stimuli and observing are not appropriate. The pain needs to be assessed and treated. 2. Incorrect: Not necessary, it will sedate them, but not help the pain. Sedation is not a necessary intervention for pain. A medication to relieve pain is needed. 3. Incorrect: The nurse can administer pain medication as ordered. There is no need to contact the healthcare provider.

An unlicensed assistive personal (UAP) has been floated to the emergency department (ED) because of several staff call offs. Since the UAP has never worked in the ED, what is the most appropriate task the charge nurse could assign? 1. Clean and restock exam rooms after client discharge. 2. Follow another UAP who has worked there previously. 3. Sit at the reception desk and answer incoming calls. 4. Escort clients from the ED to other areas for tests.

4. Correct: Clients seen in the emergency room are often taken to other hospital departments for tests such as X-rays, Cat scans or MRI's. If ordered to another department for testing, such clients are generally stable and could therefore be transported by unlicensed assistive personnel. This is a task UAP's often do on other hospital floors and would be an appropriate assignment. 1. Incorrect: Having never worked the emergency department before, this UAP would not be aware of even basic exam room requirements, particularly involving specialized equipment. Because supplies must be readily available in critical situations, personnel familiar with those requirements and provisions needed for each room should complete restocking of the rooms. 2. Incorrect: A thorough orientation for this UAP would be ideal, especially if there is a chance of being floated to the emergency room again. However, doing so during a staffing crisis is neither appropriate nor efficient, since the UAP is being utilized out of a desperate need for adequate staffing. 3. Incorrect: The reception area personnel are the first staff that encounter incoming clients. This position usually requires some type of training with interviewing techniques or how to determine an acute situation requiring immediate triage. Even answering the phone would involve understanding the necessary prerequisites for that position. This is not an appropriate task for the UAP.

A client with a rare disorder has been admitted to a teaching hospital. The primary healthcare provider includes this client in medical students' morning rounds without notifying the client. When the angry client reports this to the charge nurse, what response by the nurse would be most appropriate? 1. "Consent is implied because this is a teaching hospital." 2. "These students will provide excellent care for you." 3. "I will call your primary healthcare provider to report how upset you are." 4. "You can refuse to be part of the students' study."

4. Correct: Clients' rights (still referred to in a hospital setting as the" Patient Bill of Rights") is a written code of ethical behavior describing the relationship that exists between the client and any facility to which they are admitted, including mental health units and hospice care. These guidelines provide the client a specified level of expectations regarding, for example, access to care, confidentiality and personal dignity. Regardless of the circumstances of the disease or location of treatment, clients have the right to refuse care from any professional personnel, including medical and nursing students. 1. Incorrect: Implied consent is an inferred agreement in which medical interventions are provided when the client cannot formally agree, as in the case of unconsciousness or incompetence. However, this client is clearly conscious and able to choose whether care by students is acceptable. The fact that the facility is a teaching hospital in no way deprives this client of the right to refuse student involvement. 2. Incorrect: The issue is the client's rights were violated when medical students were allowed involvement in this case without express consent or acknowledgement by the client. This response by the nurse ignores the client's rights or feelings by focusing on student abilities to provide care. It is demeaning to the client and does not address the client's concerns or provide alternatives. 3. Incorrect: Alerting the primary healthcare provider will be one component needed to resolve this situation. However, this initial response by the nurse is inappropriate for two reasons; first, this process transfers care of the client away from the nurse. Secondly, it does not provide the client with specific information about rights or resolutions.

A newly married wife tells the nurse, "I told my husband that I may not know how to cook, but I can sure do the dishes!" Which defense mechanism is the client displaying? 1. Projection 2. Displacement 3. Sublimation 4. Compensation

4. Correct: Compensation is consciously or unconsciously overemphasizing a characteristic to compensate for a real or imagined deficiency. Making up for deficits in one area by excelling in another helps to raise or maintain the client's self-esteem. 1. Incorrect: Projection is attributing one's own thoughts or impulses to another person as if they had originated in the other person. This is unconsiously done and usually includes intolerable wishes and emotional feelings. 2. Incorrect: Displacement is shifting or transferring the emotional element of a situation from a threatening object to a non-threatening object. This could include transferring emotions from a person, object or situation to another person, object or situation. 3. Incorrect: Sublimation is redirecting a socially unacceptable impulse into socially acceptable behavior. Examples include strong aggressive or sexual drives.

The nurse is caring for a 5 year old client who is 12 hours post tonsillectomy. The client is pain free and has advanced to a soft diet. What is the priority nursing intervention? 1. Apply warm compresses to the throat. 2. Encourage gargling to reduce discomfort. 3. Position the child supine. 4. Monitor for frequent clearing of the throat.

4. Correct: Continuous swallowing and frequent clearing of the throat are signs of bleeding. 1. Incorrect: This would increase blood flow, causing edema and bleeding, so this should not be done. 2. Incorrect: Gargling increases motion of throat and may cause bleeding. This is also something that could be a developmental challenge for a 5 year old. 3. Incorrect: The blood can drip down into the stomach and the client will wake up and vomit the old blood while lying flat. This puts the client at risk for aspiration so the nurse should place the client in a side lying position.

The nurse is caring for a client diagnosed with Obsessive Compulsive Disorder (OCD). Which statement, made by the client, would be the best indicator of improvement? 1. "My friends don't know I have OCD." 2. "I only do my hand washing to reward myself when I am good." 3. "I know my thoughts and behaviors aren't very normal." 4. "I have more control over my thoughts and behaviors."

4. Correct: Control is an issue for those with OCD. Appropriate goals for this client would be to control unwanted behaviors and thoughts. 1. Incorrect: Doesn't indicate control over behavior. Not telling their friends indicates the client is ashamed of disease and doesn't show improvement. 2. Incorrect: The behaviors are not reward for good behavior; they are utilized to decrease anxiety. Compulsions are ritualistic behavior that the individual feels driven to perform in an attempt to reduce anxiety. 3. Incorrect: Clients with OCD are aware their behavior is not normal, so this would not be an indicator of improvement. Obsessions are defined as thoughts, impulses or images that persist and recur, so that they cannot be dismissed from the mind.

The parents of a child hospitalized with cystic fibrosis have been given discharge instructions. The nurse knows that teaching has been successful when the parents make what statement? 1. "Our child will need to have a gluten free diet." 2. "The enzymes should be given at bedtime daily." 3. "Salt needs to be decreased in our child's diet." 4. "We need to prepare high calorie, high fat meals."

4. Correct: Cystic fibrosis is an inherited disorder in which abnormally viscous secretions affect the respiratory and digestive systems. Because the client is unable to absorb nutrients, several dietary adaptations are crucial, including frequent small meals along with digestive enzymes to help the client process food. The meals should be high calorie, high fat with increased amounts of sodium to help stabilize fluids. 1. Incorrect: A gluten free diet is not associated with cystic fibrosis. This special diet is generally required for clients with Celiac disease and certain food allergies, although clients with either of these diseases will need the addition of fat soluble vitamins A, D, E and K. This statement by the parents indicates the need for further teaching. 2. Incorrect: Pancreatic digestive enzymes, such as Creon or Pancreaze, must be given with every meal or snack in order to help the digestive system absorb nutrients properly. Because clients with cystic fibrosis need frequent small meals throughout the day, digestive enzymes must also be provided throughout the day with any food. 3. Incorrect: Clients with cystic fibrosis lose abnormally large amounts of salt in sweat, and the glands are unable to reabsorb needed sodium into the body system. Rapid dehydration is common due to decreased sodium levels, which are exacerbated during exercise or hot weather. These clients are encouraged to increase salt intake.

Which comment by the mother indicates understanding of the diet needed to maintain health and adequate nutrition in the toddler? 1. "It is important to give my child low fat milk after one year of age". 2. "If the child won't eat new foods after three tries, he is not going to eat it". 3. "I think that the sooner one starts to give vitamins to children, the better". 4. "I try to provide whole grains, fruits, vegetables, and meat daily".

4. Correct: Depending on their age, size, and activity level, toddlers need about 1,000-1,400 calories a day. A health promotion strategy to help meet the nutritional needs of the toddler includes offering a wide variety of healthy foods and from all food groups based on the "my plate" food guide. 1. Incorrect: Fat should not be limited in the child under two years of age. In general, kids ages 12 to 24 months old should drink whole milk to help provide the dietary fats they need for normal growth and brain development. 2. Incorrect: Learning to eat new foods is a process that requires many attempts. Keep offering the food. 3. Incorrect: If children eat a wide variety of foods, it is unlikely that vitamin supplementation will be needed.

The charge nurse is making assignments for the evening shift. Which client would be an appropriate assignment for a new LPN/VN graduate? 1. A middle aged adult admitted with syncope. 2. An adolescent with skin grafts to right hand. 3. A young adult receiving IV chemotherapy. 4. An elderly adult diagnosed with diverticulitis.

4. Correct: Diverticulitis is a bowel disorder of undetermined origin, characterized by pain to the lower abdomen, along with bloating, fever and diarrhea. Treatment may include hospitalization, antibiotics, liquid diet and bedrest. Because there is usually no bleeding involved, this would be an appropriate assignment for a new LPN/VN graduate. 1. Incorrect: A new admission requires assessments that must be completed by a registered nurse. Additionally, syncope could be an indication of a serious cardiac issue; therefore, this would not be a client suitable for the LPN/VN. 2. Incorrect: Although sterile wound care is within the scope of practice for an LPN/VN, skin grafts require special assessment techniques during dressing changes, in order to determine quality of tissue perfusion and potential for rejection. This client should be assigned to a more experienced nurse. 3. Incorrect: This client will need extensive assessments of multiple body systems while receiving I.V. chemotherapy, requiring an experienced registered nurse with knowledge of both chemo drugs and different types of cancer. This client would not be appropriate for a new LPN/VN.

Which risk factor should the nurse include when planning to educate a group of women about breast cancer? 1. Menopause before the age of 50 2. Drinking one glass of wine daily 3. Multiparity 4. Early menarche

4. Correct: Early menarche before age 12 is a known risk factor for breast cancer. The increased risk of breast cancer linked to a younger age at first period is likely due, at least in part, to the amount of estrogen a woman is exposed to in her life. A higher lifetime exposure to estrogen is linked to an increase in breast cancer risk. The earlier a woman starts having periods, the longer her breast tissue is exposed to estrogens released during the menstrual cycle and the greater her lifetime exposure to estrogen. 1. Incorrect: Studies show women who go through menopause after age 50 have increased risk of breast cancer. The risk for breast cancer increases as time period between menarche and menopause increases. 2. Incorrect: Small increase in risk with moderate alcohol consumption, not one glass of wine daily. Drinking low to moderate amounts of alcohol, however, may lower the risks of heart disease, high blood pressure and death. But, drinking more than one drink per day (for women) and more than two drinks per day (for men) has no health benefits and many serious health risks, including breast cancer. Alcohol can change the way a woman's body metabolizes estrogen (how estrogen works in the body). This can cause blood estrogen levels to rise. Estrogen levels are higher in women who drink alcohol than in non-drinkers. These higher estrogen levels may in turn, increase the risk of breast cancer. 3. Incorrect: Nulliparity (no pregnancies) is a known risk factor for breast cancer. Factors that increase the number of menstrual cycles also increase the risk of breast cancer, probably due to increased endogenous estrogen exposure. The test taker must be aware of risk factors that cause breast cancer. Factors that are associated with an increased risk of breast cancer include: Being female, increasing age, a personal history of breast cancer, a family history of breast cancer, inherited genes, such as BRCA1 and BRCA2, that increase cancer risk, radiation exposure to chest as a child or young adult, obesity, menarche at a younger age (before 12), beginning menopause at an older age, having first child at an older age (after 35), having never been pregnant, postmenopausal hormone therapy, drinking alcohol.

What is most important for the nurse to monitor when administering intravenous erythromycin to a client? 1. Nausea and vomiting. 2. Clotting studies. 3. Premature atrial contractions. 4. Prolonged QT interval.

4. Correct: Erythromycin is a macrolide antibiotic that is linked to QT prolongation. Pharmacologic agents capable of prolonging the QT interval are capable of causing ventricular tachyarrhythmias. 1. Incorrect: This is a side effect but not as life threatening as a prolonged QT. 2. Incorrect: This medication does not alter clotting factors unless there is liver dysfunction, which is a contraindication of the medication. 3. Incorrect: PACs are not a common problem with this medication but prolonged QT intervals are, and the associated ventricular tachyarrhythmias are more life-threatening.

A client who has chronic renal failure has been prescribed synthetic erythropoietin for the prevention of anemia. Which assessment finding should be reported to the primary healthcare provider? 1. Hemoglobin level of 10 g/dl (1.6 mmol/L) 2. Blood pressure of 120/84 3. Constipation 4. Swelling of feet and ankles

4. Correct: Erythropoietin is generally well tolerated. Swelling of feet and ankles may indicate the beginning of a cardiovascular problem. Clients taking this drug are at risk for myocardial infarctions and risk of blood clots. 1. Incorrect: The purpose of this drug is to increase hemoglobin levels. A level of 10g/dL (1.6 mmol/L) would be considered favorable even though still low. The client would still need the medication since anemia still exists. If hgb is above 12 g/dl (1.9 mmol/l), the level should be reported as the client does not need the med any longer. 2. Incorrect: An elevated blood pressure is one of the more common and major side effects. If elevated it should be reported, but this blood pressure is within normal limits. 3. Incorrect: Constipation may be caused by iron preparations. Increasing fiber in the diet may improve that symptom. A common side effect of synthetic erythropoietin is darrhea.

The primary healthcare provider has prescribed ampicillin and ciprofloxacin piggyback in the same hour, every 6 hours. How will the nurse administer these medications? 1. Administer one of the medications every 4 hours and the other every 6 hours. 2. Administer the medications by combining them into 150 mL of normal saline (NS). 3. Administer the medications at the same time by connecting the secondary tubing to two separate ports on the primary tubing. 4. Administer the medications separately, flushing with normal saline (NS) between medications.

4. Correct: Even though two IV piggyback medications have been ordered at the same time, they can both be infused separately on time. It just takes planning. The nurse must follow the medication rights (right client, right medication, right route, right dose, right time). The antibiotics need to be administered one at a time and normal saline is used to flush the remaining medication of the first antibiotic before the second is administered. 1. Incorrect: The primary healthcare provider will prescribe the dosing schedule. Its beyond the scope of practice for the nurse to independently the dosing schedule. 2. Incorrect: The properties of each antibiotic are different. The two different antibiotics cannot be mixed together. 3. Incorrect: Administering the antibiotic into different parts of the IV tubing is the same as mixing the IVs together. Only one antibiotic should be administered at a time.

A client diagnosed with Alzheimer's disease tells the nurse, "I haven't eaten all day. When am I going to eat?" The nurse noted that the client ate 100% of the provided lunch 45 minutes ago. What would be the best way for the nurse to respond? 1. "I'll ask the kitchen if they can send you up another lunch." 2. "What makes you think you didn't eat lunch?" 3. "You ate lunch less than 1 hour ago." 4. "Would you like me to get you some crackers and milk?"

4. Correct: The client believes that he/she has not eaten. Do not argue with the client. Offer the client something to eat. Fix the problem that the client believes he/she has. 1. Incorrect: The client wants to eat but another lunch is not needed since the client ate 100% of the provided lunch, just 45 minutes ago. A snack can be given. 2. Incorrect: Do not argue with the client. This will cause agitation and possible aggression. 3. Incorrect: Again do not argue with the client. The client does not believe he/she has eaten.

Which action by the nurse is most likely to result in a possible breach of confidentiality of medical records? 1. Entering the data on clients only at computers in nurse's station. 2. Recording the client history of an abortion. 3. Sharing access controls like passwords with other healthcare professionals. 4. Leaving the computer terminal before logging off.

4. Correct: Failing to log out may allow persons not concerned with the care of the client to access private information. 1. Incorrect: Computers for entering data, may be moved from room to room by the nurse. This is a much better use of time and ensures comprehensive accurate data. 2. Incorrect: The medical record routinely contains sensitive information, so this does not breach confidentiality. 3. Incorrect: This should not be done but not as likely to result in breach of confidentiality since this is restricted to other healthcare professionals.

A client is to be discharged following cataract removal with lens implantation. What statement by the client indicates to the nurse that teaching has been successful? 1. "I must keep both eyes covered till my check-up." 2. "I should only have pain for about two days." 3. "I will no longer have to wear reading glasses." 4. "My vision will be blurry for a couple weeks."

4. Correct: Following cataract removal, a new lens is sutured in place, which slightly alters the corneal curve. Newer surgical approaches involve the use of a "suture-less glue" but that method is less common. Although the client's vision will eventually improve and stabilize, minor blurring may exist during the 6 to 12 week healing period. After that time, any remaining visual issues can be corrected with glasses. 1. Incorrect: Only the operative eye is protected by an eye patch during the healing process. The primary healthcare provider will remove that covering at the first post-operative checkup. Covering both eyes would pose a greater safety risk and decrease the client's self care abilities. 2. Incorrect: Pain following cataract surgery is the sign of a serious complication and should be reported to the surgeon immediately. Clients may experience a small amount of serous drainage or scratchy sensation, but should not have pain. 3. Incorrect: Implantation of a new lens causes a mild astigmatism that will be permanent. The client may still need to use corrective lenses, even if just for reading. NCLEX questions about client teaching will focus on whether that teaching was successful, or whether the client will need further instruction. So be sure to carefully determine which direction the question is leading you. This question asks for a correct statement by the client that reflects a positive learning outcome. The easiest way is to look at each option and ask of the statement is correct. Option 1: Obviously incorrect, since covering both eyes would create a safety issue for the client. The first checkup is usually within a day or two of discharge and the primary healthcare provider will remove the single eye patch that covers the operative eye. Option 2: Always be suspicious when the presence of pain is not addressed prior to discharge. Drawing on your knowledge of cataracts and surgery, you will remember that this surgery is relatively painless, including during the recovery period. Some clients indicate a "scratchy sensation", likely due to the lens sutures, but not pain. This statement is not a reflection of successful teaching. Option 3: The phrase "no longer need" is similar to a 'never' response. Such a broad statement by the client does not indicate an understanding of the cataract removal process, in which an artificial lens replaces the client's own lens, thus changing the shape of the eyeball slightly. Some clients do need reading glasses, and some even need everyday glasses. This does not reflect an understanding of the teaching. Option 4: Excellent! The presence of sutures and a new lens does impact the client's vision for a period of time, and is an expected side effect of this surgery. Eventually the client's vision will stabilize, once the sutures are removed in several months, and the client will have brighter, clearer vision than before surgery, even with the use of corrective glasses.

What personal protective equipment should the nurse wear into the room of a client who has been placed on droplet precaution? 1. Gloves 2. Gown 3. Goggles 4. Mask

4. Correct: For a client on droplet precautions, a facemask is worn for close contact with the client. 1. Incorrect: The only requirement for droplet precautions is that a mask needs to be worn. However, if there is a reasonable expectation of contact with secretions, such as through contact or spraying, additional standard precautions would be needed, such as gown, gloves, and goggles. There is not indication in this stem about additional precautions needed. 2. Incorrect: The only requirement for droplet precautions is that a mask needs to be worn. However, if there is a reasonable expectation of contact with secretions, such as through contact or spraying, additional standard precautions would be needed, such as gown, gloves, and goggles. There is not indication in this stem about additional precautions needed. 3. Incorrect: The only requirement for droplet precautions is that a mask needs to be worn. However, if there is a reasonable expectation of contact with secretions, such as through contact or spraying, additional standard precautions would be needed, such as gown, gloves, and goggles. There is not indication in this stem about additional precautions needed.

Which nursing intervention is likely to be most helpful in providing adequate nutrition while the client is in the acute phase of mania? 1. Sit with the client during meals and encourage the client to eat all foods on the tray. 2. Assess the client's food preferences and provide only those foods for the client at meal time. 3. Allow the client to eat in the dining room with other clients. 4. Provide high-protein, high calorie snacks to the client between meals.

4. Correct: Having nutritious foods available between meals may help to increase the client's food intake. Nutritious intake is required on a regular basis to compensate for increased caloric requirements due to the hyperactivity during the manic phase. 1. Incorrect: The client in the acute phase of mania may have difficulty sitting still long enough to eat a meal. By offering finger foods or foods that can be eaten on the run, the client's food intake may increase. Also the word "all" is too limiting. 2. Incorrect: The client's diet should include the client's food preferences, but not just those foods. The word "only" is too limiting. 3. Incorrect: The client will be easily distracted when manic. Eating in the dining room with other clients may decrease the amount of food intake.

The nurse is caring for a client with hyperemesis gravidarum. What electrolyte imbalance is most likely? 1. Hypocalcemia 2. Hypomagnesemia 3. Hyponatremia 4. Hypokalemia

4. Correct: Hyperemesis gravidarum is characterized by persistent severe pregnancy related nausea and vomiting. There is a large amount of potassium in the upper GI tract. A client with prolonged vomiting will lose potassium in the emesis. Additionally, the client is unable to replace the lost potassium due to the persistent nausea and vomiting. 1. Incorrect: Hypocalcemia results from any condition that causes a decrease in the production of parathyroid hormone (PTH). Hyperemesis gravidarum does not affect PTH. 2. Incorrect: The lower GI tract has a lot of magnesium; this client is at risk for hypomagnesemia, but not more than hypokalemia. The client with hyperemesis gravidarum is losing upper GI contents. 3. Incorrect: The client with hyperemesis gravidarum is at high risk for being dehydrated. The electrolyte imbalance associated with dehydration is hypernatremia, not hyponatremia. Remember, this client's blood will be concentrated, and concentrated makes numbers go up i.e. sodium, hematocrit and specific gravity.

A client admitted with a diagnosis of end stage kidney disease (ESKD) has been prescribed a diet containing no more than 1 gram of phosphate per day. Which food item, if found on the client's meal tray should be removed by the nurse? 1. Skinless chicken breast 2. Green beans 3. Asparagus 4. Ice cream

4. Correct: Ice cream, a milk product is high in phosphate. 1. Incorrect: Fresh or frozen red meats without breading, marinades or sauce are better choices for a kidney diet. On average, fresh meat contains 65 mg of phosphorus per ounce and 7 grams of protein per ounce. A 3 ounce serving would have 150 mg of phosphorus. 2. Incorrect: Green peas, green beans and wax beans are low in phosphate, whereas black-eyed peas, lima beans, kidney beans, pinto beans, and lentils are high in phosphate. 3. Incorrect: A serving of asparagus has approximately 20 mg of phosphorus. As kidney function deteriorates, phosphate elimination by the kidneys is diminished and the client begins to develop hyperphosphatemia. By the time a client reaches ESKD, phosphate should be limited to approximately 1 gram/day. Foods that are high in phosphate include meat, dairy products (milk, ice cream, cheese, yogurt), and foods containing dairy products (pudding). Many foods that are high in phosphate are also high in protein. Since some clients on dialysis are encouraged to eat a diet containing protein, phosphate binders are essential to control phosphate. Fresh or frozen red meats without breading, marinades or sauce are better choices for a kidney diet. On average, fresh meat contains 65 mg of phosphorus per ounce and 7 grams of protein per ounce. Most fruits and vegetables are low in phosphorus, except for black-eyed peas, lima beans, kidney beans, pinto beans, and lentils.

A client with cervical cancer received an internal cervical radiation implant. What should be the initial nursing action if the radiation implant becomes dislodged and is found lying in the bed? 1. Call the client's primary healthcare provider. 2. Pick up the implant immediately with gloved hands and place it in double biohazard bags. 3. Notify the radiology department. 4. Utilize long-handled forceps to pick up the implant and dispose of it in a lead container.

4. Correct: If a client is receiving a radiation implant, a lead container and long-handled forceps should be placed in the client's room and kept for the duration of the therapy. If the implant becomes dislodged, the nurse should pick up the implant with long-handled forceps and place it in the lead container. 1. Incorrect: The placement of the implant into the lead container should be done initially. The primary healthcare provider may be notified but this is not the initial nursing action needed. 2. The implant should be picked up with long forceps for distance and reduction of contact. In addition, a biohazard bag is not sufficient for proper disposal of the radiation implant. 3. The initial action is to use long-handled forceps and dispose of the implant in a lead container. Calling the radiology department is delaying care and exposing individuals to the implant.

The edrophonium (Tensilon) test has been prescribed for a client. Which statement by the client would indicate to the nurse that the client understands this test? 1. "This medication will be given to me as an IM injection immediately after my muscles are tired." 2. "This test will determine if I have multiple sclerosis." 3. "The test is positive if my muscles do not get stronger after injection with this medication." 4. "I will be asked to perform a repetitive movement to test my muscles."

4. Correct: If the primary healthcare provider suspects myasthenia gravis (MG), the client will be asked to perform a repetitive movement to test a group of muscles. 1. Incorrect: The medication is given IVP after the muscle group has become fatigued. 2. Incorrect: The edrophonium (Tensilon) test is used to diagnose myasthenia gravis. 3. Incorrect: A person tests positive for MG if their muscles get stronger after being injected with edrophonium (Tensilon). The Tensilon test in a client with MG reveals improved muscle contractility after IV injection of the anticholinesterase agent edrophonium chloride (Tensilon). Anticholinesterase blocks the enzyme acetylcholinesterase. This test also aids in the diagnosis of cholinergic crisis (secondary to overdose of anticholinesterase drug), which occurs when there is too much cholinesterase inhibition. Clinical features include muscle fasciculation, sweating, excessive salivation, and constricted pupils. In this condition, edrophonium chloride (Tensilon) does not improve muscle weakness but actually increases it. Atropine, a cholinergic antagonist, should be readily available to counteract the effects of edrophonium chloride (Tensilon) when used diagnostically. If the primary healthcare provider suspects MG, the client will be asked to perform a repetitive movement to test a group of muscles. This may be: getting up and down from your chair; crossing and uncrossing your legs; holding your arms overhead until they get tired; counting backward from 100 until your voice starts to weaken. Each time the client gets tired, they'll give another dose of edrophonium (Tensilon) to observe whether the shot revives muscle strength each time. If so, the client may be diagnosed with MG.

A client with chronic arterial occlusive disease has a bypass graft of the left femoral artery. Postoperatively, the client develops left leg pain and coolness in the left foot. What is the priority action by the nurse? 1. Elevate the leg. 2. Check distal pulses. 3. Increase the IV rate. 4. Notify the primary healthcare provider.

4. Correct: In this case, there is nothing on the list the nurse can do to fix the problem. The primary healthcare provider must be notified immediately. Anticipate that the client will be returning to surgery because these are symptoms of an arterial problem that needs to be addressed immediately. 1. Incorrect: Arterial circulation is improved by lowering the extremity. Remember to raise venous problems; lower arterial problems. These signs/symptoms indicate an acute, emergent change in the client's condition. In this case, the nurse is "worried" the client will lose the extremity. There is nothing the nurse can do to fix the problem, so calling the primary healthcare provider is the best answer. 2. Incorrect: Assessing the pulses is delaying treatment and does not fix the problem. In this question you have only 1 option, so you must go with what is best for the client. 3. Incorrect: Increasing the IV rate does nothing to fix the problem, and you have only 1 chance in this question to show the NCLEX lady that you are a safe nurse.

A child is admitted to the emergency department due to suspected ruptured appendicitis with perforation. What would be the priority nursing assessment for this client? 1. Monitor for the Rovsing sign. 2. Assess for an increase in temperature. 3. Check for rebound tenderness at McBurney's point. 4. Monitor for increasing pain and rigidity of the abdomen.

4. Correct: Increasing pain and rigid, board-like abdomen are signs that the appendix may have ruptured, with resulting peritonitis developing. 1. Incorrect: The Rovsing Sign results in RLQ pain that occurs with palpation of the LLQ. This suggests peritoneal irritation due to palpation of a remote location and would indicate appendicitis. 2. Incorrect: Although children with appendicitis may have an elevated temperature, the priority would be assessing for the signs of peritonitis which include increasing pain and rigidity of the abdomen. Children can have an increased temperature with many different types of inflammation and infections. 3. Incorrect: Although rebound tenderness at McBurney's point is indicative of appendicitis, the nurse should not check for this due to the possibility of rupturing the appendix.

The nurse is caring for a client with tuberculosis receiving isoniazid therapy. Because of the possible peripheral neuropathy that can occur, which supplementary nutritional agents would the nurse expect to administer? 1. Cyanocobalamin 2. Vitamin D 3. Ascorbic acid 4. Pyridoxine

4. Correct: Isoniazid interferes with vitamin B6 (pyridoxine) metabolism by inhibiting the formation of the active form of vitamin B6. This interference often results in peripheral neuropathy. 1. Incorrect: Vitamin B12 (Cyanocobalamin) is not given to prevent peripheral neuropathy caused from isoniazid therapy. It is used to treat vitamin B12 deficiency often caused by pernicious anemia. It may be given in client's with peripheral neuropathy, but is not beneficial in clients whose neuropathy is due to isoniazid therapy. 2. Incorrect: Vitamin D is not given to prevent peripheral neuropathy. It is used in the treatment of weak bones, bone pain and/or bone loss. 3. Incorrect: Vitamin C is not given to prevent peripheral neuropathy cause from isoniazid therapy. It's use can be beneficial in clients with diabetic peripheral neuropathy.

A nine year old child with attention deficit hyperactivity disorder (ADHD) is being admitted to the pediatric unit. Who should the charge nurse assign this client to room with? 1. Ten year old with Crohn's disease. 2. Eight year old with a history of seizures. 3. Six year old admitted with asthma. 4. Seven year old with a urinary tract infection.

4. Correct: It would be best to pair this child with the child with a urinary tract infection. They are close to the same age and this child's condition does not require a quiet environment that could be interrupted by a hyperactive child. 1. Incorrect: The ADHD clients behavior could be detrimental to the client's Crohn's disease. Crohn's disease is not caused by stress but it can make signs and symptoms worse and may trigger flare-ups. 2. Incorrect: You want to decrease stimulation when you have a client with seizures not increase it. Seizures can be precipitated by sensory stimuli. 3. Incorrect: Again, the client with asthma does not need any stimuli that could cause agitation and stress. Stress can trigger the release of chemicals like histamine and leukotriences which can trigger narrowing of the airways.

A term primipara is admitted in active labor and with rupture of membranes. Her last vaginal exam one hour ago revealed that she was dilated to 6 centimeters, 100% effaced, and at -1 station. The client calls out "My belly really hurts. I feel like I have to have a bowel movement!" Which action should the nurse perform first? 1. Offer her a bedpan. 2. Call the primary healthcare provider. 3. Prepare for epidural administration. 4. Perform a sterile vaginal exam.

4. Correct: Labor can progress rapidly even in a primipara. As the fetal presenting part descends, pressure is placed on the rectum and many women report that it feels as though they need to have a bowel movement. The symptoms described indicate that the client has fully dilated and is at +1 or better station, and delivery may be imminent. 1. Incorrect: The nurse should first determine labor progress with a vaginal exam since this complaint is a common symptom of labor progressing and the fetus descending through the birth canal. Often, the client has had an enema to cleanse the colon prior to delivery so there usually is no fecal material present. 2. Incorrect: First determine labor progress with a vaginal exam. This might be necessary later, but is not the first action to perform. 3. Incorrect: First determine labor progress with a vaginal exam prior to preparing the client for anesthesia.

A client asks the nurse, "What causes hypermagnesemia?" The nurse should explain to the client that hypermagnesemia can occur secondary to what health problem? 1. Peripheral vascular disease 2. Dehydration 3. Liver failure 4. Renal insufficiency

4. Correct: Magnesium is excreted primarily through the kidneys. When the client experiences renal insufficiency, magnesium is held. The incidence of hypermagnesemia is rare in comparison with hypomagnesemia, and it occurs secondary to renal insufficiency. 1. Incorrect: Peripheral vascular disease does not lead to hypermagnesemia 2. Incorrect: Dehydration leads to the electrolyte imbalance of hypernatremia, it does not cause hypermagnesemia. A client who has become dehydrated due to excessive urination may experience hypomagnesemia. 3. Incorrect: Liver failure does not lead to hypermagnesemia. Magnesium is regulated by GI absorption and renal excretion.

A first generation Hispanic-American has been admitted to the psychiatric unit after being diagnosed with severe panic disorder. When developing the plan of care for this client, to which cultural background information should the nurse give priority? 1. Discuss treatment in terms of future plans for this client. 2. Do not use touch when communicating with this client. 3. Include the Protestant minister in the spiritual care of the client. 4. Allow family members to visit regularly.

4. Correct: Make allowances for individuals from other cultures to have family members around them and even participate in their care. Large numbers of extended family members are very important to African Americans, Native Americans, Asian Americans, and Hispanic-Americans. Denying access to these family support systems could interfere with the healing process. 1. Incorrect: Hispanic-Americans are more present oriented. They may be late to appointments and appear to be indifferent to some aspects of their therapy. Be accepting of these differences and refrain from allowing existing attitudes to interfere with delivery of care. 2. Incorrect: Hispanic-Americans have close personal space and use lots of touching and embracing. 3. Incorrect: Hispanic-Americans are most often Roman Catholic. The nurse needs to ask about religious preferences first.

How does the nurse identify the correct size of crutches for a client? 1. Turn the crutches upside down and measure from the heel to the shoulder. 2. Obtain a set of crutches and adjust the height until the client can stand comfortably while resting the axilla on the crutch pad. 3. Measure the client while standing upright from the axilla to the heel then adjust the crutches so that the elbow flexion is a 30-degree angle. 4. Measure the client from 2 inches below the axilla to 6 inches lateral to the client's heel.

4. Correct: Measuring the client from 2 inches below the axilla to 6 inches lateral to the client's heel correctly measures a client for crutches. This is the correct size while a client is standing. 1. Incorrect: This is not the correct way to choose the correct size crutches. Without the proper fit safety is a concern. 2. Incorrect: This is not how to choose the correct size of crutches. The client should not rest their weight on the crutch pad as this can cause damage to the brachial plexus nerve. 3. Incorrect: This is not how to choose the correct size of crutches. The shoulders should be relaxed, the hand piece should be adjusted to provide a 20°- 30° elbow flexion. The 2 inch drop below the axilla allows the weight to be pressed against the sides and the hands absorb the weight. The crutch should not be placed against the axilla or the brachial plexus nerve could be damag

A client has received 850 mL of an isotonic solution intravenously in less than 60 minutes. Which central venous pressure (CVP) reading noted by the nurse indicates a problem related to the amount of intravenous fluids infused? 1. 1 mm of Hg 2. 3 mm of Hg 3. 6 mm of Hg 4. 10 mm of Hg

4. Correct: Normal CVP is 2-6 mmHg. This client has received an isotonic solution amount of time. Remember that isotonic fluids stay "where I put them". The vascular space will increase in volume. More volume, more pressure! 1. Incorrect: This CVP reading indicates fluid volume deficit. There is no indication in the stem that the client is losing fluid. 2. Incorrect: This is a normal CVP reading. Normal CVP is 2 to 6 mm of Hg. 3. Incorrect: This is a normal CVP reading. Normal CVP is 2 to 6 mm of Hg.

A post-operative client has received morphine for pain. The nurse re-assesses the client 10 minutes later. Which assessment data warrants further action by the nurse? *Exhibit (nursing notes)* B/P 110/76, Pulse 68, Respirations 8, Pain level of 5, dressing dry and intact. 1. Blood pressure 94/60 2. Pulse rate 72/min 3. Pain level 3/10 4. Respiratory rate at 8/min

4. Correct: Normal respiratory rate is 12-20 per minute. The respiratory rate indicates respiratory depression following administration of an opioid. Care should be taken to titrate the dose so that the patients pain is controlled without depressing the respiratory function. 1. Incorrect: Respiratory rate warrants immediate action. However, blood pressure will continue to be monitored. 2. Incorrect: Pulse rate warrants no further action. Pulse rate is normal. 3. Incorrect: The pain level is expected following surgery. The client should continue to have a reduction on pain, as 10 minutes is not long enough to fully evaluate.

What action is most important for the nurse to take when a client receiving a cephalosporin develops abdominal cramping and diarrhea? 1. Administer antidiarrheal medication. 2. Increase fluid intake. 3. Provide food with the medication. 4. Notify the healthcare provider.

4. Correct: Notify the healthcare provider if diarrhea occurs as it can promote the development of Clostridium difficile infection. Cephalosporin difficile is a toxin producing bacteria that causes antibiotics-associated colitis, and can occur with antibiotic therapy. Cephalosporin is one of the most common antibiotics that cause clostridium difficile. 1. Incorrect: Taking a probiotic, stopping the antibiotic or switching to another antibiotic are standard treatments for antibiotic induced diarrhea. Administering an anti-diarrheal is not recommended for antibiotic induced diarrhea. 2. Incorrect: Increasing fluid intake will help with the associated dehydration seen with diarrhea, but will not correct the problem or decrease the risk of clostridium difficile. 3. Incorrect: If the client has GI upset, then cephalosporin may be given with food, however, the most important thing to worry about is the development of Clostridium difficile infection. So notifying the healthcare provider is the most important action.

The head nurse on a busy surgical unit is evaluating several fresh post-operative clients. Which observation should the nurse report immediately to the primary healthcare provider? 1. A post transurethral resection client with cherry colored urine 2. A post mastectomy client drains 40 mL of bloody drainage within 3 hours of the surgery 3. A post ileostomy client with a beefy red stoma and mucus drainage 4. A post thyroidectomy client reporting tingling in toes and fingers

4. Correct: One potential risk during a thyroidectomy is the accidental removal of some or all of the parathyroid glands. The client would develop signs and symptoms of hypocalcemia from decreased blood levels of calcium. As muscles become rigid and twitch, the resulting tetany would cause the client to experience a tingling sensation in toes and fingers. The nurse needs to notify the primary healthcare provider so that a calcium level can be drawn and the client given supplemental calcium. 1. Incorrect: Following a transurethral resection of the prostate (TURP), it is normal for urine to be cherry red in color. Continuous bladder irrigation will keep clots from developing over several days and the deep red color of urine is an expected finding following this procedure. 2. Incorrect: Mastectomy clients return from surgery with one or more drains placed under skin flaps in the breast tissue. These drains are part of a collection system that allows serous drainage to be removed from the surgical site, thus enhancing the healing process. Because this client has just returned from surgery, 40 mL over 3 hours is not an excessive amount. This is an expected finding that does not need to be reported. 3. Incorrect: The sign of a healthy stoma immediately post-op is a beefy red appearance and slightly elevated above the level of the abdomen. It is expected that the stoma will have a mucoid discharge for a day or so until normal stool begins to form again. This client displays normal post-operative findings with nothing unexpected.

What room assignment would be best for the nurse to make for a primigravida with gestational diabetes who was admitted for glycemic control? 1. A private room near the nurses' station. 2. A room with a client admitted with a placenta previa. 3. A room with a client in preterm labor. 4. A room with a client admitted with pregestational diabetes.

4. Correct: Placing clients with similar diagnoses together can result in information sharing and emotional support. It is ok to put these two clients together. 1. Incorrect: A private room is not required since the client has no emotional or infection control issues. Also, it is not necessary to place them near the nursing station because they do not need monitoring on that close of a level. 2. Incorrect: A client with placenta previa is in an unstable state and can have emotional issues concerning this diagnosis. The client would be best in a private room. 3. Incorrect: The client in preterm labor needs a private room that is quiet with limited visitors, she is having issues herself and concerned about her unborn child.

A military veteran with a history of post-traumatic stress disorder (PTSD) has arrived at the Crisis Center reporting an increase in nightmares, depression and anxiety. The nurse is aware the client would obtain the most immediate relief with what intervention? 1. Increase dose of antianxiety medications. 2. Greater family support interaction. 3. Referral to community support group. 4. Opportunity to verbalize memories.

4. Correct: PTSD is an emotional response to a traumatic event, usually beginning within several months of the event, although it can be delayed for years. When a client in severe distress arrives at the Crisis Center, the priority intervention must focus on relief of client's symptoms. The best non-invasive method to alleviate symptoms is encouraging the client to verbalize both memories and feelings. Though some individuals want to forget the incident, most clients experience a decrease in anxiety by discussing the event. 1. Incorrect: While it is true that a component of PTSD treatment involves either antianxiety or antidepressant medications, these drugs take several weeks to become effective. The nurse needs to provide an intervention that will give the client more immediate relief. 2. Incorrect: Clients with PTSD generally do benefit from family support and interaction, especially during periods of increased symptoms. However, the question requests a nursing action that would assist the client in crisis now. 3. Incorrect: Support groups are always beneficial for individuals experiencing long term problems, and are an invaluable resource to both client and family members. However, this question asks how the Crisis Center nurse could intervene to assist at this moment. This choice is a long term solution.

The nurse instructs a client taking isoniazid for the treatment of tuberculosis (TB) regarding appropriate food choices. Which food choices indicate to the nurse that teaching has been successful? 1. Salad with bleu cheese dressing. 2. Smothered liver with onions. 3. Smoked salmon with crackers. 4. Pear salad with lettuce.

4. Correct: Pears are acceptable fruit. Foods high in tyramine can cause headaches, fast or irregular heartbeats, nausea and vomiting and sensitivity to light. Foods high in tyramine such as aged cheeses, certain meats, liver, moked fish, sour cream, raisins, bananas and avocados should not be eaten when taking isoniazid. 1. Incorrect: Avoid foods high in tyramine. Foods high in tyramine such as salad with bleu cheese dressing can result in severe reactions when client is taking isoniazid. 2. Incorrect: Avoid foods high in tyramine. Foods high in tyramine such as smothered liver with onions can result in severe reactions when client is taking isoniazid. 3. Incorrect: Avoid foods high in tyramine. Foods high in tyramine such as smoked salmon can result in severe reactions when client is taking isoniazid.

Which intervention should the nurse recommend to the adult child who is caring for an elderly parent diagnosed with Alzheimer's disease (AD)? 1. Give parent a small dog for company and comfort. 2. Reset the water heater to 125 degrees Fahrenheit (51.67 degree Celsius) to prevent burns. 3. Place mirrors in multiple locations so parent sees images of self. 4. Make floors and walls different colors.

4. Correct: People with Alzheimer's disease (AD) get more confused over time. They also may not see, smell, touch, hear and/or taste things they once did. By creating a contrast in color between the floors and walls makes it easier for the person with AD to see. 1. Incorrect: Be careful with small pets. The person with AD may not see the pet and trip over it. This is a safety issue. A fall could cause a major injury to the client. 2. Incorrect: The water heater should be set below 120 degrees Fahrenheit (48.8 degrees Celsius) to prevent burns. 3. Incorrect: Limit the size and number of mirrors in the home. Mirror images may confuse the person with AD. They may not recognize self and may see the person as a stranger.

A nurse enters a client's room to find the client on the floor having a seizure. Which nursing action is appropriate for this client? 1. Hold the client's arms and legs. 2. Insert a padded tongue blade in the client's mouth. 3. Assist the client back into the bed. 4. Place a rolled towel under the client's head.

4. Correct: Placing a rolled towel under the client's head prevents further injury to the client. 1. Incorrect: Restraining the client may cause further injury to the client. 2. Incorrect: Forcing an object into the client's mouth can result in choking the client or injuring the client's teeth and mouth. 3. Incorrect: Lifting the client may cause injury to the nurse and client.

A nurse is providing education to a client regarding the use of an inhaler for acute asthma symptoms. Which statement made by the client would indicate the need for further teaching? 1. "I should shake the inhaler well before use." 2. "I should breathe out slowly and completely through my mouth before placing the mouthpiece of the inhaler in my mouth." 3. "I should hold my breath for approximately 8-10 seconds before exhaling slowly." 4. "I should administer the two puffs that are ordered in rapid sequence."

4. Correct: Rapid sequencing of the puffs is not a correct measure for using an inhaler. The client should wait 1 minute between puffs. This statement indicates the need for further teaching. 1. Incorrect: This is a correct measure that should be followed when using an inhaler. Clients are instructed to shake the inhaler well before use. 2. Incorrect: This is a correct measure that should be followed when using an inhaler. Clients are instructed to exhale slowly before bringing the inhaler to the mouth. 3. Incorrect: This is a correct measure that should be followed when using an inhaler. After removing the inhaler from the mouth, clients are instructed to hold their breath for 10 seconds, then breath out slowly. Proper use of an inhaler: -Shake the inhaler well before use (3 or 4 shakes) -Remove the cap -Breathe out, away from inhaler -Bring the inhaler to mouth. Place it in mouth between teeth and close mouth around it. -Start to breathe in slowly. Press the top of inhaler once and keep breathing in slowly until a full breath has been taken. -Remove the inhaler from mouth, and hold breath for about 10 seconds, then breathe out. -If a second puff is needed, wait 30-60 seconds, shake inhaler again, and repeat steps 3-6.

A client comes into the clinic reporting muscle pain and tenderness but denies previous injury. Based on data gathered by the nurse, what client medication does the nurse suspect is causing this problem? *Exhibit (labs and Nursing notes)* *labs* CPK: 300 U/L ALT: 38 U/L AST: 42 U/L *nursing notes* Alert, oriented client with general weakness and muscle tenderness noted. Reports myalgia, muscle weakness, fatigue, and joint pain. Urine sample obtained; urine cola-colored. 1. Captopril 2. Furosemide 3. Nadolol 4. Rosuvastatin

4. Correct: Rosuvastatin is a lipid-lowering agent. All lab work is abnormal and indicates muscle and liver damage. Lipid-lowering drugs can cause liver damage. The assessment of the lab values reveals muscle damage and could indicate rhabdomyolysis development. Creatine phosphokinase: males 55-170 u/L and females 30-135 u/L; Alanine aminotransferase (ALT): 4-36 u/L; Aspartate aminotransferase (AST): 0-35 u/L 1. Incorrect: Captopril does not affect the liver or muscle. Captopril is an angiotensin-converting enzyme (ACE) prescribed for treatment of hypertension. 2. Incorrect: Furosemide does not affect the liver or muscle. Furosemide is a loop diuretic which promotes diuresis. 3. Incorrect: Nadolol does not affect the liver or muscle. Nadolol is a non-selective beta blocker. The action of nadolol is to treat arterial fibrillation, hypertension, migraines and chest pain

What instruction would the nurse give a client about a newly prescribed salmeterol inhaler? 1. "Use the inhaler immediately if wheezing and shortness of breath occur during exercise." 2. "Use the inhaler when you experience a stuffy nose due to seasonal allergies." 3. "Carry the inhaler with you at all times and take 2 puffs anytime you experience an exacerbation." 4. "This inhaler should be used routinely as prescribed even when free of symptoms."

4. Correct: Salmeterol is a maintenance medication. It can prevent asthma attacks and exercise induced bronchospasm. Salmeterol acts as a bronchodilator. It works by relaxing muscles in the airways to improve breathing. 1. Incorrect: Salmeterol can be used to help prevent exercise induced bronchospasm, but it should be taken 30 to 60 minutes before exercise. 2. Incorrect: Salmeterol is indicated for asthma only and bronchospasm induced by obstructive pulmonary disease. It is not indicated for seasonal allergies. 3. Incorrect: Salmeterol is a maintenance medication. Albuterol is used as a "rescue inhaler" for bronchospasms. Option 1, 2, and 3 are false. Salmeterol is a beta2-agonist, a maintenance medication that clients with asthma use twice a day every 12 hours. Clients should be taught that if symptoms occur before next scheduled dose, use a rapid acting inhaled bronchodilator. Clients should be cautioned not to use salmeterol to treat acute symptoms. Clients using salmeterol for prevention of exercise induced bronchospasm should not use additional doses for 12 hours.

A client is brought into the emergency department (ED) with nausea, vomiting and diarrhea after eating chicken at a picnic. The nurse suspects that this client has most likely contracted which infection? 1. Shigella 2. Escherichia coli 3. Clostridium Difficile 4. Salmonella

4. Correct: Salmonella is a gram negative bacillus found in animal sources such as chicken products, eggs, turkey, and some beef. Nausea, vomiting, and diarrhea after ingesting infected chicken would be the classic signs/symptoms. 1. Incorrect: Shigella infection is a gram negative organism that invades the lumen of the intestine and causes severe runny, bloody diarrhea which can be transmitted through the fecal/oral route. Improper hygiene is most likely cause. 2. Incorrect: Escherichia coli is the most common aerobic organism colonizing the large bowel. It is often linked to ingestion of undercooked contaminated beef and vegetables that have been contaminated by animal waste water. Signs and symptoms of E. coli include bloody diarrhea, severe cramps, nausea and vomiting, and renal failure. 3. Incorrect: Clostridium Difficile is a spore-forming bacterium usually preceded by antibiotics, which disrupt normal intestinal flora and allow the C. Difficile spores to proliferate within the intestine. C. difficile signs and symptoms can range from mild diarrhea to severe colitis.

An elderly client comes to the clinic for a check-up. The client's daughter tells the nurse that her father's dementia symptoms become increasingly more difficult to handle in the evening. How would the nurse document this symptom? 1. Confabulation 2. Apraxia 3. Pseudodementia 4. Sundowning

4. Correct: Sundowning is a phenomenon where symptoms seem to worsen in the late afternoon and evening. Communication becomes more difficult, with increasing loss of language skills. Institutional care is usually required at this stage. 1. Incorrect: Confabulation is the term used for creating imaginary events to fill in memory gaps. This is sometimes associated with dementia, but more often with disorders like Korsakoff's syndrome, traumatic brain injuries or tumors. 2. Incorrect: Apraxia is the term used for the inability to carry out motor activities despite intact motor function. 3. Incorrect: Pseudodementia is depression. Depression is the most common mental illness in the elderly, but it is often misdiagnosed and treated inadequately. Cognitive symptoms of depression may mimic dementia.

A client was admitted to the medical unit after an acute stroke. Which nursing activity can the registered nurse delegate to the LPN/VN? 1. Screen client for contraindications for tissue plasminogen activator (tPA) therapy. 2. Place seizure precaution equipment in client's room. 3. Perform passive range of motion (ROM) exercises. 4. Administer enoxaparin 1 mg/kg subcutaneously every 12 hours.

4. Correct: The LPN/VN can administer subcutaneously medications. 1. Incorrect: This is an RN only responsibility and cannot be delegated. 2. Incorrect: The unlicensed assistive personnel (UAP) can be assigned to place equipment in a client's room. 3. Incorrect: Passive ROM exercises can be done by the UAP. During the first 48 hours after an acute stroke, some nursing activities can be delegated to LPNs/VNs and unlicensed assistive personnel (UAPs). The RN should communicate frequently with these staff members to rapidly intervene based on changes in the client's status. The RN is responsible for assessment, evaluation, formulating the care plan, and teaching plan, and teaching complex topics. The RN cannot delegate these tasks to the PN and UAP.

While preparing an information sheet for a client diagnosed with a vancomycin-resistant enterococcus (VRE) urinary tract infection (UTI), the home health nurse should include which instructions? 1. Wash hands with hot water and soap when hands are soiled. 2. Gloves are not needed in the home since contamination with VRE has already occurred. 3. Wash hands before using the bathroom and after preparing food. 4. Clean the bathroom and kitchen with warm water and bleach.

4. Correct: The bathroom and kitchen should be cleaned with warm water and bleach to decrease contamination. The client should wash hands after using the bathroom and before preparing food. 1. Incorrect: Instructing the client and family to wash with hot water can cause drying and cracking of the skin. Hands should be washed with all contacts. Washing hands is the single most important thing to do to prevent infection. 2. Incorrect: Gloves are needed with VRE to prevent spread of infection. Gloves are especially needed if contact with blood or other infectious materials is anticipated. 3. Incorrect: Hands should be washed after using the bathroom and prior to handling or preparing food.

A client has been on the nursing unit for two hours following a retropubic prostatectomy for the treatment of prostate cancer. The client is receiving a continuous bladder irrigation of normal saline infusing at 1000 mL/hr. The client's urine output for the past two hours is 410 mL. What is the nurse's first action? 1. Inspect the catheter tubing for obstruction. 2. Irrigate the catheter with a large piston syringe. 3. Notify the primary healthcare provider. 4. Stop the irrigation flow.

4. Correct: The catheter output should be at least the volume of irrigation input plus the client's actual urine. A severe decrease in output indicates obstruction in the drainage system. The first action is to stop the irrigation flow to prevent further bladder distention. Bladder distention is one of the main causes of hemorrhage in the fresh post op period. 1. Incorrect: The next action is to check the external system for kinks or obstruction to assess if this is the cause of the decreased urine output. Obstruction of the catheter tubing can also cause bladder distention. 2. Incorrect: After the external system is checked for kinks or obstruction, and the client's urine output doesn't change, then the catheter is irrigated with 30 to 50 mL of normal saline using a large piston syringe. However, irrigating a new post-op client is not the safest or first action for the nurse. 3. Incorrect: Of the options listed here, this is the last intervention. If the obstruction is not resolved after irrigating the system, the primary healthcare provider must be notified.

During evening rounds on a medical unit, a client is discovered in cardiac arrest. After activating the code button, the nurse initiates chest compressions. A second nurse enters the room to assist. What priority task could be delegated to the second nurse? 1. Retrieve the crash cart. 2. Document the code events. 3. Notify the primary healthcare provider 4. Begin oxygenating the client.

4. Correct: The chest compressions, airway, and breathing (CAB) sequence is always of primary concern. The first nurse correctly activated a code and then began chest compressions. The second nurse will assist by oxygenating the client, using a bag valve mask. 1. Incorrect: Although it will be necessary to bring the crash cart into the room, the initial priority should focus on the client's needs. In the case of a client in cardiac arrest, the first personnel to respond must focus on CPR protocols, including compressions and oxygenation. Other personnel can bring the crash cart into the room. 2. Incorrect: Documenting all the events that occur during a code is vital for both legal and quality assurance purposes; however, the initial priority must focus on stabilizing the client. 3. Incorrect: The Healthcare Provider does need to be notified, but it is not an immediate priority for either nurse. Ancillary personnel, such as the unit secretary, can complete this task. Professional personnel must focus on the client's immediate needs at this critical point.

Which prevention strategy should the nurse consider when developing a health promotion plan for new parents concerning sudden infant death syndrome (SIDS)? 1. Place the infant in the prone position when placing the infant in the bed. 2. The child should sleep in a separate room from the parents. 3. The child should not have a pacifier in place when sleeping. 4. The child should be placed in the supine position when sleeping.

4. Correct: The child should be placed in the supine position when being put to bed for naps or for the night. This position has helped to reduce the incidence of SIDS by as much as 50% since the 1990s. 1. Incorrect: The child should not be placed in the prone position when being put to bed for sleep. If the child is awake and supervised, the prone position encourages muscular development. 2. Incorrect: The American Academy of Pediatrics recommends that the infant sleep in the same room with the parents, but not in the same bed. 3. Incorrect: The American Academy of Pediatrics recommends that a pacifier be used for the first 6 months because of the benefit in regard to pain management and prevention of sudden infant death syndrome, but recommends the child be weaned from the pacifier during the second 6 months of life.

The nurse is preparing to bathe a client who is confined to the bed. Which action by the nurse is important to preserve client's self-esteem as the task is completed? 1. Closes the door for privacy. 2. Introduces self and explains the procedure. 3. Bathes the client without the help of others. 4. Covers the client with a bath blanket.

4. Correct: The client does not have to be exposed during the bed change and should be covered with a bath blanket as the top sheet is removed. A bath blanket covers the client as once section at a time of the body is exposed and bathed. This allows for the most privacy and protects self-esteem. 1. Incorrect: Closing the door is very important but the client's privacy should be maintained at all times even from the nurse administering the bath. A bath blanket promotes privacy and protects the self-esteem. 2. Incorrect: Introducing yourself to the client and explain procedures shows respect. These two actions do not provide for privacy and preserve the self-esteem of the client. 3. Incorrect: If help is needed during an occupied bed bath to protect the client and provide for safety, help should be obtained. The nurse should use measures to protect privacy and preserve the client's self-esteem.

A client who is 3 days post abdominal hysterectomy reports left sided chest and back pain on inspiration. The nurse notes hemoptysis, shortness of breath, and auscultates bilateral rales. Vital signs are BP 140/90, HR 122, Resp 28, T 100°F (37.78°C) . O2 sat is 89%. Based on this assessment, what is the nurse's priority action? 1. Obtain 12 lead ECG. 2. Draw arterial blood for ABGs. 3. Administer morphine for pain. 4. Notify the primary healthcare provider.

4. Correct: The client has likely developed a pulmonary embolus post abdominal hysterectomy. The classic symptoms are chest and back pain on inspiration, hemoptysis, and dyspnea. This is a potentially life threatening complication and must be reported to the primary healthcare provider immediately. 1. Incorrect: Obtaining a 12 lead ECG would delay treatment. Remember, the nurse needs to decide on one option that best fixes the problem. 2. Incorrect: Obtaining ABGs would be delaying treatment and would not fix the problem. 3. Incorrect: Morphine is given for chest pain related to an MI. If given to this client, it would depress respirations in a client that already has respiratory distress. This would be unsafe and could kill the client. Don't be a killer nurse.

A client admitted with a myocardial infarction has developed crackles in bilateral lung bases. Which prescription written by the primary healthcare provider should the nurse complete first? 1. Draw blood for arterial blood gases. 2. Place compression hose on legs. 3. Insert indwelling catheter for hourly urinary output. 4. Administer furosemide 20 mg intravenous push (IVP).

4. Correct: The client is developing pulmonary edema or heart failure and needs to be diuresed to remove excess fluid. The question stem tells you that you have prescriptions for these four options so what are you going to do first. All prescriptions are possible but furosemide will fix the problem. 1. Incorrect: You may need to draw these to evaluate the effect of the fluid on oxygenation but this option will not be priority over removing the fluid. 2. Incorrect: Compression hose will help prevent stagnation of blood in the lower extremities to prevent deep vein thrombosis (DVT). Great nursing care but not priority when fluid is developing in the lungs. 3. Incorrect: The indwelling catheter can be inserted after administration of the diuretic. You will be glad you have that indwelling catheter but it is not the prescription that will help with the fluid overload problem.

An 18 year old football player is admitted to the ortho unit after a femur fracture. He is scheduled for a rod to be placed in the morning, but suddenly develops severe shortness of breath, a petechial rash on his chest, and unstable vital signs. What should the nurse do first? 1. Decrease rate of IV fluids. 2. Neurovascular checks of affected leg. 3. Elevate the head of the bed. 4. Call the active response team.

4. Correct: The client is exhibiting symptoms of a fat embolism, particularly with the petechial rash on his chest and severe shortness of breath. Due to his age, high risk behaviors with contact sports, and the large long bone fracture, he is the classic example of a client that may experience a fat embolus. This constitutes a medical emergency and activation of the response team. 1. Incorrect: This does not affect breathing here and will do nothing to resolve the fat embolism. 2. Incorrect: Neurovascular checks of the leg will not help the client's breathing and are not the first priority for the nurse. 3. Incorrect: The nurse may elevate HOB to assist with breathing unless client is hypotensive. Either way, this is not the best first answer.

Which type of comment should the nurse expect from a client exhibiting clang associations? You answered this question 1. Concrete explanations for abstract ideas 2. Reporting very small details when explaining something 3. Comments that are illogically associated 4. Use of rhyming words when talking

4. Correct: The client may use rhyming words, such as dog, bog, cog, jog. It is the meaningless rhyming of words, often in a forceful manner. 1. Incorrect: This type of comment indicates concrete thought. Concrete thinking is characterized by immediate experience rather than abstraction. 2. Incorrect: This type of comment indicates circumstantiality. This is characterized by indirectness and delay before the person gets to the point or answers a question. The person gets caught up in countless details and explanations. 3. Incorrect: These indicate loose associations or derailment. It is a sequence of unrelated or only remotely related ideas.

The nurse has been working with a client who has a diagnosis of schizophrenia. The client has had three inpatient admissions in the past, but none in the past 6 months. Which statement by the client indicates adequate understanding of the medication treatment regimen? 1. I am feeling better so I hope that I don't have to take the medication for long. 2. I can stop the medication after I have been out of the hospital for a year. 3. The medicine is good for me now; however, I don't want to take it forever. 4. The medication keeps me out of the hospital, and I don't want to hear voices again.

4. Correct: The client must take the medicine long-term. If the client makes the connection between the medicine and feeling better, adherence is more likely. 1. Incorrect: This statement indicates lack of understanding of the disorder and required outcomes. Medication is likely to be required indefinitely. 2. Incorrect: The client does not understand the connection between adherence to medication and staying out of the hospital. 3. Incorrect: It is very likely that the client will be on the medication for a lifetime. This information should be a part of the plan of care and not withheld from the client's knowledge of the disorder and effective control.

A client who is in the manic phase of bipolar disorder was admitted to the psychiatric unit two days ago. Since admission, the client has been overly active, dressing bizarrely and sleeping very little. What type of activity should be planned for this client for the period following the evening meal? 1. Encourage the client to watch TV with the other clients on the unit. 2. Engage the client in a game of ping pong. 3. Suggest that the client play monopoly with other clients. 4. Provide soft lighting in the client's room for reading.

4. Correct: The client needs minimal stimulation to help reduce activity level and encourage sleep. A quiet environment that is calm and dimly lit is ideal for the manic client. 1. Incorrect: TV and other clients will increase stimulation for the client, which may cause this client to become more active. This client should be directed away from active environmental stimuli to minimize the escalation of mania. 2. Incorrect: The less activity in the evening, the more likely the client will sleep. Ping pong is a constructive outlet for energy but should not be performed this close to bedtime. 3. Incorrect: The client would not be able to attend to a long monopoly game, and the other clients would provide too much stimulation. The board game is not the best choice for the manic client who needs sleep.

A client with diabetes is hospitalized for debridement of a non-healing foot ulcer. Following the procedure, the nurse notes that the client has become confused and combative. The family expresses concern with the behavioral changes and requests that the client be restrained in bed. What is the nurse's priority action? 1. Notify the primary healthcare provider. 2. Apply a vest restraint as requested by family. 3. Move client to a room near the nurse's desk. 4. Obtain a finger-stick blood glucose level.

4. Correct: The client's behavior has negatively changed following the ulcer debridement procedure. The nurse's priority is to determine the cause of the client's confusion. The nurse is correct to investigate other possible causes for the behavior changes, including an abnormal glucose level in this diabetic client. 1. Incorrect: The nurse will indeed have to contact the primary healthcare provider about the client's change in behavior. However, the first priority would be to assess the client and collect data prior to placing that phone call. 2. Incorrect: The nurse understands that restraints cannot be applied by family request. Additionally, applying a restraint can often increase negative behavior while ignoring the actual cause. 3. Incorrect: Although assigning confused clients to a room near the nurses' station is an accepted practice, this does not determine the cause for the changing behavior and is not a priority at this time. The nursing priority is to assess the client for possible factors causing the behavior changes.

A client with nausea, vomiting, and diarrhea for the past three days has been prescribed one liter of normal saline with 40 mEq (40 mmol/L) of potassium chloride to infuse at 250 mL per hour. Which assessment would the nurse report to the primary healthcare provider prior to initiating the infusion? 1. Blood pressure of 106/54 2. Apical pulse of 112 per minute 3. Tenting of the skin over the sternum 4. Urinary output of 148 mL for the past 6 hours

4. Correct: The client's output is below normal. This could indicate a problem with renal perfusion. Potassium is excreted through the kidneys, so if the kidneys are not being perfused, the client would retain potassium. The healthcare provider would need to be aware of the client's low urine output. 1. Incorrect: A client in fluid volume deficit would have a low blood pressure. This is an expected assessment prior to fluid resuscitation. 2. Incorrect: A client in fluid volume deficit would have a fast pulse rate. This is an expected assessment prior to fluid resuscitation. 3. Incorrect: A client in fluid volume deficit would have tenting of skin. This is an expected assessment prior to fluid resuscitation. Look at the clues in the stem: nausea, vomiting, diarrhea for 3 days. NS with potassium at 250 mL/hour. What are we worried about? Fluid volume deficit? Yes. But which option would interfere with giving the prescribe IV fluid with potassium? The low UOP. The number one way to get rid of potassium is through the kidneys. So if the kidneys are not working properly, then the potassium could get too high.

In what position should the nurse place a client diagnosed with gastric reflux? 1. Orthopneic 2. Semi-Fowler's 3. Sims' 4. Reverse Trendelenburg

4. Correct: The entire bed is tilted with the foot of the bed lower than the head of the bed. This position promotes gastric emptying and prevents esophageal reflux. 1. Incorrect: Orthopneic position has the client sit in the bed or at the bedside. A pillow is placed on the over-bed table, which is placed across the client's lap. The client rests arms on the over-bed table. This position allows for chest expansion and is especially beneficial to clients with COPD. 2. Incorrect: The head of the bed is elevated 30 degrees. This position is useful for clients who have cardiac, respiratory, and neurological problems and is often optimal for clients who have a nasogastric tube in place. 3. Incorrect: Sims' or semi-prone position has the client on the side halfway between lateral and prone positions. Weight is on the anterior ileum, humerus, and clavicle. The lower arm is behind the client while the upper arm is in front. Both legs are flexed but the upper leg is flexed at a greater angle than the lower leg at the hip as well as at the knee. This is a comfortable sleeping position for many clients, and it promotes oral drainage.

The charge nurse identifies that three admissions were received during the night shift, one nurse has called in sick, and the clients on the unit have high acuity levels. What action should the nurse implement first to ensure client safety? 1. Take report on the most critical clients first. 2. Encourage the staff to help each other. 3. Assign one additional client to each nurse. 4. Call the nursing supervisor to request additional staff immediately.

4. Correct: The hospital nurse to client staffing ratio should reflect the complexity of nursing care for high acuity clients. The nurse should call for immediate help so that a safe care environment is maintained for all clients. The charge nurse should notify the nursing supervisor who will seek additional staff. The nursing supervisor may be able to assist with client care until another nurse can come in to work. 1. Incorrect: The critical clients are important, but all clients must be considered. The charge nurse must evaluate each client's status and needs to assign the appropriate staff to care for them. The safety of each client must be reviewed. 2. Incorrect: The charge nurse may encourage the staff to work together. This is a positive action but the priority for the charge nurse is to ask for additional staff to maintain safe nursing care. 3. Incorrect: Each nurse may have to increase his/her client load until adequate staffing can be obtained. However, calling the nursing supervisor to request help is the first action.

A nurse is conducting an initial admission history on a client who is reporting bone pain secondary to cancer with metastasis to the bone. What does the nurse determine is the most important information to gather during this initial screening? 1. The physical assessment of the client 2. The hemoglobin and hematocrit levels 3. The amount of pain medication the client is receiving 4. The client's description of the pain

4. Correct: The most important information to gather during the initial screening is the client's perception and description of the pain. Pain is subjective, based on the client's perception. This is also the primary complaint of the client upon admission. 1. Incorrect: The question is asking about the client's pain. The physical assessment is important but does not address the client's perception of their own pain. 2. Incorrect: RBCs are produced in the bone marrow. The H&H might be affected but will not be the cause of the pain and assessed later with admission lab and diagnostics. 3. Incorrect: The amount of pain medication is important, but is not the most important information to gather from a client who is reporting pain, particularly with cancer and metastatic bone pain.

While a nurse was in shift report, four clients called the nurses' station. Which client should the nurse see first? 1. Child whose colostomy bag is leaking. 2. Three day post op client requesting pain medication. 3. Child admitted with failure to thrive, whose mother requested formula. 4. Client who needs a peak blood level drawn because the antibiotic just finished infusing.

4. Correct: The most urgent task is the peak medication level that needs to be drawn. If the level is not drawn at the appropriate time, the results may not give an accurate report of whether the medication is at the appropriate dosage or not, and if the dosage is safe. 1. Incorrect: A leaking colostomy bag is uncomfortable and should be seen, but this is not time sensitive like the peak blood level. 2. Incorrect: Pain needs assessing and treated appropriately. The key here is three days post op so the administration of the pain medication does not take priority over the need to draw the blood levels at this time. 3. Incorrect: Nutrition for a baby that is admitted for failure to thrive is important, but can wait a few minutes until blood levels are drawn.

A client who is occasionally confused states that the medication is the wrong color when the nurse hands it to the client. What action should the nurse take? 1. Encourage the client to take the medication. 2. Tell the client that the medication is correct. 3. Explain that generic medications may be different colors. 4. Double check the medication before administering.

4. Correct: The nurse cannot assume that the client is confused. Assessing orientation, LOC and asking client to state his/her name would help identify if the client is confused. The nurse must double-check. An error may be prevented by doing this. Seeking clarification is the safest option. 1. Incorrect: The nurse may make a medication error if she/he encourages the client to take the medication without double checking. To prevent errors, the nurse must adhere to the five rights of medication administration: right drug, right dose, right time, right route and right patient. 2. Incorrect: The client may be identifying an error. The nurse should double check that this is the correct medication. Determine if the client understands the purpose of the medication. 3. Incorrect: This statement is true; however, to maintain safety of the client, the medication should be checked again. Seeking clarification is the safest option.

A client states, "I really do not want to go through open heart surgery. I have told my children this, but they still want me to go through with the surgery. I don't know what to do." What is the best response for the nurse as client advocate? 1. Your children are correct. The open heart surgery is the best thing for your health. 2. You feel as if your children are not addressing your concerns. You and your family will need to resolve this before you go to surgery. 3. I can contact your primary healthcare provider so that you can discuss your concerns regarding open heart surgery. 4. You have some genuine concerns about the open heart surgery, and you feel as if your children are not addressing your concerns.

4. Correct: The nurse has a duty to advocate for the client if there is a discrepancy between the care or proposed care and the client's wishes regarding treatment. It is important to acknowledge the client's feelings, and to demonstrate compassion and a willingness to understand. This presents an opportunity for additional communication to help answer some of the client's questions, or set up a client-family conference with the client, the client's family, and the primary healthcare provider. 1. Incorrect: When the nurse agrees with the client's children, the nurse ignores the client's feelings and does not address the issue of the client's treatment wishes. 2. Incorrect: When the nurse restates the client's comment without investigating the client's concerns, the issue goes unresolved. 3. Incorrect: Offering only to contact the primary healthcare provider is an incomplete solution and hints of the nurse not taking responsibility to investigate the client's concerns. The client may be uncomfortable addressing concerns with the primary healthcare provider before resolving the issue of treatment wishes with family members.

The nurse is working with a new unlicensed assistive personnel (UAP) on a post-operative unit. The nurse received a client following surgery 8 hours ago. The first vital sign check was performed by the nurse. As the evening progressed, the unit tasks became very demanding and the nurse had to delegate several actions to the UAP. In planning care for the post-operative client, the nurse has decided to retain the task of vital sign assessment. What was the rationale for this plan? 1. The nurse did not trust the new UAP. 2. The nurse prefers to check all vital signs on all clients. 3. The nurse is responsible for the assessment of all vital signs of post-op clients. 4. The nurse does not know the skills of the new UAP.

4. Correct: The nurse has not been able to determine the skill of vital sign assessment for this new UAP. When the licensed person cannot determine this, the task should not be delegated. This determination is needed to assure client safety is being considered. 1. Incorrect: The nurse may trust the UAP; however, the nurse has not been able to determine the competency of the new staff member. Since this is a postoperative client, It is important that the vital sign measurement is accurate to detect any changes or possible complications. 2. Incorrect: When a unit is very busy, the nurse should rely on the UAP if the person is competent to perform the tasks. In this situation, it is not a matter of the nurse preferring to take all the vital signs, but the nurse needs to know the competency level of the UAP before delegating this task. 3. Incorrect: The nurse can measure vital signs; however, agency policy usually states that UAP can perform this task also. If the client is unstable, the nurse would retain the role of measuring the vital signs. Once the client is stable, the UAP could perform this task. However, since the new UAP's competency level is not known, the nurse does not delegate this task for the safety of the client.

The nurse notices the primary healthcare provider removes gloves after performing an invasive procedure on a client. The healthcare provider then enters another client's room without washing hands. What is the initial action by the nurse? 1. Ignore it since the primary healthcare provider knows best. 2. Contact the nursing supervisor. 3. Notify the chief of medical staff. 4. Remind the primary healthcare provider of the importance of standard precautions.

4. Correct: The nurse is the client's advocate and can remind the primary healthcare provider of the importance of washing hands before entering a client's room. Hand washing should be performed when going from one room to another. 1. Incorrect: Nurses are to be client advocates and resolve a problem that they see. The primary healthcare provider should wash their hands prior to entering another client's room. 2. Incorrect: The nursing supervisor is not the first step, the nurse is. This incident may be reported to the charge nurse at a later time but the client's safety is priority. 3. Incorrect: This is not the first step. The nurse should address the problem when it is witnessed. The nurse should follow the chain of command when reporting a problem but speaking to the chief of medical staff is not the best action at this time.

During client care rounds, the nurse reports that a client coughs frequently after taking anything by mouth. The dietician recommends a swallow evaluation for the client. The primary healthcare provider writes the prescription. Which statement best describes this process? 1. Collaboration with the ancillary care providers. 2. Collaboration between the primary healthcare provider and the dietician. 3. Collaboration with the risk management team. 4. Collaboration among members of the multi-disciplinary team.

4. Correct: The nurse reporting assessment findings, the dietician suggesting a swallow evaluation, and the primary healthcare provider ordering the swallow evaluation are an example of collaboration of care among members of the multi-disciplinary team. 1. Incorrect: Collaboration of care with the ancillary providers is a partial answer, as is collaboration of care between the primary healthcare provider and the dietary department. These healthcare team members all are part of the multi-disciplinary team. 2. Incorrect: Collaboration of care with the ancillary providers is a partial answer as is collaboration of care between the primary healthcare provider and the dietary department. These healthcare team members all are part of the multi-disciplinary team. 3. Incorrect: Risk management is a formal process through which a healthcare facility or provider agency tracks client outcomes to identify potential problems and ensure safe delivery of care.

The nurse is bathing a confused client in the acute care unit. The nurse talks with the client and explains each procedure. During the bath, the client becomes very agitated. What should the nurse do? 1. Complete the bath as quickly as possible. 2. Reassure the client and request them to stop acting out. 3. Continue bathing with assistance from an unlicensed assistive personnel. 4. Stop the bath, dress and reassure the client.

4. Correct: The nurse should not continue bathing if the client is becoming so distressed. Perhaps the bath can be completed at a later time. Safety is the priority. 1. Incorrect: The client is obviously distressed. Continuing the bath could jeopardize the safety of the nurse and client. 2. Incorrect: Reassurance may not work with the confused client. It is difficult to know exactly why the client is becoming so distressed. The safety of the client is important. 3. Incorrect: Adding a second person will increase the feelings of powerlessness in the client. This could and will add to the client's distress. Stopping the procedure is the safest answer.

The nurse is making rounds on the psychiatric unit at the beginning of the shift. Which client should be seen first? 1. Client with somatoform disorder. 2. Client with depression. 3. Client with panic attacks. 4. Client with hallucinations.

4. Correct: The nurse will need to assess the client with hallucinations first. The client who is actively hallucinating may be hearing voices, and the voices may order the client to do something harmful to self or others. 1. Incorrect: The client with Somatoform disorder has a physical symptom with a psychological cause and does not require immediate attention. Although the symptoms are real to the client and not under voluntary control as with fictitious disorders. They do not have a physical cause that would warrant immediate attention. Therefore, this client does not have a need to be seen before the client having hallucinations who may be at risk for harming self or others. 2. Incorrect: The client with depression should be seen second. There is no indication that the client is currently on suicide precautions. However, as the client receives medication and the energy level rises, the client may be at an increased risk of suicide. 3. Incorrect: The client with panic attacks is likely to summon the nurse if needed immediately. This client would not need immediate attention at this time. The client with panic attacks is not considered at risk of harm to self or others and would not have to be seen before the clients with hallucinations or depression.

A client has been diagnosed with genital herpes. Which comment indicates understanding of the disease and prevention of the spread of the disease? 1. "I can be treated and then no one else is at risk." 2. "Using condoms will keep my sex partner from acquiring the disease." 3. "If I have no sores, I am not contagious to anyone." 4. "My sex partner should be tested because we have not always used condoms."

4. Correct: The sex partner may become infected even if using a condom. The condom does not always cover all lesions. Condoms do, however, reduce the likelihood of getting/transmitting the disease. 1. Incorrect: Sex partners can acquire the disease even if no open sores are present. Treatment manages outbreaks but does not cure the disease. 2. Incorrect: Condoms decrease the risk. Abstinence is the only guaranteed way to not expose your partner. 3. Incorrect: Sex partners may get the disease even if no open sores are present; therefore, they should be tested for the disease.

A client has arrived at the emergency room reporting tingling to both lower legs over the past 24 hours. The only significant health history is a cold for the past week. During the nursing assessment, the client indicates that both thighs are feeling numb. What priority action should the nurse initiate immediately? 1. Assess bilateral pedal pulses. 2. Initiate a Code Blue. 3. Roll client onto left side. 4. Prepare for intubation.

4. Correct: The symptoms reported by this client indicate the onset of Guillian-Barre syndrome, an acute inflammatory disease that may occur following a respiratory illness and is characterized by progressive, ascending paralysis. The extent of paralysis varies by client, but airway is always the greatest concern. Because the client's lack of sensation is progressing, the nurse should anticipate the health care provider may need to immediately intubate to protect the airway. 1. Incorrect: Assessment is a primary essential part of the nursing process and provides clues to a client's status. This client has indicated a numbness to lower extremities for the last 24 hours, which is now extending up to the thighs. Determining the presence of pedal pulses is not the priority issue at this time. Another area presents a greater concern. 2. Incorrect: The code blue alarm is only initiated when a client has no heartbeat or spontaneous breathing. This client is still breathing and, therefore, a code blue is not necessary at this time. However, there is a potential problem that urgently needs to be addressed by the nurse. 3. Incorrect: This action serves no purpose for Guillian-Barre. Positioning is not beneficial for this client. Option 1: Not a good choice! Although assessment is the first part of the nursing process, the nurse has already gathered sufficient data to begin implementation. In this option, assessing pulses is not a priority action at this time. Option 2: This implementation is obviously premature since the client currently has both a pulse and an airway! The nurse should initiate an action that will prevent the client from deteriorating into a cardiac arrest. Option 3: You may have realized that this action serves absolutely no purpose for a client with Guillian-Barre! Rolling a client to the left side may be appropriate for obstetrical clients, or even clients experiencing dumping syndrome, but would not be beneficial for this client. Positioning is not the most important priority for this client. Option 4: Good choice! You have realized that clients with Guillian-Barre experience an ascending paralysis that can reach the diaphragm and the respiratory muscles in just a matter of hours, requiring airway support. The nurse has determined that based on the client's health history, along with the present symptoms, prophylactic intubation is the priority concern at this time.

The nurse is caring for a client with a diagnosis of major depression. The client began taking a selective serotonin reuptake inhibitor (SSRI) three days ago. The client says, "I am just not feeling well. My medicine is not working." Which reply by the nurse indicates adequate understanding of treatment? 1. "I agree, your medication is not working." 2. "Your treatment may have to be changed." 3. "Most SSRIs take about 5 days to work." 4. "You should reach the desired effect in 1-3 weeks."

4. Correct: Therapeutic effect is usually reached in one to three weeks, or longer. Encourage the client to continue taking the medication as prescribed. Provide supportive care and reassurance during this time. 1. Incorrect: This response demonstrates that the nurse is not familiar with the time for therapeutic onset. This response would discourage the client. 2. Incorrect: It is too soon to determine if treatment should be changed. It may take several weeks to reach therapeutic effects. 3. Incorrect: While some clients may be more calm within a short period of time, therapeutic effect cannot be evaluated at this point. Initial effects may be seen in as little as 1-3 weeks, while full therapeutic effects may fake up to 4-6 weeks.

A fully alert and competent client is in end-stage cardiac disease. The client says, "I'm ready to die," and refuses to take nourishment. The family urges the client to allow the nurse to insert a feeding tube. What action should the nurse take? 1. Tell the family that the feeding tube will be inserted after the client becomes unresponsive. 2. Ask the primary healthcare provider to have the dietician talk with the client about food preferences. 3. Notify the case manager to arrange a meeting with the client's family . 4. Provide additional information as requested by the client concerning nourishment.

4. Correct: This client is alert and competent, and has the right to make healthcare decisions and the right to die with dignity. The nurse should provide any additional information as requested by the client. 1. Incorrect: This is inappropriate, as it does not follow the client's wishes and would be a violation of client rights. 2. Incorrect: The client has made the decision to refuse nourishment so this action ignores this decision and violates client rights. 3. Incorrect: The nurse should honor the client's wishes first. The family would only need to meet if the client became unable to make decisions on their own. Even so, these decisions could not violate any advance directives that were in place.

After making rounds on clients, a primary healthcare provider hands the nurse a client record and gives the following verbal order: Administer cisplatin 1 mg IV over 6 hours. What should be the first action by the nurse following this verbal prescription? 1. Call the pharmacy to prepare the drug. 2. Repeat the prescription back to the primary healthcare provider. 3. Ask the primary healthcare provider to spell the drug name for clarification. 4. Inform the healthcare provider that this medication requires a written prescription.

4. Correct: This drug is a high alert drug that should be given careful consideration. Verbal orders for antineoplastic agents should NOT be permitted under any circumstances. These medications are not administered in emergency or urgent situations, and they have a narrow margin of safety. 1. Incorrect: The pharmacy should not be called to prepare this drug as it is unsafe to follow a verbal prescription for an antineoplastic. Verbal orders for antineoplastic agents should NOT be permitted under any circumstances. These medications are not administered in emergency or urgent situations, and they have a narrow margin of safety. 2. Incorrect: The first action by the nurse should be to inform the primary healthcare provider that a verbal prescription is not adequate for this particular category of drug. Verbal orders for antineoplastic agents should NOT be permitted under any circumstances. These medications are not administered in emergency or urgent situations, and they have a narrow margin of safety. 3. Incorrect: Use the testing strategy of finding similar options to eliminate incorrect answers. Options 1, 2 and 3 insinuate the nurse is going to proceed with the prescription, which is an unsafe practice for antineoplastics. Verbal orders for antineoplastic agents should NOT be permitted under any circumstances. These medications are not administered in emergency or urgent situations, and they have a narrow margin of safety.

A client with a terminal illness, asks the nurse about palliative care. What would be the nurse's best response? 1. Palliative care is a holistic way of finding a cure for a serious illness. 2. Palliative care begins when the client has 3 months or less to live. 3. Palliative care will require you to change to a palliative care healthcare provider. 4. Palliative care prevents and treats symptoms and side effects of disease and treatments.

4. Correct: This is a correct statement. The goal of palliative care is to help the client living with a chronic, life threatening illness. It focuses on the client's symptoms and the relief of these symptoms. Palliative care helps the client obtain their best quality of life throughout the course of their illness. 1. Incorrect: Palliative care is not aimed at cure. It is provided to clients who have chronic, life threatening illnesses. 2. Incorrect: Palliative care can begin at diagnosis. Hospice care is usually offered when the person has 6-12 months or less to live. 3. Incorrect: The client does not need to give up his or her primary healthcare provider. This is not a requirement of palliative care.

After a heart catheterization a client reports severe foot pain on the side of the femoral insertion site. The nurse notes pulselessness, pallor, and a cold extremity. What should be the nurse's first action? 1. Administer an anticoagulant. 2. Warm the room. 3. Increase intravenous fluids. 4. Notify the primary healthcare provider.

4. Correct: This is an emergency and the primary healthcare provider (PHCP) is the only one that can save this foot from ischemia. Don't delay. 1. Incorrect: The primary healthcare provider may treat with an embolectomy, bypass surgery or a thrombolytic. Giving an anticoagulant could alter treatment options. 2. Incorrect: The cold extremity is caused by decreased arterial perfusion, not room temperature. 3. Incorrect: In theory, increasing blood volume increases blood flow, but this client has an arterial obstruction. Blood cannot get passed the occlusion.

A nurse is triaging a 2 year old child in the pediatric emergency department. The nurse notes that the child will not lie down and is consistently drooling. A croaking sound is heard on inspiration. What is the priority nursing intervention? 1. Examine the oral pharynx using a tongue depressor. 2. Administer a sedative so the child can be examined. 3. Have a second nurse hold the child down for the assessment. 4. Notify the primary healthcare provider immediately.

4. Correct: This is the safest answer. The child could suddenly obstruct the airway upon examination of throat. 1. Incorrect: If it looks like epiglottitis, do not examine as this could cause sudden airway obstruction which could be fatal. 2. Incorrect: The client is having trouble breathing, so do not sedate the client. Sedatives would depress the respirations more and potentially cause the client to go into respiratory arrest. Remember, the NCLEX® lady does not want you to be a killer nurse. 3. Incorrect: This will cause more respiratory and emotional distress to the child. This is an unsafe answer.

A psychiatric client tells the day shift nurse, "The night nurses have been stealing from all of us while we are sleeping." What is the nurse's best response? 1. "Can you prove what the nurses are stealing?" 2. "No nurse working here would steal." 3. "You must have misunderstood what you were seeing." 4. "Tell me more about what you saw."

4. Correct: This open ended question allows for exploring the idea the client has. This statement does not accuse anyone or deny the possibility of stealing. This statement allows the nurse to remain nonthreatening and nonjudgmental. 1. Incorrect: This response shows disapproval. This statement could make the client feel uncomfortable and seem like the nurse is taking the side of the other nurses. 2. Incorrect: This response is disagreeing with the client. This is a closed-ended statement that does not allow the client to discuss this topic further. Since the nurse has said no nurse would steal then the client most likely will become defensive. 3. Incorrect: This response is defending. It also makes the client feel that the nurse does not believe them.

The nurse is caring for a client prescribed ondansetron due to postoperative nausea. Which side effect is the nurse most worried about the client experiencing with administration of this medication? 1. Respiratory depression 2. Hyperglycemia 3. Malignant hypertension 4. Torsades de pointes

4. Correct: Torsades de pointes is a life threatening dysrhythmia which can occur with administration of ondansetron. Clients who are at increased risk for Torsades de pointes are those with underlying heart conditions and those with hypomagnesemia or hypokalemia. 1. Incorrect: Respiratory depression is not a common side effect of ondansetron. Headache and drowsiness are more common. 2. Incorrect: Hyperglycemia is also not a side effect of ondansetron. Hyperglycemia is high blood sugar and may produce symptoms of urinary frequency, increased thirst and increased appetite. Hyperglycemia is not related to ondansetron. 3. Incorrect: Malignant hypertension is extremely high blood pressure that develops rapidly and causes some type of organ damage. Although it is a serious condition this is not a side effect of ondansetron.

A renal transplant client has received discharge education. Which statement by the client indicates that further teaching is necessary? 1. "I will need to notify my primary healthcare provider if I develop a fever." 2. "I need to check my BP daily and report an increased B/P." 3. "I will tell my primary healthcare provider if I become easily fatigued." 4. "I will be on steroids for 3 months, then I will not have to take them."

4. Correct: This statement indicates a need for further teaching. Doses of immunosupressive agents are often adjusted, but the client will be required to take some form of immunosuppressive therapy for the entire time that the client has the transplanted kidney. 1. Incorrect: Yes, fever is a sign of post transplant rejection. Additional signs and symptoms include oliguria, edema, increasing blood pressure, weight gain and swelling or tenderness over the transplanted kidney, flu-like symptoms, shortness of breath and/or fatigue. 2. Incorrect: Yes, hypertension is a sign of post transplant rejection. Additional signs and symptoms include oliguria, edema, increasing blood pressure, weight gain and swelling or tenderness over the transplanted kidney, flu-like symptoms, shortness of breath and/or fatigue. 3. Incorrect: Yes, fatigue is a sign of post transplant rejection. Additional signs and symptoms include oliguria, edema, increasing blood pressure, weight gain and swelling or tenderness over the transplanted kidney, flu-like symptoms, shortness of breath. "Transplant" is important because the client will be taking anti-rejection medication. What will this do to the immune system? Decrease it. Option 1 is true as fever indicates infection. Option 2 is true, because hypertension is a sign of rejection. Option 3 is true, because fatigue is also a sign of rejection. Option 4 is false. Steroids are not stopped abruptly.

A mother of a newborn is crying and tells the nurse, "I am worried about my baby. His Apgar score was 6 and the nurses had to help him breath for a while." What response should the nurse make to this mother? 1. "Don't worry about what score your baby received on the Apgar. The nurses know how to take care of him." 2. "Stop crying. Your baby is fine now and will continue to get stronger as the day progresses." 3. "Your baby's Apgar score was normal. The score was 6 at 1 minute which is typical." 4. "It is normal for you to feel this way. Let me explain what the Apgar score is used for."

4. Correct: This statement recognizes the mother's feelings and seeks to educate. Providing relevant information may decrease her anxiety and encourage further communication. 1. Incorrect: This statement belittles the mother's feelings and communicates that the nurse is not taking her concerns seriously. 2. Incorrect: This is nontheraputic and will discourage further expression of feelings. This response also gives false reassurance because the nurse does not know if the baby will continue to improve. 3. Incorrect: An Apgar score of 7 to 10 indicates a newborn in good condition. An Apgar score of 6 is not normal and indicates the need for interventions.

A charge nurse receives a report of back discomfort after work every day from a surgical nurse. Which action by the surgical nurse should be addressed by the charge nurse? 1. Frequently shifting weight from one foot to the other. 2. Standing straight with knees slightly bent. 3. Raising work station to waist level. 4. Twisting at the waist to reach for an object during cleanup.

4. Correct: To reduce back injuries at work, it is important to find and avoid risk factors that increase the chance of injury. When any of the following occur in combination your risk of a back injury is increased: 1. Awkward posture 2. Overexertion 3. Repetition 4. Fatigue Your body posture determines which joints and muscles are used and the amount of force that is generated. Whether standing or sitting, there is a neutral position for the back. Postures that differ from the neutral position increase stress on the back, especially when combined with other risk factors. Avoid strenuous activity while the body is in a twisted or bent position, repeatedly bending, twisting and reaching, or bending forward while lifting.​ 1. Incorrect: The nurse should be alternating their weight frequently during surgery. Alternating weight from one foot to the other increases circulation and avoids strain on the back. Alternate tasks and postures that use different motions and muscles groups, such as sitting and standing. Take time to stretch during scheduled breaks. Break standing tasks with seated.​ 2. Incorrect: This action will decrease the possibility of back injury. The knees should never be locked. Locking your knees restricts blood flow and is a sure fire way to make yourself pass out. 3. Incorrect: The nurse should change the work height so that items can be handled with the back in a neutral position. It will alleviate unnecessary bending.

What would be the best way for the nurse to evaluate the effectiveness of fluid resuscitation during the emergent phase of burn management? 1. Weight increases by 2 pounds in 24 hours 2. Urinary output is greater than fluid intake 3. Blood pressure is 90/60 mmHg 4. Urine output greater than 35mL/hour

4. Correct: Urine output of 30 to 50 mL/hour indicates adequate fluid replacement. 1. Incorrect: May indicate fluid retention. 2. Incorrect: Does not indicate fluid balance. 3. Incorrect: Blood pressure alone does not indicate adequate fluid balance. 1. Look at each option as True or False. 2. Option 1 is false. The emergent phase is the first 24 hours post burn. Timing is one of the most important considerations in calculating fluid needs in the first 24 hours. While body weight is closely monitored, weight gain may indicate fluid retention. 3. Option 2 is false. Intake and urine output are assessed hourly, but urine output is the best indicator and is used to evaluate fluid resuscitation. 4. Option 3 is false. Vital signs are monitored closely. A blood pressure of 90/60 mmHg could indicate a hypovolemic shock. 5. Option 4 is true. For adults, a urine output of 30 to 50 mL/hour is used as an indication of appropriate resuscitation in thermal and chemical burns. A urine output of 75 to 100 mL/ hour is the goal with electrical burns. An indwelling catheter is inserted to permit accurate monitoring of urine output as a measure of kidney function and fluid needs for the client.

A client admitted to the mental health unit for a suicidal attempt has been progressing slowly in treatment. Suddenly, the client has voiced a much more positive outlook and tells the nurse "I am going to be fine now." What is significant about this situation? 1. The nurse should expect that the treatment has been effective. 2. The client is developing a more positive outlook. 3. The client sees hope for the future. 4. The client may have decided to kill himself.

4. Correct: When a depressed mood suddenly lifts, the client may have decided to kill himself. One would normally expect an incremental return to a positive mood with treatment. This client would need close monitoring to prevent another suicide attempt. 1. Incorrect: The effective treatment plan results in a gradual improvement in mood and energy level, unless the client is suicidal. 2. Incorrect: The client will demonstrate a positive outlook with treatment over the course of a few weeks, if he is not suicidal. 3. Incorrect: Hopeful comments should emerge more gradually, unless the client is suicidal.

A client asks the nurse, "How is relaxation therapy going to help reduce my stress?" What would be the nurse's best response? 1. Relaxation therapy leads to more awareness of potential stressors 2. Relaxation therapy reduces stress by releasing small doses of epinephrine into the body. 3. Stress can be eliminated from your life when you use this therapy. 4. Relaxation therapy can counteract the flight or fight response.

4. Correct: When stress overwhelms the nervous system, the body is flooded with chemicals that prepare for fight or flight. To counteract this stress response, relaxation techniques can be used. 1. Incorrect: Identifying stressors is part of the solution. The relaxation techniques themselves bring about the counter production of the stress response. 2. Incorrect: Epinephrine is released during stress and increases anxiety. Relaxation therapy counteracts symptoms of stress, such as increasing heart rate and respiratory rate, increased blood pressure, pupil dilation, and increased metabolism. 3. Incorrect: The goal of relaxation therapy is not to improve the stress response of fight or flight but to reduce or counteract the response. Relaxation therapy can help reduce the fight or flight response before it becomes severe.

The nurse is admitting an 8 month old infant to the pediatric unit. For what major developmental stressor in this infant should the nurse plan interventions? 1. Fear of unknown 2. Loss of daily routine 3. Body image disturbance 4. Separation anxiety

4. Correct: Yes, they are afraid of being without the caregiver. Separation anxiety develops after a child gains an understanding of object permanence. Once your infant realizes the parent is gone, it may leave him unsettled. Although some babies display object permanence and separation anxiety as early as 4 to 5 months of age, most develop more robust separation anxiety at around 8 months. Separation anxiety can be worse if the infant is hungry, tired, or not feeling well. Keep the good-bye short and sweet. 1. Incorrect: Fear of the unknown is not a concern at this age, but rather between the age of 2-3 years. Separation anxiety is their immediate concern. 2. Incorrect: Keeping family routines and providing quality time with trusted adults is reassuring once the child reaches the age of 2. 3. Incorrect: The preschooler fears mutilation resulting in body image disturbance.

The nurse is caring for a post op client who is drowsy but arousable. The client will take a few deep breaths when instructed but drifts to sleep when left alone. The O2 saturation while sleeping drops to 82% on 3 liters of nasal oxygen. The client received a dose of oxycodone/acetaminophen 2 tabs one hour ago. What is the nurse's best action at this time? 1. Keep the O2 sat machine at the bedside and set the alarm to beep loudly when O2 sat drops below 93%. 2. Give bath to arouse client and then report that oxycodone/acetaminophen 2 tabs is too much for next dose. 3. Let the client sleep until he has rested, then discuss abuse potential of narcotics. 4. Call the primary healthcare provider and report client assessment findings.

4. Correct: Yes, this client has unstable respirations and is in respiratory distress. The client needs naloxone,the antidote for narcotic overdose. Since that is not an option, you need to call the primary healthcare provider to get a prescription for the antidote. 1. Incorrect: That will work the first time, but the client is too sedated to remain awake and take deep breaths. The client will continue to have respiratory distress until naloxone can be given. 2. Incorrect: No, that won't fix the problem of too much medication. We need to fix the problem now. 3. Incorrect: No, client is too sedated. Naloxone is needed, so the nurse needs to notify the primary healthcare provider.

The client with obsessive-compulsive disorder (OCD) asks the nurse for help with a repetitive behavior. What is the most likely origin of this behavior? 1. Fear 2. Depression 3. Delusions 4. Anxiety

4. Correct: Yes, this is how they deal with anxiety. The obsession causes the anxiety such as a thought that can't be dismissed from the mind. The ritualistic behavior that the client is driven to perform is an attempt to reduce anxiety. The compulsive act temporarily reduces high levels of anxiety. 1. Incorrect: No, is phobia that deals with fears. A phobia is an excessive and irrational fear reaction. If you have a phobia you may experience a deep sense of dread or panic when you encounter the source of your fear. 2. Incorrect: OCD is not about depression. Depression is a mood disorder that causes persistent feelings of sadness and loss of interest. Not the origin of OCD. 3. Incorrect: Delusions are not associated with OCD. Delusions are most often defined as false fixed beliefs that cannot be corrected by reasoning.

The night nurse on a step down unit suspects another nurse may be intoxicated. What initial action should the nurse take? 1. Ask another nurse to confirm suspicions. 2. Call supervisor to report the intoxication. 3. Confront the nurse privately in person. 4. Discuss suspicions with unit nurse manager.

4. Correct:The greatest concern at this time is the safety of the clients to whom the intoxicated nurse is providing care. The nurses Code of Ethics dictates safe, effective care for the public with protection from incompetent or unethical practice. The chain of command for this floor nurse is to report directly to the unit nurse manager. 1. Incorrect: When dealing with ethical or legal issues, the chain of command starts with the nurse manager of the unit in question. Asking another staff nurse for a personal opinion would not provide any pertinent data and instead amounts to gossip. 2. Incorrect: In order to avoid undue conflict, the nurse needs to immediately alert the unit nurse manager and not the facility supervisor. The nurse manager must then manage any conflict that may result and bears the responsibility to control possible disruption resulting from re-assigning the impaired nurse's clients. 3. Incorrect: Direct confrontation of the allegedly impaired nurse would most likely result in denial or defensive behaviors which could place the clients at further risk. The chain of command for this staff nurse starts with the unit nurse manager who would be more qualified to deal with conflict resolution in this matter.

A client newly diagnosed with Celiac disease is being instructed on a gluten-free diet. What statement by the client would indicate to the nurse that further teaching is needed? 1. "I will still have occasional abdominal discomfort." 2. "I may need to take iron or vitamin supplements." 3. "I can have eggs but no wheat toast for breakfast." 4. "I should avoid fresh apples and strawberries."

4. Correct:The nurse is evaluating client statements for any lack of understanding and the need to provide further instruction. With Celiac disease, intestinal villi become inflamed whenever gluten is introduced to the gut through food intake. However, fresh fruits and vegetables do not contain gluten; therefore, fresh apples and strawberries would definitely be acceptable foods for this client. This statement by the client is inaccurate, indicating the need for further explanation by the nurse. 1. Incorrect:The client correctly acknowledges that some episodes of abdominal discomfort may still occur, since it is nearly impossible to totally eliminate gluten. Despite buying "gluten-free" products, occasionally small amounts of gluten may contaminate foods and causing symptoms to resurface. Eating in a restaurant may also be a challenge for those with Celiac disease. Because the client recognizes these possible symptoms, teaching was successful. However, the question asks for evidence the client needs further instruction. 2. Incorrect: This is an accurate statement by the client about Celiac disease. Because inflammation of the intestinal villi may lead to poor absorption of nutrients or anemia, clients may indeed need to take supplements for extended periods of time. This response does not indicate any problems with the client's comprehension of teaching. 3. Incorrect: It is important for a Celiac client to eat as healthy and diverse a diet as possible, since malnutrition occurs secondary to poor nutrient absorption in the bowel. Protein is a vital component in the diet, including such choices as eggs, dairy and beans. Those foods creating the worst symptoms include grains like wheat, rye, and barley as well as the "malt barley" used as a thickening agent in certain products. The client has precisely stated that a breakfast including eggs but minus the wheat toast would be appropriate, evidence that teaching was successful.

A client with a history of syncope and transient arrhythmias has been ordered a Holter monitor for 48 hours. The nurse knows that teaching has been effective when the client makes what statement? 1. No follow up care will be needed after the monitor is removed. 2. It is okay to shower or bath while wearing this equipment. 3. I have to take it easy and not exercise for the next two days. 4. It's important to write down all my activities during this time.

4. Correct:The purpose of the Holter monitor is to detect cardiac irregularities over an extended period of time, in this case 48 hours. Although the monitor will record heart rate and rhythm for two days, it is vital for the client to keep a log or diary during that time, indicating the precise time and type of every activity. Additionally, this log needs to indicate any chest pain or palpitations the client experiences during that time, to assist the primary healthcare provider in diagnosing cardiac dysfunctions. 1. Incorrect: A Holter monitor is a mobile diagnostic test utilized by the cardiologist to help determine a cause for this client's syncopal episodes or arrhythmias. Once the client has the monitor and electrodes removed, the primary healthcare provider will analyze the data before meeting with the client to discuss the findings. Regardless of any suggested treatment options, the cardiologist needs a follow up visit with the client. 2. Incorrect: Showering or tub bathing is not permitted while wearing the Holter monitor as this may interfere with the functioning of the equipment. Only a careful sponge bath is permitted. Clients are also instructed to avoid heavy machinery, electric razors, microwave ovens and even hair dryers since can also affect accuracy and performance of the monitor. 3. Incorrect: The purpose of wearing Holter monitor for 24-48 hours is to diagnose cardiac arrhythmias during ADL's or exercise. The client cannot remove the monitor at any time during that period since that would cause inaccurate readings, or even the loss of valuable data. The client is instructed to complete all routine daily activities during that time, including work or exercise, to help identify actions that contribute to the symptoms or cardiac irregularities.

The nurse is caring for a client who is wheezing and struggling to breathe. Which inhaled medications might be indicated at this time? 1. Fluticasone 2. Salmeterol 3. Theophylline 4. Albuterol 5. Levalbuterol

4., & 5. Correct: Albuterol and levalbuterol are both rapid acting bronchodilators, that will quickly relieve shortness of breath, chest tightness and wheezing. This client is in distress now. Either medication would be indicated. 1. Incorrect: Fluticasone is a corticosteroid that is used regularly to receive the most benefit. It does not work immediately but may take 12 hours to several days to get the full benefit. Steroid use is for control of symptoms. This client is having symptoms now. 2. Incorrect: Long acting bronchodilators are not for use in an emergency. Salmeterol is an inhaled corticosteroid. It will not stop an asthma attack or breathing problem once it has begun. 3. Incorrect: Theophylline is inexpensive but it is often not utilized as a first line treatment. Takes a long time for this to work, and its purpose is to prevent frequency of attacks, not for emergency use.

The nurse is reviewing the plan of care for a client during the first day post-craniotomy. Which actions can the nurse delegate to an experienced LPN/LVN working in the ICU? 1. Determine Glasgow Coma Score. 2. Check endotracheal tube (ET) cuff pressure every shift. 3. Reposition client from side to side every 2 hours. 4. Administer acetaminophen via nasogastric tube for temperature greater than 101ºF (38.3ºC). 5. Monitor intake and output every hour.

4., & 5. Correct: Both of these actions are within the scope of practice for the LPN/LVN. 1. Incorrect: Assessing the Glasgow Coma Score should be done by the RN. 2. Incorrect: ET tube cuff assessment is accomplished by an experienced RN. 3. Incorrect: Usually, repositioning a client would be within the scope of practice for the LPN/LVN; however, this client is at risk for increased ICP during position changes. The RN must monitor. First of all, this question is talking about caring for a client in ICU who is day one post craniotomy. Is this a critical client? Yes. So the LPN will not be providing any care alone. What can the LPN do? Certain safe tasks that would not require the RN to perform. Well you know that only the RN can assess, evaluate, educate, and plan care. So based on this knowledge, which options can you eliminate? Options 1, and 2 can be eliminated. Assessing the Glasgow Coma Score should be done by the RN. ET tube cuff assessment is accomplished by an experienced RN. This is beyond the scope of practice for the LPN. What about option 3. Can the LPN reposition a client every 2 hours? Generally, yes. But what do we know about a client with a head injury or surgery? The client's ICP can increase with position changes! So should the LPN reposition this client? No. The RN needs to do this and assess the ICP after the position change to see if the client is tolerating the position or not. Option 4. Can the LPN give medication via a nasogastric tube? Yes, this is within the scope of practice of the LPN, so this task can be delegated to the LPN to perform. Option 5. Can the LPN monitor I&O? Yes. The LPN can monitor and report changes in the I&O status to the RN. The RN will then be responsible for planning care based on the results.

What discharge instructions should the nurse provide to the parents of a child diagnosed with sickle cell anemia? 1. Provide high-calorie, low protein diet. 2. Inheritance is by autosomal dominate genes. 3. Restrict all activities for 3 months. 4. Deferasirox helps prevent liver damage from iron deposits. 5. Avoid high altitudes.

4., & 5. Correct: Deferasirox is an orally administered iron chelation agent shown to reduce the liver iron concentration due to repeated RBC transfusions. It binds iron. Low oxygen environments such as airplanes and high altitudes should be avoided. 1. Incorrect: Provide a diet that is high-calorie and high-protein to promote growth and health. 2. Incorrect: Sickle cell anemia is an autosomal recessive disease. 3. Incorrect: Activities are not generally restricted. The client may not be able to tolerate vigorous exercise or exertion. Encourage children to participate in physical activities.

A client is admitted to the emergency department following a motor vehicle accident (MVA). The client reports abdominal discomfort, weakness, and nausea. Vital signs: BP 88/52, HR 118/min, RR 24/ min. Which tasks can the nurse delegate to an unlicensed assistive personnel (UAP)? 1. Obtain a baseline weight to guide therapy. 2. Administer an antiemetic suppository. 3. Assess client's abdominal girth. 4. Measure urinary output every hour. 5. Obtain vital signs every 15 minutes.

4., & 5. Correct: It is within the UAP ability to measure UOP hourly and obtain vital signs every 15 minutes. The nurse needs to make sure the UAP knows reportable values. 1. Incorrect: The RN would need to make sure of the client's weight to guide therapy. Emergency medications are often given based on weight. So the weight would need to be accurate. 2. Incorrect: The UAP cannot administer any medications. 3. Incorrect: The UAP cannot assess abdominal girth.

The nurse is preparing to teach a client about post percutaneous transluminal coronary angioplasty (PTCA) care. Which teaching points should the nurse include? 1. Restricting oral fluids until the gag reflex has returned. 2. Encouraging early ambulation and deep breathing exercises. 3. Discontinuing medicines following percutaneous intervention. 4. Reporting any chest discomfort following percutaneous intervention. 5. Avoid lifting more than 10 pounds until approved by healthcare provider.

4., & 5. Correct: The number one thing you are "worried" about post PTCA is re-occlusion or re-infarction, so report chest discomfort at once. Lifting more than 10 pounds can make the client bleed and would be contraindicated until cleared by the primary healthcare provider. 1. Incorrect: Fluids need to be increased to flush the dye used during the procedure from the kidneys. Oral fluids do not have to be restricted because the client does not have to be intubated for the procedure. 2. Incorrect: To ensure a stable clot is formed at the femoral access site, the client must remain on bed rest for a minimum of 4 hours. The client is at risk for hemorrhaging at the insertion site. DO NOT ambulate until it is certain that the clot is stable. 3. Incorrect: Medications are generally continued as before the procedure. Certain medications, like anticoagulants, may be held prior to the procedure, but typically all pre-procedure medications are resumed after PTCA. Before we review the options, let's look at the question. The key words in a question should be identified. The key words in this question are teaching points and PTCA. This is a select all question so there will be 2 or more options correct. Also each option stands alone with the question. After reviewing the question, look at each option and identify if it is true or false. Remember client safety is always a priority. This question is asking which teaching points should be implemented for a post PTCA. So let's look at the options. Option 1 is false. Fluids need to be increased to flush the dye used during the procedure from the kidneys. Oral fluids do not have to be restricted because the client did not have to be intubated for the procedure. The client should also not exhibit any issues with the gag reflex due to not being intubated. Option 2 is false .To ensure a stable clot is formed at the femoral access site, the client must remain on bed rest for a minimum of 4 hours. The client is at risk for hemorrhaging at the insertion site. DO NOT ambulate until it is certain that the clot is stable.

The nurse is planning care for a client admitted with Alzheimer's Disease. What interventions can the nurse delegate to the LPN/VN? 1. Teach caregivers memory enhancement aids. 2. Evaluate client's safety risk factors. 3. Make referrals to community services. 4. Determine caregiver's stress level and coping strategies. 5. Monitor for behavioral changes. 6. Check environment for potential safety hazards.

5. & 6. Correct: The LPN/VN can monitor for behavioral changes and can look for potential safety hazards. 1. Incorrect: The RN is responsible for teaching. This task cannot be delegated to the LPN/N. The LPN/VN can reinforce teaching. 2. Incorrect: The RN is responsible for assessment and evaluation of clients. The LPN/VN can gather data, but the RN is responsible for validating and interpreting that data to assess and evaluate. 3. Incorrect:The RN is responsible for developing the plan of care which would include necessary referrals. 4. Incorrect: This again is assessment which is the role of the RN only. If you look at the NCLEX-PN detailed test-plan, then you know that behavior management is one component of psychosocial integrity that the PN is responsible for ensuring. Specifically, the PN should monitor client appearance, mood, and psychomotor behavior and observe for changes. Under the safety and infection control client needs category, it states that the PN should monitor the client care environment for safety hazards and report problems to appropriate personnel. What options match? Options 5 and 6, right? Yes. What is wrong with option 1? The RN is responsible for teaching. Option 2? Assessment and evaluation are parts of the nursing process done by the RN only. Option 3? The RN makes referrals since that is part of developing the plan of care.

A client admitted with heart failure is experiencing severe shortness of breath and states, "I feel like something is terribly wrong!" The client is restless and begins to cough up large amounts of pink frothy sputum. The client's skin is a dusky grayish color and the oxygen saturation levels have decreased from 92% to 76% in the last hour. What is the first action the nurse should take? Check vital signs Administer the PRN ordered oxygen Call the health care provider Place the bed in high Fowler's position

Administer the PRN ordered oxygen When dealing with a medical emergency, the rule is to assess airway first, then breathing, and then circulation. Starting oxygen is the priority. The other actions should also be implemented as quickly as possible, including activation of the rapid response team. The client is experiencing an acute episode of fulminant pulmonary edema, likely as a result of a new and severe cardiac event and possible cardiogenic shock. Emergency assessment and intervention is indicated to prevent cardiac arrest and possible death.

A client who has been admitted to the intensive care unit with malignant hypertension has been prescribed nitroprusside IV. BP on admit is 210/112. Weight - 56 kg. Based on the prescription, what should the flow rate for a volumetric pump be set at initially? Round to the whole number. *exhibit (prescriptions)* Titrate nitroprusside 50 mg in 250 mL D5W at 3 to 6 mcg/kg/min to maintain client's systolic blood pressure below 140 mm Hg.

Answer: 50 mL/hour Rationale: Convert to like units. Equivalent: 1,000 mcg = 1 mg Therefore 50 mg = 50,000 mcg Calculate the concentration of solution in mcg/mL. 50,000 mcg : 250 mL = x mcg : 1 mL 250x/250 = 50,000/250 X = 200 mcg/mL The concentration is 200 mcg/mL Always start with the lowest dosage when beginning nitropusside. 3 mcg/kg x 56 kg = 168 mcg/min Convert dosage range to mL/min 200 mcg : 1 mL = 168 mcg : x mL 200x/200 = 168/200 X = 0.84 mL/min Convert mL/min to mL/ hr. 0.84 mL x 60 min = 50.4 or 50 mL/hr.

Which action by the nurse administering intravenous ciprofloxacin would require intervention by the charge nurse? 1. Sets IV pump to administer ciprofloxacin over a period of 30 minutes. 2. Educates client that medication may cause dizziness. 3. Instructs client to notify nurse for any tendon pain. 4. Administers ciprofloxacin through 20 gauge catheter into the cephalic vein.

Correct: Cipro IV should be administered to by intravenous infusion over a period of 60 minutes. Slow infusion of a dilute solution into a larger vein will minimize client discomfort and reduce the risk of venous irritation. Incorrect: This action does not require intervention by the charge nurse as dizziness is a side effect of this medication. Incorrect: This is a correct action. Fluoroquinolones, including Cipro IV, are associated with an increased risk of tendinitis and tendon rupture in all ages. This adverse reaction most frequently involves the Achilles tendon, and rupture of the Achilles tendon may require surgical repair. Tendinitis and tendon rupture in the rotator cuff (the shoulder), the hand, the biceps, the thumb, and other tendon sites have also been reported. Incorrect: Slow infusion of a dilute solution into a larger vein will minimize client discomfort and reduce the risk of venous irritation.

The nurse is caring for a client who just had a central venous catheter line inserted at the bedside. Which of these assessments requires immediate attention by the nurse? Pallor in the extremities Increased temperature by one degree Involuntary coughing spells Dyspnea at rest

Dyspnea at rest Complications of central catheter insertion include pneumothorax and hemothorax. Air embolism is another potential complication. Dyspnea, shallow respirations, sudden sharp chest pain that worsens with coughing or deep breathing are indications of pneumothorax. Other potential complications of central catheters may include thrombosis, local or systemic infection, or even cardiac tamponade (if the central line perforates the heart). When considering the options listed, the client who is dyspneic after central line insertion would be the greatest concern for the nurse.

An 18 month-old weighing 22 pounds is admitted to the pediatric unit with a diagnosis of dehydration. A replacement bolus of normal saline at 20 mL/kg is ordered to be administered intravenously over 40 minutes.

Using ratio proportion: First, convert 22 pounds to kilograms (22/2.2) = 10 kg 20 mL/kg = 20 x 10 kg = 200 mL 200 mL/40 minutes = x mL/60 minutes (in an hour) 200 x 60 = 12000/40 = 300 mL/hr Using dimensional analysis: 20 mL/kg x 1 kg/2.2 lb x 22 lb x 60 min/hr x 1/40 min = 300 mL/hr

The nurse is to review the topic of caring for clients with Guillain-Barré syndrome with other staff members at a monthly meeting. Which of these findings should the nurse include in the discussion? (Select all that apply.) Weakness, tingling or loss of sensation in legs and feet occur first Rapidly progressive ascending paralysis of the legs, arms, respiratory muscles and face! Difficulty with bladder control or intestinal functions Hypertension Difficulty with eye movement, facial movement, speaking, chewing or swallowing Numbness, tingling, prickling sensation or moderate pain throughout the body

Weakness, tingling or loss of sensation in legs and feet occur first Rapidly progressive ascending paralysis of the legs, arms, respiratory muscles and face! Difficulty with bladder control or intestinal functions Difficulty with eye movement, facial movement, speaking, chewing or swallowing Numbness, tingling, prickling sensation or moderate pain throughout the body Guillian-Barré is an autoimmune disease. The symptoms of weakness or tingling sensation begins in the legs and progresses to the arms and upper body, resulting in almost complete paralysis. The client is often put on a ventilator during the worst part of the disease to assist breathing. The client may have low blood pressure or poor blood pressure control.

Place the steps in order that the nurse should take to administer a subcutaneous injection. remove the needle cap by pulling it straight off inject the needle and administer the medication cleanse site with antiseptic swab hold the syringe and pinch skin with non-dominant hand perform hand hygiene Dispose the syringe in the sharps container apply gloves and locate the injection site

perform hand hygiene apply gloves and locate the injection site cleanse site with antiseptic swab remove the needle cap by pulling it straight off hold the syringe and pinch skin with non-dominant hand inject the needle and administer the medication Dispose the syringe in the sharps container First perform hand hygiene. Then apply gloves and locate injection site using anatomical landmarks. Start at the center of the site and rotate outward in a circular direction to cleanse the site. Remove the needle cap by pulling the cap straight off. Next, hold the syringe and pinch the skin with nondominant hand. Inject the needle quickly then administer the medication slowly. Finally, dispose of the syringe in the sharps container.

A nurse is in the mall when a shopper suddenly becomes non-responsive. Taking an available automatic external defibrillator (AED) from the wall, the nurse would immediately initiate interventions in what order? press the shock button uncover the clients chest tell everyone to stand clear place pads on client's torso await analysis rhythm turn on the AED machine

uncover the clients chest place pads on client's torso await analysis rhythm turn on the AED machine tell everyone to stand clear press the shock button Initially, the AED should be turned on and the prompts followed. These prompts will either be visual or audio. The client must then be properly positioned and "prepped". You are aware that the client must be placed supine, although the question does not indicate what transpired prior to finding the client non-responsive. As a nurse, you know two things immediately - that electrode pads must be placed on bare skin, and that ANY client must have their dignity protected. So, the client's chest skin must be uncovered (#5) - and that you only expose the area necessary to apply the electrodes. Then the pads are placed in the correct positions (#4). Next, the AED should be allowed to analyze the client's heart rhythm (#6). The machine will determine if a shock needs to be delivered, or if CPR should be initiated/continued. Remember that the safety of the 'rescuers' is also vital; therefore, if the machine calls for a shock to be delivered, it is vital to 'clear' anybody in the vicinity (#1) prior to pushing the shock button (#3).

The client was admitted to CCU with a diagnosis of acute coronary syndrome. Continuous cardiac monitoring has been implemented. Which assessment finding is most significant? 1. Ventricular fibrillation 2. Ventricular tachycardia 3. Premature ventricular contractions 4. ST segment depression of 0.5 mm

1. Correct: V fib is the most common lethal dysrhythmia in the initial period following a myocardial infarction. 2. Incorrect: V tach is significant as it may occur prior to V fib. 3. Incorrect: PVCs may precede V fib. 4. Incorrect: ST segment depression of 1 mm or more signifies myocardial ischemia. Remember, acute coronary syndrome is an umbrella term for myocardial infarction and unstable angina. With acute coronary syndrome, this client is at risk for any of these arrhythmias however, one is most life threatening to the client. With ventricular fibrillation there is NO cardiac output, so the client is dead, unless the nurse can do something to reverse this arrhythmia. V-tach and frequent PVCs may lead to V-fib, so the nurse will worry about them, but the most life threatening is V-fib.

What should a nurse include when planning an educational program for a group of women on how to prevent a urinary tract infection (UTI)? 1. Increase daily intake to at least 9 cups (2160 mL) of water. 2. Urinate within one hour after sexual intercourse. 3. Help soothe the peritoneum using bubble baths. 4. Wipe from the anal area to the vaginal area after a bowel movement. 5. Void when the urge occurs.

1., & 5. Correct: Water helps to dilute urine and flush out bacteria. Do not hold urine. When the urge occurs, void to prevent accumulation of bacteria. 2. Incorrect: Urination should occur within 15 minutes after sexual intercourse. 3. Incorrect: Avoid bubble baths. They cause irritation and lead to UTIs. 4. Incorrect: After bowel movements, always wipe from the vaginal area toward the rectum. This helps prevent bacteria in the anal area from spreading to the vagina and urethra.

During the admission examination of a client diagnosed with acute pyelonephritis, what signs or symptoms would the nurse expect to find? 1. Dysuria 2. Costovertebral angle tenderness 3. Pale conjunctiva 4. Chills 5. Urinary frequency

1., 2., 4., & 5. Correct: The client with acute pyelonephritis, will often exhibit these signs/symptoms. 3. Incorrect: Pale conjunctiva is seen in chronic renal failure. Option 1: True. Dysuria is painful urination sometimes described as "burning". Dysuria is mors commonly caused by bacterial infections of the urinary tract. Option 2: True. CVA tenderness occurs because the tapping disturbs the inflamed tissue, causing pain. Option 3: False. Pale conjunctiva is seen in chronic renal failure since the kidneys are unable to stimulate erythropoietin. Option 4: True. High fever can result in chills. Option 5: True. Frequency commonly accompanies the dysuria associated with urinary tract infections.

The nurse is assessing a pregnant client who thinks she is in labor. Which statements by the client would alert the nurse that the client is likely experiencing true labor? 1. "The pains are in my back, and come around to my abdomen" 2. "Walking around seems to help the pain" 3. "Contractions are irregular" 4. "My contractions are coming every 3 minutes" 5. "Changing my position does not help with the pain intensity"

1., 4., & 5. Correct: All of these are signs of true labor. 2. Incorrect: True labor pain is not alleviated by walking. 3. Incorrect: Contractions associated with true labor occur at regular intervals.

A client has been admitted to the orthopedic surgical unit after repair of a fractured right arm and right leg due to a motor vehicle accident. Preoperative vital signs are heart rate 88/min, respiration 16/min, BP 132/86, Temp. 98.6° F (37° C). The unlicensed assistive personnel reports the 2 hour post op vital signs as heart rate 98/min, respirations 20/min, BP 138/88, Temp. 98.6° F (37° C). What are the appropriate nursing interventions? 1. Notify the primary healthcare provider of the change in vital signs. 2. Assess the client for pain using a pain scale. 3. Perform neurovascular check of distal extremities. 4. Instruct the unlicensed assistive personnel to take another blood pressure reading manually. 5. Reassess the vital signs in 15 minutes.

2. & 3. Correct: Acute pain is a nursing diagnosis that can be associated with this client's elevated BP and heart rate. Since pain is suspected always do a neurovascular check to assure that pain is not related to neurovascular damage. 1. Incorrect: All vital signs are within normal limits. 4. Incorrect: There is no problem with the blood pressure reading. 5. Incorrect: Reassessing vital signs in 15 minutes delays care. Find the problem and fix it.

What should the nurse monitor for when caring for a client receiving an IV of 1/2 Normal Saline at 100 mL/hr? 1. Hypertension 2. Fluid volume deficit 3. Hypernatremia 4. Pulmonary edema

2. Correct: 1/2 Normal Saline is a hypotonic solution. Monitor for cellular edema because the fluid is moving out to the cell which could lead to fluid volume deficit and decreased blood pressure. 1. Incorrect: Hypertension can occur with isotonic and hypertonic IV solutions. Hypotension can occur with hypotonic IV solutions such as 1/2 Normal Saline, 3. Incorrect: Hypernatremia can occur with isotonic and hypertonic sodium solutions. 4. Incorrect: This is a nursing alert for hypertonic IV solutions. You must identify 1/2 normal saline as a hypotonic solution. Hypotonic solutions go into the vascular space, hang out for a while to rehydrate the client, then moves into the cell. The cell uses it up for energy. So fluid is then leaving the vascular space, going to the cell. This means the nurse should monitor for cellular edema because the fluid is moving out to the cell which could lead to fluid volume deficit and decreased blood pressure.​

The nurse is caring for a client in the emergency department after a violent altercation with her husband. She describes increasingly violent episodes over the past 10 years. She says, "This is the last time he will hit me." Which response by the nurse demonstrates understanding of the violence cycle? 1. When you leave, you don't have to worry anymore. 2. You are at greatest risk when you leave. 3. That is the best decision you can make. 4. I am glad that you won't be hurt ever again.

2. Correct: Violence is likely to escalate and may become lethal when the spouse leaves the abusive partner. The risk of death or injury is highest at the time the abused person decides to leave the abusive relationship or shortly after leaving. 1. Incorrect: Just because the victim leaves does not guarantee that the abuser will not follow or find her. The threat of injury or death increases at the time the abused person leaves. This response is giving false reassurance to the abused person. 3. Incorrect: The client should be praised; however, there are risks with both leaving and staying. The client should be informed. The nurse should acknowledge the fear of staying in the relationship and guide the client to resources that can be used to help make informed decisions. 4. Incorrect: Leaving the home and the perpetrator do not guarantee cessation of violence. Again, this only provides false reassurance that the abuser will not find the client and inflict harm.

In order to maintain asepsis, what should the nurse teach the client on home peritoneal dialysis? 1. Drink only bottled water. 2. Cap Tenckhoff catheter when not in use. 3. Soak the dialysate in warm water. 4. Clean the arteriovenous fistula with hydrogen peroxide twice a day. 5. Wash around the catheter insertion site daily.

2., & 5. Correct: Capping the Tenckhoff catheter prevents dialysate leakage and bacterial invasion. Clean around insertion site to decrease risk of bacterial infection. 1. Incorrect: What does drinking bottled water have to do with it? Nothing 3. Incorrect: Soaking the bags of solution in warm water can introduce bacteria to the exterior of the bags of solution . 4. Incorrect: Do you have an arteriovenous fistula? Not with peritoneal dialysis. Option 1: False. Do you see the word "only"? This is an "always" or "never" type word. Does the client "only" have to drink bottled water? No. This does not maintain asepsis to prevent infection. Option 2: True. The Tenckhoff catheter is the catheter used for peritoneal dialysis. A sterile cap should be applied when not in use. Otherwise the tip of the catheter would touch non-sterile items. Option 3: False. Soaking the bags of solution in warm water can introduce bacteria to the exterior of the bags of solution and increase the chance of peritonitis. Option 4: False. AV fistulas are for hemodialysis, not peritoneal dialysis. Option 5: True. Doing this will decrease the chance of bacteria being introduced during dialysis.

Question: The labor nurse is assessing a client admitted in preterm labor. Which client finding would require a social service consult? 1. Very quiet and avoids eye contact. 2. Reports that she is not married. 3. Has injuries in various stages of healing. 4. Reports frequent arguments with her partner.

3. Correct: Injuries in various stages of healing indicate a pattern of abuse. Abuse not only harms the mother, but also increases the risk of fetal harm or death and preterm delivery. 1. Incorrect: While these signs may indicate abuse, cultural differences may explain her demeanor. 2. Incorrect: Single parenthood is not an indicator for referral to social services. 4. Incorrect: Frequent arguments with her partner are not an indicator for referral to social services. This could also be from other stressors in the client's life, emotional mood swings from hormone changes, or other factors unrelated to the pregnancy.

Which postpartum client should the nurse assign to the last private room in the Women's Health Center? 1. Had an abruption during delivery 22 hours ago 2. Had a boggy fundus five hours post-delivery 3. Was pre-eclamptic prior to delivery 30 hours ago 4. Delivered by c-section whose WBC count is 24,000/mm3 or 5-10 x 10^9​/L

3. Correct: This pre-eclamptic client delivered 30 hours ago. They are trying to make you think that everything is OKAY because they say AFTER delivery... they must have a private room because any stimulus can precipitate a seizure. 1. Incorrect: People who are at risk for bleeding and shock do not require private rooms. 2. Incorrect: Boggy fundus....doesn't have anything to do with a private room. 4. Incorrect: This is the one most people jump on.... They thought you would jump on this...all ladies who have had babies have elevated white counts post-delivery. Normal white count is 5,000-10,000/ mm3 or 5-10 x 109/L

A nurse educator has completed an educational program on interpreting arterial blood gases (ABGs). The educator recognizes that education was successful when a nurse selects which set of ABGs as compensated respiratory alkalosis? 1. pH - 7.4, PaCO2 - 40, HCO3 - 24 2. pH - 7.48, PaCO2 - 29, HCO3 - 22 3. pH - 7.44, PaCO2 - 30, HCO3 - 18 4. pH - 7.46, PaCO2 - 32, HCO3 - 20

3. Correct: This set of ABGs indicate compensated respiratory alkalosis. The pH is normal. Both the PaCO2 and Bicarb are abnormal, but the PaCO2 is low. The bicarb is low to get rid of base. Compensation has occurred. 1. Incorrect: These are normal ABGs. 2. Incorrect: This set of ABGs indicates respiratory alkalosis. Compensation has not occurred as the pH is still abnormal and the bicarb is still normal. 4. Incorrect: This set of ABGs indicates partially compensated respiratory alkalosis. The pH is still abnormal. Both the PaCO2 and HCO3 are abnormal. The pH matches the CO2 as both are alkalotic. The reduction in bicarb shows attempt to compensate. This question is asking you to identify compensated respiratory alkalosis. When ABGs are compensated, that means that the pH is back to normal. Which options have a normal pH between 7.35-7.45? Options 1 and 3. So the test taker can eliminate options 2 and 4 since the pH is not normal. Look at option 1. The pH is normal at 7.4. The PaCO2 of 40 (normal 35-45) is normal. The HCO3 of 24 (normal 22-26) is normal. Option 1 can be eliminated. So the answer is option 3. The pH of 7.44 is normal. The PaCO2 of 30 (normal 35-45) is alkalosis. The HCO3 of 18 (normal 22-26) is acidosis. So the PaCO2 matches the pH since 7.44 is on the alkaline side of normal. These ABGs indicate compensated respiratory alkalosis.

The nurse manager of an Alzheimer's unit as completed inservice education to new nursing staff regarding guidelines for dealing with dementia. Which identified guidelines by the new nursing staff indicates to the nurse manager that education was successful? 1. Use a firm touch to guide the client to a different location when needed. 2. Be persistent when getting the client to do something. 3. Provide simple directions using gestures or pictures. 4. Do not argue with the client. 5. Play memory games to decrease dementia. 6. Require participation in daily activities.

3., & 4. Correct: When a person is confused and has dementia, we need to communicate in a simple manner. Provide simple directions or instructions, short sentences, and gestures. Use pictures. Do not give instructions on multiple things. Do not argue, criticize, or correct the client. This can increase anxiety, agitation, and anger. 1. Incorrect: Use a gentle touch rather than a firm touch with these clients. You do not want to be confrontational or evoke fear in the client. 2. Incorrect: Be flexible. If one approach does not work, try another. 5. Incorrect: Avoid questions or topics that require extensive thought, memory, or words. This can increase anxiety, frustration, and agitation. 6. Incorrect: Do not require or force participation in activities or events. This can increase anxiety, frustration, and agitation.

What teaching should the nurse provide the client regarding prevention of deep vein thrombosis when traveling by plane for a long period of time? 1. Do not cross legs longer than 15 minutes at a time. 2. Get up and move around the plane every 4 hours. 3. Wear compression stockings while traveling. 4. Move legs frequently while sitting. 5. Avoid alcohol and coffee while traveling.

3., 4., & 5. Correct: Compression stockings put gentle pressure on the leg muscles. It is important for passengers to keep moving their legs to help the blood flow, even when waiting in the airport terminal. Alcohol and coffee contribute to thickened blood and increased risk for clot formation. 1. Incorrect: Do not cross legs at all. 2. Incorrect: The client should get up and move around at least every 2 hours. While walking the muscles of the legs squeeze the veins and move blood to the heart.

A client is being discharged with halo traction. What should the nurse teach the client and family about home management of this traction? 1. Showering is permitted. 2. Apply baby powder under the halo vest to prevent irritation. 3. Sleep in whatever position is found to be most comfortable. 4. Never pull on any part of the halo traction. 5. Clean around pins at least twice a day with a new q-tip for each pin site. 6. Driving is allowed after discharge.

3., 4., & 5. Correct: The client may sleep in whatever position is most comfortable. The placement of a rolled-up towel, or pillow, either under the neck, if on back, or under the cheek, if side lying, may be helpful. Never pull on any part of the halo traction. It can damage or loosen the traction. Pin care is done to prevent infection. Clean around pins at least twice daily with q-tips. Use a new q-tip for each pin site to decrease contamination from one pin site to another. Do not use ointments or antiseptics unless prescribed. 1. Incorrect: Client should never attempt a shower since there is no reliable way to keep vest liner dry. Take sponge baths or sit in a bathtub with about 2-3 inches (5.08-7.62 cm) of water. Use towels or plastic to keep vest from getting wet. 2. Incorrect: Do not use soaps, creams, lotions, or powders beneath the vest as these may irritate the skin. 6. Incorrect: Absolutely do not operate a motor vehicle until healthcare provider allow this activity: field of vision and movement is narrowed. The client is an impaired driver and could cause an accident.

Standard orders on the nurse's unit include an intravenous infusion of D5 1/4 NS 1000 mL with 20 mEq (20 mmol/L) potassium chloride to run at 100 mL per hour. This IV solution would be appropriate for which clients? 1. Client diagnosed with Addison's disease. 2. Client diagnosed with hypertension. 3. Client diagnosed with chronic renal failure. 4. Client diagnosed with Cushing's disease. 5. Client diagnosed with hypokalemia.

4. & 5. Correct: Clients with abdominal cramping, Cushing's disease, and hypokalemia are safe to receive normal saline with potassium chloride. 1. Incorrect: Clients with Addison's disease can have hyperkalemia if they experience an Addisonian crisis due to lack of aldosterone. When aldosterone is not secreted, sodium and water is released and potassium levels elevate in response to the hyponatremia. 2. Incorrect: Clients with hypertension need a hypotonic solution which will not increase the client's blood pressure. However, the client may or may not have hypokalemia. 3. Incorrect: Clients in chronic renal failure are retaining fluid and potassium. They do not need more potassium. Option 1: false. With Addison's disease the client does not have enough aldosterone, so the client loses too much sodium and water and retains too much potassium. This client needs an isotonic solution to keep fluid in the vascular space, but potassium is not needed. Normal saline is an isotonic solution which will go into the vascular space and stay there, increasing vascular volume. More volume means more pressure. Option 2: false. The client with hypertension can be hydrated with a hypotonic solution, such as D5 1/4 NS. There is no indication that the client needs potassium. Option 3: false. The client will chronic renal failure will have fluid volume retention and hyperkalemia due to poor renal perfusion and low UOP. This client does not need potassium. Option 4: true. The client with Cushing's has too much sodium and water and not enough potassium. The potassium is needed for this client. This hypotonic solution will go into the vascular space for hydration, but then will shift to the cells. Option 5: true. The client with a low potassium will need IV potassium.

What is most important for the nurse to do prior to initiating peritoneal dialysis? 1. Aspirate for placement. 2. Have the client void. 3. Irrigate the catheter for patency. 4. Warm the dialysate fluid.

4. Correct: The peritoneal fluid is inserted into the abdominal cavity. To promote the exchange of wastes and fluid through the peritoneal membrane, the peritoneal fluid should be warmed. This will promote vasodilation of the capillaries in the peritoneal cavity. 1. Incorrect: The peritoneal catheter should not be aspirated. This would not tell you anything and could irritate the peritoneal membrane. After the cover of the dialysate fluid is removed, the tubing should be connected to the peritoneal catheter. 2. Incorrect: This is not a bad choice, just not the most important. Voiding would make the client more comfortable during the procedure but will not affect the success. 3. Incorrect: It is not necessary to irrigate a peritoneal catheter because you are irrigating with the dialysate.

A client has been unable to eat due to protracted vomiting. Which alterations in the arterial blood gases would the nurse expect to find? 1. pH: 7.40, PaCO2: 38, HCO3: 23 2. pH: 7.33, PaCO2: 46, HCO3: 18 3. pH: 7.28, PaCO2: 48, HCO3: 29 4. pH: 7.46, PaCO2: 35, HCO3: 28

4. Correct: This client has metabolic alkalosis. 1. Incorrect: These are normal ABGs. 2. Incorrect: This is metabolic acidosis. 3. Incorrect: This is respiratory acidosis. Option 1 shows a normal pH at 7.4 so this is not the correct answer. Option 2 shows a low pH (acidosis) at 7.33 so this is not the correct answer. Option 3 shows a low pH (acidosis) at 7.28 so this is not the correct answer. Option 4 is the only option showing a high (alkalosis) pH at 7.46. If the client is vomiting, vomiting, vomiting....then that client is losing acid, acid, acid! Which means metabolic alkalosis with an elevated HCO3.

Which signs and symptoms would concern the nurse if assessed in a client post radical neck surgery? 1. Bradypnea 2. Flaccid muscle tone 3. Flushed, warm skin 4. Positive trousseau's sign 5. Dysphagia 6. Decreased deep tendon reflexes

4., & 5. Correct: A positive Trousseau's sign indicates that muscles are rigid and tight due to a low calcium level. Some of the parathyroids could have been removed resulting in hypocalcemia. Dysphagia may result from a rigid and tight esophagus. This may cause aspiration. 1. Incorrect: The respiratory rate will decrease with hypermagnesemia and hypercalcemia. 2. Incorrect: Weak, flaccid muscle tone is seen with hypercalcemia. 3. Incorrect: Flushed and warm skin would be seen with hypermagnesemia due to vasodilation. 6. Incorrect: Decreased deep tendon reflexes would occur with hypermagnesemia or hypercalcemia. Option 1: False. Calcium acts like a sedative, so if the calcium level were high, the client would exhibit bradypnea. Option 2: False. Because calcium acts like a sedative, too much calcium would cause flaccid muscle done. A low calcium would result in rigid, and tight muscles which could lead to seizures. Option 3: False. Vasodilation is seen with increased magnesium, which acts like a sedative. Magnesium causes vasodilation and flushed, warm skin. Option 4: True. A Trousseau's is performed by blowing up a BP cuff. It is a positive Trousseau's sign when the hand tremors when the cuff is inflated. This indicates that muscles are rigid and tight due to a low calcium level. This is something that you worry about post radical neck surgery. Some of the parathyroids could have been removed resulting in hypocalcemia. Option 5: True. Remember, to worry about rigid and tight muscles. The esophagus is a smooth muscle. The esophagus will become rigid and tight when serum calcium is low, leading to dysphagia, and aspiration.

A nurse is planning care for a newly admitted client diagnosed with acute nephrotic syndrome. What interventions should the nurse include in the plan of care? 1. Monitor triglyceride level 2. Assess hydration status 3. Educate client on a 3 gm sodium diet. 4. Auscultate lung sounds. 5. Monitor blood pressure every 4 hours. 6. Assess for venous thrombo-embolism (VTE).

1, 2, 4, 5, & 6. Correct: The liver increases the release of cholesterol and triglycerides while producing more needed albumin. The client could go into a fluid volume excess or fluid volume deficit depending on the severity of the disease. This client is at risk for heart failure, and pulmonary edema so the lungs should be auscultated and the blood pressure should be monitored every 4 hours. Without proteins, the blood can clot and put the client at risk for thrombosis or embolism. 3. Incorrect: The client should be on a low sodium diet (3 grams is too high). Carbohydrates are given liberally to provide energy. Option 1: True. The client is hypoalbuminemic, so the liver compensates by synthesizing more albumin. However, the liver also starts producing more cholesterol and triglycerides. So triglyceride, cholesterol, and albumin levels should be monitored. Option 2: True. Depending on the stage that the nephrotic syndrome is in, the client will go from fluid volume deficit to fluid volume excess. As the client loses albumin, fluids shift out of the vascular space to the tissue, so vascular volume goes down. Once the holes in the glomerulus start to shrink, then more albumin will stay in the vascular space. Albumin pulls fluid back into the vascular space, so all of this excess fluid (anasarca) enters the vascular space causing fluid volume excess. So hydration status must be monitored . Option 3: False. The client is extremely edematous, so they do not need a diet high in sodium. Remember, water follows that sodium. Option 4: True. With fluid volume excess, the client is at risk for pulmonary edema and heart failure. So blood backs up to the lungs, causing wet lung sounds. Option 5: True. While in fluid volume deficit, the blood pressure may be low. Less volume = less pressure. During fluid volume excess, the blood pressure could be high. More volume = more pressure. Option 6: True. The proteins that are leaking out of the vascular space also prevent clotting. Without these proteins, lots of little clots can form throughout the body.

Which signs or symptoms would a nurse expect to find in a client admitted to the hospital in the oliguric phase of acute kidney injury (AKI)? 1. Edema 2. Hypotension 3. Hyperkalemia 4. Decreased blood urea nitrogen 5. Metabolic acidosis

1, 3. & 5. Correct: Edema, fluid volume excess, hypertension will be seen in this phase of AKI. Oliguria reflects kidney damage so potassium, BUN and creatinine levels will increase. Metabolic acidosis occurs as well. 2. Incorrect: Due to the client retaining fluid, the nurse would see hypertension rather than hypotension. 4. Incorrect: Renal damage would cause the BUN to increase. Option 1: True. Edema occurs because fluid is leaking from the vascular space. Edema is typically seen around the eyes, in the legs, ankles, and feet. Option 2: False. Fluid volume excess causes hypertension. More volume = more pressure. Option 3: True. Hyperkalemia occurs because the number one way to get rid of potassium is through the kidneys. Option 4: False. When the kidneys are not functioning, BUN will go up. Option 5: True. Metabolic acidosis occurs because the kidneys are not able to excrete hydrogen or manage bicarb.

Which statements should the nurse include when teaching a client about osteomyelitis? 1. Osteomyelitis is a risk factor for people who have chronic illnesses. 2. Activity restriction is necessary to avoid stress on the affected bone. 3. Oral antibiotics must be taken for at least 3 to 6 weeks. 4. High calcium levels may occur so report muscle weakness, anorexia, nausea and vomiting. 5. Osteomyelitis requires subcutaneous administration of calcitonin to reverse the course of the disease.

1. & 2. Correct: Clients who are at high risk for osteomyelitis include those who are poorly nourished, elderly, or obese. Others at risk include those with impaired immune systems and chronic illnesses such as diabetes and rheumatoid arthritis. Treatment regimens restrict activity. The bone is weakened by the infective process and must be protected by immobilization devices and by avoidance of stress on the bone. 3. Incorrect: IV antibiotic therapy is provided for a period of 3-6 weeks with around the clock dosing to maintain a high therapeutic blood level. 4. Incorrect: Hypercalcemia is a complication associated with bone cancer, not osteomyelitis. 5. Incorrect: Calcitonin is used for Paget's disease. A disorder of localized rapid bone turnover, most commonly affecting the skull, femur, tibia, pelvic bones, and vertebrae.

A client is being treated for fluid volume deficit with D5W, oral hydration, and management of viral symptoms. Which client data would indicate to the nurse that treatment has been successful? 1. BP 104/70 lying; 94/68 standing. 2. Moist mucous membranes. 3. Skin turgor recoil below clavicle is 3 seconds. 4. Urine specific gravity of 1.032 5. Serum sodium 152 mEq (152 mmol/L)

1. & 2. Correct: These BP readings are within normal limits. 3. Incorrect: Skin recoil delayed for more than 2 seconds indicates severe dehydration. 4. Incorrect: With fluid volume deficit, the specific gravity can be expected to be abnormally high. 5. Incorrect: This indicates hypernatremia, which is the same thing as dehydration.. Option 1: True. At first glance, the test taker may think that this is false because orthostatic hypotension occurs when blood volume is low. However, these BP readings do not indicate orthostatic hypotension. A drop of > 20 mm Hg systolic, 10 mm Hg diastolic, or both is considered orthostatic hypotension. Option 2: True. Moist mucous membranes is a good sign of fluid volume. Mucous membranes would be dry if the client were still dehydrated. Option 3: False. This is one sign that fluid volume deficit still exists. Skin recoil delayed for more than 2 seconds indicates severe dehydration. Less than 2 seconds = moderate dehydration. Skin recoils immediately = good skin turgor. Option 4: False. Normal urine specific gravity is 1.000 to 1.030 so this result is high indicating dehydration. Remember, the urine output will be low and concentrated. Concentrated makes numbers go up. What three numbers? Sodium, specific gravity, and hematocrit. Option 5: False. If your serum sodium is above 148 mEq or 148 mmol/L (normal is 134-145mEq or 134-145 mmol/L), hypernatremia exists. Hypernatremia is the same thing as dehydration, so this means that the client is still sick.

The nurse receives new healthcare provider prescriptions on a client diagnosed with Addison's disease. Which prescriptions should the nurse recognize as being inappropriately written and requiring clarification from the prescriber? 1. Weigh QD 2. IV of normal saline at 125 mL/hr 3. MRI of pituitary gland 4. Fludrocortisone acetate 0.1 mg by mouth T.I.W. 5. Dehydroepiandrosterone DHEA sulfate 5 mg by mouth every other day

1. & 4. Correct: Use "daily" or "every day". QD is an unapproved abbreviation. T.I.W. stands for three times a week; however, it is an unapproved abbreviation. Use "three times a week". 2. Incorrect: This is a correct action and is written properly. 3. Incorrect: The primary healthcare provider may suggest an MRI scan of the pituitary gland if testing indicates the client might have secondary adrenal insufficiency. This is an approved abbreviation. 5. Incorrect: This is written correctly and may be given to women to treat androgen deficiency.

A nurse assessing a client who is one day post thyroidectomy and identifies an arrhythmia on auscultation. While taking the blood pressure, the nurse notices the client's hand starts to tremble. What interventions are priority? 1. Initiate seizure precautions 2. Monitor potassium level 3. Monitor BUN and Creatinine 4. Restrict fluids 5. Check for airway patency

1. & 5. Correct: The parathyroid glands can accidentally be removed with a thyroidectomy. Low calcium causes rigid and tight muscles. 2. Incorrect: What about potassium? Is this the problem chemical? No, Calcium is. But, can calcium cause an arrhythmia? Yes, it can! See, the NCLEX Lady thought you would see arrhythmia and say that must be potassium, but don't let them get you off track.. Remember, no doubts or hesitation. But, What other chemical should you think about with Calcium? Phosphorous, because we just said they have an INVERSE Relationship . 3. Incorrect: What about BUN and Creatinine? Are my kidney's involved, no not here. 4. Incorrect: Do I restrict fluids, not unless you are a mean nurse. This is not a fluid problem. Option 1 is true. The client can have a seizure. Option 2 is false. The problem chemical is calcium. And checking the potassium is not a safety precaution. Option 3 is false. The kidneys are not the problem and will not promote safety. Option 4 is false. This is a calcium problem not a fluid problem. Fluid restriction will not promote safety.

A client, scheduled for a total hysterectomy for advanced cervical cancer, is crying and states, "I want to have more children! I do not know if I should have this procedure." Which responses by the nurse are best? 1. Allow the client to discuss her fears. 2. Tell client good things that she will be able to do without more children. 3. Explain to the client that her ovaries can be frozen for egg harvesting at a later time. 4. Advise the client to put off having the surgery until she is sure. 5. Encourage client to talk to primary healthcare provider again.

1. & 5. Correct: This may be anticipatory grieving and being scared. Let the person talk and encourage them to talk again to the primary healthcare provider. They need reassurance that they are making the right decision. 2. Incorrect: This is not her fear and not helpful in this situation. 3. Incorrect: Not an appropriate answer and we don't freeze ovaries. 4. Incorrect: The cancer is already advanced stages. Will the waiting help her survive?

The nurse is to administer oxytocin 0.5 milliunits/min IV to a client admitted for labor induction. Oxytocin is available as 10 units/1000 ml 0.9% normal saline. How many mL/hour of the oxytocin should be administered? 1. 3 mL/hour 2. 6 mL/hour 3. 10 mL/hour 4. 12 mL/hour

1. Correct: 3 mL/hour will deliver oxytocin 0.5 milliunits/min. Calculations for IV Oxytocin Solution NOTE 1 Unit = 1,000 milliunits TO MAKE SINGLE-STRENGTH IV SOLUTION: Add 10 Units of Oxytocin to 1 liter of compatible IV fluid. TO INFUSE: Convert prescribed milliunits/min to mL/hr and set infusion pump. AMOUNT Prescribed: 0.5 milliunits/min CALCULATIONS: 10 Units/1 L = 10,000 milliunits/1,000 mL OR 10 milliunits/1 mL. 10 milliunits/1 mL = 0.5 milliunits/X mL Cross-multiply to get: 0.5 = 10X X = 0.05, so 0.05 mL/min Multiply by 60 minutes to get amount infused per hour. THINK: 0.05 milliunits = 0.05 mL/min 0.05 mL/min x 60 min/hr = 3 mL/hr (0.05 x 60 = 3) Set the infusion pump for 3 mL/hr.

The nurse is caring for a client following spinal surgery. The client is placed on methylprednisolone. What additional drug therapy would the nurse expect to be prescribed with methylprednisolone? 1. Pantoprazole 2. Phenytoin 3. Imipramine HCI 4. Aminocaproic acid

1. Correct: A potential side effect of methylprednisolone is a peptic ulcer. The primary healthcare provider will prescribe a proton pump inhibitor or H2 blocker to prevent this side effect. 2. Incorrect: Phenytoin is an anticonvulsant. Seizures are not a side effect of methylprednisolone. 3. Incorrect: Imipramine HCI is an antidepressant which is not routinely given with methylprednisolone (Although mood changes can occur with steroid administration, anti-depressants are not routinely given). 4. Incorrect: Aminocaproic acid is given when clients are bleeding. Bleeding is not a side effect of methylprednisolone.

An elderly, bed-bound client receiving G-tube feedings at home is transported to the emergency department after onset of behavioral changes and hallucinations. Which nursing action is priority while diagnostic testing is underway? 1. Seizure precautions 2. Monitor for signs of increased intracranial pressure 3. Orient to time, place, and person 4. Obtain vital signs q 15 minutes

1. Correct: Feeding tube clients tend to get dehydrated, especially clients on bed rest, because bed rest induces diuresis! If the client is already having neurological signs, a grand-mal seizure may be next! Better take seizure precautions while awaiting the serum sodium results. 2. Incorrect: When hypernatremia is present, the brain cells shrink because when the body is dehydrated, water is drawn from the cells into the vascular space. 3. Incorrect: Until serum sodium is corrected, the client will be unable to process information regarding time, place, and person. The brain does not like it when the sodium is messed up. 4. Incorrect: While you're taking vital signs, your client is having a seizure! Don't delay care! The word "priority" is used in this question, so you must place a value on what to do first. Look at the clues: "elderly", "tube-feeding", "behavioral changes", and 'hallucinations". So what should you worry about. Maybe MI because of elderly and behavioral changes. But look at the other hints: "tube feeding", and "hallucinations". You should worry about which electrolyte when providing tube feeding? Sodium. The sodium level will go up when the client is dehydrated. Tube feedings will make the client dehydrated if the client is not receiving water. High sodium can lead to seizures. So the answer that relates to this complication is option 1: seizure precautions.

Which statement by a student nurse indicates to the nurse educator that teaching regarding witnessing consent signatures has been successful? 1. "Two people must witness a consent signature." 2. "A RN must witness a consent signature." 3. "Signing as a witness implies that the client willingly signed the consent." 4. "A witness must be over the age of 21."

3. Correct: Signing as a witness implies that the witness has observed the client personally signing the consent form with no coercion. 1. Incorrect: Only one signature is required as a witness. 2. Incorrect: The witness does not have to be an RN. 4. Incorrect: A witness is required to be over the age of 18.

A client with a diagnosis of pulmonary edema is observed sitting up in bed stating, "I can't get my breath". What is the priority intervention for this client? 1. Dangle the client's legs over the side of the bed. 2. Auscultate anterior and posterior lung fields bilaterally. 3. Call respiratory therapy to the room stat to bring an oxygen mask. 4. Administer PRN morphine sulfate 2 mg IVP via existing venous access device.

1. Correct: Since the client is already sitting up, the most immediate nursing intervention is to dangle the client's legs over the side of the bed. 2. Incorrect: This is assessment. The question is asking for an intervention. 3. Incorrect: This may take time. Do something to help the client immediately first. 4. Incorrect: Medication will require time to prepare and dangling the legs will help the client now. This question asks you to pick out the "priority" intervention which places value on that intervention. It helps to know about pulmonary edema, but note the clue: "sitting up in bed", stating "I can't get my breath". So what is the fastest and easiest intervention that would help the client breathe? Option 1. The problem is too much fluid entering the heart, so pool blood in the periphery by placing the legs in a dependent position. Dangling the lower extremities in a dependent position will trap venous blood in the periphery which will decrease venous return and reduce preload. Look at option 2. This is an assessment. The question is asking for an intervention. So this one is wrong. Options 3 and 4 will help the client but will take longer. Do something that can help the client now.

The client asks the nurse what the primary healthcare provider means when he says that she has right sided heart failure. What should the nurse include in the teaching plan? 1. There is a backup of blood in the right upper chamber of the heart. 2. There is swelling of lower extremities. 3. The heart rate decreases. 4. You may experience fatigue and depression. 5. You may experience nausea and loss of appetite.

1., 2. 4. & 5. Correct: The blood backs up into the right atrium and venous circulation. Vascular congestion is evident by swelling of the lower extremities. Clients usually experience fatigue and depression. Nausea and loss of appetite may be a result of the hepatomegaly that is also seen with right sided failure. 3. Incorrect: The heart rate increases in an attempt to increase cardiac output.

What interventions can an occupational health nurse discuss with a client in an effort to improve lateral epicondylitis (tennis elbow) pain? 1. Avoid activities that make the pain worse. 2. The primary healthcare provider may prescribe an oral nonsteroidal anti-inflammatory drug. 3. Apply an epicondylitis strap 2 to 3 centimeters above the elbow. 4. Stretching and strengthening the muscle and tendon should be started immediately. 5. If pain persists, a cortisone injection into the inflamed area may be recommended by the healthcare provider. 6. Apply ice for 45 minutes six times a day.

1., 2., & 5. Correct: General activities that make the pain worse should be avoided or at least cut back. While continued activity in the presence of mild discomfort is not harmful, severe pain will only prolong the necessary recovery time and should be avoided. Oral nonsteroidal anti-inflammatory drugs are very helpful in controlling the pain and inflammation of tennis elbow. The medicine is taken daily for at least four to six weeks when treating severe cases. For less severe cases, these medicines may be taken only when needed. Cortisone injections are considered when the other measures have not worked and the pain is severe. The cortisone is injected into the area of the inflamed tendons in order to decrease the inflammation. 3. Incorrect: Tennis elbow straps are found to be helpful by some clients. They are designed to be worn 2 to 3 centimeters below the elbow. This is intended to take the stress off the tendon where it attaches to the bone. 4. Incorrect: Stretching and strengthening of the involved muscle and tendon unit is one of the mainstays of treatment for this condition once pain and inflammation have subsided. A gentle stretching program is started through a range of motion at the elbow and wrist. This is combined with a program of muscle strengthening. 6. Incorrect: It is recommended that the area be iced two to three times a day, for 20 to 30 minutes each time.

What nursing interventions should a nurse initiate for a client diagnosed with pyelonephritis? 1. Monitor for dark, cloudy, foul smelling urine. 2. Monitor intake and output. 3. Decrease fluid intake to 1 liter/day. 4. Advise that urine may turn blue with administration of nitrofurantoin. 5. Monitor for hypotension, tachycardia, and chills.

1., 2., & 5. Correct: With pyelonephritis, urine will be dark, cloudy, and foul smelling. The client should be placed on intake and output. Monitor for septic shock, which is a complication of pyelonephritis. 3. Incorrect: This is not enough fluid. 4. Incorrect: This medication will not turn the urine blue. Option 1: True. The nurse would expect to see dark, cloudy, foul smelling urine when assessing the client's urine. Option 2: True. When you have a fluid volume problem, heart problem, or renal problem, place the client on intake and output. Option 3: False. Fluid intake should be increased to 2-3 liters/day unless contraindicated, to flush out the kidneys. Option 4: False. Nitrofuranoin, an antibiotic, will turn the urine dark yellow or brown, not blue. Option 5: True. This client is at risk for septic shock. Shock is indicated by hypotension, tachycardia, and chills.

In which client should the nurse question a prescription for an oxytocin challenge test? 1. Client at 26 weeks gestation. 2. Client at 38 weeks with 4 Cesarean section deliveries. 3. Client at 38 weeks with a history of gestational diabetes. 4. Client at 37 weeks gestation. 5. Client that is 35 weeks gestation and has preeclampsia. 6. Client with a current history of placenta previa.

1., 2., & 6. Correct: 26 weeks is too early to stimulate contractions. This could lead to a precipitous delivery. Stimulating contractions in a client with previous cesarean deliveries is not recommended. This may lead to uterine rupture. Stimulating contractions in a client with placenta previa is not recommended. This may lead to hemorrhage. 3. Incorrect: There is no reason to suspect complications from an oxytocin challenge test for this client. 4. Incorrect: There is no reason to suspect complications from an oxytocin challenge test for this client. 5. Incorrect: There is no reason to suspect complications from an oxytocin challenge test for this client.

The nurse is monitoring the client's heart rhythm. The monitor shows sinus tachycardia. What is expected with this assessment finding? 1. Regular rhythm 2. Rate of 101-200 3. P-wave normal 4. P-R interval not measurable 5. QRS complex normal

1., 2., 3. & 5. Correct: Sinus tachycardia indicates a regular rhythm, although the rate is elevated. The term tachycardia is defined as a heart rate above 100. The P-wave is normal in a sinus rhythm. Sinus rhythms have a normal QRS complex. 4. Incorrect: P-R interval is not measurable in atrial flutter, atrial fib, PVCs, V tach or V fib. Option 1: True. Regular rhythm means all the p waves, and QRS complexes will be evenly spaced or regular. Option 2: True. Sinus tachycardia means that the HR is greater than 100 beats/minute up to 180-200 beats/min. Option 3: True. The p wave is normal which means that it is upright, and consistent. Option 4: False. The P-R interval will measure between 0.12-0.20 seconds and shortens with increasing heart rate. Option 5: True. The QRS complex is normal, measuring less than 0.12 seconds and is consistent.

A client has been admitted to the unit with acute pyelonephritis. What interventions should the nurse include in this client's plan of care? 1. Observe for changes in mental status. 2. Assist client to restroom. 3. Palpate the bladder every 4 hours 4. Help the client get in a comfortable position to void. 5. Instruct client to void every 30 minutes while ill.

1., 2., 3., & 4. Correct: Changes in mental status may signify septic shock. Help clients to the restroom, use bedpans/urinals, and get is a comfortable position to facilitate urination. Palpate the bladder every 4 hours to determine the presence of bladder distension. 5. Incorrect: Instruct to void every 2-3 hours. This will prevent the buildup of urine in the bladder. Every 30 minutes is excessive. Option 1: True. Changes in mental status can occur due to accumulation of urea and electrolyte imbalances which can be toxic to the central nervous system. It may also signify septic shock. Option 2: True. The client can cause fatigue and confusion which places the client at risk for falls. Option 3: True. Clients on bedrest and who are very ill may not sense the need to void, so the bladder should be palpated to assess for a full bladder. Option 4: True. Clients often have difficulty voiding when lying flat. Sitting upright or standing may facilitate urination. Option 5: False. This is excessive. The bladder will not be full if voiding every 30 minutes.

A client arrives at the emergency department (ED) after sustaining a high-voltage electrical injury. Which interventions should the nurse initiate in the ED? 1. Assess entry and exit wound. 2. Monitor vital signs. 3. Test for myoglobinuria. 4. Connect to cardiac monitor. 5. Perform the rule of nines.

1., 2., 3., & 4. Correct: These are correct interventions for the nurse to initiate when caring for a client who has sustained a high-voltage electrical injury. Remember, Electricity kills vessels, nerves, and organs. 5. Incorrect: The rule of nines would not be used for an electrical injury. Visual examination is not predictive of burn size and severity with an electrical burn injury. Option 1: This is a true statement. Electrical burns have two wounds: an entrance burn wound that is generally small and an exit burn wound that is much larger. The electricity goes throughout the body causing damage, and then exits the body. So look for 2 burn wounds. Option 2: This is a true statement. Remember, vessels, nerves, and organs can be damaged. The nurse needs to monitor vital signs frequently, especially those assessing the respiratory and cardiac systems, since we worry about organ damage. Option 3: This is true. Electricity kills cells and muscle causing myoglobin to be released. When myoglobin gets into the kidneys, it can clog up the glomerulus. This can damage the kidneys - so we need IV fluid to keep the kidneys flushed out. Option 4: This is true. Electricity can damage the heart muscle, so the client is at risk for dysrhythmias within 24 hours following an electrical burn. Put the client on continuous cardiac monitoring during this time. Option 5: This statement is false. Even if you did not know this, but knew the other 4 options were true, then the you should be able to eliminate this option since with SATA questions, there must be at least one wrong answer. The rule of nines is not used for electrical burns, but for thermal burns. Most of the damage from electrical burns internal and cannot be determined by using the rule of nines.

What signs and symptoms should the nurse expect to find during the physical assessment of a client who has a history of rheumatoid arthritis? 1. Firm nodules noted on the arms 2. Reports of morning stiffness lasting for hours 3. Fever 4. Cool, swollen joints 5. Weight gain 6. Joint deformity

1., 2., 3., & 6. Correct: Firm bumps of tissue under the skin on the arms are called rheumatoid nodules, a sign of rheumatoid arthritis. Morning stiffness that can last for hours is a symptom of rheumatoid arthritis. Over time, rheumatoid arthritis can cause joints to deform and shift out of place. Fever is a common symptom of rheumatoid arthritis. 4. Incorrect: Affected joints will be tender, warm, and swollen. 5. Incorrect: Weight loss occurs rather than weight gain.

Which signs/symptoms does the nurse expect to note when caring for a client with a suspected cystitis? 1. Incontinence 2. Urgency 3. Frequency 4. Hematuria 5. Nocturia 6. Flank pain

1., 2., 3., 4. & 5. Correct: Signs and symptoms of cystitis include burning on urination, nocturia, incontinence, suprapubic or pelvic pain, hematuria, and back pain. 6. Incorrect: Flank pain is seen when the urinary tract infection progresses to the kidneys. Cystitis is a fairly common lower urinary tract infection, it refers to an inflammation of the bladder, specifically, the wall of the bladder. All this question is asking is "Hey new nurse, do you know the signs/symptoms of cystitis? Well do you? Remember to look at each option individually and decide if it is a sign or symptom of cystitis. Option 1. Incontinence - loss of bladder control? True or False? True. Option 2. Urgency - A strong desire to urinate, accompanied by a fear of leakage.? True Option 3 Frequency - the need to urinate many times during the day, at night (nocturia), or both but in normal or less-than-normal volumes. ? True Option 4 Hematuria - the presence of blood in urine.? True Option 5. Nocturia - Frequency at night? True Option 6. Flank pain? False. Flank pain occurs with a kidney infection, pyelonephritis.

A nurse is caring for a client post heart catheterization with a left femoral stick. What signs and symptoms would indicate to the nurse that the primary healthcare provider should be notified? 1. Capillary refill of 6 seconds to left toes. 2. Epigastric discomfort 3. Paresthesia to left leg 4. Left pedal pulse 0/4; Right pedal pulse 2+/4 5. Left foot pain 6. Temperature of 99.9º F (37.72º C)

1., 2., 3., 4. & 5. Correct: These signs and symptoms indicate an emergency with loss of circulation to the extremity. This is an emergency, and the primary healthcare provider is the only one that can save this foot from ischemia. Don't delay. Epigastric pain could indicate the client is having an MI. Always assume the worse! 6. Incorrect: Temperature of 101º F (38.3º C) or more indicates a problem. Look at the clues in this question: "post heart cath", "femoral stick", and "signs and symptoms" of a Problem! What are complications the test taker should worry about post heart cath? Hemorrhage, re-occlusion, and decreased circulation to the extremity. Signs and symptoms of hemorrhage include bleeding from the puncture site, decreasing BP, and increasing pulse. Signs of re-occlusion of the artery in the heart include signs of a heart attack such as pressure or tightness in the chest, pain in the chest, back, jaw, and epigastric region, shortness of breath, sweating, nausea, vomiting, anxiety, a cough, dizziness. Signs and symptoms of decreased circulation to the extremities (Remember the 5 Ps) include pain, pulselessness, pallor, paresthesia, and paralysis. Capillary refill should be less than 3 seconds.

A nurse manager has several issues regarding staff maintaining proper infection control while caring for clients. What actions should the manager take regarding this issue? 1. Place colorful posters regarding infection control in conspicuous places on unit. 2. Monitor staff providing client care for the use of appropriate infection control. 3. Give staff a written test on proper infection control. 4. Have all staff read agency policy and procedures regarding infection control. 5. Dock pay of staff who do not maintain proper infection control. 6. Provide mandatory in-service sessions on infection control for every shift.

1., 2., 3., 4., & 6. Correct. Each of these actions can be taken by the nurse manager. The staff needs further education, reminders, and follow-up observation. Posters are great reminders of concepts. All nurses need to supervise those under their direction. Testing can be done as pretest or post test along with in service education. Staff development or in service sessions are required by Joint Commission on Accreditation of Healthcare Organizations (JCHO) on infection control. 5. Incorrect. This is not the best solution, because most people want to do what is right. Education should be tried first, then documentation of the infractions. You must support, supervise and educate!

A nurse is caring for a client who is on bed rest following admission to the hospital two days ago with a diagnosis of new onset heart failure. While evaluating the client's progress, what assessment findings would indicate to the nurse that further treatment is required? 1. Sacral edema 2. Orthopnea 3. Shiny skin 4. S3 heart sound 5. Heart rate - 98/min 6. CVP - 8 mmHg

1., 2., 3., 4., & 6. Correct: These are all signs of fluid volume excess seen with heart failure. 5. Incorrect: This is a normal heart rate which would indicate the client is improving. Option 1. True. When a client has been on bed rest for a while the nurse will see sacral rather than ankle edema. Edema is seen with fluid volume excess. When the client has too much fluid in the vascular space it will eventually start to leak out into the tissue causing 3rd spacing. Option 2. True. Orthopnea is an abnormal condition in which the person must sit up or stand to breathe comfortably. This would indicate FVE. When the hear is weak it cannot pump well, so fluid backs up into the lungs. Option 3. True. Edematous skin is extremely stretched to where it appears shiny. Option 4. True. A S3 heart sound is often an indication of heart failure. The third heart sound (S3), also known as the "ventricular gallop", occurs just after S2 when the mitral valve opens allowing passive filling of the left ventricle. The S3 sound is actually produced by the large amount of blood striking a very compliant left ventricle. A S3 can be an important sign of systolic heart failure. Option 5. False. A heart rate of 98/min is normal sinus rhythm. With FVE, expect to see tachycardia. Option 6. True. Normal CVP is 2-6 mmHg. This client's CVP of 8 mmHg is high indicating FVE.

A nurse is planning to provide information to a group of adults considering smoking cessation. What information should the nurse include? 1. Nicotine is the drug in tobacco products that produces dependence. 2. Withdrawal symptoms may include irritability, difficulty concentrating, and increased appetite. 3. Stopping smoking reduces the risk of coronary heart disease. 4. All smokers need to have a prescription for bupropion SR in order to quit. 5. Refer to smoking quit-lines that offer free support, advice, and counseling from experienced coaches.

1., 2., 3., 5. Correct: These are correct statements. Nicotine is the drug in tobacco products that produces dependence. Other withdrawal symptoms include anxiety and cravings for a cigarette. There are many health benefits to smoking cessation including reducing the risk of coronary heart disease, stroke, peripheral vascular disease, COPD and reduced risk for infertility in women. Clients should be referred to educational programs and support groups. 4. Incorrect: The majority of cigarette smokers quit without using this prescription; however, treatments can help the smoker quit, so they should discuss possible medications with their primary healthcare provider. Other medications such as the nicotine patch or varenicline (chantix) may also be used to assist with smoking cessation.

The home care nurse visits a client who has moderate cognitive impairment and whose family provides care for the client. Which suggestions would be helpful for this family to reduce the risk of injury? 1. Suggest that the family lock up medications and poisons and keep the keys. 2. Encourage the family to place locks high on the door frame to make it difficult for the client to leave. 3. Suggest that the family talk with the client weekly about safety issues around the house. 4. Suggest that the family remove knobs from stove when not in use. 5. Keep fire extinguishers present and in working order.

1., 2., 4. & 5. Correct: Clients with cognitive impairment may forget that they have taken their medicines and take them again. They may also confuse harmful substances with other substances. Locks in places that are not normally expected will make it more difficult for the client with a cognitive impairment to find and open. This is especially useful if the client wanders. The client may turn the stove on and be burned or cause a fire. If the knobs are removed, the home is safer for everyone. Fires are a hazard for people with cognitive impairment; therefore, the presence of a working fire extinguisher could prevent damage from a fire. 3. Incorrect: A client with moderate cognitive impairment will need to be continually supervised to decrease their risk of injury. The retention of information this client has is too short for weekly discussions.

Question: The nurse is caring for a client who is receiving enoxaparin after a diagnosis of deep vein thrombosis of the left leg. Which nursing interventions would be appropriate for this client? 1. Monitor PT and aPTT 2. Initiate bedrest 3. Apply cool, moist packs to left leg 4. Elevate left leg 5. Monitor closely for bleeding 6. Monitor complete blood count

1., 2., 4., 5. & 6. Correct: The main complication of anticoagulant therapy is bleeding. Blood studies such as CBC should be monitored. Bedrest will reduce the risk of a clot dislodging. Elevate left leg to decrease swelling and promote venous return. 3. Incorrect: Warm, moist packs reduce discomfort. Never put cold on a vein. Option 1: True. These lab values will help to identify if the client is at increased risk for bleeding. Option 2: True. If the client gets up and ambulates with a clot in the leg, that clot can dislodge and traveling to the heart, lung, or brain, causing pulmonary embolus. Option 3: False. Never apply cold to a vein. Warm, moist heat will decrease inflammation and reduce pain. Option 4: True. Elevating the effected extremity will decrease swelling and promote venous return. Option 5: True. The client taking enoxaparin will be a risk of bleeding. Bleeding assessment and bleeding precautions are essential. Option 6: True. A CBC will let the nurse know if the client is bleeding.

A petite female client presents to the clinic with symptoms of back pain and states, "I think I am getting shorter." Which teachings would be appropriate for the nurse to provide? 1. Spend time in the sunlight twice a week for 5 to 30 minutes. 2. Wear rubber sole shoes for traction. 3. Walk at least 30 minutes most days. 4. Include yogurt and hard cheese in diet. 5. Take regularly scheduled prescribed corticosteroids.

1., 2.,3. & 4. Correct: The client with osteoporosis is usually female, small framed and back pain from collapsed vertebrae and shortening are symptoms. Exposure to light converts vitamin D stores in the skin. Rubber shoes are a safety issue to prevent loss of balance and falls. Weight bearing exercises like walking will promote bone density. Yogurt and hard cheese are high in calcium. 5 Incorrect: This individual should not be taking corticosteroids as these drugs will promote the loss of calcium.

A client admitted to ICU has a prescription for an arterial line insertion to the right radial artery. What assessment findings by the nurse would be of concern? 1. Right sided mastectomy 2. Inability to abduct fingers of right hand 3. Negative Allen's test 4. Radial pulse 3+/4 5. Presence of A-V shunt to right forearm

1., 3. & 5. Correct: Right sided mastectomy would be a contraindication. A negative Allen's test means that the ulnar artery is not patent enough to supply blood to the hand. An A-V shunt would be a contraindication. 2. Incorrect: This assesses peripheral nerve function which is used for musculoskeletal assessment. 4. Incorrect: Normal finding. Option 1: True. What do we know about care of the client post mastectomy? No blood pressures, no needle sticks, no nothing on that extremity. So that means no arterial line on the right side. Option 2: False. The movement of the fingers of the right hand will not determine if there is adequate arterial blood getting to the hand and fingers. Option 3: True. The nurse wants to see a positive Allen's test, which means that the ulnar artery is patent and can supply arterial blood to the hand and fingers. Option 4: False. The strength of the pulse here is very good, which should not cause the nurse to be concerned. Option 5: True. What do we know about care of the client with an A-V shunt to the right forearm? No blood pressures, no needle sticks, no nothing on that extremity. So that means no arterial line on the right side.

After a cholecystectomy, a client experiences palpitations, weakness and diarrhea following meals. Which teachings would be appropriate for the nurse to provide the client? 1. Drink minimum fluids with meals. 2. Follow a high carbohydrate, high protein meal plan. 3. Avoid electrolyte replacement sports drinks. 4. Lie down on right side after meals. 5. Eat at least six small meals per day.

1., 3., & 5 Correct: Dumping syndrome is associated with meals having a hyperosmolar composition. To decrease hyperosmolar components, you decrease the carbs and electrolytes. You should also limit fluids with meals because they increase the size of the food bolus. Small frequent meals decrease the extremes of the hyperosmolar content and keep a steady blood sugar level. 2. Incorrect: Carbs speed through the GI tract. Fats and proteins digest slower and stay in the stomach longer. 4. Incorrect: Lying down on the left side slows emptying of the stomach. Lying on the right side will speed up emptying. HURST TEST TAKING STRATEGY: 1. Identify clues in the stem. Generally the stem is short and contains only the information needed to make the question clear. A clue is the use of a word or phrase that leads you to the correct answer. The clue shows a relationship to word or phrase in the correct answer. 2. The test taker would need basic knowledge about dumping syndrome to answer this question easily. 3. With Select All That Apply questions, look at each option as a true/false statement. Match what is being asked in the stem with the options. If the stem is asking for a true response, then the true options should be selected. With Select All That Apply questions there will be two or more correct options, but never all.

Question: The nurse recognizes which manifestations as signs of community-acquired pneumonia? You answered this question Incorrectly 1. Cough 2. Decreased respiratory rate 3. Fever 4. Myalgia 5. Pleuritic chest pain

1., 3., 4. & 5. Correct: Signs of community-acquired pneumonia include cough, crackles, egophony, tactile fremitus, fever, dyspnea, sputum production, myalgias, and pleuritic chest pain. A client with an infection (particularly pneumonia) will exhibit these symptoms. 2. Incorrect: Decreased respiratory rate is not a sign of community-acquired pneumonia; respiratory rate increases with fever and dyspnea. Well, if you know the signs and symptoms of pneumonia, then you have it made with this question. Remember, With Select All That Apply questions, look at each option as a true/false statement. Match what is being asked in the stem with the options. If the stem is asking for a true response, then the true options should be selected. And, With Select All That Apply questions, there will be two or more correct options. So which options are true? Did you say options 1., 3., 4. & 5? Yes, these are the signs of community-acquired pneumonia. With pneumonia the client will have a cough, crackles, egophony, tactile fremitus, fever, dyspnea, sputum production, myalgias, and pleuritic chest pain. Why is Option 2 false? Decreased respiratory rate is not a sign of pneumonia; the respiratory rate increases with fever and dyspnea.

Which information should the community health nurse include when explaining to a group of college students living in a dormitory about receiving an immune globulin (IG) injection for hepatitis A virus (HAV)? 1. Immune globulin contains antibodies that destroy the hepatitis A virus (HAV), preventing infection. 2. Immune globulin protection is permanent, so no other injection is required. 3. Common side effects of the injection include soreness and swelling around the injection site. 4. The sooner you get a shot of IG after being exposed to HAV, the greater the likelihood that infection will be prevented. 5. Crowded living environments such as dormitories place people at risk for HAV.

1., 3., 4. & 5. Correct: These are all correct statements about immune globulin for Hepatitis A. 2. Incorrect: IG protection is only temporary, lasting about 3 months. Option 1. True. The client gets the antibodies immediately. Option 2. False. Passive immunity is temporary. Option 3. True. Even if the test taker did not know this, most injections cause some discomfort and swelling at the site. Option 4. True. Antibodies are received immediately with passive immunity, so the client has what it needs to fight off this infection at once. Option 5. True. The most common method of transmission for Hepatitis A is through the fecal/oral route. People living in crowded, confined places are at a higher risk of contracting Hepatitis A.

The charge nurse recognizes that a new nurse can properly perform a linear wound dressing change on a surgical client when the new nurse performs which interventions correctly? 1. Hand hygiene is done prior to the dressing change. 2. Dressing tape is removed in the direction opposite of the hair growth. 3. The soiled dressing is discarded in a biomedical waste bag. 4. Clean gloves are donned in order to clean the wound. 5. The center of the wound is cleaned first, then the wound area farthest from the nurse, then the area closest to the nurse. 6. New sterile dressing is applied to the wound.

1., 3., 5. & 6. Correct: These are all correct procedures for doing a dressing change. 2. Incorrect: Tape on the dressing is pulled parallel with the skin in the direction of hair growth. 4. Incorrect: Most dressing changes following surgery are sterile and require that the nurse use standard precautions and wear sterile gloves to clean the incision and apply sterile dressings. Clean gloves can be used to remove the old dressing. Dressings are never touched by ungloved hands. Option 1 is true. Hand hygiene is the most basic way to prevent the spread of infection. Option 2 is false. Tape should be removed in the same direction as the growth of hair. Pulling opposite hair growth direction can damage follicles and skin. Option 3 is true. Soiled dressings have blood and body fluids on them which is a method of transmission, therefore, soiled dressings are considered a biohazard. Option 4 is false. In the hospital setting, dressing changes are a sterile procedure. The attempt is to decrease the chance of a hospital acquired infection. Option 5 is true. Remember, clean to dirty. The wound is considered the cleanest area, so that is cleaned first. The area beside the wound farthest from the nurse is considered the next cleanest area. The area closes to the nurse is considered the most contaminated and is cleaned last. Option 6 is true. Sterile dressings are used to cover the wound to decrease the chance of infection.

A client who has been diagnosed with chronic venous insufficiency has received teaching regarding how to prevent venous stasis ulcerations. Which statements by the client indicate to the nurse that teaching has been effective? 1. "Stationary standing should be kept to a minimum." 2. "It is important to avoid wearing constricting clothes longer than 2 hours." 3. "Elevation of my legs should be done for 15 minutes every 4-6 hours." 4. "Protecting my legs from trauma is very important." 5. "I will wear compression stockings every day." 6. "I will elevate the foot of my bed 6 inches (15.24 cm) when I sleep." RationaleStrategies

1., 4., 5., & 6. Correct: Minimize stationary standing as much as possible. Protect legs from trauma as this can lead to ulcerations. Elastic compression stockings are recommended for clients with chronic venous insufficiency to prevent pooling and promote venous return. Leg elevation decreases edema, promotes venous return, and provides symptomatic relief. Legs should be elevated frequently throughout the day (for at least 15-30 minutes every 2 hours). During the night, the client should sleep with the foot elevated approximately 6 inches (15.24 cm). 2. Incorrect: The client should avoid wearing any constricting clothing, even for short periods of time. This will decrease blood flow. 3. Incorrect: Leg elevation decreases edema, promotes venous return, and provides symptomatic relief. Legs should be elevated frequently throughout the day (at least 15 to 30 minutes every 2 hours).

A client returned to the unit following a total hip replacement. What statement by the client would indicate to the nurse that teaching has been successful? 1. "I will try to keep my legs together as close as possible." 2. "I will not elevate the head of the bed." 3. "I know that I cannot ever swim again." 4. "I can resume my exercises at the gym within one month."

2. Correct: Flexion of the hip should be avoided after hip surgery. Elevating the HOB would cause flexion, which could cause hip dislocation. 1. Incorrect: The legs should be kept in an abducted (legs apart) position following surgery to keep the head of the femur in the acetabulum (hip in the socket). An abductor pillow is often used to accomplish this and prevent the legs being close together or crossing. 3. Incorrect: Swimming is a non-weight bearing exercise that is encouraged during rehabilitation for post hip replacement clients. Walking is another good exercise for these clients. 4. Incorrect: Stressors on the hip joint should be kept to a minimum for the first 3 to 6 months. Some exercises in the gym could put too much strain on the new hip joint and cause dislocation.

A client comes to the clinic reporting palpitations, as well as nausea and vomiting while taking metronidazole. The nurse notes that the client is flushed and has a heart rate of 118 bpm. Based on this information, what is the most important question for the nurse to ask the client? 1. "Do you take metronidazole on an empty stomach?" 2. "Are you using any products that contain alcohol?" 3. "How long have you had these symptoms?" 4. "What other medications are you currently taking?"

2. Correct: Flushing, nausea and vomiting, palpitations, tachycardia, psychosis are signs of disulfiram-type reaction seen when using products containing alcohol (cologne, after shave lotion, or path splashes) or ingesting alcohol products while taking metronidazole. 1. Incorrect: Although it is preferable to take metronidazole on an empty stomach, this is not the most important question to ask at this time. 3. Incorrect: How long the client has had these symptoms is not as important as whether the client is using any alcohol containing products. 4. Incorrect: Although the nurse needs to know what other medications the client is taking, it is not as important as knowing if the client is using any alcohol containing products.

The nurse is assessing a client with advanced cirrhosis and notes an abdominal girth increase of 5 inches (12.7 cm) since yesterday. What is the best position for the nurse to place this client? 1. Supine 2. Mid-Fowler's 3. Trendelenburg 4. Lateral, left side

2. Correct: If I've got an increasing abdominal girth, I've got lots of pressure on my abdomen, so sit me up! 1. Incorrect: Now, look this position. What will this positions do? That's right, make it harder to breathe! 3. Incorrect: Please don't put me in Trendelenburg and stand me on my head when I am not perfusing well! 4. Incorrect: What will putting them on their left side do? Nothing, that's right! Look at the clues in this question: "advanced cirrhosis", and "abdominal girth increase". What should you be worried about when asked what position to put the client in? Difficulty breathing because of the enlarging abdominal girth, which can push up on the diaphragm making the work of breathing difficult. Think about it.....You need to get their head up to improve respirations and perfusion. Which position will help the client breathe better? Mid-Fowler's

A client has sustained burns to the upper torso, face, and neck as a result of a steam injury when a pressure cooker exploded at home. Which intervention is the nurse's priority? 1. Administer fentanyl for pain. 2. Prepare for endotracheal intubation. 3. Administer 1000mL of LR. 4. Drench immediately with running water.

2. Correct: Intubation must be accomplished quickly while a tube can still be inserted. 1. Incorrect: The client definitely needs pain medication, however, airway and breathing take priority. 3. Incorrect: Fluid resuscitation is needed, however, airway comes first. 4. Incorrect: You want to cool them down, but you are in the ER now and this should have been done at the site. Option 1 would be a correct answer if this was a select all that apply question, but not when asked for the priority answer. The client definitely needs analgesia, and administration of fentanyl will not only ease the pain, but will enhance intubation. However, when you are picking priority, you better make sure the client can breathe. Option 3 would be a correct answer if this was a select all that apply question, but airway still takes priority. This is another critical answer because fluid will be lost from the vascular space quickly after a major burn due to increased capillary permeability from the thermal injury to the vessel walls. So now you have to pick between two killer answers: airway (for breathing) and circulation. Airway wins here. Option 4 should have been done before the client arrived to the emergency department. Airway is the priority here.

A client who has had a laparoscopic cholecystectomy develops pain in the left shoulder. Vital signs, laboratory studies, and an electrocardiogram are within normal limits. What does the nurse recognize as a contributing cause of the pain? 1. Surgical cannulation of the bile duct is causing spasm and pain. 2. Carbon dioxide used intraperitoneally is irritating the phrenic nerve. 3. Large abdominal retractors used in the procedure compressed a nerve. 4. Side lying position in the operating room generated pressure damage.

2. Correct: Phrenic nerve irritation can result in referred pain to the left shoulder. Carbon dioxide (CO2) is used to inflate the abdominal/chest wall during the procedure for better visualization of the internal organs. If the CO2 irritates the phrenic nerve, it radiates to the shoulder. 1. Incorrect: Surgical cannulation of the bile duct is not performed during a laparoscopic cholecystectomy. 3. Incorrect: Large abdominal retractors are not used during this procedure. This is done via a small incision to accommodate a scope. 4. Incorrect: The client is turned in several directions during the procedure to prevent damage to the abdominal viscera.

The nurse in the outpatient clinic performs an assessment on a client who takes propranolol for management of palpitations associated with mitral valve prolapse. Which statement by the client should be reported immediately to the primary healthcare provider? 1. "My resting pulse was 60 this morning." 2. "I feel a little short of breath when walking." 3. "I have lost 5 pounds in the last 2 weeks." 4. "My blood pressure (BP) was lower this visit than last time."

2. Correct: Propranolol is a non-selective beta blocker so it blocks sites in the heart and in the lungs. The shortness of breath could be the result of the adverse reactions of bronchospams or heart failure. This statement requires immediate investigation by the primary healthcare provider. 1. Incorrect: A side effect of propranolol is bradycardia. The client should be taught to contact their primary healthcare provider if their pulse is <50 beats per minute (bpm). A pulse rate of 60 bpm is acceptable. 3. Incorrect: Losing weight is not a side effect of propranolol. Weight loss regimen may be encouraged for hypertension. Losing 5 pounds in 2 weeks is within the acceptable range. 4. Incorrect: The therapeutic effect of propranolol is to reduce BP. If the client is asymptomatic, decreased BP is no big deal.

An elderly, confused client with dehydration is admitted to the medical unit. Which intervention would be appropriate for the RN to delegate to the LPN/LVN? 1. Develop a plan of care to monitor intake and output. 2. Reinforce the teaching plan with the client's family. 3. Maintain fresh fluids at bedside. 4. Assess I & O for adequate fluid replacement.

2. Correct: The LPN/LVN can reinforce teaching. 1. Incorrect: The LPN/LVN cannot develop a plan of care. 3. Incorrect: This can best be accomplished by the unlicensed assistive personnel (UAP), it can be done by LPN/LVN but not best use of resources. 4. Incorrect: Assessment is a role of the RN, not LPN. LPN can observe and data collect but not assess and evaluate on the NCLEX. Option 1 should not be delegated to the LPN. The RN must develop the plan of care. Option 2 can be delegated to the LPN. RNs do the initial teaching, but the LPN can reinforce teaching that has been initiated by the RN. Option 3 should not be delegated to the LPN. This is a waste of a resource. The LPN can do more than this. The UAP could handle this. Option 4 should not be delegated to the LPN. The LPN can collect data, but the RN must assess and evaluate.

Question: A nurse wants to find out a better way to perform oral care on unresponsive clients. What is the best first action for the nurse to take in order to achieve this goal? 1. Try different methods of oral care on unresponsive clients to see what works best. 2. Discuss the issue with the leader of the "best practices" committee. 3. Read all the current literature related to oral care on unresponsive clients. 4. Ask the primary healthcare provider to suggest the best oral care procedure.

2. Correct: The best first action for the nurse is to identify a problem, and follow up with the appropriate person. An experienced person who can research "best practice" regarding the issue is needed. The best practice committee works to improve clinical practice based on current research 1. Incorrect: This is doing research, which requires the research process be implemented, including appropriate approval. The best practice committee utilizes current research in their recommendations. 3. Incorrect: This will take a lot of time and is best initiated from the "best practice" committee. The nurse could definitely be part of the committee. But the evidence-based care leaders are trained to help nurses through the proper process of evidence based research. 4. Incorrect: This is a nursing responsibility and the best practice committee is the best place to begin. The primary healthcare provider may have suggestions but this is not the best first action.

The client expresses concern to the nurse about the ability to provide self-care and perform activities of daily living at discharge. Which member of the healthcare team should the nurse contact to provide information and assist the client with resources for an effective discharge plan? 1. Primary healthcare provider 2. Case manager 3. Physical therapist 4. Occupational therapist

2. Correct: The client's case manager should be contacted regarding the order for pending discharge from the healthcare facility. The case manager coordinates care and provides the client with information and resources for an individualized discharge plan. 1. Incorrect: The primary healthcare provider does not assume the case management role in the acute care facility setting, and generally does not coordinate the discharge planning process. 3. Incorrect: The physical therapist is a member of the multidisciplinary team and might help evaluate, but does not coordinate discharge planning. They are not responsible for case management and coordination of overall client care for discharge from the facility. 4. Incorrect: The occupational therapist is a member of the multidisciplinary team and might help evaluate, but does not coordinate discharge planning. They are not responsible for case management and coordination of overall client care for discharge from the facility.

Following escharotomy of a circumferential burn to the arm, which assessment is the best indicator when evaluating the effectiveness of this procedure? 1. Absence of pain in the extremity. 2. Prompt capillary refill after blanching. 3. Bleeding at the site of the incision. 4. Ability of the client to wiggle his/her fingers

2. Correct: The objective of creating an incision through the eschar is to relieve the pressure and restore circulation. If nail beds blanch and refill promptly, blood is flowing into the limb. 1. Incorrect: Absence of pain is not an indicator of circulation to a limb. It's a good thing, but is not a definitive evaluation of circulation. 3. Incorrect: Bleeding would indicate that circulation was improved in the incision area, but to assure improved circulation of the total arm, capillary refill is the best assessment. 4. Incorrect: Movement (motor) is a neurological check, and the right answer will involve a circulatory (vascular) check! Option 1 will not evaluate the effectiveness of restored circulation. The client may have extremity pain due to the escharotomy incision and from the burn itself. Option 3 will not evaluate the effectiveness of restored circulation. You would expect to see some bleeding at the incision site. Option 4 will not evaluate the effectiveness of restored circulation.This will tell the nurse if there is intact nerves. Option 2 is the only option that will evaluate the effectiveness of restored circulation. This tells the nurse that blood is getting to the limb appropriately.

The occupational health nurse is leading a group discussion about addiction. What should the nurse include as the primary barrier to the client with alcohol addiction seeking treatment? 1. Co-dependency 2. Denial 3. Depression 4. Stigma

2. Correct: They reject that they have a drinking problem and will argue with you if you suggest it. The client with an addiction may also use denial to lessen the impact of their addiction. 1. Incorrect: Co-dependency makes alcohol abuse last longer, but this is not the reason they do not seek treatment. Persons with co-dependency have difficulty establishing healthy relationships. 3. Incorrect: No depression associated with substance abuse. The primary reason a person does not seek treatment is denial of their addiction. 4. Incorrect: Yes, clients may be afraid of the stigma associated with addiction recovery. The person must first address their denial of addiction issues.

A nurse is caring for a client who has been connected to a ventilator after surgery. What interpretation should the nurse make based on the results of the client's ABGs? *Exhibit (ABGs)* pH - 7.32 PaO2 - 93% PaCO2 - 48 HCO3 - 24 1. Metabolic acidosis 2. Respiratory acidosis 3. Metabolic alkalosis 4. Respiratory alkalosis

2. Correct: This arterial blood gas results indicate that the client is in respiratory acidosis. 1. Incorrect: This is a respiratory problem. The bicarb is within normal limits, eliminating a metabolic problem. 3. Incorrect: The pH is low which indicates acidosis. The bicarb is within normal limits, eliminating a metabolic problem. 4. Incorrect: The pH is low, which indicates acidosis. The clues in this question are surgery and blood gas results found in the exhibit. In order to answer the question correctly you must open the exhibit to interpret the blood gases. Look at the options. There are opposites in these options. The answer is either acidosis or alkalosis. So look at the pH. The pH is 7.32 (normal 7.35-7.45) which means acidosis. So the test taker can eliminate options 3 and 4 since these options are alkalosis. Now determine if the problem is respiratory or metabolic. What other chemical says acidosis? The paCO2 of 48 (normal 35-45). So this is a respiratory problem isn't it? Yes, respiratory acidosis which is option 2. You might have guessed this from the clue "surgery". When clients have surgery, especially abdominal and thoracic surgery, taking deep breaths is difficult.

A client who delivered a 9-pound 12-ounce (4.17 kg) baby 1 hour ago, has saturated 2 peri-pads in 15 minutes. Which nursing action should take priority? 1. Notify the primary healthcare provider. 2. Massage the fundus. 3. Obtain a blood pressure. 4. Administer oxygen.

2. Correct: This is the only answer that will STOP BLEEDING! The fundus is boggy! 1. Incorrect: Doesn't stop the bleeding. Since it says priority you have to say, if I could only do ONE thing. If you choose this answer you have not STOPPED THE BLEEDING. 3. Incorrect: This is good, but how will it stop the bleeding. 4. Incorrect: The most common reason for saturating 2 peri-pads is a boggy fundus. The priority is to massage the fundus and stop the bleeding!

As a member of the emergency preparedness planning team at the hospital, which action should the nurse encourage the team to implement? 1. Developing a response plan for each potential disaster. 2. Providing education to employees on the response plan. 3. Practicing the response plan on a regular basis. 4. Evaluating the hospital's level of preparedness. 5. Preparing every hospital for all the same emergencies.

2., 3. & 4. Correct: Developing a single response plan, educating individuals to the specifics of the response plan, and practicing the plan and evaluating the facility's level of preparedness are effective means of implementing emergency preparedness. 1. Incorrect: One good response plan should be developed rather than multiple plans. 5. Incorrect: Consideration must be given to the proximity of chemical plants, nuclear facilities, schools, and areas where large groups gather. The basic principle of emergency preparedness are the same for all types of disasters. Only the response interventions vary to address the specific needs of the situation. So, one good response plan should be developed rather than multiple plans. This will ensure adequate understanding of the plan and decrease confusion of roles that could occur with multiple plans. Why is option 5 wrong. "Every" is one of those "always" or "never" type words. Is every hospital in an area the would need to prepare for a chemical disaster if there is not chemical plant near by? No. Remember education, practice, and evaluation are key to a good response plan.

A nurse is caring for a client in an outpatient clinic. The client lost her husband of 51 years three months ago. Which findings support that the client is experiencing normal grief reactions rather than clinical depression? 1. The client is experiencing anhedonia. 2. The client states, "I have good and bad days." 3. The client smiles at the nurse while talking about her grandchild. 4. The client has a persistent state of dysphoria. 5. The client states, "I am having fewer crying spells."

2., 3. & 5. Correct: A client going through a normal grieving process will experience a mixture of good and bad days. The client experiences moments of pleasure and cries less. 1. Incorrect: Anhedonia is the inability to experience pleasure seen in clinical depression. This would not be a positive sign of normal grieving in a client. 4. Incorrect: Dysphoria is a mood of general dissatisfaction, restlessness, depression, and anxiety. This is often seen in clinical depression.

What assessment findings would the nurse expect when evaluating whether treatment has been effective for a client hospitalized with systolic heart failure? 1. S3 heart sound 2. CVP of 6 mm Hg 3. One day weight loss of 2 pounds (0.91 kg) 4. Hepatomegaly 5. Increasing BNP level 6. Urine output at 50 mL/hr

2., 3. & 6. Correct: Normal CVP is 2-6 mm Hg. This CVP is within normal range so treatment is effective. Weight loss indicates that fluid is being removed and a urine output of 50mL/hour indicates that renal perfusion is adequate. All three assessments indicate improvement. 1. Incorrect: S3 would indicate that the client is not better. S3 is heard when the client is in fluid overload. 4. Incorrect: Hepatomegaly and tenderness in the right upper quadrant of the abdomen result from venous engorgement of the liver. The client is not better. 5. Incorrect: An increase in BNP level would indicate that the heart failure was getting worse not better.

Post thyroidectomy, the nurse assesses the client for complications by performing which assessment? 1. Perform blood glucose monitoring every 6 hours 2. Check for a positive Chvostek's 3. Assess swallowing reflex 4. Monitor neck dressings for change in fit and comfort 5. Administer desmopressin per nasal spray for urinary output (UOP) greater than 200 mL/hr

2., 3., & 4. Correct: A positive Chvostek's and Trousseau's is indicative of tetany (low calcium). This can occur when one or more of the parathyroids are accidently removed when the thyroid is removed. A weak, raspy voice, swallowing difficulty, and impaired respiratory status can be caused by nerve injury. Change in fit and comfort of the dressing can indicate possible neck swelling, which can affect the airway. 1. Incorrect: A possible complication of a thyroidectomy is to remove one or more parathyroid glands. The parathyroids' action is to regulate the serum calcium levels. The parathyroid does not regulate the blood glucose levels. 5. Incorrect: The action of desmopressin is to increase the reabsorption of water in the kidney. A decrease in vasopressin, (antidiuretic hormone) is not a complication of a thyroidectomy.

A nurse is planning to teach a group who works at a local mall about proper use of automated external defibrillators (AED). Which points should the nurse emphasize? 1. The standard AED can be used on children over the age of 5. 2. All users of the AED must be trained in its operation. 3. CPR should be taught to users. 4. Primary healthcare provider oversight is needed to ensure proper maintenance. 5. The local EMS should be notified of the type and location of AEDs.

2., 3., 4. & 5. Correct: These are appropriate actions. 1. Incorrect: Standard AEDs can be used on children over the age of 8. For children ages 1-8, the AHA recommends the pediatric attenuated pads that are purchased separately.

The nurse is preparing a seminar for a group of clients diagnosed with irritable bowel syndrome. Which points should the nurse include? 1. Teach a low fiber diet to fight the constipation. 2. Emphasize the importance of eating at regular times. 3. Drink at least one 8 ounce (240 mL) glass of water with meals. 4. Become active in yoga classes for stress reduction. 5. Keep a food diary for 1 week to determine irritating foods.

2., 4. & 5. Correct: Eating at regular intervals and chewing foods slowly and thoroughly will help to manage symptoms. Additional strategies include maintaining good dietary habits with avoidance of food triggers. Identify irritating foods by keeping a food diary for 1-2 weeks. Although adequate fluid intake is necessary, fluid should not be taken with meals because this results in abdominal distention. Alcohol use and cigarette smoking are discouraged. Stress management via relaxation techniques, yoga, or exercise are recommended. 1. Incorrect: This client needs a high fiber diet to help control diarrhea and constipation. 3. Incorrect: Although adequate fluid intake is necessary, fluid should not be taken with meals because this results in abdominal distention.

The client has pustules on the arm from intravenous drug abuse. The microbiology laboratory informs the nurse that the client's cultures are growing methicillin-resistant Staphylococcus aureus (MRSA). Which action should the nurse take? 1. Droplet isolation should be started immediately. 2. Inform the client to stay in the hospital room. 3. Instruct the client on the importance of hand hygiene. 4. Cover the pustules to prevent drainage. 5. Open all pustules to allow drainage.

3. & 4. Correct: It is important that the nurse implement these intervention in order to prevent the spread of infection. If the client refuses to follow instructions, then isolation precautions are warranted. 1. Incorrect: Contact isolation should be instituted.. 2. Incorrect: The client may leave room after proper hand washing with dressings that cover the pustules. 5. Incorrect: Opening the pustules will increase the chance of spreading the infection. Option 1 is false. MRSA is transmitted via contact, so contact isolation is needed. Option 2 is false. With contact precautions the client can leave the room. Option 3 is true. Hand hygiene is the number 1 way to prevent the spread of infection. Option 4 is true. The drainage is what spreads the infection. Option 5 is false. Don't open the pustules as drainage has the bacteria that can spread.

What should the nurse instruct a client to avoid when prescribed digoxin? 1. Ginkgo 2. Grapefruit 3. Black licorice 4. Aged cheese

3. Correct: Black licorice can deplete the body of potassium which can result in digoxin toxicity. 1. Incorrect: Ginkgo should not be taken with SSRI's. 2. Incorrect: Grapefruit should be avoided when taking statin medications. 4. Incorrect: Aged cheese should be avoided when taking MAOIs.

A new nurse asks the charge nurse for assistance in interpreting arterial blood gases for a client. What acid/base imbalance should the charge nurse tell the new nurse these ABGs indicate in the client? *Exhibit (ABGs)* pH - 7.5 PaO2 - 94% PaCO2 - 58 HCO3 - 35 1. Partially compensated metabolic acidosis 2. Partially compensated respiratory alkalosis 3. Partially compensated metabolic alkalosis 4. Partially compensated respiratory acidosis

3. Correct: Partially compensated metabolic alkalosis is indicated by these ABGs. 1. Incorrect: Partially compensated metabolic alkalosis is indicated by these ABGs. 2. Incorrect: Partially compensated metabolic alkalosis is indicated by these ABGs. 4. Incorrect: Partially compensated metabolic alkalosis is indicated by these ABGs. You must open the provided exhibit in order to see the ABG results. We know that there is partial compensation going on since all options say partially compensated. This should tell the test taker to expect the pH, PaCO2, and HCO3 to be abnormal. There are opposites in the options. The results are metabolic or respiratory and acidosis or alkalosis. Look at the pH of 7.5 (normal 7.35-7.45) which is high or alkalosis. So options 1 and 4 can be eliminated since these indicate acidosis. Next, look at the PaCO2 of 58 (normal 35-45) which is high or acidosis. Look at the HCO3 of 35 (normal 22-26) which is high or alkalosis. The HCO3 matches the pH as both indicate alkalosis. So the correct answer is Option 3: Partially compensated metabolic alkalosis.

A client was admitted to the unit during the night shift with chronic hypertension. At 0830, the unlicensed assistive personnel (UAP) reports that the client's blood pressure is 198/94 mm Hg. What would be the best action for the charge nurse to delegate at this time? 1. Ask a nursing student to put the client back in bed immediately. 2. Tell the UAP to take the BP in the opposite arm in 15 minutes. 3. Have the staff RN administer the 0900 furosemide and enalapril now. 4. Ask the LPN/LVN to assess the client for pain.

3. Correct: These medications lower blood pressure. 1. Incorrect: This client is unstable and needs a licensed nurse caring for them. 2. Incorrect: This client is considered unstable and the UAP should not have assignments until stable. 4. Incorrect: This is not an appropriate action for the LPN/VN. This question is looking for the best action. What is the safest or most appropriate action for the client to receive? What is the problem? A high BP. Is there something the nurse can do to fix this problem? Yes. Option 3, have the RN give furosemide, a diuretic, and enalopril, an ACE inhibitor to decrease BP. This will fix the problem and can be delegated to another RN. Do the other options fix the problem? No. This client is not stable, so the RN should not delegate options 1 and 2 to an unlicensed person. The LPN/VN cannot assess. This is an RN only task.

A client who is four days post-op cholecystectomy complains of severe abdominal pain. During the initial assessment the client states, "I have had two almost black stools today." Which nursing action is most important? 1. Start an IV with D5W at 125 mL/hr 2. Insert a nasogastric tube 3. Contact the primary healthcare provider 4. Obtain a stool specimen

3. Correct: What's going on inside? They are hemorrhaging. Assume the worst. The primary healthcare provider is the only one who can stop the bleeding. 1. Incorrect: There's nothing wrong with starting an IV, but isn't the client bleeding while you do this? 2. Incorrect: How does that help the bleeding stop? It doesn't. 4. Incorrect: You are going to wait on a stool specimen and Hemoccult? Don't delay care! Notify the primary healthcare provider first.

What information should a nurse include when preparing discharge education for a client diagnosed with gastroesophageal reflux disease (GERD)? 1. Raise head when sleeping on 2-3 pillows. 2. Foods that may trigger an attack include apple juice, cream cheese, and oatmeal. 3. Lose weight slowly at a rate of 1 kilogram per week. 4. Only eat three small meals per day. 5. Avoid tight-fitting clothing. 6. Wait at least 1 hour after eating to lie down.

3., & 5. Correct: Excess pounds (kg) put pressure on the abdomen, pushing up the stomach and causing acid to back up into the esophagus. Work to slowly lose weight, no more than 1-2 pounds (0.5-1 kg) per week. Avoid tight-fitting clothing. Clothes that fit tightly around the waist put pressure on the abdomen and the lower esophageal sphincter. 1. Incorrect: Raising the head with additional pillows is not effective. Elevate the head of the bed by placing wood or cement blocks under the feet of the bed so that the head end is raised by 6-9 inches (15.24-22.86 cm). If it's not possible to elevate the bed, the client can insert a wedge between the mattress and box spring to elevate the body from the waist up. 2. Incorrect: These foods are safe for the client with GERD to eat. Common triggers include fatty or fried foods, tomato sauce, alcohol, chocolate, mint, garlic, onion, and caffeine. 4. Incorrect: Eat 6 small meals per day. Avoid eating 3 large meals. 6. Incorrect: Don't lie down after a meal. Wait at least 3 hours after eating. Gastric acid is more likely to go up into the esophagus if the client lies down immediately.

A construction worker comes into the occupational health nurse's clinic reporting chest heaviness. What additional signs and symptoms should the nurse assess for? 1. Severe headache 2. Dry, flushed skin 3. Lightheadedness 4. Dyspnea 5. Irregular pulse

3., 4. & 5. Correct: The nurse should be thinking myocardial infarction! 1. Incorrect: Headaches do not commonly occur with MI. 2. Incorrect: Skin would be cool and clammy. Option 1: False. A severe headache is not a symptom of an MI. It may be a symptom of neurological disorders. Option 2: False. The client having an MI, will develop hypotension and possibly cardiogenic shock due to decreasing cardiac output. Remember, dead tissue doesn't pump well. The skin would be cool and clammy rather than warm, dry, and flushed. Option 3: True. Lightheadedness is a symptom of an MI. When cardiac output goes down, perfusion to the brain goes down. Option 4: True. Dyspnea is difficulty breathing. Lack of oxygen is reaching the lungs. Option 5: True. The heart muscle is irritable and leads to arrhythmias.

Question: Which client would the nurse monitor for the development of hypovolemic shock? 1. Client admitted with acute myocardial infarction. 2. Elderly client post-operative hip replacement with spinal anesthesia. 3. Client diagnosed with Addisonian crisis. 4. A 10 year old client with 40% Total body surface area (BSA) burns. 5. Client with Type 2 diabetes, who has a current blood sugar of 425 mg/dL (2.359 mmol/L)

3., 4. & 5. Correct: These clients are at risk for hypovolemic shock due to the loss of fluid from the vascular space. 1. Incorrect: I would worry about cardiogenic shock with this client. 2. Incorrect: I would worry about neurogenic shock with this client. Option 1 is false. With MI, think cardiogenic shock. Option 2 is false. With spinal anesthesia, think neurogenic shock. Option 3 is true. With addisonian crisis, the client is losing too much sodium and water from the vascular space which can lead to hypovolemic shock. Option 4 is true. A 10 year old child with 40% burns is considered to have a major burn. With major burns, the client loses fluid from the vascular space because of increased permeability. This sends fluid to the tissues and leads to hypovolemic shock.​ Option 5 is true. An adult with type 2 diabetes and an infection can develop HHNK. Diabetics who have a high blood sugar lose the glucose through the volume: Polyuria. Polyuria, think hypovolemic shock!

A client has been admitted to the psychiatric unit with a diagnosis of schizophrenia. Which client behaviors does the nurse anticipate? 1. Abstract reasoning 2. Waxy flexibility 3. Grandiose delusions 4. Anxiety 5. Agitated behavior

3., 4. & 5. Correct: Waxy flexibility describes a condition in which the client allows body parts to be placed in bizarre or uncomfortable positions for long periods of time. Delusions of grandiosity like believing they are a famous person or religious figure is a false fixed belief experienced by the client. If the client is in the acute phase of schizophrenia, the person may be overwhelmed by anxiety and is not able to distinguish thoughts from reality. It is thought that delusions may develop to cope with the anxiety. Agitated behavior like running about and going from one location to another can lead to exhaustion in this client. 1. Incorrect: This client has concrete thinking which implies over emphasis on specific details and an impairment in the ability to use abstract concepts. For example, during the nursing history you may ask the client what brought them to the hospital and the answer will be "a cab"

An occupational health nurse is planning to teach a group of manufacturing workers how to prevent back injuries. What teaching points should the nurse plan to include? 1. Assistive devices should be used when lifting greater than 50 pounds (22.72 kg). 2. When sitting, keep knees slightly lower than the hips. 3. Avoid movements that require spinal flexion with straight legs. 4. Squarely face the direction of anticipated movement. 5. Pivot to turn while holding an object. 6. Wear comfortable, low-heeled shoes.

3., 4., 5., & 6. Correct: Flexion the spine with the legs straight (toe-touches, sit-ups) will injure the back. Avoid twisting of the back by squarely facing the direction of movement. Move toward or away from your center of gravity. Pivoting is a technique in which the body is turned in a way that avoids twisting of the spine. Comfortable, low heeled shoes provide good foot support ad reduce the risk of slipping stumbling, or turning your ankle. 1. Incorrect: Assistive devices should be used when lifting greater than 35 pounds (15.9 kg). 2. Incorrect: When sitting, keep knees slightly higher than the hips.

The family of an elderly client are concerned about emotional well-being since the loss of the spouse two years ago. What alternative therapy could the nurse recommend for this client? 1. Massage 2. Bioelectromagnetics 3. Accupressure 4. Animal-assisted therapy

4. Correct: Animal-assisted therapy is the use of specifically selected animals as a treatment modality in health and human service settings. It has been shown to be a successful intervention for people with a variety of physical or psychological conditions. The contributions companion animals make to the emotional well-being of people include providing unconditional love and opportunities for affection; achievement of trust, responsibility, and empathy toward others; a reason to get up in the morning, and a source of reassurance. 1. Incorrect: Massage therapy is the scientific manipulation of the soft tissues of the body. It is believed to aid the body to heal itself. 2. Incorrect: This uses electromagnetic fields to affect the functioning of cells, tissues, organs and systems. 3. Incorrect: Acupressure is a treatment rooted in the traditional Eastern philosophy of life energy, that flows through the body along pathways. It opens up blocked pathways to relieve pain.

A nurse attaches a client to continuous cardiac monitoring due to a potassium level of 2.8 mEq (2.8 mmol). The nurse should monitor for which dysrhythmia? 1. Third degree heart block 2. Atrial fibrillation 3. Premature atrial contractions 4. Premature ventricular contractions

4. Correct: Hypokalemia is reflected by the EKG. The earliest EKG change is often premature ventricular contractions (PVCs) which can deteriorate into ventricular tachycardia or fibrillation (VT/VF) without appropriate potassium replacement. 1. Incorrect: Atrial-ventricular blocks are not often seen initially with hypokalemia. 2. Incorrect: Atrial fibrillation is not often seen with hypokalemia. 3. Incorrect: PACs are not often seen initially with hypokalemia.

A first generation Hispanic-American has been admitted to the psychiatric unit after being diagnosed with severe panic disorder. When developing the plan of care for this client, to which cultural background information should the nurse give priority? 1. Discuss treatment in terms of future plans for this client. 2. Do not use touch when communicating with this client. 3. Include the Protestant minister in the spiritual care of the client. 4. Allow family members to visit regularly.

4. Correct: Make allowances for individuals from other cultures to have family members around them and even participate in their care. Large numbers of extended family members are very important to African Americans, Native Americans, Asian Americans, and Hispanic-Americans. Denying access to these family support systems could interfere with the healing process. 1. Incorrect: Hispanic-Americans are more present oriented. They may be late to appointments and appear to be indifferent to some aspects of their therapy. Be accepting of these differences and refrain from allowing existing attitudes to interfere with delivery of care. 2. Incorrect: Hispanic-Americans have close personal space and use lots of touching and embracing. 3. Incorrect: Hispanic-Americans are most often Roman Catholic. The nurse needs to ask about religious preferences first.

What task would be appropriate for a nurse caring for a client diagnosed with gastroesophageal reflux to delegate to an unlicensed assistive personnel (UAP)? 1. Discuss methods of losing weight. 2. Explain reasons client should eat small frequent meals. 3. Monitor for GI upset after meals. 4. Remind the client to sit up for 2 hours after eating.

4. Correct: The UAP can remind the client to do something that has already been taught by the nurse. 1. Incorrect: The RN is responsible for teaching. 2. Incorrect: The RN is responsible for teaching. 3. Incorrect: The RN is responsible for assessment and evaluation. RNs cannot delegate any parts of the nursing process, assessment, evaluation, education, or preparation of the care plan. Knowing this will allow the test taker to eliminate options 1 and 2 because these involve teaching, and option 3 because it involves assessment.

How should the nurse interpret the ABG results of a client admitted with dehydration? *Exhibit (ABGs)* pH - 7.49 PaO2 - 99% PaCO2 - 29 HCO3 - 23 1. Metabolic acidosis 2. Respiratory acidosis 3. Metabolic alkalosis 4. Respiratory alkalosis

4. Correct: The blood gases confirm respiratory alkalosis. 1. Incorrect: The blood gases confirm respiratory alkalosis. 2. Incorrect: The blood gases confirm respiratory alkalosis. 3. Incorrect: The blood gases confirm respiratory alkalosis. You need knowledge of ABG interpretation. There are opposites in these options. You need to first decide if the client is in acidosis or alkalosis. So look at the pH of 7.49 (normal 7.35-7.45). This pH indicates alkalosis since it is high. You can eliminate options 1 and 2 since these are acidosis. That leaves options 3 and 4. Which other chemical says alkalosis? The PaCO2 of 29 (normal 35-45) is low which indicates alkalosis. The HCO3 is normal. This means that option 4, respiratory alkalosis is correct. How did this happen? The client must have been breathing too fast, blowing off CO2, an acid.

A client is admitted for treatment of fluid volume excess. The nurse reviews the admitting lab work and the primary healthcare provider's prescriptions. Which prescription would be of concern to the nurse? *lab* Sodium 138 mEq/L Potassium 5.4 mEq/L Calcium 9.0 mg/dL Glucose 108 mg/dL *HCP prescriptions* Bedrest 2 gm Na diet Spironolactone 25 mg by mouth once per day KCL 20 mEq by mouth twice a day 1. Bedrest 2. 2 gm Na diet 3. Spironolactone 4. KCL

4. Correct: The client has been prescribed spironolactone, a potassium sparing diuretic, so KCL supplement is not necessary. 1. Incorrect: Bed rest induces diuresis, which is good for this client. 2. Incorrect: This client needs to be on a low sodium diet to reduce fluid retention. 3. Incorrect: Spironolactone is a potassium sparing diuretic which can be prescribed for this client. Look at the hints. The client is receiving treatment for fluid volume excess with the prescriptions listed in the options: bedrest, 2 gm sodium diet, spironolactone, and potassium. What should you know about spironolactone? It is a potassium sparing diuretic. And the client is also receiving supplemental potassium! What are you worried about? High potassium. So look at the lab values seen in the exhibit. Sodium of 138 mEq/L or 138 mmol/L (normal 135-145 mEq/L or 135-145 mmol/L) is normal. Potassium of 5.4 mEq/L or 5.4 mmol/L (normal 3.5-5.0 mEq/L or 3.5-5.0 mmol/L) is high. The calcium of 9.0 mg/dL or 2.25 mmol/L (8.5-10.2 mg/dL or 2.125-2.55 mmol/L) is normal. Glucose of 108 mg/dL or 5.99 mmol/L (70-110 mg/dL or <6.1 mmol/L) is normal. So which prescription should the nurse be worried about? Option 4: KCL.

Preparing to administer a bolus enteral feeding to a client who is receiving a proton pump inhibitor, a nurse checks the pH of aspirated gastric fluid to determine feeding tube placement. The pH reading is 6. Which action should be taken by the nurse? 1. Initiate the tube feeding. 2. Replace the feeding tube. 3. Notify the primary healthcare provider of the assessment finding. 4. Inspect the aspirated contents for color and consistency.

4. Correct: The nurse should inspect the aspirated stomach contents for color and consistency to determine correct feeding tube placement. The normal stomach pH value is 1 to 4; however, when a client is receiving medications to decrease stomach acidity, the pH of the gastric aspirate may be as high as 6 and similar to the pH of respiratory secretions. Small intestines aspirates can also have a pH equal to or higher than 6. A pH of 6 does not confirm correct tube placement. 1. Incorrect: The tube feeding should not be initiated until feeding tube placement has been confirmed. 2. Incorrect: It is not necessary to replace the feeding tube at this time. Look at the aspirated contents for color and consistency. 3. Incorrect: Further data should be collected before notifying a primary healthcare provider.

Which client diagnosed with chronic peptic ulcer disease is at the highest risk for gastrointestinal bleeding? 1. 50 year old who consumes 4 ounces (120 mL) of wine at bedtime. 2. 55 year old who is positive for Helicobacter pylori (H. pylori). 3. 60 year old requiring corticosteroid inhalers for asthma. 4. 70 year old that takes clopidogrel daily for unstable angina.

4. Correct: The older client has a higher risk of bleeding and he is on a platelet aggregation inhibitor. This client is at highest risk for bleeding. 1. Incorrect: Alcohol is a cause of gastritis but not in this amount. Its role in bleeding is not confirmed. 2. Incorrect: A positive H. pylori is present in 50% of the world's population, not a deciding factor of bleeding. 3. Incorrect: This would be my second pick because of the corticosteroid effect on the protective lining of the stomach.

The mother of a 2 month-old baby calls a pediatrician's nurse two days after the first DTaP, inactivated polio vaccine (IPV), Hepatitis B and Haemophilus influenzae type B (HIB) immunizations. She reports that the baby feels very warm, cries inconsolably for as long as three hours, and has had several shaking spells. Which immunization would the nurse expect to be primarily responsible with these findings? DTaP IPV Hepatitis B HIB

DTaP DTaP immunization is a vaccine that protects against diptheria, tetanus and pertussis (whooping cough). The majority of reactions described in this question occur with the administration of the DTaP vaccination. Contraindications to giving repeat DTaP immunizations include the occurrence of severe side effects after a previous dose, as well as signs of encephalopathy within seven days of the immunization.

The nurse has a prescription to calculate a client's BMI. The client weighs 150 kgs and is 1.8 m tall. Determine the BMI to the whole number.

The client has a BMI of 46 BMI = weight in kilograms / height in meters squared First we multiply the client's height by itself: 1.8 x 1.8 = 3.24 ² Next we divide the client's weight by the height in meters ²: 150 / 3.24 = 46.3 The client's BMI is 46.3 or 46


Conjuntos de estudio relacionados

California Real Estate Chapter 3

View Set

G8 RELIGION CH. 12 - SAINTS IN OUR HISTORY PT. 2

View Set

Personal Lines Insurance Exam - Arizona

View Set

BUS1B Managerial Accounting Chapter 7

View Set

evrn 148 ch 6 human populations questions

View Set

Care Mgmt 1 FINAL Practice Questions

View Set